Wiley Acing the Gate: Engineering Mathematics and General Aptitude 9788126566556


270 22 33MB

English Pages 860 Year 2018

Report DMCA / Copyright

DOWNLOAD PDF FILE

Table of contents :
Cover......Page 1
Inner Title......Page 2
Preface......Page 6
Contents......Page 8
Engineering Mathematics......Page 24
CR_GATE EM_3rd_Chapter 1......Page 26
CR_GATE EM_3rd_Chapter 2......Page 96
CR_GATE EM_3rd_Chapter 3......Page 194
CR_GATE EM_4th_Chapter 4......Page 248
CR_GATE EM_4th_Chapter 5......Page 276
CR_GATE EM_4th_Chapter 6......Page 336
CR_GATE EM_4th_Chapter 7......Page 394
CR_GATE EM_4th_Chapter 8......Page 418
CR_GATE EM_3rd_Chapter 9......Page 450
CR_GATE EM_3rd_Chapter 10......Page 462
CR_GATE EM_3rd_Chapter 11......Page 488
CR_GATE GA_3rd_Verbal Ability......Page 510
VA_Chapter 1......Page 512
VA_Chapter 2......Page 526
VA_Chapter 3......Page 538
VA_Chapter 4......Page 546
VA_Chapter 5......Page 552
VA_Chapter 6......Page 556
VA_Chapter 7......Page 560
CR_GATE GA_4th_Numerical Ability......Page 566
Chapter 01-1......Page 568
Chapter 01-2......Page 584
Chapter 01-3......Page 596
Chapter 01-4......Page 606
Chapter 01-5......Page 620
Chapter 01-6......Page 634
Chapter 01-7......Page 644
Chapter 01-8......Page 654
Chapter 02-1......Page 670
Chapter 02-2......Page 680
Chapter 02-3......Page 688
Chapter 02-4......Page 702
Chapter 02-5......Page 710
Chapter 03-1......Page 720
Chapter 03-2......Page 728
Chapter 03-3......Page 734
Chapter 03-4......Page 740
Chapter 03-5......Page 746
Chapter 03-6......Page 754
Chapter 03-7......Page 762
Chapter 03-8......Page 770
GATE_GA-Questions......Page 776
Recommend Papers

Wiley Acing the Gate: Engineering Mathematics and General Aptitude
 9788126566556

  • 0 0 0
  • Like this paper and download? You can publish your own PDF file online for free in a few minutes! Sign Up
File loading please wait...
Citation preview

WILEY

GATE

ENGINEERING MATHEMATICS AND GENERAL APTITUDE SECOND

EDITION

WILEY

GATE

ENGINEERING MATHEMATICS AND GENERAL APTITUDE Dr Anil Kumar Maini

Senior Scientist and former Director of Laser Science and Technology Centre Defence Research and Development Organization, New Delhi

Varsha Agrawal

Senior Scientist, Laser Science and Technology Centre Defence Research and Development Organization, New Delhi

Nakul Maini

SECOND

EDITION

WILEY ACING THE GATE ENGINEERING MATHEMATICS AND GENERAL APTITUDE SECOND EDITION Copyright © 2017 by Wiley India Pvt. Ltd., 4435-36/7, Ansari Road, Daryaganj, New Delhi-110002. All rights reserved. No part of this book may be reproduced, stored in a retrieval system, or transmitted in any form or by any means, electronic, mechanical, photocopying, recording or scanning without the written permission of the publisher.

Limits of Liability: While the publisher and the author have used their best efforts in preparing this book, Wiley and the author make no representation or warranties with respect to the accuracy or completeness of the contents of this book, and specifically disclaim any implied warranties of merchantability or fitness for any particular purpose. There are no warranties which extend beyond the descriptions contained in this paragraph. No warranty may be created or extended by sales representatives or written sales materials.

Disclaimer: The contents of this book have been checked for accuracy. Since deviations cannot be precluded entirely, Wiley or its author cannot guarantee full agreement. As the book is intended for educational purpose, Wiley or its author shall not be responsible for any errors, omissions or damages arising out of the use of the information contained in the book. This publication is designed to provide accurate and authoritative information with regard to the subject matter covered. It is sold on the understanding that the Publisher is not engaged in rendering professional services.

Other Wiley Editorial Offices: John Wiley & Sons, Inc. 111 River Street, Hoboken, NJ 07030, USA Wiley-VCH Verlag GmbH, Pappellaee 3, D-69469 Weinheim, Germany John Wiley & Sons Australia Ltd, 42 McDougall Street, Milton, Queensland 4064, Australia John Wiley & Sons (Asia) Pte Ltd, 1 Fusionpolis Walk #07-01 Solaris, South Tower Singapore 138628 John Wiley & Sons Canada Ltd, 22 Worcester Road, Etobicoke, Ontario, Canada, M9W 1L1

First Edition: 2015 Second Edition: 2017 ISBN: 978-81-265-6655-6 ISBN: 978-81-265-8347-8 (ebk) www.wileyindia.com Printed at:

PREFACE

Wiley Acing the GATE: Engineering Mathematics and General Aptitude is intended to be the complete book for those aspiring to compete in the Graduate Aptitude Test in Engineering (GATE) in various engineering ­disciplines, including Electronics and Communication, Electrical, Mechanical, Computer Science, Civil, Chemical and Instrumentation, comprehensively covering all topics as prescribed in the syllabus in terms of study material and an elaborate question bank. There are host of salient features offered by the book as compared to the content of the other books already published for the same purpose. Some of the important ones include the following. One of the notable features of the book includes presentation of study material in simple and lucid language and in small sections while retaining focus on alignment of the material in accordance with the requirements of GATE examination. While it is important for a book that has to cover a wide range of topics in Engineering Mathematics, General Aptitude including Verbal Ability and Numerical Ability to be precise in the treatment of different topics, the present book achieves that goal without compromising completeness. The study material and also the question bank have all the three important `C’ qualities, Conciseness, Completeness and Correctness, for communicating effectively with the examinees. Another important feature of the book is its comprehensive question bank. The question bank is organized in three different categories, namely the Solved Examples, Practice Exercise and Solved GATE Previous Years’ Questions. Solved Examples contain a large number of questions of varying complexity. Each question in this category is followed by its solution. Under Practice Exercise, again there are a large number of multiple choice questions. The answers to these questions are given at the end of the section. Each of the answers is supported by an explanation unlike other books where solutions to only selected questions are given. The third category contains questions from previous GATE examinations from 2003 onwards. Each question is followed by a complete solution. Briefly outlining the length and breadth of the material presented in the book, it is divided into two broad sections, namely Engineering Mathematics and General Aptitude. Section on General Aptitude is further divided into two sections covering Verbal Ability and Numerical Ability. Engineering Mathematics is covered in eleven different chapters. These include Linear Algebra covering important topics such as matrix algebra, systems of linear equations, and eigenvalues and eigenvectors; Calculus covering important topics such as functions of single variable, limit, continuity and differentiability, mean value theorem, definite and improper integrals, partial derivatives, maxima and minima, gradient, divergence and curl, directional derivatives, line, surface and volume integrals, Stokes’ theorem, Gauss theorem and Green’s theorem, and Fourier series; Differential Equations covering linear and non-linear differential equations, Cauchy’s and Euler’s equations, Laplace transforms, partial differentiation, solutions of one-dimensional heat and wave equations, Laplace equation, and method of variation of parameters; Complex Variables covering analytic functions, Cauchy’s integral theorem, and Taylor and Laurent series; Probability and Statistics covering conditional probability, random variables, discrete and continuous distributions, Poisson, normal, uniform, exponential and binomial distributions, correlation and regression analyses, residue theorem, and solution integrals; Numerical Methods covering numerical solutions of linear and non-linear algebraic equations, integration by trapezoidal and Simpson’s rules, single and multi-step methods for differential equations, numerical solutions of non-linear algebraic equations by secant, bisection, Runge—Kutta and Newton—Raphson methods; Mathematical Logic including first-order logic and proportional logic; Set Theory and Algebra including

Gate_FM.indd 5

8/25/2017 5:38:27 PM

vi       

PREFACE 

sets, relations, functions and groups, partial orders, lattice, and Boolean algebra; Combinatory covering permutations and combinations, counting, summation, generating functions, recurrence relations, and asymptotics; Graph Theory covering connectivity and spanning trees, cut vertices and edges, covering and matching, independent sets, colouring, planarity, and isomorphism; and Transform Theory covering Fourier transform, Laplace transform and z-transform. General Aptitude comprises two sub-sections namely Verbal Ability and Numerical Ability. Important topics covered under Verbal Ability include English grammar, synonyms, antonyms, sentence completion, verbal analogies, word groups, and critical reasoning and verbal deduction. Under Numerical Ability, important topics covered include basic arithmetic, algebra, and reasoning and data interpretation. Under Basic Arithmetic, we have discussed number system; percentage; profit and loss; simple interest and compound interest; time and work; average, mixture and allegation; ratio, proportion and variation; and speed, distance and time. Under Algebra, we have discussed permutation and combination; progression; probability; set theory; and surds, indices and logarithm. The topics covered under Reasoning and Interpretation are cubes and dices, line graph, tables, blood relationship, bar diagram, pie chart, puzzles, and analytical reasoning. The Graduate Admission Test in Engineering (GATE) is an All-India level competitive examination for engineering graduates aspiring to pursue Master’s or Ph.D. programs in India. The examination evaluates the examinees in General Aptitude, Engineering Mathematics and the subject discipline. Though majority of questions is asked from the subject discipline; there are sizable number of questions set from Engineering Mathematics and General Aptitude. It is an examination where close to ten lakh students appear every year. The level of competition is therefore very fierce. While admission to a top institute for the Master’s programme continues to be the most important reason for working hard to secure a good score in the GATE examination; another great reason to appear and handsomely qualify GATE examination is that many Public Sector Undertakings (PSUs) are and probably in future almost all will be recruiting through GATE examination. And it is quite likely that even big private sector companies may start considering GATE seriously for their recruitment as GATE score can give a bigger clue about who they are recruiting. The examination today is highly competitive and the GATE score plays an important role. This only reiterates the need to have a book that prepares examinees not only to qualify the GATE examination by getting a score just above the threshold but also enabling them to achieve a competitive score. In a competition that is as fierce as the GATE is, a high score in Engineering Mathematics and General Aptitude section can be a great asset. The present book is written with the objective of fulfilling this requirement. The effort is intended to offer to the large section of GATE aspirants a self-study and do-it-yourself book providing comprehensive and step-by-step treatment of each and every aspect of the examination in terms of concise but complete study material and an exhaustive set of questions with solutions. The authors would eagerly look forward to the feedback from the readers through publishers to help them make the book better. Dr ANIL KUMAR MAINI VARSHA AGRAWAL NAKUL MAINI

Gate_FM.indd 6

8/25/2017 5:38:27 PM

CONTENTS

Preface

v

ENGINEERING MATHEMATICS

1

1 Linear Algebra Matrix Types of Matrices Types of a Square Matrix Equality of a Matrix Addition of Two Matrices Multiplication of Two Matrices Multiplication of a Matrix by a Scalar Transpose of a Matrix Adjoint of a Square Matrix Inverse of a Matrix Rank of a Matrix Determinants Minors Cofactors Solutions of Simultaneous Linear Equations Solution of Homogeneous System of Linear Equations Solution of Non-Homogeneous System of Simultaneous Linear Equations Cramer’s Rule Augmented Matrix Gauss Elimination Method Cayley—Hamilton Theorem Eigenvalues and Eigenvectors Properties of Eigenvalues and Eigenvectors Solved Examples  Practice Exercise Answers

Gate_FM.indd 7

3 3 3 4 4 5 5 5 5 6 6 6 7 7 7 8 9 9 10 10 10 11 11 12 12 15 18

8/25/2017 5:38:27 PM

viii       

CONTENTS 

Explanations and Hints Solved GATE Previous Years’ Questions 2 Calculus Limits Left-Hand and Right-Hand Limits Properties of Limits L’Hospital’s Rule Continuity and Discontinuity Differentiability Mean Value Theorems Rolle’s Theorem Lagrange’s Mean Value Theorem Cauchy’s Mean Value Theorem Taylor’s Theorem Maclaurin’s Theorem Fundamental Theorem of Calculus Differentiation Applications of Derivatives Increasing and Decreasing Functions Maxima and Minima Partial Derivatives Integration Methods of Integration Integration Using Table Integration Using Substitution Integration by Parts Integration by Partial Fraction Integration Using Trigonometric Substitution Definite Integrals Improper Integrals Double Integration Change of Order of Integration Triple Integrals Applications of Integrals Area of Curve Length of Curve Volumes of Revolution Fourier Series Conditions for Fourier Expansion Fourier Expansion of Discontinuous Function Change of Interval Fourier Series Expansion of Even and Odd Functions Half Range Series Vectors Addition of Vectors Multiplication of Vectors

Gate_FM.indd 8

19 26 73 73 73 74 74 74 74 74 74 75 75 75 75 75 75 76 76 77 78 78 78 78 79 79 80 80 80 80 81 82 82 82 82 82 83 83 83 84 84 84 84 85 85 85

8/25/2017 5:38:27 PM

CONTENTS          ix

Multiplication of Vectors Using Cross Product Derivatives of Vector Functions Gradient of a Scalar Field Divergence of a Vector Curl of a Vector Directional Derivative Scalar Triple Product Vector Triple Product Line Integrals Surface Integrals Stokes’ Theorem Green’s Theorem Gauss Divergence Theorem Solved Examples  Practice Exercise  Answers  Explanations and Hints Solved GATE Previous Years’ Questions 3 Differential Equations Introduction Solution of a Differential Equation Variable Separable Homogeneous Equation Linear Equation of First Order Exact Differential Equation Integrating Factor Clairaut’s Equation Linear Differential Equation Particular Integrals Homogeneous Linear Equation Bernoulli’s Equation Euler—Cauchy Equations Homogeneous Euler—Cauchy Equation Non-homogeneous Euler—Cauchy Equation Solving Differential Equations Using Laplace Transforms Variation of Parameters Method Separation of Variables Method One-Dimensional Diffusion (Heat Flow) Equation Second Order One-Dimensional Wave Equation Two-Dimensional Laplace Equation Solved Examples Practice Exercises Answers Explanations and Hints Solved GATE Previous Years’ Questions

Gate_FM.indd 9

86 86 86 86 86 87 87 87 87 88 88 88 88 89 99 103 103 112 171 171 171 171 172 172 172 172 172 173 173 174 175 175 175 175 176 176 176 177 178 178 179 187 192 192 198

8/25/2017 5:38:27 PM

x       

CONTENTS 

4 Complex Variables Introduction Complex Functions Exponential Function of Complex Variables Circular Function of Complex Variables Hyperbolic Functions of Complex Variables Logarithmic Function of Complex Variables Limit and Continuity of Complex Functions Derivative of Complex Variables Cauchy—Riemann Equations Integration of Complex Variables Cauchy’s Theorem Cauchy’s Integral Formula Taylor’s Series of Complex Variables Laurent’s Series of Complex Variables Zeros and Poles of an Analytic Function Residues Residue Theorem Calculation of Residues Solved Examples Practice Exercise Answers Explanations and Hints  Solved GATE Previous Years’ Questions 5 Probability and Statistics Fundamentals of Probability Types of Events Approaches to Probability Axioms of Probability Conditional Probability Geometric Probability Rules of Probability Statistics Arithmetic Mean Median Mode Relation Between Mean, Median and Mode Geometric Mean Harmonic Mean Range Mean Deviation Standard Deviation Coefficient of Variation Probability Distributions Random Variable Properties of Discrete Distribution

Gate_FM.indd 10

225 225 225 225 226 226 226 227 227 227 228 228 228 229 229 229 230 230 230 230 233 234 234 236 253 253 254 254 254 254 254 255 255 255 256 256 256 257 257 257 257 258 258 258 258 259

8/25/2017 5:38:27 PM

CONTENTS          xi

Properties of Continuous Distribution Types of Discrete Distribution Types of Continuous Distribution Correlation and Regression Analyses Correlation Lines of Regression Hypothesis Testing Hypothesis Testing Procedures for Some Common Statistical Problems Hypothesis Testing of a Proportion Hypothesis Testing of a Mean Hypothesis Testing of Difference Between Proportions Hypothesis Testing of Difference Between Means Bayes’ Theorem Solved Examples Practice Exercise Answers Explanations and Hints Solved GATE Previous Years’ Questions 6 Numerical Methods Introduction Numerical Solution of System of Linear Equations Gauss Elimination Method Matrix Decomposition Methods (LU Decomposition Method) Gauss—Jordan Method Iterative Methods of Solution Numerical Solution of Algebraic and Transcendental Equations Bisection Method Regula—Falsi Method (Method of False Position Method) Newton—Raphson Method Secant Method Jacobian Numerical Integration Newton—Cotes Formulas (General Quadrature) Rectangular Rule Trapezoidal Rule Simpson’s Rule  Numerical Solution of Ordinary Differential Equation (O.D.E.)  Picard’s Method Euler’s Method Runge—Kutta Method Euler’s Predictor—Corrector Method Accuracy and Precision Classification of Errors Significant Figures Measuring Error Propagation of Errors

Gate_FM.indd 11

259 259 260 261 261 261 262 262 262 263 263 264 265 265 272 275 275 281 313 313 313 313 314 315 315 316 316 317 317 317 318 318 318 319 319 320 320 321 321 322 322 322 323 323 324 324

8/25/2017 5:38:27 PM

xii       

CONTENTS 

Method of Least Square Approximation Lagrange Polynomials Numerical Differentiation Newton’s Forward Formula Newton’s Backward Formula Stirling’s or Bessel’s Formula Solved Examples  Practice Exercise Answers Explanations and Hints Solved GATE Previous Years’ Questions 7 Mathematical Logic Introduction Statements Atomic Statements Molecular Statements Truth Table Truth Values Connectives Types of Connectives Well-Formed Formulas Duality Law Equivalent Well-Formed Formula Logical Identities Normal Form Disjunction Normal Form Conjunctive Normal Form Propositional Calculus Rules of Inference Predicate Calculus Predicates Quantifier Solved Examples Practice Exercise  Answers  Explanations and Hints  Solved GATE Previous Years’ Questions 8 Set Theory and Algebra Introduction to Set Theory Subsets and Supersets Equal Sets Comparable Sets Universal Set Power Set Types of Sets

Gate_FM.indd 12

324 325 326 326 326 327 327 340 342 342 351 371 371 371 371 371 371 371 372 372 373 373 373 374 375 375 376 376 376 378 378 378 379 381 382 382 385 395 395 395 395 395 396 396 396

8/25/2017 5:38:27 PM

CONTENTS          xiii

Operations on Sets Important Laws and Theorems Venn Diagrams Operations on Sets Using Venn Diagrams Application of Sets Cartesian Product of Sets  Relations  Types of Relations  Properties of Relation Functions Types of Functions Compositions of Functions  Introduction to Algebra Semigroups Some Important Theorems  Group  Some Important Theorems  Residue Classes  Residue Class Addition Residual Class Multiplication Partial Ordering Hasse Diagram Lattice Sublattice Bounds Boolean Algebra of Lattices Solved Examples Practice Exercise  Answers Explanations and Hints Solved GATE Previous Years’ Questions

9 Combinatory Introduction  Counting Fundamental Principle of Addition Fundamental Principle of Multiplication Permutations Conditional Permutations  Permutations When all Objects are not Distinct  Circular Permutations  Combinations Properties of Combinations  Conditional Combinations  Generating Functions  Ordinary Generating Function

Gate_FM.indd 13

396 396 397 397 397 398 398 398 398 399 399 400 400 401 401 402 402 402 402 402 402 402 403 403 403 404 405 411 416 416 422

427 427 427 427 427 428 428 428 428 428 428 429 429 429

8/25/2017 5:38:27 PM

xiv       

CONTENTS 

Exponential Generating Function  Poisson Generating Function Recurrence Relation  Logistic Map Fibonacci Numbers  Binomial Coefficients Summation  Asymptotic Analysis  Solved Examples  Practice Exercise  Answers  Explanations and Hints  Solved GATE Previous Years’ Questions 10 Graph Theory Introduction Fundamental Concepts of Graph Common Terminologies Degree of a Vertex Multigraph Walks, Paths and Connectivity Subgraph Types of Graphs Complete Graph Regular Graph Bipartite Graph Tree Graph Trivial Graph Cycle Operations on Graphs Matrix Representation of Graphs Adjacent Matrix Incidence Matrix Cuts Spanning Trees and Algorithms Kruskal’s Algorithm Prim’s Algorithm Binary Trees Euler Tours .. Konigsberg Bridge Problem Hamiltonian Graphs Closure of a Graph Graph Isomorphism Homeomorphic Graphs Planar Graphs Matching Covering

Gate_FM.indd 14

429 429 429 429 430 430 430 430 431 432 433 434 437 439 439 439 439 440 440 440 441 441 441 441 441 442 442 443 443 443 443 444 444 444 444 445 445 445 445 445 446 446 446 447 447 447

8/25/2017 5:38:27 PM

CONTENTS          xv

Independent Set Graph Coloring Solved Examples Practice Exercise Answers Explanations and Hints Solved GATE Previous Years’ Questions

Chapter 11 Transform Theory



Introduction Laplace Transform Laplace Transform of Common Signals Properties of Laplace Transform Inverse Laplace Transform z -Transform z -Transform of Common Sequences Properties of z -Transform Inverse z-Transform Relationship between z-Transform and Laplace Transform Fourier Transform Convergence of Fourier Transforms Properties of Fourier Transform Solved Examples Practice Exercise Answers  Explanations and Hints Solved GATE Previous Years’ Questions

GENERAL APTITUDE: VERBAL ABILITY 1 English Grammar Articles Noun Use of Nouns in Singular Form Use of Nouns in Plural Form Conversion of Nouns from Singular to Plural Collective Nouns Pronoun Personal Pronoun Reflexive and Emphatic Pronoun Demonstrative Pronoun Indefinite Pronoun Distributive Pronoun Relative Pronoun Interrogative Pronoun Use of Pronouns

Gate_FM.indd 15

447 447 448 452 455 455 457

465 465 465 466 466 467 467 468 468 469 470 470 470 471 472 474 475 475 476

487 489 489 491 491 491 491 491 492 492 492 493 493 493 493 493 493

8/25/2017 5:38:28 PM

xvi       

CONTENTS 

Adjective Use of Adjectives Preposition Preposition of Time Preposition of Position Preposition of Direction Other Uses of Preposition Verbs Use of Verb Use of Infinitives Use of Gerunds Tenses Use of Tenses Adverbs PracticeExercise Answers 2 Synonyms Tips to Solve Synonym Based Questions Practice Exercise Answers 3 Antonyms Graded Antonyms Complementary Antonyms Relational Antonyms Practice Exercise Answers 4 Sentence Completion Tips to Solve Sentence Completion Based Questions Practice Exercise Answers 5 Verbal Analogies Types of Analogies Tips to Solve Verbal Analogies Questions Practice Exercise Answers Explanations and Hints 6 Word Groups Practice Exercise Answers Explanations and Hints

Gate_FM.indd 16

494 494 494 494 495 495 495 496 496 496 497 497 498 498 499 502 503 503 508 513 515 515 516 516 516 522 523 523 525 528 529 529 530 530 531 532 533 535 536 536

8/25/2017 5:38:28 PM

CONTENTS          xvii

7 Verbal Deduction Tips and Tricks to Solve Verbal Deduction Questions Practice Exercise Answers Explanations and Hints

GENERAL APTITUDE: NUMERICAL ABILITY

537 538 540 541

543

Unit 1: Basic Arithmetic

545

1 Number System

547

Numbers Classification of Numbers Progressions Arithmetic Progressions Geometric Progressions Infinite Geometric Progressions Averages, Mean, Median and Mode Relation between AM, GM and HM Some Algebraic Formulas The Remainder Theorem The Polynomial Factor Theorem Base System Counting Trailing Zeros Inequations Quadratic Equations Even and Odd Numbers Prime Numbers and Composite Numbers HCF and LCM of Numbers Cyclicity Test for Divisibility Solved Examples Practice Exercise Answers  Explanations and Hints 

2 Percentage Introduction Some Important Formulae Solved Examples Practice Exercise Answers Explanations and Hints

Gate_FM.indd 17

537

547 547 548 548 548 548 548 549 549 549 549 550 550 550 551 551 551 551 552 552 553 556 557 558

561 561 561 562 566 568 568

3 Profit and Loss

573

Introduction

573

8/25/2017 5:38:28 PM

xviii       

CONTENTS 

Some Important Formulae Margin and Markup Solved Examples Practice Exercise Answers Explanations and Hints 4 Simple Interest and Compound Interest Introduction Some Important Formulae Solved Examples Practice Exercise Answers Explanations and Hints 5 Time and Work Introduction Important Formulas and Concepts Solved Examples Practice Exercise Answers Explanations and Hints 6 Average, Mixture and Alligation Average Weighted Average Mixture and Alligation Solved Examples Practice Exercise Answers Explanations and Hints 7 Ratio, Proportion and Variation

583 583 583 584 588 591 591 597 597 597 598 601 604 605 611 611 611 611 612 615 617 617 621

Ratio Proportion Variation Solved Examples Practice Exercise Answers Explanations and Hints

621 622 622 623 625 627 627

8 Speed, Distance and Time

631

Introduction Some Important Formulas Solved Examples Practice Exercise Answers Explanations and Hints

Gate_FM.indd 18

573 574 574 577 579 579

631 631 632 636 640 640

8/25/2017 5:38:28 PM

CONTENTS          xix

Unit 2: Algebra

647

1 Permutation and Combination

649

Permutation Combination Partitions Counting Fundamental Principle of Addition Fundamental Principle of Multiplication Solved Examples Practice Exercise Answers Explanations and Hints 2 Progression Arithmetic Progression Geometric Progression Infinite Geometric Progression Harmonic Series Relation between AM, GM and HM Solved Examples Practice Exercise Answers Explanations and Hints 3 Probability Introduction Some Basic Concepts of Probability Some Important Theorems Solved Examples Practice Exercise Answers Explanations and Hints 4 Set Theory Introduction Venn Diagrams Operation on Sets Venn Diagram with Two Attributes Venn Diagram with Three Attributes Solved Examples Practice Exercise Answers Explanations and Hints 5 Surds, Indices and Logarithms Surds

Gate_FM.indd 19

649 649 650 650 650 650 650 652 653 653 657 657 657 658 658 658 658 661 662 662 665 665 665 666 666 671 673 674 679 679 679 679 680 680 681 682 684 684 687 687

8/25/2017 5:38:28 PM

xx       

CONTENTS 

Indices Logarithm Solved Examples Practice Exercise Answers Explanations and Hints Unit 3: Reasoning and Data Interpretation

697

1 Cubes and Dices

699

Cubes Dices Solved Examples Practice Exercise Answers  Explanations and Hints 2 Line Graph Introduction Solved Examples Practice Exercise Answers Explanations and Hints 3 Tables Introduction Solved Examples Practice Exercise Answers Explanations and Hints 4 Blood Relationship Introduction Standard Coding Technique Solved Examples Practice Exercise Answers  Explanations and Hints 5 Bar Diagram Introduction Solved Examples Practice Exercise Answers Explanations and Hints

Gate_FM.indd 20

687 687 688 691 692 693

699 700 700 702 703 703 705 705 706 707 709 709 711 711 711 713 715 715 717 717 718 718 719 721 721 723 723 724 728 729 729

8/25/2017 5:38:28 PM

CONTENTS          xxi

6 Pie Chart Introduction Types of Pie Charts 3-D Pie Chart Doughnut Chart  Exploded Pie Chart  Polar Area Chart  Ring Chart/Multilevel Pie Chart Solved Examples Practice Exercise Answers Explanations and Hints 7 Puzzles Introduction Types of Puzzles Solved Examples Practice Exercise Answers  Explanations and Hints 8 Analytical Reasoning Introduction Solved Examples Practice Exercise Answers  Explanations and Hints GENERAL APTITUDE: SOLVED GATE PREVIOUS YEARS’ QUESTIONS

Gate_FM.indd 21

731 731 731 731 732 732 732 732 733 736 737 738 739 739 739 739 742 744 744 747 747 747 749 751 751 753

8/25/2017 5:38:28 PM

Gate_FM.indd 22

8/25/2017 5:38:28 PM

ENGINEERING MATHEMATICS

Chapter 1.indd 1

8/22/2017 2:14:56 PM

Chapter 1.indd 2

8/22/2017 2:14:56 PM

CHAPTER 1

LINEAR ALGEBRA

MATRIX

Types of Matrices

A set of mn number (real or imaginary) arranged in the form of a rectangular array of m rows and n columns is called an m × n matrix. An m × n matrix is usually written as  a11 a12   a21 a22  A=       a  m1 am2

a13 a23   am3

… a1n  … a2n          … amn 

In compact form, the above matrix is represented by A = [aij]m×n or A = [aij]. The numbers a11, a12, …, amn are known as the elements of the matrix A. The element aij belongs to ith row and jth column and is called the (ij)th element of the matrix A = aij  .  

Chapter 1.indd 3

1. Row matrix: A matrix having only one row is called a row matrix or a row vector. Therefore, for a row matrix, m = 1.

For example, A = [1 2 −1] is a row matrix with m = 1 and n = 3. 2. Column matrix: A matrix having only one column is called a column matrix or a column vector. Therefore, for a column matrix, n = 1.  2   For example, A = 3 is a column matrix with  1  

m = 3 and n = 1. 3. Square matrix: A matrix in which the number of rows is equal to the number of columns, say n, is called a square matrix of order n. 1 2  is a square matrix of For example, A =  3 4

order 2.

8/22/2017 2:14:58 PM

4     LINEAR ALGEBRA

4. Diagonal matrix: A square matrix is called a diagonal matrix if all the elements except those in the leading diagonal are zero, i.e. aij = 0 for all i ≠ j. 1 0 0    For example, A = 0 5 0  is a diagonal matrix  0 0 10  

and is denoted by A = diag[1, 5, 10]. 5. Scalar matrix: A matrix A = [aij]n×n is called a scalar matrix if (a) aij = 0, for all i ≠ j. (b) aii = c, for all i, where c ≠ 0.  5 0 0   For example, A = 0 5 0 is a scalar matrix of  0 0 5  

order 5.

6. Identity or unit matrix: A square matrix A = [aij]n×n is called an identity or unit matrix if (a) aij = 0, for all i ≠ j. (b) aij = 1, for all i.  1 0  is an identity matrix of For example, A =   0 1 order 2. 7. Null matrix: A matrix whose all the elements are zero is called a null matrix or a zero matrix.  0 0  is a null matrix of order For example, A =   0 0 2 × 2. 8. Upper triangular matrix: A square matrix A  =  [aij] is called an upper triangular matrix if aij = 0 for i > j. 1  0 For example, A =  0 0 

2 5 0 0

6 7 9 0

3  4 is an upper tri3 2

angular matrix. 9. Lower triangular matrix: A square matrix A = [aij] is called a lower triangular matrix if aij = 0 for i < j. 1 0 0 0   2 5 0 0 For example, A =   is a lower trian6 2 9 0  3 7 4 3  

that An = 0, then n is called index of the nilpotent matrix A. 2. Symmetrical matrix: It is a square matrix in which aij = aji for all i and j. A symmetrical matrix is necessarily a square one. If A is symmetric, then AT = A. 1 2 3   For example, A = 2 5 4 . 3 4 6    3. Skew-symmetrical matrix: It is a square matrix in which aij = −aji for all i and j. In a skewsymmetrical matrix, all elements along the diagonal are zero.  0 2 3   For example, 2 0 4 .  3 4 0   4. Hermitian matrix: It is a square matrix A in which (i, j)th element is equal to complex conjugate of the (j, i)th element, i.e. aij = aji for all i and j. A necessary condition for a matrix A to be Hermitian is that A = Aq, where Aq is transposed conjugate of A.  1 1 + 4i 2 + 3i    For example, 1 − 4i 2 5 + i .  2 − 3i 5 − i 4   5. Skew-Hermitian matrix: It is a square matrix A = [aij] in which aij = −aij for all i and j.

The diagonal elements of a skew-Hermitian matrix must be pure imaginary numbers or zeroes. A necessary and sufficient condition for a matrix A to be skew-Hermitian is that Aq = −A 6. Orthogonal matrix: A square matrix A is called orthogonal matrix if AAT = ATA = I.

For example, if  cos q  cos q − sin q   and AT =  A=  sin q cos q  − sin q   1 0  = I. then AAT =   0 1

sin q   cos q 

Equality of a Matrix

gular matrix. Two matrices A = [aij]m×n and B = [bij]x×y are equal if

Types of a Square Matrix 1. Nilpotent matrix: A square matrix A is called a nilpotent matrix if there exists a positive integer n such that An = 0. If n is least positive integer such

Chapter 1.indd 4

1. m = x, i.e. the number of rows in A equals the number of rows in B. 2. n = y, i.e. the number of columns in A equals the number of columns in B. 3. aij = bij for i = 1, 2, 3, …, m and j = 1, 2, 3, …, n.

8/22/2017 10:52:29 AM

MATRIX     5

Addition of Two Matrices

1 2  4 3  and B =   For example, if A =  3 4 2 1  1 × 4 + 2 × 2 1 × 3 + 2 × 1  Then A × B =  3 × 4 + 4 × 2 3 × 3 + 4 × 1

Let A and B be two matrices, each of order m × n. Then their sum (A + B) is a matrix of order m × n and is obtained by adding the corresponding elements of A and B.

8 5  C = A×B =  20 13 Some important properties of matrix multiplication are:

Thus, if A = [aij]m×n and B = [bij]m×n are two matrices of the same order, their sum (A + B) is defined to be the matrix of order m × n such that (A + B)ij = aij + bij for i = 1, 2, …, m and j = 1, 2, …, n The sum of two matrices is defined only when they are of the same order.  2 1 7 8   and B =   For example, if A =  3 5 1 2  9 9  Hence, A + B =   4 7   1 3 1 3 1  and   is not possible. However, addition of  2 1 2 2 1 Some of the important properties of matrix addition are: 1. Commutativity: If A and B are two m × n matrices, then A + B = B + A, i.e. matrix addition is commutative. 2. Associativity: If A, B and C are three matrices of the same order, then (A + B) + C = A + (B + C) i.e.matrix addition is associative. 3. Existence of identity: The null matrix is the identity element for matrix addition. Thus, A + O = A=O+A 4. Existence of inverse: For every matrix A = [aij]m×n, there exists a matrix [aij]m×n, denoted by −A, such that A + (−A) = O = ( −A) + A 5. Cancellation laws: If A, B and C are matrices of the same order, then A+B=A+C⇒B=C B+A=C+A⇒B=C

Multiplication of Two Matrices

1. Matrix multiplication is not commutative. 2. Matrix multiplication is associative, i.e. (AB)C = A(BC). 3. Matrix multiplication is distributive over matrix addition, i.e. A(B + C) = AB + AC. 4. If A is an m×n matrix, then ImA = A = AIn. 5. The product of two matrices can be the null matrix while neither of them is the null matrix.

Multiplication of a Matrix by a Scalar If A = [aij] be an m × n matrix and k be any scalar constant, then the matrix obtained by multiplying every element of A by k is called the scalar multiple of A by k and is denoted by kA.  1 3 1   For example, if A = 2 7 3   and  k = 2.  4 9 8   2 6 2    ⇒ kA = 2 14 6  3 18 16   Some of the important properties of scalar multiplication are:

1. k(A + B) = kA + kB 2. (k + l) ⋅ A = kA + lA 3. (kl) ⋅ A = k(lA) = l(kA) 4. (−k) ⋅ A = −(kA) = k(−A) 5. 1 ⋅ A = A 6. −1 ⋅ A = −A

Here A and B are two matrices of same order and k and l are constants. If A is a matrix and A2 = A, then A is called idempotent matrix. If A is a matrix and satisfies A2 = I, then A is called involuntary matrix.

If we have two matrices A and B, such that A = [aij]m×n  and  B = [bij]x×y, then A × B is possible only if n = x, i.e. the columns of the pre-multiplier is equal to the rows of the post multiplier. Also, the order of the matrix formed after multiplying will be m × y. n

(AB)ij = Σ air brj = ai1b1j + ai2b2 j +  + ain bnj r =1

Chapter 1.indd 5

Transpose of a Matrix Consider a matrix A, then the matrix obtained by interchanging the rows and columns of A is called its transpose and is represented by AT.  1 2 5 1 3 7      For example, if A = 3 9 8 , AT = 2 9 6 . 7 6 4   5 8 4    

8/22/2017 10:52:34 AM

6     LINEAR ALGEBRA

Some of the important properties of transpose of a matrix are: 1. For any matrix A, (AT)T = A 2. For any two matrices A and B of the same order (A + B)T = AT + BT 3. If A is a matrix and k is a scalar, then (kA)T = k(AT) 4. If A and B are two matrices such that AB is defined, then (AB)T = BTAT

7. If A and B are non-singular square matrices of the same order, then adj (AB) = (adj B)(adj A) 8. If A is an invertible square matrix, then adj AT = (adj A)T 9. If A is a non-singular square matrix, then adj (adj A) = |A|n−2 A 10. If A is a non-singular matrix, then 1 |A−1| = |A|−1, i.e. | A−1 | = |A| 11. Let A, B and C be three square matrices of same type and A be a non-singular matrix. Then

Adjoint of a Square Matrix

AB = AC ⇒ B = C and      BA = CA ⇒ B = C

Let A = [aij] be a square matrix of order n and let Cij be the cofactor of aij in A. Then the transpose of the matrix of cofactors of elements of A is called the adjoint of A and is denoted by adj A. Thus, adj A aji in A.  a11  If A = a21  a31

= [Cij]T ⇒ (adj A)ij = Cji = cofactor of a12 a22 a32

a13   a23   a33 

 c11 c12  then adj(A) = c21 c22  c31 c32

The column rank of matrix A is the maximum number of linearly independent column vectors of A. The row rank of A is the maximum number of linearly independent row vectors of A. In linear algebra, column rank and row rank are always equal. This number is simply called rank of a matrix.

c13   c11   c23  = c12   c33  c13

c21 c22 c23

c31   c32   c33 

Inverse of a Matrix A square matrix of order n is invertible if there exists a square matrix B of the same order such that AB = In = BA In the above case, B is called the inverse of A and is denoted by A−1. (adj A) A−1 = |A| Some of the important properties of inverse of a matrix are: 1. A−1 exists only when A is non-singular, i.e. |A| ≠ 0. 2. The inverse of a matrix is unique. 3. Reversal laws: If A and B are invertible matrices of the same order, then (AB)−1 = B−1A−1 4. If A is an invertible square matrix, then (AT)−1 = (A−1)T 5. The inverse of an invertible symmetric matrix is a symmetric matrix. 6. Let A be a non-singular square matrix of order n. Then |adj A| = |A|n−1

Chapter 1.indd 6

Rank of a Matrix

The rank of a matrix A is commonly denoted by rank (A). Some of the important properties of rank of a matrix are:

1. The rank of a matrix is unique. 2. The rank of a null matrix is zero. 3. Every matrix has a rank. 4. If A is a matrix of order m × n, then rank (A) ≤ m × n (smaller of the two) 5. If rank (A) = n, then every minor of order n + 1, n + 2, etc., is zero. 6. If A is a matrix of order n × n, then A is nonsingular and rank (A) = n. 7. Rank of IA = n. 8. A is a matrix of order m × n. If every kth order minor (k < m, k < n) is zero, then rank (A) < k 9. A is a matrix of order m × n. If there is a minor of order (k < m, k < n) which is not zero, then rank (A) ≥ k 10. If A is a non-zero column matrix and B is a nonzero row matrix, then rank (AB) = 1. 11. The rank of a matrix is greater than or equal to the rank of every sub-matrix. 12. If A is any n-rowed square matrix of rank, n − 1, then adj A ≠ 0 13. The rank of transpose of a matrix is equal to rank of the original matrix. rank (A) = rank (AT)

8/22/2017 10:52:35 AM

DETERMINANTS     7

14. The rank of a matrix does not change by ­pre-multiplication or post-multiplication with a non-singular matrix. 15. If A − B, then rank (A) = rank (B). 16. The rank of a product of two matrices cannot exceed rank of either matrix. rank (A × B) ≤ rank A or     rank ( A × B) ≤ rank B 17. The rank of sum of two matrices cannot exceed sum of their ranks. 18. Elementary transformations do not change the rank of a matrix.

1 3 , then minors of A will be For example, say A =  2 4 M11 = 4 M12 = 2 M21 = 3 M22 = 1 Say       A =

M11 =

DETERMINANTS

M12 =

Every square matrix can be associated to an expression or a number which is known as its determinant. If A = [aij] is a square matrix of order n, then the determinant of A is denoted by det A or |A|. If A = [a11] is a square matrix of order 1, then determinant of A is defined as |A| = a11

M13 = M 21 = M 22 =

a a  If A =  11 12  is a square matrix of order 2, then a21 a23  determinant of A is defined as |A| = a11a23 − a12a21

M 23 =

 a11 a12  If A = a21 a22  a31 a32 then determinant of

M 32 =

a13   a23  is a square matrix of order 3,  a33  A is defined as

    A| = a  11 (a22a33 − a13a32) + or      |A| = a  11 (a22a33 − a23a31) +

− a23a32) a21 (a12a23 − a23a32) a13 (a21a32

− a21 (a12a33 − a13a22) − a12 (a21a33 − a22a31)

 2 3  For example, determinants of the matrices [1],  1 5 1 7 1   and 2 −7 3 will be, respectively,  5 9 8   Δ=1 Δ = (2 × 5) − (1 × 3) = 7 Δ = 1[(−7 × 8) − (3 × 9)] − 2 [(7 × 8) − (1 × 9)] + 5 [(7 × 3) − (−7 ×1)] = −83 − 94 + 140 = −177 + 40 = −37

Minors The minor Mij of A = [aij] is the determinant of the square sub-matrix of order (n − 1) obtained by removing ith row and jth column of the matrix A.

Chapter 1.indd 7

M 31 =

M 33 =

 1 2 3    3 5 1 −4 4 7     5 1   = 35 − 4 = 31  4 7   3 1   = 21 − (−4) = 25 −4 7   3 5   = 12 − (−20) = 32 −4 4  2 3   = 14 − 12 = 2  4 7   1 3   = 7 − (−12) = 19 −4 7   1 2   = 4 − (−8) = 12 −4 4  2 3   = 2 − 15 = −13 5 1  1 3   = 1 − 9 = −8 3 1  1 2   = 5 − 6 = −1 3 5

Cofactors The cofactor Cij of A = [aij] is equal to (−1)i+j times the determinant of the sub-matrix of order (n − 1) obtained by leaving ith row and jth column of A.  1 2 3   For example, say A =  3 5 1 . −4 4 7    Cofactors of A will be  5 1  = 31 C11 = (−1)1+1   4 7   3 1 C12 = (−1)1+2 −4 7  = −25    3 5 C13 = (−1)1+3 −4 4 = 32    2 3 C21 = (−1)2+1  4 7  = −2    1 3 C22 = (−1)2+2 −4 7  = 19    1 2 C23 = (−1)2+3 −4 4 = −12  

8/22/2017 10:52:43 AM

8     LINEAR ALGEBRA

 2 3 C31 = (−1)3+1 5 1 = −13    1 3 C32 = (−1)3+2 3 1 = 8    1 2 C33 = (−1)3+3 3 5 = −1   Some of the important properties of determinants are: 1. Sum of the product of elements of any row or column of a square matrix A = [aij] of order n with their cofactors is always equal to its determinant. n

n

∑ aij cij = |A| =

∑ aij cij

i =1

j =1

2. Sum of the product of elements of any row or column of a square matrix A = [aij] of order n with the cofactors of the corresponding elements of other row or column is zero. n

n

∑ aij cik = 0 =

∑ aij ckj

i =1

j =1

3. For a square matrix A = [aij] of order n, |A| = |AT|. 4. Consider a square matrix A = [aij] of order n ≥ 2 and B obtained from A by interchanging any two rows or columns of A, then |B| = −A. 5. For a square matrix A = [aij] of order (n ≥ 2), if any two rows or columns are identical, then its determinant is zero, i.e. |A| = 0. 6. If all the elements of a square matrix A = [aij] of order n are multiplied by a scalar k, then the determinant of new matrix is equal to k|A|. 7. Let A be a square matrix such that each element of a row or column of A is expressed as the sum of two or more terms. Then |A| can be expressed as the sum of the determinants of two or more matrices of the same order. 8. Let A be a square matrix and B be a matrix obtained from A by adding to a row or column of A a scalar multiple of another row or column of A, then |B| = |A|. 9. Let A be a square matrix of order n (≥ 2) and also a null matrix, then |A| = 0. 10. Consider A = [aij] as a diagonal matrix of order n (≥ 2), then |A| = a11 × a22 × a33 ×  × ann 11. Suppose A and B are square matrices of same order, then |AB| = |A| ⋅ |B|

Chapter 1.indd 8

SOLUTIONS OF SIMULTANEOUS LINEAR EQUATIONS Suppose we have a system of `i’ linear equation in `j’ unknowns, such as a11x1 + a12x2 +  + aijxj = b1 a221x1 + a22x2 +  + a2jxj = b2   :     :    :    :    :   :     :    :    :    : ai1x1  +  ai2x2 +  + aijxj = bi The above system of equations can be written in the matrix form as follows: a a  aij   x1   b1   11 12     a21 a22  a2 j   x2  b2     =                  ai1 ai2  aij   xj  bj    The equation can be represented by the form AX = B.  a11 a12  aij    a21 a22  a2 j   is of the order of i × j, where A =           ai1 ai2  aij    x   1 x  x =  2  is of the order of j × 1 and  x   j   b1    b  B =  2  is of the order of i × 1.  b   i  [A]i×j is called the coefficient matrix of system of linear equations. For example, 2x + 3y − z = 2 3x + y + 2z = 1 4x − 7y + 5z = −7 The above equation can be expressed in the matrix form as 2 3 −1  x  2       2  y  =  1  3 1  4 −7 5   z  −7        A consistent system is a system of equations that has one or more solutions. An inconsistent system is a system of equations that has no solutions. For example, consider    x + y = 4 2x + 3y = 9 The above system of equation is consistent as it has the solution, x = 3 and y = 1.

8/22/2017 10:52:48 AM

SOLUTIONS OF SIMULTANEOUS LINEAR EQUATIONS     9

Now, consider

The above system of equation is inconsistent as it has no solution for both equations simultaneously.

or AX = 0 2 1 3   A =  3 −1 1 = (2(−3 + 2) − 3(3 + 6) − (1 + 3)) −1 −2 3   = −2 − 27 − 4 = −33 ≠ 0

A homogeneous system is a system when B = 0 in the equation AX = B.

As |A| ≠ 0, given system of equations has the solution     x = y = z = 0.



x+y=4 2x + 2y = 6

A non-homogeneous system is a system when B ≠ 0 in the equation AX = B. For example, consider 3x + y = 0 6x + 2y = 0 Such a system is a homogeneous system.

Solution of Non-Homogeneous System of Simultaneous Linear Equations Non-homogeneous equations have already been discussed in the previous sections. Also, the methods to solve homogeneous system of equations were also discussed in the previous section.

Now, consider 3x + y = 1 6x + 2y = 2 Such a system is a non-homogeneous system.

Solution of Homogeneous System of Linear Equations As already discussed, for a homogeneous system of linear equation with `j’ unknowns,

AX = B becomes



AX = 0 (∵ B = 0)

There are two cases that arise for homogeneous systems: 1. Matrix A is non-singular or |A| ≠ 0.

The solution of the homogeneous system in the above equation has a unique solution, X = 0, i.e. x1 = x2 =  = xj = 0. 2. Matrix A is singular or |A| = 0, then it has infinite many solutions. To find the solution when |A| = 0, put z = k (where k is any real number) and solve any two equations for x and y using the matrix method. The values obtained for x and y with z = k is the solution of the system.

For example, given the following system of homogeneous equation

2x + y + 3z = 0 3x − y + z = 0 −x − 2y + 3z = 0

This system can 2  3 −1 

Chapter 1.indd 9

be rewritten as 1 3  x   0      −1 1   y  =  0  −2 3  z   0

In this section, we will discuss the method to solve a non-homogeneous system of simultaneous linear equations. Please note the number of unknowns and the number of equations. 1. Given that A is a non-singular matrix, then a system of equations represented by AX = B has the unique solution which can be calculated by X = A−1 B. 2. If AX = B is a system with linear equations equal to the number of unknowns, then three cases arise: (a) If A ≠ 0 , system is consistent and has a unique solution given by X = A−1 B. (b) If A = 0 and (adj A)B = 0, system is consistent and has infinite solutions. (c) If A = 0 and (adj A)B ≠ 0, system is inconsistent. For example, consider the following system of equations: x + 2y + z x + 3z 2x − 3y 1 2 1   x    1 0 3   y   2 −3 0   z     

=7 = 11 =1 7   = 11 1   1 2 1    ⇒ AX = B, where A = 1 0 3  2 −3 0    1 2 1   A = 1 0 3 = 1(0 + 9) − 1(0 + 3) + 2(6)  2 −3 0    = 9 − 3 + 12 = 18 ≠ 0 So, the given system of equations has a unique solution given by X = A−1 B. Now, calculate the adjoint of matrix.

8/22/2017 10:52:51 AM

10     LINEAR ALGEBRA

 9 −3 6    adj A =  6 −2 −2 −3 7 −2    9 −3 6  1 1   A−1 = adj A =  6 −2 −2  A 18 −3 7 −2   X = A−1B  9 −3 6   7   63 − 33 + 6  36 2 1   1   1    X =  6 −2 −2 11 =  42 − 22 − 2  = 18  = 1 18 −3 7 −2  1  18 −21 + 77 − 2 18 54 3            x   2      y  =  1  z   3     ⇒ x = 2, y = 1, z = 3 is the unique solution of the given set of equations.

Cramer’s Rule Suppose we have the following system of linear equations: a1x + b1y + c1z = k1 a2x + b2y + c2z = k2 a3x + b3y + c3z = k3

AUGMENTED MATRIX Consider the following system of equations: a11x1 + a12 x2 +  + a1n xn = b1 a21x1 + a22 x2 +  + a2n xn = b2 

  am1x1 + am2 x2 +  + amn xn = bm This system can be represented as AX = B.  x1   a11 a12  a1n      x   a21 a22  a2n  2 where A =   , X =   and         a  x   m1 am2  amn   n  b1    b  B =  2  .   b   n    a11 a12  a1n a a22  a2n The matrix  A B  =  21      a am2  amn m 1  augmented matrix.

b1   b2  is called    bm 

Now, if  a1  ∆ = a2  a3

GAUSS ELIMINATION METHOD

b1 b2 b3

c1   c2  ≠ 0  c3 

 k1  ∆1 = k2  k3

b1 b2 b3

c1   c2  ≠ 0  c3 

 a1  ∆2 = a2  a3

k1 k2 k3

c1   c2  ≠ 0  c3 

In the matrix form, the above equations can be represented as AX = B. We consider two particular cases:

 a1  ∆3 = a2  a3

b1 b2 b3

k1   k2  ≠ 0  k3 

Case 1: Suppose the coefficient matrix A is such that all elements above the leading diagonal are zero. Then A is a lower triangular matrix and the system can be written as follows:

Consider the following system of equations: a11x1 + a12 x2 +  + a1n xn = 0 a21x1 + a22 x2 +  + a2n xn = 0 



 an1x1 + an2 x2 +  + ann xn = 0

a11x1 = b1 Thus, the solution of the system of equations is given by ∆1 ∆ ∆2 y= ∆ ∆3 z= ∆ x=

Chapter 1.indd 10

a21x1 + a22 x2 = b2 a31x1 + a32 x2 + a33 x3 = b3  an1x1 + an2 x2 +  + ann xn = bn The system given above can be solved using forward substitution.

8/22/2017 10:52:55 AM

EIGENVALUES AND EIGENVECTORS     11

b − (a31x1 + a32 x2 ) b1 b − a21x1 ; x2 = 2 ; x3 = 3 a11 a22 a33 and so on. ∴ x1 =

Case 2: Suppose the coefficient matrix A is such that all elements below the leading diagonal are zero. Then A is an upper triangular matrix and the system can be written in the following form: a11x1 + a12 x2 +  + a1n xn = b1

Here −a21 /a11 and −a31 /a11 are called the ­multipliers for the first stage of elimination. It is clear that we have assumed a11 ≠ 0. ′ /a22 ′ ) R2 , we get Step 3: Performing R3 → R3 − (a32 a  11 a12  0 a22 ′   0 0 

a13 ′ a23 ′′ a33

b1  b2′   b3′′

where

a22 x2 +  + a2n xn = b2

′ ′′ = a33 ′ − (a32 ′ /a22 ′ ) × a23 a33



′′ = b33 ′ − (a32 ′ /a22 ′ ) × b2′ b33

ann xn = bn The system given above can be solved using backward substitution. ∴ x3 =

b − (a12 x2 + a13 x3 ) b3 b − a23 x3 ; x2 = 2 ; x1 = 1 a33 a22 a11

This is the end of the forward elimination. From the system of equations obtained from step 3, the values of x1 , x2 and x3 can be obtained by back substitution. This procedure is called partial pivoting. If this is impossible, then the matrix is singular and the system has no solution.

and so on. Gauss elimination method is a standard elimination method for solving linear systems that proceeds systematically irrespective of particular features of the coefficients. a11x1 + a12 x2 + a13 x3 = b1 a21x1 + a22 x2 + a23 x3 = b2

CAYLEY–HAMILTON THEOREM According to the Cayley–Hamilton theorem, every square matrix satisfies its own characteristic equations. Hence, if

a31x1 + a32 x2 + a33 x3 = b3

A − lI = (−1)n (l n + a1l n −1 + a2 l n −2 +  + an )

The first equation of the above system is called the pivotal equation and the leading coefficient a11 is called first pivot.

is the characteristic polynomial of a matrix A of order n, then the matrix equation

Step 1: Form the augmented system. a  11 a12 a13 a21 a22 a23  a31 a32 a33

matrix of the above b1  b2   b3 

Step 2: Performing the following operations R2 R2 → R2 − (a21 /a11 ) R1 and R3 → R3 − (a31 /a11 ) R1 , we get a  11 a12  0 a22 ′   0 a32 ′ 

a13 b1  ′ b2′  a23  ′ b3′  a33 

where ′ = a22 − (a21 /a11 ) a12 a22 ′ = a23 − (a21 /a11 ) a13 a23 ′ = a32 − (a31 /a11 ) a12 a32 ′ = a33 − (a31 /a11 ) a13 a33 b2′ = b2 − (a21 /a11 ) b1 b3′ = b3 − (a31 /a11 ) b1

Chapter 1.indd 11

X n + a1X n −1 + a2 X n −2 +  + an I = 0 is satisfied by X = A.

EIGENVALUES AND EIGENVECTORS → R2 − (a21 /a11 ) R1 If A = [aij]n×n is a square matrix of order n, then the  x1     x2    .  vector equation AX = lX, where X =   is an unknown .  .    x   n  vector and l is an unknown scalar value, is called an eigenvalue problem. To solve the problem, we need to determine the value of X’s and l’s to satisfy the abovementioned vector. Note that the zero vector (i.e. X = 0) is not of our interest. A value of l for which the above equation has a solution X ≠ 0 is called an eigenvalue or characteristic value of the matrix A. The corresponding

8/22/2017 10:53:02 AM

12     LINEAR ALGEBRA

solutions X ≠ 0 of the equation are called the eigenvectors or characteristic vectors of A corresponding to that eigenvalue, l. The set of all the eigenvalues of A is called the spectrum of A. The largest of the absolute values of the eigenvalues of A is called the spectral radius of A. The sum of the elements of the principal diagonal of a matrix A is called the trace of A.

Properties of Eigenvalues and Eigenvectors Some of the main characteristics of eigenvalues and eigenvectors are discussed in the following points: 1. If l1, l2, l3, …, ln are the eigenvalues of A, then kl1, kl2, kl3, …, kln are eigenvalues of kA, where k is a constant scalar quantity. 2. If l1, l2, l3, …, ln are the eigenvalues of A, then 1 1 1 1 , , , ..., are the eigenvalues of A−1. l1 l2 l3 ln

3. If l1, l2, l3, …, ln are the eigenvalues of A, then l1k , l2k , l3k , ..., lnk are the eigenvalues of Ak. 4. If l1, l2, l3, …, ln are the eigenvalues of A, then A A A A , , , ..., are the eigenvalues of adj A. l1 l2 l3 ln 5. The eigenvalues of a matrix A are equal to the eigenvalues of AT. 6. The maximum number of distinct eigenvalues is n, where n is the size of the matrix A. 7. The trace of a matrix is equal to the sum of the eigenvalues of a matrix. 8. The product of the eigenvalues of a matrix is equal to the determinant of that matrix. 9. If A and B are similar matrices, i.e. A = IB, then A and B have the same eigenvalues. 10. If A and B are two matrices of same order, then the matrices AB and BA have the same eigenvalues. 11. The eigenvalues of a triangular matrix are equal to the diagonal elements of the matrix.

SOLVED EXAMPLES 1 6   4 −7   1 9  ,B =   and C =   , then 1. If A =  2 4 3 5  −8 2 find the value of 3A + 2B − 4C . Solution:  We have 1 6   4 −7   1 9  ,B =   and C =   A= 2 4 3 5  −8 2 Now, 3 18  3A =  6 12  8 −14  2B =  6 10   4 36  4C =  −32 8  Now calculating Eqs. (1) + (2) − (3), we get



a−b = 5 2a − b = 12  2c + d = 3  2a + d = 15 

(1) (2) (3) (4)

Subtracting Eq. (1) from Eq. (2), we get a=7 (1)

⇒ 7−b = 5 ⇒ b = 2 Substituting the value of a in Eq. (4), we get

(2) (3)

3 18  8 −14  4 36 + −  3A + 2B − 4C =  6 12 6 10  −32 8   3 + 8 − 4 18 − 14 − 36  7 −32 =  = 6 + 6 + 32 12 + 10 − 8   44 14   a − b 2c + d   5 3  2. Find a, b, c and d such that  = . 2a − b 2a + d  12 15 Solution:  We know that the corresponding elements of two equal matrices are equal.

Chapter 1.indd 12

Therefore,

14 + d = 15 ⇒ d = 1 Substituting the value of d in Eq. (3), we get 2c + 1 = 3 ⇒ c = 1 Hence, a = 7, b = 2, c = 1 and d = 1. 2 0 1    3. If A = 2 1 3 , then find the value of A2.  1 −1 0    Solution:  We have 2 0 1    A = 2 1 3   1 −1 0   

8/22/2017 10:53:11 AM

SOLVED EXAMPLES      13

⇒ 1(−6 − 2) − 1(6 − 6) + x(−2 − 6) = 0 ⇒ −8 − 8x = 0 ⇒ x = −1

Now, 2  A2 = A ⋅ A = 2 1  4 + 0 + 1  = 4 + 2 + 3 2 − 2 + 0 

0 1  2 0 1     1 3  ⋅ 2 1 3    −1 0 1 −1 0 0 + 0 − 1 2 + 0 + 0  0 + 1 − 3 2 + 3 + 0 0 − 1 + 0 1 − 3 + 0   5 −1 2    =  9 −2 5   0 −1 −2   

6. Solve the following system of equations using Cramer’s rule: 5x − 7 y + z = 11 6x − 8y − z = 15 3x + 2y − 6z = 7

−1   4. If A =  2  and B = [−2 −1 −4], verify that 3   (AB)T = B T A T .

Now, 5 −7 1 D = 6 −8 −1 3 2 −6

Solution:  We have −1   A =  2  and B = [−2 −1 −4] 3  

= 5(48 + 2) + 7(−36 + 3) + 1(12 + 24) = 250 − 231 + 36 = 55

−1   AB =  2  [−2 −1 −4] 3  

11 −7 1 D1 = 15 −8 −1 7 2 −6

2 1 4    = −4 −2 −8  −6 −3 −12  

= 11(48 + 2) + 7(−90 + 7) + 1(30 + 56) = 550 − 581 + 86 = 55

 2 −4 −6    (AB)T = 1 −2 −3   4 −8 −12  

5 11 1 D2 = 6 15 −1 3 7 −6 = 5(−90 + 7) − 11(−36 + 3) + 1(42 − 45) = −415 + 363 − 3 = −55

Also, B T AT

−1 T −2     = [−2 −1 −4]  2  = −1 [−1 2 3] 3 −4      2 −4 −6    =  1 −2 −3   4 −8 −12  

5 −7 11 D3 = 6 −8 15 3 2 7

T

= 5(−56 − 30) + 7(42 − 45) + 11(12 + 24) = −430 − 21 + 396 = −55 Now, using Cramer’s rule,

Hence, it can be seen that (AB)T = B T A T .  1 −2 3    1 5. For what value of x, the matrix A =  1 2  x 2 −3    is singular? Solution:  The matrix 1 −2 3 1 2 1 =0 x 2 −3

Chapter 1.indd 13

Solution:  The given set of equations is 5x − 7 y + z = 11 6x − 8y − z = 15 3x + 2y − 6z = 7

A

is

singular

if

D1 55 = = 1, D 55 D −55 y= 2 = = −1, D 55 D −55 z= 3 = = −1 D 55 x=

Hence, the solution of the given set of equations is x = 1, y = −1 and z = −1.

8/22/2017 10:53:17 AM

14     LINEAR ALGEBRA

 cos f − sin f 0   7. Find the adjoint of the matrix A =  sin f cos f 0 .  0 0 1  Solution:  Let Cij be the cofactor of A. Then C11 = (−1)1+1 = (−1)1+ 2

cos f 0 = cos f, C12 0 1 sin f 0 0 1

= − sin f, C13 = (−1)1+ 3 C21

C31

sin f cos f =0 0 0

1  5 −2   . 1 −7 3 

9. Show that the homogeneous system of equations has a non-trivial solution and find the solution. x − 2y + z = 0 x +y −z = 0 3x + 6y − 5z = 0 Solution:  The given system of equations can be written in the matrix form as follows:

 1 −2 1   x   0       − sin f 0 1 1 −1  y  =  0 = (−1)2 +1 = sin f, C22  3 6 −5   z   0  0 1        1 −2 1   x     which is similar to AX = O, where A = 1 1 −1 , X =  y  cos f 0 = (−1)2 + 2  3 6 −5  z  0 1      x  0  1 −2 1        A = 1 1 −1 , X =  y  and O =  0 . cos f − sin f = cos f, C23 = (−1)2 + 3 =0 z   0  3 6 −5  0 0       Now, − sin f 0 = (−1)3 +1 = 0, C32 cos f 0 1 −2 1 A = 1 1 −1 = 1(−5 + 6) − 1(10 − 6) cos f 0 = (−1)3 + 2 3 6 −5 sin f 0 = 0, C33 = (−1)3 + 3

+ 3(2 − 1) = 0

cos f − sin f =1 sin f cos f

Therefore,  cos f sin f 0  cos f − sin f 0 T     adj A =  sin f cos f 0 = − sin f cos f 0  0  0 0 1 0 1    3 2  , then what 8. If A =  7 5 Solution:  We have 3 A= 7 Now, A =

−1

is the value of A .

Thus, A = 0 and hence the given system of equations has a non-trivial solution. Now, to find the solution, we put z = k in the first two equations. x − 2y = −k x+y =k 1 −2  x −k     =   1 1   y  k 

 x 1 −2  ,X =   which is similar to AX = B, where A =  y  1 1     x −k  1 −2  , X =   and B =   . A= y  k  1 1      Now,

2  5

3 2 = 15 − 14 = 1 ≠ 0 7 5

A =

1 −2 =3≠0 1 1

Hence, A−1 exists.

Hence, A is invertible. The cofactors of A is given by A11 = (−1)

1+1

⋅5 = 5

A12 = (−1)1+ 2 ⋅ 7 = −7 A21 = (−1)2 +1 ⋅ 2 = −2 A22 = (−1)2 + 2 ⋅ 3 = 3 The adjoint of the matrix A is given by  5 −2   5 −7  T  =  adj A =  −2 3  −7 3 

Chapter 1.indd 14

Hence, A−1 =

 1 2 . Now, adj A =  −1 1 1  1 2  A−1 =  3 −1 1  x 1  1 2 −k  k/3     =  Now, X = A−1B ⇒   =   y  3 −1 1 k  2k/3       ⇒ x = k/3, y = 2k/3 Hence, x = k/3, y = 2k/3 and z = k, where k is any real number that satisfies the given set of equations.

8/22/2017 10:53:26 AM

PRACTICE EXERCISE     15

10. Check the following system of equations for consistency. 4x − 2y = 3 6x − 3y = 5

The cofactors can be calculated as follows: C11 = (−1)1+1(−3) = −3 C12 = (−1)1+ 2 (6) = −6 C21 = (−1)2 +1(−2) = 2

Solution:  The given system of equations can be written as  x  3  4 −2   , X =   and B =   . AX = B, where A =  y   5 6 −3     Now, A =

4 −2 = −12 + 12 = 0 6 −3

C22 = (−11)2 + 2 (4) = 4 −3 2 −3 −6 T  =  adjA =  4   2 −6 4 Thus,

So, the given system of equations is inconsistent or it has infinitely many solutions according to (adjA) B ≠ 0 or (adjA)B = 0, respectively.

−3 2 3 −9 + 10  1  = ≠0   =  (adjA)B =  −6 4 5 −18 + 20 2 Hence, the given system of equations is inconsistent.

PRACTICE EXERCISE 1 3 5 1    1. What is the rank of the matrix A = 2 4 8 0  ?    3 1 7 5   (a) 1 (b) 2 (c) 3 (d) 4  4 2 1 3   2. What is the rank of the matrix A = 6 3 4 7  ?   2 1 0 1    (a) 1 (b) 2 (c) 3 (d) 4 1 1 1    3. What is the rank of the matrix A = 1 − 1 0  ?   1 1 1    (a) 1 (b) 2 (c) 3 (d) 4 4 2 3    4. What is the rank of the matrix A = 1 0 0  ?   4 0 3    (a) 0 (b) 1 (c) 2 (d) 3  a + b 3   4 3 =  , then what are the values of 5. If  ab  5 3  5 a and b? (a) (2, 1) or (1, 2) (c) (0, 3) or (3, 0)

(b) (2, 4) or (4, 2) (d) (1, 3) or (3, 1)

 1 −2   3 4  and B =   , then what is the value 6. If A =  3 5  7 1 of 4A − 3B? 5 20  (b)    9 −17 

−5 −20 (a)   −9 17 

−5 20 (d)    9 17 

−2 −2 (c)   −4 4 

7. What is the value of  sinq −cosq   cosq  + cosq  sinq   cosq sinq  −sinq 1 1  (a)  1 1

 1 0  (b)   0 1

 sin q cos q (c)   sin q − cos q  sin q cos q (d)  0  1 8. If B =  3 then what

sinq  ? cosq 

sin q + cos q   sin q cos q   0  sin q + cos q 

4 7 , C =  1 6 is the value

 21/2 27/2  (a)  −15/2 45/2

1  , and 2A + 3B − 6C = 0, 8 of A?  21/4 27/4  (b)  −15/4 45/4

 21/4 −15/4 (d)   27/4 45/4 

 21/2 −15/2 (c)   27/2 45/2 

Chapter 1.indd 15

8/22/2017 10:53:37 AM

16     LINEAR ALGEBRA

9. Find A and B, if 8 5   6 1  and A − B =  . A+B =   8 13 2 3 7 3   1 2 , B =   (a) A =  5 8 3 5  1 2 7 3  , B =   (b) A =  3 5 5 8 5 2 3 3 , B =   (c) A =  5 6 3 7  10 5  4 4 , B =   (d) A =   5 8 3 5 10. For what values of l, the given set of equations has a unique solution? 2x + 3y + 5z = 9 7x + 3y − 2z = 8 2x + 3y + lz = 9 (a) l = 15 (b) l = 5 (c) For all values except l = 15 (d) For all values except l = 5 11. For what values of l, the given set of equations has a unique solution? x+y+z=3 x + 2y + 3z = 4 x + 4y + lz = 6 (a) 5 (c) 9

(b) 7 (d) 0

12. How many solutions does the following system of equations have? x + 2y + z = 6 2x + y + 2z = 6 x+y+z=5 (a) One solution (b) Infinite solutions (c) No solutions (d) None of the above 1 2 —7  3 —5 1     13. If A = 3 1 5  and B =  4 8 5 , then what 4 7 1  1 2 6     is the value of (A × B)?  4 −3 −31   (a) 18 3 38   41 38 45    11 −3 −31   (c) 18 9 18   41 38 35   

Chapter 1.indd 16

 4 −3 −38   (b)  −3 3 38  −31 31 45    4 3 −31   (d) 18 −3 38   45 38 41   

k 0  4 0   and B =   , then what is the 14. If A =   1 4 6 16 2 value of k for which A = B? (a) −1 (b) −2 (c) 1 (d) 2  1 0  1 0  and I =   , then what is the 15. If A =  −1 7   0 1 value of k for which A2 = 8A + kI? (a) 7 (b) −7 (c) 10 (d) 8 2 —1  —1 —8 —10     16. If 1 0  A =  1 —2 —5  ,  —3 4   9 22 15      value of A? 3 4 0  3  (a)  (b)  1 3 4  1   1 (c) 1 −2 −5 (d)  3 0  3 4 

then what is the

4 0  −2 −5 3 4  4 0

1 2 2    17. If A =  2 1 −2 is a matrix such that AAT = a 2 b    9I3, then what are the values of a and b? (a) a (b) a (c) a (d) a

= = = =

−1, −2, 1, b 2, b

b = −2 b = −1 =2 =1

 8 4 6   18. If A =  4 0 2 is singular, then what is the value  x 6 0   of x? (a) 12 (c) 4

(b) 8 (d) 1

2 3   , then what is the value of A−1? 19. If A =  5 —2 1 −2 −3 1 −2 −3     (a) (b) 19 −5 2  29 −5 2  1 2 3  1 2 3      (c) (d) 19 5 −2 29 5 −2 20. What is the value of IT, where I matrix of order 3? 1 0 0 0 0    (a) 1 0 0 (b)  0 1 1 0 0 1 0     1 1 1 1 0    (c)  0 0 0 (d)  0 1  0 0 0 0 0   

is an identity 1  0 0 0  0 1

8/22/2017 10:53:47 AM

PRACTICE EXERCISE     17

1 7 −1   21. If A = 3 2 2  , then what is the first row of AT? 4 5 1    (a) [1 7 −1] (b) [1 3 4] (c) [3 2 2] (d) [4 5 1] 1 3 3   22. If A = 1 4 3 , then what is the 1 3 4    7 −3 −3  7    (a) −1 1 0 (b) −3 −1 0  −3 1     7 −1 −1 −3    (c) −3 0 1 (d) −1 −3 1   −1 0   

value of A−1? −1 −1  1 0 0 1  −3 7   1 1 1 1

 8 4 2   23. Calculate the adjoint of the matrix A = 2 9 4 .  1 2 8   64 28 2    (a) 12 62 18   6 36 −64  

 64 −28 −2    (c) −12 62 −28  −5 −12 64   

 62 −28 −2    (b) −12 62 −38  −2 −12 −62  



64 28 24    (d) 10 62 48   6 36 −64  

 4 2 6 0  and C =   , then what is the 24. If A =  − 1 1   0 6  value of B such that AB = C?  2 1  (a)  1 2

1 −2  (b)  1 4 

 1 1  (c)  −2 4

 0 4  (d)   1 3

 2 1  —3 2  , B=  and if ABC = I2, then 25. If A =  3 2  5 —3 what is the value of C ?  2 4  (a)  1 3

 2 1  (b)  1 2

2 0   (c)  1 0

1 1  (d)  1 0

26. What is the value of (AB)−1B? (a) A−1 (c) A

Chapter 1.indd 17

(b) B (d) A−1B−1

 x 2 0   27. If A =  2 0 1 is singular, then what is the value  6 3 0   of x? (a) 0 (c) 4

(b) 2 (d) 6

0 1 1   28. What is the nullity of the matrix A =  1 −1 0 ?   −1 0 −1 (a) 0 (b) 1 (c) 2 (d) 3  4 −2  ? 29. What are the eigenvalues of A =  −2 1  (a) 1, 4 (b) 2, 3 (c) 0, 5 (d) 1, 5  3 −1 −1   30. What are the eigenvalues of A = −1 3 −1 ? −1 −1 3    (b) 1, 4, − 4 (d) 1, 2, 6

(a) 1, 4, 4 (c) 3, 3, 3

31. What is the eigenvector of the matrix A =  5 −4   ? −1 2   1 (a)   2 −2 (c)   −1

 2 (b)   2  1 (d)    0



32. What are the eigenvectors of the matrix A =  1 −1  ? −1 1   1 (a)   , 2

1   −2

1 (c)   , 1

1   −1



 2 (b)   , 1

2   −1



 1 (d)   ,  0

1   −1

33. What are the eigenvalues of the matrix A =  0 —1 1     —1 0 0  ?  1 1 1   (a)

2, − 2, 1

(c) 2, − 2, 1

(b) i, − i, 1 (d) 0,

1 1 , 2 2

8/22/2017 10:54:03 AM

18     LINEAR ALGEBRA

34. What 4 0  1 4 0 0 

are the eigenvalues of the matrix A = 0  0 ? 5

(a) 1, 2, 3 (c) 3, 5, 6

(b) 4, 4, 5 (d) 3, 3, 7

35. Which one of the following options is not the eigen 1 1 0   vector of matrix A =  0 2 2 ?  0 0 3    1   (a)  0  0  

 1   (b)  1  0  

 2   (c) 1  2  

 1   (d)  2  0  

36. What 2 5  0 3 0 1  k    (a)  0 , k   

0   k 2k   

 0   (b)  0 , k   

k     0 ,  0  

0   0 2k   

 0   (d)  0 , k   

k    k  ,  0  

0   2k  k  

 1 2 4   37. What is the sum of eigenvalues of A = 3 5 4 ? 1 6 2   (a) 8 (c) 4

(b) 10 (d) 5

x y  and 38. What is the value of x and y if A =   —4 10 eigenvalues of A are 4 and 8? (a) x = 3, y = 2

(b) x = 2, y = 4

(c) x = 4, y = 2

(d) x = 2, y = 3

 0 1 ? 39. What are the eigenvalues of A =   —1 0

are the eigenvectors of the matrix A = 0  0 ? 1 k     0 ,  0  

k    (c)  0 ,  0  

k     0 ,  0  

0   k 2k   

(a) 1, − 1 (c) i, − i

(b) 1, i (d) 0, 1

3 0 0    40. What are the values of x and y, if A = 5 x 0  3 6 y    and eigenvalues of A are 3, 4 and 1? (a) x = 2, y = 2 (c) x = 2, y = 3

(b) x = 3, y = 2 (d) x = 4, y = 1

ANSWERS 1. (c)

8. (c)

15. (b)

22. (a)

29. (c)

36. (b)

2. (b)

9. (a)

16. (c)

23. (c)

30. (a)

37. (a)

3. (b)

10. (d)

17. (b)

24. (b)

31. (b)

38. (d)

4. (d)

11. (b)

18. (a)

25. (d)

32. (c)

39. (c)

5. (d)

12. (c)

19. (c)

26. (a)

33. (a)

40. (d)

6. (a)

13. (a)

20. (d)

27. (c)

34. (b)

7. (b)

14. (d)

21. (b)

28. (b)

35. (d)

Chapter 1.indd 18

8/22/2017 10:54:13 AM

EXPLANATIONS AND HINTS     19

EXPLANATIONS AND HINTS 1 3 5 1 1. (c) Matrix A = 2 4 8 0 3 1 7 5

Hence, rank ≠ 3. Now, let us consider 2 × 2 minors 1 1 = ( —1 — 1) = —2 ≠ 0 1 —1

Maximum possible rank = 3 Now, consider 3 × 3 minors 1 3 5 2 4 8 = (28 — 8) — 2 (21 — 5) + 3 (24 — 20) 3 1 7 = 20 − 32 + 12 = 0 3 5 1 4 8 0 = 3(40 − 0) − 4(25 − 7) + 1(0 − 8) 1 7 5 = 120 − 72 − 8 = 40 ≠ 0 Hence, rank of A = 3. 4 2 1 3 2. (b) Matrix A = 6 3 4 7 2 1 0 1 Maximum possible rank = 3 Now, consider 3 × 4 6 2

3 minors 2 1 3 4 =0 1 0

2 1 3 3 4 7 =0 1 0 1 4 1 3 6 3 7 =0 2 0 1 and

4 2 3 6 3 7 =0 2 1 1

Now, because all 3 × 3 minors are zero, let us consider 2 × 2 minors 4 2 =0 6 3 2 1 = 8−3 = 5 ≠ 0 3 4 Hence, rank of A is 1  3. (b) Matrix A = 1 1  Maximum 1 1  1 —1 1 1 

2. 1 1  —1 0 1 1

rank = 3 1  0 = ( —1 — 0) — 1 (1 — 1) + 1 (0 + 1) 1 = −1 + 1 = 0

Chapter 1.indd 19

Hence, rank of A is 2.  4 2 3   4. (d) Matrix A = 1 0 0  4 0 3   Maximum rank = 3  4 2 3   1 0 0 = 4 (0) — 1 (6 — 0) + 4 (0) = —6 ≠ 0  4 0 3   Hence, rank of A = 3. 5. (d) Since both the matrices are equal  a + b 3   4 3  =  ab  5 3  5

⇒ a + b = 4 (1) ab = 3 (2) From Eq. (1), we have a=4−b Substituting the value of a in Eq. (2), we get (4 − b) b = 3 ⇒ 4b − b2 = 3 ⇒ b2 − 4b + 3 = 0 ⇒ (b − 3) (b − 1) = 0 ⇒ b = 3, 1 For values of b, a = 4 − 3, 4 − 1 = 1, 3 Therefore, the values of a and b = (1, 3) or (3, 1)

 3 4 1 —2 6. (a) A =   ,   B = 7 1   3 5   4 —8  9 12 4A =   ,   3B = 21 3    12 20   4 — 9 —8 — 12  —5 —20 4A − 3B =  =  12 — 21 20 — 3   —9 17  7. (b) We have  cos q sin q   sin q — cos q   + sin q   cos q   — sin q cos q  cos q sin q   cos2 q sin q cos q   sin2 q — sin q cos q  =  +  2 cos q   sin q cos q sin2 q   — sin q cos q  cos2 q + sin2 q sin q cos q — sin q cos q  =   cos2 q + sin2 q  — sin q cos q + sin q cos q   1 0  =  0 1

8/22/2017 11:06:18 AM

20     LINEAR ALGEBRA

8. (c) We have

11. (b) The given set of equations can be written as 2A + 3B − 6C = 0 1 A = (6C — 3B) 2

1 7   and C = Also, B =  3 1 1 1   4 1  −3 A = 6  3 2  6 8 1  24 6  3 − =   2  36 48  9 =

 4 1   6 8 7    1  21   3  

1   21 −15    2  27 45  

 21/2 −15/2  = 27/2 45/2 

1 1 1 X  3      1 2 3  Y  =  4 1 4 k   Z  6       1 1 1   Thus, 1 2 3 =1 (2k − 12) − 1 (k − 4) + 1 (3 − 2) 1 4 k    = 2k − 12 − k + 4 + 1

=k−7

Now, for a system 1  1 1 

to be unique 1 1  2 3 ≠ 0 4 k 

⇒k−7≠0 ⇒k≠7

9. (a) We have 8 5   6 1  and A − B =   A+B =   8 13 2 3  8 5   6 1 +  ∴ (A + B ) + (A − B ) =   8 13 2 3 14 6  1 14 6  7 3 ⇒A=  =  2A =  10 16 2 10 16 5 8  8 5   6 1 −  Also, (A + B) − (A − B) =   8 13 2 3 2 4  1  2 4   1 2 ⇒B=  =  2B =  2 6 10 3 5 6 10 7 3   1 2  and B =  . Thus, A =  5 8 3 5 10. (d) The given set of equations can be written as  2 3 5   x   9      7 3 −2  y  =  8  2 3 l   z   9       The system will have a unique solution if the rank of coefficient matrix is 3. Thus, 2 3 5 7 3 −2 = 2(3l + 6) − 7(3l − 15) + 2(−6 − 15) 2 3 l = 6 l + 12 − 21l + 105 − 12 − 30 = —115l + 75 = 15(5 − l ) For rank = 3, 2 3 5 7 3 −2 ≠ 0 2 3 l ∴ 15(5 − l) ≠ 0 l≠5

Chapter 1.indd 20

Thus, the value of k for which the given set of equations does not have a unique solution is 7. 12. (c) The given system can be written as  1 2 1  X   6        2 1 2  Y  =  6   1 1 1  Z   5         1 2 1   Now, 2 1 2 = 1 (1 − 2) − 2 (2 − 1) + 1 (4 − 1)  1 1 1  = −1 − 2 + 3 = 0  Hence, rank of matrix is not 3. Now, taking a minor from the matrix 1 2 = 1 − 4 = −3 ≠ 0 2 1 Hence, rank of matrix = 2. Now, rank of matrix is less than the number of variables. Hence, the system is inconsistent or has no solution. 13. (a) We have  1 2 −7   3 −5 1      A = 3 1 5    and  B =  4 8 5 4 7 1  1 2 6     3 + 8 + (−7) −5 + 16 − 14 1 + 10 − 42   A×B =  9 + 4 + 5 −15 + 8 + 10 3 + 5 + 30   12 + 28 + 1 −20 + 56 + 2 4 + 35 + 6     4 −3 −31   = 18 3 38   41 38 45    14. (d) We have k 0  4 0     and  B =   A=  1 4 6 16

8/22/2017 10:54:32 AM

EXPLANATIONS AND HINTS     21

k 0 k  A2 =   1 4  1  2 = k + 0 k+4 

Also, 2z1 − z2 = −10, z1 = −5

0  4

0 + 0  =  k2 0  0 + 16 k + 4 16

Now, since A2 = B

⇒ z2 = 0 1 —2 —5 . Thus, A =  3 4 0  17. (b) We have

 k2 0  =  4 0   k + 4 16 6 16  

1 2 2    A =  2 1 −2  a 2 b    1 2 a    AT = 2 1 2   2 −2 b   

k2 = 4 and k + 4 = 6 k = ± 2 and k = 2 Hence, value of k = 2. AAT = 9I3

15. (b) We have  1 0  1 0    and  I =   A=  —1 7   0 1  1 0  1 0  1 + 0 0   =  A2   —1 7  −1 1  —1 — 7 0 + 49 1 0  =  —8 49  1 0  8 0  =  = 8A = 8   —1 7   —8 56  1 0  k 0  =  kI = k   0 1  0 k  Now, A2 = 8A + kI  1 0   8 0  k 0  = +  ⇒  —8 49  —8 56  0 k   1 0  8 + k 0   =   —8 49  —8 56 + k  or

8 + k = 1  and  56 + k = 49 ⇒ k = −7

16. (c) As product is a 3 × 3 matrix and one of the matrix is 3 × 2, the order of A is 2 × 3.  x y1 z1   , then Consider A =  1  x2 y2 z2  2  —1 —8 —1    x1 y1 z1    =  1 —2 0  1  —3 4   x2 y2 z2   9 22     2x1 − x2 2y1 — y2 2z1 — z2     = x1 y1 z1   — + — + — + 3 x 4 x 3 y 4 y 3 z 4 z  1 2 1 2 1 2  Now, 2x1 − x2 = −1, x1 = 1

 —1 —8 —10    1 —2 —5   9 22 15   

Solving above equations, 3b = −3 ⇒ b = −1 a = −4 + 2 = −2 Hence, a = −2 and b = −1.  8 4 6   18. (a) We know that A =  4 0 2 is singular.  x 6 0   8 4 0 Hence, 4 0 2 = 0 x 6 0 8(0 − 12) − 4(0) + x (8 − 0) = 0 −96 + 8x = 0 8x = 96 x = 12 19. (c) We have 2 3   A= 5 —2 |A|= {[2 × (—2)] — (3 × 5)} = —4 — 15 = —19 ≠ 0 Hence, A is invertible. Now, cofactors of the matrix A are C11 = −2

⇒ 2 − x2 = −1

C12 = −5

⇒ x2 = 3

C21 = −3

Also, 2y1 − y2 = −8, y1 = −2 ⇒ −4 − y2 = −8 ⇒ y2 = 4

Chapter 1.indd 21

—10  —5  15 

 1 2 2  1 2 a   1 0 0       2 1 —2 2 1 2  = 9  0 1 0 a 2 b  2 —2 b   0 0 1        9 0 a + 2b + 4   9 0 0     0 9 2a + 2 — 2b  =  0 9 0  a + 2b + 4 2a + 2 — 2b a2 + 4 + b2   0 0 9   a + 2b + 4 = 0  ⇒ a + 2b = −4 2a + 2 −2b = 0  ⇒ a − b = −1

C22 = 2  —2 —5  T  —2 —3  =  adjA =  — 3 2    —5 2 

8/25/2017 6:14:34 PM

22     LINEAR ALGEBRA

A−1 =

1 −1  —2 —3 2 / 19 3 / 19   =  adj A = A 19  —5 2  5 / 19 —2 / 19

 1 0 0  1 0 0     20. (d) I =  0 1 0 IT =  0 1 0  0 0 1  0 0 1     1 7 —1   21. (b) A = 3 2 2  4 5 1   

23. (c) We have  8 4 2   A =  2 9 4  1 2 8   Now, cofactors of the matrix A are given as

 1 3 4   A =  7 2 5 −1 2 1   The first row of transport of A = [1 3 4] . T

1 3  22. (a) A = 1 4 1 3  A = 1(16 −

3  3 4 9) − 1(12 − 9) + 1(9 − 12)

= 7 − 3 − 3 = 1≠ 0

C11 =

9 4 (—1)1+1 = 64 2 8

C12 =

2 4 (—1)1+ 2 = —12 1 8

C13 =

2 9 (—1)1+ 3 = —5 1 2

C21 =

4 2 (—1)2 +1 = —28 2 8

C22 =

8 2 (—1)2 + 2 = 62 1 8

C23 =

8 4 (—1)2 + 3 = —12 1 2

C31 =

4 2 (—1)3 +1 = —2 9 4

C32 =

8 2 (—1)3 + 2 = —28 2 4

C33 =

8 4 (—1)3 + 3 = 64 2 9

Hence, A is invertible. Now, cofactors of the matrix A are given as C11 = (−1)1+1 C12 = (1)1+ 2

4 3 =7 3 4

1 3 = —1 1 4

 64 —12 —5  T  64 —28 —2      adj A =  —28 62 —12 =  —12 62 —28  —2 —28 64   —5 —12 64     

1+ 3

C13 = (−1)

1 4 = —1 1 3

C21 = (−1)2 +1

3 3 = —3 3 4

C22 = (−1)2 + 2

1 3 =1 1 4

C23 = (−1)2 + 3

1 3 =0 1 3

C31 = (−1)3 +1

3 3 = —3 4 3

Now,        AB = C

3+2

C32 = (−1)

1 3 =0 1 3

Now, we can calculate A−1 as follows:

C33 = (−1)3 + 3

1 3 =1 1 4

 7 —1 —1   Thus, adj A =  —3 1 0   —3 0 0    1 adj A Hence, A−1 = A

T

=

Chapter 1.indd 22

Thus,

 7 —3 —3 1   —1 1 0  1  —1 0 1   

 7 —3 —3    =  —1 1 0   —1 0 1   

24. (b) We know 4 A=  —1 6 C=  0

2  1 0  6

B = CA−1 |A| = 4 + 2 = 6 ≠ 0 Hence, A is invertible. Now, cofactor of the matrix A are given as C11 = (−1)1+1 (1) = 1 C12 = (−1)2+1 (−1) = 1 C21 = (−1)2+1 (2) = −2 C22 = (−1)2+2 (4) = 4  1 1 T 1 —2  =  adjA =   —2 4 1 4 

8/22/2017 10:54:54 AM

EXPLANATIONS AND HINTS     23

1 1 —2   6 1 4  6 0 1 1 —2 ×   B=  0 6 6 1 4   1 0 1 —2 1 —2  =  =  0 1 1 4  1 4 

A —1 =

Now,  2 —1 3 2 6 — 5 —9 + 10  =  C = A−1B−1 =   —3 2  5 3  4 — 3 —6 + 6  1 1  = 1 0 26. (a) (AB)−1 = (A−1B−1) B = A —1[(B —1 ) ⋅ B ]

25. (d) We know that 2 A= 3  —3 B=  5

1  2

2  —3

Now, |A| = (4 − 3) = 1 ≠ 0 |B| = (9 − 10) = −1 ≠ 0

= A−1I = A—1 27. (c) We know  x 2 0   A =  2 0 1  6 3 0   For A to be singular, |A| = 0

Hence, both A and B are invertible. Now,    

x(0 — 3) — 2(0 — 0) + 6(2) = 0 −12 −3x + 12 = 0 ⇒ x = −3 ⇒x=4



ABC = I C = A−1B−1I



or         C = A−1B−1

28. (b) We know

Calculating cofactors of A, CA = ( —1)

(2) = 2

CA = ( —1)

(3) = —3

CA = ( —1)

(1) = —1

1+1

0 1 1   A =  1 —1 0   —1 0 —1  

11

1+ 2

12

2 +1

|A| = 0(1 — 0) — 1(—1 — 0) + (—1) (0 + 1) =0+1—1=0

21

2+2

CA = (—1)

Hence, rank is not 3.

(2) = 2

22

 2 —3 T  2 —1  =  adjA =   —1 2   —3 2  1  2 —1  A —1 =  1  —3 2  Calculating cofactors of B, CB = (—1)1+1(—3) = —3 11

CB = (—1)1+ 2 (5) = —5

Choosing a minor from A, we get 0 1 = 0 — 1 = —1 ≠ 0 1 —1 Therefore, rank (A) = 2 Now, nullity = Number of columns − Rank of matrix =3—2 =1 29. (c) We know  4 —2   A=  —2 1 

12

CB

= (—1)2 +1(2) = —2 21

Now, CB

= (—1)2 + 2 (—3) = —3 22

 —3 —5  —3  = adj B =   —2 —3  —5 1 1 −3  B −1 = adj B = |B| (−1) −5  3 2  B —1 =  5 3 T

Chapter 1.indd 23

[A — lI ] =

4—l —2 =0 —2 1 — l

—2  —3

(4 − l )(1 − l ) − (−2)(−2) = 0

−2   −3

l 2 — 5l + 4 — 4 = 0 l(l − 5) = 0 l = 0, 5 Hence, eigenvalues are 0 and 5.

8/22/2017 10:55:04 AM

24     LINEAR ALGEBRA

1 — l —1    =0  —1 1 — l 

30. (a) We know  3 —1 —1   A =  —1 3 —1  —1 —1 3   

(1 — l )2 — 1 = 0

[(1 — l ) + 1][(1 — l ) — 1] = 0

Now, 3 — l —1 —1    A — lI =  —1 3 — l —1  = 0  —1 —1 3 — l   (3 — l ) (3 — l )2 — 1 — (—1)[ —(3 — l ) — 1] + (—1)[1 + (3 — l )] = 0  

[(3 — l ) + 1]{(3 — l )[(3 — l ) — 1] — (1) — 1} = 0

⇒ (2 — l )(— l ) = 0 l = 2, 0 Hence, eigenvalues of A = 2, 0  =0 Now, using A — lI X Putting l = 0, we have  1 —1  x1     =0  —1 1   x2  x1 − x2 = 0

(4 — l )  l 2 — 5l + 4 = 0 ⇒ (4 — l )(l — 1)(l — 4) = 0   l = 1, 4, 4

−x1 + x2 = 0

31. (b) We have

x1 = x2

 5 —4  A=  —1 2  The characteristic equation is given by

Putting l = 2, we have  —1 —1  x1     =0  —1 —1  x2  −x1 − x2 = 0

|A − lI| = 0 5—l —4 =0 —1 2 — l (5 — l )(2 — l ) — 4 = 0 l 2 — 7l + 6 = 0

x1 = −x2 Hence, from the given options, eigenvectors for the 1 1 corresponding eigenvalues can be   and   . 1 −1 33. (a) We have  0 —1 1   A =  —1 0 0  1 1 1  

(l − 6)(l − 1) = 0 l = 6, 1 ∴ Eigenvalues of A = 1, 6  =0 Now, using A — l X and substituting l = 1, we get  4 —4  X1   0    =    —1 1   X2   0 4X1 − 4X2 = 0 −X1 + X2 = 0

0 — l —1 1    A — lI =  —1 0 — l 0 =0  1 1 1 — l   ⇒ (0 — l )[(0 — l )(1 − l ) — 0 ] + 1[(l — 1) — 1] + 1[0 — (0 — l )] = 0 ⇒ — l  — l + l 2  + 1[ l — 2] + 1[ l ] = 0   ⇒ —l3 + l2 + l − 2 + l = 0

or             X1 = X2

⇒ l 3 — l 2 — 2l + 2 = 0

Now, the solution is X1 = X2 = k.

⇒ l 2 (l — 1) — 2(l — 1) = 0

 2 Hence, from the given options, the solution is   . 2 32. (c) We have  1 —1  A=  —1 1  The characteristic equation is given by A — lI = 0

Chapter 1.indd 24

Now,

⇒ (l 2 — 2)(l — 1) = 0 ⇒ (l + 2 )(l — 2 )(l — 1) = 0 Hence, eigenvalues are l = 2 , — 2 , 1. 34. (b) We have  4 0 0   A =  1 4 0  0 0 5  

8/22/2017 10:55:16 AM

EXPLANATIONS AND HINTS     25

Now,

4 — l 0 0    (A — lI ) =  1 4—l 0 =0  0 0 5 — l  

⇒ (4 — l )[(4 — l )(5 — l ) — 0 ] — 1[0 — 0 ] + 0 [0 — 0 ] = 0

Now putting l =  —2  0 0 

35. (d) We have  1 1 0   A =  0 2 2  0 6 3   The characteristic equation is given by A — lI = 0 1— l 1 0 0 2—l 2 =0 0 0 3—l ⇒ (1 — l )[(2 — l )(3 — l ) — 0 ] — 0 [(3 — l )(2 — l ) — 0 ] +0 [2 — 0 ] = 0 ⇒ (1 — l )(2 — l )(3 — l ) = 0 l = 1, 2, 3  =0 Using | A − lI | X and putting l = 1, we get  0 1 0   x  0        0 1 2  y  =  0  0 0 2  z   0       y=0 y − 2z = 0 2z = y 2z = 0 Hence, x = k, y = 0, z = 0. k    Therefore, the eigenvector can be of the form  0 .  0   Now putting l = 2 , we get  —1 1 0  x  0       0 0 2  y  =  0  0 0 1  z   0       −x + y = 0    ⇒ x = y 2z = 0    ⇒ z = 0

 0    0  0  

−2x + y = 0 ⇒ x = y/2

⇒ (4 — l )(4 — l )(5 — l ) = 0 ∴ Eigenvalues of the matrix are l = 4, 4, 5.

3, we get 1 0   x   —1 2   y  = 0 0  z 

−y + 2z = 0 ⇒ y = 2z 2k    Therefore, the eigenvector can be of the form  k  . 2k    Hence, x = 2k, y = k, z = 2k.

Hence, the eigenvector which is not of the matrix  1   A is  2 .  0   36. (b) We have  2 5 0   A =  0 3 0  0 1 1   The characteristic equation is given by A — lI = 0 2—l 5 0 0 3—l 0 0 1 1— l ⇒ (2 — l )(3 — l )(1 — l ) = 0 l = 1, 2, 3  =0 Now, using A — lI X Putting l = 1, we get  1 5 0   x  0        0 2 0  y  =  0  0 1 0  z   0       x + 5y = 0 2y = 0 y=0

 0   Thus, the eigenvector can be of the form  0 . k    Putting l = 2, we get  0 5 0   x  0        0 1 0   y  =  0  0 1 —1  z   0       5y = 0 y=0

z=0 Hence, x = k, y = k, z = 0. k    Therefore, the eigenvector can be of the form k  .  0  

Chapter 1.indd 25

y−z=0⇒y=z k    Thus, the eigenvector can be of the form  0 .  0  

8/22/2017 10:55:26 AM

26     LINEAR ALGEBRA

Putting »l = 3, we get  —1 5 0   x  0       0 0 0   y  =  0  0 1 —2  z   0       −x = 0 ⇒ x = 0 y − 2z = 0 ⇒ y = 2z 0   Thus, the eigenvector can be of the form  k  . 2k    Hence, the eigenvectors for the corresponding eigenvalues are given as

⇒y =

12 =3 4

Hence, x = 2 and y = 3. 39. (c) We have  0 1  A=  —1 0 The characteristic equation is given by A — lI = 0 0—l 1 =0 —1 0 — l

 0  k   0         0 ,  0 ,  k  k   0 2k       

⇒ l2 + 1 = 0 ⇒ l 2 = —1 ⇒l=±i

37. (a) We have  1 2 4   A =  3 5 4 1 6 2   Sum of eigenvalues = sum of diagonal elements of matrix A

40. (d) We know 3 0 0    A = 5 x 0  3 6 y    Now, eigenvalues of A are 1, 3 and 4. Sum of eigenvalues = Sum of diagonal

=1+5+2

∴3+x+y=1+3+4

=8

⇒x+y=5 Also, product of eigenvalues = |A|

38. (d) We know that x y  A=  —4 10 Also, we know that the eigenvalues of A are 4 and 8. Now, sum of eigenvalues = sum of diagonal elements ∴ 4 + 8 = x + 10 ⇒ x = 12 − 10 = 2 Also, product of eigenvalues = |A| ∴ 4 × 8 = 10x + 4y ⇒ 4 × 8 = 10 × 2 + 4y

∴ 3(xy − 0) − 5(0 − 0) + 3(0 − 0) = 1 × 3 × 4 ⇒ xy = 4 Now, y = 5 − x ⇒ (5 − x)x = 4 ⇒ 5x − x2 = 4 ⇒ x2 − 5x + 4 = 0 ⇒ (x − 4)(x − 1) = 0 ⇒ x = 1, 4 ∴ y = 4, 1 Hence, from the given options x = 4, y = 1

⇒ 4y = 32 − 20

SOLVED GATE PREVIOUS YEARS’ QUESTIONS  4 2 1 3   1. Given a matrix [A ] = 6 3 4 7  , the rank of the  2 1 0 1   matrix is (a) 4 (c) 2

(b) 3 (d) 1 (GATE 2003, 1 Mark)

Chapter 1.indd 26

Solution:  Consider four 3 × 3 minors because maximum possible rank is 3. 4 2 1 6 3 4 =0 2 1 0 2 1 3 3 4 7 =0 1 0 1

8/22/2017 10:55:32 AM

SOLVED GATE PREVIOUS YEARS’ QUESTIONS     27

Solution:  The given equations are

4 1 3 6 4 7 =0 2 0 1

x + 2y + z = 6 2x + y + 2z = 6 x+y +z = 5

4 2 3 6 3 7 =0 2 1 1

The given system can be written as  1 2 1  x 6       2 1 2  y  =  6   1 1 1  z   5      

As all 3 × 3 minors are zero, rank cannot be 3. We now try 2 × 2 minors.

 1 2 1 6   The augmented matrix is given by 2 1 2 6 .    1 1 1 5

4 2 =0 6 3 2 1 = 8−3 = 5 ≠ 0 3 4 Hence, rank = 2. 

Ans. (c)

 4 1  , the eigenvalues are 2. For the matrix   1 4 (a) 3 and −3 (b) −3 and −5 (c) 3 and 5 (d) 5 and 0

 1 2 1   Now, rank of the matrix A = 2 1 2 can be cal  culated as  1 1 1 1 2 1 2 1 2 1 1 1 = (1)(1 − 2) − (2)(2 − 1) + (1)(4 − 1) = 0

(GATE 2003, 1 Mark) Solution:  We have  4 1  A=  1 4 Now, the characteristic equation is given by

Hence, rank (A) is not 3. Taking a minor from the matrix A, we get 1 2 2 1 = 1 − 4 = −3 ≠ 0

A − lI = 0

Hence, rank (A) is 2. Now taking a minor from the augmented matrix, we get

4 − l 1 =0 ⇒ 1 4 − l   ⇒ (4 − l )2 − 1 = 0 ⇒ (4 − l )2 − (1)2 = 0 ⇒ (5 − l )(3 − l ) = 0

Therefore, the eigenvalues are given by l = 3 and l = 5. l = 3 and l = 5. 

Ans. (c)

3. Consider the following system of simultaneous equations: x + 2y + z = 6 2x + y + 2z = 6 x+y +z = 5

1 2 6 2 1 6 1 1 5 = (1)(5 − 6) − (2)(10 − 6) + (1)(12 − 6) = −1 − 8 + 6 = − 3

Hence, the rank of augment matrix is 3. As rank of coefficient matrix ≠ rank of augmented matrix, the system has no solution, i.e. system is inconsistent. 

Ans. (c)

4. Consider the following system of linear equations: This system has (a) a unique solution (b) infinite number of solutions (c) no solution (d) exactly two solutions (GATE 2003, 2 Marks)

Chapter 1.indd 27

 2 1 −4  x a        4 3 −12  y  =  5  1 2 −8   z   7        Notice that the second and third columns of the coefficient matrix are linearly dependent.

8/22/2017 10:55:40 AM

28     LINEAR ALGEBRA

For how many values of a, does this system of equations have infinitely many solutions?

Thus, [F]T [C]T [B] [C] [F] has a size 1 × 1. Therefore, it is a scalar.

(a) 0 (c) 2

Hence, statement 1 is true.

(b) 1 (d) Infinitely many (GATE 2003, 2 Marks)

Solution:  The given system of equations is  2 1 −4  x a        4 3 −12  y  =  5  1 2 −8   z   7        The augmented matrix for the given system is  2 1 −4 a     4 3 −12 5    1 2 −8 7  For infinite solutions to exist, the rank of the augmented matrix should be less than the total unknown variables, i.e. 3. Therefore, 2 1 a 4 3 5 =0 1 2 7

Consider statement 2: [D]T [F] [D] is always symmetric. Now, [D]T [F] [D] does not exist because DT3×5, F5×1 and D5×3 are not compatible for multiplication as DT 3×5 F5×1 = X3×1 and X3×1 D5×3 do not exist. Hence, statement 2 is false. 

6. Let A, B, C and D be n × n matrices, each with non-zero determinant. If ABCD = I, then B−1 (a) is D−1C−1A−1 (b) is CDA (c) is ADC (d) does not necessarily exist (GATE 2004, 1 Mark)

⇒ a (8 − 3) − 5(4 − 1) + 7(6 − 4) = 0 1 ⇒a = 5

Solution:  Given that A, B, C and D are n × n matrices. Also, ABCD = I − ABCDD−1C −1 = D−1C −1 − AB = D−1C −1

Therefore, there is only one value of α for which infinite solutions exist.  Ans. (b) 5. Real matrices [A]3×1, [B]3×3, [C ]3×5, [D]5×3, [E ]5×5 and [F]5×1 are given. Matrices [B] and [E] are symmetric. The following statements are made with respect to these matrices. T

T



1. Matrix product [F] [C] [B] [C] [F] is a scalar.



2. Matrix product [D]T [F] [D] is always symmetric. With reference to the above statements, which one of the following options applies? (a) Statement 1 is true but statement 2 is false (b) Statement 1 is false but statement 2 is true (c) Both the statements are true (d) Both the statements are false (GATE 2004, 1 Mark) Solution:  Statement 1 is true as shown below. [F]T has a size 1 × 5 [C]T has a size 5 × 3 [B] has a size 3 × 3 [C] has a size 3 × 5 [F] has a size 5 × 1

Chapter 1.indd 28

Ans. (b)

− A−1AB = A−1D−1C −1 − B = A−1D−1C −1 Now, B−1 = (A−1D−1C −1 )−1 = (C −1 )−1(D−1 )−1(A−1 )−1 = CDA  Ans. (b) 7. The sum of the eigenvalues of the matrix given  1 2 3   below is  1 5 1 . 3 1 1   (a) 5 (c) 9

(b) 7 (d) 18 (GATE 2004, 1 Mark)

Solution:  Sum of eigenvalues of given matrix = sum of diagonal elements of given matrix = 1 + 5 + 1 = 7.  Ans. (b) 8. For what value of x become singular?  8   4 12 

will the matrix given below x 0  0 2 6 0

8/22/2017 10:55:45 AM

SOLVED GATE PREVIOUS YEARS’ QUESTIONS     29

(a) 4 (c) 8

(b) 6 (d) 12 (GATE 2004, 2 Marks)

 8 x 0   Solution:  For singularity of matrix,  4 0 2 = 0 12 6 0   ⇒ 8 (0 − 12) − x (0 − 2 × 12) = 0 ⇒ −96 + 24x = 0 ⇒x=4 Therefore, x = 4. 

 4 −2   10. The eigenvalues of the matrix  1 −2 (a) are 1 and 4 (b) are −1 and 2 (c) are 0 and 5 (d) cannot be determined (GATE 2004, 2 Marks) Solution:  We have  4 −2   A= 1 −2

Ans. (a)

9. How many solutions does the following system of linear equations have? −x + 5y = −1 x−y = 2 x + 3y = 3 (a) infinitely many (b) two distinct solutions (c) unique solution (d) no solution (GATE 2004, 2 Marks)

Characteristic equation is given by A − lI = 0 ⇒

−2 4−l =0 −2 1 − l

⇒ [(4 − l ) × (1 − l )] − [(−2) × (−2)] = 0 ⇒ l 2 − 5l = 0 ⇒ l(l − 5) = 0 Hence, l = 0 and 5 are the eigenvalues. 

Solution:  The given equations are −x + 5y = −1 x−y = 2 x + 3y = 3

Ans. (c)

11. Consider the matrices X(4×3) , Y(4×3) and P(2×3) . The order of [P(XTY)−1 PT]T will be (a) (2 × 2) (c) (4 × 3)

The system of equations can be represented as −1 5    −1   x    1 −1   =  2   1 3  y   3     

(b) (3 × 3) (d) (3 × 4) (GATE 2005, 1 Mark)

Solution:  We know that the order of X → 4 × 3, Y → 4 × 3 and P → 2 × 3.

−1 5 −1   The augmented matrix is given by  1 −1 2 .   3 3  1

Hence, we can calculate the following orders:

Using Gauss elimination on above matrix, we get

Also, P T → 3 × 2

−1 5 −1 −1 5 −1     R2 + R1  1 −1 2   →  0 4 1 R + R     1 3 3 3  0 8 2  1

XT → 3 × 4, XT Y → 3 × 3 and (XT Y )−1 → 3 × 3

Therefore, (P (XT Y )−1 P T )T → 2×2 and P (XT Y )−1 P T → (2 × 3)(3 × 3) P (X Y ) P T → (2 × 3)(3 × 3)(3 × 2) → 2 × 2 T

−1 5 −1     →  0 4 1    0 0 0

−1



Ans. (a)

R3 −2R2

12. Consider a non-homogeneous system of linear equation representing mathematically an overdetermined system. Such a system will be

Rank[A| B ] = 2(number of non-zero rows in [A| B ]) Rank[A ] = 2(number of non-zero rows in [ A ]) Rank[A| B ] = Rank [A ] = 2 = number of variables Therefore, the system has a unique solution. 

Chapter 1.indd 29

Ans. (c)

(a) consistent having a unique solution (b) consistent having many solutions (c) inconsistent having a unique solution (d) inconsistent having no solution (GATE 2005, 1 Mark)

8/22/2017 10:55:53 AM

30     LINEAR ALGEBRA

Solution:  In an over-determined system having more equations than variables, the three possibilities that still exist are consistent unique, consistent infinite and inconsistent with no solution. Therefore, options (a), (b) and (d) are correct.  Ans. (a), (b), (d)

1  0 (c)  0 0 

0 1 0 0

0 0 1 0

13. A is a 3 × 4 real matrix and Ax = b is an inconsistent system of equations. This highest possible rank of A is

1  4  0  (d)   0   0 

0  0 0 1



0

0

1 4

0

0

1 4

0

0

 0   0   0  1  4 

(GATE 2005, 2 Marks) (a) 1    (b) 2    (c) 3    (d) 4 (GATE 2005, 1 Mark) Solution:  We know that rank (Am×n ) ≤ min(m, n) Therefore, the highest possible rank is 3. If the rank of A is 3, then the rank of [A | B ] would also be 3, which means the system would become consistent. However, it is given that the system is inconsistent. Hence, the maximum rank of A is only 2.  Ans. (b) 14. In the matrix equation Px = q, which one of the following options is a necessary condition for the existence of at least one solution for the unknown vector x? (a) Augmented matrix [Pq] must have the same rank as matrix P (b) Vector q must have only non-zero elements (c) Matrix P must be singular (d) Matrix P must be square (GATE 2005, 1 Mark) Solution:  Rank [Pq] = Rank [P] is necessary for existence of at least one solution to Px = q.  Ans. (a) 1 1 1 1   1 −1 −1 1 15. Given an orthogonal matrix A =  . 0 0  1 −1 0 0 1 −1  −1 What is the value of  AA T  ?   1  4  0  (a)   0   0 

Chapter 1.indd 30

0

0

1 4

0

0

1 2

0

0

 0   0   0  1  2 

1   0 0 0 2     0 1 0 0   2  (b)    1 0 0 0   2   1 0 0 0   2 

Solution:  For orthogonal matrix, we know that A ⋅ AT = I, i.e. identity matrix Therefore, (A ⋅ AT)−1 = I  —1 = 1. 

Ans. (c)

 1 0 −1   16. If R = 2 1 −1 , then top row of R−1 is 2 3 2  (b) [5 −3 1] (d) [2 −1 1/2]

(a) [5 6 4] (c) [2 0 −1]

(GATE 2005, 2 Marks) Solution:  We have  1 0 −1   R = 2 1 −1 2 3 2  R −1 =

adj(R) |R|

1 0 −1 ⇒ |R| = 2 1 −1 = 1(2 + 3) − 0 (4 + 2) − 1(6 − 2) 2 3 2 = 5−4 =1 As we need only the top row of R−1, we need to find only the first row of adj (R). adj(1, 1) = +

1 −1 = 2+3 = 5 3 2

adj(1, 2) = +

0 −1 = −3 1 −1

adj(1, 3) = +

0 −1 = +1 1 −1

Dividing by |R| = 1 gives the top row of R−1 = [5 −3 1]. 

Ans. (b)

8/22/2017 10:55:58 AM

SOLVED GATE PREVIOUS YEARS’ QUESTIONS     31

1   2 −0.1  a  and A−1 =  2 17. Let A =  , then (a + b) is 3  0  0 b   7 3 (a) (b) 20 20 11 19 (c) (d) 20 60 (GATE 2005, 2 Marks) Solution:  We know that [AA−1 ] = 1

2 −1 3 3 −2 5 = (2)(−2 + 20) − (3)(−1 + 12) −1 −4 1 + (−1)(−5 + 6) = 36 − 33 − 1 = 2 ≠ 0 Hence, rank (A) = 3 and rank (A|B) = 3. As rank [A| B ] = rank [A ] = number of  variables, the system has a unique solution. 

 2 −0.1 1 / 2 a  1 0  =  ⇒ 3  0 b   0 1  0

Ans. (b)

19. Which one of the following options is an eigenvec-

 1 2a − 0.1b   1 0 =  ⇒ 3b   0 1  0 ⇒ 2a − 0.1b = 0 ⇒ a = 0.1b/2 

(1)

1 3 Now substituting b in equation (1), we get 2b = 1 ⇒ b =

1 60 1 1 1 + 20 21 7 So, a + b = + = = = 60 30 60 60 20 a=



Now,

Ans. (a)

5  0 tor of the matrix  0 0 

3x1 − 2x2 + 5x3 = 2 −x1 − 4x2 + x3 = 3 This system of equations has (a) no solution (b) a unique solution (c) more than one but a finite number of solutions (d) infinite number of solutions (GATE 2005, 2 Marks) Solution:  The equations can be represented as follows:  2 −1 3  x1     3 −2 5  x2  = −1 −4 1     x3    

1     2    3  

Chapter 1.indd 31

0  0 ? 1 1

(GATE 2005, 2 Marks) Solution:  We first calculate the eigenvalues by solving characteristic equation A − lI = 0. 5−l 0 0 0 0 5−l 5 0 |A − lI | = =0 0 0 2−l 1 0 0 3 1− l ⇒ (5 − l )(5 − l )[(2 − l )(1 − l ) − 3] = 0 ⇒ (5 − l )(5 − l )(l 2 − 3l − 1) = 0 l = 5, 5,

3 ± 13 2

 = 0, we get Putting l = 5 in [A − lI ]X 5 − 5 0 0 0  x1   0      0 5−5 5 0  x2   0   =  0 0 2−5 1  x3   0   0 0 3 1 − 5  x4   0 

The augmented matrix for the given system is  2 −1 3 1    3 −2 5 2    −1 −4 1 3

0 5 2 3

(b)  0    0  1    0   (d)  1   −1  2    1  

(a)  1   −2  0    0   (c)  1    0  0   −2  

18. Consider the following system of equations in three real variables x1, x2 and x3: 2x1 − x2 + 3x3 = 1

0 5 0 0

0  0 ⇒ 0 0 

0 0 0  x1   0     0 5 0  x2   0  = 0 −3 1  x3   0 0 3 −4  x4   0

8/22/2017 10:56:09 AM

32     LINEAR ALGEBRA

which gives the equations

Putting x2 = k and x3 = 0 in Eq. (1), we get

5x3 = 0

(1)

−3x3 + x4 = 0

(2)

3x3 − 4x4 = 0

(3)

5 x1 − 2(k) + 2 × 0 = 0 ⇒ x1 = 2/5k Therefore, eigenvectors are of the form  x1  2/5k       x2  =  k       x3   0

Solving Eqs. (1)−(3), we get x3 = 0, x4 = 0, and x1 and x2 may take any value. The eigenvector corresponding to l = 5 may be written as  x1   k1       1 =  x2  = k2  X  x3   0  x   0  4   

⇒ x1 : x2 : x3 = 2/5k : k : 0 = 2/5 : 1 : 0 = 2 : 5 : 0.

where k1 and k2 may be any real number. As only option (a) has x3 and x4 equal to 0, this is our desired answer. 

Ans. (a)

 3 −2 2    20. For the matrix A =  0 −2 1 , one of the eigen0 0 1  values is equal to −2. Which one of the following options is an eigenvector? (a)  3 (b) −3     − 2    2  1  −1     (c)  1   −2  3  

 x1   2     Therefore,  x2  =  5 is an eigenvector of matrix A.      x3   0  Ans. (d) −4 2  , the eigenvector is 21. Given the matrix   4 3 (a) 3 (b)  4     2  3 (c)  2 (d) −1     −1  2 (GATE 2005, 2 Marks) Solution:  First, find the eigenvalues of A = −4 2  .  4 3

(d)  2    5  0  

The characteristic equation is given by A − lI = 0. ⇒

(GATE 2005, 2 Marks)

−4 − l 2 =0 4 3−l

⇒ (−4 − l )(3 − l ) − 8 = 0 Solution:  As matrix is triangular, the eigenvalues are the diagonal elements themselves, namely l = 3, −2 and 1. Corresponding to eigenvalue l = −2, let us find the eigenvector using the following equation:

⇒ l 2 + l − 20 = 0 ⇒ (l + 5)(l − 4) = 0 ⇒ l1 = −5 and l2 = 4

 =0 [A − lI ]X 3 − l −2 2  x1   0      0 −2 − l 1  x2  =  0     0 0 1 − l   x3   0 

Corresponding to l1 = −5, we need to find the eigenvector.  = 0. The eigenvalue problem is [ A − lI ] X

Putting l = −2 in the above equation, we get



 5 −2 2  x1   0      0 1  x2  =  0 0   0 0 3  x3   0  which gives the equations

Chapter 1.indd 32

−4 − l 2 =0 4 3−l

Putting l = −5, we get

5x1 − 2x2 + 2x3 = 0 

(1)

x3 = 0

(2)



 1 2  x1   0    =    4 8  x2   0 x1 + 2x2 = 0 

3x3 = 0

(3)



4x1 + 8x2 = 0 

(1) (2)

8/22/2017 10:56:20 AM

SOLVED GATE PREVIOUS YEARS’ QUESTIONS     33

As Eqs. (1) and (2) are same, we take

x1 + 2x2 = 0 

(3)

x1 = −2x2 x1 : x2 = −2 : 1 x ⇒ 1 = −2 x2



Now from the given options, we look for the eigenvector having the ratio −2 : 1 and find that option (c) is the appropriate one. So, option (c) is an eigenvector corresponding to l = −5.  Ans. (c) 22. What are the eigenvalues of the following 2 × 2 matrix?  2 −1   5 −4 (a) −1 and 1 (c) 2 and 5

Solution:  Although lim will be the corresponding eigenvalues of Am , xim need not be corresponding eigenvectors. Ans. (b)

1 1 1   24. The rank of the matrix 1 −1 0 is 1 1 1  (a) 0 (b) 1 (c) 2 (d) 3 (GATE 2006, 1 Mark) Solution:  We have 1 1 1   1 −1 0 1 1 1  Now, 1 1 1 1 −1 0 = 1(−1 − 0) − 1(1 − 1) + 1(0 + 1) = 0 1 1 1

(b) 1 and 6 (d) 4 and −1 (GATE 2005, 2 Marks)

Hence, rank is not 3. Now considering a minor of matrix, we have

Solution:  We have  2 −1  A= 5 −4

1 1 = (−1 − 1) = −2 ≠ 0 1 −1

The characteristic equation of this matrix is given by A − lI = 0 −1 2−l =0 −4 5 − l

Hence, rank = 2. 

(2 − l )(5 − l ) − 4 = 0 l = 1, 6

Ans. (b)

23. Consider the system of equations A(n × n )x(n × t ) = l(n × I ) , where l is a scalar. Let (li , xi ) be an eigen-pair of an eigenvalue and its corresponding eigenvector for real matrix A. Let I be a (n × n) unit matrix. Which one of the following statements is not correct? (a) For a homogeneous n × n system of linear equations, (A − lI )x = 0 has a non-trivial solution, the rank of (A − lI ) is less than n. (b) For matrix Am, m being a positive integer, (lim , lim ) will be the eigen-pair for all i. (c) If AT = A−1, then −li− = 1 for all i. (d) If AT = A, then li is real for all i. (GATE 2005, 2 Marks)

Chapter 1.indd 33

Ans. (c)

25. Solution for the system defined by the set of equations 4 y + 3 z = 8, 2 x − z = 2 and 3 x + 2 y = 5 is

Therefore, the eigenvalues of A are 1 and 6. 

Therefore, its rank is the number of non-zero rows in this form.

(a) x = 0, y = 1, z = 4/3 (b) x = 0, y = 1/2, z = 2 (c) x = 1, y = 1/2, z = 2 (d) non-existent (GATE 2006, 1 Mark) Solution:  The augmented matrix for given system is 0 4  2 0 −1 2  3 8     Exchange 1st and 2nd row  2 0 −1 2    →  0 4 3 8     0 5 0 5  3 2  3 2 Now, by Gauss elimination procedure  2 0 −1 2  2 0 3 −1 2  R3 − R1     2 0 4 3 8   3 8 → 0 4     0 5  0 2 3 / 2 2  3 2  2 0 −1 2  1 R3 − R2   2  3 8   →0 4   0 −2  0 0

8/22/2017 10:56:28 AM

34     LINEAR ALGEBRA

27. Match List I with List II and select the correct answer using the codes given below the lists:

Now, considering A. 2 0 −1 0 4 3 =0 0 0 0

List I

List II

A. Singular matrix

1. Determinant defined

Taking a minor of matrix A, we get 2 0 = 8−0 ≠ 0 0 4

not

B. Non-square matrix 2. Determinant is always one C. Real symmetric

Hence, rank (A) = 2. Now, taking a minor of matrix (A|B).



5. Eigenvalues defined

are

not

Codes:

Hence, rank (A|B) = 3. Since rank (A) ≠ rank (A|B), the system of equations is inconsistent. Therefore, solution is non-existent for above system.  Ans. (d) 26. The multiplication of matrices E and Matrices E and G are 1 0  cos q − sin q 0    E =  sin q cos q 0 and G =  0 1   0 0 0 0 1  

3. Determinant is zero

D. Orthogonal matrix 4. Eigenvalues are always real

0 −1 2 4 3 8 = −2(0 + 4) = −8 ≠ 0 0 0 −2

What is the matrix F ? (a)  cos q − sin q 0   cos q 0  sin q  0 0 1    (c) cos q sin q 0   − sin q cos q 0  0 0 1 

is

F is G.

   A  (a) 3  (b) 2  (c) 3  (d) 3 

B  C  D 1  4  2 3  4  1 2   5  4 4   2  1 (GATE 2006, 2 Marks)

0  0 1

(b)  cos q cos q 0   − cos q sin q 0  0 0 1  (d)  sin q − cos q 0   sin q 0  cos q  0 0 1 

Solution: A. Singular matrix → Determinant is zero B. Non-square matrix → Determinant is not defined C. Real symmetric → Eigenvalues are always real D. Orthogonal matrix → Determinant is always one  Common Data for Questions 28 and 29 −10T   P =  −1 , Q =  3  

Ans. (a)

−2T  2T     −5 and R = −7  are three vectors.  9  12    

(GATE 2006, 2 Marks) Solution:  We have  cos q − sin q 0   E =  sin q cos q 0 and G =  0 0 1  Now, it is given that E×F=G 1 0  cos q − sin q 0    ⇒  sin q cos q 0 × F =  0 1  0 0 0 0 1   We can see that product of E and matrix, so F has to be E −1.

28. An orthogonal set of vectors having a span that contains P, Q and R is  1 0 0    0 1 0  0 0 1  

0  0 1 F is a unit

 cos q sin q 0 adj(E ) 1   = − sin q cos q 0 |E | 1 0 0 1   cos q sin q 0   = − sin q cos q 0  0 0 1 

Chapter 1.indd 34

 4   −2  3   −3  3      2  9 −2 −4    

(b) −4    2  4   (d)  4    3 11  

 5    7 −11    1   31  3  

 8    2 −3    5    3  4  

(GATE 2006, 2 Marks)

F = E −1 =



(a) −6   − 3   6   (c)  6    7 −1  

Solution:  We are looking for orthogonal vectors having a span that contains P, Q and R.  4 −6     Considering option (a), −3 and −2  3  6     Firstly, these are orthogonal because their dot product is given by (−6 × 4) + (−3 ×−2) + (6 × 3) = 0

Ans. (c)

8/22/2017 10:56:37 AM

SOLVED GATE PREVIOUS YEARS’ QUESTIONS     35



The space spanned by these two vectors is −6  4     k1 −3 + k2 −2   6  3    

(1)

−6  4     The span of −3 and −2 contains P, Q and R.  6  3     We can show this by successively setting Eq. (1) to P, Q and R one by one and solving for k1 and k2 uniquely. Also, the options (b), (c) and (d) are wrong because none of them are orthogonal as can be seen by taking pairwise dot products. 

Ans. (a)

29. The following vector is linearly dependent upon the solution to the previous problem: (a)  8 (b)  −2      9 −17   3  30     (c)  4 (d)  13      4  2  5 −3    

 4 2 30. For the matrix   , the eigenvalue correspond 2 4 101 ing to the eigenvector   is 101   (a) 2    (b) 4    (c) 6    (d) 8 (GATE 2006, 2 Marks) Solution:  We have  4 2  M=  2 4 The characteristic equation is given by 4 − l 2  [M − lI ] =  2 4 − l   101 The given eigenvector is   . 101    [M − lI ]X = 0 4 − l 2 101  ⇒ =0 2 4 − l  101  ⇒ (4 − l )(101) + 2 × 101 = 0 ⇒ 404 − 101l + 202 = 0 ⇒l = 6 

Ans. (c)

(GATE 2006, 2 Marks) Solution:  We find the linear dependency by finding the determinant of the vectors taken as a matrix. Considering option (a) and finding the determinant −6 −3 6   of  4 −2 3 , we get  8 9 3  −6 −3 6 4 −2 3 = −6(−6 − 27) + 3(12 − 24) 8 9 3 + 6(36 + 16) ≠ 0

−6 −3 6 4 −2 3 = −6(−60 + 51) + 3(120 + 6) −2 −17 30 + 6(−68 − 4) = 0

(b) 3, −5 (d) 3, 5



(GATE 2006, 2 Marks)

Solution:  We know that

∑ li = Trace (A) l 1 + l 2 + l 3 = Trace (A) = 2 + (−1) + 0 = 1 ∴ 3+l2 +l3 =1 ⇒ l 2 + l 3 = −2 Only option (b) satisfies the condition.  Ans. (b)  3 2  are 5 and 1. 32. The eigenvalues of a matrix S =  2 3 What are the eigenvalues of the matrix S 2 = SS ? (a) 1 and 25 (c) 5 and 1

Hence, it is linearly dependent.

Chapter 1.indd 35

(a) 2, −5 (c) 2, 5

Now l 1 = 3

Considering option (b) and finding the determi−6 −3 6   nant of  4 −2 3 , we get −2 −17 30  



 2 −2 3    31. For a given matrix A = −2 −1 6 , one of the  1 2 0  eigenvalues is 3. The other two eigenvalues are

Ans. (b)

(b) 6 and 4 (d) 2 and 10 (GATE 2006, 2 Marks)

8/22/2017 10:56:47 AM

36     LINEAR ALGEBRA

Solution:  If l 1 , l 2 , l 3 ,… are the eigenvalues of A. m m Then the eigenvalues of Am are l m 1 , l 2 , l 3 ,…. Here, S has eigenvalues 1 and 5. So, S 2 has eigenvalues 12 and 52, i.e. 1 and 25. 

Ans. (a)

33. The eigenvalues and the corresponding eigenvectors or a 2 × 2 matrix are given by eigenvalues l1 = 8 and l2 = 4 and eigenvectors

35. X = [x1 , x2 ,..., xn ]T is an n-tuple non-zero vector. The n × n matrix V = XX T (a) has rank zero (b) has rank 1 (c) is orthogonal (d) has rank n (GATE 2007, 1 Mark) Solution:  If X = [x1 , x2 ,..., xn ]T

1 1  v1 =   , v2 =   1 −1    

Now, it is clear that rank (X) = 1. rank (XT) = rank (X) = 1

The matrix is  6 2  4 6   (a)  (b)   2 6 6 4  2 4  4 8   (c)  (d)   8 4  4 2

Now, rank (X T ) ≤ (rank X , rank X T ) ⇒ rank(XX T ) ≤ 1 So, XXT has a rank of either 0 or 1.

(GATE 2006, 2 Marks) Solution:  According to the property of eigenvalues, sum of diagonal elements should be equal to sum of values of l. So, ∑ l i = l 1 + l 2 = 8 + 4 = 12 = Trace (A) Hence, trace (A) = 12. Only option (a) satisfies the property.  Ans. (a) 34. [A] is a square matrix which is neither symmetric nor skew-symmetric and [A]T is its transpose. The sum and difference of these matrices are defined as [S] = [A] + [A]T and [D] = [A] − [A]T, respectively. Which one of the following statements is true? (a) Both [S] and [D] are symmetric (b) Both [S] and [D] are skew-symmetric (c) [S] is skew-symmetric and [D] is symmetric (d) [S] is symmetric and [D] is skew-symmetric

However, as both X and XT are non-zero vectors, neither of their ranks can be zero. Therefore, XXT has a rank 1.  Ans. (b) 36. The minimum and maximum given values of the  1 1 3   matrix  1 5 1 are −2 and 6, respectively. What 3 1 1   is the other eigenvalue? (a) 5 (c) 1

(b) 3 (d) −1 (GATE 2007, 1 Mark)

Solution:  We know that

∑ l1 = Trace(A) ⇒ l1 + l2 + l3 = 1+ 5 +1 = 7 Now, given that l 1 = −2 and l 2 = 6.

(GATE 2007, 1 Mark)

∴ −2 + 6 + l 3 = 7 ⇒ l 3 = 3

Solution:  Since 

Ans. (b)

S T = (A + A T )T = A T + (A T )T = A T + A = S 37. If a square matrix A is real and symmetric, then the eigenvalues

⇒ ST = S Therefore, [S] is symmetric. Since D T = (A − A T )T = A T − (A T )T = A T − A

(a) are always real (b) are always real and positive (c) are always real and non-negative (d) occur in complex conjugate pairs

= −(A − A T ) = −D

(GATE 2007, 1 Mark)

⇒ D T = −D Therefore, [D] is skew-symmetric. 

Chapter 1.indd 36

Solution:  The eigenvalues of any symmetric matrix are always real. Ans. (d)



Ans. (a)

8/22/2017 10:56:56 AM

SOLVED GATE PREVIOUS YEARS’ QUESTIONS     37

 1 2  is 38. The inverse of the 2 × 2 matrix  5 7  1 −7 2 (a)   5 − 1  3 1  7 −2  (c)   1 3 −5



Here, X = {x ∈ R 3 | x1 + x2 + x3 = 0}

1 7 2  (b)   3  5 1 1 −7 −2 (d)   3 −5 −1

xT = {x1 , x2 , x3 }T Now, {[1, −1, 0]T , [1, 0, −1] } is a linearly independent set because it cannot be obtained from another by scalar multiplication. The fact that it is independent can also be established by seeing that 1 −1 0  is 2. the rank of  0 −1 1 T

(GATE 2007, 2 Marks)  1 2  is given by Solution:  Inverse of A =  5 7  1 (adj(A)) A

Next, we need to check if the set spans X.

 7 −2  1 1  7 −2 1 −7 2  =  =   ⇒ 1 −3 −5 1 3  5 −1 (7 − 10) −5 

2. They must span X.

Ans. (a)

Here,   X = {x ∈ R 3 | x1 + x2 + x3 = 0} −k1 − k2    The general infinite solution of X is  k1  .   k2  

39. It is given that X1 , X2 ,..., XM are M non-zero,  k   0  k  k  orthogonal vectors. The dimension of the vector k1, k2 as  1  =   and  1  =   , we get X2 ,..., − X space spanned by the 2M vectors X 1 , X2 ,...., XM , −X1 , −Choosing k2  k  k2   0 M X 1 , X2 ,...., XM , −X1 , −X2 ,..., − XM is two linearly independent solutions, for X, (a) 2M −k  −k  (b) M + 1     (c) M X =  0 or  k   k  0 (d) dependent on the choice of X1 , X2 ,..., XM     (GATE 2007, 2 Marks) Solution:  Since (X1 , X2 ,..., XM ) are orthogonal, they span a vector space of dimension M.

Now, since both of these can be generated by linear combinations of [1, −1, 0 ]T and [1, 0, −1]T , the set spans X. Also we have shown that the set is not only linearly independent but also spans X, therefore by definition it is for the subspace X.

Since (−X1 , −X2 ,..., XM ) are linearly dependent on X1 , X2 ,..., XM , the set X 1 , X2 ,...., XM , −X1 , −X2 ,..., − XM Ans. (a) X 1 , X2 ,...., XM , −X1 , −X2 ,..., − XM will also span a vector 41. For what values of a and b, the following space of dimension M only. ­simultaneous equations have infinite number of  Ans. (c) solutions? 40. Consider the set of (column) vector defined 3 T = (x , x , x )T by    X = {x ∈ R | x 1 + x2 + x3 = 0, where x 1 2 3 x}+ y + z = 5; x + 3y + 3z = 9; x + 2y + αz = b

x2 + x3 = 0, where xT = (x1 , x2 , x3 ) }. Which one of the following options is true? T

(a) {(1, −1, 0)T ,(1, 0, −1)T } is a basis for the subspace X. (b) {(1, −1, 0)T ,(1, 0, −1)T } is a linearly independent set, but it does not span X and therefore is not a basis of X. (c) X is a subspace for R3 (d) None of the above (GATE 2007, 2 Marks) Solution:  To be basis for subspace X, two conditions are to be satisfied.

Chapter 1.indd 37

1. The vectors have to be linearly independent.

(a) 2, 7 (c) 8, 3

(b) 3, 8 (d) 7, 2 (GATE 2007, 2 Marks)

Solution:  The augmented matrix for this system is given by 1 1 1 5   1 3 3 9   1 2 a b  Now, for the system to have infinite solutions, rank of the augmented matrix should be less than the total number of unknown variables, i.e. 3.

8/22/2017 10:57:08 AM

38     LINEAR ALGEBRA

Hence,

The solution is therefore x2 = 0 and x1 can take any value  x1  k   =   x   0  2  

1 1 1 1 3 3 =0 1 2 a ⇒ (1)(3a − 6) − (1)(a − 2) + (1)(3 − 3) = 0 ⇒ 3a − 6 − a + 2 = 0 ⇒ 2a = 4 ⇒ a = 2

As there is only one parameter in infinite solutions, there is only one linearly independent eigenvector for  1 k  this problem which may be written as   or as  0 .  0  

Also,

 1 1 5 1 3 9 =0 1 2 b

43. The linear operation L(x) is defined by the cross product L(x) = b × X, where b = [0 1 0]T and X = [x1 x2 x3]T are three-dimensional vectors. The 3 × 3 matrix M of this operation satisfies

⇒ (1)(3b − 18) − (1)(b − 10) + (1)(9 − 15) = 0 ⇒ 3b − 18 − b + 10 − 6 = 0 ⇒ 2b = 14 ⇒ b = 7 Hence, for the system to have infinite solutions, a = 2 and b = 7.  Ans. (a) 42. The number of linearly independent eigenvectors  2 1  is of   0 2 (a) 0 (c) 2

(b) 1 (d) infinite (GATE 2007, 2 Marks)

Solution:  We have  2 1  A=  0 2 Eigenvalues can be calculated using the following characteristic equation: A − lI = 0 2 − l 1 =0 ⇒ 0 2 − l   ⇒ (2 − l )2 = 0

 =0 A − lI X 2 − l 1  x1   0   =   ⇒ − l 0 2    x2   0

(a) 0, + 1, −1 (c) i, −i, 1

(b) 1, −1, 1 (d) i, −i, 0 (GATE 2007, 2 Marks)

Solution:  We have b = [0 1 0]T and X = [x1 x2 x3]T. The cross product of b and X can be written as i j k b×x = 0 1 0 = x3 i + 0 j − x1 k x1 x2 x3 = [x3

0 x1 ]

 x1    Now, let L(x) = b × x = A  x2  , where A is a 3 × 3    x3  matrix.  x1  c3     c6  . Now, A  x2  = b × x    c9   x3  c2 c3   x1   x3    c5 c6   x2  =  0    c8 c9   x3  −x1   

Comparing L.H.S.  0 0   0 0 −1 0 

Putting l = 2, we get  0 1  x1   0    =    0 0  x2   0

Chapter 1.indd 38

Then the eigenvalues of M are

 c1   c4  c7

Now, consider the eigenvalue problem

x2 = 0

 x1    L(x) = M  x2     x3 

 c1 c2  Let A = c4 c5  c7 c8

⇒l = 2



Ans. (b)

(1)

and R.H.S., we get 1  x1   x3      0  x2  =  0     0  x3  −x1 

 0 0 1   So, A =  0 0 0 −1 0 0  

8/22/2017 10:57:18 AM

SOLVED GATE PREVIOUS YEARS’ QUESTIONS     39

Now, eigenvalues of A are calculated using characteristic equation

Solution:  In the previous question, we derived A2 + 3A + 2I = 0.

|A − lI | = 0

⇒ A2= − 3A − 21

−l 0 1 ⇒ 0 −l 0 =0 −1 0 −l

A 4 = A2 × A2 = (−3A − 2I )(−3A − 2I ) = 9A2 + 12A + 4I = 9(−3A − 2I ) + 12A + 4I = −15A − 14I

⇒ −l(l 2 − 0) − 1(0 + l ) = 0 ⇒ l3 + l = 0

A8 = A4 × A4 = (−15A − 141)(−15A − 141)

⇒ l (l + l ) = 0 2

⇒ l = 0, l = ±1

= 225A2 + 420A + 156I = 225(−3A − 21) + 420A + 156I = −225A − 254I

So, the eigenvalues of A are i, −i and 0.  Ans. (d) Common Data for Questions 44 and 45  Cayley— Hamilton theorem states that a square matrix satisfies its own characteristic equation. Consider a matrix

A9 = A × A8 = A( − 255A − 254I ) = −255A2 − 254A = −255(−3A − 2I ) − 254A = 511A + 510I

−3 2  A=  −1 0



Hence, A9 = 511A + 510I .

44. A satisfies the relation (a) A + 3I + 2 A−1 = 0 (b) A2 + 2A + 2I = 0 (c) (A + 1) (A + 2) (d) exp (A) = 0 (GATE 2007, 2 Marks)



Ans. (a)

46. The product of matrices (PQ) −1

−1

P is

−1

(a) P (c) P−1 Q−1 P

(b) Q (d) PQ P−1 (GATE 2008, 1 Mark)

Solution:  We have −3 2  A=  −1 0

Solution:  We have (PQ)−1 P = (Q−1P −1 )P = (Q−1 )(P −1P ) = (Q−1 )(1) = Q−1

Now, A − lI = 0 

−3 − l 2 ⇒ =0 −1 0 − l

the four entries of the 2 × 2 matrix  p11 p12   are non-zero, and one of its eigenP =  p21 p22  values is zero. Which one of the following statements is true?

47. All

⇒ (−3 − l )(−l ) + 2 = 0 ⇒ l 2 + 3l + 2 = 0 A will satisfy this equation according to Cayley— Hamilton theorem, i.e.       A2 + 3A + 2I = 0 Multiplying by A−1 on both sides, we get

(a) p11 p22 − p12 p21 = 1 (b) p11 p22 − p12 p21 = −1 (c) p11 p22 − p12 p21 = 0 (d) p11 p22 + p12 p21 = 0 (GATE 2008, 1 Mark)

A−1A2 + 3A−1A + 2A−1I = 0

Solution:  Since Πl i = A and if one of the eigenvalues is zero, then

⇒ A + 3I + 2A−1 = 0 

Ans. (b)

Ans. (a)

Πli = A = 0

9

45. A equals Now, (a) 511 A + 510 I (c) 154 A + 155 I

(b) 309 A + 104 I (d) exp (9A) (GATE 2007, 2 Marks)

Chapter 1.indd 39

A =

p11 p21

p12 =0 p22

⇒ p11 p22 − p12 p21 = 0

Ans. (c)

8/22/2017 10:57:29 AM

40     LINEAR ALGEBRA

48. The characteristic equation of a (3 × 3) matrix P is defined as a(l ) = lI − P = l 3 + l 2 + 2l + 1 = 0 If I denotes identity matrix, then the inverse of matrix P will be (a) (P 2 + P + 2I )

(b) (P 2 + P + 1)

(c) (−P 2 + P + 1)

(d) − (P 2 + P + 2I ) (GATE 2008, 1 Mark)

4 2 7  The augmented matrix [A|B] is given by   2 1 6  4 2  is given by Rank of the matrix A =   2 1 4 2 = 4−4 = 0 2 1 Hence, the rank of the matrix A is 1. The rank of the augmented matrix [A|B] is calculated by considering the minor

Solution:  If characteristic equation is l 3 + l 2 + 2l + 1 = 0

2 7 = 12 − 7 = 5 ≠ 0 1 6

By Cayley—Hamilton theorem, P 3 + P 2 + 2P + I = 0

Hence, the rank of the augmented matrix is 2. As rank [A|B ] ≠ rank[A ], the system has no solution.  Ans. (b)

I = −P − P − 2P 3

Solution:  The system can be written in the matrix form as  4 2  x 7     =    2 1  y  6

2

Multiplying by P−1 on both sides, we get P −1 = −P 2 − P − 2I = − (P 2 + P + 2I )

51. The following system of equations: x1 + x2 + 2x3 = 1

Ans. (d)

x1 + 2x3 + 3x3 = 2

49. If the rank of a (5 × 6) matrix Q is 4, then which one of the following statements is correct?

x1 + 4x2 + ax3 = 4



(a) Q will have four linearly independent rows and four linearly independent columns (b) Q will have four linearly independent rows and five linearly independent columns (c) QQT will be invertible (d) QT Q will be invertible

(a) 0 (b) either 0 or 1 (c) one of 0, 1 or −1 (d) any real number other than 5 (GATE 2008, 1 Mark)

(GATE 2008, 1 Mark)

Solution:  The equations can be represented as follows:

Solution:  If rank of (5 × 6) matrix is 4, then surely it must have exactly 4 linearly independent rows as well as 4 linearly independent columns, since rank = row rank = column rank.  Ans. (a) 50. The system of linear equations 4x + 2y = 7 2x + y = 6 has (a) a unique solution (b) no solution (c) infinite number of solutions (d) exactly two distinct solutions (GATE 2008, 1 Mark)

Chapter 1.indd 40

has a unique solution. The only possible value(s) for a is/are

1 1 2   x1  1    1 2 3   x2  = 2 1 4 a        x3   4   The augmented matrix for the above system is 1 1 2 1 1 1 2 1     R2 −R1 1 2 3 2   1 1 → 0 1 R − R     1 3  0 3 a − 2 3 1 4 a 4 1 1 2 1   R3 −3R2 1 1   → 0 1    0 0 a − 5 0 Now as long as a − 5 ≠ 0, rank(A) = rank(A | B) = 3. Therefore, a can take any real value except 5.  Ans. (d)

8/22/2017 10:57:38 AM

SOLVED GATE PREVIOUS YEARS’ QUESTIONS     41

 1 2 4   52. The matrix 3 0 6 has one eigenvalue equal to 3.  1 1 p   The sum of the two eigenvalue is

For the system to have a unique solution, rank of the augmented matrix should be equal to the number of unknown variables. Hence, system will not have a unique solution if rank of augmented matrix is less than 3.

(a) p (b) p − 1 (c) p − 2 (d) p − 3

1 1 1 1 2 3 =0 1 4 k ⇒ (1)(2k − 12) − (1)(k − 4) + (1)(3 − 2) = 0 ⇒ k − 12 + 4 + 1 = 0 ⇒ k = 7

(GATE 2008, 1 Mark) Solution:  We know that Sum of the eigenvalues of the matrix = Sum of diagonal values present in the matrix = Trace of matrix ∴ 1+ 0 + p = 3+ l2 + l3



⇒ p +1 = 3 + l2 + l3

2x + 3y = 4 x+y +z = 4 x + 2y − z = a

Ans. (c)

53. A is m × n full rank matrix with m > n and I is the identity matrix. Let matrix A ′ = (A T A)−1 A T , then which one of the following statements is true? (a) AA ′A = A (c) AA ′A = 1

Ans. (d)

55. For what value of a, if any, will the following system of equations in x, y and z have a solution?

⇒ l 2 + l 3 = p +1−3 = p −2 

Therefore, when k = 7, unique solution is not possible and only infinite solutions are possible.

(b) (AA ′)2 (d) AA ′A = A ′

(a) Any real number (b) 0 (c) 1 (d) There is no such value (GATE 2008, 2 Marks)

(GATE 2008, 2 Marks) Solution:  The system can be represented as Solution:  Since

2 3 0  x  4     1  y  =  4 1 1    1 2 −1   a   z   

AA ′A = A[(A T A)−1 A T ]A = A (A T A)−1 A T A   T Let        A A = X Then     A[X −1X ] = A ⋅ I = A Hence, the correct option is (a). 

Ans. (a)

54. The following simultaneous equations: x+y +z = 3 x + 2y + 3z = 4 x + 4y + kz = 6 will not have a unique solution for k equal to (a) 0 (c) 6

(b) 5 (d) 7 (GATE 2008, 2 Marks)

Solution:  The augmented matrix for given system 1 1 1 3   is 1 2 3 4   1 4 k 6

Chapter 1.indd 41

2 3 0 4   1 4 Augmented matrix is given by  1 1    1 2 −1 a Performing Gauss elimination on this matrix, we get 2 2 3 0 4 R − 1 R 3 0 4     1 2 2 1 1 1 4   1 2 →  0 −1/2     R3 − 1 R1 1/2 −1 a − 2  0  1 2 −1 a 2 2 3 0 4   R3 +R2   → 0 −1/2 1 2   0 0 a 0 If a ≠ 0, r(A) = 2 and r(A | B) = 3, then the system will have no solutions. If a = 0, r(A) = r(A | B) = 2, then the system will be consistent and will have infinite solutions. 

Ans. (b)

8/22/2017 10:57:48 AM

42     LINEAR ALGEBRA

 =X  2 = 1 , we get Putting the value of l = 2 and X b  −1 2 1    =0  0 0 b

4 5  are 56. The eigenvalues of the matrix P =   2 −5 (a) −7 and 8 (b) −6 and 5 (c) 3 and 4 (d) 1 and 2



(GATE 2008, 2 Marks)

Solution:  We have 4 5  A=  2 −5 Characteristic equation of A is given by

⇒ −1 + 2b = 0 1           ⇒ b =  2

From Eqs. (1) and (2), we get a+b = 0+

A − lI = 0

 1 0  0 1 1 −1 −1 0  , ,  and    0 0  0 0 1 −1  1 −1

⇒ l 2 + l − 30 = 0

Ans. (b)

 1 2  are written 57. The eigenvectors of the matrix   0 2  1 1 in the from   and   . What is a + b? a b  (b) 1/2 (d) 2 (GATE 2008, 2 Marks) Solution:  We have  1 2  A=  0 2 Eigenvalues can be calculated using the following characteristic equation: A − lI = 0 (1 − l ) 2 =0 0 (2 − l ) ⇒ (1 − l )(2 − l ) = 0 ⇒ l = 1 and 2 Now, since the eigenvalue problem is  =0 [A − lI ]X 1 − l 2   0   X =   0 2 − l    0  =X  1 =  1 , we Putting the value of l = 1 and X a get  0 2  1   =0 ⇒  0 1 a      ⇒ a = 0  (1)

Chapter 1.indd 42

Ans. (b)

58. How many of the following matrices have an eigenvalue 1?

⇒ (4 − l )(−5 − l ) − (2 × 5) = 0

(a) 0 (c) 1

1 1 = 2 2



4−l 5 ⇒ =0 2 −5 − l

The eigenvalues are l = 5 and − 6. 

(2)

(a) One (c) Three

(b) Two (d) Four (GATE 2008, 2 Marks)

 1 0  can be calculated Solution:  Eigenvalues of   0 0 using characteristic equation, 1− l 0 =0 0 0−l (1 − l ) − l = 0 l = 0, 1  0 1  is given by Eigenvalues of   0 0 −l 1  =0 0 − l   l2 = 0 l = 0, 0 1 −1  is given by Eigenvalues of  1 1 1 − l −1  =0 1 1 − l   (1 − l )2 + 1 = 0 (1 − l )2 = −1 1 − l = i, −i l = (1 − i), (1 + i)) −1 0  is given by Eigenvalues of   1 −1

8/22/2017 10:58:02 AM

SOLVED GATE PREVIOUS YEARS’ QUESTIONS     43

−1 − l 0   1 −1 − l  

⇒ 

(−1 − l )(−1 − l ) = 0

Ans. (a)

61. The trace and determinant of a 2 × 2 matrix are known to be −2 and −35, respectively. Its eigenvalues are

(1 + l )2 = 0 l = −1, − 1 So, only one matrix has an eigenvalue of 1 which  1 0 . is   0 0  Ans. (a)

(a) −30 and −5 (c) −7 and 5

(GATE 2009, 1 Mark)

Trace(A) = −2

(b) BT = B (d) B−1 = BT Also, (GATE 2009, 1 Mark)

Solution:  A square matrix B is defined as skewsymmetric if and only if BT = −B, by definition. 

∑ l 1 = Trace(A)

Therefore, l 1 + l 2 = −2 

(a) − 4 5 3 (c) 5

(b) − 3 5 4 (d) 5 (GATE 2009, 1 Mark)

Solution:  Given that MT = M 1 So, MTM = 1 3  ⇒  5 4 5 

 3 4  x    5 5  =  1 0   3  3   0 1 x 5   5 

  3 2    + x2   5  ⇒   4 3  3  ⋅  + ⋅ x  5 5  5

 3 4  3   ⋅  + x  5 5  5   1 0 =    4 2  3 2   0 1   +     5   5  

Comparing both sides of a12, we get  3  4   3  a12 =     +   x = 0  5   5   5 

(1)

Also, Πl i = A = −35 ⇒ l 1l 2 = −35 ⇒ l 2 =

Ans. (a)

3 4   60. For a matrix [M] =  5 5  , the transpose of the x 3  5   matrix is equal to the inverse of the matrix, [M]T = [M]−1. The value of x is given by

Chapter 1.indd 43

(b) −37 and −1 (d) 17.5 and −2

Solution:  We know that

59. A square matrix B is skew-symmetric if (a) BT = −B (c) B−1 = B

3 3 4 4 x=− ⋅ ⇒x=− 5 5 5 5

−35 (2) l1

Now substituting l 2 from Eq. (2) in Eq. (1), we get l1 +

−35 = −2 ⇒ l12 + 2l 1 − 35 = 0 λ1 ⇒ (l 1 + 7)(l 1 − 5) = 0

l 1 = −7 or 2 If l 1 = −7 , then l 2 = 5. 

Ans. (c)

 −1 3 5   62. The eigenvalues of the matrix −3 −1 6 are  0 0 3  (a) 3, 3 + 5j, 6 − j (b) −6 + 5j, 3 + j, 3 − j (c) 3 + j, 3 − j, 5 + j (d) 3, −1 + 3j, −1 − 3j (GATE 2009, 2 Marks) Solution:  Sum of eigenvalues = Trace(A) = (−1) + (−1) + 3 = 1 So, Σl i = 1. Only option (d) (3, −1 + 3j, − 1 − 3j) gives Σl i = 1. 

Ans. (d)

63. The eigenvalues of a skew-symmetric matrix are (a) always zero (b) always pure imaginary

8/22/2017 10:58:14 AM

44     LINEAR ALGEBRA

(c) either zero or pure imaginary (d) always real

Performing Gauss elimination on this, we get  1 2 1 4 2  1 2 1 4 2 R2 −3R1      → 3 6 3 12 6  0 0 0 0 0

(GATE 2010, 1 Mark) Solution:  Eigenvalues of a skew-symmetric matrix are either zero or pure imaginary.  Ans. (c)

rank(A) = rank(A | B) = 1 Therefore, the system is consistent. As the system has rank(A) = rank(A | B) = 1 which is less than the number of variables, only infinite (multiple) non-trivial solutions exist.  Ans. (d)

 3 + 2i i  is 64. The inverse of the matrix   −i 3 − 2i 1  3 + 2i −i   i 3 − 2i 12 

(a)

(b)

1 3 − 2i −i   i 3 + 2i 12 

1  3 + 2i −i   i 3 − 2i  14 

(c)

(d)

1 3 − 2i −i   i 3 + 2i  14 

(GATE 2010, 2 Marks) Solution:  We have 3 + 2i i  A=  −i 3 − 2i Therefore,

 2 2  is 66. One of the eigenvectors of the matrix A =   1 3  2  2 (a)   (b)   − 1    1  4  1 (c)   (d)    1 −1 (GATE 2010, 2 Marks) Solution:  We have

−1

A

2 A=  1 Eigenvalues of A are ­characteristic equation,

−1  3 + 2i 1 i  = = (adj(A))  −i 3 − 2i A

3 − 2i 1 −i   = 2  i 3 + 2i [(3 + 2i)(3 − 2i) + i ]  1 3 − 2i −i   = i 3 + 2i 12  

2  3 calculated

using

the

|A − lI | = 0 2 − l 2  =0 1 3 − l   ⇒ (2 − l )(3 − l ) − 2 = 0 ⇒ l 2 − 5l + 4 = 0 ⇒ l = 1, 4

Ans. (b)

65. For the set of equations

 =0 The eigenvalue is given by [A − lI ]X

x1 + 2x2 + x3 + 4x4 = 2

2 − l 2  x1   0    =   1 3 − l   x2   0 

3x1 + 6x2 + 3x3 + 12x4 = 6 the following statement is true:

Putting l = 1, we get

(a) Only the trivial solution x1 = x2 = x3 = x4 = 0 exists (b) There is no solution (c) A unique non-trivial solution exists (d) Multiple non-trivial solutions exist

1 2  x1   0    =   1 2  x2   0 x1 + 2x2 = 0 

(1)



x1 + 2x2 = 0 

(2)



(GATE 2010, 2 Marks) Solution:  The given equations are x1 + 2x2 + x3 + 4x4 = 2 3x1 + 6x2 + 3x3 + 12x4 = 6 The augmented matrix is given by  1 2 1 4 2   3 6 3 12 6

Chapter 1.indd 44

Solution is x2 = k and x1 = −2k  1 = −2k  ⇒ x : x = −2 : 1 X 1 2  k   2 The option (a)   is in same ratio of x1 : x2 . −1  Ans. (a)

8/22/2017 10:58:28 AM

SOLVED GATE PREVIOUS YEARS’ QUESTIONS     45

 1 1 0   67. An eigenvector of P =  0 2 2 is  0 0 3   (a) [−1 1 1]T

(b) [1 2 1]T

(c) [1 − 1

(d) [2

2]T

1 − 1]T

(GATE 2010, 2 Marks)  1 1 0   Solution:  Given, P =  0 2 2  0 0 3  

Hence, the solution is x3 = 0, x1 = k, x2 = k. k   2 = k  ⇒ x : x : x = 1 : 1 : 0. Therefore, X 1 2 3  0   As none of the eigenvectors given in the options is of this ratio, we need the third eigenvector also. By putting l = 3 in the eigenvalue problem, we get −2 1 0  x1   0       0 −1 2  x2  =  0    0 0 0  x3   0  −2x1 + x2 = 0

Since P is triangular, the eigenvalues are the diagonal elements themselves. Eigenvalues are therefore l 1 = 1, l 2 = 2 and l 3 = 3.  =0 Now, the eigenvalue problem is [A − l ]X 1 − l 1 0  x1   0      0 2−l 2  x2  =  0     0 0 3 − l   x3   0  Putting l1 = 1, we get the eigenvector corresponding to this eigenvalue  0 1 0  x1   0       0 1 2  x2  =  0  0 0 2    0     x3    which gives the equations x2 = 0 x2 + 2x3 = 0 2x3 = 0 The solution is x2 = 0, x3 = 0, and x1 = k. k     So, one eigenvector is X =  0 .  0   Hence, x1 : x2 : x3 = k : 0 : 0. We need to find the other eigenvectors corresponding to the other eigenvalues because none of the eigenvectors given in the options matches with this ratio. Now, corresponding ing set of equations eigenvalue problem, −1 1   0 0  0 0  which gives

to l2 = 2, we get the followby substituting l = 2 in the 0  x1   0     2  x2  =  0   1  x3   0

−x1 + x2 = 0 2x3 = 0 x3 = 0

Chapter 1.indd 45

−x2 + 2x3 = 0 Putting x1 = k, we get x2 = 2k and x3 = x2/2 = k  k  3 = 2k  ⇒ x : x : x = 1 : 2 : 1. Therefore, X 1 2 3  k    1   Only the eigenvector given in option (b) 2 is in  1   this ratio.  Ans. (b) 68. Consider the following matrix:  2 3  A=  x y  If the eigenvalues of A are 4 and 8, then (a) x = 4, y = 10 (c) x = −3, y = 9

(b) x = 5, y = 8 (d) x = −4, y = 10 (GATE 2010, 2 Marks)

Solution:  Sum of eigenvalues = Trace (A) = 2 + y Product of eigenvalues = |A| = 2y − 3x Therefore,

4 + 8 = 2 + y y = 10 4 × 8 = 2y − 3x

(1) (2)

Substituting the value of y in Eq. (2), we get

20 − 3x = 32 ⇒ x = −12 / 4 = −3

Hence, x = −4 and y = 10.  Ans. (d) 69. Eigenvalues of a real symmetric matrix are always (a) positive (c) real

(b) negative (d) complex (GATE 2011, 1 Mark)

8/22/2017 10:58:39 AM

46     LINEAR ALGEBRA

Solution:  Eigenvalues of a symmetric matrix are always real.  Ans. (c)

Solution:  The augmented matrix for the system of equations is 1 1 1 6   R3 = R3 −R2 [A | B ] = 1 4 6 20   →[A | B ] 1 4 λ m   

70. Consider the following system of equations: 2x1 + x2 + x3 = 0 x2 − x3 = 0

1 1 6 1    = 1 4 6 20  0 0 λ − 6 m − 20  

x1 + x2 = 0 This system has

(a) a unique solution (b) no solution If l = 6 and m ≠ 20, then rank(A | B) = 3 and rank(A) = 2, (c) infinite number of solutions rank(A | B) = 3 and rank(A) = 2, which is the required condition. (d) five solutions Hence, the given system of equations has no (GATE 2011, 2 Marks) ­solution for l = 6 and m ≠ 2. Solution:  The system can be represented as  Ans. (b) 2 1 1  x  0      72. Consider the following matrix:  0 1 −1  y  =  0 1 1      0  z   0  1 2 3    0 4 7 The augmented matrix is given by  0 0 3   2 1 1 0   Which one of the following options provides the [ A | B ] =  0 1 −1 0    correct values of the eigenvalues of the matrix? 1 1 0 0   (a) 1, 4, 3 (b) 3, 7, 3 Now, rank of matrix A is given by (c) 7, 3, 2 (d) 1, 2, 3 2 1 1 (GATE 2011, 2 Marks) 0 1 −1 1 1 0 Solution:  As the given matrix is upper triangu= (2)(0 + 1) + (1)(−1 − 1) = 0 lar, its eigenvalues are the diagonal elements themselves, which are 1, 4 and 3. Hence, the correct Hence, rank (A) ≠ 3. Now, considering a minor option is the first one. from the matrix, we get 

2 1 = 2−0 = 2 ≠ 0 0 1 Thus, rank (A) = 2. Therefore, the rank of augmented matrix is 2. Now, rank (A) = rank (A|B) = 2 < number of unknown variables = 3 So, the system has infinite number of solutions. 

Ans. (c)

71. The system of equations x+y +z = 6 x + 4y + 6z = 20 x + 4y + l z = m has no solution for values of l and m given by (a) l = 6, m = 20

(b) l = 6, m ≠ 20

(c) l ≠ 6, m = 20

(d) l ≠ 6, m ≠ 20 (GATE 2011, 2 Marks)

Chapter 1.indd 46

Ans. (a)

73. Let A be the 2 × 2 matrix with elements a11 = a12 = a21 = +1 and a22 = −1. Then the eigenvalues of the matrix A19 are (a) 1024 and −1024 (b) 1024 2 and −1024 2 (c) 4 2 and −4 2 (d) 512 2 and −512 2 (GATE 2012, 1 Mark) Solution:  We have 1 1  A= 1 −1 Eigenvalues of A are the roots of the characteristic polynomial given as follows:

8/22/2017 10:58:51 AM

SOLVED GATE PREVIOUS YEARS’ QUESTIONS     47

 5 3 , one of the normalized 75. For the matrix A =   1 3 eigenvectors is given as  1   1         2  (a)   (b)  2   3   −1       2   2 

A − lI = 0 1− l 1 =0 1 −1 − l (1 − l )(−1 − l ) − 1 = 0 −(1 − l )(1 + l ) − 1 = 0 l2 − 2 = 0 l =± 2 Eigenvalues of A are

   (c)    

2 and − 2 , respectively.

So, eigenvalues of A19 = ( 2 ) , (− 2 ) 19

19

3   10   −1   10 

   (d)    

1   5   2   5 

(GATE 2012, 2 Marks) = 219/2 , − 219/2 Solution:  The matrix is given by = 29 ⋅ 21/2 , − 29 ⋅ 21/2

 5 3  A=  1 3 Characteristic equation is given by

= 512 2 , − 512 2 . 

Ans. (d)

74. The system of algebraic given below has x + 2y + z = 4 2x + y + 2z = 5 x−y +z =1

A − lI = 0 ⇒

⇒ (5 − l )(3 − l ) − 3 = 0

(a) a unique solution of x = 1, y = 1 and z = 1 (b) only two solutions of (x = 1, y = 1, z = 1) and (x = 2, y = 1, z = 0) (c) infinite number of solutions (d) no feasible solution

⇒ l 2 − 8l + 12 = 0 ⇒ l = 2, 6 Now, to find eigenvectors  =0 [A − lI ]X 5 − l 3  X1   0  ⇒ = − 1 3 l    X2   0

(GATE 2012, 2 Marks) Solution:  The given system is x + 2y + z = 4 2x + y + 2z = 5 x−y +z =1

Putting l = 2 in the above equation, we get 3 3  x1   0    =    1 1  x2   0

Use Gauss elimination method as follows: The augmented matrix is given by 1 1 2 1 4 2 1 4     R2 −2R1 [A|B ] = 2 1 2 5   →  0 − 3 0 −3  R3 −R1      1 −1 1 1  0 −3 0 −3 1 2 1 4   R3 −R2   →  0 −3 0 −3   0 0 0  0 Hence, rank (A) = rank (A|B) = 2 and the system is consistent. Now, rank (A|B) is less than the number of unknown variables. Therefore, we have infinite number of solutions.  Ans. (c)

Chapter 1.indd 47

5−l 3 =0 1 3−l

⇒ 3x1 + 3x2 = 0 



(1)

(2)               x1 + x2 = 0  Eqs. (1) and (2) are the same equations whose solution is given by  x1  −k   =   x2   k   1      Hence, the eigenvector is  2  .  −1     2  

Ans. (b)

8/22/2017 10:59:02 AM

48     LINEAR ALGEBRA

 9 5  are 76. The eigenvalues of matrix   5 8 (a) −2.42 and 6.86 (b) 3.48 and 13.53 (c) 4.70 and 6.86 (d) 6.86 and 9.50 (GATE 2012, 2 Marks)

78. There are three matrixes P(4 × 2), Q(2 × 4) and R(4 × 1). The minimum number of multiplication required to compute the matrix PQR is (GATE 2013, 1 Mark) Solution:  If we multiply QR first, then Q2×4 × R4×1 will have multiplication number 8. Therefore, P4×2 QR2×1 will have minimum number of multiplication = (8 + 8) = 16.

Solution:  We have  9 5  A=  5 8 The characteristic equation is given by

79. The dimension of the null space of the matrix  0 1 1   0 is  1 −1 −1 0 −1  (a) 0 (b) 1 (c) 2 (d) 3 (GATE 2013, 1 Mark)

A − lI = 0 9−l 5 =0 5 8−l (9 − l )(8 − l ) − 25 = 0

Solution:  We know that  0 1 1   A =  1 −1 0 −1 0 −1 

⇒ l − 17 l + 47 = 0 2

⇒l=

17 ± 289 − 188 2

So, eigenvalues are l = 3.48 and 13.53. 

Order of matrix = 3 Now, A = (−1)(−1 − 0) + (−1)(0 + 1) = 0

Ans. (b)

Hence, rank (A) = 2 Therefore, dimension of null space of A = 3 − 2 = 1.  Ans. (b)

77. Given that  1 0 −5 −3  and I =  , A= 0  2  0 1

80. Which one of the following options does not equal

the value A3 is (a) 15A + 12I (c) 17A + 15I

1 x x2 1 y y ?

(b) 19A + 30I (d) 17A + 15I

1 z (GATE 2012, 2 Marks)

1 x + 1 x2 + 1 1 x(x + 1) x + 1 (a) 1 y(y + 1) y + 1 (b) 1 y + 1 y 2 + 1 1 z(z + 1) z + 1 1 z + 1 z2 + 1

 1 0 −5 −3  and I =   Solution:  We have A =  0  0 1  2 Characteristic equation of A is given by

0 x − y x2 − y 2

−5 − l −3 =0 2 0−l (−5 − l )(−l ) + 6 = 0 ⇒ l 2 + 5l + 6 = 0 So, A2 + 5A + 6I = 0 theorem)

(by

Cayley—Hamilton



Multiplying by A on both sides, we have A 3 = − 5A 2 − 6 A ⇒ A3 = −5(−5A − 6I ) − 6A = 19A + 30I

Chapter 1.indd 48

(c) 0

y −z

1

z

2 x + y x2 + y 2

y 2 − z 2 (d) 2 z2

1

y +z

y2 + z2

z

z2

(GATE 2013, 1 Mark)

⇒ A 2 = − 5A − 6 I



z2

Ans. (b)



Solution:  We know that (i) In a matrix if we add or subtract any constant to any row or column, then the determinant of the matrix remains same. (ii) In a matrix if we apply any row or column transformation, then also the determinant of the matrix remains same. (iii) In a matrix if we interchange two rows or columns, then the determinant of the resultant matrix is multiplied by −1.

8/22/2017 10:59:12 AM

SOLVED GATE PREVIOUS YEARS’ QUESTIONS     49

So by concepts (i) and (ii), matrix in options (b), (c) and (d) gives the same result as our given matrix. By concept (iii), matrix in option a gives a different result 1 x x2 1 y

y2

1 z

z2

Ans. (a)

81. If the A-matrix of the state space model of a SISO linear time interval system is rank deficient, the transfer function of the system must have (a) a pole with a positive real part (b) a pole with a negative real part (c) a pole with a positive imaginary part (d) a pole at the origin (GATE 2013, 1 Mark) Solution:  A pole at the origin. 

Ans. (d)

sin x, sin2 x and cos2 x cos x, sin x and tan x cos 2x, sin2 x and cos2 x cos 2x, sin x and cos x (GATE 2013, 1 Mark)

Solution:  The correct set of linearly dependent functions is cos x, sin x and tan x.  Ans. (b) 2 −2  x1   0    =   has 83. The equation   1 −1  x2   0 (a) no solution  x1   0 (b) only one solution  x  =  0  2   (c) non-zero unique solution (d) multiple solutions (GATE 2013, 1 Mark) Solution:  The system of equations given is 2 −2  x1   0    =    1 −1  x2   0 2x1 − 2x2 = 0 x1 − x2 = 0 ⇒ x1 = x2

Chapter 1.indd 49

Solution:  All the eigenvalues of symmetric matrix [AT = A] are purely real.  Ans. (c) 85. The minimum eigenvalue of the following matrix is  3 5 2   5 12 7   2 7 5   (a) 0 (c) 2

(b) 1 (d) 3 (GATE 2013, 1 Mark)

82. Choose the correct set of functions, which are linearly dependent. (a) (b) (c) (d)

84. All the eigenvalues of a symmetric matrix are (a) complex with non-zero positive imaginary part (b) complex with non-zero negative imaginary part (c) real (d) pure imaginary (GATE 2013, 1 Mark)

1 x(x + 1) y + 1 = − 1 y(y + 1) y + 1 1 z(z + 1) z + 1

Hence, the answer is (a). 

Hence, x1 and x2 are having infinite number of solutions or multiple solutions.  Ans. (d)

Solution:  Characteristic equation is given by A − lI = 0 3−l 5 2 5 12 − l 7 =0 2 7 5−l ⇒ (3 − λ)[(12 − λ)(5 − λ) − 49] − 5[5(5 − λ) − 14] + 2[35 − 2(12 − λ)] = 0 ⇒ (3 − λ)[60 − 17 λ + λ2 − 49] − 5(25 − 5λ − 14) + 2(35 − 242λ) = 0 ⇒ (3 − λ)(λ2 − 17 λ + 11 − 5)((11 − 5λ) + (11 − 2λ) = 0 ⇒ 3λ2 − 51λ + 33 − λ3 + 17 λ2 − 11λ − 55 + 25λ + 22 + 4λ2 = 0 −l 3 + 24l 2 − 37 l = 0 l 3 − 24l 2 + 37 l = 0 l(l 2 − 24l + 37) = 0 l1 = 0 l 2, l 3 =

24 ± (24)2 − 4(37) 2

l 2 = 22 l3 = 2 Hence, lmin = 0. 

Ans. (a)

8/22/2017 10:59:20 AM

50     LINEAR ALGEBRA

86. Let A be an m × n matrix and B be an n × m matrix. It is given that determinant (I m + AB) = determinant (I n + BA), where I K is the k × k identity matrix. Using the above property, the determinant of the matrix given below is 2  1 1  1  (a) 2 (c) 8

1 2 1 1

1  1 1 2

1 1 2 1

(b) 5 (d) 16 (GATE 2013, 2 Marks)

Solution:  Take the determinant of given matrix Determinant = 2[2(4 − 1) − 1(2 − 1) + 1(1 − 2)] − 1[1(4 − 1) − 1(2 − 1) + 1(1 − 2)] + 1[1(2 − 1) − 2(2 − 1) + 1(1 − 1)] − 1[1(1 − 2) − 2(1 − 2) + 1(1 − 1)] = 2[6 − 1 − 1] − 1[3 − 1 − 1] + 1[1 − 2 + 0 ] − 1[−1 + 2 + 0 ] = 2(4) − 1(1) + 1(−1) − 1(1) = 8 −1−1−1 = 5 Hence, the answer is 5.  Ans. (b) 87. One pair of eigenvectors corresponding to the two  0 −1  is eigenvalues of the matrix  0  1 (a)  1  j   ,  −j  −1 (c)  1  0  ,   j   1

(b)  0 −1  ,   1  0 (d)  1  j   ,   j   1 (GATE 2013, 2 Marks)

Solution:  We have  0 −1  A= 0  1 Eigenvalues are calculated using characteristic equation, A − lI = 0 −l −1 =0 1 −l l2 + 1 = 0 l 2 = −1

∴ l = ±i

Therefore,

−i 1  X1   0    =    1 −i  X2   0 −ix1 − x2 = 0 x1 − ix2 = 0

Chapter 1.indd 50

Now, for l = −i  i −1  x1   0    =   i  x2   0 1 Therefore, ix1 − x2 = 0 x1 + ix2 = 0  x   j  1 Clearly,  1  =   and   satisfy  x2  −1  j  Thus, the two eigenvalues of the matrix are  1  j   1  j    ,   or   ,   . −j  −1  j   1  Ans. (d) 88. A matrix has eigenvalues −1 and −2. The corre 1  1 sponding eigenvectors are   and   , respec−2 −1 tively. The matrix is (a)  1 1   −1 −2 0 (c) −1    0 −2

(b)  1 2   −2 −4 1 (d)  0   −2 −3 (GATE 2013, 2 Marks)

Solution:  We know that AX = lX a b  1  1     = (−1)    c d  −1 −1              a − b = −1          c − d = 1  a b   1  1     = (−2)    c d  −2 −2               a − 2b = −2           c − 2d = 4 

(1) (2)

(3) (4)

Subtracting Eq. (1) from Eq. (3), we get b=1 Substituting value of b in Eq. (1), we get a − 1 = −1 ⇒ a = 0 Subtracting Eq. (2) from Eq. (4), we get

To find eigenvector, For l = +i

 X   1  j Clearly,  1  =   and   satisfy  X2  −j   1

−d = 3 ⇒ d = −3 Substituting value of d in Eq. (2), we get c − (−3) = 1 ⇒ c = 4 a b   0 1 =  Therefore, A =   c d  −2 −3 

Ans. (d)

8/22/2017 10:59:40 AM

SOLVED GATE PREVIOUS YEARS’ QUESTIONS     51

89. Consider the following system of equations: 3x + 2y = 1 4x + 7z = 1 x+y+z=3 x - 2y + 7z = 0 .

The number of solutions for this system is

(GATE 2014, 1 Mark) Solution:  The given equations are 3x + 2y = 1

= {2((−36 × 35) − (−20 × 63)) + 4((18 × 35) − (10 × 63)) + 7((−20 × 18) − (−36 × 10))} = {2((−1260) − (−1260)) + 4((630) − (630)) + 7((−360) − (−360))} = {2(0) + 4(0) + 7(0)} =0 Ans. 0

92. Which one of the following statements is TRUE about every n × n matrix with only real eigenvalues? (a) If the trace of the matrix is positive and the determinant of the matrix is negative, at least one of its eigenvalues is negative.

4x + 7z = 1 (b) If the trace of the matrix is positive, all its eigenvalues are positive.

x+y+z=3 x - 2y + 7z = 0





3 2 0 1    4 0 7 1 Augmented matrix r(A : B) =   1 1 1 3  1 −2 7 0    Applying row operations, the resultant matrix will be 1 1 1 3    0 − 4 3 − 11   0 0 −15 −21   0 0  0 0  

90. The value of dot product of the eigenvectors corresponding to any pair of different eigenvalues of a 4-by-4 symmetric positive definite matrix is . (GATE 2014, 1 Mark) Solution:  The eigenvectors for symmetric positive matrix corresponding to different eigenvalues are orthogonal. Hence, their dot product is zero always. Ans. 0 91. If the matrix A is such that 2   A = −4 [1 9 5] 7    .

(GATE 2014, 1 Mark) Solution:  The determinant of the given matrix A is 2 18 10 Determinant of A = −4 −36 −20 7 63 35

Chapter 1.indd 51

(d) If the product of the trace and determinant of the matrix is positive, all its eigenvalues are positive. (GATE 2014, 1 Mark) Solution:  If the trace of the matrix is positive and the determinant of the matrix is negative, then at least one of its eigenvalues is negative.  Determinant of matrices = Product of eigenvalues

r(A) = 3 = number of variables. So equations have a unique solution. Ans. unique

then the determinant of A is equal to

(c) If the determinant of the matrix is positive, all its eigenvalues are positive.

Ans. (a) 93. For matrices of same dimension M, N and scalar c, which one of these properties DOES NOT ALWAYS hold? (a) (MT)T = M (b) (cM)T = c(M)T (c) (M + N)T = MT + NT (d) MN = NM (GATE 2014, 1 Mark) Solution:  Matrix multiplication is not commutative in general. Ans. (d) 94. A real (4 × 4) matrix A satisfies the equation A2 = I, where I is the (4 × 4) identity matrix. The positive eigenvalue of A is . (GATE 2014, 1 Mark) Solution:  A2 = I ⇒ A = A−1 ⇒ if l is an eigen1 value of A, then is also its eigenvalue. Since, we l require positive eigenvalue, l = 1 is the only possibility as no other positive number is self-inversed. Ans. 1

8/22/2017 10:59:46 AM

52     LINEAR ALGEBRA

95. The determinant of matrix A is 5 and the determinant of matrix B is 40. The determinant of matrix AB is . (GATE 2014, 1 Mark)

98. Two matrices A and B are given below:  p2 + q 2 pr + qs  B =  2 2  pr + qs r + s 

 p q ; A=  r s

Solution:  AB = A ⋅ B = (5) ⋅ (40) = 200 Ans. 200 96. Given a system of equations: x + 2y + 2z = b1

If the rank of matrix A is N, then the rank of matrix B is (a) N/2

(c) N

(d) 2 N

(GATE 2014, 1 Mark)

5x + y + 3z = b2 Which of the following is true regarding its solutions? (a) The system has a unique solution for any given b1 and b2 (b) The system will have infinitely many solutions for any given b1 and b2 (c) Whether or not a solution exists depends on the given b1 and b2 (d) The system would have no solution for any values of b1 and b2 (GATE 2014, 1 Mark) Solution:  We are given a set of two equations x + 2y + 2z = b1 5x + y + 3z = b2

Solution:  Rank of a matrix is unaltered by the elementary transformations, i.e. row/column operators. (Here B is obtained from A by applying row/ column operations on A). Since rank of A is N. Therefore, rank of B is also N. 99. Given  1 3   2 6 −1 0 

Ans. (c) that the determinant of the matrix 0  4 is −12, the determinant of the matrix 2

 2 6 0    4 12 8 is −2 0 4   (a) −96

1 2 2 b  1 [ A / B ] =  b2  5 1 3   Applying Row transformation,

(b) −24

(c) 24

(d) 96

(GATE 2014, 1 Mark) Solution:  Taking factor 2 common from all the three rows, we have

R2 → R2 − 5R1

2 6 0 1 3 0 4 12 8 = (2)3 2 6 4 = 8 × −12 = −96 −2 0 4 −1 0 2

1 2 2 b1    0 −9 −7 b2 − 5b1    Thus, rank(A) = rank(A/B) < number of unknowns, for all values of b1 and b2. Therefore, the equations have infinitely many solutions, for any given b1 and b2.

Ans. (a) dx 100. The matrix form of the linear system = 3x − 5y dt dy and = 4x + 8y is dt (a)

d  x   3 −5  x    = 8  y  dt  y   4

(b)

d  x   = dt  y 

(c)

d  x   4 −5  x    = 8  y  dt  y   3

(d)

d  x   = dt  y 

Ans. (b) 97. Which one of the following statements is true for all real symmetric matrices? (a) All the eigenvalues are real. (b) All the eigenvalues are positive. (c) All the eigenvalues are distinct. (d) Sum of all the eigenvalues is zero. (GATE 2014, 1 Mark) Solution:  Eigenvalues of a real symmetric matrix are all real. Ans. (a)

Chapter 1.indd 52

(b) N-1

3 4  x      8 −5  y 

4 8  x      3 −5  y  (GATE 2014, 1 Mark)

8/22/2017 10:59:54 AM

SOLVED GATE PREVIOUS YEARS’ QUESTIONS     53

dx dy = 3x − 5y and = 4x + 8y , dt dt d  x   3 −5  x   the matrix form is  = 8  y  dt  y   4 Ans. (a) Solution:  As

103. Which one of the following equations is a correct identity for arbitrary 3 × 3 real matrices P, Q and R? (a) (Q + R)=PQ + RP (b) (P −  Q)2 = P2 − 2PQ + Q2 (c) det(P +  Q) = det P + det Q

−5 2  is 101. One of the eigenvectors of the matrix  −9 6 −1 −2  2 1 (a)   (b)   (c)   (d)    1  9 −1 1

(d) (P + Q)2 = P2 + PQ + QP + Q2 (GATE 2014, 1 Mark) Solution:  The correct answer is option (d). Ans. (d)

(GATE 2014, 1 Mark) −5 2  , the eigenvalues Solution:  For the matrix  −9 6 are found as follows: −5 2 1 0 −5 − l 2 =0⇒ =0 −l 6−l −9 6 0 1 −9

3 2 1 1     104. Given the matrices J = 2 4 2 and K =  2  , 1 2 6  −1     T the product K JK is . (GATE 2014, 1 Mark)

l − l − 12 = 0 ⇒ l = 4, −3 2

Solution:  We are given, 3 2 1 1     J = 2 4 2 and K =  2  1 2 6  −1    

For finding eigenvectors −5 − l 2   x1     =0 6 − l   x2   −9 For l = 4, we have the characteristic equation as −9x1 + 2x2 = 0 and for l = −3, we have −2x1 + 2x2 = 0 So, eigenvector corresponding to eigenvalue 4 is  2 / 9   and that corresponding to eigenvalue −3 is  9 / 2 1  . 1 Ans. (d)

Now we have to calculate K T JK , we get 3 2 1  1  1      = − 1 2 − 1 2 4 2 2 6 8 1 [ ] ]  2  = 23   [ 1 2 6 −1 −1       Ans. 23 105. The sum of eigenvalues of the matrix, [M ] is 215 650 795   where [M] = 655 150 835  485 355 550  

102. Consider a 3×3 real symmetric matrix S such that two of its eigenvalues are a ≠ 0, b ≠ 0 with respec x1   y1     tive eigenvectors  x2   y2  . If a ≠ b then     x3   y3 

(a) 915

(b) b

(c) ab

(d) 0

(GATE 2014, 1 Mark) Solution:  Since the matrix is symmetric with distinct eigenvalues, the eigenvectors of the matrix are orthogonal. So,  x1   y1      x2   y2  = 0 ⇒ x1y1 + x2 y2 + x3 y3 = 0     x3   y3  Ans. (d)

Chapter 1.indd 53

(c) 1640

(d) 2180

(GATE 2014, 1 Mark)

x1y1 + x2 y2 + x3 y3 equals (a) a

(b) 1355



Solution:  Sum of the eigenvalues = Trace of the matrix = 215 + 150 + 550 = 915 Ans. (a)

0  1 106. The determinant of matrix  2 3 

1 0 3 0

2 3 0 1

3  0 is 1 2

.

(GATE 2014, 1 Mark) 0  1 Solution:  The given matrix is  2 3 

1 0 3 0

2 3 0 1

3  0 1 2

8/22/2017 11:00:05 AM

54     LINEAR ALGEBRA

1  0 0  0  1 

The determinant is given by 0 1 2 3

1 0 3 0

2 3 0 1

3 1 2 3 0 1 3 0 = −1 3 0 1 − 3 2 3 1 1 0 1 2 3 0 2 2 = 1 (0 − 1) − 2 (6 − 0) + 3 (3 − 0) = −3  0 − 1 (4 − 3) + 3 (0 − 9) = 88

is

0 1 1 1 0

0 1 1 1 0

0 1 1 1 0

1  0 0 0 1

. (GATE 2014, 2 Marks)

Solution:  Eigenvector is AX = lX Ans. 88 107. There are 5 bags labeled 1 to 5. All the coins in a given bag have the same weight. Some bags have coins of weight 10 gm, others have coins of weight 11 gm. I pick 1, 2, 4, 8, 16 coins, respectively, from bags 1 to 5. Their total weight comes out to be 323 gm. Then the product of the labels of the bags having 11 gm coins is . (GATE 2014, 2 Marks) Solution:  Let the weight of coins in the respective bags (1 through 5) be a, b, c, d and e — each of which can take one of two values namely 10 or 11 (gm). Now, the given information on total weight can be expressed as the following equation:

1  0 A =  0 0  1 

0 1 1 1 0

1  0 0 0 1

0 1 1 1 0

Convert the given matrix in diagonal matrix. The diagonal entries are called eigenvalues. Eigenvalues = 2, 3 Product of eigenvalues = 2 × 3 = 6 Ans. 6 109. Consider the matrix 0  0 0 J 6 =  0 0  1

a + 2b + 4c + 8d + 16e = 323 ⇒ a must be odd ⇒ a = 11

The equation then becomes 11 + 2b + 4c + 8d + 16e = 323 ⇒ 2b + 4c + 8d + 16e = 312 ⇒ b + 2c + 4d + 8e = 156 ⇒ b must be odd ⇒ b = 10

0 1 1 1 0

0 0 0 0 1 0

0 0 0 1 0 0

0 0 1 0 0 0

0 1 0 0 0 0

1  0 0 0 0  0

which is obtained by reversing the order of the columns of the identity matrix I6. Let P = I6 + aJ6, where a is a non-negative real number. The value of a for which det(P) = 0 is .

The equation then becomes (GATE 2014, 2 Marks) 10 + 2c + 4d + 8e = 156 ⇒ 2c + 4d + 8e = 146 ⇒ c + 2d + 4e = 73 ⇒ c must be odd ⇒ c = 11 The equation now becomes 11 + 2d + 4e = 73 ⇒ 2d + 4e = 62 ⇒ d + 2e = 31 ⇒ d = 11 and e = 10 Therefore, bags labeled 1, 3 and 4 contain 11 gm coins ⇒ Required product = 1 × 3 × 4 = 12. Ans. 12 108. The product of the non-zero eigenvalues of the matrix

Chapter 1.indd 54

Solution:  Let P = 1 − a 2.  1 0  +a P = I2 + a J 2 =   0 1

 0 1  1 a   =   1 0 a 1 

1 0 0 a   0 1 a 0 P = I4 + a J 4 =   0 a 1 0 a 0 0 1    Now, let 1 a P = (1) a 1 0 0

0 0 1 a 0 − (a ) 0 a 1 1 a 0 0

= (1 − a 2 ) − (a )[a (1 − a 2 )] = (1 − a 2 )2

8/22/2017 11:00:13 AM

SOLVED GATE PREVIOUS YEARS’ QUESTIONS     55

112. Which one of the following statements is NOT true for a square matrix A?

Similarly, if P = I6 + aJ 6, then we get

(

P = 1− a2

)

3

(a) If A is upper triangular, the eigenvalues of A are the diagonal elements of it

Therefore P = 0 ⇒ a = −1, 1

(b) If A is real symmetric, the eigenvalues of A are always real and positive (c) If A is real, the eigenvalues of A and AT are always the same

Since a is non-negative, so a = 1. Ans. 1 110. The system of linear equations

(d) If all the principal minors of A are positive, all the eigenvalues of A are also positive

2 1 3 a  5       3 0 1  b  = −4 1 2 5 c   14      

(GATE 2014, 2 Marks) Solution:  Consider a real symmetric matrix

has −1 1   A=  1 −1

(a) a unique solution (b) infinitely many solutions

Characteristic equation is

(c) no solution (d) exactly two solutions (GATE 2014, 2 Marks) Solution:  2 1 3 5    [ A / B ] =  3 0 1 −4    1 2 5 14  Using row transformation, R2 → 2R2 − 3R1 and R3 → 2R3 − R1 2 1 2 1 3 5  3 5       0 −3 −7 −23 R 3 = R3 + R2  0 −3 −7 −23       7 23  0 0   0 3  0 0 Since rank (A) = rank (A/B) < number of unkowns, equations have infinitely many solutions. Ans. (a) 111. The maximum value of the determinant among all 2 × 2 real symmetric matrices with trace 14 is . (GATE 2014, 2 Marks) Solution:  Let the 2 × 2 real symmetric matrix be  y x  .  x z 

Chapter 1.indd 55

A − lI = 0 ⇒ (1 + l ) − 1 = 0 ⇒ l + 1 = ±1 2

Therefore l = 0, −2 (not positive) Thus, option (b) is not true. Options (a), (c) and (d) are true using properties of eigenvalues. Ans. (b) 113. A system matrix is given as follows:  0 1 −1   A = −6 −11 6  −6 −11 5    The absolute value of the ratio of the maximum eigenvalue to the minimum eigenvalue is . (GATE 2014, 2 Marks) Solution:  The characteristic equation is A − lI = 0 −l 1 −1 − 6 − 11 − l 6 =0 ⇒ −6 −11 5−l ⇒ l 3 + 6 l 2 + 11l + 6 = 0

Now det = yz − x2 and trace is given as y + z = 14

So l = −1, −2, and −3 are the eigenvalues of A.

⇒ z = 14 − y  (1) Let f = yz − x2 (det) = − x2 − y 2 + 14y, using Eq. (1). Using maxima and minima of a function of two variables, f is maximum at x = 0 and y = 7. Therefore, maximum value of the determinant is 49. Ans. 49

Now lmax = −1 and lmin = −3. Hence lmax −1 1 = = −3 3 lmin Ans. (1/3)

8/22/2017 11:00:26 AM

56     LINEAR ALGEBRA

6 0 4 4    114. The rank of the matrix −2 14 8 18  is  14 −14 0 −10   . (GATE 2014, 2 Marks) Solution:  We have the matrix, 6 0 4 4    −2 14 8 18   14 −14 0 −10  

 1 2 3  1    [ A ] 7 8 9 = 4  4 5 6 7    1 2 3 1 ⇒ A 7 8 9 = 4 4 5 6 7

2 3  5 6 8 9 2 3 5 6 8 9

1 2 3 1 2 3 ⇒ A 7 8 9 = − 7 8 9 ⇒ A = −1 4 5 6 4 5 6 Product of eigenvalues = −1 Thus, option (c) is correct.

Using row transformation, R2 → 3R2 + R1 ; R3 → 6R3 − 14R1

Ans. (c)

6 0 4 4    28 58   0 42  0 −84 −56 −116  

117. The larger of the two eigenvalues of the matrix  4 5   is . 2 1 (GATE 2015, 1 Mark)

R3 → R3 + 2R2

 4 5 . Solution:  The given matrix is  2 1

6 0 4 4     0 42 28 58 0 0 0 0    Rank = number of non-zero rows = 2.

The characteristic equation is given by A − lI = 0 Ans. 2

115. A scalar valued function is defined as f (x) = xT Ax + b T x + c, where A is a symmetric positive definite matrix with dimension n × n; b and x are vectors of dimension n × 1. The minimum value of f(x) will occur when x equals (a) (AT A)−1 b

(b) −(AT A)−1 b

 A−1b   (c) −   2 

A−1b (d) 2 (GATE 2014, 2 Marks)

Solution:  The minimum value of f(x) will occur  A−1b    . when x = −   2  Ans. (c) 116. For the matrix A satisfying the equation given below, the eigenvalues are  1 2 3  1 2 3     [A ] 7 8 9 =  4 5 6  4 5 6  7 8 9      (a) (1, −j, j)

(b) (1, 1, 0)

(c) (1, 1, −1)

(d) (1, 0, 0) (GATE 2014, 2 Marks)

Chapter 1.indd 56

Solution: 

Hence, 4−l 5 =0 2 1− l ⇒ l 2 − 5l − 6 = 0 ⇒ (l − 6)(l + 1) = 0 ⇒ l = 6, − 1 Therefore, larger eigenvalue is 6. Ans. 6  1 −1 2    118. In the given matrix  0 1 0 , one of the eigen 1 2 1   values is 1. The eigenvectors corresponding to the eigenvalue 1 are (a) {a (4, 2, 1) | a ≠ 0, a ∈ R} (b) {a (−4, 2, 1) | a ≠ 0, a ∈ R} (c) {a ( 2 , 0, 1) | a ≠ 0, a ∈ R} (d) {a (− 2 , 0, 1) | a ≠ 0, a ∈ R} (GATE 2015, 1 Mark) Solution:  Let X be an eigenvector corresponding to eigenvalue l = 1, then

8/22/2017 11:00:38 AM

SOLVED GATE PREVIOUS YEARS’ QUESTIONS     57

AX = lX ⇒ (A − l )X = 0

120. Consider the system of linear equations:

 0 −1 2  X    ⇒  0 0 0  Y  = 0  1 2 0     Z  ⇒ −y + 2z = 0 and x + 2y = 0

x - 2y + 3z = -1 x - 3y + 4z = 1 and -2x + 4y - 3z = k The value of k for which the system has infinitely many solutions is .

x =y −2

⇒ y = 2z and

(GATE 2015, 1 Mark)

Therefore, x x y z x = y = 2z ⇒ x = y = z = a (say) −2 = y = 2z ⇒ −4 = 2 = 1 = a (say) −2 −4 2 1 −4  4   − ⇒ X = 2  a; a ≠ 0 ⇒ X =   21  a ; a ≠ 0   1    

Solution:  We are given, x - 2y + 3z = -1 x - 3y + 4z = 1 -2x + 4y - 3z = k  1 −2 3 −1    Augmented matrix (A/B) =  1 −3 4 1  −2 4 −6 k  

Thus, eigenvectors are {a (−4, 2, 1) a ≠ 0, a ∈ R } Ans. (b)

Using row transformation

 1   119. The value of p such that the vector 2 is an eigen 3   4 1 2   vector of the matrix  p 2 1  is 14 −4 10  

R2 → R2 − R1 , R3 → R3 + 2R1 We get  1 −2 3 −1    2   0 −1 1 0 0 0 k − 2 

.

(GATE 2015, 1 Mark)

The system will have infinitely many solutions if p(A/B) = p(A) = r < number of variables

Solution:  We are given,

⇒ k−2 = 0 ⇒k=2

4 1 2   1 p 2 14 −4 10  



Ans. 2

121. Two sequences [a, b, c] and [A, B, C] are related as We have,  A  1 1        B  = 1 w3−1       −2 C  1 w3

4  1 1 2   1      AX = lX ⇒  p 2 1   2 = l  2 14 −4 10 3  3        12   l      ⇒  p + 7  = 2l  ⇒ l = 12 (1)  36  3l      2l = p + 7 (22)

j

where w3 = e

2p 3

If another sequence [p, q, r] is derived as p  q   r

and 3l = 36 ⇒ l = 12 Thus, Eq. (2) gives p + 7 = 24 ⇒ p = 17 Ans. 17

Chapter 1.indd 57

1  a      w3−2   b      w3−4   c 

1  1    = 1 w3−1    1 w2 3 

1  1 0  w32   0 w32 w34   0 0

0   A/3  0   B/3 w34   C /3

     

then the relationship between the sequences [p, q, r] and [a, b, c] is

8/22/2017 11:00:48 AM

58     LINEAR ALGEBRA

A+ = A , i.e. (A)T = A

(a) [p, q, r] = [b, a, c]

(b) [p, q, r] = [b, c, a]

(c) [p, q, r] = [c, a, b]

(d) [p, q, r] = [c, b, a]  10 x 4  10 5 + j 4     2 =  x 20 2 ⇒ x = 5 − j 5 − j 20  4   2 −10  4 2 −10 

(GATE 2015, 1 Mark) Solution:  Consider, 1 1  1 w1 3  1 w2  3

1  1 0  w32   0 w32 w31   0 0 (3 +1)

Since w34 = w3 1 w2 3 1  1 1  3 1 1 w3

w31   A 1   B w32   C

2 0  1 w3   0  = 1 w33  w34  1 w34 

w34   w36   w35  

Ans. (b)  1 tan x , A= 1  − tan x ATA-1 is

123. For

the determinant of

= w31 ; w36 = w33 = w30 = 1; w35 = w32

2    1 1 w3  = 1 1  3 1 w1    3

1 w31  1   1  1 w3−1 w32  1 w3−2

1  a  w3−2   b w3−1   c

1 + w2 + w1 1 + w + w−1 1 + w30 + w30   a 3 3 3 3  1 =  1 + 1 + 1 1 + w3−1 + w3−2 1 + w3−2 + w3−1   b 3  1 + w30 + w30 1 + w3−1 + w31   c 1 + w31 + w32   Since, w3 = e



     

w32 + w31 = w3 + w3−1 = w3−1 + w3−2 = −1 Thus,  p  0 0 3 a   c    1      q  =  3 0 0   b  = a   r  3  0 3 0  c   b         

Ans. (c)

122. The value of x for which all the eigenvalues of the matrix given below are real is 10 5 + j 4    20 2  x 4 2 −10  (a) 5 + j

(b) 5 - j

(c) 1 - 5j

(d) 1 + 5j (GATE 2015, 1 Mark)

Solution:  The matrix is given by, 10 5 + j 4    A = x 20 2  4 2 −10 

(b) cos 4x

(c) 1

(d) 0 (GATE 2015, 1 Mark)

     

j 2p 3

(a) sec2 x

Solution:  We are given,  1 tan x  A= − tan x 1  

 − tan x  1   1 − tan  A T x=   tan x AT =  1  1   tan x A = 1 + tan2 x = sec2 x A = 1 + tan2 x = sec2 x 1  1 − tan x 1  1A−1 −   =tan x 2   tan x  A−1 = 1  sec 1 x  sec2 x  tan x  − tan x tan x 1   1 T tan  1 A−   11  − −tan x  1  A−1x= 2   tan   A T A−1 =  x 1  sec x  tan x 1  1  sec2 x  tan x 1   tan x 1  1 − tan2 x  − tan x − tan x 1  1 − tan=2 x − tan x − tan x = sec2 x  tan x2+ tan x − tan2 x + 1  2  − tan x + 1  sec x  tan x + tan x 2 2  1 − 1 2  x −2 tan x   2 xtan 1 1 − tan =x −2 tan  = x 1 − tan2 x sec2 x  22 tan 1 − tan x sec2 x  2 tan x 1 2 2 2 1 A T A−1 = 2 2 2 [(1 −2 tan x) + 4 tan x] A T A−1 = 1 x ) 4 tan x ] + [( − tan sec x sec2 x 1 1 [1 + tan 4 x − 2 tan2 x + 4 tan2 x] 4= 2 tan2 x + 4 tan2 x ] [ 1 tan x − 2 = + sec x sec2 x 1 1 1 (12+ tan 4 x1+ 2 tan2 x) 2= 2 2 (1 + tan2 x)2 4= 1 x + 2 tan x ) = ( 1 + tan x ) = ( + tan sec x sec x sec2 x sec2 x 1 2 2 2 1 = (sec x) = sec x = (sec2 x)2 =sec sec2 2xx sec2 x

Ans. (a)

124. If the sum of the diagonal elements of a 2 × 2 matrix is −6, then the maximum possible value of determinant of the matrix is .

Given that all eigenvalues of A are real. Thus, A is Hermitian.

Chapter 1.indd 58

(GATE 2015, 1 Mark)

8/22/2017 11:01:00 AM

SOLVED GATE PREVIOUS YEARS’ QUESTIONS     59

Solution:  Sum of the diagonals elements is −6 for 2 × 2 matrix. The possible eigenvalues are

statements and `X ≡  Y ’ means X and Y are not equivalent statements.

−1, −5 −5, −1 −8, 2 −2, −3 −4, −2 −9, 3 − − − −3, −1 −3, −3 −10, 4

P: There is a unique solution. Q: The equations are linearly independent. R: All eigenvalues of the coefficient matrix are non-zero. S: The determinant of the coefficient matrix is nonzero.

Maximum possible (−3) × (−3) = 9.

value

of

determinant

is

Ans. 9 125. The maximum value of `a’ such that the matrix −3 0 −2    1 −1 0  0 a −2  has three linearly independent real eigenvectors is 2 (a)

1

(b)

3 3

1+ 3

3 3

(a) P ≡ Q ≡ R ≡ S (b) P ≡ R ≡ Q ≡ S (c) P ≡ Q ≡ R ≡ S (d) P ≡ Q ≡ R ≡ S (GATE 2015, 1 Mark) Solution:  The correct answer is option (a) Ans. (a)

3 3

1+2 3 (c)

Which one of the following is TRUE?

127. At least one eigenvalue of a singular matrix is

(d) 3 3 (GATE 2015, 1 Mark)

Solution:  The characteristic equation of A is A − XI = 0 ⇒ f (x) = x3 + 6x2 + 11x + 6 + 2a = (x + 1)(x + 2)(x + 3) + 2a = 0 f(x) cannot have all 3 roots (if any) equal since if f (x) = (x − k)3 , then comparing coefficients, we get

(a) Positive (b) Zero (c) Negative (d) Imaginary (GATE 2015, 1 Mark) Solution:  The correct answer is `Zero’. Ans. (b)  4 2  128. The lowest eigenvalue of the 2 × 2 matrix  1 3 is .

6 = −3k, 3k2 = 11 No such k exists. (a) Thus f(x) = 0 has repeated (2) roots (say) a, a, b or (b) f(x) = 0 has real roots (say) a , b , g .

(GATE 2015, 1 Mark) Solution:  Let  4 2  A= 1 3 Characteristic equation of A is A − lI = 0

Now, −6 − 3 ≈ −2.577a f ′(x) = 0 ⇒ x1 = 3 −6 + 3 ≈ −1.422 x2 = 3 At x1, f (x) has relative maxima and at x2, f(x) has relative minima. In all possible cases, we have  3  1  ≤ 0 ⇒ a ≤ f (x2 ) ≤ 0 ⇒ 2 a −  9  3 3 Ans. (b) 126. We have a set of three linear equations in three unknowns. `X ≡Y ’ means X and Y are equivalent

Chapter 1.indd 59



4−l 2 =0 1 3−l

⇒ l 2 − 7 l + 10 = 0 ⇒ l = 2, 5 The lowest eigenvalue is 2. 

Ans. 2

 4 7 8   129. If any two columns of a determinant P = 3 1 5  9 6 2   are interchanged, which one of the following statements regarding the value of the determinant is CORRECT?

8/22/2017 11:01:14 AM

60     LINEAR ALGEBRA

 1 2 3 − − −    2 4 6 − − − ⇒A=   3 6 9 − − − − − − − − −   Applying row transformation,

(a) A  bsolute value remains unchanged but sign will change. (b) Both absolute value and sign will change. (c)  Absolute value will change but sign will not change. (d) Both absolute value and sign will remain unchanged.

R2 − 2R1 , R3 − 3R1 , …

(GATE 2015, 1 Mark) Thus, every row will be zero row, except first row in echelon form. Thus, r(A) = 1. Ans. (b)

Solution:  The correct answer is option (a). Ans. (a) 130. For what value of p, the following set of equations will have no solution? 2x + 3y = 5

132. Let A be an n × n matrix with rank r (0 < r < n). Then Ax = 0 has p independent solutions, where p is

3x + py = 10 (a) r

(b) n

(c) n - r

(d) n + r

(GATE 2015, 1 Mark) (GATE 2015, 1 Mark) Solution:  We are given the equations, Solution:  Given AX = 0 2x + 3y = 5 3x + py = 10 2 3   x 5    =   ⇒ 3 p   y  10 AX = B

r(An×n ) = r (0 < r < n) where r is the number of independent solution. That is, nullity = 0 We know that,

Augmented matrix Rank + Nullity = n

2 3 5   [ A / B] =   3 p 10

r + p = n ⇒ p = n−r Ans. (c)

Using, row transformation, we get

133. The following set of three vectors

R2 → 2R2 − 3R1

   x 1 6 2 , x 1

3 4 2 is linearly dependent when x is equal to (a) 0 (b) 1 (c) 2 (d) 3

2 3 5   0 2 p − 9 5   System will have no solution if r (A / B ) ≠ r (A) ⇒ 2p − 9 = 0 9 ⇒ p = = 4. 5 2 





(GATE 2015, 1 Mark) Solution:  For linearly depending vectors, A = 0. 1 x 3 2 6 4 =0 1 x 2

Ans. 4.5

131. Let A = [aij], 1 ≤ i, j ≤ n with n ≥ 3 and aij = i, j. The rank of A is (a) 0

(b) 1

(c) n - 1

(d) n

⇒ (12 − 4x) − x(4 − 4) + 3(2x − 6) = 0 ⇒ 2x = 6 ⇒x=3

(GATE 2015, 1 Mark) Solution:  Given A = aij  , 1 ≤ i, j ≤ n, n ≥ 3 and   aij = i ⋅ j

Chapter 1.indd 60

and

Ans. (d) 4 3 1 134. For the matrix , if is an eigenvector, the 3 4 1 corresponding eigenvalue be .

  

(GATE 2015, 1 Mark)

8/22/2017 11:01:24 AM

SOLVED GATE PREVIOUS YEARS’ QUESTIONS     61

Solution:  Let A =

  4 3 3 4

and x =



The determinant of the resultant matrix is

1 1

(GATE 2015, 2 Marks)

Let eigenvalue be l. So,

Solution:

Ax = lx ⇒ (A − lI )(x) = 0

  

 I =

where       





3 4−l





4−l 3

 3 4 45    Let A =  7 9 105 13 2 195  

1 0 0 1 

l So,       lI =   0  

.

Applying row transformation,

0   l 

R2 → R2 + R3  3 4 45    ∼ 20 11 300 13 2 195   

 

1 =0 1

Applying column transformation,

4−l +3 ⇒ 3+4−l

=0⇒l =7

C1 → C2 − C3



Ans. 7

135. Consider the following 2 × 2 matrix A where two elements are unknown and are marked by a and b. The eigenvalues of this matrix are −1 and 7. What are the values of a and b?

 −42 4 45    ∼ −280 11 300 = B (resultant mattrix ) −182 2 195    Now, −42 4 3 3 4 3 B = 280 11 300 = (−14)(15) 20 11 20 = 0 −182 2 195 13 2 13

1 4 A =   b a (a) a = 6, b = 4 (c) a = 3, b = 5

(b) a = 4, b = 6 (d) a = 5, b = 3

Ans. 0

(GATE 2015, 2 Marks) Solution:  Given that l1 = −1 and l2 = 7 are eigenvalues of A. Using the properties of matrices,

137. If the following system has non-trivial solution px + qy + rz = 0 qx + ry + pz = 0 rx + py + qz = 0

 l1 + l2 = tr(A)

l1 ⋅ l2 = A

(1) 

(2)

⇒ 6 = 1+a ⇒ a = 5

then which one of the following options is TRUE? (a) p - q + r = 0 or p = q = −r (b) p + q - r = 0 or p = -q = r (c) p + q + r = 0 or p = q = r (d) p - q + r = 0 or p = −q = −r

Also,

(GATE 2015, 2 Marks) a − 4b = −7 ⇒ 5 − 4b = −7 ⇒ b = 3 Solution:  For non-trivial solution, A = 0. Ans. (d)

136. Perform the following operations on the matrix  3 4 45     7 9 105 13 2 195   (i) Add the third row to the second row. (ii) Subtract the third column from the first column.

Chapter 1.indd 61

p q r That is q r p = 0 r p q Applying column transformation, C1 → C1 + C2 + C3

8/22/2017 11:01:35 AM

62     LINEAR ALGEBRA

 3 −2 2     4 −4 6   2 −3 5   

1 q r ( p + q + r) 1 r p = 0 1 p q (a) 1.5 and 2.5 Applying row transformation, (b) 0.5 and 2.5

[R2 → R2 − R1 ; R3 → R3 − R1 ]

(c) 1.0 and 3.0 (d) 1.0 and 2.0

11 qq p+ + qq + − qq p + rr = = 00 or or 00 rr − 00 p p− − qq

rr p− − rr = p = 00 qq − r −r

(GATE 2015, 2 Marks)  3 −2 2    Solution:  Let A =  4 −4 6  2 −3 5   

⇒ ((rr − − qq))22 − − (( p p− − qq)( )( p p− − rr)) = = 00 ⇒ 2 ⇒p + qq 22 + + rr22 − − pq pq − − qr qr − − pr pr = = 00 ⇒ p2 +

⇒ (( p p − qq))22 + + ((qq − rr))22 + + ((rr − p p))22 = = 00 ⇒ ⇒ p q = 0 , q r = 0 , r p = − − − ⇒ p q = 0, q r = 0, r p = 00 ⇒ ⇒p p= = qq = = rr

Characteristic equation is A − l I = 0



Ans. (c)  4 + 3i −i   , where 138. For a given matrix P =  4 − 3 j   i

⇒ l 3− 4l 2+ 5l − 2 = 0 ⇒ (l − 1)(l 2− 3l + 2) = 0 ⇒ (l − 1)((l − 1)(l − 2) = 0 ⇒ l = 1, 2

i = −1, the inverse of matrix P is

(a)

1  4 − 3i i    − i 4 + 3i  24 

Ans. (d)

1  i 4 − 3i   (b) −i  25  4 + 3i (c)

1  4 + 3i −i    4 − 3i 24  i

(d)

1  4 + 3i −i    4 − 3i 25  i

3−l −2 2 4 −4 − l 6 =0 2 −3 5−l

2 1   have a 140. The two eigenvalues of the matrix  1 p  ratio of 3:1 for p = 2. What is another value of p for which the eigenvalues have the same ratio of 3:1?

(GATE 2015, 2 Marks)

(a) -2

(b) 1

(c) 7/3

(d) 14/3

Solution:  We are given

(GATE 2015, 2 Marks)

 4 + 3i −i  , P = 4 − 3 j   i P = (4 + 3i)(4 − 3i) − (i)(−i) = 16 + 9 − 1 = 24

Given that two eigenvalues of A are in 3:1. Ratio for p = 2. Characteristic equation l 2 − 4l + 3 = 0 (by substituting p = 2)

 4 + 3i −i   adj P =  4 − 3i  i Therefore, P −1 =

2 1   Solution:  Let A =  1 p 

⇒ l = 1, 3

1  4 + 3i −i    4 − 3i 24  i Ans. (c) If we take p =

139. The smallest and largest eigenvalues of the following matrix are:

Chapter 1.indd 62

2 1  14   then A =  14  3 1   3 

8/22/2017 11:01:48 AM

SOLVED GATE PREVIOUS YEARS’ QUESTIONS     63

  28  14  ⇒ l 2− 2 +  l +  − 1 = 0   3  3 20 25 ⇒ l 2− l + = 0 ⇒ 3l 2 − 20 l + 25 = 0 3 3 5 ⇒ l = 5, 3

143. Suppose that the eigenvalues of matrix A are 1, 2, 4. The determinant of (A−1)T is __________. (GATE 2016, 1 Mark) Solution:  It is given that for matrix A, the eigenvalues are 1, 2, 4. Therefore, det(A) = Product of eigenvalues

Eigenvalues are in the ratio 3:1.

trace(A) = Sum of eigenvalues Now, the eigenvalues of A−1 are 1, 1/2, 1/4. Therefore,

14 Therefore, p = 3 Ans. (d)

|A −1 | = 1 ×

141. A system is represented in state-space as X˙ = Ax +  1 2 1  and B =   . The value of a for Bu, where A =  a 6 1 which the system is not controllable is .

1 1 1 × = 2 4 8

|A −1 |T = |A −1 | =

1 = 0.125 8 Ans. 0.125

(GATE 2015, 2 Marks) Solution:  For a system to be uncontrollable, its controllability determinant should be equal to zero. Qc = BAB = 0 1 3  2     =     6 2×2 1 a + 6 2×2 1 3 Qc = BAB → = 0 ⇒ a +6−3 = 0 1 a +6

1 AB =  a

⇒ a = −3 Ans. -3



142. Two eigenvalues of a 3 × 3 real matrix P are (2 + −1) and 3. The determinant of P is __________. (GATE 2016, 1 Mark)

144. Let M    4 = I, (where I denotes the identity matrix) and M ≠ I, M    2 ≠ I and M  3 ≠ I. Then, for any natural number k, M − 1 equals: (a) M    4k + 1

(b) M    4k + 2

(c) M    4k + 3

(d) M    4k (GATE 2016, 1 Mark)

Solution:  It is given that M    4 = I (1) −1 On multiplying both sides of Eq. (1) with M  , we get M    −1(M  4) = M  −1 ⇒ M  −1 = M  3 = M  0 + 3 = M  4k + 3 Therefore,

(for k = 0)

M  −1 = M  4k + 3

Solution:  For 3 × 3 real matrix, the eigenvalues must be (2 + −1, 2 − −1 and 3) = (2 + i, 2 − i and 3). Now, determinant of the matrix P

Ans. (c) 145. The

value of x for which the matrix 2 4   3  A= 9 7 13  has zero as an eigenvalue    −6 −4 −9 + x is __________.

= Product of eigenvalues = (2 + i)(2 − i)(3) = (22 − i2 ) × 3 = (4 + 1) × 3 = 15

(GATE 2016, 1 Mark)

Therefore, the determinant of the matrix P is 15. Ans. 15

Chapter 1.indd 63

Solution:  For the eigenvalue of a matrix is to be zero, then the determinant of the matrix is also zero:

8/22/2017 11:01:55 AM

64     LINEAR ALGEBRA

148. A 3 × 3 matrix P is such that, P3 = P. Then the eigenvalues of P are

3 2 4 A =0⇒ 9 7 13 = 0 −6 −4 −9 + x ⇒ 3[( − 63 + 7 x) + 52] − 2[(− 81 + 9x) + 78] + 4( −36 + 42) = 0 ⇒ 21x − 33 − 18x + 6 + 24 = 0 ⇒ 3x − 3 = 0 ⇒x=1



(a) 1, 1, -1 (b) 1, 0.5 + j0.866, 0.5 - j0.866 (c) 1, -0.5 + j0.866, -0.5 - j0.866 (d) 0, 1, -1 (GATE 2016, 1 Mark) Solution:  Using Cayley—Hamilton theorem, we get

Ans. 1 s x  146. Consider a 2 × 2 square matrix: A =   , where w s  x is unknown. If the eigenvalues of the matrix A are (σ  +  jω) and (σ  −  jω), then x is equal to (a) +jω

(b) −jω

(c) +ω

(d) −ω

l3 = l



Þ l = 0, 1, -1 Ans. (d)

149. The solution to the system of equations  2 5  x   2   −4 3  y  = −30  is

(GATE 2016, 1 Mark) Solution: 



(a) 6, 2   (b) −6, 2  (c) −6, −2  (d) 6, −2 (GATE 2016, 1 Mark)

It is given that s

Solution:  Given that

x w s

A=

D=

2 5 = 26 −4 3

Now, Determinant of A  =  Product of eigenvalues

D1 =

2 5 = 156 −30 3

D2 =

2 5 = −52 −4 −30

⇒ σ2  −  ωx = (σ  +  jω)(σ   −  jω) = σ2  +  ω2 ⇒ − ωx = ω2 ⇒ x = − ω Ans. (d)

Thus,

147. Consider a 3 × 3 matrix with every element being equal to 1. Its only non-zero eigenvalue is ________.

D1 =6 D D y = 2 = −2 D x=

(GATE 2016, 1 Mark)

Ans. (d)

Solution:  Det [A - lI ] 1− l 1 1 = 1 1− l 1 1 1 1− l





Chapter 1.indd 64

150. The condition for which the eigenvalues of the matrix 2 1  A= 1 k 



l = 1 × order of matrix

are positive is

=1×3=3

(a) k > 1/2  (b) k > −2  (c) k > 0  (d) k < −1/2

Eigenvalues are 0, 0, 3. Ans. 3

(GATE 2016, 1 Mark)

8/22/2017 11:02:02 AM

SOLVED GATE PREVIOUS YEARS’ QUESTIONS     65

Solution:  Given that

Solution:  Eigenvalue increases by increasing the values of entries. If it is increased by 1 then one eigenvalue will be equal to 1.

2 1  A= 1 k 

Ans. (d) Characteristic equation is given by |A - lI | = 0 2−λ 1 =0 ⇒ 1 k−λ ⇒ (2k − 2l - kl + l2) - 1 = 0

153. Let A1, A2, A3, and A4 be four matrices of dimensions 10 × 5, 5 × 20, 20 × 10, and 10 × 5, respectively. The minimum number of scalar multiplications required to find the product A1A2A3A4 using the basic matrix multiplication method is __________.

⇒ l2 - (k + 2)l + 2k - 1 = 0

(GATE 2016, 2 Marks)

2

⇒ λ=

(k + 2) ± (k + 2) − 4(2k − 1) 2

Solution:   Here, there are five possible cases:

For l to be positive, we have

A(B(CD)) A((BC)D)

(k + 2) > (k + 2)2 − 4(2k − 1)

((AB)C)D ⇒ (k + 2)2 > (k + 2)2 - 4(2k - 1) ⇒ −4(2k - 1) < 0 ⇒ 2k - 1 > 0 ⇒ k > 1/2 Ans. (a)

(A(BC))D (AB)(CD) The scalar multiplications required are 1750, 1500, 3500, 2000, 3000, respectively. Therefore, the minimum number of scalar multiplications is 1500.

151. A real square matrix A is called skew-symmetric if (a) AT = A (c) AT = −A

Ans. 1500

(b) AT = A−1 (d) AT = A + A−1

 x[n ]  1 1  154. A sequence x[n] is specified as  =  , x[n − 1] 1 0  for n ≥ 2. The initial conditions are x[0]  =  1, x[1]  =  1, and x[n] = 0 for n  median > mode

(GATE 2005, 2 Marks) Solution:  We know that P (A | B ) =

(d) In a negatively skewed distribution: mode > mean > median

=

15. A lot has 10% defective items. Ten items are chosen randomly from this lot. The probability that exactly two of the chosen items are defective is (a) 0.0036

(b) 0.1937

(c) 0.2234

(d) 0.3874

P (B )

∴ P(2 heads in 3 tosses | first toss is head)

(GATE 2005, 1 Mark) Solution:  Options (a), (b), (c) are true but option (d) is not true because in a negatively skewed distribution, mode > median > mean.  Ans. (d)

P (A ∩ B )

P (2 heads in 3 tosses and first toss is head ) P (first toss is head)

P(first toss is head) = 1/2 P(2 head in 3 tosses and first toss is head) = 1 1 1 1 1 1 1 P(HHT) + P(HTH) = × × + × × = 2 2 2 2 2 2 4 ∴

1 1 Required probability = 4 = 1 2 2

(GATE 2005, 1 Mark)  Solution:  This problem can be done using binomial distribution because population is infinite. Probability of defective item, p = 0.1

Chapter 5.indd 284

Ans. (b) ∞

18. The value of the integral I =

1 2p

∫ 0

 x2  exp −  dx is  8 

8/22/2017 3:35:44 PM

SOLVED GATE PREVIOUS YEARS’ QUESTIONS     285 ∞

(b) p (d) 2p

(a) 1 (c) 2



Solution:     

−∞ ∞

(GATE 2005, 2 Marks)



Solution:  We have I=

2p

(−x2 / 8) e dx



0



ax

Ke

−∞

0

(x − m )

−a x

Ke

−∞



1

p(x) dx = 1 dx = 1



dx + ∫ K e−ax dx = 1 0

2





1 Comparing with

2ps

e

dx = 1

−∞

We can provide m and s any value. Here, putting m = 0 ∞

x2

− 2 1 e 2s = 1 2ps

2∫ 0



)0

=1

Ans. (c)

21. Two fair dice are rolled and the sum r of the numbers turned up is considered: (a) P(r > 6) = (1/6)

x2 x2 =− 2 8 2s

(b) P(r/3 is an integer) = (5/6)

⇒ s = 2,

(c) P(r = 8 | r/4 is an integer) = (5/9)

Now putting s = 2 in the above equation, we get ∞

∫ 0

 

(



Given, −

( )

K ax 0 K −ax e e + −∞ a −a K K ⇒ + =1 a a 2K = a ⇒ K = 0.5a ⇒

2s 2

1 2p

x2 − e 8

(d) P(r = 6 | r/5 is an integer) = (1/18)

Solution:  If two fair dice are rolled, the probability distribution of r where r is the sum of the numbers on each dice is given by

Ans. (a)

19. A box contains 20 defective items and 80 nondefective items. If two items are selected at random without replacement, what will be the probability that both items are defective? 1 5 20 (c) 99 (a)

(GATE 2006, 2 Marks)

=1

1 25 19 (d) 495

gg

2

3

4

5

6

7

8

9

10 11 12

1 2 3 4 5 6 5 4 3 2 1 P( (gg))) P( 36 36 36 36 36 36 36 36 36 36 36

(b)



The above table has been obtained by taking all different ways of obtaining a particular sum. For example, a sum of 5 can be obtained by (1, 4), (2, 3), (3, 2) and (4, 1).

(GATE 2006, 1 Mark) P(x = 5) = 4/36

Solution:  The problem can be solved by hypergeometric distribution, C2 × 80C2

P(r > 6) = P(r > 7) 6 5 4 3 2 1 21 7 = + + + + + = = 36 36 36 36 36 36 36 12

20

P (X = 2) =

100

C2



19 = 495 Ans. (d)

20. A probability density function is of the form p(x) = Ke−a |x|, x ∈ (−∞, ∞). The value of K is (a) 0.5

(b) 1

(c) 0.5a

(d) a (GATE 2006, 1 Mark)

Chapter 5.indd 285

Now let us consider option (a),

Therefore, option (a) is wrong. Consider option (b), P(r/3 is an integer) = P(r = 3) + P(r = 6) + 2 5 4 1 12 1 P(r = 9) + P(r = 12) = + + + = = 36 36 36 36 36 3 Therefore, option (b) is wrong.

8/22/2017 3:35:56 PM

286     Probability and Statistics 

Consider option (c),

Solution:  The probability that exactly n elements are chosen is equal to the probability of getting n heads out of 2n tosses, that is = 2n Cn (1/2)n (1/2)2n −n (Binomial formula)

1 P(r = 8 | r/4 is an integer) = 36 Now, P(r/4 is an integer) = P(r = 4) + P(r = 8) + 3 5 1 9 1 P(r = 12) = + + = = 36 36 36 36 4 P(r = 8 and r/4 is an integer) = P(r = 8) = 5/36 5 20 5 P(r = 8 | r/4 is an integer) = 36 = = 1 36 9 4 Hence, option (c) is correct.  Ans. (c) 22. Three companies X, Y and Z supply computers to a university. The percentage of computers supplied by them and the probability of those being defective are tabulated below:

=

2n

Cn (1/2)n (1/2)n

2n

Cn

=

2n

=

2n

Cn

(2 )

2n

=

2 n

2

4n



Ans. (a)

24. There are 25 calculators in a box. Two of them are defective. Suppose five calculators are randomly picked for inspection (i.e., each has the same chance of being selected). What is the probability that only one of the defective calculators will be included in the inspection? 1 2 1 (c) 4

1 3 1 (d) 5

(a) Company % of Computer P ­ robability of ­Being Defective

Cn

(b)

X

60%

0.01

Y

30%

0.02

(GATE 2006, 2 Marks)

Z

10%

0.03

Solution:  As population is finite, hypergeometric distribution is applicable:

Given that a computer is defective, the probability that it was supplied by Y is (a) 0.1 (c) 0.3

C1 × 23C4

2

P(1 defective in 5 calculators) =

25

(b) 0.2 (d) 0.4

C5



=

1 3

Ans. (b)

(GATE 2006, 2 Marks) 25. Consider the continuous random variable with probability density function,

Solution: S → supply by Y, d → defective Probability that the computer was supplied by Y, if the product is defective P (S ∩ d ) P (S /d ) = P (d ) P (S ∩ d ) = 0.3 × 0.02 = 0.006 P (d ) = 0.6 × 0.1 + 0.3 × 0.02 + 0.1 × 0.03 = 0.015 P (S /d ) =

The standard deviation of the random variable is 1 1 (a) (b) 6 3 1 1 (c) (d) 3 6 (GATE 2006, 2 Marks)

Ans. (d)

23. For each element in a set of size 2n, an unbiased coin is tossed. The 2n coins tossed are independent. An element is chosen if the corresponding coin toss were head. The probability that exactly n elements are chosen is 2n 2n (a)   4n (b)   2n n n (d)

1 2

(GATE 2006, 2 Marks)

Chapter 5.indd 286

 = 1 − t for 0 ≤ t ≤ 1

0.006 = 0.4 0.015



2n (c) 1   n

f(t) = 1 + t for — 1 ≤ t ≤ 0

Solution:  Mean is given by ∞

1

mt = E(t) ∫ t ⋅ f (t)dt = −∞

0

=



−1

∫ t ⋅ f(t) dt

−1

1

t(1 + t)dt + ∫ t(1 − t) dt 0 3 0

 t2 t  t2 t3  1 =  +  +  −  3  3   2  2 0 −1 1 1 1 1 1 1 = − −  +  −  −  +  = 0  2 3   2 3   6 6 

8/22/2017 3:36:05 PM

SOLVED GATE PREVIOUS YEARS’ QUESTIONS     287

Variance = E(t2 ) − [ E(t)]2 ∞

=





t f (t)dt − [ E(t)] = 2

2

−∞



t2f (t)dt − (0)2

−∞

0

=

27. Suppose we uniformly and randomly select a permutation from the 20! permutations of 1, 2, 3, …, 20. What is the probability that 2 appears at an earlier position than any other even number in the selected permutation? 1 1 (a) (b) 2 10 9! (c) (d) None of these 20! (GATE 2007, 2 Marks)

1

∫ (t

2

−1

+ t3 )dt + ∫ t2 (1 − t)dt 0

 t3 t 4  1  t3 t =  +  +  −  4  4   3  3 0 −1 1 1 1 =− + = 12 12 6 4 0

Standard deviation =

Solution:  Number of permutations with `2’ in the first place = 19! Variance =



1 6

Number of permutations with `2’ in the second place = 10 × 18 !

Ans. (b)

Number of permutations with `2’ in the third place = 10 × 9 × 17 !

26. A class of first year B.Tech. students is composed of four batches A, B, C and D, each consisting of 30 students. It is found that the sessional marks of students in Engineering Drawing in batch C have a mean of 6.6 and standard deviation of 2.3. The mean and standard deviation of the marks for the entire class are 5.5 and 4.2, respectively. It is decided by the course instructor to normalize the marks of the students of all batches to have the same mean and standard deviation as that of the entire class. Due to this, the marks of a student in batch C are changed from 8.5 to (a) 6.0

(b) 7.0

(c) 8.0

(d) 9.0

Solution:  Let the mean and standard deviation of the students of batch C be mc and s c, respectively, and the mean and standard deviation of the entire class of first year students be m and s, respectively. Now given, mc = 6.6, s c = 2.3, m = 5.5 and s = 4.2 In order to normalize batch C to entire class, the normalized score (Z scores) must be equated. As x−m x − 5.5 Z= = s 4.2 xc − mc 8. 5 − 6. 6 Zc = = sc 2.3

(a) 0.5 (c) 0.12

(b) 0.18 (d) 0.06 (GATE 2007, 2 Marks)

Solution:  Let A denote the event of failing in Paper 1 and B denote the event of failing in Paper 2. Then we are given that P ( A ) = 0. 3 P (A/B ) = 0.6

8. 5 − 6. 6 x − 5. 5 = 2. 3 4.2

Chapter 5.indd 287

28. An examination consists of two papers, Paper 1 and Paper 2. The probability of failing in Paper 1 is 0.3 and that in Paper 2 is 0.2. Given that a student has failed in Paper 2, the probability of failing in Paper 1 is 0.6. The probability of a student failing in both the papers is

P ( B ) = 0. 2

Equating these two and solving, we get



19! + 10 ×18! + 10 × 9 × 17! + 10 × 9 × 8 × 16! +  + 10! × 9! Now, the probability of this happening is given by 19! + 10 × 18! + 10 × 9 × 17! +  + 10! × 9! 20! which is clearly not options (a), (b) or (c). Therefore, correct option is (d), that is, none of these.  Ans. (d)

(GATE 2006, 2 Marks)



and so on until `2’ is in the eleventh place. After that, it is not possible to satisfy the given condition, because there are only 10 odd numbers available to fill before `2’. So, the desired number of permutations which satisfies the given condition is

Probability of failing in both, P (A ∩ B ) = P (B ) ∗ P (A / B ) = 0.2 ∗ 0.6 = 0.12

x = 8.969  9.0 Ans. (d)



Ans. (c)

8/22/2017 3:36:14 PM

288     Probability and Statistics 

29. A loaded dice has the following probability distribution of occurrences Dice value

1

2

3

4

5

6

Probability

1 4

1 8

1 8

1 8

1 8

1 4

(c) Var(X + Y ) = Var(X ) + Var(Y ) (d) E(X 2Y 2 ) = (E(X ))2 (E(Y ))2 (GATE 2007, 2 Marks) Solution: Option (a) is true.

If three identical dice as the above are thrown, the probability of occurrence of values 1, 5 and 6 on the three dice is

Option (b) is true. Option (c) is true. Option (d) is false.

(a) same as that of occurrence of 3, 4, 5 Because E(X 2Y 2 ) = (E(X ))2 (E(Y ))2 (b) same as that of occurrence of 1, 2, 5 However, as X is not independent of X,

(c) 1/128

E(X 2 ) ≠ [ E(X )] ‚ 2

(d) 5/8 (GATE 2007, 2 Marks) Solution: Dice value

1

2

3

4

5

6

Probability

1 4

1 8

1 8

1 8

1 8

1 4

2 2 ∴ E(X 2Y 2 ) = E(X 2 )E(Y 2 ) ≠ [ E(X )]  E(Y 2 )    Ans. (d)

32. A coin is tossed 4 times. What is the probability of getting heads exactly 3 times? (a) 1/4

(b) 3/8

(c) 1/2

(d) 3/4

As the dice are independent, (GATE 2008, 1 Mark)

P (1, 5, 6) =

1 1 1 1 × × = 4 8 4 128 1 1 1 1 P (3, 4, 5) = × × = 8 8 8 512 1 1 1 1 P (1, 2, 5) = × × = 4 8 8 256 1 Therefore, option (c), P(1, 5 and 6) = is correct. 128  Ans. (c) 30. If the standard deviation of the spot speed of vehicles in a highway is 8.8 kmph and the mean speed of the vehicles is 33 kmph, the coefficient of variation in speed is (a) 0.1517

(b) 0.1867

(c) 0.2666

(d) 0.3646 (GATE 2007, 2 Marks)

Solution:  We know that P = P(H) = 0.5 Probability of getting head exactly 3 times is P (X = 3) =4 C3 (0.5)1 = 

1 4 Ans. (a)

33. A person on a trip has a choice between private car and public transport. The probability of using a private car is 0.45. While using the public transport, further choices available are bus and metro, out of which the probability of commuting by a bus is 0.55. In such a situation, the probability (rounded up to two decimals) of using a car, bus and metro, respectively, would be (a) 0.45, 0.30 and 0.25 (b) 0.45, 0.25 and 0.30

Solution: s 8.8 C.V. = = = 0.2666 33 m 

(c) 0.45, 0.55 and 0.00 (d) 0.45, 0.35 and 0.20 Ans. (c) (GATE 2008, 2 Marks)

31. Let X and Y be two independent random variables. Which one of the relations between expectation (E), variance (Var) and covariance (Cov) given below is FALSE? (a) E(XY ) = E(X )E(Y ) (b) Cov(X , Y ) = 0

Chapter 5.indd 288

Solution:  We are given that P(Car) = 0.45 Now, probability of choosing a public transport = 0.55 Furthermore, probability of commuting by a bus = 0.55

8/22/2017 3:36:23 PM

SOLVED GATE PREVIOUS YEARS’ QUESTIONS     289

Hence, P(Bus) = 0.55 × 0.55 = 0.30  1 P − < x <  3

Now, probability of commuting by a metro = 0.45

1  = 3 

P(Metro) = 0.55 × 0.45 = 0.25 

(c) 0.4

(d) 0.6

1

 x3  3 2   f ( x ) dx = x dx = ∫ ∫ 3 1   − 1 1 − − 3

3

34. Aishwarya studies either computer science or mathematics everyday. If she studies computer science on a day, then the probability that she studies mathematics the next day is 0.6. If she studies mathematics on a day, then the probability that she studies computer science the next day is 0.4. Given that Aishwarya studies computer science on Monday, what is the probability that she studies computer science on Wednesday? (b) 0.36

1 3

3

2 = 81

Ans. (a)

(a) 0.24

1 3

The probability expressed in percentage, P  =

2 × 100 = 2.469% = 2.47% 81



Ans. (b)

36. Let X be a random variable following normal distribution with mean +1 and variance 4. Let Y be another normal variable with mean −1 and variance unknown. If P(X ≤ −1) = P(Y ≥ 2), the standard deviation of Y is (a) 3

(b) 2

(c)

(d) 1

(GATE 2008, 2 Marks) Solution:  Let C denote computer science study and M denote maths study.

2

(GATE 2008, 2 Marks) Solution:  Given, mx = 1, σx2 = 4 ⇒ s y = 2

Now by rule of total probability, we total up the desired branches and get the answer as shown below:

Also given, my = —1 and s y is unknown Given, P(X ≤ −1) = P(Y ≥ 2) Converting into standard normal variates.

P(C on Monday and C on Wednesday) = P(C on Monday, C on Tuesday and C on Wednesday) + P(C on Monday, M on Tuesday and C on Wednesday)

  2 − my  −1 − mx    = P z ≥ P z ≤   s x  s y 

= 1 × 0.6 × 0.4 + 1 × 0.4 × 0.4 = 0.24 + 0.16

  2 − (−1) −1 − 1  P z ≤  = P z ≥    2  s y 

= 0.40 

Ans. (c)

35. If probability density function of a random variable X is f (X ) = X 2 for − 1 ≤ X ≤ 1 = 0 for any other value of x



 1 1 then the percentage probability P − < x2 <  is  3 3 (a) 0.247

(b) 2.47

(c) 24.7

(d) 247 (GATE 2008, 2 Marks)

Solution:  Given f (x) = x2 for − 1 < x < 1 = 0 elsewhere

 3   P (z ≤ −1) = P z ≥   s y 



(i)

Now, we know that in standard normal distribution, P(z ≤ −1) = P(z ≥ 1)



(ii)

Comparing Eqs. (i) and (ii), we can say that 3 = 1 ⇒ sy = 3 sy 

Ans. (a)

37. A fair coin is tossed 10 times. What is the ­probability that only the  1 2first two tosses will yield (a )   heads? 2 1 (b) 10 C2  

1 (a )  

2

2

2

2



1 (b) 10 C2  

1 (c)  

1 (c)  

1 (d) 10 C2  

2

2

10

2 10

10

2

2

1 (d) 10 C2  

10

2

Chapter 5.indd 289

8/22/2017 3:36:30 PM

1 (a )  

2

2

1 (b) 10 C2  

1 (a )  

2

2

2

2

 1   1  and Statistics  290     Probability 10   2

10

(b) C2   2

(c)  

1 (c)  

1 (d) 10 C2  

2

10

10

2

2

1 (d) 10 C2  

10

(GATE 2009, 1 Mark) 2 Solution:  Probably of only the first two tosses being heads = P(H, H, T, T, T, …, T) As each toss is independent, the required ­probability = P (H ) × P (H ) × [P (T )]8  1 2  1 2  1 10 =     =    2   2   2  

Ans. (c)

38. If two fair coins are flipped and at least one of the outcomes is known to be a head, then what is the probability that both outcomes are heads?

value is odd is 90% of the probability that the face value is even. The probability of getting any evennumbered face is the same. If the probability that the face is even given that it is greater than 3 is 0.75, which one of the following options is closest to the probability that the face value exceeds 3? (a) 0.453

(b) 0.468

(c) 0.485

(d) 0.492 (GATE 2009, 2 Marks)

Solution:  It is given that P(odd) = 0.9P(even) ∑ p(x) = 1 Now, P(odd) + P(even) = 1 0.9P(even) + P(even) = 1

(a) 1/3

(b) 1/4

(c) 1/2

(d) 2/3

P(even) = 1/1.9 = 0.5263 Now, it is given that P(any even face) is same

(GATE 2009, 1 Mark) Solution:  Let A be the event of head in one coin and B be the event of head in second coin. The required probability is P (A ∩ B ) ∩ (A ∪ B ) P (A ∩ B | A ∪ B ) =  P (A ∪ B ) P (A ∩ B ) = P (A ∪ B ) P (A ∩ B ) = P (both coin heads ) = P (H, H ) =

⇒ P (2) = P (4) = P (6) Now, P(even) = P(2) or P(4) or P(6) = P(2) + P(4) + P(6) ∴

P(2) = P(4) = P(6) = 1/3P(even)



= 1/3(0.5263) = 0.1754

It is given that P(even | face > 3) = 0.75

1 4



P (even ∩ face > 3) = 0.75 P (face > 3)



P (face = 4, 6) = 0.75 P (face > 3)

P(HH, HT, TH) = 3/4 1 So, required probability = 4 = 1 3 3 4 

⇒ P (face > 3) =

Ans. (a)

39. If three coins are tossed simultaneously, the ­probability of getting at least one head is (a) 1/8

(b) 3/8

(c) 1/2

(d) 7/8 (GATE 2009, 1 Mark)

Solution:  Here, total outcomes = 23 = 8. The probability of getting zero heads when three coins are tossed = 1/8 (when all three are tails). Hence, the probability of getting at least one head when three coins are tossed = 1 — 1/8 = 7/8.  Ans. (d) 40. An unbalanced dice (with 6 faces, numbered from 1 to 6) is thrown. The probability that the face

Chapter 5.indd 290

P (face = 4, 6) 0.75

P (4) + P (6) 0.75 0.1754 + 0.1754 = = 0.4677  0.468 0.75

=



Ans. (b)

41. The standard deviation of a uniformly distributed random variable between 0 and 1 is 1 1 (a) (b) 12 3 5 7 (c) (d) 12 12 (GATE 2009, 2 Marks)

8/22/2017 3:36:36 PM

SOLVED GATE PREVIOUS YEARS’ QUESTIONS     291

Solution:  s =

(b − a )2 (1 − 0)2 1 = = 12 12 12



Ans. (a)

42. The standard normal probability function can be approximated as 1 F(X ) = N 1 + exp(−1.7255 xn|Xn|0.12 ) where XN    = standard normal deviate. If mean and standard deviation of annual precipitation are 102 cm and 27 cm, respectively, the probability that the annual precipitation will be between 90 cm and 102 cm is (a) 66.7% (c) 33.3%

2 1 Probability of drawing 2 washers first =  ×   9 8  Probability of drawing 3 nuts after drawing 3 2 1 2 washers =  × ×   7 6 5  Probability of drawing 4 bolts after drawing 4  4 3 2 1 bolts =  × × ×   4 3 2 1 P(2 washers, then 3 nuts, then 4 bolts)  2 1   3 2 1   4 3 2 1 1 =  ×  ×  × ×  ×  × × ×  =  9 8   7 6 5   4 3 2 1 1260 

Ans. (c)

(b) 50.0% (d) 16.7%

44. A box contains 4 white balls and 3 red balls. In succession, two balls are randomly selected and removed from the box. Given that the first (GATE 2009, 2 Marks) removed ball is white, the probability that the Solution:  Here m = 102 cm and s = 27 cm. second removed ball is red is m = 102 cm and s = 27 cm.  90 − 102 − 102 102   ≤x≤ P (90 90 ≤− x≤ 102) = P 102 (a) 1/3 (b) 3/7   − 102 102   27 ≤ x ≤  27  P (90 ≤ x ≤ 102) = P   27 (c) 1/2 (d) 4/7 = P (−027 .44 ≤ x ≤ 0) (GATE 2010, 2 Marks) = P (−0.44 ≤ x ≤ 0) Solution:  We have to calculate, This area is shown as follows: P (II is red and I is white)

P(II is red | I is white) =

−0.44

=

The shaded area in the above figure is given by F (0) − F (−0.44) =

P (I is white and II is red) P (I is white)

Probability of first removed ball being white =

4 7

Probability of second removed ball being red =

3 6

1 1 + exp(0)

1 = 1 + exp[−1.7255(−0.44)(0.44)0.12)] = 0.5 − 0.3345 = 1.1655  16.55% Hence, the closest answer is 16.7%. 

Ans. (d)

43. A box contains 2 washers, 3 nuts and 4 bolts. Items are drawn from the box at random one at a time without replacement. The probability of drawing 2 washers first followed by 3 nuts and subsequently the 4 bolts is (a) 2/315

(b) 1/630

(c) 1/1260

 (d) 1/2520 (GATE 2010, 2 Marks)

Solution:  Box contains 2 washers, 3 nuts and 4 bolts.

Chapter 5.indd 291

P (I is white)

4 3 × 3 1 P(II is red | I is white) = 7 6 = = 4 6 2 7 

Ans. (c)

45. Consider a company that assembles computers. The probability of a faulty assembly of any computer is p. The company therefore subjects each computer to a testing process. This testing process gives the correct result for any computer with a probability of q. What is the probability of a computer being declared faulty? (a) pq + (1 − p) (1 − q)

(b) (1 − q)p

(c) (1 − p)q

(d) pq (GATE 2010, 2 Marks)

8/22/2017 3:36:42 PM

292     Probability and Statistics 

Solution:  The probability of a faulty assembly of any computer = p The probability of a computer being declared faulty from a faulty assembly = pq The probability of a non-faulty assembly of any computer = 1 − q The probability of a computer being declared faulty from a non-faulty assembly = (1 − p)(1 − q) Required probability = pq + (1 − p)(1 − q)  Ans. (a) 46. A fair coin is tossed independently four times. The probability of the event “the number of times heads show up is more than the number of times tails show up” is 1 1 (a) (b) 8 16 1 5 (c) (d) 4 16 (GATE 2010, 2 Marks) Solution:  Coin is tossed 4 times. Hence, the total outcomes = 24 = 16.

Solution:  In the first toss, results can be 1, 2, 3, 4, 5. For 1, the second toss results can be 2, 3, 4, 5, 6. For 2, the second toss results can be 3, 4, 5, 6. For 3, the second toss results can be 4, 5, 6. For 4, the second toss results can be 5, 6. For 5, the second toss results can be 6. The required probability =

1 5 1 4 1 3 1 2 1 1 5 ⋅ + ⋅ + ⋅ + ⋅ + ⋅ = 6 6 6 6 6 6 6 6 6 6 12



Ans. (c)

49. If the difference between the expectation of the 2 square of a random variable (E[x]) and the square of the expectation of the random variable 2 (E[x]) is denoted by R, then (a) R = 0

(b) R < 0

(c) R ≥ 0

(d) R > 0



(GATE 2011, 1 Mark)

Solution:  V(x) = E(x2) − [E(x)]2 = R where V(x) is the variance of x.

Favorable outcomes (the number of times heads show up is more than the number of times tails show up) = {HHHH, HHTH, HHHT, HTHH, THHH} = 5 5 Hence, the required probability = . 16  Ans. (d) 47. There are two containers, with one containing 4 red and 3 green balls and the other containing 3 blue and 4 green balls. One ball is drawn at random from each container. The probability that one of the ball is red and the other is blue will be 1 9 (a)     (b) 7 49 12 3 (c) (d) 49 7 (GATE 2011, 1 Mark) 4 Solution:  Probability that one of the ball is red = 7 3 Probability that one of the ball is blue = 7 Probability that one of the ball is red and the other 4 3 12 is blue = × = 7 7 49  Ans. (c) 48. A fair dice is tossed two times. The probability that the second toss results in a value that is higher than the first toss is (a) 2/36 (c) 5/12

Chapter 5.indd 292

(b) 2/6 (d) 1/2 (GATE 2011, 1 Mark)

As variance is s x2 and hence never negative, R ≥ 0. 

Ans. (c)

50. A deck of five cards (each carrying a distinct number from 1 to 5) is shuffled thoroughly. Two cards are then removed one at a time from the deck. What is the probability that the two cards are selected with the number on the first card being one higher than the number on the second card? 1 4 (a)   (b) 5 25 2 (c) 1.4 (d) 5 (GATE 2011, 2 Marks) Solution:  The five cards are {1, 2, 3, 4, 5} Sample space = 5 × 4 Favorable outcome = P {(2,1) , (3,2) , (4,3) , (5,4)} = 

4 1 = 5×4 5 Ans. (a)

51. Consider the finite sequence of random values X  = [x1 , x2 , … , xn ]. Let mx be the mean and σx be the standard deviation of X. Let another finite sequence Y of equal length be derived from this as yi = a ∗ xi + b where a and b are positive constant. Let my be the mean and s y be the standard deviation of this sequence. Which one of the following statements is incorrect?

8/22/2017 3:36:52 PM

SOLVED GATE PREVIOUS YEARS’ QUESTIONS     293

(a) Index position of mode of X in X is the same as the index position of mode of Y in Y. (b) Index position of median of X in X is the same as the index position of median of Y in Y. (c) my = amx + b (d) σy = aσx + b (GATE 2011, 2 Marks) Solution:  Standard deviation is affected by scale but not by shift of origin. So, yi = axi + b ⇒

sy = asx

and not

sy = asx + b



54. The annual precipitation data of a city is normally distributed with mean and standard deviation as 1000 mm and 200 mm, respectively. The probability that the annual precipitation will be more than 1200 mm is (a) 1200) = P z >    200 = P (z > 1)

Ans. (d)

52. An unbiased coin is tossed five times. The outcome of each toss is either a head or a tail. The probability of getting at least one head is 1 13 (a) (b) 32 32 16 31 (c) (d) 32 32 (GATE 2011, 2 Marks)

where z is the standard normal variate. In normal distribution, P(−1 < z < 1) ≈ 0.68 (  68% of data is within one standard deviation of mean) 0.68 P(0 < z < 1) = = 0.34 2

Solution:  Here, total outcomes = 25 = 32.

So, P(z > 1) = 0.5 − 0.34 = 0.16  16%

The probability of getting zero heads when three coins are tossed = 1/32 (when all five are tails).

which is < 50%. 

Hence, the probability of getting at least one head when three coins are tossed = 1 − 1/32 = 31/32.  Ans. (d) 53. Consider a random variable X that takes values +1 and −1 with probability 0.5 each. The values of the cumulative distribution function F(x) at x  =  −1 and +1 are (a) 0 and 0.5 (c) 0.5 and 1

(b) 0 and 1 (d) 0.25 and 0.75



(GATE 2012, 1 Mark)

Solution:  The probability distribution table of the random variable is x

−1

+1

P(x)

0.5

0.5

Ans. (a)

55. Two independent random variables X and Y are uniformly distributed in the interval [−1, 1]. The probability that max[X, Y] is less than 1/2 is (a) 3/4 (c) 1/4

(GATE 2012, 1 Mark)

Solution:  −1< x < 1 and −1 ≤ y ≤ 1 is the entire rectangle. The region in which maximum of (x, y) is less than 1 is shown below as the shaded region inside the 2 rectangle. y 1

(−1, 1)

The cumulative distribution function F(x) is the probability up to x as follows: x

−1

+1

(b) 9/16 (d) 2/3

(1, 1)

1/2 −1

1/2

1 x

0 F(x)

0.5

1.0

Hence, the values of cumulative distributive function are 0.5 and 1.0.  Ans. (c)

Chapter 5.indd 293

(−1, −1)

−1

(1, −1)

8/22/2017 3:36:57 PM

294     Probability and Statistics 

 1 Area of shaded region P max x, y <  =  2  Area of entire rectanglle

1 9 15 5 + = = 6 36 36 12

= 

3 3 × 9 = 2 2 = 2×2 16 

Ans. (b)

56. A fair coin is tossed till a head appears for the first time. The probability that the number of required tosses is odd is 1 1 (a) (b) 3 2 2 3 (c) (d) 3 4 (GATE 2012, 2 Marks)

Ans. (b)

58. In an experiment, positive and negative values are equally likely to occur. The probability of obtaining at most one negative value in five trials is (a) 1/32

(b) 2/32

(c) 3/32

(d) 6/32 (GATE 2012, 2 Marks)

Solution:  As negative and positive are equally likely, the distribution of number of negative values is binomial with n = 5 and p = 1/2. Let X represents the number of negative values in 5 trials.

Solution:  Probability that the number of toss is odd = Probability of the number of toss is 1, 3, 5, 7, …

P(at most 1 negative value) = P(x ≤ 1)

Probability that the number of toss is 1 = Probability of getting head in first toss = 1/2

 1 0  1 5  1 1  1 4 6 = 5C0     + 5C1     =  2   5   2   2  32 6 . Hence, required probability is 32  Ans. (d)

Probability that the number of toss is 3 = Probability of getting tail in first toss, tail in second toss and head in third toss 1 1 1 1 × × = 2 2 2 8 Probability that the number of toss is 5 = P(T, T, =

 1 5 1 T, T, H) =   =  2  32

= P(x = 0) + P(x =1)

59. A box contains 4 red balls and 6 black balls. Three balls are selected randomly from the box one after another, without replacement. The probability that the selected set contains 1 red ball and 2 black balls is (a) 1/20 (c) 3/10

(b) 1/12 (d) 1/2

So, probability that the number of tosses are odd (GATE 2012, 2 Marks) 1 1 1 + = + + 2 8 32 Sum of infinite geometric series with 1 1 1 and r = = 2 = a= 2 4 1− 1 4 

Solution:  P(1 red and 2 black) C1 × 6C2

4

1 2 =2 3 3 4 Ans. (c)

57. Suppose a fair six-sided die is rolled once. If the value on the die is 1, 2 or 3, the die is rolled a second time. What is the probability that the total sum of values that turn up is at least 6?

=

10

C3

=

60 1 = 120 2



Ans. (d)

60. A continuous random variable X has a probability density function f (x) = e−x , 0 < x < ∞. Then P{X > 1} is (a) 0.368 (c) 0.632

(b) 0.5 (d) 1.0 (GATE 2013, 1 Mark)

Solution:  (a) 10/21 (c) 2/3

(b) 5/12 (d) 1/6



P = (GATE 2012, 2 Marks) Solution:  P(sum > 6) = P(6 on first throw) + P(1, 5) + P(1, 6) + P(2, 4) + P(2, 5) + P(2, 6) + P(3, 3) + P(3, 4) + P(3, 5) + P(3, 6)

Chapter 5.indd 294

∫ 1





f (x)dx =

−x

∫e

dx = e−x

1 ∞

= e−1 = 0.368

1

Ans. (a)

61. Suppose p is the number of cars per minute passing through a certain road junction around

8/22/2017 3:37:03 PM

SOLVED GATE PREVIOUS YEARS’ QUESTIONS     295

 8 1 3  ⇒ l −  −  + (4 − 1) − 2(2 − 1) = 1   3 3  2   7 9  ⇒ l − + − 2 = 1  9 2 

5  PM, and p has Poisson distribution with mean  3. What is the probability of observing fewer than 3 cars during any given minute in this interval? (a) 8 (2e3)

(b) 9 (2e3 )

(c) 17 (2e3 )

(d) 26 (2e3 )

 −14 + 27 − 12   =1 ⇒l  6 

(GATE 2013, 1 Mark) Solution:  Poisson formula for (p = x) is given as e−x l x . x! l: mean of Poisson distribution = 3 (given) Probability of observing fewer than 3 cars = (p = 0) + (p = 1) + (p = 2) =

e−3 l 0 e−3 l 1 e−3 l 2 17 + + = 3 0! 1! 2! 2e



Ans. (c)

62. Let X be a normal random variable with mean 1 and variance 4. The probability P{X < 0} is

⇒l=

6 =6 1 l=6

64. The security system at an IT office is composed of 10 computers of which exactly four are working. To check whether the system is functional, the officials inspect four of the computers picked at random (without replacement). The system is deemed functional if at least three of the four computers inspected are working. Let the probability that the system is deemed functional be denoted by p then 100p = . (GATE 2014, 1 Mark)

(a) 0.5 Solution:  If p is the probability that system is functional means at least three systems are working, p will be calculated as:

(b) greater than zero and less than 0.5 (c) greater than 0.5 and less than 1.0 (d) 1.0

p for at least 3 systems are working All 4 systems working + 3 systems working = Number of ways to pick 4 systems from 100

(GATE 2013, 1 Mark) Solution: x − m 0 − m  P (x < 0) = P  <  − P (Z < −0.5)  s s 

p that 3 systems are working = (Number of systems not selected) ×(Number of ways to pick 3 from 4 selected systems) = 6×4×3×2×4

= P (Z > 0.5) = 0.5 − P (0 < Z < 0.55) Hence, the probability is greater than zero and less than 0.5. Ans. (b) 63. Find the value of l such that function f(x) is valid probability density function, =−l1()( x 2− −1)( − x1) ≤ forx1≤≤2x ≤ 2 f (x) f=(xl)(x x2) for 0 otherwise = 0= otherwise (GATE 2013, 2 Marks) ∞



Solution:

  f (x) = l(−x + 3x − 2) 1 ≤ x ≤ 2   0 otherwise 2

∫ l(−x 1

2

+ 3x − 2)dx = 1

2  x3 x2  ⇒ l − +3 − 2x = 1 3 2 1 

Chapter 5.indd 295



p for at least 3 systems are working (4 × 3 × 2 × 1) + (6 × 4 × 3 × 2 × 4) = = 0.1192 10 × 9 × 8 × 7 So, 100p = 11.92

f (x)dx = 1

−∞ 2



Therefore,

Ans. 11.92 65. Each of the nine words in the sentence `The quick brown fox jumps over the lazy dog’ is written on a separate piece of paper. These nine pieces of paper are kept in a box. One of the pieces is drawn at random from the box. The expected length of the word drawn is . (The answer should be rounded to one decimal place.) (GATE 2014, 1 Mark)

8/22/2017 3:37:11 PM

296     Probability and Statistics 

Solution:  The given 9 words are as follows: The, Quick, Brown, Fox, Jumps, Over, The, Lazy, Dog Words with length 3 are THE, FOX, THE, DOG (4) Words with length 4 are OVER, LAZY (2) Words with length 5 are QUICK, BROWN, JUMPS (3) Probability for drawn 3 length words = 4/9 Probability for drawn 4 length words = 2/9 Probability for drawn 5 length words = 3/9 Expected word length  4  2  3  = 3 ×  + 4 ×  + 5 ×  = 3.88        9 9 9 



Ans. 3.88 66. In a housing society, half of the families have a single child per family, while the remaining half have two children per family. The probability that a child picked at random has a sibling is . (GATE 2014, 1 Mark) Solution:  Let E1 = one child family, E          2 = two children family and A = picking a child. Then by Bayes’ theorem, required probability is given by 1 x 2 2 P (E2 /A) = = = 0.667 1.x 1 + x 3 2 2 2 where x is the number of families. Ans. 0.667 67. Let X1, X2 and X3 be independent and identically distributed random variables with the uniform distribution on [0, 1]. The probability P{X1 is the largest} is . (GATE 2014, 1 Mark)

68. Let X be a random variable which is uniformly chosen from the set of positive odd numbers less than 100. The expectation, E[X ], is . (GATE 2014, 1 Mark) Solution:  X = 1, 3, 5,..., 99 ⇒ n = 50 (number of observations) Therefore E (x ) =

=

1 n 1 ∑ x = 50 [1 + 3 + 5 +  + 99] n i= j i 1 2 ⋅ (50) = 50 50

Ans. 50 69. An unbiased coin is tossed an infinite number of times. The probability that the fourth head appears at the tenth toss is (a) 0.067

(b) 0.073

(c) 0.082 (d) 0.091 (GATE 2014, 1 Mark)

Solution:  P[fourth head appears at the tenth toss] = P[getting 3 heads in the first 9 tosses and one head at tenth toss]   1 9   1  21 =  9C3 ⋅      = = 0.082  2   2  256   Ans. (c) 70. Let X be a zero mean unit variance Gaussian random variable. E[|X|] is equal to . (GATE 2014, 1 Mark) Solution: X − N (0.1) ⇒ f (x) =

e−x

1 2p

2

/2

, −∞ < x < ∞

Therefore ∞

E{ x } =



x f (x )

−∞ ∞

Solution:  Given that three random variables X1, X2, and X3 are uniformly distributed on [0, 1], we have the following possible values: X1 X1 X2 X2 X3 X3

> > > > > >

X2 X3 X1 X3 X1 X2

> > > > > >

X3 X2 X3 X1 X2 X1

Since all the three variables are identical, the probabilities for all the above inequalities are same. Hence, the probability that X1 is the largest is P{X1 is the largest} = 2/6 = 1/3 = 0.33 Ans. 0.33

Chapter 5.indd 296

1 =

=

2p

x2 ∫ xe−x



2

2

/2

dx =

0

2p

−u

∫e

du

0

2 = 0.797  0.8 p Ans. 0.8

71. Consider a dice with the property that the probability of a face with n dots showing up is proportional to n. The probability of the face with three dots showing up is . (GATE 2014, 1 Mark) Solution:  P(n) = K ⋅ n where n = 1 to 6. We know that,

8/22/2017 3:37:15 PM

SOLVED GATE PREVIOUS YEARS’ QUESTIONS     297

∑ P (x) = 1 ⇒ K [1 + 2 + 3 + 4 + 5 + 6] = 1 n

⇒K =

1 21

Therefore, required probability is P(3) = 3K =

1 . 7

72. Lifetime of an electric bulb is a random variable with density f(x) = kx2, where x is measured in years. If the minimum and maximum lifetimes of bulb are 1 and 2 years, respectively, then the value of k is . (GATE 2014, 1 Mark) Solution:  Density of the random variable is f(x) = kx2. Since f(x) is a PDF  2 f (x) = kx , 1 < x < 2  0, otherwise Therefore, 2

∫ 1

 x3  2 3 f (x) dx = 1 ⇒ k   = 1 ⇒ k = = 0.428  0.43 7  3 1

73. A box contains 25 parts of which 10 are defective. Two parts are being drawn simultaneously in a random manner from the box. The probability of both the parts being good (a)

7 20

(b)

42 125

(c)

25 29

(d)

5 9

(GATE 2014, 1 Mark) Solution:  The probability of selecting two parts from 25 is 25C2. The probability of selecting two good parts is (25−10)C2. 15

C2

105 7 So, the required probability is 25 = = 300 20 C2 Ans. (a) 74. A group consists of equal number of men and women. Of this group, 20% of the men and 50% of the women are unemployed. If a person is selected at random from this group, the probability of the selected person being employed is . (GATE 2014, 1 Mark) Solution:  Let M be the event of person selected is a man, W be the event of person selected is a woman, E be the event of person selected being employed and U be the event of person selected being unemployed.

Chapter 5.indd 297



Given that P(M) = 0.5, P(W) = 0.5, P(U/M) = 0.2 and P(U/W) = 0.5 The probability of selecting an unemployed person is P(U) = P(M) × (P(U/M) + P(W) × (P(U/W) = 0.5 × 0.2 + 0.5 × 0.5 = 0.35 So, the probability of selecting an employed person = 1 − P(U) = 1 − 0.35 = 0.65 Ans. 0.65

75. A nationalized bank has found that the daily ­balance available in its savings account follows a normal distribution with a mean of H 500 and a standard deviation of H 50. The percentage of savings account holders, who maintain an average daily balance more than H 500 is . (GATE 2014, 1 Mark) Solution:  Given that mean (m) = H 500 and standard deviation (s) = H 50 If x follows the normal distribution, then the standard normal variable, z=

x−m 500 − 500 = =0 s 50

Hence, if x > 500 then z > 0 and so the percentage of savings account holders, who maintain an average daily balance more than H 500 is ≈50%. Ans. 50% 76. The probability density function of evaporation E on any day during a year in a watershed is given by  1 f (E ) =  5  0

0 ≤ E ≤ 5 mm/day otherwise

The probability that E lies in between 2 and 4 mm/day in a day in the watershed is (in decimal) . (GATE 2014, 1 Mark) Solution:  The probability density function of E on any day is given by  1 f (E ) =  5  0 4

P  (2 < E < 4) =

0 ≤ E ≤ 5 mm/day otherwise 4

∫ f (E ) dE = ∫ 5 dE = 5 [E ]2 1

2

1

4

2

1 2 = (4 − 2) = = 0.4 5 5 Ans. 0.4

8/22/2017 3:37:20 PM

298     Probability and Statistics 

77. A fair (unbiased) coin was tossed four times in succession and resulted in the following outcomes:

79. Given that x is a random variable in the range x

e2 [0, ∞] with a probability density function , the K value of the constant K is .

(i) Head, (ii) Head, (iii) Head, (iv) Head. The probability of obtaining a `Tail’ when the coin is tossed again is 1 4 1 (a) 0 (b) (c) (d) 2 5 5 (GATE 2014, 1 Mark) Solution:  Since the coin is unbiased, the probabil1 ity of getting a Tail is . 2  Ans. (b) 78. If {x} is a continuous, real-valued random variable defined over the interval (−∞, +∞) and its occurrence is defined by the density function given as 1  x−a    2  b 

(GATE 2014, 1 Mark) Solution:  We have, ∞ −x / 2



e

=1 K

0

1 e−x /2 ⇒ . K (−1 / 2) ⇒



=1 0

−2 2 (0 − 1) = 1 ⇒ = 1 ⇒ K = 2 K K

2

1

e where `a’ and `b’ are the 2p +b statistical attributes of the random variable {x}.

f (x) =

1  x−a    2  b 

80. Four fair six-sided dice are rolled. The probability that the sum of the results being 22 is X/1296. The value of X is . (GATE 2014, 2 Marks)

2

The value of the integral ∫

a

−∞

(a) 1

1 2p +b

(c) p

(b) 0.5

e

(d)

dx is

p 2

(GATE 2014, 1 Mark) Solution:  We have ∞



f (x) dx = 1

(GATE 2014, 2 Marks)

−∞ a





−∞ a







Solution:  Let A event → Divisible by 2, B event → Divisible by 3, C event → Divisible by 5. Then, P(A′ ∩ B ′ ∩ C ′) = 1 − P(A ∪ B ∪ C ) P(A) = 50/100, P(B) = 33/100, P(C ) = 20/100 P(A ∩ B) = 16/100, P(B ∩ C ) = 6/100, P(A ∩ C ) = 10/100, P(A ∩ B ∩ C ) = 3/100 P(A ∪ B ∪ C ) = P(A) + P(B) + P(C ) − P(A ∩ B) − P(B ∩ C ) − P(A ∩ C ) + P(A ∩ B ∩ C ) P(A ∪ B ∪ C ) = (50 + 33 + 20 − 16 − 6 − 10 + 3)/ 100 = 74/100 P(A′ ∩ B ′ ∩ C ′) = 1 − 74/100 = 26/100 = 0.26 Ans. 0.26

f (x) dx + ∫ f (x) dx = 1 a

f (x) dx = 0.5

−∞

−∞

x=a 0.5

Chapter 5.indd 298

Solution:  Sum 22 comes with following possibilities: 6 + 6 + 6 + 4 = 22 (numbers can be arranged in 4 ways) 6 + 6 + 5 + 5 = 22 (numbers can arranged in 6 ways) Therefore, the total ways X = 4 + 6 = 10 Ans. 10 81. The probability that a given positive integer lying between 1 and 100 (both inclusive) is NOT divisible by 2, 3 or 5 is .

−∞

82. Let S be a sample space and two mutually exclusive events A and B be such that A∪B = S. If P(.) denotes the probability of the event, the maximum value of P(A)P(B) is .

Ans. (b)

(GATE 2014, 2 Marks)

0.5

8/22/2017 3:37:26 PM

SOLVED GATE PREVIOUS YEARS’ QUESTIONS     299

distribution on [0, 1]. The probability P{x1 + x2 ≤ x3} is .

Solution:  Maximum value of P(A) P(B) = ? P(A)P(B) = P(A)[1 − P(A)]

(GATE 2014, 2 Marks) Let P(A) = x f (x) = x − x2 f ′(x) = 1 − 2x = 0 So, x = 1/2 and f ˝(x) = −2 and x is maximum at 1/2. Hence

Solution:  Given x1, x2 and x3 be independent and identically distributed with uniform distribution on [0, 1]. Let z = x1 + x2 = x3. Then P {x1 + x2 ≤ x3 } = P {x1 + x2 − x3 ≤ 0} = P {z ≤ 0}

P(A)P(B) = 0.25 Ans. 0.25 83. A fair coin is tossed repeatedly till both head and tail appear at least once. The average number of tosses required is . (GATE 2014, 2 Marks)

Let us find probability density function of random variable z. Since z is summation of three random variables x1 , x2 and −x3 , overall PDF of z is convolution of the PDF of x1 , x2 and −x3 . PDF of {x1 + x2} is

Solution:  Let the first toss be head. Let x denotes the number of tosses (after getting first head) to get first tail. We can summarize the event as: Event (After getting the first H)

x

T

1

1/2

HT

2

1/2 × 1/2 = 1/4

HHT and so on …

3

1/8

E (x ) =



1

Probability, p(x)

0

1

2

PDF of {−x3} is 1

−1

∑ xp (x) = 1 × 2 + 2 × 4 + 3 × 8 +  1

1

1

x =1

Let,



0

1 1 1 S = 1× + 2 × + 3 × +   2 4 8 1 1 1 1 ⇒ S = + 2× + 3× +  2 4 8 16

(i)



−1

(z + 1)

2

dz = 2

= 6

−1

1 = 0.16 6 Ans. 0.16

(ii)

Subtracting Eq. (ii) from Eq. (i) we get   1 − 1  S = 1 + 1 + 1 + 1 +   2  2 4 8 16 1 1 ⇒ S = 2 = 1 ⇒ S = 2 ⇒ E (x ) = 2 1 2 1− 2 Hence, the expected number of tosses (after first head) to get first tail is 2 and same can be applicable if first toss results in tail. Thus, the average number of tosses is 1 + 2 = 3. Ans. 3 84. Let x1, x2 and x3 be independent and identically distributed random variables with the uniform

Chapter 5.indd 299

P {z ≤ 0} =

3 0

(z + 1)

85. Parcels from sender S to receiver R pass sequentially through two post offices. Each post office has a probability 5 of losing an incoming parcel, independently of all other parcels. Given that a parcel is lost, the probability that it was lost by the second post office is . (GATE 2014, 2 Marks) Solution:  Parcel will be lost if (a) It is lost by the first post office (b) It is passed by first post office but lost by the second post office 1 4 1 9 P(parcel is lost) = + × = 5 5 5 25 P(parcel lost by second post office if it passes first post office) = P(Parcel passed by first post office) × 4 1 4 P(Parcel lost by second post office) = × = 5 5 25

8/22/2017 3:37:31 PM

300     Probability and Statistics 

P(parcel lost by second post office | parcel lost) = 4/25 4 = = 0.44 9/25 9 Ans. 0.44 86. A fair coin is tossed n times. The probability that the difference between the number of heads and tails is (n-3) is (a) 2-n

(c) nCn-32-n (d) 2-n + 3

(b) 0

(GATE 2014, 2 Marks)

88. The mean thickness and variance of silicon steel laminations are 0.2 mm and 0.02, respectively. The varnish insulation is applied on both die sides of the laminations. The mean thickness of one side insulation and its variance are 0.1 mm and 0.01, respectively. If the transformer core is made using 100 such varnish coated laminations, the mean thickness and variance of the core, respectively, are (a) 30 mm and 0.22 (c) 40 mm and 2.44

(b) 30 mm and 2.44 (d) 40 mm and 0.24

Solution:  Let X be the difference between number of heads and tails. Take n = 2 ⇒ S = {HH,HT,TH,TT} and X = -2, 0, 2. Here, n — 3 = —1 is not possible.

(GATE 2014, 2 Marks) Solution:  Mean thickness of laminations = 0.2 mm Mean thickness of one side varnish = 0.1 mm

Take n = 3 ⇒ S = {HHH,HHT,HTH,HTT,THH,THT,TTH,TTT Mean}thickness of varnish including both sides = 0.2 mm H,HHT,HTH,HTT,THH,THT,TTH,TTT and X = -3, -1, 1, 3.

}

Therefore, mean thickness of one lamination varnished on both sides = 0.4 mm

Here, n — 3 = 0 is not possible. Similarly, if a coin is tossed n times, then the difference between heads and tails is n — 3, which is possible. Thus, required probability is 0. Ans. (b) 87. Let X be a random variable with probability density function 0.2,  f (x) = 0.1,  0, 

x ≤1

for

otherwise

The probability P(0.5 < x < 5) is

(d1 − 0.2)2 + (d2 − 0.2)2 + … + (d100 − 0.2)2/100 = 0.02 (x1 − 0.1)2 + (x2 − 0.1)2 + … + (x100 − 0.1)2 = 0.01

.

(GATE 2014, 2 Marks) Solution:  The probability density function of the random variable is 0.2, for x ≤ 1  f (x) = 0.1, for 1 < x ≤ 4   otherwise  0,

d = 0.3414 and x = 0.2 Each lamination thickness therefore including steel and two-side varnish coating thicknesses = 0.3414 + 0.4 = 0.7414 mm. Variance of overall core is therefore given by 100 × (0.7414 − 0.4)2/100 = 0.1166 Therefore, none of the given answers matches the correct answer. 89. In the following table, x is a discrete random variable and p(x) is the probability density. The standard deviation of x is

5



If there are 100 laminations with mean thickness = 0.2 mm and variance = 0.1 mm, then we can write the following expressions:

Assuming all thicknesses to be equal to d and each side lamination varnish thickness to be equal to x, the two equations reduce to:

for 1 < x ≤ 4

P (0.5 < x < 5) =

Thickness of the stack of 100 such laminations = 0.4 × 100 = 40 mm

f (x) dx

0. 5 1

=



4

5

f (x) dx + ∫ f (x) dx + ∫ f (x) dx

0. 5

1

= 0.1 + 0.3 = 0.4

Chapter 5.indd 300

1

2

3

P(x)

0.3

0.6

0.1

4

= (0.2)(x)0.5 + (0.1)(x)1 + 0 1

x

4

(a) 0.18

(b) 0.36

(c) 0.54

(d) 0.6

(GATE 2014, 2 Marks)

8/22/2017 3:37:34 PM

SOLVED GATE PREVIOUS YEARS’ QUESTIONS     301

Solution:  From the given table, we have Mean (m) = ∑ xP (x) = 1 × 0.3 + 2 × 0.6 + 3 × 0.1 = 1.8

∑ x2P (x) = 1 × 0.3 + 4 × 0.6 + 9 × 0.1 = 3.6 2 Variance = ∑ x2P (x) −  ∑ xP (x) = 3.6 − (1.8)2

92. The number of accidents occurring in a plant in a month follows Poisson distribution with mean as 5.2. The probability of occurrence of less than 2 accidents in the plant during a randomly selected month is (a) 0.029

(b) 0.034

(c) 0.039

(d) 0.044

= 0.36 (GATE 2014, 2 Marks) Standard deviation =

Variance = 0.36 = 0.6 Ans. (d)

90. Consider an unbiased cubic dice with opposite faces coloured identically and each face coloured red, blue or green such that each colour appears only two times on the dice. If the dice is thrown thrice, the probability of obtaining red colour on top face of the dice at least twice is .

Solution:  For Poisson distribution, P(l) = e−l l n . Given that l = 5.2 and n < 2, so n! e−l l 0 e−1l 1 + = e−5.2 (1 + 5.2) 0! 1! = 0.00552 × 6.2 = 0.034

P (l ) =

(GATE 2014, 2 Marks)

Ans. (b)

Solution:  The probability of each colour appearing only two times on the dice is p = 2/6 = 1/3 and the probability of obtaining red colour at the top face is q = 1 − (1/3).

93. A traffic office imposes on an average 5 number of penalties daily on traffic violators. Assume that the number of penalties on different days is independent and follows a Poisson distribution. The probability that there will be less than 4 penalties in a day is .

Therefore, the required probability of obtaining red colour on top face of the dice at least twice is

(GATE 2014, 2 Marks) 1 p (x ≥ 2) = 3C2    3 

2

 2      + 3C3  1   3   3  1

3

 2    = 7  3  27 0

91. A machine produces 0, 1 or 2 defective pieces in a day with associated probability of 1/6, 2/3 and 1/6, respectively. The mean value and the variance of the number of defective pieces produced by the machine in a day, respectively, are (a) 1 and 1/3 (c) 1 and 4/3

(b) 1/3 and 1 (d) 1/3 and 4/3

Solution: e−l l x ,l = 5 x! P (x < 4) = P (x = 0) + P (x = 1) + P (x = 2) + P (x = 3) P (x ) =

e−5 51 e−5 52 e−5 53 e−5 50 + + + 0! 1! 2! 3!   25 125  = 0.265 = e−5 1 + 5 + +  2 6  =

(GATE 2014, 2 Marks) Solution:  From the question, we have x

0

1

2

P(x)

1/6

2/3

1/6

Ans. 0.265 94. An observer counts 240 vehicles/h at a specific highway location. Assume that the vehicle arrival at the location is Poission distributed, the probability of having one vehicle arriving over a 30-second tome interval is .

where x is the number of defective pieces. Mean (m) = ∑ xP (x) = 0 × (1/6) + 1 × (2/3) + 2 × (1/6) = 1

∑ x2P (x) = 0 × (1/6) + 1 × (2/3) + 4 × (1/6)

(GATE 2014, 2 Marks) Solution:  Average number of vehicles per hour, l = 240/h =

= 4/3 Variance = ∑ x2P (x) −  ∑ xP (x) = 4/3 − (1)2 = 1/3 Ans. (a) 2

Chapter 5.indd 301

P  (x = 1) =

240 / min = 4/ min = 2/30 s 60

e−2 ⋅ 2 = 0.27 1! Ans. 0.27

8/22/2017 3:37:39 PM

302     Probability and Statistics 

95. Consider the following two normal distributions

(

f1 (x) = exp −px f2 (x) =

2

)

 1 2  1 exp − x + 2x + 1   4p 2p 

(

)

If m and s denote the mean and standard deviation, respectively, then (a) m1 < m2 and s 12 < s 22 (b) m1 < m2 and s 12 > s 22

P(x) = corresponding probabilities Thus, the probability is highest for the outcome 6 `7,' i.e. . 36 Ans. 0.1667 97. Suppose A and B .are two independent events with probabilities P(A) ≠ 0 and P(B) ≠ 0. Let A and B be their complements. Which of the following statement is FALSE? (a) (b) (c) (d)

(c) m1 > m2 and s 12 < s 22 (d) m1 > m2 and s 12 > s 22

P (A ∩ B) = P (A)P (B) P(A/B) = P(A) P (A ∪ B) = P (A) + P (B) P (A ∩ B) = P (A) + P (B) (GATE 2015, 1 Mark)

(GATE 2014, 2 Marks)

Solution:  We know that A and B are independent, then

Solution:   We are given that f1 (x) = e−px

2

P (A ∩ B) = P (A) ⋅ P (B) 1  x− m  −   2  s 

2

Comparing with f (x) =

1

e

s 2p

m1 = 0 and s 1 =

Now, f2 (x) =

1 2p

1 − (x2 + 2x +1) 4 e p

, we get

1

A and B are independent then P (A/B) = P (A) and P (B/A) = P (B). Also, if A and B are independent then A and B are also independent, that is,

2p =

P (A ∩ B) = P (A) ⋅ P (B)

1 2p

1 2 − (x +1) p 4 e

1  x− m  −   2  s 

Thus, (a), (b) and (d) are correct. Hence, (c) is false. Ans. (c)

2

Comparing with f (x) =

1 s 2p

e

we get

m2 = −1 and s 2 = 2p

98. The variance of the random variable X with 1 −| x| probability density function f (x) = | x | e is 2 . (GATE 2015, 1 Mark)

So

Ans. (c) 96. In rolling of two fair dice, the outcome of an experiment is considered to be the sum of the numbers appearing on the dice. The probability is highest for the outcome of . (GATE 2014, 2 Marks) Solution:  We can form the table as follows: x

2

P(x)

1 2 3 4 5 6 5 4 3 2 1 36 36 36 36 36 36 36 36 36 36 36

3

4

5

6

7

8

9

1 −x xe is prob2 ability density function of random variable X. Solution:  Given that f (x) =

m1 > m2 and s 12 < s 22

V (x) = E(x2 ) − {E(x)}2 ∞



E(x) =



xf (x)dx =

−∞



−∞

1 x x e xdx = 0 2

nction is odd) ( the fun ∞

E(x)2 =





x2f (x)dx =

−∞ ∞

10 11 12 =



−∞

x2

1 −x x e dx 2

2 x3 e−x dx ( function is even) 3∫ 2

Where x is a random variable and denotes the sum of the numbers appearing on the dice.

Chapter 5.indd 302

= 3! = 6 Ans. 6

8/22/2017 3:37:54 PM

SOLVED GATE PREVIOUS YEARS’ QUESTIONS     303

If A starts the game, probability that A wins the

99. A random variable X has the probability density function f(x) as given below: a + bx for 0 < x < 1 f (x) =  otherwise  0

game = P (A) + P (A)P (B)P (A) + P (A)P (B)P (A)P (B)P (A) +  P (A) + P (A)P (B)P (A) + P (A)P (B)P (A)P (B)P (A) + 

If the expected value E[X] = 2/3, then Pr[X < 0.5] is .

=

1 551 55551 + + + 6 666 66666

  1  5 2  5 4 1 55 5555 1 + + +  = 1 +   +   +   6   6   6  6  66 6666        1 36 1  1 6 = × =  = 2 6 6 11 11 5   1 −     6    Ans. (d) =

(GATE 2015, 1 Mark) Solution:  We have a + bx for 0 < x < 1 f (x) =  otherwise  0 ∞



f (x)dx = 1

−∞

101. If P(X) = 1/4, P(Y) = 1/3, and P (X ∩ Y ) = 1/12, the value of P(Y/X) is 1 4 1 29 (a) (b) (c) (d) 4 25 3 50

1

∫ (a + bx)dx = 1

So,

0

a+

b = 1 ⇒ 2a + b = 2 2 

(i)

Given that,

(GATE 2015, 1 Mark) Solution:

1

E[X ] =

2 = 3

P (Y /X ) =

∫ x[a + bx]dx

P (X ∩ Y ) =

P (X )

1/12 1 = 1/4 3

0

Ans. (c) 2 a b = + ⇒ 3a + 2b = 4 3 2 3



(ii)

From Eqs. (i) and (ii), we get a = 0 and b = 2 Pr[X < 0.5] =

0.5

0.5



f (x)dx = 2 ∫ xdx = 0.25

0

0

102. Among the four normal distribution with probability density functions as shown below, which one has the lowest variance? IV III II

Ans. 0.25 I

100. Two players, A and B, alternately keep rolling a fair dice. The person to get a six first wins the game. Given that player A starts the game, the probability that A wins the game is (a) 5/11

(b) 1/2

(c) 7/13

(d) 6/11

-2

(a) I

-1

0

(b) II

1

(c) III

2

(d) IV

(GATE 2015, 1 Mark) (GATE 2015, 1 Mark) 6 1 = 36 6 1 That is, probability of A wins the game = 6 1 5 Probability of A not wins the game = 1 − = 6 6 1 Probability of B wins the game = 6 5 Probability of B not winning the game = 6 Solution:  Probability of getting 6 =

Chapter 5.indd 303

Solution:  The correct answer is option (d). Ans. (d) 103. Consider the following probability mass function (p.m.f) of a random variable X. q,  p(x, q) = 1 − q,  0,

if X = 0 if X = 1 otherwise

If q = 0.4, the variance of X is .­ (GATE 2015, 1 Mark)

8/22/2017 3:38:03 PM

304     Probability and Statistics 

Solution:  We are given, The value of the autocorrelation q,  p(x, q) = 1 − q,  0,

if X = 0 if X = 1 otherwise

  3T   3T  Ryy   ∆E  y(t)y t −  equals  4     4 

   3T  3T  Ryy   ∆E  y(t)y t −    4   4 

.

(GATE 2015, 2 Marks)

Given q = 0.4, hence Solution:  We are given, 0.4 , if X = 0  p (x, q ) = 0.6, if X = 1  0, otherwise X

0

1

p(X = x)

0.4

0.6

 t  Xn p(t − nT − f ) Ryy (z ) = 1 −   T   n =−∞  3T   3p /4  1  = = 0.25 Ryy   = 1 −  4   p  4 ∞

y(t) =





Ans. 0.25

106. The chance of a student passing an exam is 20%. The chance of a student passing the exam and getting above 90% marks in it is 5%. Given that a student passes the examination, the probability that the student gets above 90% marks is

Required value = V (X ) = E(X 2 ) − {E(X )}

2

E(X ) = ∑ xi pi = 0 × 0.4 + 1 × 0.6 = 0.6

1 18 2 (c) 9

E(X 2 ) = ∑ xi2 pi = 02 × 0.4 + 12 × 0.6 = 0.6

1 4 5 (d) 18

(a)

V (X ) = 0.6 − (0.6)2 = 0.6 − 0.36 = 0.24

(b)

Ans. 0.24 (GATE 2015, 2 Marks) 104. Let X and Y denote the sets containing 2 and 20 distinct objects, respectively, and F denote the set of all possible functions defined from X to Y. Let f be randomly chosen from F. The probability of f being chosen from F. The probability of f being one-to-one is . (GATE 2015, 2 Marks)

Solution:  Let A be the event that a student passes the exam and B be the event that student gets above 90% marks. We are given, P(A) = 0.2; P (A ∩ B ) = 0.05 Required probability is P (B /A) =

Solution:  X = 2, Y = 20 Number of functions from X to Y is 202, that is 400 and number of one-one functions from X to Y is 20 × 19 = 380. Therefore, probability of a function f  being one380 one is = 0.95. 400 Ans. 0.95

P (A ∩ B) 0.05 1 = = P (A) 0. 2 4 Ans. (b)

107. The probability of obtaining at least two `SIX’ in throwing a fair dice 4 times is (a) 425/432 (c) 13/144

(b) 19/144 (d) 125/432 (GATE 2015, 2 Marks)

105. A

random ∞

y(t) =



binary

wave

y(t)

is

given

by Solution:  From the given data, n = 4 and p = 1/6

Xn p(t − nT − f )

n =−∞

where p(t) = u(t) - u(t - T ), u(t) is the unit-step function and f is an independent random variable with uniform distribution in [0, T  ]. The sequence {Xn} consists of independent and identically distributed binary valued random variables with P{Xn = +1}= P{Xn = -1} = 0.5 for each n.

Chapter 5.indd 304

⇒ q = 1−

1 5 = 6 6

p(x ≥ 2) = 1 − p(x < 2) = 1 − [ p(x = 0) + p(x = 1)]   1 1  5 3   1 0  5 4 19 = 1 −  4C0     + 4C1      =  6   6   6   6    144 Ans. (b)

8/22/2017 3:38:12 PM

SOLVED GATE PREVIOUS YEARS’ QUESTIONS     305

108. For probability density function of a random variable, x is

Solution:  Probability density function of x is given as e−x , x ≥ 0 px (x) =  0, otherwise 

x (4 − x2 ) for 0 ≤ x ≤ 2 4 = 0 otherwise

f (x) =

The mean mx of the random variable is

.

(GATE 2015, 2 Marks)

and

gx (x) =

3x e4



Solution:  We are given E[ gx (x)] = x f (x) = (4 − x2 ), 0 ≤ x ≤ 2 4

∫ fx (x)gx (x)dx 0 ∞

Mean = m x = E(x)

=

−x

∫e

3x e 4 dx

0 2

=

2

∫ xf(x)dx = ∫ 0

=

∞ 3x −x e 4 dx





=



∫e

0

x 4 dx

0

−x / 4  ∞

x x   (4 − x2 )dx  4 

e  = −4(e−∞ − 1) = −4(0 − 1) = 4 =   − 1 / 4   0

0

2 2 1 1  4x3 x5  2 4 = ∫ (4x − x )dx =  −  5  4 4  3 0 0 1  8 32  32  1 1   −  =  4. −  = 4  3 5  4  3 5 

Ans. 4 111. A probability density function on the interval [a, 1] is given by 1/x2 and outside this interval the value of the function is zero. The value of a is  .

 2  16 = 8  = = 1.0667  15  15

(GATE 2016, 1 Mark) Ans. 1.0667

109. The probability that a thermistor randomly picked up from a production unit is defective is 0.1. The probability that out of 10 thermistors randomly picked up 3 are defective is (a) 0.001

(b) 0.057

(c) 0.107

(d) 0.3

(GATE 2015, 2 Marks)

Solution:  It is given that 1/x2 , for a ≤ x ≤ 1 f (x) =  otherwise  0, That is, ∞



−∞

Solution:  Let p be probability of a thermistor is defective = 0.1 q = 1 − p = 1 − 0.1 = 0.9 n = 10 Let X be the random variable which is number of defective pieces. Required probability = P (x = 3) =

10

C3 p3q 7 =

C3 (0.1)3 (0.9)7 = 0.057

1

f (x)dx = 1 ⇒

∫x

1 2

dx = 1

a

 1 ⇒ −  = 1  x a 1 ⇒ −1 = 1 a 1 ⇒ a = = 0. 5 2 1

Ans. 0.5

10

110. The probability-density function of a random variable x is px (x) = e-x for x ≥ 0 and 0 otherwise. The expected value of the function gx(x) = e3x/4 is .

112. Suppose that a shop has an equal number of LED bulbs of two different types. The probability of an LED bulb lasting more than 100 hours given that it is of Type 1 is 0.7, and given that it is of Type 2 is 0.4. The probability that an LED bulb chosen uniformly at random lasts more than 100 hours is .

(GATE 2015, 2 Marks)

(GATE 2016, 1 Mark)



Chapter 5.indd 305

Ans. (b)

8/22/2017 3:38:17 PM

306     Probability and Statistics 

Solution: (i) Type 1 LED bulb: The probability that the bulb is of Type 1 and it lasts more than 100 hours is

The probability of getting `head’ for the first time in the fifth toss: P(Tail)⋅P(Tail)⋅P(Tail)⋅P(Tail)⋅P(Head) = (0.7)4 (0.3) = 0.07203

1 × 0.7 2

Ans. 0.07203

(ii) Type 2 LED bulb: The probability that the bulb is of Type 2 and it lasts for more than 100 hours is

115. Consider a Poisson distribution for the tossing of a biased coin. The mean for this distribution is m. The standard deviation for this distribution is given by (a)

1 × 0.4 2

µ   (b) m2  (c) m  (d) 1/m (GATE 2016, 1 Mark)

Therefore, the probability that an LED bulb chosen uniformly at random and lasts more than 100 hours is

Solution:  For Poisson distribution,

σ2 = µ ⇒σ =

1  1   × 0.7 +  × 0.4 = 0.55 2 2

Ans. (a)

Ans. 0.55 113. The second moment of a Poisson-distributed random variable is 2. The mean of the random variable is . (GATE 2016, 1 Mark) Solution: 

µ

For this Poisson distribution,

116. The area (in percentage) under standard normal distribution curve of random variable Z within limits from −3 to +3 is     . (GATE 2016, 1 Mark) Solution:  A standard normal curve (as shown in  figure) has 68% are in limits −1 to +1, 95% area is limits −2 to +2 and 99.7% area in limits −3 to +3.

Mean = Variance = 2 However, we know that variance is

σ2 = m2 − m21

(i)

where m2 is the second central moment and m1 is the mean. From Eq. (i), we get

λ = 2 − λ2

−3

−2

−1

0

+0

+2

+3

λ2 + λ − 2 = 0 ⇒ (λ + 2) (λ − 1) = 0 Therefore,

99.7% of the data are within 3 standard deviations of the mean

λ = −2 or λ = 1 Since variance (λ) is positive, λ = 1.

Ans. 99.7 Ans. 1 117. Type II error in hypothesis testing is 114. The probability of getting a `Head’ in a single toss of a biased coin is 0.3. The coin is tossed repeatedly till a `Head’ is obtained. If the tosses are independent, then the probability of getting `Head’ for the first time in the fifth toss is . 

(GATE 2016, 1 Mark) Solution: 

We have

P(Head) = 0.3 = p and P(Tail) = 0.7 = q

Chapter 5.indd 306

(a) a cceptance of the null hypothesis when it is false and should be rejected (b) rejection of the null hypothesis when it is true and should be accepted (c) rejection of the null hypothesis when it is false and should be rejected (d) acceptance of the null hypothesis when it is true and should be accepted (GATE 2016, 1 Mark)

8/22/2017 3:38:20 PM

SOLVED GATE PREVIOUS YEARS’ QUESTIONS     307

Solution:  In hypothesis testing, type II error has acceptance of null hypothesis when it is false and should be rejected. Ans. (a)

1st time HEADS HEADS TAILS TAILS 2nd time HEADS TAILS HEADS TAILS

118. X and Y are two random independent events. It is known that P(X) = 0.40 and P (X ∪ Y C ) = 0.7. Which one of the following is the value of P (X ∪ Y )?

N

Ans. 0.33 120. Two random variables x and y are distributed according to

(a) 0.7  (b) 0.5  (c) 0.4  (d) 0.3 (GATE 2016, 1 Mark)

(x + y), 0 ≤ x ≤ 1 0 ≤ y ≤ 1 fx,y (x, y) =  otherwise  0,

Solution:  P(X) = 0.4 and P (X ∪ Y C ) = 0.7 P (X ) + P (Y C ) − P (X ∩ Y C ) = 0.7

The probability P(X + Y ≤ 1) is

P(X) + P(Y    ) - P(X)P  (Y    ) = 0.7



{

C

}

C

0.4 + P(Y    C) - 0.4 P  (Y    C) = 0.7

Solution: 

We have 1

P (x + y ≤ 1) =

0. 7 − 0. 4 P (Y C ) = 0.6



∫ ∫

1 1− x

∫ ∫

x=0 0

Now,

fxy (x, y) dx dy 1− x

 y2  (x + y)dx dy = ∫  xy +  dx 2 0 x=0  1

1  (1 − x)2  = ∫  x(1 − x) + dx 2  0 

P(Y    C) = 1 - P(Y    ) Therefore, P(Y    ) = 1 - 0.5 = 0.5  P (X ∪ Y ) = P (X ) + P (Y ) − P (X ∩ Y )

=

= 0.4 + 0.5 - 0.4 × 0.5 = 0.7

x x3 − 2 6

1

= 0.33 0

Ans. 0.33 Ans. (a)

119. Consider the following experiment. Step 1. Flip a fair coin twice. Step 2. If the outcomes are (TAILS, HEADS) then output Y and stop. Step 3. If the outcomes are either (HEADS, HEADS) or (HEADS, TAILS), then output N and stop. Step 4. If the outcomes are (TAILS, TAILS), then go to Step 1. The probability that the output of the experiment is Y is (up to two decimal places) . (GATE 2016, 2 Marks)

Solution:  The probability, that the output of the given experiment is Y, can be written as follows:  1  1 1  1 1 1 P (Y ) =   +  ×  +  × ×   4  4 4  4 4 4 1/4 1 = = = 0.33 1 − (1/4) 3

Chapter 5.indd 307

1

x=0 y =0

P(Y    C) = 0.5



.

(GATE 2016, 2 Marks)

0.4 + 0.6 P  (Y    C) = 0.7



Y

121. Candidates were asked to come to an interview with 3 pens each. Black, blue, green and red were the permitted pen colours that the candidate could bring. The probability that a candidate comes with all 3 pens having the same colour is ________. 

(GATE 2016, 2 Marks)

Solution:  Let Black = a Blue = b

Green = c



Red = d

For all three pens to be of the same colour, four cases are possible:

aaa



bbb



ccc



ddd

Let first pen be a. Next, choose 1 from 4 = 4C1

8/22/2017 3:38:24 PM

308     Probability and Statistics 

Next, choose 1 from 4 = 4C1 Similarly

a guarantee of replacement if one or more screws in the packet are found to be defective. The probability that a packet would have to be replaced is     .

       b ⋅ 4C1 ⋅ 4C1 = 16 cases



= a ⋅ 4C1 ⋅ 4C1 = 16 cases



(GATE 2016, 2 Marks)

       c ⋅ 4C1 ⋅ 4C1 = 16 cases        d ⋅ 4C1 ⋅ 4C1 = 16 cases

Solution:  P(screw is defective) = 0.1

4 1 Total 64 cases. Therefore, probability = = 64 6 or 0.167

P (that a packet would have to be replaced) = P (one screw is found defective) + P (two screws defective) +  + P(five screws are defective)



Ans. 0.167

122. Let the probability density function of a random variable, X, be given as 3 fX (x) = e−3x u(x) + ae4x u(−x) 2 where u(x) is the unit step function. Then the value of `a’ and Prob{X ≤ 0}, respectively, are 1 (a) 2, 2 1 (c) 2, 4

1 2 1 (d) 4, 4

= 1 − P (no screw is defective) = 1 - 0.9 × 0.9 × 0.9 × 0.9 × 0.9 = 0.4095 Ans. 0.4095

124. Three cards were drawn from a pack of 52 cards. The probability that they are a king, a queen, and a jack is

(b) 4,







(a) 

(GATE 2016, 2 Marks)

Solution:  For the given function:  ae4x , x 0 is the parameter of distribution. Exponential = 1

Chapter 5.indd 310

1 16 Required probability = = 0.5 1 8 Ans. (0.5) 136. If a random variable X has a Poisson distribution with mean 5, then the expectation E[(X + 2)2] equals ______. (GATE 2017, 2 Marks) Solution:

Therefore, the total parameters are Gaussian = 2

1 1 1 1 × × × 2 2 2 2 1 = 16 1 1 1 Probability of getting (H, H, H) = × × 2 2 2 1 = 8

Probability of getting (H, H, H, H) =

I n Poisson distribution, if X is a random variable, then Ans. (b)

E(X) = Mean = l = 5

8/22/2017 3:38:44 PM

SOLVED GATE PREVIOUS YEARS’ QUESTIONS     311

and

7 8 2 7 7 7 Var{Y } = −   = 8 8 64 E(Y 2 ) =

E(X) = V(X) = 5 Now, E(X2) = V(X) + (E(X))2 = 5 + 52 = 30

Ans. (c)

Therefore,

138. For the function f(x) = a + bx, 0 ≤ x ≤ 1, to be valid probability density function, which one of the following statements is correct?

E[(X + 2)2] = E (X2 + 4X + 4)  = E (X2) + 4 E (X) + 4                                            = 30 + 4 × 5 + 4 = 54

Ans. (54.0)

137. A person decides to toss a fair coin repeatedly until he gets a head. He will make at most 3 tosses. Let the random variable Y denote the number of heads. The value of var(Y ), where var {•} denotes the variance, equals

7 49 7 105 (a)      (b)       (c)       (d) 8 64 64 64

(a) a = 1, b = 4 (c) a = 0, b = 1

(GATE 2017, 2 Marks) Solution: For probability density function, f(x), to be valid, we have a

∫a

f( x ) = 1



(a + bx) = 1





(GATE 2017, 2 Marks) ⇒

Solution:

(b) a = 0.5, b = 1 (d) a = 1, b = −1

a

−a 1



(a + bx)dx = 1

0

 bx2  ⇒ ax + =1 2  0  b ⇒a+ =1 2 1

No. of Head (Y ) P(Y)

E(Y ) = 0 ×

Chapter 5.indd 311

0

1

1  3 2 

7 23

1 7 7 + 1× = 8 8 8

For equation to be satisfied, a = 0.5, b = 1. Ans. (b)

8/22/2017 3:38:48 PM

Chapter 5.indd 312

8/22/2017 3:38:48 PM

CHAPTER 6

NUMERICAL METHODS

INTRODUCTION The limitations of analytical methods have led the engineers and scientists to evolve graphical and numerical methods. However, numerical methods can be derived which are more accurate. These have recently become highly important due to increasing demand for numerical answers to various problems. In this chapter, we shall discuss some numerical methods for the solution of linear and algebraic and transcendental equations. The chapter also covers numerical integration and numerical solutions for ordinary differential equations.

are very tedious and lengthy. Therefore, the scope of a mistake increases while solving the problem. Hence, there exist other numerical methods of solution which are well-suited for computing machines. This section explains the various numerical methods of solution.

Gauss Elimination Method The Gauss elimination method is a basic method of obtaining solutions. In this method, the unknowns are eliminated successively and the system is reduced to an upper triangular system from which the unknowns are found by back substitution. Now, consider the following equations:

NUMERICAL SOLUTION OF SYSTEM OF LINEAR EQUATIONS Linear equations occur in various mathematical problems and though the system of equations can be solved by Cramer’s rule or by Matrix method, these methods

Chapter 6.indd 313

a1x + b1y + c1z = d1

(1)



a2 x + b2 y + c2z = d2

(2)



a3 x + b3 y + c3z = d3

(3)



Step 1: Eliminate x from Eqs. (2) and (3)

8/22/2017 4:27:35 PM

314     Numerical Methods 

Assuming a1 ≠ 0, we eliminate x from Eq. (2) by subtracting (a2 /a1 ) times Eq. (1) from Eq. (2). Similarly, we eliminate x from Eq. (3) by subtracting (a3 /a1 ) times Eq. (1) from Eq. (3). Hence, the new set of equations is given by a1x + b1y + c1z = d1

(4)



b2′ y + c2′ z = d2′

(5)



b3′ y + c3′ z = d3′ (6) Here, Eq. (4) is called the pivotal equation and a1 is called the first pivot.



 u11 u12 u13   l11 0 0     L = l21 l22 0 and U =  0 u22 u23      0 u 33   0 l31 l32 l33  Since AX = B and A = LU,



(7)

b2′ y + c2′ z = d2′

(8)

c3′′z = d3′′

(9)

Now, Eq. (8) is called the pivotal equation and b2′ is called the new pivot. Step 3: Evaluate the unknowns

LUX = B

(12)

A = LU reduces to  l11 0 0  u11 u12 u13   a11 a12     l21 l22 0  0 u22 u23  = a21 a22     0 u 33  a31 a32 l31 l32 l33   0

Assuming b2 ≠ 0, we eliminate y from Eq. (3), by subtracting (b3′ /b2′ ) times Eq. (2) from Eq. (3). Hence, the new set of equations is given by a1x + b1y + c1z = d1

(11)

where L is the lower triangular and U is the upper triangular. For example, say

Step 2: Eliminate y from Eq. (3)



A = LU

 l11u11 l11u12  l21u11 l21u12 + l22 u22  l31u11 l31u12 + l32 u22  a11 a12  = a21 a22  a31 a32

l31u13

a13   a23   a33 

l11u13   l21u13 + l22 u23   + l32 u23 + l33 u33 

a13   a23   a33 

 y1    Putting UX = Y where Y =  y2  , we get    y3 

The values of x, y and z are found from Eqs. (7), (8) and (9) by back substitution.



LY = B

(13)

The equivalent system is

Matrix Decomposition Methods (LU Decomposition Method)



l11y1 = b1

(14)



l21y1 + l22 y2 = b2

(15) (16)



l31y1 + l32 y2 + l33 y3 = b3

In this section, we will discuss some more numeric methods for solving linear systems of n equations in n unknowns x1, …, xn,

AX = B

(10)

where  a11 a12  A = [ajk ] = a21 a22  a31 a32

a13   a23   a33 

is the n × n coefficient matrix, X =  x1 … xn  and B = b1 … bn  . We discuss two related methods, namely Doolittle’s and Crout’s that are modifications of the Gauss elimination, which require fewer arithmetic operations. They use the idea of the LU-factorization of A. This method is based on the fact that every matrix can be expressed as the product of a lower and an upper triangular matrix, provided all the principal minors are non-singular. An LU-factorization of a given square matrix A is of the form

Chapter 6.indd 314

This can be solved by forward substitution for  y1    y1 , y2 and y3 . Now, using UX = Y and Y =  y2  , we get    y3 

u11x1 + u12 x2 + l13 x3 = y1

(17)

u22 x2 + u23 x3 = y2

(18)

u33 x3 = y3

(19)



This can be solved by backward substitution for x1 , x2 and x3 .

Doolittle’s Method Now, we can conclude that L and U in Eq. (11) can be computed directly, without solving simultaneous equations (thus, without using the Gauss elimination method). Once we have Eq. (11), we can use it for solving AX = B in two steps, simply by noting that AX = LUX = B may be written as

8/22/2017 4:27:45 PM

NUMERICAL SOLUTION OF SYSTEM OF LINEAR EQUATIONS      315

LY = B (20) If we assume | a1 | ≥ | b1 | + | c1 |; | b2 | ≥ | a2 | + | c2 |; | c3 | ≥ | a3 | + | b3 | | aUX | ≥ | b | + | c | ; | b | ≥ | a | + | c | ; | c3 | ≥ | a3 | + | b3 | to be true, then this iterative method = Y (21) 1 1 1 2 2 2 can be used for the above system to calculate x, y and z. and solving first Eq. (20) for Y and then Eq. (21) for X. Step 2: Suppose Here we require that L have main diagonal 1, …, 1 then this is called Doolittle’s method. Both systems in 1 x = (d1 − b1y − c1z) (22) Eqs. (20) and (21) are triangular, so we can solve them a1 as in the back substitution for the Gauss elimination. 1 y = (d2 − a2 x) (23) b1 Crout’s Reduction 1 z = (d3 − a3 x − b3 y) (24) c3 A similar method is obtained from Eq. (11) if U (instead

of L) is required to have main diagonal 1, …, 1, and this method is called Crout’s method. The rest of the steps for obtaining the solution are similar to those in Section 6.2.2.

Gauss–Jordan Method Gauss —Jordan method is a modification of the Gauss elimination method. In this method, the elimination of the unknowns is performed not only in the equations below the primary diagonal, but also in the equations above it. Hence, the system is ultimately reduced to a diagonal matrix form; that is, each equation involving only one unknown. From these equations, x, y and z are obtained easily.

Step 3: If x(0) , y(0) , z(0) are the initial values of x, y, z, respectively, then (25)

y(1)

(26)

z(1)

1 (d1 − b1y(0) − c1z(0) ) a1 1 = (d2 − a2 x(0) − c2z(0) ) b2 1 = (d3 − a3 x(0) − b3 y(0) ) c3

x(1) =

Step 4: Similarly, we can conclude that

Iterative Methods of Solution

(28)

y(r +1)

(29)

z(r +1)

1 (d − b1y(r) − c1z(r) ) a1 1 1 = (d2 − a2 x(r) − c2z(r) ) b2 1 = (d3 − a3 x(r) − b3 y(r) ) c3

x(r +1) =

The methods described till now are known as direct methods as they give exact solutions. In this section, we discuss the iterative methods, in which we start from an approximation to the true solution and obtain better approximations from a computation cycle repeated until we obtain the desired accuracy.

(27)

(30)

This process is continued till the convergence is obtained.

Gauss–Jacobi Method

Gauss–Seidel Method

An iteration method is called a method of simultaneous corrections if no component of an approximation x(m) is used until all the components of x(m) have been computed. A method of this type is the Jacobi iteration which involves not using improved values until a step has been completed and then replacing x(m) by x(m +1) at once, directly before the beginning of the next step.

The Gauss— Seidel iteration is a modification of the Gauss—Jacobi iteration method. It is a method of successive corrections because for each component we successively replace an approximation of a component by a corresponding new approximation as soon as the latter has been computed. It can also be shown that the Gauss—Seidel method converges twice as fast as the Jacobi method.

The Jacobi iteration method can be explained by the following steps:

The Gauss—Seidel iteration method can be explained by the following steps:

Step 1: Consider the system of equation as a1x + b1y + c1z = d1   a2 x + b2 y + c2z = d2  → 0  a3 x + b3 y + c3z = d3 

Chapter 6.indd 315

Step 1: Consider the system of equation as



a1x + b1y + c1z = d1   a2 x + b2 y + c2z = d2  → 0  a3 x + b3 y + c3z = d3 

8/22/2017 4:27:51 PM

316     Numerical Methods 

If we assume | a1 | ≥ | b1 | + | c1 |; | b2 | ≥ | a2 | + | c2 |; | c3 There | ≥ | a3 |are + | some b3 | numerical methods for the solutions of b2 | ≥ | a2 | + | c2 |; | c3 | ≥ | a3 | + | b3 | to be true, then the iterative method equations of the above form, where f(x) is algebraic or transcendental or combinations of both. These methods can be used for the above system to find the values of are discussed in this section. x, y and z. Step 2: Suppose

Bisection Method

1 (d − b1y − c1z) a1 1 1 y = (d2 − a2 x − c2z) b2 1 z = (d3 − a3 x − b3 y) c3 x=



(31) (32) (33)

Step 3: We start with the initial values y(0) , z(0) for y and z, respectively, and get x(1) from the first equation: x(1) =

1 (d − b1y(0) − c1z(0) ) a1 1

(34)

Step 4: For the second equation, substitute z(0) for z and x(1) for x instead of x(0) .

This is one of the simplest methods and convergence is guaranteed but slow; that is, the solution of f(x) can always be obtained. This method is also known as the method of successive bisection. This method is based on the principle that if f(x) is continuous between x0 and x1 , and f (x0 ), f (x1 ) are of opposite signs, then there exists at least one root between x = x0 and x = x1. We begin the iterative cycle by choosing two trial  points x0 and x1 , which enclose the actual root. Then  f (x0 ) and f (x1 ) are of opposite signs. The interval  (x0 , x1 ) is bisected and its midpoint x2 is obtained as x2 = (x1 + x0 ) / 2

y

(1)



1 = (d2 − a2 x(1) − c2z(0) ) b2

(35)

Step 5: Now substitute x(1) for x and y(1) for y in the third equation. z(1) =

1 (d − a3 x(1) − b3 y(1) ) c3 3

(36)

Step 6: In finding the values of the unknowns, we use the latest available values on R.H.S. If x(r) , y(r) , z(r) are the rth iterates, then the iteration scheme will be



y

(r +1)

z

1 (d − b1y(r) − c1z(r) ) a1 1

(37)

1 = (d2 − a2 x(r +1) − c2z(r) ) b2

(38)

1 = (d3 − a3 x(r +1) − b3 y(r +1) ) c3

(39)

x(r +1) =

(r +1)

(40)

If f (x2 ) = 0 , then x2 itself is the root. If not, the root lies either between x0 and x2 or between x1 and x2 . That is, if f (x0 ) and f (x1 ) are of opposite signs, then root lies between x0 and x2 ; otherwise if f (x1) and f(x2) are of opposite signs, the root lies between x1 and x2 . The new interval for searching the root is therefore (x0 , x1 ) in the first case and (x1 , x2 ) in the second case, which is much less compared to the first interval (x0 , x1 ). This process is illustrated in Fig. 1. This new interval will be the beginning interval for the next iteration, and the processes of bisection and finding another such x2 are repeated. y or f(x) f(x)

Actual root

x0

This process is continued till the convergence is assumed. x′

NUMERICAL SOLUTION OF ALGEBRAIC AND TRANSCENDENTAL EQUATIONS In scientific and engineering work, a frequently occurring problem is to find the roots of equations of the form f (x) = 0

Chapter 6.indd 316

x =(x0+x1) x1 2

x

Third iteration Second iteration Starting interval Figure 1 |   Illustration of bisection method.

8/22/2017 4:28:06 PM

NUMERICAL SOLUTION OF ALGEBRAIC AND TRANSCENDENTAL EQUATIONS      317

Regula–Falsi Method (Method of False Position Method)

Newton–Raphson Method

The Regula—Falsi method is better than the bisection method in the sense that it guarantees convergence at a faster rate (that is, solution can be obtained in less number of iterations, all other parameters being same). As before, the iterative procedure is started by choosing two values x0 and x1 such that f (x0 ) and f (x1 ) are of opposite signs. Then the two points [x0 , f (x0 )] and [x1 , f (x1 )] are joined by a straight line. The intersection of this line with the x-axis gives x2. If f (x2 ) and f (x0 ) are of opposite signs, then replace x1 by x2; otherwise, replace x0 by x2. This yields a new set of values for x0 and x1 . The present range is much smaller than the range or interval between the first chosen set of x0 and x1. The convergence is thus established, and the iterations are carried over with the new set of x0 and x1. Another x2 is found by the intersection of the straight line joining the new f (x0 ) and f (x1 ) points with x-axis. Each new or successive interval is smaller than the previous interval, and it is guaranteed to converge to the root.

Newton’s method, also known as Newton—Raphson method, is another iteration method for solving ­equations f (x) = 0 , where f is assumed to have a continuous derivative f ′. The underlying idea is that we approximate the graph of f by suitable tangents. The Newton—Raphson method has second-order convergence. Using an approximate value x0 obtained from the graph of f, we let x1 be the point of intersection of the x-axis and the tangent to the curve of f at x0 (Fig. 3). Then tan b = f ′(x0 ) = Hence, x1 = x0 −

f (x0 ) x0 − x1

(42)

f (x0 ) f ′(x0 )

(43)

In the second step, we compute x2 =

x1 − f (x1 ) f ′(x1 )

in the third step x3 from x2 again by the same formula, and so on.

The procedure is illustrated in Fig. 2. y

y A (x0, f(x0))

y =f(x)

f(x0) x3 0

x0

x2 x1

x

P(x)

b B (x1, f(x1))

x

Figure 2 |   Illustration of Regula—Falsi method.

From Fig. 2, it can be seen (from the equation of the straight line) that y − f (x0 ) = where x2 = x0 −

f (x1 ) − f (x0 ) (x − x0 ) x1 − x0

x0 − x1 f (x0 ) f (x1 ) − f (x0 )

which is an approximation to the root.

Chapter 6.indd 317

x2

x1

x0

Figure 3 |   Newton—Raphson method.

Secant Method (41) Another method for finding the roots using a succession of roots of secant lines to better approximate a root of a function f is called secant method. The secant method can be thought of as a finite difference approximation of Newton—Raphson method.

8/22/2017 4:28:14 PM

318     Numerical Methods 

NUMERICAL INTEGRATION The process of evaluating a definite integral from a set of tabulated values of the integrand f(x) is called numerical differentiation. The problem of numerical integration is solved by representing f(x) by an interpolation formula and then integrating it between the given limits. x3

x0

x2

Numerical integration means the numeric evaluation of integrals

x1

b

J=

∫ f(x)dx

(45)

a



where a and b are given and f is a function given analytically by a formula or empirically by a table of values. Geometrically, J is the area under the curve of f between a and b (Fig. 5).

f(x)

We know that if f is such that we can find a differentiable function F whose derivative is f, then we can evaluate J by applying the familiar formula,

Figure 4 |   Secant method.

Consider Fig. 4. The slope of line from points (x0, f(x0)) to (x1, f(x1)) can be given by the following equation. f (x1 ) − f (x0 ) y=

x1 − x0

b

J=

(x − x1 ) + f (x1 )

∫ f(x)dx = F (b) − F (a) [where F ′(x) = f(x)]

(46)

a

y

We now calculate the solution of x by putting y = 0. x = x1 − f (x1 )

x1 − x0 f (x1 ) − f (x0 )

y =f(x)

We use the new value of x as x2 and repeat the same process using x1 and x2 instead of x0 and x1. This process is continued in the same manner unless we obtain xn = xn −1 . x2 = x1 − f (x1 )

x1 − x0 f (x1 ) − f (x0 )

x3 = x2 − f (x2 )

x2 − x1 f (x2 ) − f (x1 )



xn −1 − xn −2 f (xn −1 ) − f (xn −2 )

(44)

Jacobian If u and v are functions of two independent variables x ∂u / ∂x ∂u / ∂y and y, then the determinant is called ∂v / ∂x ∂v / ∂y the Jacobian of u, v with respect to x, y and is written as  u, v  ∂ (u, v ) . or J   x, y  ∂ (x, y )

Chapter 6.indd 318

x b

Figure 5 |   Geometric interpretation of a definite integral.

Hence, we obtain the generalized solution for the secant method as xn = xn −1 − f (xn −1 )

a

0

Newton–Cotes Formulas (General Quadrature) Let y0 , y1 , y2 ,… , yn be the values of y = f (x) corresponding to x = x0 , x1 , x2 ,..., xn, which are equally spaced with interval as h. Then by Newton’s forward interpolation formula, y = y(x0 + uh) = y0 + u∆y0 + +

u(u − 1) 2 ∆ y0 2!

u(u − 1)(u − 2) 3 ∆ y0 +  3!

(47)

8/22/2017 4:28:19 PM

NUMERICAL INTEGRATION     319

where u = (x − x0 )/h. Now,

y

x0 + nh

xn

∫ ydx = ∫ x0

y =f(x)

ydx

x0 h

=



∫ y0 + 1 ! ∆y0 + 0

u

 u(u − 1) 2 ∆ y0 +  hdu  2!

On replacing y by the interpolating polynomial in Eq. (47) and putting dx = hdu, we get

x a x* 1

xn

∫ x0 n

ydx = h ∫

n    u3 − 3u2 + 2u  u2 − u 2  ∆3 y0 + ⋅ ⋅ ⋅ du ydx = h ∫  y + u∆y0 + ∆ y0 +     2 6  0 

x2*

xn* b

Figure 6 |   Rectangular rule.

Trapezoidal Rule

2 2    3   y + u∆y + u − u ∆2 y +  u − 3u + 2u  ∆3 y + ⋅ ⋅ ⋅ du  0 0 0     2 6   

The trapezoidal rule is generally more accurate. We obtain it if we take the same subdivision as before and approximate f by a broken line of segments (chords) with n 2 3 2 4    endpoints [a, f (a)], [x1, f (x1)], …, [b, f (b)] on the curve 1 u 1 u u u  2 3 3   y0 + 7). = h  y0 u + ∆y0 +  −  ∆ y0 +  − u +ofu∆f (Fig. ⋅ ⋅ ⋅ Then the area under the curve of f between 2 2  3 2  6  4   0 a and b is approximated by n trapezoids of areas n  1  u3 u2  1  u4 u2 1 1 y0 u + ∆y0 +  −  ∆2 y0 +  − u3 + u∆3 y0 + ⋅ ⋅ ⋅ f (a) + f (x1 ) h, f (x1 ) + f (x2 ) h, … ,    2 2  3 2  6  4 2 2  0 1 f (x ) + f (b) h 2  n − 1 2 3 2 4     1 n 1 n n n   = h  ny0 + ∆y0 +  −  ∆3 y0 +  − n3 − n2  ∆3 y0 +    By taking their sum, we obtain the trapezoidal rule    2 2 3 2 6 4      b 2 3 2 4  1    1 n 1 n n n   f (x)dx = b  f (a) + f (x1 ) + f (x2 ) +  y0 + ∆y0 +  −  ∆3 y0 +  − n3 − n2  ∆3 y0 +   (48) ∫   2 2 2  3 2  6  4 (50)  a  1 +f (xn −1 ) + f (b)  2 0

Rectangular Rule

Numerical integration methods are obtained by approximating the integrand f by functions that can easily be integrated. The simplest formula, the rectangular rule, is obtained if we subdivide the interval of integration a ≤ x ≤ b into n subintervals of equal length h = (b − a)/n and in each subinterval approximate f by the constant f(xj*), which is the value of f at the midpoint (xj*) of the jth subinterval (Fig. 6). Then f is approximated by a step function. The n rectangles in Fig. 6 have the areas f(x1*)h, …, f(xn*)h, and the rectangular rule is

where h = (b − a)/n , as in Eq. (49). The xj ’s and a and b are called nodes. y y =f(x)

b

J=

∫ f(x)dx ≈ h f(x1 *) + f(x2 *) +  + f(xn *) a

  h = b − a   n      

Chapter 6.indd 319

(49)

x a

x1

x2

xn−1

b

Figure 7 |   Trapezoidal rule.

8/22/2017 4:28:25 PM

320     Numerical Methods 

Now, let us derive the trapezoidal rule. By putting n = 1 in Eq. (48), we get x0 + h

x1



f (x) =



x0



x0

x0 + 2h

xn



  1 ydx = h  y0 + ∆y0    2

ydx =

x0

x0

So

(51)

=

x0 + nh

xn

∫ f(x)dx = ∫ x0

ydx

x0

x1

=

x2

x1

ydx 

(52)

∫ f(x)dx = 2 [(y0 + y1 ) + (y1 + y2 ) + (y2 + y3 ) h

+  + (yn −1 + yn )] h [(y + yn ) + 2(y1 + y2 +  + yn −1 )] 2 0

Simpson’s Rule

Simpson’s Three-Eighth (3/8) Rule

 9 1 9 ydx = h 3y0 + ∆y0 +   ∆2 y0  2 2  2  x0   1  81 +  − 27 + 9 ∆3 y0     6 4  9 9 = h 3y0 + (y1 − y0 ) + (y2 − 2y1 + y0 )  2 4  3 + (y3 − 3y2 + 3y1 − y0 )  8 3h = (y + 3y2 + 3y1 + y0 ) (56) 8 3 Therefore, x0 + 3h

xn

Simpson’s One-Third (1/3) Rule



To apply this rule, the number of intervals should be even. That is, the no of ordinates must be odd. The error in Simpson’s one third rule is of the order h4 .

  4 1  8 4 ydx = h 2y0 + ∆y0 +  −  ∆3 y0 +    2 2  3 2 

ydx =

x0

∫ x0

x0 + 6h

ydx +

∫ x3

xn

ydx +  +



ydx

xn −3

3h = [(y0 + 3y1 + 3y2 + y3 ) + (y3 + 3y4 8 + 3y5 + y6 ) +  + (yn −3 + 3yn −2 + 3yn −1 + yn )]

Putting n = 2 in Eq. (48), we get

x0

(55)



Equation (53) is called the trapezoidal rule.



4 [(sum of first and last ordinates) 3 + 2(sum of remaining odd ordinates) + 4(sum of even ordinates)]

x0 + 3h

= h/2[(sum of first and last ordinates) + 2(sum of the remaining ordinates)] (53)

x0 + 2h

xn −2

This rule is applicable if n (the number of intervals) is a multiple of 3; that is, y(x) is a polynomial of degree three. Putting n = 3 in Eq. (48), we get

xn

=

ydx

xn −1

Here xi − xi−1 = h, i = 0, 1, 2,..., n . So we get

x0

x2



xn

∫ ydx + ∫ ydx +  + ∫ x0



xn

ydx +  +

4 4 = (y2 + 4y1 + y0 ) + (y2 + 4y3 + y4 ) +  3 3 4 + (yn −2 + 4yn −1 + yn ) 3 4 = [(y0 + yn ) + 2(y2 + y4 + ) + 4(y1 + y3 + )] 3

(since higher order derivatives do not exist)   h 1 = h  y0 + (y1 − y0 ) − [y0 + y1 ]   2 2



x0 + 4h

ydx +

=

3h [(y0 + yn ) + 3(y1 + y2 + y4 + y5 8  +  + yn −1 ) + 2(y3 + y6 +  + yn −2 )]

(57)

(higher order derivatives do not exist)   1 = h 2y0 + 2(y1 − y0 ) + (y2 − 2y1 + y0 )   3  1  1 2 = h 2y0 + 2y1 − 2y0 + y2 − y1 + y0   3  3 3 h = (y2 + 4y1 + y0 )  3 Therefore,

Chapter 6.indd 320

NUMERICAL SOLUTION OF ORDINARY DIFFERENTIAL EQUATION (O.D.E.)

(54)

Analytical methods of solutions are applicable only to a few cases and hence are used very rarely. The differential equations which appear in the physical problems frequently do not belong to any of these familiar types and hence one has to resort to numerical methods.

8/22/2017 4:28:32 PM

NUMERICAL SOLUTION OF ORDINARY DIFFERENTIAL EQUATION (O.D.E.)      321

A number of numerical methods are available for the solution of first-order differential equations of the form

Proceeding in this way, we get y(3), y(4), …, y(n − 1) and y(n), where

dy = f (x, y), given that y(x0 ) = y0 dx

y(n) = y0 + ∫ f (x, y(n −1) )dx

x

(61)

x0

These methods yield solutions either as a power series in x from which the values of y can be found by direct substitution or as a set of values of x and y. Methods such as Picard and Taylor series belong to the former class and methods such as Euler, Runge—Kutta, Milne, etc., belong to the latter one.

with n = 1, 2, 3, etc., and when x = x0, we have y = y0 as the initial condition, or y(0) = y0 becomes the initial or starting value for this iterative method. We thus proceed from y(0) to y(n) till two successive approximations are sufficiently close. This method is not of much practical use.

This section contains the most popular used methods, namely Picard’s, Euler’s, modified Euler’s and Runge— Kutta method of fourth order.

Euler’s Method In Taylor series method, we obtain approximate solution of the initial value problem

Picard’s Method

dy = f (x, y), y (x0 ) = y0 dx

This method is also known as Picard’s method of successive approximations. Consider dy = f (x, y) and y = y0 at x = x0 dx

(58)

Integrating the given equation, we get

as a power series in x and the solution can be used to compute y numerically using specified value of x near x0. Let y1 = y(xi ), where x1 = x0 + h. Then y1 = y(x + h). Then by Taylor series, y1 = y (x0 ) +

x

y=

∫ f(x, y)dx + C



x0

(62)

Neglecting the terms with h2 and higher powers of h in Eq. (62), we get

As y = y0 at x = x0, we have y0 = C. Therefore,

y1 = y0 + hf (x0 , y0 )

x

y = y0 + ∫ f (x, y)dx + C

h h2 y ′ (x0 ) + y ′′ (x0 ) +  1! 2!

(59)

x0 Since y is a function of x, Eq. (58) can also be expressed as



(63)

Expression from Eq. (63) gives an approximate value of y at x1 = x0 + h. Similarly, we get

x



y2 = y1 + hf (x1 , y1 ) for x2 = x1 + h

y(x) = y0 + ∫ f (x, y)dx x0

For any n,

Equation (58) can be solved by successive approximation method (also known as the method of iteration).

yn = yn −1 + hf (xn −1 , yn −1 ) for n = 1, 2, 3, …

The first approximation of y is obtained by putting y0 for y on the right-hand side of Eq. (58). Then y = y(6.1) is the value of y and is given by

Figure 8 shows the Euler’s method.

(64)

y

x

y

(1)

= y0 + ∫ f (x, y0 )dx

True value of y

(60) Q Q1

x0 As y0 is a constant, the integral on the right-hand side of Eq. (60) can be solved, which yields y(6.1). This is again substituted on the right-hand side of Eq. (60) to get the second approximation to y, which is y(6.2).

Pn P

y(2) = y0 + ∫ f (x, y(1) )dx x0

Chapter 6.indd 321

R1

R2

y0

0 x

P2

P1

Error Approximate Rn value of y

x L L1 L2 x0 x0 + h x0 + 2h

M x0 + nh

Figure 8|   Illustration of Euler’s method.

8/22/2017 4:28:40 PM

322     Numerical Methods 

Modified Euler’s Method

  h k2 = hf x + , y + k   2

(70)



k4 = hf (x + h, y + k3 )

(71)

We know by Eq. (63) that y1 = y0 + hf (x0 , y0 )



Now, in modified Euler’s method, we first compute y1(1) = y0 + hf (x0 , y0 ) by Euler’s method and y1 = y (x1 ) is given by

(

)

h (1)  (65) f (x0 , y0 ) + f x1 , y1    2 This formula is known as improved Euler’s formula or modified Euler’s formula. Now yn+1 for xn +1 = xn + h is computed as follows: y1 = y0 +

   h h y (x + h) = y (x) + h f x + , y + f (x, y )     2 2   or



1 (k + 2k2 + 2k3 + k4 ) 6 1 y(x + h) = y(x) + ∆y

∆y =

(72) (73)

Euler’s Predictor–Corrector Method If xn−1 and xn are two consecutive points, then xn = xn −1 + h. Hence, by Euler’s method, we know that yn = yn −1 + hf (xn −1 , yn −1 ) for n = 1, 2, 3, …

(74)

(66) The modified Euler’s method gives

    h h y(n) = yn −1 + hf  xn −1 + , yn −1 + f (xn −1 , yn −1 )  (75)     2 2    h h y(n) = yn −1 + h f xn −1 + , yn −1 + f (xn −1 , yn −1 )   2 2   where yn = y (xn −1 + h) , h is the step size and n = 1,    h h 2, 3, …, etc. y(n) = yn −1 + h f xn −1 + , yn −1 + f (xn −1 , yn −1 ) (67)    2 2 The value of yn is first estimated by using Eq. (74), then this value is inserted on the right-hand side of Eq. (75), giving a better approximation of yn. This value where yn = y (xn −1 + h) , h is the step size and n = 1, of yn is again substituted in Eq. (75) to find a still better 2, 3, …, etc. approximation of yn. This step is repeated till two consecutive values of yn are equal. This method is known as predictor—corrector method. Equation (74) is called Runge–Kutta Method the predictor and Eq. (75) is called the corrector of yn.

A method of great practical importance and much greater accuracy than that of the improved Euler’s method is the Runge—Kutta method. In each step, we first compute four auxiliary quantities k1 , k2 , k3 , k4 and then the new value yn+1. This method needs no special starting procedure and does not require a lot of storage and repeatedly uses the same straightforward computational procedure. Hence, the method is well suited to the computer. Moreover, it is numerically stable. The use of the previous methods to solve the differential equation numerically is restricted due to either slow convergence or labor involved, especially in Taylor’s series method. But in Runge—Kutta methods, the derivatives of higher order are not required and we require only two given functional values at different points. The fourth-order Runge—Kutta method algorithm is mostly used for solving problems unless otherwise mentioned. It is given as follows:

Chapter 6.indd 322

k1 = h + (x, y)  h k k2 = hf x + , y +   2 2

(68) (69)

ACCURACY AND PRECISION Accuracy describes how close the measured value of a quantity is to its true or actual value. For example, if a weighing machine measures the weight of a person, who actually weighs 72 kg, as let us say 71.9 kg, the measurement can be considered fairly accurate. Precision is defined as the ability of a measurement to be consistently reproduced and the number of significant digits to which a value has been reliably measured. If on making several measurements, the measured value matches the actual value while the actual values held constant, then the measurement is considered as precise also in addition to being accurate. As far as the number of significant digits with reference to precision is concerned, the number 71.9135 is more precise than 71.91. It may be mentioned that a measurement can be accurate and imprecise at the same time. Similarly, measurements can be precise and inaccurate, inaccurate and imprecise and accurate and precise. The following examples illustrate this further.

8/22/2017 4:28:48 PM

SIGNIFICANT FIGURES     323

1. A temperature sensor is used for measuring the temperature inside a refrigerator whose temperature is maintained constant at 8°C. Ten measurements are made with the sensor. The results are 8.1°C, 7.9°C, 7.8°C, 8.2°C, 8.1°C, 7.9°C, 8.2°C, 8.1°C, 7.9°C and 8.1°C. The measurements show a tendency towards a particular temperature value, which is also very close to the true value. This is an example of good accuracy and good precision. 2. In the above example, if the results of 10 measurements were 8.2°C, 7.9°C, 7.5°C, 8.7°C, 7.3°C, 8.1°C, 7.9°C, 8.6°C, 7.4°C and 7.2°C, the measurement is not only inaccurate but also imprecise. 3. Again, in the above example, if the measurements were 8.1°C, 8.2°C, 8.3°C, 7.8°C, 7.7°C, 8.2°C, 8.3°C, 7.8°C and 8.4°C, they can be considered fairly accurate but lack precision. 4. Yet in another case, in the above example, if the measurements were 7.1°C, 7.2°C, 6.9°C, 7.0°C, 7.1°C, 7.2°C, 6.9°C, 7.1°C, 7.0°C and 7.1°C, they are fairly precise but highly inaccurate.

device. Random errors are also called statistical errors because they can be eliminated in a measurement by statistical means due to their random nature.

To sum up, for evaluating the accuracy of a measurement, the measured value must be compared to the true value. To evaluate the precision of a measurement, you must compare the values of two or preferably more measurements.

SIGNIFICANT FIGURES

CLASSIFICATION OF ERRORS Classification of errors is important for understanding of error analysis. Errors are broadly classified as 1. Systematic or determinate errors 2. Random or indeterminate errors Systematic errors are those that can be avoided or whose magnitude can be determined. Systematic errors are further classified as (a) operational errors, (b) instrument errors, (c) errors of method and (d) additive or proportional error. Operational errors are those for which the individual analyst is responsible. These are not connected with the procedure. Instrument errors are the errors that occur due to the instrument used for making a measurement. Errors of method occur due to the method. These errors are difficult to correct. Proportional error is a type of systematic error that changes as the variable being observed changes, although the change is predictable. Random errors, as the name suggests, are random in nature and very difficult to predict. Random errors are statistical fluctuations (in either direction) in the measured data due to precision limitations of a measurement

Chapter 6.indd 323

Also, the error can be specified as an absolute error or a relative error. The absolute error is the measure of uncertainty in the quantity and has the same units as the quantity itself. For example, if the weight of a commodity is specified as 535 ± 20 grams, then 20 grams is the absolute error. The relative error, also called fractional error, is obtained by dividing absolute error by the quantity itself and is a dimensionless quantity. When expressed as a percentage, it is multiplied by 100. For example, the floor area of a room when specified as 10 ± 0.5 m2 has an absolute error of 0.5 m2 and relative error of 0.05 or 5%. Relative error is usually more significant than the absolute error. For example, a 1 mm error in a skate wheel is more serious than a 1 mm error in the diameter of a truck tyre.

Significant figures of a measured quantity are the meaningful digits in it. Following are the important facts about significant figures: 1. Any digit that is not zero is significant. For example, 346 has three significant figures and 2.5478 has five significant figures. 2. Zeros between non-zero digits are significant. For example, 3046 has four significant figures. 3. Zeros to the left of first non-zero digit are not significant. For example, 0.000346 has only three significant figures. 4. For numbers with decimal points, zeros to the right of a non-zero digit are significant 23.00 and 10.0430, respectively, have four and three significant figures. 5. For numbers without decimal points, trailing zeros may or may not be significant. For example, 500 and 500.5, respectively, have one and four significant figures. Even 500. has three significant figures due to addition for a decimal point. 6. Exact numbers have an infinite number of significant digits. For example, the number representing p has infinite number of significant digits. With reference to significant figures, while expressing uncertainty, last significant figure in any result should be of the same order by magnitude, that is, in the same decimal position as the uncertainty. For example, 9.81 ± 0.03, 15.2 ± 1.3 and 7 ± 1 are correct representation; 9.81 ± 0.03567 is not correct.

8/22/2017 4:28:48 PM

324     Numerical Methods 

MEASURING ERROR There are several different ways in which the distribution of the measured values of a repeated experiment can be specified. Errors may be measured as maximum error, probable error, average deviation, mean and standard deviation. Maximum error is given by x − xmin = max 2

∆xmax

Virtually, no measurements should ever fall outside (x ± ∆xmax ), where x is the mean. Probable error, ∆xprob, specifies the range x ± ∆xprob, which contains 50% of the measured values. Average deviation is the average of the deviations from the mean and is given by N

∑ xk − x k =1

result. In such cases, you may arrive at the final result using some formula. This calculated result or the propagated error can be obtained from a few simple rules provided that the errors in the measured quantities are random in nature and are independent, that is, if one quantity is measured as being larger than it actually is, another quantity is still just as likely to be smaller or larger. These rules are summarized as follows and do not apply to systematic errors. 1. If z = x ± y, then ∆z = [(∆x)2 + (∆y)2 ]1/2 Here, ∆z is the propagated error. 2. If z = ax ± b, then ∆z = a∆x 3. If z = cxy, then ∆z /z = [(∆x/x)2 + (∆y /y)2 ]1/2 y 4. If z = c , then ∆z /z = [(∆x/x)2 + (∆y /y)2 ]1/2 x

5. If 6. If 7. If 8. If 9. If

z z z z z

= = = = =

cxa, then ∆z /z = a(∆x/x) cxayb, then  ∆z /z = [(a∆x/x)2 + (b∆y/y)2 ]1/2 sin x, then ∆z /z = ∆x cot x cos x, then ∆z /z = ∆x tan x tan x, then ∆z /z = ∆x/ sin x cos x

N In the above formulas, errors in a, b and c are considered negligible.

Mean is given by 1 N

N

∑ xk

METHOD OF LEAST SQUARE APPROXIMATION

k =1

Standard deviation is given by 1 s x =   N

N



k =1



1/ 2

∑ (xk − x)2 

The square of standard deviation is called variance. Mean and standard deviation are important parameters to estimate random error. If a measurement, which is subject to only random fluctuations, is repeated many times, then approximately 68% of the measured values will fall in the range x ± s x, where xx and sx are, respectively, mean and standard deviation of measured quantity x. If the measurement is repeated more number of times, we become sure that x represents the true value of the quantity. A useful parameter, in this regard, is the standard deviation s x of the mean and is given by (s x / N ) . s x is a good estimate of uncertainty in x. The precision of measurement increases in proportion to N , as we increase the number of measurements.

PROPAGATION OF ERRORS

The principle of least squares provides an elegant procedure for fitting a unique curve to a given data. The procedure or method is illustrated as follows with the help of an example. Let us suppose that it is required to fit a curve given by y = a + bx + cx2 (a parabola) to a given set of observations defined by (x1, y1), (x2, y2), …, (x4, y4). Now for any xi, the observed value is yi and the expected value is a + bxi + cx12. The error is ei = yi − (a + bxi + cxi2 ). The sum of squares of these errors (E) is given by E = e12 + e22 + e32 + e42 = [y1 − (a + bx1 + cx12 )]2 + [y2 − (a + bx2 + cx22 )]2 +  + [y4 − (a + bx4 + cx4 )]2 For E to be minimum, ∂E ∂E ∂E = = =0 ∂a ∂b ∂c This leads to the following three equations: 2[y1 − (a + bx1 + cx12 ) − 2[y2 − (a + bx2 + cx22 )]

In real-life experimental measurements, there may be more than one parameter that contributes to the final

Chapter 6.indd 324

− − 2[y4 − (a + bx4 + cx42 )] = 0

8/22/2017 4:28:57 PM

LAGRANGE POLYNOMIALS     325

—2x1[y1 — (a + bx1 + cx12 )] — 2x2 [y2 — (a + bx2 + cx22 )] —  — 2x4 [y4 — (a + bx4 + cx42 )] = 0 —2x12 [y1 — (a + bx1 + cx12 )] — 2x22 [y2 — (a + bx2 + cx22 )] —  — 2x42 [y4 — (a + bx4 + cx42 )] = 0

For example, for n = 3,

These equations can be further simplified to

∑ yi = 4a + b∑ xi + c ∑ xi2 

smooth-fitting function. More the number of data points, higher is the degree of Lagrange polynomial. Higher degree polynomial produces greater oscillation between data points. As a result, a higher degree interpolation may be a poor predictor of function between data points, though the accuracy at the data points will be higher.

(76)

∑ xiyi = a∑ xi + b∑ xi2 + c ∑ xi3 

(77)



∑ xi2yi = a∑ xi2 + b∑ xi3 + c ∑ xi4 

(78)



(x − x2 )(x − x3 ) (x − x1 )(x − x2 ) y y1 + (x1 − x2 )(x1 − x3 ) (x2 − x1 )(x2 − x3 ) 2

P (x) =

+

(x − x1 )(x − x2 ) y (x3 − x1 )(x3 − x2 ) 3

Now

Equations (76), (77) and (78) are known as normal equations. These equations are solved simultaneously to determine a, b and c. Values of a, b and c are substituted in the equation of the curve to be filled (in this case, y = a + bx + cx2) to get the desired curve of best fit. The procedure is summarized as follows: 1. Substitute the observed set of values in the equation of desired curve of fit. 2. Form normal equations for each constant. 3. Solve normal equations as simultaneous equations to determine values of constants. 4. Substitute the values of constants in the equation of curve to be fitted to obtain the desired curve of best fit.

P ′(x) =

+

(x1 − x2 )(x1 − x3 ) (x − x1 )(x1 − x3 ) y1 + 1 y (x1 − x2 )(x1 − x3 ) (x2 − x1 )(x2 − x3 ) 2

P (x1 ) =

+

LAGRANGE POLYNOMIALS

P (x3 ) =

(x2 − x1 )(x2 − x2 ) y = y2 (x3 − x1 )(x3 − x2 ) 3

(x3 − x2 )(x3 − x3 ) (x − x1 )(x3 − x3 ) y y1 + 3 (x1 − x2 )(x1 − x3 ) (x2 − x1 )(x2 − x3 ) 2 +

∑ Pj (x)

(x1 − x1 )(x1 − x2 ) y = y1 (x3 − x1 )(x3 − x2 ) 3

(x1 − x2 )(x2 − x3 ) (x − x1 )(x2 − x3 ) y + 2 y (x1 − x2 )(x1 − x3 ) 1 (x2 − x1 )(x2 − x3 ) 2 +

n

P (x) =

2x − x1 − x2 y (x3 − x1 )(x3 − x2 ) 3

It can be seen from the following equations that function P (x) passes through the points (xi, yi)

P (x2 ) =

Lagrange polynomial is a polynomial P(x) of degree ≤ (n − 1) that passes through the n points (x1, y = f(x1)), (x2, y2 = f(x2)), … (xn, yn = f(xn)). It is given by

2x − x2 − x3 2x − x1 − x3 y1 + y (x1 − x2 )(x1 − x3 ) (x2 − x1 )(x2 − x3 ) 2

(x3 − x1 )(x3 − x2 ) y = y3 (x3 − x1 )(x3 − x2 ) 3

j =1

Generalizing to arbitrary (n), x − xk where Pj (x) = y j Π . k =1 xj − xk n

n

P (xj ) =

k≠ j

n

∑ Pk (xj ) =

∑ djk yk = yj

k =1

k =1

It can also be written as (x − x2 )(x − x3 )… (x − xn ) P (x) = y (x1 − x2 )(x1 − x3 )… (x − xn ) 1 +

(x − x1 )(x − x3 )… (x − xn ) y + (x2 − x1 )(x2 − x3 )… (x2 − xn ) 2

(x − x1 )(x − x2 )… (x − xn−1 ) + y (xn − x1 )(xn − x2 )… (xn − xn−1 ) n While using Lagrange polynomials for interpolation, there is a trade-off between having a better fit and a

Chapter 6.indd 325

Lagrange interpolating polynomials can also be written using Lagrange’s fundamental interpolating polynomials. n

Let

p (x) ≡ Π (x − xk ) k =1 n

p (xj ) = Π (xj − xk ) k =1 n  dp  p ′(xj) =   = Π (xj − xk )  dx  x=x k =1 j k≠ j

8/22/2017 4:29:03 PM

326     Numerical Methods 

So that p(x) is an nth degree polynomials with zeros at x1, x2, …, xn. Fundamental polynomials are defined by p n (x) =

Differentiating both sides with respect to p, we get dy 2p − 1 2 3 p2 − 6 p + 2 3 ∆ y0 + ∆ y0 +  = ∆y0 + dp 2! 3!

p (x) ′ p (xn )(x − xn )

which satisfy p n (xm ) = dnm .

Since, p =

x − x0 , h

(dnm ) is the Kronecker delta Let y1 = P (x1), y2 = P(x2), …, yn = P(xn), then

Therefore,

dp 1 = dx h

n

n

k =1

k =1

P (x) =

p (x)

dy dy dp ⋅ = dx dp dx  1 2p − 1 2 3 p2 − 6 p + 2 3 ∆ y0 + ∆ y0 +  = ∆y0 + 2! 3! h   At x = x0, p = 0. Substituting p = 0, we get

∑ p k (x)yk = ∑ (x − x )p ′(x ) yk k

k

This is the unique Lagrange interpolating polynomial assuming the values yk at xk.

NUMERICAL DIFFERENTIATION

 dy     dx  x

0

Numerical differentiation is the process of determining the numerical value of a derivative of a function at some assigned value of x from the given set of values (xi, yi). The interpolation formula to be used depends on the assigned value of x at which it is desired to find (dy/dx). Three commonly used formulas include the following:

 1 1 1 1 1 = ∆y0 − ∆2 y0 + ∆3 y0 − ∆4 y0 + ∆5 y0 −  h  2 3 4 5 d2y = dx

1. Newton’s forward interpolation formula: It is used if the values of x are equispaced and it is desired to find (dy/dx) near the beginning of the table. 2. Newton’s backward interpolation formula: It is used if it is desired to find (dy/dx) near the end of the table. 3. Stirling’s or Bessel’s formula: It is used when (dy/dx) is desired to be computed near the middle of the table.

Newton’s Forward Formula Let the function y =f(x) take values y1, y2, y3, … corresponding to the values x0, x0 + h, x0 + 2h, … of (x). If p is any real number and it is required to evaluate f(x) for x = x0 + ph, we can write −p

y p = f (x0 + ph) = E p f (x0 ) = (1 + ∆ )

y0

E p f (x) = f (x + ph)

 1  2 2 6p − 6 3 12 p2 − 36 p + 22 4 ∆ y0 + ∆ y0 + ∆ y0 +  3! 4! h  2 ! 

Substituting p = 0, we get  d 2y     dx2   x0  =

 1  2 11 5 137 6 ∆ y0 − ∆3y0 + ∆4y0 − ∆5y0 + ∆ y0 + 2   12 6 180 h 

 d3y   1  3 Similarly,  3  = 3 ∆3 y0 − ∆4 y0 +   2  dx  h 

Newton’s Backward Formula In this case, we will write the following expression for yp: (∵ E −1 = 1 − ∇)

( ∵ E = 1 + ∆)

Therefore,   p( p − 1) 2 p( p − 1)( p − 2) 3 y p = 1 + p∆ + ∆ + ∆ +  y0   2! 3!

Chapter 6.indd 326

d  dy  dp   dp  dp  dx

y p = f (xn + ph) = E p f (xn ) = (1 − ∇)−p yn

E is defined as

y p = y0 + p∆y0 +

=

2

p( p − 1) 2 p( p − 1)( p − 2) 3 ∆ y0 + ∆ y0 +  2! 3!

  p( p + 1) 2 p( p + 1)( p + 2) 3 y p = 1 + p∇ + ∇ + ∇ +  yn   2! 3! p( p + 1) 2 ∇ yn 2! p( p + 1)( p + 2) 3 ∇ yn +  + 3!

y p = yn + p∇yn +

8/22/2017 4:29:10 PM

SOLVED EXAMPLES      327

Differentiating with respect to (p), we get

Gauss’ formula is written as

dy 2p + 1 2 3 p2 + 6 p + 2 3 = ∇yn + ∇ yn + ∇ yn +  dp 2! 3!

p( p + 1) 2 ∆ y−1 2! p( p + 1)( p − 1) 3 ∆ y−1 + 3! ( p + 1)( p − 1)( p − 2) 4 ∆ y−2 +  + 4!

y p = y0 + p∆y0 +

Since p = (x − xn )/h , dp/dx = 1/h . Now dy dy dp = ⋅ dx dp dx  1 2p + 1 2 3 p2 + 6 p + 2 3 = ∇yn + ∇ yn + ∇ yn +  2! 3! h  

Gauss’ back interpolation formula is given by ( p + 1)( p) 2 ∆ y−1 2! p( p + 1)( p − 1) 3 ∆ y−2 + 3! ( p + 2)( p + 1)(p p)( p − 1) 4 ∆ y−2 + 4!

y p = y0 + p∆y−1 +

At x = xn, p = 0. Substituting p = 0, we get  dy   1 1 1 1   = ∇yn + ∇2 yn + ∇3 yn + ∇4 yn +   dx  x   h 2 3 4 0

Taking the mean of the two, we get  ∆y + ∆y−1  p2 2 + y p = y0 + p  0 ∆ y−1   2 ! 2 2 3 p( p2 − 1)  ∆ y−1 + ∆ y−2  × +  3! 2  

Differentiating again with respect to (x), we get d2y = dx =

2

d  dy  dp   dp  dx  dx

 1  2 6p + 6 3 6 p2 + 18 p + 11 4  ∇ ∇ y y ∇ y +  + + n n n   3! 12 h2  

+

p2 ( p2 − 1) 4 ∆ y−2 +  4!

Substituting p = 0, we get This is known as Gauss’ backward interpolation ­formula; is also known as Stirling formula.

 d 2y     dx2   x0  =

In the case of central difference equation, the equation takes the form:

 1  2 11 5 137 6 ∇ yn + ∇3yn + ∇4yn + ∆5yn + ∇ yn +  2   12 6 180 h 

y p = y0 + pmdy0 +

Similarly,

+  d2y  1    3 = 3 h  dx  x0

 3  3 ∇ yn + ∇4 yn +    2

p( p2 + 12 ) 3 p2 2 ∆ y−1 + md y0 2! 3!

p2 ( p2 − 12 ) 4 d y0 4!

or 1 1 (∆y0 + ∆y−1 ) = (dy1/2 + dy−1/2 ) 2 2

Stirling’s or Bessel’s Formula

or

Stirling’s formula is the mean of Gauss’ forward interpolation formula and Gauss’ back interpolation formula.

1 1 (∆y0 + ∆y−1 ) = (dy1/2 + dy−1/2 ) = mdy0 2 2 1 2 1 (∆ y−1 + ∆3 y−2 ) = (d 2 y1/2 + d 3 y−1/2 ) = md 3 y3 2 2

SOLVED EXAMPLES 1. Solve the following set of equations using Gauss elimination method: x + 4y − z = −5 x + y − 6z = −12 3x − y − z = 4

Chapter 6.indd 327

x + 4y − z = −5 x + 4y − z = − 5 x + y − 6z = −12 x + y − 6z = −12 3x − y − z = 4 3x − y − z = 4

8/22/2017 4:29:16 PM

328        Numerical Methods 

Solution:  We have x + 4y − z = − 5



Thus, we get     1 1 1   1 0 0   1   A = LU = 2 −3 0   0 1 −  3   3 −2 − 14   0 0 1    3   The given system is AX = B. This gives

(1)

x + y − 6z = −12

(2)



3x − y − z = 4

(3)



Now, performing Eq. (2) — Eq. (1) and Eq. (3) — 3 × Eq. (1) to eliminate x from Eq. (2) and (3), we get −3y − 5z = −7



(4)

−13y + 2z = 19

(5) 13 Now, eliminating y by performing Eq. (5) — × 3 Eq. (4), we get



71 148 z= 3 3



(6)

Now, by back substitution, we get z=

148 = 2.0845 71



LUX = B Let UX = Y, so from Eq. (1), we have     0   y1   3  1 0      LY = B ⇒ 2 −3 0   y2  =  16       3 −2 − 14   y3  −3  3  which gives y1 = 3 3y1 − 3y2 = 16 ⇒ 9 − 3y2 = 16 ⇒ y2 = −

From Eq. (6), we get 7  5  148  81 y = −     = − = −1.1408   3  3   71  71 From Eq. (i), we get  81 148  117 x = −5 − 4 −  +  = = 1.6479  71   71  71 Hence, x = 1.6479, y = −1.1408, z = 2.0845. 2. Solve the following system of equations by Crout’s method: x+y +z = 3 2x − y + 3z = 16 3x + y − z = −3

(1)

10 3

14 20 14 y = −3 ⇒ 9 + − y3 3 3 3 3 = − 3 ⇒ y3 = 4 1 1 1   3     x   1     10   Now, UX = Y ⇒  0 1 −   y  = −  3    3    0 0 1   z   4      which gives x+y +z = 3 3y1 − 2y2 −

z=4 1 10 10 4 y − z = − ⇒ y = − + = −2 3 3 3 3 x−2+4 = 3 ⇒ x = 1 By back substitution, x = 1, y = −2, z = 4.

Solution:  We choose uii = 1 and write 1 1 1   l11    2 −1 3  = l21 3 1 −1    l31 l  11 = l21  l31

0 l22 l32

A = LU 0   1 u12  0  0 1  l33   0 0

l11u12 l21u12 + l22 l31u12 + l32

u13   u23   1 

 l11u13  l21u13 + l22 u23   l31u13 + l32 u23 + l33 

Equating, we get l11 = 1, l21 = 2, l31 = 3 l11u12 = 1 ⇒ u12 = 1,

l11u13 = 1 ⇒ u13 = 1

l21u12 + l22 = −1 ⇒ l22 = −3, 1 l31u13 + l22 u23 = 3 ⇒ u23 = − , 3 l31u13 + l32 u23 + l33 = −1 ⇒ l33 = −

Chapter 6.indd 328

14 3

3. Solve the following system of equations using Doolittle’s method: 8x2 + 2x3 = −7 3x1 + 5x2 + 2x3 = 8 6x1 + 2x2 + 8x3 = 26 Solution:  The decomposition is obtained from  a11 a12 a13   3 5 2     A = [ajk ] = a21 a22 a23  =  0 8 2     a31 a32 a33   6 2 8  1 0 0  u11 u12 u13     = l21 1 0  0 u22 u23     0 u33  l31 l32 1  0 By comparing, we get a11 = 3 = 1 × u11 = u11 a21 = 0 = l21u11 ,

l21 = 0

a31 = 6 = l31u11 = l31 × 3,

l31 = 2

8/23/2017 5:21:37 PM

SOLVED EXAMPLES         329

a12 = 5 = 1 × u12 = u12 a22 = 8 = l21u12 + u22 ,

x+y +z = 9 2x − 3y + 4z = 13 3x + 4y + 5z = 40

u22 = 8

a32 = 2 = l31u12 + l32 u22 = 2 × 5 + l32 × 8,

l32 = −1

Solution:  We are given that

a13 = 2 = 1 × u13 = u13 x+y +z = 9 a23 = 2 = l23 u13 + u23 ,

u23 = 2

a33 = 8 = l31u13 + l32 u23 + u33 = 2 × 2 − 1 × 2 + u33 ⇒ u33 = 6



We first solve LY = B, determining values of y1 , y2 and y3 1 0 0  y1   8    1 0  y2  = −7  0  2 −1 1        y3   26   

3x + 4y + 5z = 40

(3)

x+y +z = 9

(4)

−5y + 2z = −5

(5)

y + 2z = 13

(6)

1 1 × Eq. (5) and Eq. (6) — × 5 5 Eq. (5) to eliminate y from Eq. (4) and Eq. (6), we get Performing Eq. (4) +



y2 = − 7

(2)

Performing Eq. (2) — 2 × Eq. (1) and Eq. (3) — 3 × Eq. (1) to eliminate x from Eq. (2) and Eq. (3), we get

By solving the above equation, we get y1 = 8

2x − 3y + 4z = 13



Using these values, we get 1  3 5 2 0 0     L = 0 1 0  ; U =  0 8 2  2 −1 1  0 0 6    

(1)



2y1 − y2 + y3 = 26 ⇒ 2(8) − (−7) + y3 = 26

7 x+ z = 8 5 −5y + 2z = −5 12 z = 12 5

(7) (8) (9)

y3 = 3  8   Hence, Y = −7     3   Then we solve UX = Y , determining values of x1 , x2 and x3 ,  3 5 2  x1   8    0 8 2  x2  = −7   0 0 8        x3   3    So, 1 2 8x2 + 2x3 = −7 ⇒ x2 = −1

7 5 × Eq. (9) and Eq. (8) — × 12 6 Eq. (9) to eliminate y from Eq. (7) and Eq. (8), we get Performing Eq. (7) +

x=1 −5y = −15 ⇒ y = 3 12 z = 12 ⇒ z = 5 5 Hence, the solution is x = 1, y = 3, z = 5. 5. Solve the following system of equations using Gauss—Seidel iterative method: 2x + 10y + z = 51; 10x + y + 2z = 44; x + 2y + 10z = 61

x3 =

2x1 − 5x2 + 2x3 = 8 ⇒ x1 = 4 4    Hence, X = −1  .   1/2   4. Apply Gauss—Jordan method to solve the following set of equations:

Chapter 6.indd 329

Solution:  The given system is not diagonally dominant. Hence, rearrange the equations as follows to make them diagonally dominant: 10x + y + 2z = 44 2x + 10y + z = 51 x + 2y + 10z = 61 Now, Gauss—Seidel’s iterative method can be applied. From the above equations, we get

8/23/2017 5:21:49 PM

330     Numerical Methods 

Solution:  Here, we see that the diagonal elements are dominant. Hence, the coefficient matrix

1 (1) (44 − y − 2z) 10 1 y = (51 − 2x − z) (2) 10 1 z = (61 − x − 2y) (3) 10 The first approximation is obtained by putting y = z = 0 in Eq. (1), x=



10 −5 −2   3  4 −10  1 6 10  is diagonally dominant since | 10 |>| −5 | + | −2 |, | −10 |>| 4 | + | 3 |, | 10 |>| 1 | + | 6 |

1 x = (44 − 0 − 0) = 4.4 10 Now, putting x = 4.4 and z = 0 in Eq. (2), we get (1)

y(1) =

1 [51 − (2)(4.4) − 0 ] = 4.2 10

Again putting x = 4.4 and y = 4.22 in Eq. (3), we get 1 z(1) = [61 − 4.4 − (2)(4.2)] = −2.9753 10 The second approximation is given by 1 x(2) = [44 − 4.2 − 2(−2.9753)] = 3.0148 10 1 (2) y = [51 − (2)(3.0148) − (−2.9753)] = 4.01544 10 1 z(2) = [61 − 3.0148 − 2(4.01544)] = 4.995432 10 Similarly, the third approximation is given by 1 x(3) = [44 − 4.01544 − 2(4.995432)] 10 = 2.9993696 1 y(3) = [51 − (2)(2.9993696) − (4.995432)] 10 = 4.00058288 1 z(3) = [61 − 2.9993696 − 2(4.00058288)] 10 = 4.999946464

Therefore, the iteration process can be applied. Solving for x, y, z, we get x=

1 (3 + 5y + 2z) 10

(1)

y=

1 (3 + 4x + 3z) 10

(2)

z=

1 (−3 − x − 6y) 10

(3)







The first iteration is given by substituting x = 0, y = 0 and z = 0. Therefore, 1 [3 + 5(0) + 2(0)] = 0.3 10 1 = [3 + 4(0) + 3(0)] = 0.3 10 1 = [−3 − (0) − 6(0)] = −0.3 10

x(1) = y(1) z(1)

The second iteration is given by 1 [3 + 5(0.3) + 2(−0.3)] = 0.39 10 1 = [3 + 4(0.3) + 3(−0.3)] = 0.33 10 1 = [−3 − (0.3) − 6(0.3)] = −0.51 10

x(2) = y(2) z(2)

The third iteration is given by The fourth approximation is given by 1 [44 − 4.00058288 − 2(4.999946464)] 10 = 2.9993696 ≈ 3 1 y(4) = [51 − (2)(3) − (4.995432)] = 4.00058288 ≈ 4 10 1 (4) z = [61 − 3 − 2(4)] = 4.999946464 ≈ 5 10 Hence, after four iterations, we obtain x = 3, y = 4, z = 5. x(4) =

y(3) z(3)

The fourth iteration is given by 1 [3 + 5(0.303) + 2(−0.537)] = 0.3441 10 1 = [3 + 4(0.363) + 3(−0.537)] = 0.2841 10 1 = [−3 − 0.363 − 6(0.303)] = −0.5181 10

x(4) =

6. Solve the following system of equations by Gauss— y(4) Jacobi method: 10x − 5y − 2z = 3; 4x − 10y + 3z = −3; x + 6y + 10z = −3 z(4) − 5y − 2z = 3; 4x − 10y + 3z = −3; x + 6y + 10z = −3

Chapter 6.indd 330

1 [3 + 5(0.33) + 2(−0.51)] = 0.363 10 1 = [3 + 4(0.39) + 3(−0.51)] = 0.303 10 1 = [−3 − (0.39) − 6(0.33)] = −0.537 10

x(3) =

8/22/2017 4:29:51 PM

SOLVED EXAMPLES      331

The fifth iteration is given by 1 x(5) = [3 + 5(0.284) + 2(−0.5181)] = 0.33843 10 1 (5) y = [3 + 4(0.3441) + 3(−0.5181)] = 0.2822 10 1 (5) z = [−3 − (0.3441) − 6(0.2841)] = −0.50487 10 The sixth iteration is given by 1 x(6) = [3 + 5(0.2822) + 2(−0.50487)] = 0.340126 10 1 y(6) = [3 + 4(0.33843) + 3(−0.50487)] = 0.283911 10 1 (6) z = [−3 − (0.33843) − 6(0.2822)] = −0.503163 10 The seventh iteration is given by 1 x(7) = [3 + 5(0.283911) + 2(−0.503163)] = 0.3413229 10 1 y(7) = [3 + 4(0.340126) + 3(−0.503163)] = 0.2851015 10 1 z(7) = [−3 − (0.340126) − 6(0.283911)] = −0.5043592 10 The eighth iteration is given by 1 x(8) = [3 + 5(0.2851015) + 2(−0.5043592)] 10 = 0.34167891 1 y(8) = [3 + 4(0.3413229) + 3(−0.5043592)] 10 = 0.2852214 1 z(8) = [−3 − (0.3413229) − 6(0.2851015)] 10 = −0.50519319 The ninth iteration is given by 1 x(9) = [3 + 5(0.2852214) + 2(−0.50519319)] 10 = 0.341572062 1 y(9) = [3 + 4(0.34167891) + 3(−0.50519319)] 10 = 0.285113607 1 z(9) = [−3 − (0.34167891) − 6(0.2852214)] 10 = −0.505300731 Hence, the values correct to three decimal places are x = 0.342, y = 0.285, z = −0.505. 7. Find a positive root of x3 − x − 1 = 0 correct to two decimal places by bisection method.

Using x0 = 1 and x1 = 2 in the given function, we get f(1) = (1)3 − 1 − 1 = −1 < 0 and f(2) = (2)3 − 2 − 1 = 5 > 0. Therefore, one root lies between 1 and 2. By bisection method, the next approximation x2 is given by x + x1 1+2 x2 = 0 = = 1. 5 2 2 ⇒ f (1.5) = (1.5)3 − (1.5) − 1 = 0.875 > 0. Therefore, the root lies between 1 and 1.5. The next approximation x3 is given by x1 + x2 1 + 1. 5 = = 1.2 2 2

x3 =

⇒ f (1.25) = (1.25)3 − (1.25) − 1 = −0.2968 < 0 Therefore, the root lies between 1.25 and 1.5. The next approximation x4 is given by x4 =

x2 + x3 1.25 + 1.5 = = 1.375 2 2

⇒ f (1.375) = (1.375)3 − (1.375) − 1 = 0.224 > 0. Therefore, the root lies between 1.25 and 1.375. The next approximation x5 is given by x5 =

x3 + x4 1.25 + 1.375 = = 1.3125 2 2

⇒ f (1.3125) = (1.3125)3 − (1.3125) − 1 = −0.0515 < 0 Therefore, the root lies between 1.3125 and 1.375. The next approximation x6 is given by x6 =

x4 + x5 1.375 + 1.3125 = = 1.34375 2 2

⇒ f (1.34375) = (1.34375)3 − (1.34375) − 1 = 0.0826 > 0 Therefore, the root lies between 1.3125 and 1.34375. The next approximation x7 is given by x5 + x6 1.3125 + 1.34375 = 2 2 = 1.3281 ⇒ f (1.3281)

x7 =

= (1.3281)3 − (1.3281) − 1 = 0.01447 > 0 Therefore, the root lies between 1.3125 and 1.3281. The next approximation x8 is given by x8 =

x5 + x7

=

2 Solution:  Let f (x) = x3 − x − 1 = 0

Chapter 6.indd 331

1.3125 + 1.3281 = 1.32 2

⇒ f (1.32) = (1.32)3 − (1.32) − 1 = −0.0187 < 0 Hence, the root is 1.32.

8/22/2017 4:29:59 PM

332     Numerical Methods 

8. By using Regula—Falsi method, find an approximate root of the equation x4 − x − 10 = 0 that lies between 1.8 and 3. Carry out three approximations.

Therefore, the root lies between 1 and 2. Now, f (1.5) = (1.5)4 − (1.5) − 9 = −5.4375 f (1.75) = (1.75)4 − (1.75) − 9 = −1.3711

Solution:  Let f (x) = x4 − x − 10. Given x0 = 1.8, x1 = 2. x0 = 1.8, x1 = 2. Then f (x0 ) = f (1.8) = −1.3 < 0 and

f (1.8) = (1.8)4 − (1.8) − 9 = 0.3024 f (1.9) = (1.9)4 − (1.9) − 9 = 2.1321 f(2) = (2)4 − (2) − 9 = 5

f (x1 ) = f (2) = 4 > 0 Since f (x0 ) and f (x1 ) are of opposite signs, f (x) = 0 has a root between x0 and x1 . The firstorder approximation of this root is given by x2 = =

There is a change of sign between 1.75 and 1.8. Hence, the root lies between 1.75 and 1.8. By Newton—Raphson method, we have

x0 f (x1 ) − x1f (x0 ) (1.8)(4) − 2(−1.3) = 4 − (−1.3) f (x1 ) − f (x0 ) 7.2 + 2.6 9. 8 = = 1.849 5.3 5. 3

⇒ f (x2 ) = (1.849)4 − 1.849 − 10 = −0.161

xi +1 = xi −

Since f (x) and f ′(x) have same sign at 1.8, we choose 1.8 as a starting point, that is x0 = 1.8. x1 = x0 −

Now f (x2 ) and f (x1 ) are of opposite signs. Hence, the root lies between x2 and x1 . The second-order approximation of the root is given by x3 = =

f (xi ) f’(xi )

= 1.8 −

f (x) f (1.8) = 1. 8 − f’(x) f ′(1.8) 0.3024 = 1.8 − 0.0135 = 1.7865 22.328

x1f (x2 ) − x2f (x1 ) (2)(−0.161) − 1.849(4) = f (x2 ) − f (x1 ) −0.161 − 4

The second approximation is given by

7.7182 = 1.8549 4.161

x2 = x1 −

⇒ f (x3 ) = (1.8549)4 − (1.8549) − 10 = −0.019

f (x1 ) (1.7865)4 − 1.7865 − 9 = 1.7865 − f ′(x1 ) 4(1.7865)3 − 1 0.6003 = 1.814 21.807

= 1.77865 + Now f (x3 ) and f (x2 ) are of same sign. Hence, the root does not lie between x2 and x3 . But f (x3 ) and f (x1 ) are of opposite signs. So the root lies between x3 and x1 and the third-order approximation of the root is x4 =

x2f (x3 ) − x3f (x2 ) f (x3 ) − f (x2 )

1.849(−0.019) − 1.8549(−0.161) −0.019 + 0.161 0.2635 = = 1.8557 0.142

The third approximation is given by x3 = x2 −

f (x2 ) (1.814)4 − 1.814 − 9 = 1.814 − f’(x2 ) 4(1.814)3 − 1

= 1.814 −

0.014 = 1.8134 22.8766

The fourth approximation is given by

=

9. Using Newton—Raphson method, find a positive root of x 4 − x − 9 = 0.

x4 = x3 −

f (x3 ) (1.8134)4 − 1.8134 − 9 = 1.8134 − f’(x3 ) 4(1.8134)3 − 1

= 1.8134 −

0.000303 = 1.8134 22.8529

Since x3 = x4 = 1.8134, the desired root is 1.8134. Solution:  Let f (x) = x 4 − x − 9. π /2

Now, f (0) = −9 < 0 f (1) = −9 < 0 f (2) = 5 > 0

Chapter 6.indd 332

10. Compute



sin x dx using Simpson’s three-eighth

0

rule of integration, taking h =

π . 18

8/22/2017 4:30:15 PM

SOLVED EXAMPLES      333

0

p 6

2p p = 6 3

3p p = 6 2

1 (y0)

1.6487 (y1)

2.3774 (y2)

2.71828 (y3)

Solution:  The values of f(x) = sin x are as follows: x

x

p 18

2p 18

3p 18

4p 18

0.17365

0.342020

0.5

0.64279

0

y = esinx f(x) 0 x

5p 18

6p 18

7p 18

8p 18

9p p = 18 2

f(x) 0.76604 0.86603 0.93969 0.984808

1

Here n = 3. Using trapezoidal rule, we get p /2



esin x dx =

h (y + y3 ) + 2(y1 + y2 ) 2 0

0

p [(1 + 2.71828) + 2(1.6487 + 2.3774)] 2 p = [(11.77048)] = 3.0815 2

By Simpson’s 3/8 rule,

=

p /2



3h  (y + y9 ) + 3 (y1 + y2 + y4 8  0

sin x dx =

0

+ y5 + y7 + y8 ) + 2 (y3 + y6 )

2

3p  =   (0 + 1) + 3 (0.17365 + 0.342020) 8 18  + 0.64279 + 0.76604 + 0.93969 + 0.984808) + 2(0.5 + 0.86603)]

13. Evaluate



dx

to three decimals, dividing x x + + 1 0 the range of integration into 8 equal parts. 2

Solution:  Dividing the interval (0, 2) into 8 equal parts, each of width 2−0 1 = 8 4

h= = 1.06589044 11. The table below shows the temperature f (t) as a function of time:

Therefore, the values of f (x) = 1/(x2 + x + 1) are as follows: x

t

1

2

3

4

5

6

7

f(t)

81

75

80

83

78

70

60

0

f(x) 1 x f(x)

7

Use Simpson’s 1/3 method to estimate



1/4

1/2

3/4

1

0.76190

0.57143

0.43243

0.3333

5/4

3/2

7/4

2

0.2623

0.215

0.17204

0.1429

f (t) dt.

1

By trapezoidal rule, 2

Solution:  Say f(t) = y. Also we have taken h = 1. By Simpson’s 1/3 rule,



dx

1 [(y + y8 ) + 2(y1 + y2 8 0 + y3 + y4 + y5 + y6 + y7 )

= 2

x + x +1

0

7

∫ f(t) dt = 3 [(y0 + y6 ) + 4(y1 + y3 + y5 ) + 2(y2 + y4 )] h

=

1

1 = [(81 + 60) + 4(75 + 83 + 70) + 2(80 + 78)] 3 1 1369 = [141 + 912 + 316 ] = = 456.3333. 3 3

1

12. Evaluate



esin x dx taking h = p /6.

0

Solution:  Let y = esin x . Given h = p /6. Length p  p of interval is  − 0 = . The table can be con 2  2 structed as follows:

Chapter 6.indd 333

∫ 1 + x dx 1

14. Evaluate p /2

1 [(1 + 0.1429) + 2(0.76190 + 0.57143 8 + 0.43243 + 0.2623 + 0.2105 + 0.17204 + 0.3333)] = 0.8288

by

0

(a) trapezoidal rule (b) Simpson’s 1/3 rule (c) Simpson’s 3/8 rule Solution:  We divide the interval [0, 1] into six (multiple of 3) subintervals. Let y = 1/(1 + x).

8/22/2017 4:30:26 PM

334     Numerical Methods 

By Picard’s method,

The table is constructed as follows:

x

x

0

1/6

2/6

3/6

1 y= 1+x

1 (y0)

x 1 1+x

y=

0.8571 (y1)

0.75 (y2)

0.6666 (y3)

4/6

5/6

1

0.6 (y4)

0.5454 (y5)

0.5 (y6)

(a) By trapezoidal rule 1

∫ 1 + x dx 1

x

y = y0 + ∫ f (x, y ) dx = y0 + ∫ x0

y −x dx y+x

(1)

0

For the first approximation, in the integrand on the R.H.S. of Eq. (1), y is replaced by its initial value which is 1. Therefore, y(1) = 1 + ∫

1− x dx = 1 + ∫ 1+ x

0

0

x

x

= 1 + −x + 2 log (1 + x)

  −1 + 2  dx  1 + x 

x 0

= 1 + −x + 2 log (1 + x) −  0 + 2 log (1 + 0)

0

h [(y + y6 ) + 2(y1 + y2 + y3 + y4 + y5 )] 2 0 1 = [(1 + 0.5) + 2(0.8571 + 0.75 + 0.6666 12 + 0.6 + 0.5454)] = 0.69485 =

(b) By Simpson’s 1/3 rule

= 1 — x + 2 log (1 + x)

(2)

For the second approximation, from Eq. (1), 1 − x + 2 log (1 + x) − x 2 y( ) = 1 + ∫ dx 1 − x + 2 log (1 + x) + x x

0 x

= 1+ ∫

1

∫ 1 + x dx 1

0

1 − 2x + 2 log (1 + x) 1 + 2 log (1 + x)

dx

0 x   2x  dx = 1 + ∫ 1 − 1 + 2 log (1 + x)   0 x x = 1 + x − 2∫ dx 1 + x log (1 + x)

h [(y + y6 ) + 4(y1 + y3 + y5 ) + 2(y2 + y4 )] 3 0 1 = [(1 + 0.5) + 4(0.8571 + 0.6666 + 0.5454) 18 + 2(0.75 + 0.6)] = 0.6931

=

0

(c) By Simpson’s 3/8 rule which is very difficult to integrate. 1

∫ 1 + x dx 1

Hence, we use the first approximation, Eq. (2), itself as the value of y.

0

3h [(y0 + y6 ) + 3(y1 + y2 + y4 + y5 ) + 2(y3 )] 8 3 = [(1 + 0.5) + 3(0.8571 + 0.75 + 0.6 (6)(8) + 0.5454) + 2(0.6666)] 11.0907 1 = [1.5 + 8.2575 + 1.33332] = 16 16 = 0.6932

y(x) = y(1) = 1 — x + 2 log (1 + x)

=

15. Find the value of y at x = 0.1 by Picard’s method, dy y −x = given that , y(0) = 1. dx y+x

Putting x = 0.1, we obtain y(0.1) = 1 − 0.1 + 2 log (1.1) — 1 — 0.1 + 0.1906203 = 1.0906204 = 1.0906 (Correct to 4 decimals) 16. Using Euler’s method, compute y(0.04) for the difdy ferential equation = −y; y (0) = 1. dx Take h = 0.01. Solution:  We have x0 = 0, y0 = 1, h = 0.01

Solution:  Here

Chapter 6.indd 334

f (x, y ) =

y −x , x = 0 and y0 = 1 y+x 0

and

dy = f (x, y ) = −y h = 0.01 dx

8/22/2017 4:30:32 PM

SOLVED EXAMPLES      335

Third approximation:

First approximation: y1 = y (0.01) = y0 + hf (x0 , y0 )

Second approximation: y2 = y (0.02) = y1 + hf (x1 , y1 ) = 0.99 + (0.01)(0.99) = 0.9999 Third approximation: = 0.9999 + (0.01)(−0.9999) = 1.009899 Fourth approximation: y4 = y (0.04) = y3 + h f (x3 , y3 ) = 1.009899 + (0.01)(−0.009899) = 1.01999799 17. Find y(0.1) using Euler’s modified formula given dy = x2 − y, y (0) = 1. that dx Solution:  We have dy = f (x, y ) = x2 − y, x0 = 0, y0 = 1 and h = 0.1 dx Now to find y1 = y(0.1), we have f (x0 , y0 ) = f (0, 1) = 0 − 1 = −1 Using Euler’s formula, we get y1

= y0 + hf (x0 , y0 ) = 1 + (0.1)(−1) = 0.9

Now x1 = 0.1 and

(

(0 )

f x1 , y1

) = f (0.1, 0.9) = (0.1)

2

− 0.9 = −0.89

First approximation: (1)

(

)

h (0 )  f (x0 , y0 ) + f x1 , y1    2 0. 1 0. 1  −1 + f (0.1, −0.89) = 1 + = 1+ (−1 + 0.9) 2 2 

y1 = y0 +

)

Fourth approximation:

y3 = y (0.03) = y2 + h f (x2 , y2 )

(0 )

(

h (2)  f (x0 , y0 ) + f x1 , y1  2 0. 1  = 1+ −1 + f (0.1, 0.90075) 2  0.1 = 1+ (−1 − 0.89075) = 0.90546 2

y1(3) = y0 +

= 1 + (0.01)(−1) = 0.99

(

)

(

)

h (3)  f (x0 , y0 ) + f x1 , y1  4 0. 1  = 1+ −1 + f (0.1, 0.90546) 2  0.1 = 1+ (−1 − 0.89546) 2 = −1 − 0.09477 = 0.90523

y1(4) = y0 +

Fifth approximation: h (4)  f (x0 , y0 ) + f x1 , y1  2 0. 1  = 1+ −1 + f (0.1, 0.90523) 2  0.1 = 1+ (−1 − 0.89523) 2 = −1 − 0.09476 = 0.90523

y1(5) = y0 +

Since y1(4) = y1(5), we have y1 = y(0.1) = 0.90523 18. Use Runge—Kutta method of fourth order, solve the following differential equation: dy y 2 − x2 with y(0) = 1 at x = 0.2, 0.4. = 2 dx y + x2 Solution:  Here, f (x, y ) =

y 2 − x2 y 2 + x2

, x0 = 0, y0 = 1

and h = 0.2

= 1 − 0.005 = 0.995 By Runge—Kutta method, Second approximation: h y1(2) = y0 + f (x0 , y0 ) + f x1 , y1(1)   2 0.1  = 1+ −1 + f (0.1, 0.995) 2  0.1 = 1+ (−1 − 0.985) = 1 − 0.09925 2 = 0.90075

(

Chapter 6.indd 335

)

k1 = hf (x0 , y0 ) = hf (0, 1) = 0.2  k  h k2 = hf x0 + , y0 = 1  = hf (0.1, 1.1) = 0.19672  2 2  k  h k3 = hf x0 + , y0 + 2  = hf (0.1, 1.09836) = 0.239279  2 2 k4 = hf (x0 + h, y0 + k3 ) = hf (0.2, 1.239279) = 0.29916

8/22/2017 4:30:40 PM

336     Numerical Methods 

Therefore, 1 ∆y = (k1 + 2k2 + 2k3 + k4 ) 6 1 = [ 0.2 + 2(0.19672) + 2(0.239279) + 0.29916 ] 6 = 0.228526

Therefore, 1 ∆y = (k1 + 2k2 + 2k3 + k4 ) 6 1 = [ 0.1 + 2(0.11525) + 2(0.116857) + 0.134737 ] 6 = 0.116492

and y1 = y(0.2) = y0 + ∆y = 1 + 0.228526 = 1.228526

Thus,

Also x1 = x0 + h = 0.2, y1 = 1.228526 and h = 0.2

x1 = x0 + h = 0.1.

Again, we calculate

Now, x1 = 0.1, y1 = 1.116492 and h = 0.1

k1 = hf (x1 , y1 ) = hf (0.2, 1.228526) = 0.19045

Again,

 k  h k2 = hf x1 + , y1 + 1  = hf (0.3, 1.32375)  2 2 = 0.18046  k  h k3 = hf x1 + , y1 + 2  = hf (0.3, 1.31876)  2 2 = 0.180318 k4 = hf (x1 + h, y1 + k3 ) = hf (0.4, 1.40884) = 0.170161 Therefore, 1 (k + 2k2 + 2k3 + k4 ) 6 1 1 = [ 0.19045 + 2(0.18046) + 2(0.180318) 6 +0.170161]

∆y =

y1 = y (0.1) = y0 + ∆y = 1.116492

(

and

)

k1 = hf (x1 , y1 ) = h x1 + y12 = 0.134655 2   k  k   h h  k2 = hf x1 + , y1 + 1  = h  x1 + + y1 + 1    2 2 2  2    = 0.155143 2   k  k   h  h k3 = hf x1 + , y1 + 2  = h  x1 + + y1 + 2    2 2  2  2   = 0.157579 2 k4 = hf (x1 + h, y1 + k3 ) = h  x1 + h + (y1 + k3 )    = 0.1823257 Therefore, 1 (k + 2k2 + 2k3 + k4 ) 6 1 1 = [0.134655 + 2(0.155143) 6 + 2(0.157579) + 0.1823257 ]

∆y =

= 0.18036115 y2 = y (0.4) = y1 + ∆y = 1.40889.

= 0.15707 dy = x + y 2 , use Runge—Kutta method of fourth 19. If dx order to find an approximate value of y for x = 0.2 given that y = 1 when x = 0. (Take h = 0.1.) Solution:  We are given that f (x, y ) = x + y 2 , x0 = 0, y0 = 1 and h = 0.1

Hence, y2 = y (0.2) = y1 + ∆y = 1.273562 x2 = x1 + h = 0.2.

and

20. Given y ′ = x2 − y, y (0) = 1, find correct to four decimal places the value of y (0.1) , by using Euler’s method.

By Runge—Kutta method, we have

(

)

k1 = hf (x0 , y0 ) = h x0 + y02 = 0.1 2   k1  k1   h h    k2 = hf x0 + , y0 +  = h  x0 + + y0 +    2  2  2 2   = 0.11525 2   k  k   h h  k3 = hf x0 + , y0 + 2  = h  x0 + + y0 + 2    2 2 2  2    = 0.116857 2 k4 = hf (x0 + h, y0 + k3 ) = h  x0 + h + (y0 + k3 )    = 0.134737

Chapter 6.indd 336

Solution:  We are given that x0 = 0, y0 = 1, h = 0.1 and f (x, y ) = x2 − y. By Euler’s method, we have yn +1 = yn + hf (xn , yn ) Therefore, y1 = y(0.1) = y0 + hf (x0 , y0 ) = 1 + (0.1) f (0, 1) = 1 + (0.1)(0 − 1) = 1 − 0.1 = 0.9 Hence, the solution is y (0.1) = 0.9.

8/22/2017 4:30:51 PM

SOLVED EXAMPLES      337

21. While measuring a certain parameter, the mean was determined to be 25. The measurements were repeated four times and the mean in the other three measurements were observed to be 26, 24 and 27. Determine the resultant value. (a) 25 ± 1

Normal equation for a is ∑y = na + b∑x, where n is number of observations = 5. Normal equation for b is ∑xy = a∑x + b∑x2. Now ∑x = 15, ∑x2 = 55, ∑xy = 88 and ∑y = 25. Therefore, the normal equations become 5a + 15b = 25 and 15a + 55b = 88 Solving these equations simultaneously, we get a = 1.1 and b = 1.3. Therefore, equation of least square line is y = 1.1 + 1.3x.

(b) 25 ± 2 (c) 25.5 ± 1 (d) 25.5 ± 0.6 Solution:  The deviations of mean in the four measurements are 0, 1, 1 and 2. Sum of squares of these deviations is 6. The standard deviation is 6 / 4 = 1.22. The mean is 25.5; standard deviation of the mean is 1.22/ 4 = 0.61. The result therefore is 25.5 ± 0.6. 22. A certain velocity was measured by measuring displacement x over a period of time t. The two quantities were measured as 5.1 ± 0.4 m and 0.4 ± 0.1  s. What would be the uncertainty in the velocity?

25. In equation obtained in Question 24, if y are the trend values, then ∑(y − y ) would be equal to Solution:  Trend values are 2.4 (for x = 1), 3.7 (for x = 2), 5.0 (for x = 3), 6.3 (for x = 4) and 7.6 (for x = 5). Corresponding values of (y − y ) are −0.4, +1.3, −2, +1.7 and −0.6. Therefore, ∑ y − y = −0.4 + 1.3 − 2 + 1.7 − 0.6 = 0. 26. If a second-degree parabola, defined by y = a + bx + cx2, is to be made to fit to the below-mentioned data, then determine the expression for best-fit curve.

Solution:  We have that x Velocity, v = t Therefore, uncertainty in velocity  dx 2  dt 2  0.4 2  0.1 2 dv =   +   =   +    x   t   5.1   0.4  = 3.34 m/s 23. An object is released from rest in a free fall. At a certain point during its free fall, its velocity was measured as 5 ± 0.2 m/s. How long it has been in free fall given that g = 9.81 m/s2?

x

0

1

2

3

4

y

1

1.8

1.3

2.5

6.3

Solution:  The parabola of fit is given by y = a + bx + cx2. Assume the following substitution to simplify ­calculations u = x − 2 and v = y . The equation of parabola becomes v = A + Bu + Cu2. The normal equations are ∑v = 5A + B ∑u + C  ∑u2

which gives 5A + 10C = 12.9 (1) 2 (∑u and ∑u can be computed from given data). Solution:  Time t is expressed as t = −v/g = −(1/g) × v t = −v/g = −(1/g) × v ; g is constant. ∑uv = A∑u + B ∑u2 + c∑u3 Therefore, which gives 10B = 11.3 (2) dt = −

1 1 × dv = × 0.2 = 0.0.0204 ≅ 0.02 9.81 g

24. Determine the equation of the desired straight line, when a straight line given by y = a + bx, is fitted to the below-mentioned data using method of least squares.

which gives 10A + 34C = 33.5 (3) Now B can be computed as B = 11.3/10 = 1.13. A and C can be determined by simultaneous solution of Eqs. (1) and (3). Multiplying Eq. (1) by 2, we get

x

1

2

3

4

5



y

2

5

3

8

7

Subtracting Eq. (4) from Eq. (3), we get

Solution:  The equation of least square line is y = a + bx.

Chapter 6.indd 337

∑u2v = A∑u2 + B ∑u3 + C  ∑u4

10A + 20C = 25.8

14C = 7.7; C =

(4)

7. 7 = 0.55 14

8/22/2017 4:30:55 PM

338     Numerical Methods 

Substituting the value of C in Eq. (3), we get 10A + 34 × 0.55 = 33.5 33.5 − 34 × 0.55 = 1.48 10 Substituting the values of A, B and C, we get A=

v = 1.48 + 1.13u + 0.55u2 Now u = x − 2 and v = y. Therefore,

L5 (x) =

(0.5)(0.2)(−0.1)(−0.4) 40 × 10−4 =− = 0.020 (1.2)(0.9)(0.6)(0.3) 1944 × 10−4

f(1.5) = sin(4.5) ≅ −0.9975 P (1.5) = −0.1411 × 0.0288 − 0.6878 × 0.288 − 0.9962 × 0.864 − 0.5507 × (−0.144) + 0.3115 × 0.020 ≅ −0.0040 − 0.1980 − 0.8607 + 0.0793 + 0.0062

y = 1.48 + 1.13 (x − 2) + 0.55 (x − 2)2 = 1.48 + 1.13x − 2.26 + 0.55x2 − 2.2x +2.2 = 1.42 − 1.07x + 0.55x2 27. Consider the following data: X

1

1.3

1.6

1.9

2.2

f(x) 0.1411 −0.6878 −0.9962 −0.5507 −0.3115 where f(x) = sin 3x.

≅ −0.9772 Therefore, P(1.5) very closely approximates f(1.5). In fact in the interval x = 1 to 2.2, P (x) approximates f(x). 28. The following data give the velocity of a particle for 20 s at an interval of 5 s. Find the initial acceleration using the entire data.

Use Lagrange polynomials to estimate f(1.5) Solution:  As a first step, we find Lagrange polynomials Lk(x) for k = 1, 2, 3, 4 and 5. L1(x) =

(x − 1.3)(x − 1.6)(x − 1.9)(x − 2.2) (1 − 1.3)(1 − 1.6)(1 − 1.9)(1 − 2.2)

L2 (x) =

(x − 1)(x − 1.6)(x − 1.9)(x − 2.2) (1.3 − 1)(1.3 − 1.6)(1.3 − 1.9)(1.3 − 2.2)

L3 (x) =

(x − 1)(x − 1.3)(x − 1.9)(x − 2.2) (1.6 − 1)(1.6 − 1.3)(1.6 − 1.9)(1.6 − 2.2)

(x − 1)(x − 1.3)(x − 1.6)(x − 2.2) (1.9 − 1)(1.9 − 1.3)(1.9 − 1.6)(1.9 − 2.2) (x − 1)(x − 1.3)(x − 1.6)(x − 1.9) L5 (x) = (2.2 − 1)(2.2 − 1.3)(2.2 − 1.6)(2.2 − 1.9)

Time (t) in seconds

0

5

10

15

20

Velocity (v) in m/s

0

3

14

69

228

Solution:  The difference table can be drawn from the given data as follows: t

v

0

0

5

3

∆v

∆2v

∆3v

∆4v

3

L4 (x) =

8 11

10

14

36 44

55 15

69

24 60

104 159

Now 20

228

P (x) = 0.1411 × L1(x) − 0.6878 × L2 (x) − 0.9962 × L3 (x) − 0.5507 × L4 (x) + 0.3115 × L5 (x) For x = 1.5, (0.2)(−0.1)(−0.4)(−0.7) 56 ×110−4 L1(x) = =− (−0.3)(−0.6)(−0.9)(−1.2) 1944 × 10−4 = −0.0288 L2 (x) =

(0.5)(−0.1)(−0.4)(−0.7) 140 ×110−4 =− = 0.288 (0.3)(−0.3)(−0.6)(−0.9) 486 × 10−4

L3 (x) =

(0.5)(0.2)(−0.4)(−0.7) 280 × 10−4 =− = 0.864 (0.6)(0.3)(−0.3)(−0.6) 324 × 10−4

L4 (x) =

(0.5)(0.2)(−0.1)(−0.7) 70 × 10−4 =− = −0.144 (0.9)(0.6)(0.3)(−0.3) 486 × 10−4

Chapter 6.indd 338

The aim is to find initial acceleration, that is, dv/dt at t = 0. Therefore, we will use Newton’s forward formula.  dv   1 1 1 1   = ∆v0 − ∆2 v0 + ∆3 v0 − ∆4 v0 +   dt  t = 0   h 2 3 4 1 8 36 24  3 − + −  5  2 3 4  1 = (3 − 4 + 12 − 6) = 1 m/s 5 =

29. In a given machine, a slider moves along a straight rod. The distance x that it moves along the rod for different values of t is given below. Find the acceleration and velocity of the slider when t = 0.3 s.

8/22/2017 4:31:00 PM

SOLVED EXAMPLES      339

t

x

0.0

30.13

0.1

31.62

∆2x

∆x

∆3x

∆4x

∆5x

∆6x

1.49 — 0.24 1.25 0.2

— 0.24

32.87

— 0.48 0.77

0.3

33.64

— 0.01

0.29

0.01

33.95

0.02

— 0.45 — 0.14

0.5

— 0.27

— 0.46 0.31

0.4

0.26 0.02

0.01 0.02

33.81

— 0.43 — 0.57

0.6

0.1

33.24

t

0

x

30.13 31.62 32.87 33.64 33.95 33.81 33.24

Solution:  The follows:

0.2

0.3

difference

0.4

table

0.5

is

0.6

given

as

Substituting these values in expressions for velocity and acceleration, we get   dx  1  0.31 + 0.77 1  0.01 + 0.02  1  0.02 − 0.27     = −   −   +      dt  0.3     0.1  2 6 2 30 2

  dx  31 + 0.77 1  0.01 + 0.02  1  0.02 − 0.27  Since velocity and acceleration are 1to 0.be deter  = −   −   +      0.3table,   30  0.1 we 2 use 6  2 2 mined near the middle of dt the  will  Stirling’s formula. = 5.33 3 3  dx  1  ∆x + ∆x−1  1  ∆ x−1 + ∆ x−2  −    =  0   6   dt  t h  2 2 0   5 5 1  ∆ x−2 + ∆ x−3  +   + 30  2  

 d2x   1  1 1   −0.46 − (−0.01) + (0.29) −  2 = 2   12 90  dt  0.3 (0.1)  = −45.6

 d2x   1  1 1 6 ∆ x−3  and   2  = 2 ∆2 x−1 − ∆4 x−2 +  12 90  dt  t0 h 

30. Obtain Newton’s forward interpolating polynomial P5(x) for the following tabular data. What would be the interpolated value of the function at x = 0.0045? X

0

0.001

0.002

0.003

0.004

0.005

Y

1.121

1.123

1.1255 1.127

1.128

1.1285

It is given that h = 0.1, t0 = 0.3. From the difference table, ∆x0 = 0.31, ∆x−1 = 0.77, ∆2x−1 = −0.46, ∆3x−1 = 0.01, ∆3x−2 = 0.02, ∆5x−2 = 0.2 and ∆5x−3 = 0.2

xi

yi

0

1.121

∆yi 0.002

∆2yi

Solution:  The forward difference table is drawn as follows: ∆3yi

0.005

∆4yi

∆5yi

0.002

−0.0025

−0.0015

Chapter 6.indd 339

0.001

1.123

0.0025

−0.0010

0.0005

0.002

1.1255

0.0015

−0.0005

0.0

0.003

1.127

0.001

−0.0005

0.004

1.128

0.005

0.005

1.1285

−0.0005

8/22/2017 4:31:02 PM

340     Numerical Methods 

Now p =

x − x0 0.0045 − 0 = = 4.5 0.001 h

= 1.121 + 0.002 × 4.5 + ×3.5 −

(4.5)(4.5 − 1) P5 (x) = 1.121 + 0.002 × 4.5 + × 0.0005 2! 4.5(4.5 − 1)(4.5 − 2) × (−0.0015) + 3!

0.0005 × 4.5 2

0.0015 × 4.5 × 3.5 × 2.5 6

0.002 0.0025 × 4.5 × 3.5 × 2.5 × 1.5 − × 4.5 24 120 ×3.5 × 2.5 × 1.5 × 0.5 +

4.5(4.5 − 1)(4.5 − 2)(4.5 − 3) × (0.002) 4! 4.5(4.5 − 1)(4.5 − 2)(4.5 − 3)(4.5 − 4) × (−0.0025) + 5! +

= 1.12840045

PRACTICE EXERCISE 1. Solve the following systems of linear equation: x1 + x2 + x3 = 1 4x1 + 3x2 − x3 = 6 3x1 + 5x2 + 3x3 = 4

(c) x = −2, y = 1, z = −3 (d) x = −1, y = −1, z = −4 5. Solve the following system of equations by using Gauss—Jacobi method (correct to three decimal places):

(a) x1 = 3/2, x2 = 1/2, x3 = − 1 8x − 3y + 2z = 20; 4x + 11y − z = 33; 6x + 3y + 12z = 35 (b) x1 = 1, x2 = 1/2, x3 = − 1/2 (c) x1 = 1, x2 = 1, x3 = − 81/2 x − 3y + 2z = 20; 4x + 11y − z = 33; 6x + 3y + 12z = 35 (d) x1 = 1, x2 = 2, x3 = − 1/2

(a) x = 3.102, y = 2.122, z = 1.052 2. Solve the following system using Crout’s decomposition method: 3x − y + 2z = 12, x + 2y + 3z = 11, 2x − 2y − z(b) = 3x. = 3.123, y = 2.102, z = 0.901 z = 12, x + 2y + 3z = 11, 2x − 2y − z = 3. (c) x = 2.999, y = 1.912, z = 1.101 (a) x = 3, y = 1, z = 2 (b) x = 3, y = 1, z = 2 (c) x = 3, y = 1, z = 2 (d) x = 3, y = 1, z = 2 3. Solve the following system of equations using Gauss elimination method:

(d) x = 3.016, y = 1.986, z = 0.912 6. Find a real root of the equation x3 − 6x − 4 = 0 by bisection method. (a) 2.0575 (c) 3.0252

(b) 2.71875 (d) 2.85578

7. Find a root of the equation x3 − 4x − 9 = 0 by 10a − 7b + 3c + 5d = 6, − 6a + 8b − c − 4d = 5, 3a + b + 4c + 11d = 2, 5a − 9b − 2c + 4d = 7 bisection method correct to three decimal places. 6, − 6a + 8b − c − 4d = 5, 3a + b + 4c + 11d = 2, 5a − 9b − 2c + 4d = 7 (a) 2.706 (b) 2.890 (c) 2.656 (d) 2.901 (a) a = 2, b = 4, c = −1, d = 1 (b) a = 1, b = 3, c = 2, d = 1 8. Use Regula—Falsi method to find the smallest posi(c) a = 5, b = 4, c = −7, d = 1 tive root of the following equation correct to four (d) a = 3, b = 4, c = −7, d = −1 significant digits x3 − 5x + 1 = 0. 4. Use Gauss—Seidel iterative method to solve the following system of simultaneous equation: 9x + 4y + z = −17; x − 2y − 6z; x + 6y = 4 Perform four iterations. (a) x = 2, y = −3, z = −2 (b) x = −2, y = 1, z = −2

Chapter 6.indd 340

(a) 0.1626 (c) 0.1832

(b) 0.2016 (d) 0.2805

9. Find an approximate root of x log10 x — 1.2 = 0 by Regula—Falsi method. (a) 2.1550 (c) 2.4022

(b) 3.0125 (d) 2.7405

8/22/2017 4:31:10 PM

PRACTICE EXERCISE     341

10. Find the real root of the equation x log10 x = 4.77 correct to four decimal places using Newton— Raphson method. (a) 5.5152 (c) 6.0831

(b) 4.4784 (d) 7.0125

11. Find the real root of the equation 3x = cos x + 1 using Newton—Raphson method. (a) 0.5887 (c) 0.6071

(b) 0.6255 (d) 0.7015 1.4

x ∫ (sin x − log x = e ) dx

12. Compute the value of

0.2

using Simpson’s 3/8 rule. (a) 4.053 (c) 5.052

(b) 3.755 (d) 2.784 0.6

13. Compute the value of

−x2

∫e

dx using Simpson’s

0

1/3 rule by taking seven ordinates. (a) 0.6012 (c) 0.1255

(b) 0.5351 (d) 0.4292

2

14. Evaluate ∫

dx by Simpson’s 1/3 rule with four strips. x

1

(a) 0.55253 (c) 0.59874

(b) 0.72456 (d) 0.69326

6

∫ 1 + x dx approximately using Simpson’s e

15. Evaluate

x

0

19. Given that y ′ = 1 − y, y (0) , find the value of y(0.2) using modified Euler’s method. (a) 0.07111 (c) 0.18098

(b) 0.22345 (d) 0.56522

20. Find y(1) and y(2) using Runge—Kutta fourthorder formula y ′ = x2 − y and y(0) = 1. (a) 0.8213 (c) 0.5212

(b) 0.9052 (d) 1.2428

21. Five measurements were made of the weight of certain article. The results obtained were 9.8 kg, 9.9  kg, 10.2 kg, 10.1 kg and 9.9 kg. If the true weight is 10  kg, then the measurement can be termed as (a) accurate and precise (b) accurate but lacking precision (c) inaccurate but precise (d) inaccurate and imprecise 22. In the above question, if the true value of the weight were 12 kg, then the measurement can be termed as (a) accurate and precise (b) precise but inaccurate (c) inaccurate and imprecise (d) accurate but imprecise 23. The uncertainty in a certain measurement was observed to be 5% (relative error). The measurement was repeated 25 times. The uncertainty would now be (assume errors are random)

3/8 rule on integration. (a) 0.653556 (c) 0.801256

(b) 0.553313 (d) 0.701652

2.0

16. Evaluate

∫ y dx

using trapezoidal rule.

(a) 5% only (b) 0.2% (c) 1% (d) None of these 24. While measuring a certain length, whose true value is 100 m, the measured value was observed to be 98 m. Relative error in this case is

0.6

(a) 8.112 (c) 9.845

(b) 10.416 (d) 1.565 6

∫ 1 + x2 dx

17. Evaluate

(a) 2 m (b) 0.02 (c) Indeterminate from given data (d) None of these

by using trapezoidal rule. 25. Which one of the following error types is random?

0

(a) 5.1414 (b) 3.4422 (c) 1.4108 (d) 2.3432 dy 18. If = 1 + y 2 , y (0) = 1, find y(0.4) by using dx Euler’s method. Take h = 0.02. (a) 1.992 (c) 2.014

Chapter 6.indd 341

(b) 1.564 (d) 1.664

(a) E  rrors in the calibration of measuring instruments (b) Parallax error (c) Bias of the person making measurement (d) AC noise causing voltmeter needle to fluctuate 26. An experiment resulted in the standard deviation of the quantity under measurement to be equal to 0.2.

8/22/2017 4:31:15 PM

342     Numerical Methods 

The experiment was repeated 16 times. The standard deviation of the mean is equal to (a) 3.2 (b) 0.0125 (c) Indeterminate from given data since mean value is not given (d) 0.05

(a) y = 1.1 − 14x (b) y = 14 − 1.1x (c) y = 14 + 1.1x (d) y = −1.1 + 14x 30. If f(x) is known at the following data points, find f(0.5) using Newton’s forward different formula.

27. A quantity R depends upon quantities X, Y and Z. The value of R is given by R = X + Y − Z. The uncertainty dR in the value of R in terms of uncertainties dX, dY and dZ, respectively, in the values of X, Y and Z is given by

2

2

2

2

2

2

1

2

3

4

fi

1

7

23

55

109

31. If it is desired to find derivative of a function for a given data near the beginning of the table, one would preferably employ

(c) (dR ) = (dX ) + (dY ) − (dZ ) 2

0

(a) 3.125 (b) 3.215 (c) 2.315 (d) 2.325

(a) dR = dX + dY − dZ (b) (dR ) = (dX ) + (dY ) + (dZ )

xi

2

(d) dR = dX + dY + dZ

(a) Stirling’s formula (b) Newton’s backward formula (c) Newton’s forward formula (d) None of these

28. The starting and finishing positions in a 100 m race were measured as 0 ± 0.05 m and 100 ± 0.15 m. The error in the measured track length would be (a) 0.151 m (b) 0.2 m (c) Indeterminate from given data (d) None of these

32. A function f(x) is defined for five different values of (x) at x = 1, 2, 3, 4 and 5. It is desired to determine f ′(x) at x = 3. One of the following formulas would preferably be used for computation:

29. Use least square method to determine the equation of the line of best fit for the below given data. x

8

2

11

6

5

4

12

9

6

1

y

3

10

3

6

8

12

1

4

9

14

(a) Stirling’s formula (b) Newton’s backward formula (c) Newton’s forward formula (d) None of these

ANSWERS 1. (b)

8. (b)

15. (d)

22. (b)

29. (b)

2. (a)

9. (d)

16. (b)

23. (c)

30. (a)

3. (c)

10. (c)

17. (c)

24. (b)

31. (c)

4. (c)

11. (c)

18. (a)

25. (d)

32. (a)

5. (d)

12. (a)

19. (c)

26. (d)

6. (b)

13. (b)

20. (d)

27. (b)

7. (a)

14. (d)

21. (a)

28. (a)

EXPLANATIONS AND HINTS 1. (b) We choose uii = 1 and write A = LU

Chapter 6.indd 342

1 1 1   l11 0     4 3 −1 = l21 l22 3 5 3     l31 l32

0   1 u12  0  0 1  l33   0 0

u13   u23   1 

8/22/2017 4:31:17 PM

EXPLANATIONS AND HINTS     343

 l11 l11u12  = l21 l21u12 + l22  l31 l31u12 + l32

 l11u13  l21u13 + l22 u23   l31u13 + l32 u23 + l33 

l21u12 = −1 ⇒ u12 = l11u13 = 2 ⇒ u13 =

−1 3

2 3

By equating, we get l21u12 + l22 = 2 ⇒ l22 =

l11 = 1, l21 = 4, l31 = 3 and l11u12 = 1 ⇒ u12 = 1, l11u13 = 1 ⇒ u13 = 1 l21u12 + l22 = 3 ⇒ 4 + l22 = 3,

7 3

l31u12 + l32 = −2 ⇒ l32 =

l22 = 3 ⇒ l22 = −1

−4 3 =1

l21u13 + l22 u23 = −1 ⇒ 4 + (−1)u23 = −1 ⇒ u23 = 5

l21u13 + l22 u23 = 3 ⇒ u23

l31u12 + l32 = 5 ⇒ 3 + l32 = 5 ⇒ l32 = 2

l31u13 + l32 u23 + l33 = −1 ⇒ l33 = −1

l31u13 + l32 u23 + l33 = 3 ⇒ 3 + 2.5 + l33 = 3 ⇒ l33 = −10

Thus, we get 3 0 0   1 −1/3 2/3    A = LU = 1 7/3 0  0 1 1  2 −4/3 −1  0 0 1    

1 0 0   1 1 1    Thus, we get A = LU =  4 −1 0   0 1 5 3 2 −10  0 0 1     The given system is AX = B ⇒ LUX = B .

The given system is AX = B ⇒ LUX = B. Let UX = Y , so LY = B .

(1)

Let UX = Y. So from Eq. (1), we have

3 0 0   y1  12      ⇒ 1 7/3 0   y2  = 11 2 −4/3 −1    2     y3   

1 0 0   y1  1      LY = B ⇒  4 −1 0   y2  = 6 3 2 −10    4    y3    By solving, we get y1 = 1

By solving, we get 3y1 = 12 ⇒ y1 = 4

4y1 − y2 = 6 ⇒ y2 = −2

7 y = 11 ⇒ y2 = 3 3 2 4 2y1 − y2 − y3 = 2 ⇒ y3 = 2 3 y1 +

−1 3y1 + 2y2 − 10y3 = 4 ⇒ y3 = 2 Now,  1 1 1  x1   1       UX = Y ⇒  0 1 5  x2  =  −2   0 0 1   −1 / 2    x3   

Now,

By solving, we get −1 x3 = 2 5 1 x2 + 5x3 = −2 ⇒ x2 − = −2 ⇒ x2 = 2 2 1 1 x1 + x2 + x3 = 1 ⇒ x1 + − = 1 ⇒ x1 = 1 2 2 Hence, the solution of the system of equations is 1 1 x1 = 1, x2 = and x3 = − . 2 2 2. (a) We choose uii = 1 and write

A = LU 0 0   1 u12  l22 0   0 1  l32 l33   0 0

3 −1 2   l11    3  = l21 1 2 2 −2 −1    l31  l11 l11u12  = l21 l21u12 + l22  l31 l31u12 + l32

u13   u23   1 

 l11u13  l21u13 + l22 u23   l31u13 + l32 u23 + l33 

By equating, we get l11 = 3, l21 = 1, l31 = 2 and

Chapter 6.indd 343

 1 −1/3 2/3  x  4      1 1   y  =  3 UX = Y ⇒  0 0 0 1   z  2 

By solving, we get z = 2 y +z = 3 ⇒ y =1 y 2 + z=4⇒x=3 3 3 Hence, the solution is x = 3, y = 1, z = 2. x−

3. (c) We have

10a − 7b + 3c + 5d = 6

(1)



− 6a + 8b − c − 4d = 5

(2)



3a + b + 4c + 11d = 2

(3)



5a − 9b − 2c + 4d = 7

(4)

 −3  To eliminate a, let us perform Eq. (2) −   Eq . (1),  5  3 1 Eq. (3) −   Eq . (1) and Eq . (4) −   Eq . (1).  2  10  Therefore, we get

8/22/2017 4:31:30 PM

344     Numerical Methods 



3.8b + 0.8c − d = 8.6

(5)



3.1b + 3.1c + 9.5d = 0.2

(6)



−5.5b − 3.5c + 1.5d = 4

(7)

z(1) =

1 [−1.8888 − 2(0.9815) − 14] = −2.9753 6

The first iteration is given by x(1) = −1.8888

 3. 1  To eliminate b, let us perform Eq. (6) −   Eq. (5)  3.8 

y(1) = 0.9815 z(1) = −2.9753

 −5.5  and Eq. (7) −  Eq. (5) . Therefore, we get  3.8 

The second iteration is given by

1 (−17 − 4y(1) − z(1) ) = −1.9945 9 (9) −2.3421053c + 0.0526315d = 16.447368 1 y(2) = (4 − x(2) ) = 0.9991 6  −2.3421053  To eliminate c, let us perform Eq. (9) −  Eq. (8).  1  2.4473684  z(2) = (x(2) − 2y(2) − 14) = −2.9988 6  −2.3421053  Eq. (9) −  Eq. (8). Therefore, we get  2.4473684  The third iteration is given by

2.4473684c + 10.315789d = −6.8157895

(8)

x(2) =



9.9249319d = 9.9245977

(10)

x(3) =

From Eq. (10), we have d=1 Putting the value of d in Eq. (9), we get c = −7 Putting the value of c and d in Eq. (7), we get b=4 Putting the value of b, c and d in Eq. (4), we get a = 5. 4. (c) The given system is not diagonally dominant. Hence, rearranging the equations as

y(3)

z(3) = (x(3) − 2y(3) − 14) = −2.9999 The fourth iteration is given by 1 x(4) = (−17 − 4y(3) − z(3) ) = −1.9999  −2; 9 1 y(4) = (4 − x(4) ) = 0.9999 = 1 6 z(4) = (x(4) − 2y(4) − 14) = −2.9999 = −3 Hence, after four x = −2, y = 1, z = −3.

9x + 4y + z = −17; x + 6y = 4; x − 2y = 14 Now, Gauss—Seidel’s iterative method can be applied.



From the above equations, we get 1 x = (−17 − 4y − z ) 9 1 y = (4 − x ) 6 1 z = (x − 2y − 14) 6

1 (−17 − 4y(2) − z(2) ) = −1.9997 9 1 = (4 − x(3) ) = 0.9999 6

(1)

we

obtain

5. (d) Since the diagonal elements are dominant in the coefficient matrix, we write x, y, z as follows:

(2) (3)

iterations,

1 (20 + 3y − 2z) 8 1 y = (33 − 4x + z) 11 1 z = (35 − 6x − 3y) 12 x=

(1) (2) (3)

Gauss—Jacobi method: Starting with y = 0 = z, we obtain 17 1 x( ) = − = −1.8888 9 Now, putting x = −1.888 and z = 0 in Eq. (2), we get y(1) =

1 [4 − (−1.8888)] = 0.9815 6

Again putting x = −1.888 and y = 0.9815 in Eq. (3), we get

Chapter 6.indd 344

Let the initial values be x = 0, y = 0, z = 0 Using the values x = 0, y = 0, z = 0 in Eqs. (1), (2), (3), we get the first iteration as 1 x(1) = [20 + 3(0) − 2(0)] = 2.5 8 1 (1) y = [33 − 4(0) + 0 ] = 3.0 11 1 z(1) = [35 − 6(0) − 3(0)] = 2.916666 12

8/22/2017 4:31:42 PM

EXPLANATIONS AND HINTS     345

The second iteration is given by The ninth iteration is given by 1 1 x(2) = [20 + 3(3.0) − 2(2.916666)] = 2.895833 x(9) = [20 + 3(1.986535) − 2(0.911644)] = 3.017039 8 8 1 (2) 1 y = [33 − 4(2.5) + (2.916666)] = 2.356060 y(9) = [33 − 4(3.016946) + (0.911644)] = 1.985805 11 11 1 (2) 1 z = [35 − 6(2.5) − 3(3.0)] = 0.916666 z(9) = [35 − 6(3.016946) − 3(1.986335)] = 0.911560 12 12 The third iteration is given by The tenth iteration is given by 1 (3) x = [20 + 3(2.356060) − 2(0.916666)] = 3.154356 1 8 x(10) = [20 + 3(1.985805) − 2(0.911560)] = 3.016786 8 1 y(3) = [33 − 4(2.895833) + (0.916666)] = 2.030303 1 11 y(10) = [33 − 4(3.017039) + (0.911560)] = 1.985764 11 1 z(3) = [35 − 6(2.895833) − 3(2.356060)] = 0.879735 1 12 z(10) = [35 − 6(3.017039) − 3(1.985805)] = 0.911696 12 The fourth iteration is given by 1 In 8th, 9th and 10th iteration, the values of x, y and x(4) = [20 + 3(2.030303) − 2(0.879735)] = 3.041430 8 z are same correct to three decimal places. Hence, 1 (4) we stop at this level. y = [33 − 4(3.154356) + (0.879735)] = 1.932937 11 6. (b) Let 1 (4) z = [35 − 6(3.154356) − 3(2.030303)] = 0.831913 f (x) = x3 − 6x − 4 12 Now, The fifth iteration is given by f (2) = 8 − 12 − 4 = −8 < 0, f (3) = 27 − 18 − 4 = 5 > 0 1 x(5) = [20 + 3(1.932937) − 2(0.831913)] = 3.016873 8 f (2) = 8 − 12 − 4 = −8 < 0, f (3) = 27 − 18 − 4 = 5 > 0 1 (5) y = [33 − 4(3.041430) + (0.831913)] = 1.969654 Therefore, one root lies between 2 and 3. Consider 11 x0 = 2 and x1 = 3. By bisection method, the next 1 (5) approximation is z = [35 − 6(3.041430) − 3(1.932939)] = 0.912717 12 x + x1 The sixth iteration is given by x2 = 0 = 2.5 ⇒ f (x2 ) = (2.5)3 − 6(2.5) − 4 = −3.375 < 0 1 2 x(6) = [20 + 3(1.969654) − 2(0.912717)] = 3.010441 x + x1 8 x2 = 0 = 2.5 ⇒ f (x2 ) = (2.5)3 − 6(2.5) − 4 = −3.375 < 0 and f(3) > 0. 2 1 (6) y = [33 − 4(3.016873) + (0.912717)] = 1.985930 Therefore, the root lies between 2.5 and 3. The 11 next approximation x3 is given by 1 z(6) = [35 − 6(3.016873) − 3(1.969654)] = 0.915817 2.5 + 3 5.5 12 x3 = = = 2.75 ⇒ f (x3 ) = (2.75)3 − 6(2.75) − 4 = 0.2968 > 2 2 The seventh iteration is given 2.5by+ 3 5.5 x = = = 2.75 ⇒ f (x3 ) = (2.75)3 − 6(2.75) − 4 = 0.2968 > 0 and f(2.5) < 0. 3 1 (7) 2 2 x = [20 + 3(1.985930) − 2(0.915817)] = 3.015770 8 Therefore, the root lies between 2.5 and 2.75. The 1 next approximation x4 is given by (7) y = [33 − 4(3.010441) + (0.915817)] = 1.988550 11 2.5 + 2.75 5.25 x4 = = = 2.625 ⇒ f (x4 ) = (2.625)3 − 6(2.625) − 4 = − 1 z(7) = [35 − 6(3.010441) − 3(1.985930)] = 0.914964 2 2 12 2.5 + 2.75 5.25 x 2.625 ⇒ f (x4 ) = (2.625)3 − 6(2.625) − 4 = −1.6621 < 0 and f(2.75) > 0. 4 = iteration is=given = The eighth 2 2 by Therefore, the root lies between 2.625 and 2.75. 1 x(8) = [20 + 3(1.988550) − 2(0.914964)] = 3.016946 The next approximation x5 is given by 8

1 2.625 + 2.75 [33 − 4(3.015770) + (0.914964)] = 1.986535 x5 = = 2.6875 ⇒ f (x5 ) = (2.6875)3 − 6(2.6875) − 4 = −0.7 11 2 1 2.625 + 2.75 = [35 − 6(3.015770 0.911644 x5 = ) − 3(1.988550=)] 2=.6875 ⇒ f (x5 ) = (2.6875)3 − 6(2.6875) − 4 = −0.7141 < 0 and f(2.75) > 0. 12 2

y(8) = z(8)

Chapter 6.indd 345

8/22/2017 4:31:51 PM

346     Numerical Methods 

Therefore, the root lies between 2.6875 and 2.75. The next approximation x6 is given by

Thus,

2.6875 + 2.75 = 2.71875 2 This is the approximate value of the root of the given equation.

Hence, the root lies between x6 and x7. Thus, the eighth approximation to the root is

x6 =

7. (a) Let f (x) = x3 − 4x − 9

f (x7 ) = (2.71094)3 − 4(2.71094) − 9 = 0.0795 > 0

1 (2.71094 + 2.703125) = 2.70703 2

x8 = Thus,

Now, f (2) = −9 and f (3) = 6.

f (x8 ) = (2.70703)3 − 4(2.70703) − 9 = 0.00879 > 0

Since f(2) is negative and f(3) is positive, a root lies between 2 and 3.

Hence, the root lies between x6 and x8. Thus, the ninth approximation to the root is

Therefore, the first approximate of the root is

1 (2.70703 + 2.703125) = 2.70508 2

x9 = x1 =

1 (2 + 3) = 2.5 2

Thus, f (x1 ) = (2.5)3 − 4(2.5) − 9 = −3.375 < 0

f (x9 ) = (2.70508)3 − 4(2.70508) − 9 = −0.0260 < 0

Hence, the root lies between x1 and 3. Thus, the second approximation to the root is

Hence, the root lies between x8 and x9. Thus, the tenth approximation to the root is 1 x10 = (2.70703 + 2.70508) = 2.70605 2

x2 =

1 (2.5 + 3) = 2.75 2

Thus, f (x2 ) = (2.75)3 − 4(2.75) − 9 = 0.7969 > 0 Hence, the root lies between x1 and x2. Thus, the third approximation to the root is x3 =

1 (2.5 + 2.75) = 2.625 2

Thus, f (x3 ) = (2.625)3 − 4(2.625) − 9 = −1.4121 < 0 Hence, the root lies between x2 and x3. Thus, the fourth approximation to the root is 1 x4 = (2.5 + 2.625) = 2.6875 2 Thus, f (x4 ) = (2.6875)3 − 4(2.6875) − 9 = −0.3391 < 0 Hence, the root lies between x2 and x4. Thus, the fifth approximation to the root is 1 x5 = (2.75 + 2.6875) = 2.71875 2 Thus, f (x5 ) = (2.71875)3 − 4(2.71875) − 9 = 0.2209 > 0 Hence, the root lies between x4 and x5. Thus, the sixth approximation to the root is 1 x6 = (2.71875 + 2.6875) = 2.703125 2 Thus, f (x6 ) = (2.703125)3 − 4(2.703125) − 9 = −0.0611 < 0 Hence, the root lies between x5 and x6. Thus, the seventh approximation to the root is 1 x7 = (2.71875 + 2.703125) = 2.71094 2

Chapter 6.indd 346

Thus,

Thus, f (x10 ) = (2.70605)3 − 4(2.70508) − 9 = −0.0086 < 0 Hence, the root lies between x8 and x10. Thus, the eleventh approximation to the root is x11 =

1 (2.70703 + 2.70605) = 2.70654 2

Thus, f (x11 ) = (2.70654)3 − 4(2.70654) − 9 = 0.000216 > 0 Hence, the root lies between x10 and x11. Thus, the twelfth approximation to the root is 1 (2.70654 + 2.70605) = 2.706295 2 Hence, the root is 2.706. x12 =

8. (b) Consider f (x) = x3 − 5x + 1 = 0 Let x0 = 0.2016 and x1 = 0.2017. Then f (x0 ) = f (0.2016) = 0.000935 > 0 and f (x1 ) = f (0.2017) = −0.0002943 < 0. Hence, the root lies between 0.2016 and 0.2017. The first approximation to the root is given by  x1 − x0  x2 = x1 −   f (x0 )  f (x1 ) − f (x0 )    0.2017 − 0.22016  = 0.2016 −   f (0.2016)  f (0.2017) − f (0.2016) = 0.201641984

8/22/2017 4:32:03 PM

EXPLANATIONS AND HINTS     347

Hence, the root correct to2.four decimal places is × f (2.7402) 2.7402 × 0.00011998 + 2.7406 × 0.00038905 7402 × f (2.7406 ) − 2.7406 0.0013950 x4 = = = = 2.74 0.2016. f (2.7406) − f (2.7402) 0.00011998 + 0.00038905 0.00050903 9. (d) Let 2.7402 × f (2.7406) − 2.7406 × f (2.7402) 2.7402 × 0.00011998 + 2.7406 × 0.00038905 0.0013950 = x4 = = = 2.7405 f (2.7406) − f (2.7402) f (x) = x log10 x − 1.02.00011998 + 0.00038905 0.00050903

2.7405. ⇒f (1) = − 1.2 < 0; f (2) = 2× 0.30103 − 1.2 = − 0.597940; f (3) = 3× 0Hence, .47712 the − 1.2root = 0.is 231364 >0 10. (c) Let 103 − 1.2 = − 0.597940; f (3) = 3× 0.47712 − 1.2 = 0.231364 > 0 f (x) = x log10 x − 4.77 Hence, the root lies between 2 and 3. Therefore, the next approximation is given by

Now,

f(6) = −0.1010 < 0

f(7) = 1.1456 > 0 2f (3) − 3f (2) 2 × 0.23136 − 3 × (−0.59794) = = 2.721014 ⇒ f (x1 ) = (2.7210) log10 (2.7210) − 1.2 = −0.017104 f (3) − f (2) 0.23136 + 0.59794 So, a root of f(x) lies between 6 and 7. Let us take 0.23136 − 3 × (−0.59794) x = 6.5, we get = 2.721014 ⇒ f (x1 ) = (2.7210) log10 (2.7210) − 1.2 = −0.017104 0 0.23136 + 0.59794 f ′(x) = log10 x + log10 e = log10 x + 0.43429 7210) log (2.7210) − 1.2 = −0.017104 x1 =

10

The root lies between 2.721014 and 3. Therefore, the next approximation is given by

The iteration formula is given by

 x log x − 4.77  f (xi ) 10 i  = 0.43429xi + 4.77 = xi −  i x × f (3) − 3 × f (2) 2.721014 × 0.231364 − 3 × (−x 0.i017104 +1 = x) i − 0.68084  log10 xi + 0.43429  log10 xi + 0.43429 x2 = = = f ′(xi ) = 2.740211   f (3) − f (x1 ) 0.23136 + 0.017104 0.2486 x2 =

 x log x − 4.77  f (x ) −0.017104 i  = 0.43429xi + 4.77 x × f (3) − 3 × f (2) 2.721014 × 0= .231364 xi + xi − − 3i × (= xi −  i ) =100.68084 = = 2.740211 1  ′ + ( ) log 0 . 43429 f x x  log10 xi + 0.43429 f (3) − f (x1 ) 0.23136 + 0.017104 i  10 i 0.2486 Putting n = 0, 1, 2, 3, …, we get

21014 × 0.231364 − 3 × (−0.017104) 0.68084 = = 2.740211 0.23136 + 0.017104 0.2486 ⇒ f (x2 ) = f (2.7402) = 2.7402 × log(2.7402) − 1.2 = 0.00038905

02) = 2.7402 × log(2.7402) − 1.2 = 0.00038905 Therefore, the root lies between 2.740211 and 3. The next approximation is given by x3 =

2.7402 × f (3) − 3 × f (2.7402) f (3) − f (2.7402)

2.7402 × 0.23136 + 3 × 0.00038905 = 0.23136 + 0.00038905 =

x1 =

0.43429x0 + 4.77 = 6.08792861 log10 x0 + 0.43429

x2 =

0.43429x1 + 4.777 = 6.083173814 log10 6.0879286 + 0.43429

x3 =

0.43429x1 + 4.77 = 6.083173439 log10 6.08317381 + 0.43429

Hence, the required root is 6.0831. 11. (c) Let f (x) = 3x − cos x − 1 f(0) = 3(0) − cos(0) − 1 = −2 < 0   and f(1) = 3(1) − cos 1 − 1 = 1.4597 > 0

063514 = 2.740627 0.23175

Hence, a root of f (x) = 0 lies between 0 and 1. It is nearer to 1. Thus, we take x0 = 0.6. Also,

⇒ f (x3 ) = (2.7406) log10 (2.7406) − 1.2

f ′(x) = 3 + sin x

= 0.00011998 Using Newton’s iteration formula, we have Therefore, the root lies between 2.740211 and 2.740627. The next approximation is given by

x4 =

Chapter 6.indd 347



xn +1 = xn −

3x − cos xn − 1 xn sin xn + cos xn + f (xn ) = xn − n = 3 + sin xn 3 + sin xn f ′(xn )

2.7402 × f (2.7406) − 2.7406 × f (2.7402 +12.7406 3x0.00011998 x × sin0.x00038905 1.0013950 f (xn) ) 2.7402 × − cos xn − n + cos xn=+ 0 == xn − n = 2.7405  (1) xn +1 = xn − = n f (2.7406) − f (2.7402) 0.300011998 0.00050903 + sin xn + 0.00038905 3 + sin xn f ′(xn )

8/22/2017 4:32:13 PM

348     Numerical Methods 

Putting n = 0, the first approximation x1 is given by

By Simpson’s 1/3 rule, we have 0.6

h  −x2  + 4  y+1 1+ y3 + y5 + 2(y2 + y4 ) } x0 sin x0 + cos x0 + 1 (0.6) sin(0.6) + cos(0.6)∫+e1 dx { y ) + 0y6.82533 0.6= (0.5729    x1 = = = = 0.6071 3  0 0 3 sin(0.6) 3 + 0.5729 3 + sin x0 0.1 x sin x0 + cos x0 + 1 (0.6) sin(0.6) + cos(0.6) + 1 0.6(0.5729) + 0.82533 + 1 = {[1 + 0.6977 ] + 4 [ 0.99 + 0.9139 + 0.7788 x1 = 0 = = = 0.6071 3 3 sin(0.6) 3 + 0.5729 3 + sin x0 + 2(0.9608 + 0.8521)]} + 1 (0.6) sin(0.6) + cos(0.6) + 1 0.6(0.5729) + 0.82533 + 1 0.1 = = 0.6071 = = (1.6977 + 10.7308 + 3.6258) 3 + 0.5729 3 sin(0.6) 3 0.1 Putting n = 1 in Eq. (1), the second approxima= (16.0543) = 0.5351 3 tion is 0.6

x sin x1 + cos x1 + 1 (0.6071) sin(0.6071) + cos(0.6071)∴+ 1 e−(x02 dx .6071 .57049 = )( 0.05351 . ) + 0.8213 + 1 = 0.6071 x2 = 1 = = ∫ 3 + sin x1 3 sin(0.6071) 3 + 0.57049 0

x1 sin x1 + cos x1 + 1 (0.6071) sin(0.6071) + cos(0.6071) + 1 (0.6071)(0.57049) + 0.8213 + 1 14. (d) Dividing the interval = = = 0.6071(1, 2) into four equal 3 + sin x1 3 sin(0.6071) 3+ 0.57049 parts, each of width 071) sin(0.6071) + cos(0.6071) + 1 (0.6071)(0.57049) + 0.8213 + 1 = = 0.6071 2 −1 h= = 0.25 3 sin(0.6071) 3 + 0.57049 4 Since x1 = x2 , the desired root is 0.6071. Therefore, the values of y = f(x) = 1/x are as follows: x2 =

12. (a) Let y = sin x − log x + ex and h = 0.2, n = 6.

x f(x)

1 1

1.25 0.8

1.50 0.6667

1.75 0.57143

2.00 0.5

The values of y are given as follows: x

0.2

0.4

0.6

0.8

1.0

1.2

By Simpson’s 1/3 rule, we have

1.4 2

3.0295 2.7975 2.8976 3.1660 3.5597 4.698 4.4042 y0 y1 y2 y3 y4 y5 y6

y



dx h = [(y0 + y4 ) + 4(y1 + y3 ) + 2(y2 )] x 3

1

By Simpson’s 3/8 rule, we have

(0.25) [(1 + 0.5) + 4(0.8 + 0.57143) + 2(0.6667)] 3 = 0.69326

= 1.4

∫ ydx =

3h  (y + y6 ) + 2 (y3 ) + 3 (y1 + y2 + y4 + y5 ) 8  0

0.2 2

3 = (0.2) 7.7336 + 2 (3.1660) + 3 (13.3247 ) = 4.053 8





dx = 0.69326. x

1 1.4



x ∫ (sin x − log x + e ) dx = 4.053.

0.2

13. (b) Say y = f (x) = e−x

15. (d) Divide the given integral of integration into six equal subintervals. The arguments are 0, 1, 2, 3, 4, 5, 6 and h = 1. We have

2

f (x) = Since we have to divide the interval into six parts, 0.6 − 0 = 0.1. The values of y are given width h = 6 as follows: x x

0 2

y

Chapter 6.indd 348

0.1

0.2

0.3 0.4 0.5 0.6 e y = f ( 0 ) = 1 , y = f ( 1 ) = , y2 = f (2) = 0 0.01 0.04 0 0.09 0.161 0.25 20.36 1 0.9900 0.9608 0.9139 0.8521 0.7788e40.6977 yy4 = f (4)y= , yy5 = f (5) = y0 y1 y2 y3 4 5 5 6

ex 1+ x

So y0 = f (0) = 1, y1 = f (1) =

e e2 e3 , y2 = f (2) = , y3 = f (3) = , 2 3 4

e3 e2 e4 e5 e6 , y3 = f (3) = ,, y4 = f (4) = , y5 = f (5) = , y6 = f (6) = 3 4 5 6 7 e6 e5 , y6 = f (6) = 6 7

8/22/2017 4:32:22 PM

EXPLANATIONS AND HINTS     349

The table is as follows: x

0

1

6

2

3

4

5



6

∫ 1 + x2 1

= 1.4108.

0

e 2

1 y (y0)

(y1)

e2 3 (y2)

e3 4 (y3)

e4 5 (y4)

e5 6 (y5)

e6 7 (y6)

18. (a) We have x0 = 0, y0 = 1 and h = 0.2 . dy = f (x, y ) = 1 + y 2 dx

Applying Simpson’s three-eighth rule, we have 6

ex

∫ 1 + x dx =

3h [(y0 + y6 ) + 3(y1 + y2 + y4 + y5 ) + 2y3 ] 8

First approximation

(

)

(

)

y1 = y(0.2) = y0 + hf (x0 , y0 ) = y0 + h 1 + y02 = 1 + (0.2)(1 + 1) = 1.4 2 4 5 3 6    3 e e e e  e e  2 + +  + 2  = 1 +  + 3  + y =  2 31 y5(0.2) 6=y0 + 4hf (x0 , y0 ) = y0 + h 1 + y0 = 1 + (0.2)(1 + 1) = 1.4 8  7    3 Second approximation = {[1 + 57.66327 ] + 3 [1.3591 + 2.463 + 10.9196 8 y2 = y(0.4) = y1 + h f (x1 , y1 ) = y1 + h 1 + y12 = 1.4 + (0.2)(1 + 1.96) +24.7355 + 2(5.0214)} = 0.701652 y2 = y(0.4) = y1 + h f (x1 , y1 ) = y1 + h 1 + y12 = 1.4 + (0.2)(1 + 1.96) = 1.992 6 x e ∴∫ dx = 0.701652. Hence, y(0.4) = 1.992. 1+ x 0 19. (c) Given 16. (b) Here h = 0.2. f (x, y) = 1 − y, x0 = 0, y0 = 0, x1 = 0.1. x 0.6 0.8 1.0 1.2 1.4 1.6 1.8 2.0 By modified Euler’s method, we get y 1.23 1.58 2.03 4.32 6.25 8.38 10.23 12.45   1 h yn +1 = yn + hf  xn + , yn + hf (xn , yn ) By trapezoidal rule,   2 2 2.0   h h h y1 = y0 + hf  x0 + , y0 + f (x0 , y0 ) ∫ y dx = 2 [(y0 + y7 ) + 2(y1 + y2 + y3 + y4 + y5 + y6 )] 2 2   0

(

(

)

  0. 1 0.1 y1 = 1 + (0.1)f  0 + ,0 + f (0, 0)   2 2 y1 = 0.095

0.6

0.2 [(1.23 + 12.45) + 2(1.58 + 2.03 + 4.32 = 2 + 6.25 + 8.38 + 10.23 + 12.45)] = (0.1)[13.68 + 90.48] = 10.416

  h h y2 = y1 + hf  x1 + , y1 + f (x1 , y1 )   2 2  0.1 , 0.095 = (0.095) + (0.1)f  0.1 +  2

2.0



)

∫ y dx = 10.416. 0.6

17. (c) Divide the interval (0, 6) into six parts, each of 1 width h = 1. The value of f (x) = are given 1 + x2 as follows:

+

 0.1 f (0.1, 0.095)  2

y2 = 0.18098 Hence, the value of y(0.2) is 0.18098.

x

0

1

2

3

4

5

6

1 0.5 0.2 0.1 0.05884 0.0385 0.027 y = f(x) (y0) (y1) (y2) (y3) (y4) (y5) (y6) By trapezoidal rule, 6

∫ 1 + x2 1

=

h (y + y6 ) + 2(y1 + y2 + y3 + y4 + y5 ) 2 0

0

1 = [(1 + 0.027) + 2(0.5 + 0.2 + 0.1 2 + 0.0588 + 0.0385)] = 1.4108

Chapter 6.indd 349

20. (d) Here x0 = 0, y0 = 1, h = 0.2, f (x0 , y0 ) = 1 k1 = hf (x0 , y0 ) = (0.2)(1) = 0.2  1 1  k2 = hf x0 + h, y0 + k1  = 0.2 × f (0.1, 1.1)  2 2  = 0.2400  1 1  k3 = hf x0 + h, y0 + k2  = 0.2 × f (0.1, 1.12)  2 2  = 0.2440 k4 = hf (x0 + h, y0 + k3 ) = 0.2 × f (0.2, 1.244) = 0.2888

8/22/2017 4:32:32 PM

350     Numerical Methods 

1 (k + 2k2 + 2k3 + k4 ) 6 1 1 = (0.2000 + 0.4800 + 0.4880 + 0.2888) 6 1 = (1.4568) = 0.2468 6

∴k=

29. (b) Let the equation of the line be y = ax + bx. Normal equation for a is ∑y = 10a + b∑x Normal equation for b is ∑xy = a∑x + b∑x2. Now   ∑x = 64 ∑y = 70 ∑xy = 317 ∑x2 = 528

Hence, the required approximate value of y is 1.2428. 21. (a) All measured values tend to be close to the actual value, therefore it is accurate. All measured values are close to each other, therefore it is precise too.

Therefore, the normal equations are and

22. (b) The measured values are far from true value. However, all measured values are close to each other. So it is pre use but inaccurate. 23. (c) If the errors are random, uncertainty would reduce by a factor N , where N is number of measurements.

(1)

64a + 528b = 317 

(2)

Multiplying Eq. (1) by 6.4, we get 64a + 409.6b = 448 



121.1b = −131

\

Therefore,

(3)

Subtracting Eq. (3) from Eq. (2), we get

25 = 5 (    N = 25) 5% Now uncertainty would be = 1% 5 24. (b) Relative error is the ratio of measured error to the true value.

10a + 64b = 70

Therefore, b = −

131 ≅ − 1. 1 121.1

Substituting the value of b in Eq. (1), we get a ≅ 14; therefore, the equation of the line is y = (14 − 1.1x).

100 − 98 2 = = 0.02 100 100

30. (a) The forward difference table is drawn as follows: 25. (d) The first three cases represent systematic errors. The error due to AC noise is equally likely to occur in both positive and negative directions.

xi

fi

0

1

26. (d) Standard deviation of mean is given by

1

7

∆fi

∆2fi

∆3fi

∆4fi

6 sx =

sx

10 16

0.2 =

N

= 0.05 16

2

23

6 16

32 27. (b) According to the laws relating to propagation of errors, If Z = X ± Y 2 Then ∆Z = ( ∆X ) + ( ∆Y  Extending this further,

3



109 From Newton’s forward difference formula, p( p − 1) 2 ∆ f0 2! p( p − 1)( p − 2) 3 + ∆ f0 3!

f (x) = f0 + p∆f0 +

If R = X ± Y ± Z 1

2 2 2 Then ∆R = ( ∆X ) + ( ∆Y ) + ( ∆Z )  2   1 2 2  Therefore, δ R = (δ X ) + (δ Y ) + (δ Z )2  2  

or (δ R)2 = (δ X )2 + (δ Y )2 + (δ Z )2 28. (a) Error = (0.15)2 + (0.05)2 = 225 × 10−4 + 25 × 10−4 = 250 × 10−2 = 0.151

Chapter 6.indd 350

22 54

4 1 22 )

55

0 6

p=

x − x0 ; h = 1 and x = 0.5 h

Therefore, p=

0.5 − 0 = 0.5 1

∆f0 = 6, ∆2f0 = 10, ∆3fi = 6 … (from difference table).

8/22/2017 4:32:41 PM

SOLVED GATE PREVIOUS YEARS’ QUESTIONS     351

Therefore, 0.5(0.5 − 1) × 10 2 0.5(0.5 − 1)(0.5 − 2) + ×6 6 = 1 + 3 + (−1.25) + 0.375

f(0.5) = 1 + 0.5 × 6 +

31. (c) Newton's forward formula is employed to find derivative of a function for a given data near the beginning of the table. 32. (a) Since derivative is desired to be computed at the centre of the given data table, Stirling’s ­formula will be used.

= 4.375 − 1.25 = 3.125 and hence the answer.

SOLVED GATE PREVIOUS YEARS’ QUESTIONS 1. The accuracy of Simpson’s rule quadrature for a step size h is

( ) (c) O (h4 ) (a) O h

2

Solution:  Starting

from

x0,

slope

of

1 − 0. 5 = −1 0 − 0.5

( ) (d) O (h5 )



[GATE 2003, 1 Mark]

This line will cut x-axis (i.e. y = 0), at x = 1

(b) O h

a=

3

line

y-intercept = 1

Equation of a is y = mx + c = —x + 1 Since x =1 > x = 0.8, a perpendicular at x = 1 will cut the line c and not line b.

Solution:  Option (c). 

Ans. (c) Therefore, the root will be 1.3.

2. A piecewise linear function f (x) is plotted using thick solid lines in the figure below (the plot is drawn to scale).

Starting from x1, the perpendicular at x1 is cutting line b and the root will be 0.6. Starting from x2,

f(x) 1.0



slope of line, d =

(2.05, 1.0) a

1 − 0. 5 =1 2.05 − 1.55

d Equation of d is given by (1.55, 0.5) (0.5, 0.5) x1 1.3

x0

i.e. 1.55 x2 2.05

0.6

b

y = x − 1.05

This line will cut x-axis at x = 1.05. Since x = 1.05 is greater than x = 0.8, the perpendicular at x = 1.05 will cut line c and not line b. The root will be therefore equal to 1.3.

c

(0.8, 1.0) If we use the Newton—Raphson method to find the roots of f (x) = 0 using x0, x1 and x2, respectively, as initial guesses, the roots obtained would be (a) 1.3, 0.6 and 0.6, respectively (b) 0.6, 0.6 and 1.3, respectively (c) 1.3, 1.3 and 0.6, respectively (d) 1.3, 0.6 and 1.3, respectively [GATE 2003, 2 Marks]

Chapter 6.indd 351

y − 0.5 = 1 (x − 1.55)

So starting from x0, x1 and x2, the roots will be 1.3, 0.6 and 1.3, respectively. 

Ans. (d)

3. With a 1 unit change in b, what is the change in x in the solution of the system of equations x + y = 2, 1.01x + 0.99y = b ? (a) Zero (c) 50 units

(b) 2 units (d) 100 units [GATE 2005, 2 Marks]

8/22/2017 4:32:47 PM

352        Numerical Methods 

Solution:  Given that

x + y = 2

1.01 x + 0.99 y = b

(1) (2)



xk +1 = xk −

(1/xk − a) −

Multiplying Eq. (1) by 0.99 and subtracting it from Eq. (2), we get





1 1.98 b− 0.02 0.02

So, for 1 unit change in b, change in x would be 50 units.

(a) 0.11, 0.1299 (c) 0.12, 0.1416

(b) 0.12, 0.1392 (d) 0.13, 0.1428 [GATE 2005, 2 Marks]

Ans. (c)

Statement for Linked Answer ­Questions 4 and 5 Given a > 0, we wish to calculate the recipro1 cal value by Newton—Raphson method for a f (x) = 0.

Solution:  For a = 7, the iteration equation is given by xk +1 = 2xk − 7 xk2 with

4. The Newton—Raphson algorithm for the function will be  1 a a  (a) xk +1 = xk +  (b) xk +1 = xk + xk2   2  xk  2  a (c) xk +1 = 2xk − axk2 (d) xk +1 = xk − xk2 2 [GATE 2005, 2 Marks] 1 Solution:  To calculate using Newton—Raphson a 1 method, set up the equation as x = . a 1 =a x Therefore, 1 ⇒ −a = 0 x 1 f (x ) = − a = 0 i.e. x 1 f ′ (x ) = − 2 Now, x 1 f (xk ) = −a xk 1 f ′ (xk ) = − 2 xk

xk +1

x1 = 2x0 − 7 x02 = 2 × 0.2 − 7 (0.2) = 0.12

and x2 = 2x1 − 7 x12 = 2 × 0.2 − 7 (0.12) = 0.1392 2



Ans. (b)

6. Starting from x0 = 1, one step of Newton—Raphson method in solving the equation x3 + 3x − 7 = 0 gives the next value (x1 ) as (a) x1 = 0.5

(b) x1 = 1.406

(c) x1 = 1.5

(d) x1 = 2 [GATE 2005, 2 Marks]

Solution:  Using Newton—Raphson method, we have

x1 = x0 −

f (x0 ) f ′ (x0 )

(1)

Given function is f (x) = x3 + 3x − 7 and

f ′ ( x ) = 3x 2 + 3

Putting x0 = 1, we get

f (x0 ) = f (1) = (1) + 3 × (1) − 7 = −3



f ′ (x0 ) = f ′ (1) = 3 × (1) + 3 = 6

3

2

Substituting x0 , f (x0 ) and f ′ (x0 ) values into Eq. (1), we get

For Newton—Raphson method, f (xk ) = xk − f ′ (xk )

x0 = 0.2 2



Chapter 6.indd 352

Ans. (c)

5. For a = 7 and starting with x0 = 0.2, the first two iterations will be

∆x dx 1 = = = 50 ∆b db 0.02



xk +1 = 2xk − axk2



0.02x = b − 1.98 ⇒ x=

xk2

Simplifying the above equation, we get

(1.01 − 0.99) x = b − 2 × 0.99



1



 −3  x1 = 1 −   = 1.5  6  Ans. (c)

25-08-17 2:51:26 PM

SOLVED GATE PREVIOUS YEARS’ QUESTIONS     353

7. Match List I with List II and select the correct answer using the codes given below the lists: List I A. Newton—Raphson method B. Runge—Kutta method equations C. Simpson’s rule equations D. Gauss elimination

1. 2. 3. 4. 5. 6.

9. A 2nd degree polynomial, f (x) has values of 1, 4 and 15 at x = 0, 1 and 2, respectively. The inte2

List II Solving non-linear equations Solving simultaneous linear equations Solving ordinary differential Numerical integration Interpolation C  alculation of Eigenvalues

gral

∫ f (x) dx is to be estimated by applying the 0

trapezoidal rule to this data. What is the error (defined as “true value — approximate value”) in the estimate? 4 2 (a) − (b) − 3 3 2 (c) 0 (d) 3 [GATE 2006, 2 Marks]

Codes: A   B   C   D (a) (b) (c) (d)

6   1   1   5  

1   6   3   3  

5   4   4   4  

Solution:  We are given that f (x) = 1, 4, 15 at x = 0, 1 and 2, respectively.

3 3 2 1

2

∫ f (x) dx = 2 (f1 + 2f2 + f3 ) h



(three-point trap-

0

[GATE 2005, 2 Marks] Ans. (c)



ezoidal rule) Here, h = 1.

8. The differential equation (dy /dx) = 0.25 y is to be solved using the backward (implicit) Euler’s method with the boundary condition y = 1 at x = 0 and with a step size of 1. What would be the value of y at x = 1? 2

(a) 1.33 (c) 2.00

2

1

0

Therefore, approximate value by trapezoidal rule = 12 Since f (x) is a second-degree polynomial, let

(b) 1.67 (d) 2.33

f (x) = a0 + a1x + a2 x2

[GATE 2006, 1 Mark]

f (0 ) = 1

Solution:  We have dy = 0.25y 2 (y = 1 at x = 0) dx h=1 Iterative equation for backward (implicit) Euler’s methods for the above equation is given by



a0 + 0 + 0 = 1



a0 = 1

yk +1 = yk + h f (xk +1 , yk +1 )



yk +1 = ⇒    

yk + h × 0.25yk2+1

0.25h yk2+1

  



− yk +1 + yk = 0

⇒ ⇒ 



a0 = a1 + a2 = 4



1 + a1 + a2 = 4 a1 + a2 = 3 f (2) = 15





a0 + 2a1 + 4a2 = 15



1 + 2a1 + 4a2 = 15



0.25h y12 − y1 + y0 = 0

2a1 + 4a2 = 14

(1)

(2)

Solving Eqs. (1) and (2), a1 = −1 and a2 = 4

Since y0 = 1 and h = 1

f (1) = 4



Putting k = 0 in the above equation, we get

Chapter 6.indd 353

∫ f (x) dx = 2 (1 + 2 × 4 + 15) = 12



f ( x ) = 1 − x + 4x 2

0.25h y12 − y1 + 1 = 0



1 ± 1−1 =2 2 × 0.25 y1 = 2

Now, the exact value of

y1 =

2

∫ Ans. (c)

0

2

f (x) dx =

∫ 0

2  32 x2 4x2  (1 − x + 4x2 ) dx =  x − +  = 3 2 3   0

8/22/2017 4:33:18 PM

354     Numerical Methods 

Error = Exact — Approximate value =

32 4 − 12 = − 3 3



Ans. (a)

Newton—Raphson equation f (xk ) xk +1 = xk − f ′ (xk )

10. Given that one root of the equation x3 − 10x2 + 31x − 30 = 0 x3 − 10x2 + 31x − 30 = 0 is 5, the other two roots are (a) 2 and 3 (b) 2 and 4 (c) 3 and 4 (d) −2 and −3

Solution:  We have f (x) = x3 − 10x2 + 31x − 30 = 0



x 2 − 5x + 6 x − 5 x3 − 10x2 + 31s − 30

)

x − 5x 3

−5x2 + 31x − 30 −5x2 + 25x 6x − 30 6x − 30 0 x3 − 10x2 + 31x − 30 = 0



(x − 5)(x2 − 5x + 6) = 0

3xk3 + 4xk ) − (xk3 + 4xk − 9) ( = 3xk2 + 4

xk +1 =

2xk3 + 9 3xk2 + 4 Ans. (a)

12. The equation x3 − x2 + 4x − 4 = 0 is to be solved using the Newton—Raphson method. If x = 2 is taken as the initial approximation of the solution, then the next approximation using this method will be 2 4 (a) (b) 3 3 3 (c) 1 (d) 2

2



xk3 + 4xk − 9) ( xk +1 = xk − (3xk2 + 4)

xk +1

x — 5, we get

is

f ′ (xk ) = 3xk2 + 4



Since 5 is a root of f (x) , it is divisible by x — 5. Now dividing f (x) by

iteration

f (xk ) = xk3 + 4xk − 9

[GATE 2007, 2 Marks]



for

[GATE 2007, 2 Marks]

Roots of x2 − 5x + 6 are 2 and 3.

Solution:  Here,

x0 = 2

We are given Hence, the other two roots are 2 and 3. 

Ans. (a)



f (x) = x3 − x2 + 4x − 4

11. The following equation needs to be numerically solved using the Newton—Raphson method:

Then,

f ′ (x) = 3x2 − 2x + 4

x 3 + 4x − 9 = 0

Also,

f (x0 ) = f (2) = 8



f ′ (x0 ) = f ′ (2) = 12

The iterative equation for this purpose is (k indicates the iteration level) (a) xk +1 =

2xk3 + 9 3xk2 + 4



(b) xk +1 =

(c) xk +1 = xk − 3xk2 + 4 (d) xk +1 =



3xk3 + 4 2xk2 + 9 4xk2 + 3 9xk2 + 2

x1 = x0 −

f (x0 ) 8 4 = 2− = f ′ (x0 ) 12 3



Ans. (b)

xn 9 + , x = 0.5 2 8xn 0 obtained from the Newton—Raphson method.

13. Consider the series xn +1 =

[GATE 2007, 2 Marks] The series converges to Solution:  We have

Chapter 6.indd 354



f (x) = x3 + 4x − 9 = 0



f ′ ( x ) = 3x 2 + 4

(a) 1.5 (c) 1.6

(b) 2 (d) 1.4 [GATE 2007, 2 Marks]

8/22/2017 4:33:32 PM

SOLVED GATE PREVIOUS YEARS’ QUESTIONS     355

Solution:  We are given that x 9 xn +1 = n + , x = 0.5 2 8xn 0 As n → ∞ , when the series converges xn +1 = xn = a = root of equation a= ⇒a = ⇒

a 9 + 2 8a 4a 2 + 9 8a 2



2

⇒a =



y = sinx

6

6p 4

 6p  y6 = sin   = −1  4 

7

7p 4

 7p  y7 = sin   = −0.70710  4 

8

2p

 8p  y8 = sin   = 0  4 

x0 + nh

9 a = 4



x

Trapezoidal rule is given by

8a = 4a + 9 2

i

f (x) ⋅ dx =

h (y + yn ) + 2 (y1 + y2 +  + yn−1 ) 2  0

x0 2p

p

∫ sin x ⋅ dx = 8 × (0 + 0) + 2

3 = 1.5 2

0



Ans. (a)

14. A calculator has accuracy up to 8 digits after deci-



(0.70710 + 1 + 0.70710 + 0 − 0.70710 − 0.70710)

= 0.00000 Ans. (a)

2p

mal place. The value of

∫ sin x dx

when evaluated

0

using this calculator by trapezoidal method with 8 equal intervals, to 5 significant digits is (a) 0.00000 (c) 0.00500

(b) 1.0000 (d) 0.00025 [GATE 2007, 2 Marks]

15. The differential equation (dx / dt) = (1 − x) / t  is discretised using Euler’s numerical integration method with a time step ∆T > 0. What is the maximum permissible value of ∆T to ensure stability of the solution of the corresponding discrete time equation? (b) t/2 (d) 2t [GATE 2007, 2 Marks]

(a) 1 (c) t

Solution:  We know that h=

2p − 0 p = 8 4

We can tabulate as follows: i

x

y = sinx

0

0

y0 = sin (0) = 0

dx 1 − x = dt t 1− x f (x, y ) = t

Solution:  Here, Here,

The Euler’s method equation is given by

1 − xi  xi +1 = xi + h    t   h h xi +1 = 1 −  xi +  t t

1

p 4

p  y1 = sin   = 0.70710  4 

p 2

p  y2 = sin   = 1  2 



2

3p 4

 3p  y3 = sin   = 0.7010  4 

For stability 1 −

3

h x1 > 0 is evalu-

1

ated analytically as well as numerically using a single application of the trapezoidal rule. If I is the exact value of the integral obtained analytically and J is the approximate value obtained using the trapezoidal rule, which of the following statement is correct about their relationship? (a) J > 1 (b) J < 1 (c) J = 1 (d) Insufficient data to determine the relationship [GATE 2015, 2 Marks]

Using trapezoidal rule with step size of 0.1, the 0.4

value of

∫ f(x)dx

Solution:  We know that the approximated value

.

is

0

[GATE 2015, 2 Marks] Solution:  We are given x

0

0.1

0.2

0.3

0.4

f(x)

0

10

40

90

160

y0 0.4

y1

y2

y3

y4

0. 4

∫ f(x)dx = ∫ ydx = 2 [(y0 + y4 ) + 2(y1 + y2 + y3 )] h

0

0



0.1 [(0 + 160) + 2(10 + 40 + 90)] = 22 = 2 Ans. 22

55. Newton—Raphson method is used to find the roots of the equation, x3 + 2x2 + 3x — 1 = 0. If the initial guess is x0 = 1, then the value of x after second iteration is . [GATE 2015, 2 Marks] Solution:  By Newton—Raphson method, x1 = x0 −

Chapter 6.indd 366

∫a f(x)dx b

of

f (x0 ) f (1) 5 1 = 1− = = 1− f ′(x0 ) f ′(1) 10 2

obtained by trapezoidal rule is always

greater than the analytical value. Thus, J > 1, where J is the approximate value and 1 is the analytical value.  Ans. (a) 57. The quadratic equation x2 - 4x + 4 = 0 is to be solved numerically, starting with the initial guess x0 = 3. The Newton-Raphson method is applied once to get a new estimate and then the secant method is applied once using in the initial guess and this new estimate. The estimated value of the root after the application of the secant method is . [GATE 2015, 2 Marks] Solution:  We have, f (x) = x2 − 4x + 4 x0 = 3 f ′(x) = 2x − 4 By Newton—Raphson method, x1 = x0 −

f (x0 ) 1 = 3 − = 2. 5 f ′(x0 ) 2

For secant method, let x0 = 2.5 and x1 = 3 By secant method,

8/22/2017 4:36:06 PM

SOLVED GATE PREVIOUS YEARS’ QUESTIONS     367

x2 = x1 − = 3−

x1 − x0 (3 − 2.5) f (x1 ) = 3 − f (3) f (x1 ) − f (x0 ) f (3) − f (2.5)

0.5 0.5 = 3 − 0.6667 = 2.333 ×1 = 3 − 1 − (0.25) 0.75



Ans. 2.333

60. Consider the first-order initial value problem y′ = y + 2x − x2, y(0) = 1, (0 ≤ x  log2 1000 ⇒

[GATE 2017, 2 Marks]

⇒ n > 9.96 ⇒ n = 10

Solution:  F (x) = x3 + x − 1 = 0     (i) 

Ans. (10)

By Newton—Raphson’s iterative method, we have x

n +1

= xn −

F (xn ) F ′(xn )



(ii)

Given x0 = 1 ⇒ F (x0 ) = 1 + 1 − 1 = 1 F ′(x) = 3x2 + 1 F ′(x0 ) = 3 + 1 = 4

69. P(0, 3), Q(0.5, 4) and R(1, 5) are three points on the curve defined by f(x). Numerical integration is carried out using both trapezoidal rule and Simpson’s rule within limits x = 0 and x = 1 for the curve. The difference between the two results will be (a) 0

(b) 0.25

(d) 1

[GATE 2017, 2 Marks]

So, from Eq. (ii), we have x1 = x0 − x1 = 1 −

(c) 0.5

Solution:  Let x = 0, 0.5, 1 and y = 3, 4, 5. Using Trapezoidal rule,

F (x0 ) F ′(x0 )

1 3 = 4 4

1

∫ f(x) dx = 0

Therefore, from Eq. (i),

0. 5 ( 3 + 5 ) + 2 ( 4 )  2 

0. 5 × 16 2 =4 =

33

33 33 FF((xx11)) ==   ++ −−11 == 00..1718 1718 44 44 22

Using Simpson's rule,

⇒ FF′(′(xx11)) == 3333 ++11 == 22..687 68755 44

1

∫ f(x) dx = 0

Therefore,

0. 5 × 24 3 =4

F (x1 ) x2 = x1 − F ′(x1 ) x2 = 

Chapter 6.indd 369

0. 5 ( 3 + 5 ) + 0 + 4 ( 4 )  3 

=

3 0.1718 − = 0.686 4 2.6875

The difference between the two results is zero. Ans. (0.68)

Ans. (a)

8/22/2017 4:36:32 PM

Chapter 6.indd 370

8/22/2017 4:36:32 PM

CHAPTER 7

MATHEMATICAL LOGIC

INTRODUCTION Mathematical logic is a subfield of mathematics that explores the applications of logic to mathematics. It is closely linked to the foundations of mathematics, metamathematics and theoretical computer science.

STATEMENTS

Truth Table A table showing the truth value of a statement formula for every possible combination of truth values of component statements is called its truth table.

Truth Values

Atomic Statements

A statement is a declarative sentence which has one and only one of the two possible values called truth values. The two truth values are TRUE and FALSE. These can also be denoted as T and F or 1 and 0, respectively.

The simple statements without any connectives are called atomic or primary statements.

As there are only two possible truth values for all the statements, the logic is called two-valued logic. For example, consider the statement 10 + 10 = 100.

Molecular Statements The statements formed by joining two or more statements through connectives are called molecular or compound statements.

Chapter 7.indd 371

Now, the statement mentioned above has a truth value, which depends on the system. If we consider the statement in context of a binary system, then it is true. On the contrary, if we consider the statement in context of a decimal system, then it is false.

22-08-2017 06:29:42

372     Mathematical Logic 

CONNECTIVES New sentences can be formed from given ­sentences using phrases or logical operators called connectives. A symbol or word used to connect two or more sentences in a grammatically valid way is called a logical connective.

(denoted by X ∨ Y ) is T when the truth value of either X or Y is T and F when the truth value of both X and Y is F. Table 3 depicts the truth table for disjunction. Table  3|   Truth table for disjunction

The logical relations when expressed by phrases are called propositional connectives. The sentence obtained will be a proposition only when the new sentence has a truth value T or F but not both.

X

Y

X∨Y

T

F

T

T

F

T

F

T

T

F

F

F

Types of Connectives Some of the commonly used connectives are explained in this section. They are described as follows: 1. Negation (NOT): If X is a proposition, then negation X or NOT X (denoted by X) is a proposition whose value is T when the truth value of X is F and F when truth value of X is T. Table 1 depicts the truth table for negation. Table 1|   Truth table for negation

Table 4|   Truth table for implication

X

Y

X→Y

X

ùX

T

T

T

T

F

T

F

F

F

T

F

T

T

F

F

T

2. Conjunction (AND): If X and Y are two propositions, then conjunction of X and Y or X AND Y (denoted by X ∧ Y ) is T only when the truth value of both X and Y is T. For all other values, X AND Y is F. Table 2 depicts the truth table for conjunction. Table 2|   Truth value for conjunction

X

Y

X∧Y

T

T

T

T

F

F

F

T

F

F

F

F

3. Disjunction (OR): If X and Y are two propositions, then disjunction of X and Y or X OR Y

Chapter 7.indd 372

4. Implication (IF … THEN …): If X and Y are two propositions, then “IF X THEN Y ” denoted by X → Y is a proposition whose value is T when the truth value of X is F and whose value is equal to the value of Y when value of X is T.  Table 4  depicts the truth table for implication.

5. IF AND ONLY IF: If X and Y are two statements, then X IF AND ONLY IF Y, denoted by X  Y, is a statement whose truth value is T when the truth values of X and Y are same and F when the truth values of X and Y are different. Table 5  depicts the truth table for IF AND ONLY IF. Table 5|   Truth table for IF AND ONLY IF

X

Y

→  Y X  ←

T

T

T

T

F

F

F

T

F

F

F

T

22-08-2017 06:29:44

WELL-FORMED FORMULAS     373

Table 6 summarizes all the connectives discussed thus far.     Table 6|   Representation and meaning of logical

If we have a WFF involving n propositional constants, we have 2n possible combinations of truth values of propositions replacing the variables. For example,

­connectives Connective Symbol

Read as

 or ¬ or ∼ NOT X

Negation

X

Conjunction



Disjunction



Implication



X OR Y (or X∨Y both) IF X THEN Y X → Y

IF AND ONLY IF

→ ←

X IF AND ONLY IF

X AND Y

Obtain the truth table for = (X ∨ Y ) ∧ (X → Y ).

Denoted as

X∧Y

→ Y X ←

X

Y

X∨Y

X→Y

A

T T F F

T F T F

T T T F

T F T T

T F T F

→ ((X ∨ Z) Construct the truth table for α = (X ∧ Y ) ← → (Z ∨ Y )). X Y

Z (X ∧Y ) Z ∨ Y

T T

T T

T F

(X ∨ Z) (X ∧ Y ) a → (Z ∨ Y )

For example Translate the following sentences into propositional forms: (a) If my course is complete and I have time, then I will go to play. (b) I do not have time and I will not go to play. (c) My course is not complete and I do not have time. (d) I will go to play only if my course is not complete. Solution:  Let X be proposition “My course is complete.” Y be proposition “I have time.” Z be proposition “I will go to play.” Then (a) X ∧ Y → Z (c) X ∧ Y

(b) Y ∧ Z → X (d) Z ←

WELL-FORMED FORMULAS A propositional variable is a variable which can take the value T or F. A propositional variable is not a proposition but can be replaced by a proposition. A well-formed formula (WFF), or simply formula, can be defined in the following ways: 1. X is a WFF if it is a propositional variable. 2. If X is a WFF, then X is also a WFF. 3. If X and Y are WFFs, then (X ∧ Y ), (X ∨ Y ), (X → Y ) → Y ) are all WFFs. and (X ← 4. A combination of symbols is a WFF if it is obtained by finite amount of applications of conditions (1)—(3).

Chapter 7.indd 373

T T

T T

T T

T T

T T

T

F

T

T

T

T

F

F

T

F

F

T

F

F

F

T

F

T

T

T

T

T

F

F

F

T

F

F

T

T

F

F

F F

F F

T F

T F

T F

T T

F F

F F

Duality Law Two WFFs A and A ′ are said to be duals of each other if either one can be obtained from the other by interchanging ∧ to ∨ and T to F. Also, if we have A and A ′ as duals of each other and X1, X2 … Xn are their propositional variables, then using De Morgan’s law, we can show that A (X1, X2 … Xn) = A ′ (X1, X2 … Xn) For example, Find the dual of A = (X ∧ Y ) ∨ (X ∧ Z). Let us say A ′ is the dual of A. Therefore, A ′ = (X ∧ Y ) ∨ (X ∧ Z).

Equivalent Well-Formed Formula A WFF whose value is T for all possible assignments of truth values to the propositional variables is called a tautology or a universally true formula. For example, Show that a = ((X → Y ) ∧ (Y → Z)) → (X → Z) is a tautology.

23-08-2017 05:32:01

374     Mathematical Logic 

X

Y

Z

X  → Y

Y  → Z

(X  → Y ) ∧ (Y → Z)

(X → Z)

a

T T T T F F F F

T T F F T T F F

T F T F T F T F

T T F F T T T T

T F T T T F T T

T F F F T F T T

T F T F T T T T

T T T T T T T T

As for all possible values of X, Y and Z we have a = T, a is a tautology.

LOGICAL IDENTITIES

A WFF whose value is F for all possible assignments of truth values to the propositional variables is called a contradiction.

There are certain laws or identities which are used for deducing other equivalences. Some of the important identities are:

For example, Show that a = (X ∧Y ) ∧ Y is a contradiction. X

Y

X  ∧ Y

|Y

a

T T F F

T F T F

T F F F

F T F T

F F F F

As for all possible values of X and Y, the value of a is F, a = (X ∧ Y ) ∧ Y is a contradiction. Two WFFs, say a and b, in propositional variables X1, X2, …, Xn are said to be logically equivalent if the formula → b is a tautology. Two equivalent WFFs are denoted as a← a≡b In a simpler language, a and b are equivalent if their truth table is same for all possible values of the proportional variables. For example, Show that (X ∨ (Y ∧ Z)) ≡ (X ∨ Y ) ∧ (X ∨ Z). Let a = X ∨ (Y ∧ Z) and b = (X ∨ Y ) ∧ (X ∨ Z). X

Y

Z

Y∧Z

a

X∨Y

X∨Z

b

T T T T F F F F

T T F F T T F F

T F T F T F T F

T F F F T F F F

T T T T T F F F

T T T T T T F F

T T T T T F T F

T T T T T F F F

1. Idempotent laws X ∧ X ≡ X and X ∨ X ≡ X 2. Commutative laws X ∨ Y ≡ Y ∨ X and X ∧ Y ≡ Y ∧ X 3. Absorption laws X ∨ (X ∨ Y ) = X and X ∧ (X ∧ Y ) ≡ X 4. Associative laws X ∨ (Y ∨ Z) ≡ (X ∨ Z) ∨ Z and X ∧ (Y ∧ Z) ≡ (X ∧ Y ) ∧ Z 5. Distributed laws X ∨ (Y ∧ Z) ≡ (X ∨ Y ) ∧ (X ∨ Z) and X ∧ (Y ∨ Z) = (X ∧ Y ) ∨ (X ∧ Z) 6. De Morgan’s laws (X ∨ Y ) = X ∧ Y, (X ∧ Y ) = X ∨ Y 7. Involution laws X = (X) 8. X ∨ T ≡ T, X∨F=X 9. X ∧ T ≡ X, X∧F=F 10. X ∧ X = F, X ∨ X = T 11. (X → Y ) ∧ (Y ) ≡ X 12. X → Y ≡ Y → X 13. X → Y ≡ (X ∨ Y ) For example,

As the truth table for a and b is same, a ≡ b.

Chapter 7.indd 374

 sing the identities described before, show that (X ∧ Y ) U ∨ (X ∧ Y ) ≡ X. L.H.S. = (X ∧ Y ) ∨ (X ∧ Y ) = X ∧ (Y ∨ Y )  [By Distributive law] = X ∧ T  [Using Identity 8] = X  [Using Identity 9] R.H.S. = X

22-08-2017 06:29:45

LOGICAL IDENTITIES     375

Normal Form We know that various WFFs having two propositional variables, say X and Y, are equivalent if they have the same truth table. Also, we know that the number of distinct truth tables for formulas in X and Y is 24. Hence, any formula in X and Y is equivalent to one of the 16 equivalent formulas. A method to reduce a given formula into an equivalent form is called a normal form. When referring to a normal form, we use “product” for conjunction (or AND), “sum” for disjunction (or OR) and “literal” for X or X, where X is a propositional variable. A product of literals is called an elementary product. A sum of literals is called an elementary sum. For example, X ∧ Y is elementary product. X ∨ Y is elementary sum.

3. If Xi and Xi are missing in the normal formula (say a), then replace a with (a ∧ Xi) ∨ (a ∧ Xi). 4. Repeat step (3) successively until all elementary products are reduced to sum of min terms. Apply idempotent laws to avoid repetition of min terms. For example, Obtain the principle disjunctive form of a = (X ∨ Z) → (X ∧ Y ). a = (X ∨ Z ) → (X ∧ Y ) = ((X ∨ Z)) ∨ (X ∧ Y ) = (X ∧ Z) ∨ (X ∧ Y ) Now, the above formula is in disjunctive normal form. a = (X ∧ Z ∧ Y ) ∨ (X ∧ Z ∧ Y ) ∨ (X ∧ Y ∧ Z) ∨ (X ∧ Y ∧ Z) Hence, a is now in principle disjunctive normal form.

Disjunction Normal Form

Functionally Complete Set of Connectives

A formula which is a sum of elementary products is in a disjunctive normal form. For example, X ∨ (Y ∧ Z) ∨ (Y ∧ Z) is in disjunctive normal form.

Any set of connectives which can be used to express every formula as an equivalent formula containing the same connectives from the set is called functionally complete set of connectives.

The following steps are used to obtain a disjunctive normal form from a given formula.

For example, {∧, } and {∨, } are functionally complete.

1. Eliminate “→" using logical identity. 2. Use De Morgan’s laws to eliminate  (negation) before sums or products. 3. Apply distributive laws successively to obtain disjunctive normal form by eliminating product of sums. For example, Obtain a disjunctive normal form from the formula a = X ∨ (Y ∨ (Y → Z)) a = X ∨ (Y ∨ (Y → Z)) = X ∨ Y ∨ Y ∨ Z

Principal Disjunctive Normal Form A min term for propositional variables X and Y is given by X ∧ Y, X ∧ Y, X ∧ Y and X ∧ Y. The number of min terms of n variable is 2n. If a formula is a sum of min terms, then it is said to be in principal disjunctive normal form. The following steps are used to obtain a principal disjunctive normal form from a given formula: 1. Follow steps (1)—(3) mentioned in Section 7.5.2 to obtain a disjunctive normal form. 2. Remove contradicting elementary products such as X ∧ X.

Chapter 7.indd 375

Some Other Connectives The other connectives used are described as follows: 1. Exclusive OR: It is denoted by v. If X and Y are two propositions, then X v Y is T when either X or Y is T but not both. The truth table for exclusive OR is given in Table 7. Table 7|   Truth table for exclusive OR

X

Y

X v Y

T T F F

T F T F

F T T F

2. NAND: NAND is denoted by ↑. If X and Y are two propositions, then X ↑ Y is T for all values expect when both X and Y are T. The connective NAND is logically equivalent to the negation of  conjunction, that is, their truth tables are same. The truth table for NAND is given in Table 8.

22-08-2017 06:29:46

376     Mathematical Logic 

Table 8 |   Truth table for NAND

X

Y

X↑Y

T T F F

T F T F

F T T T

3. NOR: NOR is denoted by ↓. If X and Y are two propositions, then X ↓ Y is T when the truth value of both X and Y is F and F for all other values. The connective NOR is logically equivalent to the negation of disjunction, that is, their truth tables are same. The truth table for NOR is given in Table 9. Table 9 |   Truth table for NOR

X

Y

X↓Y

T T F F

T F T F

F F F T

If a formula is a product of max terms, then it is said to be in principal conjunctive normal form. To obtain a principal conjunctive normal form of any formula, say a, we construct a principal disjunctive normal form of a and then apply negation on the result. For example,  ind the principal conjunctive normal form of a = X F ∨ (Y → Z).  e first calculate principal disjunctive normal form W of a. Thus,

a = (X ∨ (Y → Z)) = (X ∨ (Y ∨ Z)) = X ((Y ∧ Z)) = X ∧ Y ∧ Z

The above equation is the principal disjunction normal form of a. Hence, principal conjunction normal form of a is given by (X ∧ Y ∧ Z)

Conjunction and disjunction of X and Y can be expressed by NOR connective as follows: X ∧ Y = (X ↓ X) ↓ (Y ↓ Y ) X ∨ Y = (X ↓ Y ) ↓ (X ↓ Y ) Conjunction and disjunction of X and Y can be expressed by NAND connective as follows: X ∧ Y = (X ↑ Y ) ↑ (X ↑ Y ) X ∨ Y = (X ↑ X) ↑ (Y ↑ Y )

PROPOSITIONAL CALCULUS Propositional calculus is a formula in which formulae of a language are interpreted to represent propositions. Certain propositions are assumed to be T and based on these assumptions, other propositions are derived. A system of inference rules allow certain formulas to be derived, which are called theorems.

Conjunctive Normal Form

The propositions that are assumed to be true are called hypotheses or premises.

A formula which is a product of elementary sums is in a conjunctive normal form.

The proposition that is derived using a system of inference rules is called a conclusion.

For example, X ∧ (X ∨ Y ) ∧ (Y ∧ Z) is in conjunctive normal form.

The process of deriving a conclusion on the basis  of  assumption of hypotheses is called a valid argument.

Let us say a is in disjunctive normal form, then a is in conjunctive normal form. Hence, to obtain conjunctive form of any formula (a), we first obtain disjunctive normal form of a using the steps mentioned before and then find its negation. The resultant is a conjunctive normal form.

Principal Conjunctive Normal Form A max term for propositional variables X and Y is given by X ∨ Y, X ∨ Y, X ∨ Y and X ∨ Y. The number of max terms of n variables is 2n.

Chapter 7.indd 376

Rules of Inference In mathematical logic, the rule of inference is a logical form consisting of a function which takes premises, studies its syntax and returns a set of conclusions. The action of rule of inference is purely syntactic and does not need to preserve any semantic property. The rules of inference for valid arguments are given in Table 10.

22-08-2017 06:29:46

PROPOSITIONAL CALCULUS     377

Table 10 |   Rules of inference

S. no.

Rule

Definition

Notation

  1.

Modus ponens

X → Y ,X ∴Y

((X → Y ) ∧ X) → Y

  2.

Modus tollens

X → Y , ùY ∴ù X

((X → Y ) ∧Y ) → X

  3.

Biconditional introduction

X → Y ,Y → X ∴X↔Y

((X → Y ) ∧ (Y → X)) → (X ↔Y )

  4.

Biconditional elimination

(X ↔ Y ) ∴ (X → Y )

(X ↔ Y ) → (X → Y )

(X ↔ Y ) ∴ (Y → X )

(X ↔ Y ) → (Y → X)

X ∴ X∨Y

X → (X ∨ Y )

X → Y ,Z → Y ,X ∨ Z ∴Y

(((X → Y ) ∧ (Z → Y )) ∧ (X ∨ Z)) → Y

  5.

Disjunction introduction

  6.

Disjunction elimination

  7.

Disjunction syllogism

X ∨ Y , ùX ∴Y

((X ∨ Y ) ∧ X) → Y

  8.

Conjunction introduction

Y ∴ X∧Y

Y→X∧Y

  9.

Simplification

X∧Y ∴Y

(X ∧ Y ) → Y

10.

Hypothetical syllogism

X → Y ,Y → Z ∴X→Z

((X → Y ) ∧ (Y → Z)) → (X → Z)

11.

Constructive dilemma

X → Y ,Z → W,X ∨ Z ∴ Y ∨W

(((X → Y ) ∧ (Z →W)) ∧ (X ∨ Z)) → (Y ∨ W)

12.

Destructive dilemma

X → Y , Z → W , ù Y ∨ù W ∴ù X ∨ ùZ

(((X → Y ) ∧ (Z → W))∧ (Y ∨ W)) → (X ∨ Z)

13.

Absorption

X→Y ∴ X → (X ∧ Y )

(X → Y ) ↔ (X → (X ∧ Y ))

For example,

Let us consider the first two hypotheses/premises

Conclude W from the following premises: X→Y X→Z (Y ∧ Z) W∨X

Chapter 7.indd 377



X → Y

(1)



X → Z

(2)

Applying conjunction rule in Eqs. (1) and (2) gives

(X → Y ) ∧ (X → Z )

(3)

22-08-2017 06:29:48

378     Mathematical Logic 

Quantifier

The third hypothesis is given by

(Y ∧ Z)

(4)

Applying De Morgan’s law in Eq. (4) gives

Y ∨ Z

The phrases “for all,” “for any,” “every” and “for each” are called universal quantifiers. Universal quantifiers are denoted by ∀.

(5) For example,

Applying destructive dilemma in Eqs. (3) and (5) gives

X ∨ X = X

(6)

The fourth hypothesis is given by W ∨ X



(7)

Applying disjunctive syllogism in Eqs. (6) and (7) gives

W

PREDICATE CALCULUS Predicate calculus or first-order calculus deals with sentences having a common feature called predicates. Let us consider the following two propositions: 1. Ram is a cricketer. 2. Rahul is a cricketer. Now, there is no direct relation between the two propositions. However, we know that they have something in common, that is both Ram and Rahul share the property of being a cricketer. Hence, the above propositions can be replaced by a single statement “X is a cricketer.” Here, X is a variable and “is a cricketer” is called a predicate.

Predicates Predicates are functions of zero or more variables. They can be T or F depending on the values of their arguments. Predicates are denoted by P(x) where P denotes the common property and x is the variable.

“All students are sitting” can be denoted by ∀x c(x), where c(x) denotes “x is sitting” and takes the universe of discourse as a set of all students. The phrases “there exists,” “for some” and “for at least one” are called existential quantifiers. Existential quantifiers are denoted by ∃. For example, “Some students are sitting” can be denoted by ∃ x c(x), where c(x) denotes “X is sitting” and takes the universe of discourses as a set of all students. WFF for predicate calculus can be formed using the following rules: 1. P(x1, x2, …, xn) is a WFF if P is a predicate involving n variables x1, x2, …, xn. 2. If a is a WFF, then a is also a WFF. 3. If a and b are WFFs, then a ∧ b, a ∨ b, a → b and a→ ← b are also WFFs. 4. ∀X(a) and ∃X(a) are WFFs if a is a WFF and X is a variable. 5. An array is a WFF if and only if is obtained by finite amount of applications of rules (1)—(4) mentioned above. Consider a and b as two predicate formulas of variables x1, x2, …, xn in the universe of discourse (U), then a and b are equivalent (i.e. a ≡ b) over U for every possible value to each variable in a and b, if they have the same truth values. If ∀x P(x) or ∃x P(x) occurs as part of a, where a is a predicate formula, then such a part is called an x-bound part of a, and occurrence of x is called bound occurrence of x. If a predicate variable in a is free in any part of a, then it is not a bounded occurrence and the occurrence is called free.

It should be noted that P(x) is not a proposition since it involves a variable x and hence we cannot calculate a truth value for P(x). However, if we replace x by a constant or an individual value, then we get a proposition.

In a = ∃x P(x, Y ), the occurrence of x is a bound ­occurrence and the occurrence of y is a free occurrence.

The universe for a sentence involving a predicate is the set of all possible values that the variable of the predicate can take. For example, universe for P(x): “x is an odd number” can be taken as the set of all odd integers.

Predicate formulas where all the variables are quantified are propositional formulas. Therefore, all the rules of inference for propositional calculus are applicable to predicate calculus. Table 11 shows the rules of inference for addition and deletion of quantifiers.

Chapter 7.indd 378

For example,

22-08-2017 06:29:49

SOLVED EXAMPLES     379

Table 11|   Rules of inference for addition and deletion of quantifiers

S. no.

Rule of Inference

1.

Universal generalization

2.

Universal instantiation

3.

Existential generalization

4.

Existential instantiation

Definition

Condition

P (x) ∀xP (x)

x should not be free in any of the given premises.

∀xP (x) ∴ P (c /x)

P(x/c) is the result of substituting c for all occurrences of x in P.

P (c) ∴ ∃xP (x) ∃xP (x) ∴ P (c)

Satisfiable, Unsatisfiable and Valid Formulas A predicate formula having truth value of resulting proposition as T for some assignment of values to predicate variables is called satisfiable predicate formula.

c replaces all free instances of x within P(x).

Where c is any arbitrary element for which P(c) is true.

A predicate formula having truth value of resulting proposition as F for all possible assignment of values to predicate variables is called unsatisfiable predicate formula. A predicate formula having truth value of resulting proposition as T for all possible assignment of values to predicate variables is called valid predicate formula.

SOLVED EXAMPLES 1. Conclude X from X ∨ Y, (Y ∧ Z) and Z.

3. Prove that [ , →] is functionally complete.

Solution:  We have two premises

Z (Y ∨ Z)

(i) (ii)

Applying De Morgan’s law in Eq. (ii) gives

Y ∨ Z

Hence, { , ∨} = { , →} (iii)

Applying disjunction syllogism in Eqs. (i) and (iii) gives

Y

(iv)

We have another premise,

X ∨ Y

Solution:  We know that X → Y = X ∨ Y Also, X ∨ Y = (X ↑ Y ) ↑ (Y ↑ Y ) and X ↑ X = (X ∧ X) = X

Chapter 7.indd 379

4. Show that truth value of the following formula is independent of its components. → ( X ∨ Y ) (X → Y ) ←

2. Find the equivalent formula for (X → Y ) containing ↑ only.

= (X) ↑ (Y ) = X ↑ Y

Thus, { , →} is functionally complete.

(v)

Applying disjunction syllogism in Eqs. (iv) and (v) gives X, which is the required result.

Therefore, X ∨ Y = (X ↑ X) ↑ (Y ↑ Y )

Solution:  Let us consider X ∧ Y ≡ (X ∨ Q), hence { , ∨} is functionally complete. Now, X ∨ Y = X → Y, hence we can replace ∨ with  and →.

Solution:  Consider → ( X ∨ Y ) (X → Y ) ← Now, X ∨ Y = X → Y → (X → Y ). Therefore, (X → Y ) ← → X is T regardless of Also, truth value of X ← value X. → (X → Y ) has a truth Therefore, (X → Y ) ← value of T for all values of X and Y. Hence, it is a tautology.

22-08-2017 06:29:50

380     Mathematical Logic 

Substituting the value of X in a gives

5. Show that {∨, ∧} is functionally incomplete. Solution:  Let us consider (X ∨ X = T). This formula cannot be written as a combination of ∧ and ∨ alone. Hence, {∨, ∧} is functionally incomplete. 6. Show that [(X ∨ Y ) → (Y → X)] is not a tautology. Solution:  Let us simplify the above expression

a = (X ∧ Y ∧ Z) ∨ (X ∧ Y ∧ Z) ∨ (X ∧ Y ∧ Z) ∨ (X ∧ Y ∧ Z) ∨ (X ∧ Y ∧ Z) 9. Check the validity of the following argument: If Jack has completed B.Sc. or M.Sc., then he is assured a good job. If Jack is assured of a good job, he is content. Jack is not content, so Jack has not completed M.Sc.

[(X ∨ Y ) → (Y → X)] Solution:  The propositions can be formed as follows:

= [(X ∨ Y ) → (Y ∨ X)] = [(X ∨ Y ) ∨ (Y ∨ X)] = [(X ∧ Y ) ∨ (Y ∨X)]

W denotes “Jack has completed B.Sc.”

= [X ∨ Y]

X denotes “Jack has completed M.Sc.” Y denotes “Jack is assured of a good job.”

The above expression is F for X = 1 and Y = 0. Hence, it is not a tautology.

Z denotes “Jack is content.” The given premises are:

7. Show that proposition Z is a logical consequence of the formula X ∧ (X → (Y ∨ Z)) ∧ (Y → X) Solution:  If we want to show that any formula B is a logical consequence of formula A, then A → B is a tautology. Now if A = X ∧ (X → (Y ∨ Z)) ∧ (Y → X) and B = Z, consider the following truth table:

(W ∨ X) → Y Y → Z Z



(1) (2) (3)

Applying hypothetical syllogism in Eqs. (1) and (2) gives (W ∨ X) → Z



(4)

Applying modus tollens in Eqs. (3) and (4) gives X Y Z Y ∨ Z X → |X Y → |X A A → B (Y ∨ Z)



T

T T

T

T

F

F

F

T

Applying De Morgan’s law in Eq. (5) gives

T T

T F F T

T T

T T

F F

F T

F T

T T

T F F F

F T T F

F T F T

F T T T

F T T T

F T T T

T T T T

F F F F

T T T T

F

F

F

T

T

T

T

F

T

(W ∨ X)



W ∧ W

(5)

(6)

Simplification of Eq. (6) gives

8. Obtain principal disjunctive normal from of



X

Thus, the argument is valid. 10. Discuss the validity of the following arguments: All graduates are educated. Garima is a graduate. Therefore, Garima is educated.

a = X ∨ (X ∧ Y ∧ Z) Solution:  As we can see, a = X ∨ (X ∧ Y ∧ Z) is already in principal disjunctive normal form. However, we have to add the missing terms. Now, X = (X ∧ Y ) ∨ (X ∨ Y ) = ( X ∧ Y ∧ Z) ∨ (X ∧ Y ∧ Z) ∨ (X ∧Y ∧ Z) ∨ (X ∧Y ∧ Z)

Chapter 7.indd 380

Solution:  Let A(x) denotes “x is a graduate” B(x) denotes “x is educated” G denotes “Garima”. The premises are:

∀x (A(x) → B(x)) A(G)

(1) (2)

The conclusion is B(R).

22-08-2017 06:29:50

PRACTICE EXERCISE     381

Now, applying universal instantiation in Eq. (1) gives A(R) → B(R)



(3)

Applying modus ponens in Eqs. (2) and (3) gives B(R) Thus, the conclusion is valid.

PRACTICE EXERCISE 1. Find an equivalent formula for (X → Y ) ∨ Z containing ↑ only. (a) (b) (c) (d)

((X ↑ (Y ↑ Y )) ↑ (X ↑ (Y ↑ Y )) ↑ (Z ↑ Z )) (X ↑ Y ) ↑ ((X ↑ X ) ↑ Y ) ↑ (Y ↑ Z ) (X ↑ X ) ↑ (X ↑ (Y ↑ Y )) ↑ (Y ↑ Z ) ((X ↑ X ) ↑ Y ) ↑ (X ↑ (Y ↑ Y )) ↑ (X ↑ Y ↑ Z )

2. Find an equivalent for (X → Y ) containing ↓ only. (a) (X ↓ Y ) ↓ ( X ↓ Y ) (b) ( X ↓ Y ) ↓ ( X ↓ Y ) (c) ( X ↓ Y ) ↓ ( X ↓ Y ) (d) (X ↓ Y ) ↓ ( X ↓ Y ) 3. Substitution instance of a tautology is a (a) contradiction (b) tautology (c) can be (a) or (b) (d) none of the above

((X ((X ((X ((X

↓ Y ) ↓ (X ↓ Y )) ↓ X ) ↓ (Y ↓ Y )) ↓ Y ) ↓ (Y ↓ Y )) ↓ ((X ↓ Y ) ↓ (X ↓ X )) ↓ X ) ↓ (Y ↓ Y )) ↓ ((X ↓ X ) ↓ (Y ↓ Y ))

5. {↑} connective is

6. (X → Y ) is equivalent to (b) X ∧ Y (d) X ∨ Y

7. The translation of the sentence “Some cats are white but all swans are white” using the following notations: A(x): x is white B(x): x is cat C(x): x is swan (∀x): for all x (∃x): for some x

Chapter 7.indd 381

8. The negation of statement ∀x ∃y, P(x, Y ) is given by (a) ∃x∀y, P (x, y)

(b) ∃ x∀ y,  P (x, y)

(c) ∀ x∀ y, P (x, y)

(d) ∃ x ∃ y, P (x, y)

9. x → (y → z) is equivalent to (a) (x ∨ y) → z

(b) (x ∧ y) → z

(c) (x ∨ y) → z

(d) (x ∧ y) → z

(a) ∃x P(x) (c) ∃ x P(x)

(b) ∀x P(x) (d)  ∀x P(x)

11. Which of the following is/are tautologies? (a) (X ∨ Y ) ↔ Y

(b) (X ∨ (X → Y )) → X

(c) ((X ∨ Y ) ∧ X ) → Y (d) ((X ∨ Y )∧ X ) → Y 12. Which one of the following is a tautology?

(a) functionally complete (b) functionally incomplete (c) cannot say (d) none of the above

(a) X ∧Y (c) X ∨ Y

(∃x)( B(x) ∧ A(X )) ∧ (∀x)(C(x) → A(x)) (∃x)( B(x) ∧ A(x)) ∧ (∀x)(C(x) ∧ A(x)) (∃x)(B(x) → A(x)) ∧ (∀x)(C(x) ∧ A(x)) (∃x)(B(x) → A(x)) ∧ (∀x)(C(x) → A(x))

10. In predicate logic,  ∀ P(x) is equivalent to

4. Express X ↑ Y in terms of ↓ only. (a) (b) (c) (d)

(a) (b) (c) (d)

(a) X → (X ∨ Y )

(b) X ∧  Y

(c) X → (X ∧ Y )

(d) (X ∨ Y ) → (X ∧ Y )

13. Which one of the following is a declarative statement? (a) It is beautiful (b) It is not beautiful (c) He says “It is beautiful” (d) It may or may not be beautiful 14. Evaluate (X ∨ Y ) ∧ (X → Z) ∧ (Y → Z). (a) X (c) Z

(b) Y (d) Z ∧ (Y ∨ X)

15. If X is a tautology and Y is any other formula, then (X ∨ Y ) is a (a) tautology (b) contradiction (c) well-formed formula (d) none of these

22-08-2017 06:29:58

382     Mathematical Logic 

(a) ( X∧ Y ∧ Z) ∨ (X ∧ Y ∧ Z) ∨ (X ∧ Y ∨ (X ∧ Y ∧ Z) (b) (X ∨ Y ∨ Z) ∧ (X ∨ Y ∨ Z) ∧ (X ∨ Y ∧ (X ∨ Y ∨ Z) (c) (X ∧ Y ∧ Z) ∨ (X ∧ Y ∧ Z) ∨ (X ∧ Y ∨ (X ∧ Y ∨ Z) (d) (X ∨ Y ∨ Z) ∧ (X ∨ Y ∨ Z) ∧ (X ∨ Y ∧ (X ∨ Y ∨ Z)

16. Which one of the following is a functionally complete set? (a) {↓, ∧} (c) {↑}

(b) { ∧ ∨} (d) {→, ∧}

17. What is the conclusion of the following premises? A → B, B → C, C → (D ∧ E), A (a) E (c) C

(b) D (d) A → B

∧ Z) ∨ Z) ∧ Z) ∨ Z)

19. (X ∧ Y ) ∧ Y is a

18. For a given formula a, the truth table is given below. Find the principal disjunctive form. X

Y

Z

a

T T T T F F F F

T T F F T T F F

T F T F T F T F

T F F T T F F T

(a) tautology (c) contradiction

(b) well-formed formula (d) none of these

20. Which one of the following is True? a = (X ∨ Y ) ∧ (Y →Z) ∨ (Z ∨ X) (a) a is tautology (b) a is a contradiction (c) If X is True, Y is True and Z is False, then a is True (d) If X is True, Y is False and Z is False, then a is True

ANSWERS 1. (a)

5. (a)

9. (b)

13. (c)

17. (b)

2. (c)

6. (a)

10. (c)

14. (d)

18. (c)

3. (b)

7. (d)

11. (d)

15. (a)

19. (c)

4. (d)

8. (b)

12. (a)

16. (c)

20. (d)

EXPLANATIONS AND HINTS 1. (a) We know that X ↑ X = (X ∧ X) = X

2. (c) We know that X ↓ X = X

Also X ∧ Y = (X ↑ Y ) ↑ (X ↑ Y )

Also X ∨ Y = (X ↓ Y ) ↓ (X ↓ Y )

and X ∨ X = (X ↑ X) ↑ (Y ↑ Y )

X ∧ Y = (X ↓ X) ↓ (Y ↓ Y )

Hence, (X → X) ∨ X = X ∨ Y ∨ Z = (X ∨ Y ) ∨ Z

Hence, (X → Y ) = X ∨ Y

= ((ùX ∨ Y ) ↑ (ù X ∨ Y )) ↑ (Z ↑ Z )

= (X ↓ Y ) ↓ (X ↓ Y ) ∨ Y ) ↑ (ù X ∨ Y )) ↑ (Z ↑ Z ) = ((ùX ↑ùX ) ↑ (Y ↑ Y )) ↑ ((ù X ↑ù X) ↑ (Y ↑ Y )) ↑ (Z ↑ Z ) 3. (b) The structure of tautology will not change ↑ùX ) ↑ (Y ↑ Y )) ↑ ((ù X ↑ù X) ↑ (Y ↑ Y )) ↑ (Z ↑ Z ) after substitution of a formula. Hence, answer is = ((X ↑ (Y ↑ Y )) ↑ (X ↑ (Y ↑ Y )) ↑ (Z ↑ Z ) tautology.

Chapter 7.indd 382

22-08-2017 06:29:59

EXPLANATIONS AND HINTS      383

10. (c)  ∀ P ( x ) is equivalent to ∃ x P ( x ) .

4. (d) We know that X ↑ Y = (X ∧ Y ) = (X ∧ Y ) ↓ (X ∧ Y )

11. (d) Consider (X ∨ Y ) ↔ Y By assigning truth values, we get

= ((X ↓ X ) ↓ (Y ↓ Y )) ↓ ((X ↓ X ) ↓ (Y ↓ Y )) X

Y

XÚY

( X ∨ Y ) ↔Y

T

T

T

T

T

F

T

F

F

T

T

T

F

F

F

T

5. (a) We know that X ∧ Y = (X ↑ Y ) ↑ (X ↑ Y ) X ∧ Y = (X ↑ X ) ↑ (Y ↑ Y ) X = X ↑ X Hence, {↑} is functionally complete since any formula consisting of ∧, ∨ and  can be formed consisting of ↑ alone.

Hence, it is not a tautology. Consider (X ∨ (X ∨ Y )) → X .

6. (a) We have (X → Y )

Simplifying the above equation gives (X → Y ) = X ∨ Y

((X ∨ X )(X ∨ Y )) → X ((X ∨ X )(X ∨ Y )) → X = (X → (X ∨ Y )) → X = (X → (X ∨ Y )) → X

(X → Y ) = (X ∨ Y ) Applying De Morgan’s law, we get X ∧ Y



= (X ∨ (X ∨ Y )) → X = (X ∨ (X ∨ Y )) → X = ∨Y = ((X T ∨∨ Y(X )→ X)) → X = (T ∨ Y ) → X = ∨X Y )= →XX = (TT ∨ =T ∨X=X =T ∨X=X

7. (d) “Some cats are white” can be denoted by ∃x (B(x) → A(x)) “All Swans are white” can be denoted by ∀x (C(x) → A(x))

By assigning truth values, we get

Hence, joining the two notations, we get ∃x (B(x) → A(x)) ∧ ∀x (C(x) → A(x)) 8. (b) Negation of the statement ∀x, ∃y, P(x, Y ) is given by ∃x ∀y, P(x, Y ) 9. (b) We have, x → (y →z)

X

T↑X

T F

T F

Hence, it is not a tautology. Consider ((X ∨ Y )∨ X ) → Y (1) X Y (X ∨ Y ) (X ∨ Y ) ∧ X ((X ∨ Y ) ∨ X ) ↑ Y

Now, we know that x → y = x ∨ y Applying the identity in Eq. (i), we get x → (y ∨ z) = (x → y ) ∨ (x → z)

T T F F

T F T F

T T T F

T T F F

T T F T

Applying the identity again, we get x ∨ y ∨ x ∨ z Now, using idempotent laws, we know that x ∨ x = x Hence, x ∨ y ∨ z

Consider ((X ∨ Y )∧ X ) → Y Simplifying the above equation gives (X ∧  X ∨ (Y ∨  X )) → Y

Applying De Morgan’s law, (x ∧ y ) ∨ z = (x ∧ y ) → z

Chapter 7.indd 383

Hence, it is not a tautology.

= (Y ∧= X()Y→∧Y X) → Y = (Y ∧=  X()Y∨∧Y X )∨ Y =  Y ∨=X Y ∨∨ Y X∨ Y XT ∨ X = T ∨= =T =T

De Morgan ’s law) ’s law) (By applying (By applying De Morgan (Y ∨  Y(Y=∨T Y ) = T) (T ∨ X(T=∨TX ) = T)

22-08-2017 06:30:03

= (Y ∧ X ) → Y (Y ∧ X )∨ YLogic  384    =Mathematical = Y ∨ X ∨ Y =T∨X =T

(By applying De Morgan’s law) 17. (b) We have (Y ∨  Y = T ) (T ∨ X = T )

A → B

(1)



B → C

(2)



C → (D ∧ E)

(3)



A

(4)

Hence, it is a tautology. 12. (a) Consider X → (X ∨ Y )

Applying modus ponens in Eqs. (4) and (1) gives B (5) Applying modus ponens in Eqs. (5) and (2) gives

By assigning truth values, we get X

Y

XÚY

X ↑ (X ∨ Y )

T

T

T

T

T

F

T

T

F

T

T

T

F

F

F

T

C (6) Applying modus ponens in Eqs. (6) and (3) gives D ∧ E Simplification of Eq. (7) gives

(7)

D 18. (c) We have to write the formula for values when a is True.

Hence, it is a tautology. Note: Similarly, the rest of the three options can be proved to be not a tautology by assigning truth values.

The min terms corresponding to rows when a is True are X ∧ Y ∧ Z, X ∧ Y ∧ Z, X ∧ Y ∧ Z and X ∧ Y ∧ Z. Principal disjunctive normal form of a is

13. (c) `He says “It is beautiful.”’ is a declarative ­statement.

(X ∧ Y ∧ Z) ∨ (X ∧ Y ∧ Z) ∨ (X ∧ Y ∧ Z) ∨ (X ∧ Y ∧ Z)

14. (d) We have

(X ∨ Y ) ∧ (X → Z ) ∧ (Y → Z ) 



X → Z = X ∨ Z

and



Y → Z = Y ∨ Z 

(1)

19. (c) We have (X ∧ Y ) ∧ Y By assigning truth values and forming truth table, we get

(2) (3)

X

Y

Y

X∧Y

(X ∧ Y ) ∧Y

T T F F

T F T F

F T F T

T F F F

F F F F

Substituting Eqs. (2) and (3) in Eq. (1) gives (X ∨ Y ) ∧ ( X ∨ Z ) ∧ ( Y ∨ Z ) = ((X ∧ Z ) ∧ ( X ∧ Y )∨ (Y ∧ Z ))( Y ∨ Z ) = (X ∧ Z ∧ Y )∨ (X ∧ Z ) ∨ ( X ∧ Y ∧ Z )∨ (Y ∧ Z ) = Z ∧ ((X ∧ Y )∨ X ∨ ( X ∧ Y )∨ (Y )) = Z ∧ (Y ∨ X ) 15. (a) As X is a tautology, means X is true for all possible inputs. So, X ∨ Y is also true for values since disjunction of any variable with a true value is always true. 16. (c) Because all formulas using ∧, ∨ and  can be formed using ↑, {↑} is a functionally complete set.

Chapter 7.indd 384

Hence, the formula is a contradiction because value is False for all possible values of X and Y. 20. (d) a is not a tautology because for X = F, Y = T and Z = F, a = False. a is not a tautology because for X = F, Y = T and Z = T, a = True. Now, for X = True, Y = False and Z = False, a = True. Hence, the correct option is “If X is True, Y is False and Z is False, then a is True.”

22-08-2017 06:30:06

SOLVED GATE PREVIOUS YEARS’ QUESTIONS     385

SOLVED GATE PREVIOUS YEARS’ QUESTIONS 1. Let (S,≤) be a partial order with two minimal elements a and b, and a maximum element c. Let P : S → [True, False ] be a predicate defined on S. Suppose that P (a) = True, P (b ) = False and P (x) ⇒ P (y ) for all x, y ∈ S satisfying x ≤ y, where ⇒ stands for logical implication. Which one of the following statements CANNOT be true? (a) P(x) = True for all x ∈ S such that x ≠ b (b) P(x) = False for all x ∈ S such that x ≠ a and x ≠ c (c) P(x) = False for all x ∈ S such that b ≤ x and x≠c (d) P(x) = False for all x ∈ S such that a ≤ x and a ≤ x (GATE 2003, 2 Marks) Solution:  ((SS,≤  ≤)) is partial order in which S is reflexive, symmetric and transitive. We have,

(a) I1 satisfies a , I2 does not (b) I2 satisfies a , I1 does not (c) Neither I1 nor I2 satisfies a (d) Both I1 and I2 satisfy a (GATE 2003, 2 Marks) Solution:  We have a = ( ∀ x ) Px ⇔ ( ∀ y ) [ Qxy ⇔ ¬Qyy ]  ⇒ ( ∀ x ) [ ¬P ] I1: Domain: the set of natural numbers Px = `x is a prime number’ = 2, 3, 5, 7, 11, 13, … Qxy = `y divides x’ = 2, 3, 5, 7, 11, 13, … I2: Px = `x is a composite number means number is not a prime number’. = 4, 6, 8, 9, 10, 12, 14, … Qxy = 2, 3, 5, 7, 11, 13, … Thus, I1 satisfies a, I2 does not. Ans. (a)

P (a) = True, P (b ) = False P (x ) ⇒ P (y )

3. Consider the following logic program P A (x) ← B (x, y ) , C (y )

If P(x), then it is given that P(Y ) and a, y ∈ S satisfies x < y Therefore, −P (x) ∪ P (y )

← B (x, x) Which one of the following first-order sentences is equivalent to P? (a) (∀ x)[(∃ y)B(x, y) ∧ C(y) ⇒ A(x)] ∧ ¬(∃ x)[B(x, x)]

⇒ False ∪ False ⇒ False (∀ x)[(∃ y)B(x, y) ∧ C(y) ⇒ A(x)] ∧ ¬(∃ x)[B(x, x)]

Hence, P (x) = False for all x ∈ S, such that (b) (∀ x)[(∀ y)[B(x, y) ∧ C(y)] ⇒ A(x) ∧ ¬(∃ x)[B(x, x)] b ≤ x and x ≠ c. (∀ x)[(∀ y)[B(x, y) ∧ C(y)] ⇒ A(x) ∧ ¬(∃ x)[B(x, x)] Ans. (c) (c) (∀ x)[(∃ y)[B(x, y) ∧ C(y)] ⇒ A(x)] ∧ ¬(∃ x)[B(x, x)] 2. Consider the following formula a and its two interpretations I1 and I2(∀ x)[(∃ y)[B(x, y) ∧ C(y)] ⇒ A(x)] ∧ ¬(∃ x)[B(x, x)] a : ( ∀x) Px ⇔ ( ∀y )[Qxy ⇔ ¬Qyy ] ⇒ ( ∀x)[¬P ] (d) (∀ x)[(∀ y)[B(x, y) ∧ C(y)] ⇒ A(x)] ∧ ¬(∃ x)[B(x, x)] I1: Domain: the set of natural (∀ x)[(∀numbers y)[B(x, y) ∧ C(y)] ⇒ A(x)] ∧ ¬(∃ x)[B(x, x)] Px = `x is a prime number’ (GATE 2003, 2 Marks)

Qxy = `y divides x’ I2: Same as I1 except that Px = `x is a composite number’. Which one of the following statements is true?

Chapter 7.indd 385

Solution:  We have A(x) ← B(x, y), C(y) ← B(x, x)

22-08-2017 06:30:14

386     Mathematical Logic 

Now, we can write

Some boys in the class are taller than all the girls

B (x, x) → B (x, y ) , C (y ) → A (x)

Note: taller (x, y ) is true if x is taller than y. (a) (∃ x)(boy(x) → (∀ y)(girl(y) ∧ taller(x, y)))

B(x, x) →  [B (x, y ), C (y )] ∪ A(x) B(x, x) ∪  [B (x, y ), C (y )] ∪ A(x)

(b) (∃ x)(boy(x) ∧ (∀ y)(girl(y) ∧ taller(x, y)))

Also, we can write

(c) (∃ x)(boy(x) → (∀ y)(girl(y) → taller(x, y)))

∀ (B(x, x) ∪ (B (x, y )), C (y )) ∪ A(x)

(d) (∃ x)(boy(x) → (∀ y)(girl(y) ∧ taller(x, y)))

Rewriting option (c), (GATE 2004, 1 Mark) (∀ x) [(∃ y)[B(x, y) ∧ C(y)]] ∪ A(x) ∪  (∃ (x)[B(x, x)])

Solution: 

⇒ (∀ x) [(∀ y)  [B(x, y) ∧ C(y)] ∪ A(x) ∪ (∀ x) [B(x, x)]

(∃ x)(boy(x) → (∀ y)(girl(y) → taller(x, y)))

⇒ ( ∀xy ) [B(x, Y ) ∪ C(y ) ∪ A(x) B(x, x)]



Ans. (c)

6. The following propositional statement is

⇒ ∀ [(B(x, y ), C(y )) ∪ A(x) B(x, x)]

(P → (Q ∨ R)) → ((P ∧ Q) → R) Thus, the correct answer is (c). Ans. (c) 4. The following resolution rule is used in logic programming: Derive clause (P ∨ Q)

(P ∨ R ) , (Q ∨ ¬R )

from clauses (P ∨ R ) , (Q ∨ ¬R )

Solution:  Let us form the truth table for the given statement.

Which one of the following statements related to this rule is FALSE? (a) ((P ∨ R) ∧ (Q ∨ ¬R)) ⇒ (P ∨ Q) valid

is

(b) (P ∨ Q) ⇒ ((P ∨ R) ∧ (Q ∨ ¬R)) valid

is

P

Q

R

(P → (Q ∨ R)) ↑ ((P ∧ Q ) → R)

F F F F T T T T

F F T T F F T T

F T F T F T F T

T T T T T T F T

logically

logically

(c) (P ∨ Q) is satisfiable if and only if (P ∨ R ) ∧ (Q ∨ ¬R )

(P ∨ R ) ∧ (Q ∨ ¬R ) is satisfiable (d) (P ∨ Q) ⇒ FALSE if and only if both P and Q are unsatisfiable (GATE 2003, 2 Marks) Solution:  If P = True, Q = False, and R = True, then (T ∪ F ) ⇒ ((T ∪ T ) ∧ (F ∪ F )) T ⇒ (T ∧ F ) T ⇒F \ F ∪ F = False (is not valid)

Hence, we can see from the truth table that the statement is true for some values and false for some. Hence, it is satisfiable but not valid. Ans. (a) 7. Let P, Q and R be three atomic propositional assertions. Let X denotes (P ∨ Q) → R and Y denotes (P → R ) ∨ (Q → R ) .

Hence, “(P ∨ Q) ⇒ ((P ∨ R) ∧ (Q ∨ ¬R)) is ­logically valid” is the incorrect statement. Ans. (b) 5. Identify the correct translation into logical notation of the following assertion.

Chapter 7.indd 386

(a) satisfiable but not valid (b) valid (c) contradiction (d) none of the above (GATE 2004, 2 Marks)

Which one of the following is a tautology? (a) X  ≡Y (b) X → Y (c) Y → X (d) Y → X (GATE 2005, 2 Marks)

22-08-2017 06:30:25

SOLVED GATE PREVIOUS YEARS’ QUESTIONS     387

Solution:  Let us form the truth table for the statement.

P Q

R

X Y

F

F

F

T

T

F

F

T

T

T

F

T

F

F

T

F

T

T

T

T

T

F

F

F

T

T

F

T

T

T

T

T

F

F

F

T

T

T

T

T

Y­X T

X­Y

9. Which one of the first-order predicate calculus statements given below correctly expresses the following English statement? Tigers and lions attack if they are hungry or threatened.

(a) ∀ (x)[(tiger(x) ∧ lion(x)) → ((hungry (x)∨ threatened (x)) → attack

T

T T ∀ (x)[(tiger(x) ∧ lion(x)) → ((hungry (x)∨ threatened (x)) → attacks (x))] F T (b) ∀ (x)[(tiger(x)∨ lion(x)) → ((hungry (x)∨ threatened (x)) ∧ attacks T T ∀ (x)[(tiger(x)∨ lion(x)) → ((hungry (x)∨ threatened (x)) ∧ attacks((x))] F T (c) ∀ (x)[(tiger(x)∨ lion(x)) → (attacks(x) → (hungry(x)∨ threatened( T T ∀ (x)[(tiger (x)∨ lion(x)) → (attacks(x) → (hungry(x)∨ threatened(x)))] T T (d) ∀ (x)[(tiger (x)∨ lion(x)) → ((hungry(x)∨ threatened(x)) → attack T T ∀ (x)[(tiger (x)∨ lion(x)) → ((hungry(x)∨ threatened(x)) → attacks((x))]

Hence, from the truth table, we can see that X → Y is a tautology. Ans. (b) 8. What is the first-order predicate calculus statement equivalent to the following? Every teacher is liked by some student (a) ∀ (x)[teacher(x) → ∃ (y)[student(y) → likes(y, x)]]

her(x) → ∃ (y)[student(y) → likes(y, x)]] (b) ∀ (x)[teacher(x) → ∃ (y)[student(y) ∧ likes(y, x)]]

cher(x) → ∃ (y)[student(y) ∧ likes(y, x)]]

(c) ∃ (y)∀ (x)[teacher(x) → [student(y) ∧ likes(y, x)]] )[teacher(x) → [student(y) ∧ likes(y, x)]]

(GATE 2006, 2 Marks) Solution:  The given statement can be interpreted as follows: Consider x to be a lion or a tiger. If x is hungry or is threatened, it will attack. P (x) = lion(x)∨ tiger(x) Q(x) = hungry(x)∨ threatened(x) R(x) = attacks(x) P (x) → (Q(x) → R(x)) Expanding, we get

∀ (x)[(tiger(x)∨ lion (x)) → ((hungry (x)∨ threatened (x)) → attacks

(d) ∀ (x)[teacher(x) ∧ ∃ (y)[student(y) → likes(y, x)]] ∀ (x)[(tiger(x)∨ lion (x)) → ((hungry (x)∨ threatened (x)) → attacks (x))] cher(x) ∧ ∃ (y)[student(y) → likes(y, x)]] Ans. (d) (GATE 2005, 2 Marks) 10. Consider the following propositional statements: Solution:  For the statement “Every teacher is liked by some student”, the logical expression is P1 : ((( A ∧ B) → C )) ≡ ((A → C ) ∧ (B → C )) given by P2 : (((A ∨ B) → C )) ≡ ((A → C )∨ (B → C ))     ∀ (x)  teacher (x) → ∃ (y )  student (y ) ∧ likes (y, x)  Which one of the following is true? where likes (y, x) means y likes x, such that y ­represents the student and x represents the teacher. Ans. (b)

(a) P1 is a tautology, but not P2 (b) P2 is a tautology, but not P1 (c) P1 and P2 are both tautologies (d) Both P1 and P2 are not tautologies (GATE 2006, 2 Marks)

Chapter 7.indd 387

22-08-2017 06:30:30

388     Mathematical Logic 

Solution:  Let us make the truth table for the given statements. A

B

C

F F F F T T T T

F F T T F F T T

F T F T F T F T

A Ú B A Ù B (A ∧ B) ↑ C F F T T T T T T

F F F F F F T T

(A ∨ B) ↑ C

((A ↑ C ) ∧ (B ↑ C ))

((A ↑ C )∨ (B ↑ C ))

T T T T T T F T

T T F T F T F T

T T T T T T F T

T T F T F T F T

Hence, from the truth table, we can say that both P1 and P2 are not tautologies.  Ans. (d)

13. Which one of the following is the most appropriate logical formula to represent the statement: “Gold and silver ornaments are precious”

11. Let Graph(x) be a predicate which denotes that x is a graph. Let Connected(x) be a predicate which denotes that x is connected. Which of the following first-order logic sentences DOES NOT represent the statement: “Not every graph is connected”? (a) ¬∀ x(Graph(x) ⇒ Connected(x))

The following notations are used: G(x): x is a gold ornament. S(x): x is a silver ornament. P(x): x is precious.

(b) ∃ x(Graph(x) ∧ ¬ Connected(x))

(a) ∀ x(P (x) → (G(x) ∧ S(x)))

(c) ¬∀ x(¬ Graph(x)∨ Connected(x))

(b) ∀ x((G(x) ∧ S(x)) → P (x))

(d) ∀ x(Graph(x) ⇒ ¬ Connected(x))

(c) ∃ x((G(x) ∧ S(x)) → P (x))

(GATE 2007, 2 Marks) Solution:  ∀x (Graph (x) ⇒ ¬Connected ( x)) represents “for every x if x is a graph then it is not connected”. Ans. (d) 12. P and Q are two propositions. Which of the ­following logical expressions are equivalent? I. P∨ ∼ Q II. ∼ (∼ P ∧ Q) III. (P ∧ Q)∨ (P ∧ ∼ Q)∨ (∼ P ∧ ∼ Q) IV. (P ∧ Q)∨ (P ∧ ∼ Q)∨ (∼ P ∧ Q) (a) Only I and II (b) Only I, II and III (c) Only I, II and IV (d) All of I, II, III and IV (GATE 2008, 2 Marks) Solution:  Let us construct the truth table. P F F T T

Q F T F T

I T F T T

II T F T T

III T F T T

IV T F T T

Hence, all the four expressions are equivalent. Ans. (d)

Chapter 7.indd 388

(d) ∀ x((G(x)∨ S(x)) → P (x)) (GATE 2009, 2 Marks) Solution:  We are given that “Gold and silver ornaments are precious” Hence, we use ∀ x((G(x)∨ S(x)) → P (x)) Ans. (d) 14. Suppose the predicate F(x, y, t) is used to represent the statement that person x can fool person y at time t. Which one of the statements below expresses best the meaning of the formula ∀x ∃y ∃t(¬F (x, y, t))? (a) Everyone can fool some person at some time (b) No one can fool everyone all the time (c) Everyone cannot fool some person all the time (d) No one can fool some person at some time (GATE 2010, 2 Marks) Solution:  We are given that F(x, y, t) is a predicate.

22-08-2017 06:30:33

SOLVED GATE PREVIOUS YEARS’ QUESTIONS     389

∀ x ∃ y ∃ t(¬F (x, y, t)) ⇒ ∀ x¬(∀ y∀ tF (x, y, t))

(c) ∃x (real(x) ∧ rational(x)) (d) ∃x (rational(x) → real(x))

Hence, we can conclude that “No one can fool everyone all the time.”

(GATE 2012, 1 Mark)

Solution:  The translation of each option is as follows: Option (a): There exists an x which is either real or 15. Which one of the following options is CORRECT rational and can be both. given three positive integers x, y and z and a predicate Option (b): All real numbers are rational. P (x) = ¬(x = 1) ∧∀ y(∃ z(x = y ∗ z) ⇒ (y = x)∨ (y = 1) Option (c): There exists a real number which is rational. Option (d): There exists some number which is not P (x) = ¬(x = 1) ∧∀ y(∃ z(x = y ∗ z) ⇒ (y = x)∨ (y = 1) rational or which is real. (a) P(x) being true means that x is a prime number Ans. (c) (b) P(x) being true means that x is a number other 17. Consider the following logical inferences: than 1 Ans. (d)

(c) P(x) is always true irrespective of the value of x (d) P(x) being true means that x has exactly two factors other than 1 and x

I1: If it rains, then the cricket match will not be played. The cricket match was played.

(GATE 2011, 2 Marks) Solution:  Consider the following table of precedence of logical operators:

Inference: There was no rain. I2: If it rains, then the cricket match will not be played. It did not rain.

Operator

Precedence

¬ ∧ ∨ → ↔

1 2 3 4 5

Inference: The cricket match was played. Which of the following is TRUE? (a) Both I1 and I2 are correct inferences. (b) I1 is correct but I2 is not a correct inference. (c) I1 is not correct but I2 is a correct inference. (d) Both I1 and I2 are not correct inferences. (GATE 2012, 1 Mark)

Solution:  For I1, since we know that the cricket match was played, we can infer that there was no P (x) = ¬(x = 1) ∧∀ y (∃ z(x = y ∗ z) ⇒ (y = x)∨ (y = 1)) rain. Hence, inference 1 is correct. For I2, since we know that it did not rain. (x) = ¬(x = 1) ∧∀ y (∃ z(x = y ∗ z) ⇒ (y = x)∨ (y = 1)) However, this only confirms that the match can be played and tells nothing about whether the match P(x) being true means x ≠ 1 and for all y if there was played or not. Hence, inference 2 is incorrect. exists a z such that x = y ∗ z then y must be x (i.e. Therefore, I1 is correct and I2 is incorrect inference. z = 1) or y must be 1 (i.e. z = x). The predicate is evaluated as

Ans. (b)

Hence, only x has two factors, 1 and x itself. Thus, this predicate defines the prime number.

18. Which one of the following is NOT logically equivalent to ¬ ∃ x(∀ y(a ) ∧∀ z(b ))?

Ans. (d)

(a) ∀ x (∃ x (¬b ) → ∀ y(a ))

16. What is the correct translation of the following statement into mathematical logic?

(b) ∀ x (∀ z (b ) → ∃ y(¬a ))

“Some real numbers are rational”

(c) ∀ x (∀ y (a ) → ∃ z(¬b ))

(a) ∃x (real(x) ∨ rational(x))

(d) ∀ x (∃ y (¬a ) → ∃ z(¬b ))

(b) ∀x (real(x) → rational(x))

Chapter 7.indd 389

(GATE 2013, 2 Marks)

22-08-2017 06:30:36

390     Mathematical Logic 

Solution:  We have ¬∃x ( ∀y (a ) ∧ ∀z (b )) ,

(c) ∃d (∼Rainy(d) → Cold(d)) (d) ∃d (Rainy(d) ∧ ∼ Cold(d))

Using the identity ¬ ( p ∧ q ) ≡ p ⇒ ¬q, we get ¬∃x ( ∀y (a ) ∧ ∀z (b )) ≡ ∀ ×  ∀y (a ) → ∃z (¬b )

(GATE 2014, 2 Marks)

Using the identity p ⇒ q ≡ ¬q ⇒ ¬p, we get ¬∃x ( ∀y (a ) ∧ ∀z (b )) ≡ ∀ ×  ∀z (b ) → ∃y (¬a )



Hence, the correct options are (a) and (d).



Ans. (a) and (d) 19. Consider the statement: “Not all that glitters is gold”. Predicate glitters(x) is true if x glitters and predicate gold(x) is true if x is gold. Which one of the following logical formulae represents the above statement?

Ans. (d)

22. Which one of the following is NOT equivalent to p ↔ q? (a) ( p ∨ q) ∧ ( p ∨q) (b) ( p ∨ q) ∧ (q → p) (d) ( p ∧q) ∨ ( p ∧ q) (GATE 2015, 1 Mark)



Solution:  `Not all that glitters is gold’ means there exist some glitters that are not gold. So ∃x is used along with AND operator. ∃x; glitters(x)∧¬ gold(x)



Ans. (d)

Solution: p ↔ q ≡ ( p → q ) ∧ (q → p ) ≡ ( p ∨ q ) ∧ ( p ∨ q ) (∵ p → q ≡ p ∨ q ) ≡ ( p ∧q ) ∨ (q ∧q ) ∨ ( p ∧ p) ∨ (q ∧ p )

(using distributive laws ) ≡ ( p ∧q ) ∨ (q ∧ p )

20. Consider the following statements:

(using complement and commutative laws )

P: Good mobile phones are not cheap Q: Cheap mobile phones are not good L: P implies Q M: Q implies P N: P is equivalent to Q

Thus, p ↔ q is not equivalent to ( p ∧ q) ∨ ( p ∧q). Ans. (c)

Which one of the following about L, M, and N is CORRECT? (a) Only L is TRUE. (b) Only M is TRUE. (c) Only N is TRUE. (d) L, M and N are TRUE. (GATE 2014, 1 Mark) Solution:  P: Good mobile phones are not cheap Q: Cheap mobile phones are not good. P → Q and Q → P Hence, P « Q. So, L, M and N are true. Ans. (d) 21. The CORRECT formula for the sentence, `not all rainy days are cold’ is (a) ∀d (Rainy(d) ∧ ∼ Cold(d)) (b) ∀d (∼Rainy(d) → Cold(d))

~ ∀d (rainy (d ) → cold (d )) → ∃d (rainy (d ) ∧ ∼ cold (d ))

(c) ( p ∧ q) ∨ ( p ∧q)

(a) ∀x; glitters(x) ⇒ ¬ gold(x) (b) ∀x; gold(x) ⇒ glitters(x) (c) ∃x; gold(x) ∧ ¬ glitters(x) (d) ∃x; glitters(x) ∧¬ gold(x) (GATE 2014, 1 Mark)

Chapter 7.indd 390

Solution:  `Not all rainy days are cold’

23. Consider the following two statements: (i) If a candidate is known to be corrupt, then he will not be elected. (ii) If a candidate is kind, he will be elected. Which one of the following statements follows from (i) and (ii) as per sound inference rules of logic? (a) If a person is known to be corrupt, he is kind. (b) If a person is not known to be corrupt, he is not kind. (c) If a person is kind, he is not known to be corrupt. (d) If a person is not kind, he is not known to be corrupt. (GATE 2015, 1 Mark) Solution:  Let p: candidate known to be corrupt q: candidate will be elected r: candidate is kind

22-08-2017 06:30:39

SOLVED GATE PREVIOUS YEARS’ QUESTIONS     391

Then,

26. Let p, q, r, s represent the following propositions. s1 = p →∼ q

p: x ∈ {8, 9, 10, 11, 12}

= q →∼ p (contrapositive rule)

q: x is a composite number r: x is a perfect square

And s2 : r → q ⇒ r →∼ p (transitive rule)

s: x is a prime number

That is if a person is kind, he is not known to be corrupt.

The integer x ≥ 2 which satisfies ¬ ((P ⇒ q) ∧ (¬  rV ¬ s)) is __________.

Therefore, option (c) is correct. (GATE 2016, 1 Mark)

Ans. (c) 24. Which one of the following well-formed formulae is a tautology? (a) ∀x ∃y R(x, y) ↔ ∀xR(x, y) (b) ∀x [∃y R(x, y) → S(x, y)] → ∀x∃yS(x, y) (c) [ ∀x ∃y {P (x, y) → R(x, y)}] ↔ [ ∀x∃y {¬P (x, y) ∨ R(x, y)}] (d) ∀x∀yP (x, y) → ∀x∀yP (x, y) (GATE 2015, 1 Mark) Solution:  Since P → R = ¬P ∨ R [ ∀x∃y{P (x, y) → R(x, y)}] ↔ [ ∀x∃y{¬P (x, y) ∨ R(x, y)}] Ans. (c) 25. Consider the following grammar G

Solution:  We have ¬((P ⇒ q) ∧ (¬rV¬s)) = ¬(¬pvq)) ∨ (¬(¬rV¬S ) = ( p ∧ ¬q) ∨ (r ∧ s) which can be read as follows: (i) x ∈ {8, 9, 10, 11, 12} AND (x is not a composite number) OR (x is a perfect square and x is a prime number) Now, x is a perfect square and x is a prime number can never be true as every square has at least 3 factors 1, x and x2. Thus, the second condition due to AND can never be true. This means the first condition has to be true. Now, x ∈ {8, 9, 10, 11, 12} AND (x is not a composite number). However, here only 11 is not composite so the correct answer is x = 11.

S→F|H F→p|c

Ans. 11

H→d|c

27. Consider the following expressions: (i) False

where S, F and H are non-terminal symbols, and p, d and c are terminal symbols. Which of the following statement(s) is/are correct? S1: LL(1) can parse all strings that are generated using grammar G S2: LR(1) can parse all strings that are generated using grammar G (a) only S1 (b) only S2 (c) Both S1 and S2 (d) Neither S1 nor S2

(iii) True (iv) P ∨ Q (v) ¬ Q ∨ P The number of expression given above that are logically implied by P ∧ (P ⇒ Q) __________. (GATE 2016, 1 Mark) Solution: 

(GATE 2015, 2 Marks) Solution:  Neither S1 nor S2 is correct. Ans. (d)

Chapter 7.indd 391

(ii) Q

We know that P → Q. Therefore, P Q

(modus ponens)

Q ∨ P (by addition)

22-08-2017 06:30:41

392     Mathematical Logic 

P ∧ (P → Q) (by applying simplification

can be simplified using De Morgan’s theorem as F = (a + b + c + d ) ⋅ (b + c )

and then addition) Therefore, Q, P ∨ Q, ¬ Q ∨ P can be logically implied. Ans. 3 28. Which one of the following well-formed formulae in predicate calculus is NOT valid? (a) (∀xp(x) ⇒ ∀xq(x)) ⇒ (∃x¬p(x) ∨ ∀xq(x)) (b) (∃xp(x) ∨ ∃xq(x)) ⇒ ∃x( p(x) ∨ q(x))



= (a ⋅ b ⋅ c ⋅ d) ⋅ (bc) =0





Ans. (d)

30. The statement (¬p ) ⇒ (¬q ) is logically equivalent to which of the below statements? I. p ⇒ q II. q ⇒ p

(c) ∃x( p(x) ∧ q(x)) ⇒ (∃xp(x) ∧ ∃xq(x))

III. (¬q) ∨ p IV. (¬p) ∨ q

(d) ∀x( p(x) ∨ q(x)) ⇒ (∀xp(x) ∨ ∀xq(x))

(a) I only (b) I and IV only

(GATE 2016, 2 Marks)

(c) II only (d) II and III only

Solution:

(GATE 2017, 1 Mark)

(a) Since L.H.S. ⇔ R.H.S., the well-formed formula is valid.

Solution:  From the given statement, we have ¬p ⇒ ¬q ≡ ¬(¬p) ∨ ¬q ≡ p ∨ ¬q

(b) Same as option (a). (c) i. ∃x{p(x) ∧ q(x)} Premise

Now, considering the statements given in the options, we have

ii. p(a) ∧ q(a)

(i) E.S.

iii. p(a)

(ii) Simplification

I. p ⇒ q ≡ ¬p ∨ q, False

iv. q(a)

(ii) Simplification

II. q ⇒ p ≡ ¬q ∨ p, True

v. ∃xp(x)

(iii) E.G.

III. (¬q) ∨ p ≡ ¬q ∨ p, True

vi. ∃xq(x)

(iv) E.G.

vii. ∃xp(x) ∧ ∃xq(x)

IV. (¬p) ∨ q ≡ ¬p ∨ q, False

(v), (vi)  Conjunction Therefore, only II and III are correct.

Hence, option (c) is proved to be valid. Ans. (d) (d) Let p(x): x be a flower and q(x): x be an animal. Now, let U = {Tiger, Lion, Rose} be the universe of discourse. It is obvious that the antecedent of the implication is true but the consequent is false. Hence, option (d) is proved to be not valid. Ans. (d)

29. The Boolean expression (a + b + c + d ) + (b + c ) simplifies to

(a) 1

(b) a, b



(c) a, b

(d) 0 (GATE 2016, 2 Marks)

Solution:  The given function

Chapter 7.indd 392

F = (a + b + c + d ) + (b + c )

31. Consider the first-order logic sentence F : ∀x(∃yR(x, y)). Assuming non-empty logical domains, which of the below sentences are implied by F  ? I. ∃y(∃xR(x, y)) II. ∃y(∀xR(x, y)) III. ∀y(∃xR(x, y)) IV. ¬∃x(∀y¬R(x, y)) (a) IV only 

(b) I and IV only

(c) II only

(d) II and III only (GATE 2017, 1 Mark)

Solution:  Given F : ∀x(∃yR(x, y)) Now, considering the sentences given in the options, we have

22-08-2017 06:30:45

SOLVED GATE PREVIOUS YEARS’ QUESTIONS     393

I. ∃y(∃xR(x, y)) ∀x(∃yR(x, y)) → ∃y(∃xR(x, y)) is true, since ∃y(∃xR(x, y)) ≡ ∃x(∃yR(x, y)).

I t is not raining and it is pleasant, and it is not pleasant only if it is raining and it is cold = (¬ p ^ r ) ^ (¬ r → (p ^ q)). Ans. (a)

II. ∃y(∀xR(x, y)) ∀x(∃yR(x, y)) → ∃y(∀xR(x, y)) is false, since ∃y when it is outside is stronger when it is inside.

33. Let p, q and r be propositions and the expression (p    → q) → r be a contradiction. Then, the expression (r → p) → q is

III. ∀y(∃xR(x, y)) ∀x(∃yR(x, y)) → ∀y(∃xR(x, y)) is false, since R(x, y) may not be symmetric in x and y.

(a) a tautology

IV. ¬∃x(∀y¬R(x, y))

(c) always TRUE when p is FALSE



∀x(∃yR(x, y)) → ¬∃x(∀y¬R(x, y)) is true, since Ø($x"yØR(x, y)) º "x$yR(x, y),

(b) a contradiction

(d) always TRUE when q is TRUE (GATE 2017, 2 Marks)

So, IV will reduce to ∀x(∃yR(x, y)) → ∀x(∃yR(x, y)) which is trivially true.

Solution:  Given that (p → q) → r = False    ⇒ ~ (~ p ∨ q) ∨ r = False

Therefore, only I and IV are correct.

   ⇒ ( p ∧ ~ q) ∨ (r) = False

Ans. (b) 32. Let p, q and r denote the statements “It is raining,” “It is cold,” and “It is pleasant,” respectively. Then the statement “It is not raining and it is pleasant, and it is not pleasant only if it is raining and it is cold  ” is represented by (a) (¬p ∧ r) ∧ (¬r → ( p ∧ q)) (b) (¬p ∧ r) ∧ (( p ∧ q) → ¬r) (c) (¬p ∧ r) ∨ (( p ∧ q) → ¬r) (d) (¬p ∧ r) ∨ (r → ( p ∧ q))

This is possible when p ∧ ~ q = False and r = False We have to check (r → p) → q ⇒ ~ (~ r ∨ p) ∨ q ⇒ (r ∧ ~ p) ∨ q Given r is false

(GATE 2017, 1 Mark)

⇒ r ∧ ~ p → False

⇒ (r → p) → q ≡ q Solution:  It is not raining and it is pleasant: ¬ p ^ r. It is not pleasant only if it is raining and it is cold: ¬ r → (p ^ q).

Chapter 7.indd 393

So, (r → p) → q is true only if q is true. Ans. (d)

22-08-2017 06:30:51

Chapter 7.indd 394

22-08-2017 06:30:51

CHAPTER 8

SET THEORY AND ALGEBRA

INTRODUCTION TO SET THEORY

For example: If A = {1, 2} and B = {0, 1, 2, 3}, then A⊆B

A set is a collection of distinct objects, considered as a single entity of size equal to the number of distinct objects. For example: A is a set of sports played by a student. A = {Cricket, Football, Tennis, Basketball, Squash}. A set can be represented in two ways: 1. Set-builders form: In this representation, we first write a variable (say x or y) and then state the properties of that variable. For example: A = {x : x is a prime number and 1 < x < 100}. 2. Tabular or roaster form: In this representation, all the elements of the set are listed between the braces { } separated by commas. For example: A = {1, 2, 3, 5, 10, 20}

Subsets and Supersets If we have two sets A and B, then A is said to be a subset of B if every element of A is an element of B. It is denoted by A ⊆ B.

Chapter 8.indd 395

If every element of A is an element of B and B has at least one element which is not an element of A, then A is said to be a proper subset of B. It is denoted by A ⊂ B. For example: If A = {1, 2, 3} and B = {1, 3, 2, 4}, then A ⊂ B. If B contains all the elements of A, then B is said to be the superset of A. It is denoted by B ⊇ A.

Equal Sets Two sets A and B are said to be equal, if and only if every element of A is a member of B and every element of B is a member of A. For example: If A = {0, 1, 2} and B = {2, 0, 1}, then A = B.

Comparable Sets Two sets A and B are comparable if any one of the ­following conditions is met:

22-08-2017 06:33:19

396     Set Theory and Algebra 

1. A ⊂ B 2. B ⊂ A 3. A = B

OPERATIONS ON SETS

Similarly, if neither of the conditions mentioned above is met, then A and B are said to be incomparable.

Universal Set

1. Union of sets: The union of two sets, A and B, is the set of all elements which are present either in A or B. It is denoted by A ∪ B. For example: If A = {0, 1} and B = {1, 2, 3} A ∪ B = {0, 1, 2, 3}

Any set that contains all the sets under consideration is called a universal set. It is denoted by S or U. For example: If A = {0, 1, 2}, B = {1, 2, 3, 4} and C = {5, 6, 9}, then S = {0, 1, 2, 3, 4, 5, 6, 9}.

2. Intersection of sets: The intersection of two sets, A and B, is the set of all elements which are present in A and B. It is denoted by A ∩ B. For example: If A = {0, 1, 2} and B = {1, 2, 5} A ∩ B = {1, 2}

Power Set

3. Difference of sets: The difference of two sets, A and B, is the set of all elements of A which are not in B.

The collection or family of all subsets of A is called the power set of A and is denoted by P(A). If A has n elements, then P(A) has 2n elements. Mathematically, P(A) = {x : x ⊆ A}

A − B = {x : x ∈ A and x ∉ B} Similarly, B − A = {x : x ∈ B and x ∉ A}. For example: If A= {1, 2, 5, 6, 7, 9} and B = {2, 3, 4, 5}

Hence, x ∈ P(A)

A − B = {1, 6, 7, 9} ⇒x⊆A

For example: If A = {0, 1}, then P (A) = {f, {0}, {1}, {0, 1}}

Types of Sets The different types of sets are described as follows: 1. Empty set (or null set): A set having zero elements is called an empty or null set. It is denoted by f. For example: A = {x : x is an odd number divisible by 2} 2. Singleton set: A set having a single element is called a singleton set. For example: A = {2} 3. Pair set: A set having two elements is called a pair set. Example: A = {0, 1} 4. Finite set: A set having finite number of elements is called a finite set. For example: A = {0, 1, 2, 3, 5, 7} 5. Infinite set: A set having infinite number of elements is called an infinite set. For example: A = {x : x is a set of all the even numbers}

Chapter 8.indd 396

B − A = {3, 4} 4. Disjoint sets: Two sets A and B which have no common element are called disjoint sets. For disjoint sets, A ∩ B = f. For example: A = {0, 1, 2} and B = {5, 6} are disjoint sets. 5. Symmetric difference of sets: It is defined as a set of elements that are either a member of A or B but not both. It is denoted by A ⊕ B. Mathematically, A ⊕ B = (A ∪ B) − (A ∩ B) = (A − B) ∪ (B − A) 6. Complement of set: A set having all the elements of the universal sets which are not elements of A is called complement of A. It is denoted by Ac or A−1 or A°. Symbolically, A−1 = U − A = {x : x ∈ U and x ∈ A}

IMPORTANT LAWS AND THEOREMS 1. Idempotent laws A ∪ A = A, A ∩ A = A 2. Identity laws A ∪ f = A, A ∩ f = f

22-08-2017 06:33:19

APPLICATION OF SETS     397

3. Difference of sets: A − B and B − A

3. Commutative laws A ∪ B = B ∪ A,  A ∩ B = B ∩ A 4. Associative laws

A−B

B−A B

A



B

A



(A ∪ B) ∪ C =  A ∪ (B ∪ C ), A ∩ (B ∩ C ) = (A ∩ B) ∩ C 5. Distribution laws A ∪ (B ∩ C ) =  (A ∪ B) ∩ (A ∪ C ), A ∩ (B ∪ C ) = (A ∩ B) ∪ (A ∩ C ) 4. Complement of a set 6. De Morgan’s laws



(A ∪ B)′ = A′ ∩ B ′,  (A ∩ B)′ = A′ ∪ B ′

A

VENN DIAGRAMS The diagrams drawn to represent sets are called Venn— Euler diagrams or simply Venn diagrams. In Venn diagrams, the universal set U is represented by points within a rectangle and its subsets are represented by points in closed curves (usually circles) within the rectangle. If a set A is a subset of B, then the circle representing A is drawn inside the circle representing B. If A and B are not equal but they have some common elements, then to represent A and B we draw two intersection circles. Two disjoints sets are represented by two non-intersecting circles.

5. Subset: A ⊆ B ∪ B A

6. Disjoint sets

Operations on Sets Using Venn Diagrams

A

B



This section discusses the most commonly used operations performed on sets using Venn diagrams. 1. Union of sets: A ∪ B A

B



APPLICATION OF SETS Let A, B and C be finite sets and U be a finite universal set.

2. Intersection of sets: A ∩ B A

Chapter 8.indd 397

B



1. n(A ∪ B) = n(A) + n(B) − n(A ∩ B) Note: If A and B are disjoint sets, then n(A ∩ B) = 0. 2. n(A − B) = n(A) − n(A ∩ B) 3. Number of elements which belong to exactly one of A or B = n(A) + n(B) − 2n(A ∩ B) 4. n(A ∪ B ∪ C) = n(A) + n(B) + n(C ) − n(A ∩ B) − n(B ∩ C ) − n(A ∩ C ) + n(A ∩ B ∩ C ) 5. Number of elements in exactly two of the sets A, B, C=n  (A ∩ B) + n(B ∩ C ) + n(C ∩ A) − 3n(A ∩ B ∩ C )

22-08-2017 06:33:21

398     Set Theory and Algebra 

6. Number of elements in exactly one of the sets A, B and C =n  (A) + n(B) + n(C) − 2n(A ∩ B) − 2n(B ∩ C) − 2n(A ∩ C) + 3n(A ∩ B ∩ C) 7. n(A′ ∪ B′) = n((A ∩ B)′) = n(U) − n(A ∩ B) 8. n(A′ ∩ B′) = n((A ∪ B)′) = n(U) − n(A ∪ B)

CARTESIAN PRODUCT OF SETS Ordered pair is a pair of elements x and y of some sets, which is ordered in the sense that (x, y) ≠ (y, x) for x ≠ y. It is denoted by (x, y). Here, x is known as first coordinate and y is known as second coordinate of the ordered pair (x, y). Let us say we have two sets A and B, then Cartesian product of these two sets is the set of all those pairs whose first coordinate is an element of A and second coordinate is an element of B. The set is denoted by A × B. Symbolically, A × B = {(x, y) : x ∈ A ∧ y ∈ B}

RELATIONS If we have two sets A and B, then a relation R between two sets is a subset of A × B. Thus, R ⊆ A × B. If R is a relation from a non-empty set A to another non-empty set B and if (a, b) ∈ R, then a R b is read as `a is related to b by the relation R’. Similarly, if (a, b) ∉ R, then we write a R b, which is read as `a is not related to b by the relation R’.

It is defined by R−1 = {(x, y) : (x, y) ∈ R} It is to be noted that if R is a relation from A to B, then domain of R is the range of R−1 and range of R is the domain of R−1. For example: Let A = {1, 2}, B = {0, 3, 4} and R = {(1, 0), (1, 3), (1, 4), (2, 0)} then R−1 is given by R−1 = {(0, 1), (3, 1), (4, 1), (0, 2)} 2. Identity relation: A relation R in a set A is an identity relation if IA = {(x, x) : x ∈ A} For example: Let A = {0, 1, 2}, then IA = {(0, 0), (1, 1), (2, 2)} 3. Universal relation: If R is a relation in a set A, then R = A × A is known as a universal relation. For example: If A = {0, 1, 2}, then universal relation is given by R = { (0, 0), (0, 1), (0, 2), (1, 0), (1, 1), (1, 2), (2, 0), (2, 1), (2, 2)} 4. Void relation: If R is a relation in a set A, then R is a void relation if R = f. For example: Let A = {1, 3, 5} and R be defined as x R y if x divides y, then R = f ⊂ A × A is a void relation.

Properties of Relation The properties of relation are as follows:

If A and B are two non-empty finite sets consisting of x and y elements, respectively, then A × B consists of xy ordered pairs. Hence, the total number of subsets of A × B is 2xy. Also, since each subset defines a relation between A and B, the total number of relations is 2xy.

1. Reflexive and anti-reflexive relation: Let R be a relation in a set A, then R is a reflexive relation if ∀x ∈ A, (x, x) ∈ R. Thus, R is reflexive if x R x exists for all x ∈ A. R is an anti-reflexive relation if ∀x ∈ A, (x, x) ∉ R.

Now, consider R as a relation from set A to set B. The set of all first coordinates of the ordered pairs belonging to R is called the domain of R.

R1 = {(0, 0), (0, 1), (1, 1), (2, 2)} is reflexive.

The set of all second coordinates of the ordered pairs in R is called the range of R. If A is a non-empty set, then relation from A to itself (i.e. a subset of A × A) is called a relation on set A.

Types of Relations The different types of relations are as follows: 1. Inverse relation: If R is a relation from a set A to a set B, then inverse of R is a relation from B to A. It is denoted by R−1.

Chapter 8.indd 398

For example: Let A = {0, 1, 2}, then the relation R2 = { (0, 0), (0, 1), (2, 2)} is not reflexive since (1, 1) ∉ R2. R3 = {(0, 1), (0, 2), (1, 2), (2, 0)} is anti-reflexive. R4 = { (0, 1), (1, 1), (1, 2)} is not anti-reflexive since (1, 1) ∈ R4. 2. Symmetric and anti-symmetric relation: Let R be a relation in a set A, then R is said to be symmetric relation if ∀x ∀y ∈ A, (x, y) ∈ R → (y, x) ∈ R. R is anti-symmetric relation if ∀x ∀y ∈ A, (x, y) ∈ R ∧ (y, x) ∈ R → x = y.

22-08-2017 06:33:22

FUNCTIONS     399

For example: Let A = {0, 1, 2}, then the relation A

B

R1 = {(1, 2), (2, 1)} is symmetric. R2 = { (1, 2), (2, 1), (0, 1)} is not symmetric since R2 does not contain (1, 0).

f f(x) =y

x

R3 = {(1, 1), (2, 2)} is anti-symmetric. f −1

R4 = { (1, 2), (2, 1), (1, 0)} is not anti-symmetric since R4 contains (1, 2) and (2, 1) but 1 ≠ 2. 3. Transitive relations: Let R be a relation in set A, then R is a transitive relation if ∀x ∀y ∀z ∈ A, (x, y) ∈ R ∧ (y, z) ∈ R → (x, z) ∈ R A relation R in a set A is not transitive if ∀x ∀y ∀z ∈ A, (x, y) ∈ R ∧ (y, z) ∈ R → (x, z) ∉ R For example: Let A = {0, 1, 2}, then

Let A and B be two non-empty sets. Then a function f from set A to set B is a correspondence which associates elements of set A to elements of set B such that 1. All elements of set A are associated to elements in set B. 2. An element of set A is associated to a unique ­element in set B.

R1 = {(0, 1), (1, 2), (0, 2)} is transitive. R2 = { (0, 1), (1, 2)} is not transitive since (0, 2) ∉ R2. 4. Partial order: Let R be a relation in set A, then R is partial order if (a) R is reflexive. (b) R is anti-symmetric. (c) R is transitive. For example: Let A = {0, 1, 2}, then R = {(0, 0), (0, 1), (0, 2), (1, 1), (1, 2), (2, 2)} is partial order relation. 5. Equivalence relation: Let R be a relation in set A, then R is an equivalence relation if

A x y z

f

B a b c d

Let f  : A → B be a function from set A to set B, then set A is known as the domain of f and set B is known as the co-domain of f. The set of all the F-images of elements of A is known as the range of f and is denoted by f(A). Thus, f(A) = {f(x) : x ∈ A} = Range of f Two functions f and g are said to be equal if

(a) R is reflexive. (b) R is symmetric. (c) R is transitive. For example: Let A = {0, 1, 2}, then R = {(0, 0), (1, 1), (2, 2)} is an equivalence relation.

FUNCTIONS

1. domain of f = domain of g, 2. co-domain of f = co-domain of g, and 3. f(x) = g(x), for every x belonging to their common domain. A function f  : A → B is called a real-valued function if B is a subset of R, i.e. the set of all real numbers.

Types of Functions Let A and B be two non-empty sets. A relation f from A to B is called a function, if 1. For each x ∈ A, there exists y ∈ B such that (x, y) ∈ f. 2. (x, y) ∈ f and (x, z) ∈ f ⇒ y = c. Thus, a non-empty subset of f from A × B is a function from A to B if each element of A appears in some ordered pair in f and no two ordered pairs in f have the same first element. If (x, y) ∈ f, then y is called the image of x under f and x is called a pre-image of f.

Chapter 8.indd 399

The different types of functions are as follows: 1. Into function: If f(A) is a proper subset of B, then f  : A → B is called an into function. For example: Let f  : z → z (set of integers) be defined by f(x) = 4x, x ∈ z is an into function. 2. Onto (surjective) function: If f(A) = B, then f  : A → B is called an onto or surjective function. For example: Let f  : z → z be defined by f(x) = x + 2, x ∈ z. Then every element y in the co-domain set z has pre-image y − 2 in the domain set z. Therefore, f(z) = z and f is an onto function.

22-08-2017 06:33:23

400     Set Theory and Algebra 

3. Constant function: If k is a fixed real number, then f(x) given by f(x) = k, x ∈ R is called a constant function. For example: Let f  : R → R be defined by f(x) = 10. Hence, f is a constant function. 4. Identity function: The function f that associates each real number to itself is called the identity function. It is denoted by I. Hence, identity function for I: R → R can be defined as I(x) = x for all x ∈ R 5. Injective function: If a function f  : A → B has a distinct value for each district argument, then it is called an injective function. It is also called one-toone function. Thus, f : A → B is injective if x1 ≠ x2 in A implies f(x1) ≠ f(x2) in B. For example: Let f  : R → R be defined by f(x) = 3x + 1, x ∈ R, then for x1, x2 ∈ R and x1 ≠ x2, we have f(x1) ≠ f(x2).

Properties of Composites of Mappings 1. If f : A → B is a bijective function, then fof  −1 = IB and f  −1of = IA where IA and IB are identity mapping of sets A and B, respectively. 2. If f : A → B, g : B → C and h: C → D, then ho(goh) = (hog)of 3. If f : A → B and g : B → C are injective functions, then (fog): A → C is injective. 4. If f : A → B and g : B → C are two functions such that fog: A → C is injective. Then f is injective. 5. If f : A → B and g : B → C are surjective functions, then (fog): A → C is surjective. 6. If f : A → B and g : B → C are two functions such that fog: A → C is surjective, then f is surjective. 7. If f : A → B and g : B → C are bijective functions, then fog: A → C is bijective.

6. Bijective function: If f is both injective and surjective, then f : A → B is called a bijective function. For example: Let f  : z → z be defined by f(x) = x + 2, x ∈ 1 ∈ z. 7. Many-to-one function: If two or more distinct elements in A have the same image, i.e. if f(x) = f(y) implies x ≠ y, then f : A → A is called many-to-one function. 8. Inverse function: Let f : A → B be an onto function. Then f  −1: B → A, which associates to each element y ∈ B the element x ∈ A, such that f(x) = y is called an inverse function of f : A → B. For example: Let f : A → B is defined by f (x) =

INTRODUCTION TO ALGEBRA Sets with an algebraic operation are called groups. A binary operation on a non-empty set is a calculation that combines two elements of the set to produce another element of the set. If * be a binary operation on a non-empty set A and B ⊆ A, then B is said to be closed with respect to the binary operation. Table 1 represents the common notations for standard sets.

x−2 x−3

Table 1 |   Some commonly used notations for standard sets

Then f  −1: B → A is given by y= or f −1(x) =

x−2 x−3



x=

2 − 3y 1− y

2 − 3x 1− x

Compositions of Functions Let f : A → B and g: B → C, then composite of functions f and g is a mapping of A → C given by (fog): A → C such that

N

Set of all natural numbers

Z

Set of all integers

Q

Set of all rational numbers

R

Set of all real numbers

C

Set of all complex numbers

A

*

Set of all non-zero numbers of A where A ⊆ R

A

+

Set of all positive number of A where A ⊆ R



Set of all negative number of A where A ⊆ R

A

(gof )(x) = g f(x) = g[f(x)] = ∀x ∈ A Evidently, domain of g is equal to the co-domain of f.

Chapter 8.indd 400

Table 2 represents the symbols used to depict the usual operations.

22-08-2017 06:33:23

SEMIGROUPS     401

Table 2 |   Symbols of usual operations

Addition

+

Subtraction Multiplication

− .

Division

÷

A semigroup with an identity element is called a monoid. For example, (z, +) is a monoid. Consider (A, *) to be a semigroup with identity z. If there exists an element y ∈ A such that y * x = z, then x ∈ A is called left invertible and y is called the left inverse of x.

If * is a binary operation on a non-empty finite set A, then the elements of A after operation can be represented by binary operation tables called Cayley binary operation tables. For example: Write down binary operation table for usual multiplication on the set A = {1, 2, 3}.

If there exists an element y ∈ A such that x * y = z, then x ∈ A is called right invertible and y is called the right inverse of x. If x is both left invertible and right invertible, then x ∈ A is called invertible. If x is invertible and x ∈ A, then the element which is both left and right inverse of x is called inverse of x.



1

2

3

For example: 1 is invertible in the semigroup (N, ⋅).

1

1

2

3

If x, y, z ∈ A, x * y = x * z

2

2

4

6

3

3

6

9

A binary operation * is associative of (x * y) * z = x * (y * z), ∀x, y, z ∈ A where A is a non-empty set.

⇒y=z Then (A, *) is called a left cancellative semigroup. If x, y, z ∈ A, y * x = z * z ⇒y=z

For example: + and ⋅ are associative binary operations on N, Z, R, Q and C.

Then (A, *) is called a right cancellative semigroup.

A binary operation * is commutative if (x * y) = (y * x), ∀x, y ∈ A where A is a non-empty set.

If a semigroup (A, *) is both left cancellative and right cancellative semigroups, then it is called cancellative semigroup.

For example: + and ⋅ are commutative binary operations on N, Z, R, Q and C. If A is a non-empty set and B is a set of binary operations on A, then algebraic system/algebraic structure is defined by (A, B).

For example: (z, *) and (Q, *) are cancellative semigroups.

Some Important Theorems

For example: (R, +) is an algebraic system.

SEMIGROUPS A semigroup is an algebraic structure consisting of a set together with an associative binary operation. If A is a non-empty set and * is a binary operation on A, then algebraic system (A, *) is called a semigroup if x * (y * z) = (x * y) *z, ∀x, y, z ∈ A For example: (z, +) is a semigroup. Consider (A, *) to be a semigroup, then an element x ∈ A is called a left identity of A if x * y = y, ∀y ∈ A. Consider (A, *) to be a semigroup, then an element x ∈ A is called a right identity of A if y * x = y, ∀y ∈ A. Consider (A, *) to be a semigroup, then an element x ∈ A is called a two-sided identity of A if y * x = x * y, ∀y ∈ A.

Chapter 8.indd 401

1. Consider (A, *) to be a semigroup. If a and b are left and right identities, respectively, then a = b. 2. If (A, *) is a semigroup with identity, then identity in A is unique. 3. Consider (A, *) to be a semigroup with identity a and x ∈ A. Now, if y is a left inverse and z is a right inverse of x, then y = z. 4. Consider (A, *) to be a semigroup with identity a. If x is an invertible element, then inverse of x is unique. 5. Consider (A, *) to be a semigroup with identity a. If x is an invertible element, then x−1 is also invertible. (x−1)−1 = x 6. Consider (A, *) to be a semigroup with identity a. If x and y are two invertible elements and x, y ∈ A, then x * y is also invertible. (x * y)−1 = y−1 * x−1

22-08-2017 06:33:24

402     Set Theory and Algebra 

GROUP A semigroup satisfying associativity, identity and invertibility is called a group. Say A be a non-empty set and * be a binary operation, then (A, *) is called a group if 1. x * (y * z) = (x *  y) * z, ∀x, y, z ∈ A 2. ∃e ∈ A ⇒ x * e = e * x, ∀x ∈ A where e is identity element of the group. 3. x ∈ A ⇒ ∃y ∈ A ⇒ x *  y = y * x = e where y is the inverse of x in the group. A group (A, *) is called an abelian or commutative group if x * y = y * x, ∀x, y ∈ A For example: (R, +) and (C, +) are abelian groups. Consider (A, ⋅) to be a group. An element x ∈ A is idempotent if a2 = a. If A is a finite set, then a group (A, ⋅) is called a finite group. The number of different elements in A is called the order of the finite group, O(A). If A is an infinite set, then a group (A, ⋅) is called an infinite group. The order of an infinite group is clearly ¥. If A is a finite set containing n elements, then group of all bijections on A is called a permutation or symmetric group. It is denoted by Pn or Sn.

Some Important Theorems

1. Identity element in a group is unique. 2. Inverse of every element in a group is unique. 3. If (A, ⋅) is a group and x ∈ A, then (x−1)−1 = x. 4. If (A, ⋅) is a group and x, y ∈ A, then (x, y) −1 = y−1x−1. 5. If (A, ⋅) is a group, then (a) ∀x, y, z ∈ A,   xy = xz ⇒x=z (b) ∀x, y, z ∈ A,   yx = zx ⇒x=z

6. If (A, ⋅) is a group, x ∈ A and m, n ∈ z, then (a) xm ⋅ xn = xm+n = xn ⋅ xm (b) (xm)n = xmn

RESIDUE CLASSES Two integers are congruent `modulo n’ if they differ by an integral multiple of the integer n. Hence, if n is a positive integer and x, y ∈ z, then x is called congruent to y modulo n when n divides x − y. It is denoted by x ≡ y (mod n ) .

Chapter 8.indd 402

For example: 6 divides 40 − 4 ⇒ 40 ≡ 4 (mod 6). Also, x ≡ y (mod n) if x and y have the same remainder when divided by n. The congruence relation x ≡ y (mod n) on a set separates the set into n equivalence classes called residue classes modulo n. The residue class containing an integer x is denoted by [x] or x.

Residue Class Addition Residual class addition or addition modulo n is denoted as ⊕ and defined on Zn as x ⊕ y = x + y, ∀ x, y ∈ Zn. If x, y ∈ Zn, then x + y = r , where r is remainder of x + y when divided by n. (Zn, ⊕) is an abelian group.

Residual Class Multiplication Residual class multiplication or multiplication modulo n is defined on Zn as x, y = xy ∀ x, y ∈ Zn. It is also denoted by ⊗. If x, y ∈ Zn, then x ⊗ y = r where r is remainder of xy when divided by n. (Zn, ⊗) is a commutative semigroup with identity. (Zn*, ⊗) is an abelian group if n is prime.

PARTIAL ORDERING As already stated, a relation R is called partial order if it is 1. Reflexive, x R x, ∀x ∈ A 2. Anti-symmetric, (x R y) ∧ (y R x) ⇒ x = y 3. Transitive, (x R y) ∧ (y R z) ⇒ x R z. A partial order is denoted by ≤. A set with partial order ≤ is called a partially ordered set or poset. A poset (A, ≤) is called totally ordered set, if ∀x, y ∈ A, (x ≤ y) ∨ (y ≤ x). If x ∈ (A, ≤), then any element y ∈ (A, ≤) is called cover of x if and only if 1. x ≤ y. 2. There exists no element z ∈ A such that x ≤ z and z ≤ y.

Hasse Diagram A Hasse diagram is a type of mathematical diagram used to represent a finite poset. Consider (A, ≤) to be a partial ordered set. Then to draw the Hasse diagram, all the elements of A are

22-08-2017 06:33:27

LATTICE     403

represented as a vertex in the plane and a line segment is drawn that goes upward from x to y whenever y covers x.

If (A, ≤) is a lattice, then (A, ≥) is also a lattice, we define ≥ as:

y is the least element of A if y ≤ x for all x ∈ A.

(x ≤ y) ⇒ y ≥ x

y is the greatest element of A if x ≤ y for all x ∈ A. Consider x ⊆ A, then an element x ∈ A is called upper bound of A if y ≤ x for all y ∈ X.

Also, G.L.B. and L.U.B. are interchanged if we change ≤ with ≥.

Consider x ⊆ A, then an element x ∈ A is called lower bound of A if x ≤ y for all y ∈ X.

In other words, the operations of meet and join of (A, ≤) become the operations of join and meet of (A, ≥). (A, ≤) and (A, ≥) are called duals of each other. The dual of any poset can be obtained by turning the poset upside down. We denote x * y = meet of x and y = G.L.B. and x ⊕ y = join of x and y = L.U.B.

Least upper bound (L.U.B. or supremum) of A is the lowest upper bound belonging to A. Hence, if x and y are upper bounds of A, then x is the L.U.B. if x ≤ y. Greatest lower bounds (G.L.B. or infimum) of A is the highest lower bound belonging to A. Hence, if x and y are lower bounds of A, then y is the G.L.B. if x ≤ y.

Sublattice Consider we have a lattice (A, *, ⊕) and B ≤ A, then (B, *, ⊕) is a sublattice of (A, *, ⊕) if B is closed under operations * and ⊕.

LATTICE A lattice is a partially ordered set in which every two elements belonging to the poset have an L.U.B. (supremum) and a G.L.B. (infimum). The G.L.B. and L.U.B. are also called meet and join, respectively. Figures 1 and 2 show an example of a lattice and a non-lattice structure, respectively.

A lattice whose non-empty subsets have an L.U.B. and a G.L.B. is called a complete lattice. Every finite lattice is a complete lattice.

Bounds As already defined before, bounds are the extreme elements of lattices, i.e. the greatest and the least. They are denoted by 1 and 0, respectively.

4

Consider (L, *, ⊕, 0, 1) to be a bounded lattice, then x and y are complements of each other if x * y = 0 and x ⊕ y = 1. Consider Fig. 3, it has two complements, 2 and 3. 2

3 5

2 1 Figure 1 |   Lattice.

1 3

5

4

4

Figure 3 |   Example depicting bounds.

2

3 A complemented lattice is a bounded lattice in which every element x has at least one complement y such that x⊕y=1

1 Figure 2 |   Non-lattice.

Chapter 8.indd 403

and

   x * y = 0

22-08-2017 06:33:28

404     Set Theory and Algebra 

A relatively complemented lattice is characterized by the property that for every element x in an interval [w, z] there is an element y such that

     x * y = w

2

Such an element is called a complement of x relative to the interval. A distributive lattice is a lattice in which the operations of join and meet distribute over each other. A lattice (A, *, ⊕) is distributive if x * (y ⊕ z) = (x * y) ⊕ (x * z) x ⊕ (y * z) = (x ⊕ y) * (x ⊕ z)

2

4 3

4

0

0

Figure 5 |   Non-distributive lattices.

In a bounded distributive lattice, elements which have complements form a sub-lattice.

Figures 4 and 5 show the examples of distributive lattices and non-distributive lattices, respectively. 1

1

1

3

x⊕y=z and

1

A modular lattice is a lattice which fulfills the following condition: x ⊕ (y * z) = (x ⊕ y) * z, if x ≤ z It is to be noted that every distributive lattice is modular but the converse is not true.

2

2

3

BOOLEAN ALGEBRA OF LATTICES 0 0 Figure 4 |   Distributive lattices.

Consider we have a lattice (b, *, ⊕) with two binary operations * and ⊕. Let 0 and 1 be the least and greatest elements, respectively. Some common results for lattices are given in Table 3.

Table 3 |   Some common results for lattices

Condition

Property

Illustration

If x, y and z belong to set B.

Idempotent

x * x = x, x ⊕ x = x

Associativity

x * (y * z) = (x * y) * z, x ⊕ (y ⊕ z) = (x ⊕ y) ⊕ z

Commutativity

x * y = y * x, x ⊕ y = y ⊕ x

Absorption

x * (x ⊕ y) = x, x ⊕ (x * y) = x

Identity

x * 0 = 0, x * 1 = x x ⊕ 1 = 1, x ⊕ 0 = x

Distributivity

x ⊕ (y * z) = (x ⊕ y) * (x ⊕ z) x * (y ⊕ z) = (x * y) ⊕ (x ⊕ z)

If B is uniquely complemented.

Complements

x * x  ′ = 0, x ⊕ x  ′ = 1

De Morgan’s

(x * y)′ = x  ′ ⊕ y ′, (x ⊕ y)′ = x  ′ * y ′ x ≤ y ⇔ x * y = x, x ⊕ y = y x ≤ y ⇔ x * y ′ = 0 y ′ ≤ x  ′ ⇔ x  ′ ⊕ y = 1

Chapter 8.indd 404

22-08-2017 06:33:29

SOLVED EXAMPLES     405

SOLVED EXAMPLES 1. Let A = {−2, −1, 0, 1, 2}, B = {0, 1, 4} and f  : A → B is defined as f(x) = x2 is a function. Is the function one-to-one or bijection?

Solution:  We know a and b are natural numbers, then ab + 1 is also a natural number. Therefore, a * b is a binary operation on N. Now, a * b = ab + 1 = ba + 1 = b * a.

Solution:  We have f(x) = x2 Now, f (−2) = 4, f  (−1) = 1, f  (0) = 0, f  (1) = 1, f  (2) = 4. f : A → B is an onto function since f(A) = B, hence f : A → B is not one-to-one and therefore not a bijection. 2. If A = {a, b, c}, B = {2, 1, 0} and f : A→B, find whether the function is one-to-one, onto or bijection? Given that f = {(a, 1), (b, 0), (c, 2)}. Solution:  We have f = {(a, 1), (b, 0), (c, 2)} Different elements in A have different f-images in B and hence f is one-to-one. f(A) = {1, 0, 2} = B

Hence, * is commutative. (a * b) *c = (ab + 1) * c = (ab + 1)c + 1 = abc + c + 1 a * (b * c) = a * (bc + 1) = a(bc + 1) + 1 = abc + a + 1 Thus, (a * b) * c ≠ a * (b * c) Hence, * is not associative. 6. If A is a non-empty set and P(A) is a power set of A, then show that (P(A), ∪), (P(A), ∩) are monoids. Solution:  Let X, Y ∈ P(A). Now, X ⊆ A, Y ⊆ A and hence, X ∪ Y ⊆ A, X ∩ Y ⊆ A.

Hence, f : A → B is onto.

Hence, X ∪ Y ∈ P(A), X ∩ Y ∈ P(A).

By definition, f : A → B is a bijection, since it is one-to-one and onto.

Also, (X ∪ Y) ∪ Z = X ∩ (Y ∩ Z) (X ∩ Y) ∩ Z = X ∩ (Y ∩ Z) ∀X, Y, Z ∈ P(A)

3. Find the inverse function of f(x) = 4x + 7.

Again, X ∪ f = f ∪ X = X

Solution:  We have f(x) = 4x + 7 = y

Hence, (P(A), ∪) and (P(A), ∩) are monoids.

y −7 4 y −7 −1 ∴ f (y) = x x−7   f −1(x) = y ⇒ x=

and

and X ∩ A = A ∩ X = X. 7. Consider P = Q − {1}, and if * is defined on P as x * y = x + y − xy, prove that (P, *) is an abelian group. Solution:  We can see that P is non-empty. Now, let x, y ∈ P.

4. Form the binary operation table for a set A = {1, a, a2} under usual multiplication.

x ∈ P ⇒ x ∈ Q, x ≠ 1 Similarly, y ∈ P ⇒ y ∈ Q, y ≠ 1 Thus,

Solution:  The table can be formed as follows: ⋅ 1 a

1

a

a2

1 a

a a2

a2 a3

a2

a2

a3

a4

5. Verify if a * b = ab + 1 is a binary operation on N. Also verify whether they are associative and commutative.

Chapter 8.indd 405

x, y ∈ Q ⇒ x + y ∈ Q, xy ∈ Q and x + y − xy ∈ Q ⇒x*y∈Q Consider x * y = 1 ⇒ x + y − xy = 1 ⇒ x − 1 + y − xy = 0 ⇒ (x − 1) − y(x − 1) = 0 ⇒ (x − 1)(1 − y) = 0 ⇒ x = 1 or y = 1

22-08-2017 06:33:29

406     Set Theory and Algebra 

Now, consider x * y ≠ 1

The combination (a) is a lattice and is a chain. However, the combinations (b), (c) and (d) are not lattice because no two elements have G.L.B or L.U.B.

x*y∈Q⇒x*y∈P Therefore, * is a binary operation of P. Let x, y, z ∈ P.

9. Prove that x * (x ′ ⊕ y) = x * y.

x * (y * z) = x * (y + z − yz) = x * ((y + z) − yz) − x(y + z − yz)

Solution:  We know that x * (x ′ ⊕ y) = (x * x  ′) ⊕ (x * y)

= x + y + z − yz − xy − xz + xyz

= 0 ⊕ (x * y)

(x * y) * z = ( x + y − xy) * z = x + y − xy + z − (x + y − xy)z = x + y + z − xy − xz − yz + xyz

= x * y = R.H.S. Hence proved.

∴ x * (y * z) = (x * y) * z

10. Show that the relation R on the set A = {1, 2, 3} given by R = {(1, 2), (2, 1)} is symmetric but ­neither reflexive nor transitive.

Hence, * is associative. Let x ∈ P and y be x−1, then x*y=0

Solution:  As (1, 1), (2, 2) and (3, 3) do not belong to R, R is not reflexive.

⇒ x + y − xy = 0 x ∈P x −1 Now, say x, y ∈ P, then x * y = x + y − xy = y + x − yx = y * x ⇒ y(x − 1) = x ⇒ y =

Clearly, (1, 2) ∈ R and (2, 1) ∈ R. So, R is symmetric. As (1, 2) ∈ R and (2, 1) ∈ R but (1, 1) ∉ R, R is not transitive.

Thus, * is commutative. 11. Let a relation R on the set R of real number be defined as

Let e be identity element of P, then   x * e = x, ∀x ∈ P

(a, b) ∈ R1 ⇔ 1 + ab > 0 for all a, b ∈ R

⇒x*e=x

Show that R1 is reflexive and symmetric but not transitive.

⇒ x + e − xe = x ⇒ e(1 − x) = 0

Solution:  Let a be an arbitrary element of R. Then,

⇒ e = 0 (since x ≠ 1) Therefore, x = 0 ∈ P is the identity element of P. Hence, (P, *) is an abelian group. 8. Show that a lattice with less than four elements is always a chain. Solution:  If a lattice contains 1 or 2 elements, then it is a chain. Now, let us draw all the possible combinations with three elements. 3

(a)

3

(b) 1

a∈R ⇒ 1 + a ⋅ a = 1 + a > 0 [∵ a2 > 0 for all a ∈ R] 2

⇒ (a, a) ∈ R1 Thus, (a, a) ∈ R1 for all a ∈ R. Hence, R1 is reflexive on R. Now, let (a, b) ∈ R. Then, (a, b) ∈ R1 ⇒ 1 + ab > 0 and 1 + ba > 0 [∵ ab = ba ∀a, b ∈ R] ⇒ (b, a) ∈ R1 Thus, (a, b) ∈ R1 ⇒ (b, a) ∈ R1 for all a, b ∈ R. So, R1 is symmetric on R.

2

Now, we observe that (1, 1/2) ∈ R1 and (1/2, −1) ∈ R1 but (1, −1) ∉ R1 because 1 + 1 × (−1) = 0 > 0.

2 1

Thus, R1 is not transitive on R. (c)

3

1

1 2

Chapter 8.indd 406

3

(d)

2

12. Prove that the relation R on the set Z of all integers defined by (x, y) ∈ R ⇔ x − y is divisible by n is an equivalence relation on Z. Solution:  For any a ∈ N, we have a−a=0=0×n

22-08-2017 06:33:30

SOLVED EXAMPLES     407

a − a is divisible by n ⇒ (a, a) ∈ R Thus, (a, a) ∈ R for all a ∈ z. So, R is reflexive on Z. Let (a, b) ∈ R. Then, (a, b) ∈ R ⇒ (a − b) is divisible by n ⇒ (a − b) = np for some p ∈ Z b − a = n(−P) ⇒ b − a is divisible by n

 y − 5 1/3  3  y − 5 1/3      + 5  f (x) = f    = 3     3    3   = y −5+5 = y This shows that every element in the co-domain has its pre-image in the domain. So, f is a ­surjection. Hence, f is a bijection. 14. If the function f  : R → R be given by f(x) = x2 + 2 x and g : R → R be given by g(x) = . Find fog x −1 and gof.

⇒ (b, a) ∈ R Thus, (a, b) ∈ R ⇒ (b, a) ∈ R for all a, b ∈ Z. So, R is symmetric on Z. Now, let a, b, c ∈ Z such that (a, c) ∈ R and (b, a) ∈ R. Then, (a, b) ∈ R ⇒ (a − b) is divisible by n ⇒ a − b = np for some p ∈ z.

Solution:  Clearly, range f = domain g and range g = domain f. Hence, fog and gof both exist. Now,  x   x 2 (fog)(x) = f (g(x)) = f  = +2  x − 1  x − 1

⇒ (b, c) ∈ R

x2

⇒ (b − c) is divisible by n ⇒ b − c = nq for some q ∈ z.

=

(x − 1)2

+2

∴ (a, b) ∈ R and (b, c) ∈ R ⇒ (a − b) = np and (b − c) = nq

and

  (gof )(x) = g(f (x)) = g(x2 + 2) x2 + 2

⇒ (a − b) + (b − c) = np + nq

=

⇒ a − c = n(p + q) Hence, a − c is divisible by n.

x2 + 2

(x2 + 2) − 1

= x2 + 1

Hence, gof  : R → R and fog  : R → R are given by

⇒ (a, c) ∈ R Thus, (a, b) ∈ R and (b, c) ∈ R ⇒ (a, c) ∈ R for all a, b, c ∈ Z. So, R is a transitive relation on Z. Therefore, R is an equivalence relation on Z. 13. Show that the function f  : R → R defined by f(x) = 3x3 + 5 for all x ∈ R is a bijection. Solution:  Let x, y ∈ R. Then, f(x) = f(y) ⇒ 3x3 + 5 = 3y3 + 5 ⇒ x3 = y3 ⇒x=y Hence, f(x) = f(y) ⇒ x = y for all x, y ∈ R. Therefore, f is an injective map. Now, let y be an arbitrary element of R. Then, y −5 f(x) = y ⇒ 3x3 + 5 = y ⇒ x3 = 3 1/ 3  4 − 5  ⇒ x =   3  Hence, for all y ∈ R (co-domain) there exists  y − 5 1/3 x =  ∈ R (domain) such that  3 

Chapter 8.indd 407

x2 + 2 (gof )(x) =

x2 and (fog)(x) =

x2 + 1

(x − 1)2

+2

15. Consider f, g and h functions from R to R. Show that (f + g)oh = foh + goh Solution:  Since f, g and h are functions from R to R, (f + g)oh  : R → R and foh + goh  : R → R Now, ((f + g)oh)(x) = (f + g)(h(x)) ⇒ [(f + g)oh](x) = f [h(x)] + g[h(x)] ⇒ [(f + g)oh](x) = foh(x) + goh(x), for all x ∈ R Thus, (f + g)oh = foh + goh. 16. If f  : R → R is a bijection given by f(x) = x3 + 3, find f −1(x). Solution:  Let f(x) = y. Then, f(x) = y ⇒ x3 + 3 = y ⇒ x = (y − 3)1/3 ⇒ f −1(y) = (y − 3)1/3

22-08-2017 06:33:32

408     Set Theory and Algebra 

Thus, f −1: R → R is defined as f −1(x) = (x − 3)1/3 for all x ∈ R. 17. Show that function f  : R → R given by f(x) = x + 1 is not invertible. 2

n(A ∪ B) = n(A) + n(B) − n(A ∩ B) = 1720 + 1450 − n(A ∩ B) = 3170 − n(A ∩ B) Since A ∪ B ⊂ U

Solution:  We have f(x) = x2 + 1 Clearly, −2 ≠ 2 but f(−2) = f(2) = 5.

n(A ∪ B) ≤ n(U) 3170 − n(A ∩ B) ≤ 2000 n(A ∩ B) ≥ 1170

So, f is not a one-to-one function. Hence, f is not invertible. 18. For any two sets A and B, prove that (A ∩ B) ∪ (A − B) = A. Solution: (A ∩ B) ∪ (A − B) = (A ∩ B) ∪ (A ∩ B ′) = X ∪ (A ∩ B ′), where X = A ∩ B = (X ∪ A) ∩ (X ∪ B′) = A ∩ (A ∪ B′) ∵ X ∪ A = (A ∩ B ) ∪ A = A. Since, A ∩ B ⊂ A    X ∪ B ′ = (A ∩ B) ∪ B ′ = (A ∪ B ′) ∩ (B ∪ B ′)     = (A ∪ B′) ∩ U  = A ∪ B′

Hence, the least value of consumer who liked both the products is 1170. 21. Let F be a collection of all functions f  : {1, 2, 3} → {1, 2, 3}. If f and g ∈ F, define an equivalence relation ~ by f ~ g if and only if f(3) = g. (a) Find the equivalence classes defined by ~. (b) Find the number of elements in each equivalence class. Solution: (a) There are three equivalence classes, A = {f ∈ F, f(3) = i}, i = 1, 2. (b) Consider f(3) = 1.

 = A [∵ A ⊂ A ∪ B′]

f(3) can take any of the three values: 1, 2 or 3. Similarly, f(1) and f(2) can also take any of the three values.

19. Let R be the relation on the set N of natural numbers defined by R = {(a, b): a + 3b = 12, a ∈ N, b ∈ N}. Find R, domain and range of R.

Hence, there are 3 * 3 = 9 functions in each equivalence class.

Solution:  We have a + 3b = 12 a = 12 − 3b

22. Mr. X claims the following: If a relation R is both symmetric and transitive, then R is reflexive. For this, Mr. X offers the ­following proof.

Putting b = 1, 2, 3, we get a = 9, 6, 3, respectively. ∴ R = {(9, 1), (6, 2), (3, 3)}

“From xRy, using symmetry we get yRx. Now because R is transitive xRy and yRx together imply xRx. Therefore, R is reflexive.”

Domain of R = {9, 6, 3}

What is the flaw in Mr. X’s proof?

For b = 4, we get a = 0 ∉ N. Also, for b > 4, a ∉ N.

Range of R = {1, 2, 3} 20. In a survey of 2000 consumers, 1720 consumers liked X and 1450 consumers liked Y. What is the least number that must have liked both the products? Solution:  Let U be set of all consumers, A be set of consumers who liked X and B be set of consumers who liked Y. We are given that n(U) = 2000, n(A) = 1720, n(B) = 1450

Chapter 8.indd 408

Solution:  R is an empty set, and hence Mr. X’s proof becomes invalid. 23. Prove that P(A ∩ B) = P(A) ∩ P(B). Solution:  Say A = {0, 1, 2} and B = {1, 2} Now, P(A) = [{0}, {1}, {2}, {0, 1}, {0, 2}, {1, 2}, {0, 1, 2}, {f}] P(B) = [{1}, {2}, {1, 2}, {f}] P(A) ∩ P(B) = [{1}, {2}, {1, 2}, {f}]

22-08-2017 06:33:33

SOLVED EXAMPLES     409

A ∩ B = {1, 2}

R = (A ∩ B ∩ C) ∪ (A ∩ B ∩ C ) ∪ (A ∩ B ∩ C ) ∪ (A ∩ B ∩ C )

P(A ∩ B) = [{1}, {2}, {1, 2}, {f}] Thus, P(A ∩ B) = P(A) ∩ P(B)

Since all above regions are mutually disjoint,

Hence proved.

P(R) = P(A ∩ B ∩ C ) + P(A ∩ B ∩ C ) + P(A ∩ B ∩ C ) + P(A ∩ B ∩ C ) (1)

24. Consider the function h: N × N → N, so that h(x, y) = (2x + 1)2y − 1 where N = {0, 1 , 2, …} is a set of natural numbers:

We are given that P (A ∩ B ∩ C ) =

(a) Prove that function h is an injective function. (b) Prove that function h is a subjective function. P (C ) = Solution: (a) We have h: N * N → N, h(x, y) = (2x + 1)2y − 1

10 5 9 , P (A) = , P (B) = , 21 21 21

7 21

Also, P(A ∪ B ∪ C ) = P(A) + P(B) + P(C ) + P(A ∩ B ∩ C ) − P(A ∩ B) − P(A ∩ C ) − P(B ∩ C )

Let x1 = x2 and y1 = y2 26. Let G1 and G2 be subgroups of a group G.

∴ 2x1 = 2x2 ⇒ 2x1 + 1 = 2x2 = 1 Also, 2y1 = 2y2 

(1) (2)

Multiplying Eqs. (1) and (2), we get (2x1 + 1)2y1 = (2x2 + 1)2y2 ⇒ (2x1 + 1)2y1 − 1 = (2x2 + 1)2y2 − 1 ∴ h(x1, y1) = h(x2, y2) for x1 = x2, y1 = y2 Thus, h is injective.

(a) L  et G1 and G2 be two subgroups of G. Then, G1 ∩ G2 ≠ f since identity elements is common to both G1 and G2. Now, a ∈ G1 ∩ G2, b ∈ G1 ∩ G2 ⇒ ab−1 ∈ G1 ∩ G2 b ∈ G1 ∩ G2 ⇒ b ∈ G1 and b ∈ G2

Hence, (2x1 + 1)2y1 − 1 = 0 ⇒ (2x1 + 1)2y1 = 1 ⇒ (2x1 + 1)2y1 = 20 × 1 ⇒ 2

Solution: 

a ∈ G1 ∩ G2 ⇒ a ∈ G1 and a ∈ G2

(b) Let h(x, y) = 0

y1

(a) Show that G1 ∩ G2 is also a subgroup of G. (b) Is G1 ∪ G2 always a subgroup of G?

= 2 [∵ (2 × 1 + 1) is odd] 0

⇒ y1 = 0, x1 = 0. Therefore, h is surjective. 25. Out of a group of 21 persons, 9 eat vegetables, 10 eat fish and 7 eat eggs, 5 eat all three. How many persons eat at least two out of the three dishes? Solution:  Let A represents a person eating vegetables, B represents a person eating fish and C represents a person eating eggs.

9

a ∈ G1, b ∈ G1 ⇒ ab−1 ∈ G2, a ∈ G2, b ∈ G2 ab−1 ∈ G2 Also, ab−1 ∈ G1, ab−1 ∈ G2 ⇒ ab−1 ∈ G1 ∩ G2 Thus, a ∈ G1 ∩ G2, b ∈ G1 ∩ G2 and ab−1 ∈ G1, G2 Hence, G1 ∈ G2 is a subgroup of G.

B

A

But G1 and G2 are subgroups. Therefore,

10

(b) No, it is not necessary that G1 ∪ G2 is a ­subgroup of G. A group must satisfy four conditions: closure, ­associative, identity and inverse. However, it does not necessarily fulfill the inverse condition.

7 C The shaded region represents a person eating at least two items. The shaded region can be expressed as

Chapter 8.indd 409

27. Let (A, *) be a semigroup. Furthermore, for every a and b in A, if a ≠ b, then a * b ≠ b * a. (a) Show that for every a in A, a * a = a. (b) Show that for every a and b in A, a * b * a = a.

22-08-2017 06:33:36

410     Set Theory and Algebra 

(c) Show that for every a, b and c in A, a * b * c = a * c.

(b) Consider that a * H and b * H are not disjoint. Then there exists f such that f ∈ a * H and f ∈ b * H.

Solution: i. e. f = a * h1 = b * h2 (a) Because A is a semigroup,

where h1, h2 ∈ H

(a * b) * c = a * (b * c)

(1)

∴ a = b * h2 * h1−1

Now, putting b = a and c = a, we get (a * a) * a = a * (a * a) and A is not abelian. ⇒ a * a = a

Let x ∈ a * H or x = a * h3

(2)

⇒ x = b * (h2 * h1−1 * h3)

(b) Let b ∈ A, then

⇒ x = b * (h4)     (for h4 ∈ H)

b*b=b Multiplying both the sides of above equation by a, we get a * (b * b) = a * b

(3)

By associativity, we have (a * b) * b = a * b ⇒a*b=a Thus, a * b * a = (a * b) * a Using Eq. (2), we get a * b * a = a * a =a (c) a * b * c = (a * b) *c = a * c

(for h3 ∈ H)

Because groups are closed under * operation, they are identical. (c) B  ecause a * H and b * H are either disjoint or identical for any a and b ∈ G and cardinality of all sets is same (since |H |), there exists some partitioning of the elements in G into sets having |H | elements each. Also, each element of G belongs to some a * H. Hence, the order of any subgroup of a finite group divides the order of the group.

(4)

29. Let S = {0, 1, 2, 3, 4, 5, 6, 7} and ⊗ denotes ­multiplication modulo 8, i.e. x ⊗ y = (xy) mod 8

[using Eq. (4)]

(a) Prove that ({0, 1}, ⊗) is not a group. (b) Write three distinct groups (G, ⊗) where G ⊂ S and G has two elements.

28. Let G be a finite group and H be a subgroup of G, for a ∈ G, define aH = {ah|h ∈ H} (a) Show that |aH | = |H |. (b) Show that for every pair of elements a, b ∈ G, either aH = bH or aH and bH are disjoint. (c) Use the above conditions to argue that the order of H must divide the order of G.

Solution: (a) ⊗ does not have an inverse, hence ({0, 1}, ⊗) is not a group. (b) {1, 3}, {1, 5}, {1, 7} 30. Let ({p * q}, *) be a semigroup where p * p = q. Show that

Solution:  (a) Assume that |aH | ≠ |H | Hence, |aH | < |H | for two distinct values of h1 and h2 Now, a * h1 = a * h2 where h1, h2 ∈ H and * is the operation defined in the group. ⇒ a = a * h2 * h1−1 [inverse property of groups] ⇒ a−1 * a = h2 * h1−1 ⇒ 1 = h2 * h1−1 ⇒ h1 = h2 ∴ |aH | = |H | Hence proved.

Chapter 8.indd 410

(a) p * q = q * p

(b) q * p = q

Solution:  Let S = ({p, q}, *) be a semigroup. Because S is a semigroup, it is closed and associative, i.e. x * y ∈ S, x ∨ y ∈ S x * (y * z) = (x * y) * z, ∀x, y, z ∈ S (a) p * q = p * (p * p) = (p * p) * p = q * p     (∵ p * p = q) (b) Because S is closed, p * q ∈ S = {p, q}. Thus, p * q = p or p * q = q

22-08-2017 06:33:37

PRACTICE EXERCISE     411

Case 1: Let p * q = p

Case 2: Let p * q = q

q * q = q * (p * p)

q * q = (p * p) * q

= (q * p) * p = (p * q) * p = p * p = q

= p * (p * q) = p * q = q

PRACTICE EXERCISE 1. If f : R → R and g: R → R be two functions such that fog(x) = sinx2 and gof(x) sin2x, then find f(x) and g(x). (a) f(x) = x2 and g(x) = sin x (b) f(x) = sin x and g(x) = x2 (c) f(x) = sin x and g(x) = sin x2 (d) f(x) = cos x and g(x) = sin x cos x 2. If f  : R → R and g: R → R be functions defined by 2

f(x) = x + 1 and g(x) = sin x,

(a) fog = sin2x + 1, gof = sin x2 + 1 (b) fog = sin x2 + 1, gof = sin2 x + 1 (c) fog = (sin x + 1)2, gof = sin x + 1 (d) fog = sin x + 1, gof = (sin x + 1)2 3. Let A = {f, {f}, 1, {1, f}, 2}. Which of the following is FALSE? (b) {f} ∈ A (d) {2, f} ⊂ A

4. Two finite sets have x and y elements. The total number of subsets of the first set is 56 more than the total number of subsets of the second set. Find the values of x and y. (a) x = 6, y = 3 (c) x = 2, y = 4

(b) x = 4, y = 2 (d) x = 3, y = 6

5. A school awarded 38 prizes in football, 15 in basketball and 20 in cricket. If these prizes went to a total of 58 students and only 3 students got prizes in all three sports, how many received medals in exactly two of the three sports? (a) 18 (c) 12

(b) 15 (d) 9

6. In a class of 35 students, 17 have taken mathematics, 10 have taken mathematics but not computers. Find the students who have taken both mathematics and computers. (a) 11 (c) 9

Chapter 8.indd 411

(a) reflexive (c) transitive

(b) 7 (d) 13

(b) symmetric (d) all of these

8. Number of subsets of a set of order 3 is (a) 3 (c) 8

(b) 6 (d) 9

9. If A and B are any two sets and A ∪ B = A ∩ B, then (a) A = f (c) A = B

then find fog and gof.

(a) f ∈ A (c) {1} ∈ A

7. If R is an equivalence relation on a set A, then R−1 is

(b) B = f (d) none of these

10. Which of the following sets are null sets? (a) { } (c) {0}

(b) {f} (d) {0, f}

11. If relation R is defined on N by R = {(a, b): a divides b; a, b ∈ N}. Then R is (a) reflexive (c) transitive

(b) symmetric (d) reflexive and transitive

12. If X and Y are two sets, then X ∩ (Y ∩ X)C equals (a) f (c) X

(b) Y (d) X ∩Y C

13. Let X be any non-empty set. The identity function on set A is (a) injective (c) bijective

(b) surjective (d) none of these

14. The function f  : N → N defined by f(n) = 2n + 3 is (a) injective and surjective (b) injective and not surjective (c) not injective and surjective (d) not injective and not surjective 15. The number of distinct relations on a set of three elements is (a) 8 (c) 128

(b) 9 (d) 512

22-08-2017 06:33:37

412     Set Theory and Algebra 

16. A − (B ∪ C) is equal to (a) (A − B) ∪ (A − C ) (b) (A − B − C ) (c) (A − B) ∩ (A − C ) (d) (A − B) ∪ C Common Data for Questions 17 and 18: Let * be a Boolean operation defined as A * B = AB + AB C = A*B 17. A * A is equal to (a) 0 (c) A

(b) 1 (d) A

(a) bijective (b) injective but not surjective (c) surjective but not injective (d) neither surjective nor injective 25. The domain and range are same for (a) constant function (b) identity function (c) absolute value function (d) greatest integer function 26. If X and Y have 3 and 6 elements each, then minimum number of elements in A ∪ B is (a) 3 (c) 9

(b) 6 (d) 18

18. C * A is equal to (a) A (c) AB

(b) B (d) AB

Common Data for Questions 19−21: The following equations are given

27. If X and Y have 3 and 6 elements each, then maximum number of elements in A ∩ B is (a) 3 (c) 9

(b) 6 (d) 18

28. Hasse diagrams are drawn for AB + A C = 1 AC + B = 0 19. Value of A will be (a) 0 (c) ¥

(b) 1 (d) Indeterminant

(a) partially ordered sets (b) lattices (c) Boolean algebra (d) none of these 29. Maximal and minimal elements of the poset are

(a) 0 (c) ¥

5

3

20. Value of B will be (b) 1 (d) Indeterminant

4 2

21. Value of C will be (a) 0 (c) ¥

(b) 1 (d) Indeterminant 6

1 Common Data for Questions 22−23: In a survey on beverages, 80 people like Coca Cola, 60 like Pepsi, 50 like Fanta, 30 like Coca Cola and Pepsi, 20 like Pepsi and Fanta, 15 like Coca Cola and Fanta and 10 like all the three drinks. 22. How many people like at least one drink? (a) 135 (c) 10

(b) 30 (d) 45

23. How many people like at least two drinks? (a) 135 (c) 10

(b) 30 (d) 45

24. Let A and B be two sets. f  : A × B → B × A defined by f(a, b) = (b, a) is

Chapter 8.indd 412

(a) maximal 5, 6; minimal 2 (b) maximal 5, 6; minimal 1 (c) maximal 3, 5; minimal 1, 6 (d) none of these 30. The universal relation A × A on A is (a) all equivalence relation (b) anti-symmetric (c) a partial ordering relation (d) not symmentric and not anti-symmetric 31. Let D30 = {1, 2, 3, 5, 6, 10, 15, 30} and relation I be a partial ordering on D30. The L.U.B. of 10 and 15 is (a) 6 (c) 15

(b) 10 (d) 30

22-08-2017 06:33:38

PRACTICE EXERCISE     413

32. Let D30 ={1, 2, 3, 5, 6, 10, 15, 30} and relation I be a partial ordering on D30. The G.L.B. of 10 and 15 is (a) 10 (c) 5

(a) 1 (c) n + 1

(b) 6 (d) 1

33. If the function f  : [1, ¥] → [1, ¥] defined by f(x) = 2x(x − 1) is invertible, find f −1(x). 1 − 1 + 2 log2 x

1 + 1 + 2 log2 x (a)

(b) n (d) n2

41. The number of functions from an m element set to an n element set is (a) m + n (c) nm

(b) mn (d) m * n

(b) 4

4



1 − 1 + 4 log2 x

1 + 1 + 4 log2 x (c)

2



34. On Q, the set of all rational numbers, a binany operation * is defined by a∗b =

ab , for all a, b ∈ Q 5

(a) 1 (c) 5

(b) 10 (d) none of these

35. If f(x) and g(x) are defined on domains A and B, respectively, then domain of f(x) + g(x) is (a) A + B (c) A ∪ B

(b) A − B (d) A ∩ B

36. The binary relation S = f (empty set) on set A = {1, 2, 3} is (a) neither reflexive nor symmetric (b) symmetric and reflexive (c) transitive and reflexive (d) transitive and symmetric 37. Let X = {2, 3, 6, 12, 24} and ≤ be the partial order defined by X ≤ Y if X divides Y. Number of edges in the Hasse diagram of (X, ≤) is (a) 3 (c) 5

(b) 4 (d) none of these

38. Let P(S) denote the power set of S. Which of the following is always true? (a) P(P(S)) = P(S) (b) P(S) ∩ S = P(S) (c) P(S) ∩ P(P(S)) = [f ] (d) S ∈ P(S)

(a) 4 (c) 16

(b) 15 (d) 24

43. If A and B are sets and AC and BC denote the complements of the sets A and B, then set (A − B) ∪ (B − A) ∪ (A ∩ B) is equal to (a) A ∪ B (c) A ∩ B

(b) AC ∪ BC (d) AC ∩ BC

44. Let R be a non-empty relation on a collection of sets defined by A R B if and only if A ∩ B = f. Then (a) R is reflexive and transitive (b) R is symmetric and not transitive (c) R is an equivalence relation (d) R is not reflexive and not symmetric 45. If R is a symmetric and transitive relation on a set A, then (a) R is reflexive and hence an equivalence relation (b) R is reflexive and hence a partial order (c) R is not reflexive and hence not an equivalence relation (d) none of these 46. The number of elements in the power set P(S) of the set S = {[f ], 1, {2, 3}} is (a) 2 (c) 8

(b) 4 (d) none of the these

47. Let S be an infinite set and S1, S2, S3, …, Sn be set such that S1 ∪ S2 ∪ S3 ∪ … Sn = S, then

39. The number of binary relation on a set with n ­elements is (a) n2 2 (c) 2n

42. The number of equivalence relations of the set {1, 2, 3, 4} is

(d) 2

Chapter 8.indd 413

40. If A is a finite set with n elements, then number of elements in the largest equivalence relation of A is

(b) 2n (d) none of these

(a) at least one of the set Si is a finite set (b) not more than one of the set Si is a finite set (c) at least one of the set Si is an infinite set (d) none of the these

22-08-2017 06:33:40

414     Set Theory and Algebra 

48. If A is a finite set of size n, then number of elements in the power set of A × A is n

(a) 22 (c) 22n

2

(b) 2n (d) none of these

49. If A and B are sets with cardinalities m and n, respectively, then the number of one-to-one mappings from A to B, when m < n, is (a) mn (c) nCm

n (b) pm (d) none of these

50. Consider the following relations: R1(a, b) if (a + b) is even over the set of integers R2(a, b) if (a + b) is odd over the set of integers R3(a, b) if a ⋅ b > 0 over the set of non-zero rational numbers R4(a, b) if |a − b| ≤ 2 over the set of natural numbers Which of the following statement is correct? (a) R1 and R2 are equivalence relations, R3 and R4 are not (b) R1 and R3 are equivalence relations, R2 and R4 are not (c) R1 and R4 are equivalence relations, R2 and R3 are not (d) R1, R2, R3 and R4 are all equivalence relations 51. Consider the following statements: S 1: T  here exist infinite sets A, B and C such that A ∩ (B ∪ C) is finite. S 2: T  here exist two irrational numbers x and y such that (x + y) is rational. Which of the following is true about S1 and S2? (a) Only S1 is correct (b) Only S2 is correct (c) Both S1 or S2 are correct (d) None of S1 and S2 are correct 52. Let f  : A → B be a function, and let E and F be subsets of A. Consider the following statements about images. S1: f(E ∪ F) = f(E) ∪ f(F) S2: f(E ∩ F) = f(E) ∩ f(F) Which of the following is true about S1 and S2? (a) Only S1 is correct (b) Only S2 is correct (c) Both S1 and S2 are correct (d) None of S1 and S2 are correct

Chapter 8.indd 414

53. Let R1and R2 be two equivalence relations on a set. Consider the following statements: S1: R1 ∪ R2 is an equivalence statement. S2: R1 ∩ R2 is an equivalence statement. (a) Both statements are true (b) S1 is true but S2 is not true (c) S2 is true but S1 is not true (d) S1 and S2 are false 54. The binary relation r = {(1, 1), (2, 1), (2, 2), (2, 3), (2, 4), (3, 1), (3, 2), (3, 3), (3, 4)} on the set A = {1, 2, 3, 4} is (a) reflexive, symmetric and transitive (b) neither reflexive nor irreflexive but transitive (c) irreflexive, symmetric and transitive (d) irreflexive and anti-symmetric 55. In a room containing 28 people, there are 18 who speak English, 15 who speak Hindi and 22 who speak Kannad, 9 who speak both Hindi and English, 11 who speak both Hindi and Kannad and 13 who speak both Kannad and English. How many speak all the three languages? (a) 6 (c) 8

(b) 7 (d) 9

56. Let x and y be sets and |x | and |y| are their respective cardinalities. It is given that there are exactly 97 functions from x to y. From this, one can conclude that (a) |x | = 1, |y| = 97 (c) |x | = 97, |y| = 97

(b) |x | = 97, |y| = 1 (d) none of these

57. Let R denote the set of real numbers and f  : R × R → R × R be a bijective function defined by f (x, y ) = f (x + y, x — y). The inverse of f is given by  1 1  (a) f −1(x, y) =  ,  (x + y) (x − y)    (b) f −1(x, y) = (x − y, x + y) x + y x − y  , (c) f −1(x, y) =   2 2  (d) f −1(x, y) = [2(x − y), 2(x + y)] 58. Which of the following is true? (a) T  he set of all rational negative numbers forms a group under multiplication (b) The set of all non-singular matrices forms a group under multiplication

22-08-2017 06:33:41

PRACTICE EXERCISE     415

(c) The set of all matrices forms a group under multiplication (d) Both (b) and (c) 59. Let (Z, *) be an algebraic structure, where Z is the set of integers and the operation * is defined by n * m = maximum (n, m). Which of the following statement is true for (Z, *)? (a) (Z, *) is a monoid (b) (Z, *) is an abelian group (c) (Z, *) is a group (d) None of these 60. Which of the following statement is false? (a) The set of rational numbers is an abelian group under addition (b) The set of rational integers is an abelian group under addition (c) The set of rational numbers forms an abelian group under multiplication (d) None of these 61. Let A be the set of all non-singular matrices over real numbers and let * be the matrix multiplication operator. Then

64. The simultaneous equations on the Boolean variables x, y, z and w x + y + z = 1,     xz + w = 1,      

have the following solution for x, y, z and w, respectively (a) 0 (b) 1 (c) 1 (d) 1

1 1 0 0

0 0 1 0

0 1 1 0

65. Let L be a set with a relation R which is transitive, anti-symmetric and reflexive and for any two elements a, b ∈ L. Let least upper bound (a, b) and greatest lower bound (a, b) exist. Which of following option(s) is/are true? (a) L is a poset (b) L is a Boolean algebra (c) L is a lattice (d) Both (a) and (c) 66. In the lattice defined by the Hasse diagram, how many complements does the element “e” have? a

(a) A is close under * but is not a semigroup. (b) is a semigroup but not a monoid. (c) is a monoid but not a group. (d) is a group but not an abelian group.

b

c g

e 62. If a group (G, 0) is known to be abelian, then which one of the following option is true for G? −1

(a) g = g for every g ∈ G (b) g = g2 for every g ∈ G (c) (goh)2 = g2oh2 for every g, h ∈ G (d) G is of finite order 63. Match the following: A. Groups B. Semigroups C. Monoids D. Abelian groups

1. 2. 3. 4.

Chapter 8.indd 415

d

f (a) 2 (c) 0

(b) 3 (d) 1

67. A partial order ≤ is defined on the set S = {x, a1, a2, a3, …, an, y} as x ≤ ai for all i and ai ≤ y for all i, where n ≥ 1. The number of total order on the set S which ­contains partial order ≤ is (a) 1 (c) n + 2

(b) n (d) n!

68. What values of A, B, C and D satisfy the following simultaneously Boolean equations?

Codes A (a) 4 (b) 3 (c) 2 (d) 1

Associativity Identity Commutative Left inverse

 xy = 0  xy + z ⋅ w = 0

B 1 1 3 2

C 2 4 1 3

D 3 1 4 4

A + AB = 0, AB = AC , AB + AC + CD = CD (a) A = 1, B = 0, C = 0, D = 1 (b) A = 1, B = 1, C = 0, D = 0 (c) A = 1, B = 0, C = 1, D = 1 (d) A = 1, B = 0, C = 0, D = 0

22-08-2017 06:33:42

416     Set Theory and Algebra 

ANSWERS 1. (b)

13. (c)

25. (b)

37. (b)

49. (b)

61. (d)

2. (a)

14. (b)

26. (b)

38. (d)

50. (b)

62. (c)

3. (c)

15. (d)

27. (a)

39. (c)

51. (a)

63. (a)

4. (a)

16. (c)

28. (a)

40. (d)

52. (a)

64. (b)

5. (d)

17. (b)

29. (c)

41. (c)

53. (c)

65. (d)

6. (b)

18. (b)

30. (a)

42. (b)

54. (b)

66. (b)

7. (d)

19. (a)

31. (d)

43. (a)

55. (a)

67. (d)

8. (c)

20. (a)

32. (b)

44. (b)

56. (a)

68. (a), (d)

9. (c)

21. (b)

33. (c)

45. (d)

57. (c)

10. (a)

22. (a)

34. (c)

46. (c)

58. (b)

11. (d)

23. (d)

35. (d)

47. (c)

59. (d)

12. (a)

24. (a)

36. (d)

48. (b)

60. (d)

EXPLANATIONS AND HINTS 1. (b) We have fog(x) = sin x2 2

3. (c) It is clear that f and {f} ∈ A However, {1} ∉ A.

gof(x) = sin x

Also, {2, f} is a subset of A.

f(g(x)) = sin (x2) and g(f(x)) = (sin x)2

Hence, {1} ∉ A is a false statement.

⇒ f(x) = sin x and g(x) = x2

4. (a) Let A and B be two sets having x and y elements, respectively. The number of subsets of A = 2x.

2. (a) We have f(x) = x2 +1

Similarly, the number of subsets of B = 2y.

g(x) = sin x

We know that 2x + 2y = 56

Now, x2 ≥ 0 for all x ∈ R ⇒ x2 + 1 ≥ 1 for all x ∈ R

2y (2x−y − 1) = 23(23 − 1) = 56 ∴ 23 = 2y ⇒ y = 3

⇒ f(x) ≥ 1 for all x ∈ R

x−y=3

Range of f = [1, ¥].

x=3+3=6

Also, −1 ≤ sin x ≤ 1 for all x ∈ R.

⇒ x = 6, y = 3

Range of g = [−1, 1]. Clearly, range of f ⊆ domain of g and range of g ⊆ domain of f. So, gof  : R → R and fog: R → R are given by

5. (d) Suppose A, B and C denote the set of students who got prizes in football, basketball and cricket, respectively. Then, we have

gof(x) = g(f(x)) = g(x2 + 1) = sin x2 + 1 and fog(x) = f(g(x)) = f(sinx) = sin2 x + 1

Chapter 8.indd 416

n(A) = 38, n(B) = 15, n(C) = 20, n(A ∪ B ∪ C) = 58 and n(A ∩ B ∩ C) = 3

22-08-2017 06:33:42

EXPLANATIONS AND HINTS     417

Also, we know that n(A ∪ B ∪ C   ) = n(A) + n(B) + n(C ) − n(A ∩ B) − n (A ∩ C ) − n(B ∩ C ) + n(A ∩ B ∩ C ) ⇒ 59 = 3 8 + 15 + 20 − n(A ∩ B) − n(A ∩ C) − n(B ∩ C) + 3 ⇒ n(A ∩ B) + n(A ∩ C) + n(B ∩ C) = 18 Now, number of men who received medals in exactly two sports = n(A ∩ B) + n(A ∩ C) + n(B ∩ C) − 3n(A ∩ B ∩ C)

12. (a) We have X ∩ (Y ∪ X)C = X ∩ (Y C ∩ X C)  [Applying De Morgan’s law] = X ∩ Y C ∩ X C = f ∩ Y C [∵ X ∩ X C = f] = f [∵ f ∩ A = f] 13. (c) The identity function IA: X → X is defined as IX(a) = a for all a ∈ X.

= 18 − (3 × 3) = 9

Let a and b be any two elements of x. Then 6. (b) Suppose A denotes the set of students who have taken mathematics and B denotes the set of students who have taken computers.

IX(a) = Ix (b) ⇒ a = b Hence, Ix is an injective mapping.

Then, n(A ∪ B) = 35, n(A) = 17 and n(A − B) = 10.

Let b ⊂ X. Then, there exists a = b ∈ X such that

n(A) = n(A − B) + n(A ∩ B) ⇒ 17 = 10 + n(A ∩ B) ⇒ n(A ∩ B) = 17 − 10 = 7 7. (d) We know that R is an equivalence relation on A, then by definition R−1 is also an equivalence relation. Hence, R−1 is reflexive, symmetric and transitive.

IX(a) = a = b Hence, IX is a surjective mapping. Therefore, IX  : X → X is a bijection. 14. (b) We have f  : N → N defined by f(n) = 2n + 3 Let x and y be any two elements of N. Then f(x) = f(y)

8. (c) Number of subsets of a set of order 3 = 22 = 8.

⇒ 2x + 3 = 2y + 3 ⇒ 2x = 2y ⇒ x = y 9. (c) If the union and intersection of any two sets is same, then the sets are equal. Hence, A = B.

Hence, f(x) = f(y) ⇒ x = y for all x, y ∈ N. So, f is injective.

10. (a) All the other options apart from the first one have elements, hence they are not null or empty sets.

Now, let y be an arbitrary element of N. Then f(x) = y

11. (d) We have R = {(a, b): a divides b; a, b ∈ N} Now, for any a ∈ R, we have a divides a ⇒ (a, a) ∈ R for all a ∈ R

2x + 3 = y y −3 x= 2 y −3 Now, does not belong to N for all values of y ∈ N. 2 Hence, f is not surjective. Therefore, f is injective and not surjective.

⇒ R is reflexive. 15. (d) Total elements on set = 3 For any a, b ∈ R, if a divides b, then it is not necessary that b divides a ⇒ R is not symmetric. Let (a, b) ∈ R and (b, c) ∈ 0. Then,

Total distinct relations = 23×3 = 29 = 512 16. (c) De Morgan’s law can also be expressed as A − (B ∪ C ) = (A − B) ∩ (A − C )

a divides b and b divides c ⇒ a divides c ⇒ (a, c) ∈ R Thus, R is transitive.

Chapter 8.indd 417

17. (b) A * A = A.A + A.A =A+A=1

22-08-2017 06:33:44

418     Set Theory and Algebra 

Thus, f(a1, b1) = f(a2, b2) ⇒ (a1, b1) = (a2, b2) for all (a1, b1) (a2, b2) ∈ A × B

18. (b) C = A * B = AB + A.B Then,

So, f is injective.

C ∗ A = (AB + A ⋅ B) ⋅ A + (AB + A ⋅ B) ⋅ A

Let (b, a) be an arbitrary element of B × A. Then, b ∈ B and a ∈ A.

= (A ⋅ A ⋅ B + A ⋅ B ⋅ A) + (AB ⋅ A ⋅ B) ⋅ A

⇒ (a, b) ∈ A × B

= AB + 0 + ((A + B) ⋅ (A + B) ⋅ A)

Thus, for all (b, a) ∈ B × A there exists (a, b) ∈ A, B such that

= AB + (A ⋅ A + AB + BA + B ⋅ B)A = AB + (A ⋅ B ⋅ A + B ⋅ A ⋅ A)

f(a, b) = (b, a)

= AB + (AB) = B(A + A)

Therefore, f  : A × B → B × A is surjective.

=B Hence, f is bijective. 19. (a) We have two Boolean equations,



 AB + A C = 1    AC + B = 0

(1) (2)

25. (b) By definition, the domain and range of an identity function is same. 26. (b) For any two sets, X and Y, minimum number of elements of X ∪ Y is the elements in the bigger set.

From Eq. (2), AC = 0 and B = 0

Therefore, answer is 6. Using these in Eq. (1), we get AC = 1 and AC = 0 ⇒ AC + AC = 1 + 0 ⇒ C(A + A ) = 1 ⇒ C = 1 ∴ A(1) = 0

27. (a) For any two sets, X and Y, maximum number of elements of X ∩ Y is the elements in the smaller set. Hence, answer is 3. 28. (a) By definition, Hasse diagrams are used to represent partially ordered sets.

⇒A=0 20. (a) As already calculated in the previous question, B = 0.

29. (c) It is clear from the poset that 3 and 5 are the maximal, and 1 and 6 are the minimal.

21. (b) As already calculated in Question 19, C = 1.

30. (a) The universal relation A × A on A is an equivalence relation.

22. (a) Let A, B and C denote the sets of people who like Coca Cola, Pepsi and Fanta, respectively. Thus, we know that n(A) = 80, n(B) = 60, n(C ) = 50, n(A ∩ B) = 30, n(A ∩ C ) = 20, n(B ∩ C ) = 15, n(A ∩ B ∩ C ) = 10 Therefore, n(A ∪ B ∪ C ) = 80 + 60 + 50 − 30 − 20 − 15 + 10 = 135

31. (d) The only upper bound of 10 and 15 is 30. Hence, L.U.B. is 30. 32. (b) The greatest lower bound of 10 and 15 is 6. 33. (c) We have (fof —1 )(x) = x for all x ∈ [1, ¥) ⇒ (fof —1 )(x) = x

23. (d) Total people who like at least two drinks

⇒ 2f

= n(A ∩ B) + n(A ∩ C ) + n(B ∩ C ) − 2n(A ∩ B ∩ C )

⇒f

= 30 + 20 + 15 − 2 (10)

−1

(x){f −1 (x)−1}

−1

(x){f

−1

⇒ {f  

= 45 24. (a) Let (a1, b1) and (a2, b2) ∈ A × B such that

−1

=x

(x) − 1} = log2 x

(x)} − f  −1(x) − log2x = 0 2

⇒ f −1(x) =

1 ± 1 + 4 log2 x

1 + 1 + 4 log2 x =

2 f (a1, b1) = f(a2, b2)

∵ f 

(x) ∈ (1, ∞), f −1(x) ≥ 1 

⇒ (b1, a1) = (b2, a2)



⇒ b1 = b2 and a1 = a2

34. (c) Let e be identity element. Then

⇒ (a1, b1) = (a2, b2)

Chapter 8.indd 418

2 −1

a * e = a = e * a for all a ∈ Q

22-08-2017 06:33:46

EXPLANATIONS AND HINTS     419

ea ae = a for all a ∈ Q = a and 5 5 ⇒e=5



35. (d) If f(x) and g(x) have domains A and B, respectively, then domain of f(x) + g(x) is A ∩ B. 36. (d) In mathematics, a binary relation S over a set A is transitive if an element a is related to an element b, b is in turn related to an element c, and then a is also related to c. A binary relation S over a set A is symmetric if it holds for all a and b in A that if a is related to b then b is related to a. Hence, S is transitive and symmetric; however, it is not reflexive since (1, 1), (2, 2) and (3, 3) ∉ S. 37. (b) The Hasse diagram for (X, ≤) is given by

These are four ways to assign the four elements into one bin of size 3 and of size 1. There are three ways to assign the four elements into one bin of size 2. In this, two of the elements are related to each other and the other two are related to themselves. There are six ways to assign four elements into one bin of sizes 2, 1 and 1. There is just one way to assign the four elements into bins of sizes 1, 1, 1 and 1. Total equivalence relations = 1 + 4 + 3 + 6 + 1 = 15. 43. (a) We know that (A − B) ∪ (B − A) = (A ∪ B ) − (A ∩ B ) Hence,

12

24

(A − B) ∪ (B − A) ∪ (A ∩ B) = (A ∪ B ) − (A ∩ B ) ∪ (A ∩ B) Let X = (A ∪ B), Y = (A ∩ B)

6

Also, (X − Y ) ∪ Y = X ∪ Y 2

3

Substituting values of X and Y, we get (A − B) ∪ (B − A) ∪ (A ∩ B) = (A ∪ B) ∪ (A ∩ B)

The number of edges in the Hasse diagram is 4. 38. (d) Let S = {a1 , a2 ,… , an } where n is a finite number, then the power set of S is given by P (S ) = {a1 , a2 ,..., an , {a1 , a2 },..., {a1 , a2 ,..., an }} Hence, S ∈ P(S) is always true.

= A∪B 44. (b) Solution:  We know that A R B if and only if A ∩ B = f. A∩A≠f Hence, R is not reflexive. A∩B=B∩A

39. (c) The number of distinct relations are given by Hence, R is symmetric. 2

2n.n = 2n 40. (d) Largest equivalence relation can include all the n elements of A and the relation will be with the elements of A itself. Hence, the largest equivalent relation has n * n = n2. 41. (c) Functions are defined from m to n elements Hence, the total number of functions = nm. 42. (b) We just need to calculate the number of ways of placing the four elements of our set into the following sized bins: 4, 3 + 1, 2 + 2, 2 + 1 + 1, 1 + 1 + 1 + 1 There is just one way to put four elements into a bin of size 4.

Chapter 8.indd 419

If A ∩ B and B ∩ C, then it is not necessary that A ∩ C. Hence, R is not transitive. Therefore, R is symmetric and not transitive. 45. (d) We know that R is symmetric and transitive on set A. However, it does not say anything about reflexive. Hence, the correct option is (d). 46. (c) The power set P(S ) for S = {{f}, 1, {2, 3}} is {f, 1, {2, 3, 2, 3}} = {[f ], [1], [1, 2], [1, 3], [1, 2, 3], [2], [3], [2, 3]} Hence, P(S) contains 8 elements. 47. (c) We know that the union of two sets contains the elements of the bigger set.

22-08-2017 06:33:48

420     Set Theory and Algebra 

Hence, if Si is the union of n sets and its value is infinity, then the value of at least one set is infinity. Hence, the correct option is (c). 48. (b) Total elements in A = n. Total elements in A × A = n2. 2 Total elements in power set of A × A = 2n . 49. (b) We are given that A has m elements and B has n elements. Also, n > m. Now, the one-to-one mappings have a distinct value for each district argument. Therefore, the total number of one-to-one ­mappings is npm .

However, if |a − b| ≤ 2 and |b − c| ≤ 2, then it is not necessary that |a − c| ≤ 2. Hence, R4 is not an equivalence relation. We conclude that R1 and R3 are equivalence relations and R2 and R4 are not equivalence relations. 51. (a) S1: There exist infinite sets A, B and C such that A ∩ (B ∪ C) is finite. The union of two infinite sets is always infinite. However, the intersection of two infinite sets can be finite. Example:

B = {1} ∪ {3, 4, 5, …} C = {2} ∪ {3, 4, 5, …} A ∩ (B ∪ C) = {3, 4, 5, …}

50. (b) R1(a, b) if (a + b) is even over set of integers. We know that a + a = 2a which is even. Hence, (a, a) ∈ R1 for a∈I Therefore, R1 is reflexive. If a + b is even, then b + a is also even. Hence, (b, a) ∈ R1 Therefore, R1 is symmetric. If a + b is even and b + c is even, then a + c is also even. Hence, (a, b) ∈ R1, (b, c) ∈ R1 and (a, c) ∈ R1. Therefore, R1 is transitive. Hence, R1 is an equivalence relation. R2(a, b) if (a + b) is odd over set of integers. However, (a + a) is not odd. Hence, (a, a) ∉ R1. Therefore, R1 is not reflexive and hence not an equivalence relation. R3(a, b) if (a, b) > 0 over set of non-zero rational numbers. If a is non-zero, then a2 > 0 for all values of a being non-zero rational numbers. Hence, (a, a) ∈ R3.

Chapter 8.indd 420

A = {0} ∪ {3, 4, 5, …}

S1 is true. S2: There exist two irrational numbers x and y such that (x + y) is rational. Sum of two irrational numbers is always irrational. Hence, S2 is false. 52. (a) S1 is true since both f(E ∪ F) and f(E) ∪ f(F) will contain exactly the same images of all elements in E and F. Now, let f be a constant function such that its range is {1}. Now, let E and F be two partitions of set A, then (A ∩ B) = f and f(f ) is not defined, but f(E) ∩ f(F) is {1}. Hence, S2 is false. 53. (c) Consider R1 and R2 to be relations on set A. ∴ R1 ⊆ A × A and R2 ⊆ A × A R1 ∩ R2 ⊆ A × A ∴ R1 ∩ R2 is also a relation on A. Let a ∈ A

Thus, R3 is reflexive.

⇒ ( a, a) ∈ R1 and (a, a) ∈ R2 [∵ R1 and R2 are reflexive]

If a ⋅ b > 0, then b ⋅ a > 0. Hence, (b, a) ∈ R3.

⇒ (a, a) ∈ R1 ∩ R2

Thus, R3 is symmetric.

Hence, R1 ∩ R2 is reflexive.

If a ⋅ b > 0 and b ⋅ c > 0, then (a ⋅ c) > 0. Hence, (a, c) ∈ R3.

Let a, b ∈ A such that (a, b) ∈ R1 ∩ R2. Then

Therefore, R3 is transitive. Hence, R3 is an equivalence relation.

⇒ ( a, b) ∈ R1 and (a, b) ∈ R2 ⇒ (b, a) ∈ R1 and (b, a) ∈ R2 [∵ R1 and R2 are symmetric]

R4(a, b) if |a − b| ≤ 2 over set of natural numbers.

⇒(b, a) ∈ R1 ∩ R2

|a − a| = 0, i.e. |a − a| ≤ 2. Hence, (a, a) ∈ R4 for a∈N

Hence, R1 ∩ R2 is symmetric.

(a, b) ∈ R1 ∩ R2

Thus, R4 is reflexive.

Let a, b, c ∈ A such that (a, b) ∈ R1 ∩ R2 and (b, c) ∈ R1 ∩ R2. Then,

Also, |a − b| = |b − a|. Hence, R4 is symmetric.

(a, b) ∈ R1 ∩ R2 and (b, c) ∈ R1 ∩ R2

22-08-2017 06:33:50

EXPLANATIONS AND HINTS     421

{((a, b) ∈ R1 and (a, b) ∈ R2)} and {((b, c) ∈ R1 and (b, c) ∈ R2)} {(a, b) ∈ R1, (b, c) ∈ R1} and {(a, b) ∈ R2, (a, b) ∈ R2} ⇒ (a, c) ∈ R1 and (a, c) ∈ R2 ⇒ (a, c) ∈ R1 ∩ R2 Thus, (a, b) ∈ R1 ∩ R2 and (b, c) ∈ R1 ∩ R2 ⇒ (a, c) ∈ R1 ∩ R2 So, R1 ∩ R2 is transitive on A. Hence, R1 ∩ R2 is an equivalence relation on A. Now, let A = a, b, c and R1 and R2 be two relations on A given by R1 = {(a, a), (b, b), (c, c), (a, b) (b, a)} and R2 = {(a, a), (b, b), (c, c), (b, c), (c, b)} R1 and R2 both are equivalence relations on A. But R1 ∩ R2 is not transitive because (a, b) ∈ R1 ∪ R2 and (b, c) ∈ R1 ∪ R2 but (a, c)∉ R1 ∪ R2. Hence, R1 ∪ R2 is not an equivalence relation on A.

Hence, the correct option is (b). 59. (d) Z is closed and associative, but it does not have an identity element. Hence, it is just a semigroup. Thus, none of the options are true. 60. (d) For a group to be abelian, it should be commutative. Now, addition of numbers and integers is commutative. Also, multiplication of rational numbers is commutative. Hence, all the statements are true. 61. (d) A is a group because a * (b * c ) = (a * b ) * c for every a, b, c ∈ A and every element of A is invertible as A is a set of non-singular matrices. However, A is not an abelian group because multiplication of matrices is not commutative. Hence, is a group but not an abelian group. 62. (c) Let us assume that there exists two consecutive integers, n and n + 1, such that

Therefore, S1 is false and S2 is true. 54. (b) The relation is not reflexive because it does not contain the entry (4, 4). It is not irreflexive because it contains the entries (1, 1), (2, 2) and (3, 3). Hence, the only possible answer is (b). However, it can also be proved that the relation r is transitive since (2, 3), (3, 4) and (2, 4) ∈ r. Hence, r is neither reflexive, nor irreflexive but transitive. 55. (a) We know that n(A ∪ B ∪ C) = n(A) + n(B) + n(C) − n(A ∩ B) − n(A ∩ C) − n(B ∩ C) + n(A ∩ B ∩ C)

(goh)n = g nohn and (goh)n+1 = gn+1ohn+1 for every g, h ∈ G. Then, g n +1ohn +1 = (goh) = (goh) goh = g n ohn goh Hence, g n +1ohn +1 = g n ohn (goh) n +1

n

⇒ gohn = hn og Taking n = i and n = i + 1, we get gohi = hiog and by taking n = i + 1 and n = i + 2, we have gohi +1 = hi +1og.

Hence, this shows that gohi +1 = hi +1oa ⇒ goh = hog. i∩ +1B ∩ C) i +1 28 = 18 + 15 + 22 − 9 − 11 − 13 + n(A goh = h oa ⇒ goh = hog. Thus, G is abelian; hence, ⇒ n(A ∩ B ∩ C) = 28 − 55 + 33 = 6 (goh)2 = g2oh2 for every g, h ∈ G. Hence, the total number of people who speak all the three languages = 6.

56. (a) We know that there are exactly 97 functions from x to y. Now, the total number of element is x = 1 and y = 97. x + y x − y  , 57. (c) f −1(x, y) =   2 2  58. (b) Set of all rational negative numbers does not form a group under multiplication because it is not closed and does not contain an identity element. Set of all non-singular matrices forms a group under multiplication because it is associative, ­identity element exists and is invertible. However, set of all matrices does not form a group under multiplication since all matrices are not invertible.

Chapter 8.indd 421

63. (a) By definition, a semigroup with identity is a monoid. A semigroup follows associativity and an abelian group follows commutative. Then the correct matching would be A = 4, B = 1, C = 2 and D = 3. 64. (b) The given equations are x + y + z = 1 xy = 0 xz + w = 1 xy + z ⋅ w = 0 

(1) (2) (3) (4)

For Eq. (2) to be true, both x and y cannot be 1. Hence, answer cannot be (b). Substituting the value of Eq. (2) in Eq. (4), we get z ⋅w = 0

22-08-2017 06:33:52

422        Set Theory and Algebra 

Hence, z and w cannot be 0. Therefore, answer cannot be (a) and (d). Thus, the answer is (b).

AB = AC 

(2)

AB + AC + CD = CD 

(3)

65. (d) Because L is a set with a relation R which is transitive, anti-symmetric and ­reflexive for any two elements a, b ∈ L, it is a partially ordered set or a poset.

Considering Eq. (1),

Because L.U.B. (least upper bound) and G.L.B. (greatest lower bound) exist, L is a lattice. Hence, the correct option is (d).

Considering Eq. (2),

A + AB = 0 ⇒ A+B = 0

[∵ X + Y = 0 ] (4)

AB = AC

⇒B =C

(5)

66. (b) e has three complements a, c, d. 67. (d) The total order on the set S which ­contains partial order ≤ is given by

(a1 , a1 ) , (a1 , a2 ) ,..., (a1 , an ) (a2 , a2 ) , (a2 , a3 ) ,..., (a1 , an )

Now, because all the options have A = 1, we substitute A = 1 in Eq. (4), 1+B = 0 ⇒ B+0 = 0 ⇒ B = 0 Substituting value of B in Eq. (5), we get C=0



Substituting values of A, B and C in Eq. (3), we get

(an , an )

1.0 + 1.0 + CD = CD ⇒ CD = 1 + CD = 1

Hence, total orders are n(n − 1)(n − 2)… n = n ! 68. (a), (d) We are given A + AB = 0 

(1)

For C = 0 and D = 0, 1, the Boolean equations will be satisfied. Hence, the correct options are (a) and (d).  Ans. (a), (d)

SOLVED GATE PREVIOUS YEARS’ QUESTIONS 1. Consider the binary relation: S = {(x, y)|y = x + 1 and x, y ∈ {0, 1, 2}} The reflexive transitive closure of S is (a) {(x, y)|y > x and x, y ∈ {0, 1, 2}} (b) {(x, y)|y ≥ x and x, y ∈ {0, 1, 2}} (c) {(x, y)|y < x and x, y ∈ {0, 1, 2}} (d) {(x, y)|y ≤ x and x, y ∈ {0, 1, 2}} (GATE 2004, 1 Mark) Solution:  Reflexive closure of a relation R on set X is the smallest reflexive relation which contains R. So to find reflexive closure of R, we just take its union with set {(a,a) ∀a ∈ X}. We are given S = {(0, 1), (1, 2)} We calculate the reflexive by taking its union with set {(0, 0), (1, 1),(2, 2)}. Thus, SR = {(0, 0), (0, 1),(1, 1), (1, 2), (2, 2).

Chapter 8.indd 422

The transitive closure is also defined in the similar way, i.e. smallest transitive relation which ­contains S. So we try to make SR transitive now. For this, we have to see where does it violate property of transitivity, and then add appropriate pair. So here we see that we have (0, 1) and (1, 2), but not (0, 2), so we add that to make relation SRT = {(0, 0), (0, 1), (0, 2), (1, 1), (1, 2), (2, 2)}. Now, we can see that SRT is transitive. Among the given options, only (b) matches SRT, hence it is the correct answer.  Ans. (b) 2. The inclusion of which of the following options sets into S = {{1, 3}, {1, 2, 3}, {1, 3, 5}, {1, 2, 4}, {1, 2, 3, 4, 5}} is necessary and sufficient to make S a complete lattice under the partial order defined by set containment?

23-08-2017 05:34:30

SOLVED GATE PREVIOUS YEARS’ QUESTIONS     423

(a) {1} (b) {1}, {2, 3} (c) {1}, {1, 3} (d) {1}, {1, 3}, {1, 2, 3, 4}, {1, 2, 3, 5} 

Solution:  Number of elements in W = 2xy Number of elements in Z = z Number of functions from Z to E = z2xy

(GATE 2004, 2 Marks)

 Solution:  A lattice is complete if every subset of partial order set has a supremum and infimum element. For example, here we are given a partial order set S. Now it will be a complete lattice ­whatever the subset we choose, it has a supremum and infimum element. Here, supremum element will be the union of all sets in the subset we choose and infimum element will be the intersection of all the sets in the subset we choose.

Ans. (a)

5. If P, Q, R are subsets of the universal set U, then (P ∩ Q ∩ R) ∪ (P    C ∩ Q ∩ R) ∪ QC ∪ RC is (a) QC ∪ RC (c) P    C ∪ QC ∪ RC 

(b) P ∪ QC ∪ RC (d) U (GATE 2008, 1 Mark)

Solution:  We are given

Thus, we take the two subsets, {1, 3, 5} and {1, 2, 4} and find their intersection, i.e. {1} which is not present in S.  Ans. (a)

(P ∩ Q ∩ R) ∪ (P C ∩ Q ∩ R) ∪ QC ∪ R C = (P ∪ P C ) ∩ (Q ∩ R) ∪ (QC ∪ R C ) = U ∩ ((Q ∩ R) ∪ (Q ∩ R)C ) = U ∩U = U

3. Let A, B and C be non-empty sets and let X = (A — B) — C and Y = (A — C) — (B — C) 

Ans. (d)

Which one of the following is TRUE? (b) x ⊂ y (d) None of these

(a) x = y (c) y ⊂ x

6. Which one of the following in NOT necessarily a property of a Group? (a) Commutativity (b) Associativity (c) Existence of inverse for every element (d) Existence of indentity

(GATE 2005, 1 Mark) Solution:  We know that (A − B ) = A ∩ B ′, where B′ is complement of set B. So, X = (A − B) − C = (A ∩ B ′) ∩ C ′= A ∩ B ′ ∩ C ′



Solution:  By definition in a group, commutativity is not necessary.  Ans. (a)

Similarly, Y = (A − C ) − (B − C )

7. Consider the binary relation R = {(x, y), (x, z), (z, y), (z, y)} on the set {x, y, z}.

= (A ∩ C ′) ∩ (B ∩ C ′)′ = (A ∩ C ′) ∩ (B ′ ∪ C ) = (A ∩ C ′ ∩ B ′) ∪ (A ∩ C ′ ∩ C ) = (A ∩ C ′ ∩ B ′) ∪ (A ∩ f )

Which one of the following is TRUE? (a) R is symmetric but NOT antisymmetric (b) R is NOT symmetric but antisymmetric (c) R is both symmetric and antisymmetric (d) R is neither symmetric nor antisymetric

= (A ∩ C ′ ∩ B ′) ∪ f = (A ∩ C ′ ∩ B ′) = X



Hence, the answer is (a). 

xy



(GATE 2009, 1 Mark)

Ans. (a)

4. Let X, Y, Z be sets of sizes x, y and z, respectively. Let W = X × Y and E be the set of all subsets of W. The number of functions from Z to E is (a) Z 2 x+y (c) Z 2

(GATE 2009, 1 Mark)

(b) Z × 2xy (d) Z  xyz

Solution:  A relation is symmetric if for each (x, y) in R, (y, x) is also in R. This relation R in question is not symmetric as (x, y) is present but (y, x) is absent. A relation is anti-symmetric if for each pair of (x, y) and (y, x) in R, x = y. Here R is not ­anti-symmetric as for (x, z) and (z, x), x ≠ z.

(GATE 2006, 1 Mark) 

Chapter 8.indd 423

Ans. (d)

22-08-2017 06:33:57

424     Set Theory and Algebra 

f (A∩ B) = {2} ; f (A) = {1, 2} ; f (B) = {1, 2}

8. What is the possible number of reflexive relations on a set of 5 elements? (a) 210 (c) 220 

(b) 215 (d) 225

f (A) ∩ f (B) = {1, 2}



(GATE 2010, 1 Mark) Solution:  Consider a table of size 5 × 5 in which each possible pair is listed. In a reflexive relation, we must include all 5 diagonal elements. So from rest of the 20 elements, we have a choice whether to include them or not. So we have 220 possible reflexive relations.  Ans. (c)

Therefore, options (a), (b) and (c) are not true. Hence, option (d) is true. Ans. (d) 11. Let G be a group with 15 elements. Let L be a subgroup of G. It is known that L ≠ G and that the size of L is at least 4. The size of L is . 

9. Consider the set S = {1, w, w }, where w and w are cube roots of unity. If * denotes the multiplication operation, the structure (S, *) forms 2



Solution:  According to Lagrange’s theorem, the order of subgroup divides order of group. Order of group is 15.   3, 5, 15 divide 15. According to the given condition, size should be atleast 4 and cannot be equal to that of group. So, 5 is the order of subgroup. Ans. 5

(GATE 2010, 1 Mark)

Solution:  The answer is `a group’ since the other three options (b), (c) and (d) require two operations over structure.

12. If V1 and V2 are 4-dimensional subspaces of a 6-dimensional vector space V, then the smallest possible dimension of V1 ∩ V2 is . 

Also, we can test the set for all the properties of a group, i.e. closure, associativity, identity element and inverse element to find that it fulfills all the properties. 

10. Let X and Y be finite sets and f : X → Y be a function. Which one of the following statements is TRUE? (a) For any subsets A and B of X, |f (A ∪ B)| |f (A)| + |f (B)| (b) For any subsets A and B of X, f (A ∩ B) f (A) ∩ f (B) (c) For any subsets A and B of X, |f (A ∩ B)| min {|f (A)|, |f (B)|} (d) For any subsets S and T of Y, f  -1(S ∩ T) f  -1(S) ∩ f  -1(T).

= = = =

(GATE 2014, 1 Mark) Solution:  We have f : X ® Y defined by f(a) = 1, f(b) = 1, f(c) = 2, where X = {a, b, c}, Y = {1, 2} Let A = {a, c}, B = {b, c} be subsets of X then



Chapter 8.indd 424

(GATE 2014, 1 Mark) Solution:  Let V be {e1, e2, e3, e4, e5, e6}. Then V1 = {e1, e3, e5, e6} V2 = {e2, e3, e4, e5} Smallest possible set of V1 ∩ V2 = 2. Ans. 2

Ans. (a)

f (A∪ B) = 2; f (A) = 2; f (B) = 2

(GATE 2014, 1 Mark)

2

(a) A group (b) A ring (c) An integral domain (d) A field



f ( A∩ B ) = 1

13. Consider the following relation on subsets of the set S of integers between 1 and 2014. For two distinct subsets U and V of S we say U < V if the minimum element in the symmetric difference of the two sets is in U. Consider the following two statements: S1: There is a subset of S that is larger than every other subset. S2: There is a subset of S that is smaller than every other subset. Which one of the following is CORRECT? (a) Both S1 and S2 are true (b) S1 is true and S2 is false (c) S2 is true and S1 is false (d) Neither S1 nor S2 is true 

(GATE 2014, 2 Marks) Solution:  Both S1 and S2 are true. Ans. (a)

14. Consider the set of all functions f : {0, 1,…, 2014}→{0, 1,…, 2014} such that f (f (i)) = i, for 0 ≤ f ≤ 2014. Consider the following statements.

22-08-2017 06:33:58

SOLVED GATE PREVIOUS YEARS’ QUESTIONS     425

P. For each such function it must be the case that for every i, f (i) = i. Q. For each such function it must be the case that for some i, f (i) = i. R. Each such function must be onto.

So, the element of such group are 4 which are

{x, y, e, x × y } . Ans. 4 16. For a set A, the power set of A is denoted by 2A. If A = {5, {6}, {7}}, which of the following options are TRUE?

Which one of the following is CORRECT? (i) f ∈ 2A (a) P, Q and R are true (b)  Only Q and R are true (c) Only P and Q are true (d)  Only R is true 

(ii) f ⊆ 2A (iii) {5, {6}} ∈ 2A

(GATE 2014, 2 Marks)

(iv) {5, {6}} ⊆ 2A

Solution:  Let us consider a function as

(a) (i) and (iii) only (b) (ii) and (iii) only (c) (i), (ii) and (iii) only (d) (i), (ii) and (iv) only

f (0) = 1, f (1) = 0, f (2) = 3, f (3) = 2,… , d f (2014) = 2014 f (2012) = 2013, f (2013) = 2012 and



Clearly, f (f (i)) = i for 0 ≤ i ≤ 2014

(GATE 2015, 1 Mark) Solution:  2A → Power set of A, that is set of all subsets of A. Since empty set is a subset of every set, therefore

Here, f (i) ≠ i for every i and f(i) = i for some i. Also, f is onto. Hence, only Q and R are true. Ans. (b)

f ⊆ 2A and f ∈ 2A

15. There are two elements x, y in a group (G,*) such that every element in the group can be written as a product of some number of x’s and y’s in some order. It is known that

Since, {5, {6}} ⊆ A and 5 ∉ 2A Thus, {5, {6}} ∈ 2A and {5, {6}} ⊆ 2A

x×x=y×y=x×y×x×y=y×x×y×x=e

Hence, (i), (ii) and (iii) are true. where e is the identity element. The maximum number of elements in such a group is . 

(GATE 2014, 2 Marks) Solution:  We are given, x × x = e ⇒ x is its own inverse. y × y = e ⇒ y is its own inverse.

(x × y ) × (x × y ) = e ⇒ (x × y ) is its own inverse.

Ans. (c) 17. Suppose U is the power set of the set S = {1, 2, 3, 4, 5, 6}. For any T ∈ U , let |T | denote the number of elements in T and T ′ denote the complement of T. For any T , R ∈ U , let T\R be the set of all elements in T which are not in R. Which one of the following is true?

(y × x) × (y × x) = e ⇒ (y × x) is its own inverse.

(a) ∀X ∈ U (| X |= |X ′ |)

Also, x × x × e = e × e can be rewritten as follows:

(b) ∃X ∈ U ∃Y ∈ U ( X = 2, Y = 5 and X ∩ Y = 0)

x × y × y × x = e × y × y × e = e [∵ y × y = e]

(c) ∀X ∈ U ∀Y ∈ U ( X = 2, Y = 3 and X \ Y = 0) (d) ∀X ∈ U ∀Y ∈ U (X \ Y = Y ′ \ X ′ )

(x × y ) × (y × x) = e shows that (x × y ) and (y × x) are each other’s inverse and we already know that (x × y ) and (y × x) are inverse of its own. As per (G,∗) to be group any element should have only one inverse element. (unique) This process x × y = y × x (is one element)

Chapter 8.indd 425



(GATE 2015, 2 Marks) Solution:  X \ Y = X − Y = X ∩ Y ′ and Y ′ \ X ′ = Y ′ − X ′ = Y ′ ∩ (X ′)′ = Y ′ ∩ X = X ∩ Y ′

22-08-2017 06:34:05

426     Set Theory and Algebra 

Solution:  It is given that

Therefore, X \ Y = Y ′ \ X ′, ∀X, Y ∈ U

f(n) = f(n/2),  if n is even

Ans. (d)

f(n) = f(n + 5),  if n is odd

18. Let R be a relation on the set of ordered pairs of positive integers such that [(p, q), (r, s)] ∈ R, if and only if p — s = q — r. Which one of the following is true about R?

Let us use the definition of function to show the following: (i) f(1) = f(2) = f(3) = f(4) = f(6) = f(7) = f(8) = f(9)  = ⋅⋅⋅

(a) Both reflexive and symmetric (b) Reflexive but not symmetric (c) Not reflexive but symmetric (d) Neither reflexive nor symmetric 

(ii) f(5) = f(10) = f(15) = f(20) = ⋅⋅⋅ Hence, we conclude that the range of f(n) comprises two distinct components.

(GATE 2015, 2 Marks) Solution: Since p − q ≠ q − p

Ans. 2 20. The number of integers between 1 and 500 (both inclusive) that are divisible by 3 or 5 or 7 is _________.

Thus, ( p, q)R( p, q) ⇒ R is not reflexive.

 Let ( p, q)R(r, s) then p − s = q − r

Solution:  The number of integers divisible by 3 or 5 or 7 is given by

⇒ r −q = s− p ⇒ (r, s)R( p, q)

n(3 ∨ 5 ∨ 7) = n(3) + n(5) + n(7) − n(3 ∧ 5) − n(5 ∧ 7) − n(3 ∧ 7) + n(3 ∧ 5 ∧ 7)

Thus, R is symmetric. Ans. (c) 19. A function f :  + →  + , defined on the set of positive integers N+, satisfies the following properties: f(n) = f(n/2),  if n is even f(n) = f(n + 5),  if n is odd Let R = {i | ∃j: f(j) = i} be the set of distinct values that f takes. The maximum possible size of R is __________. 

Chapter 8.indd 426

(GATE 2017, 2 Marks)

 500   500   500   500  − + + =  3   5   7   15   500   500   500  − − +  35   21   105  = 166 + 100 + 71 - 33 - 14 - 23 + 4 = 271 Ans. (271.0)

(GATE 2016, 2 Marks)

22-08-2017 06:34:06

CHAPTER 9

COMBINATORY

INTRODUCTION

COUNTING

The selection of objects from a given set where the order of arrangement matters is known as permutation.

There are two fundamental principles of counting, which are principle of addition and principle of multiplication. The two principles form the basis of permutations and combinations.

The selection of objects from a given set where the order of arrangements does not matter is known as combination. For example, suppose we have three objects x, y and z and we have to choose two of them. Then, the two objects can be selected using permutations in six different ways, i.e., xy, yx, xz, zx, yz and zy. Also, the two objects can be selected using combinations in three different ways, i.e., xy, yz and xz. If one operation can be performed in m different ways and if corresponding to each way of performing this operation, there are n different ways of performing the second operation, the two operations can be performed in (m × n) ways.

Chapter 09.indd 427

Fundamental Principle of Addition If there are two tasks such that they can be performed independently in m and n ways, respectively, then either of the two jobs can be performed in (m + n) ways.

Fundamental Principle of Multiplication If there are two tasks such that one of them can be completed in m ways and after its completion the other one can be completed in n ways, then the two tasks in succession can be completed in (m × n) ways.

22-08-2017 06:35:29

428        Combinatory 

PERMUTATIONS As already stated, arranging or selecting objects where the order of arrangement matters is called permutation. The generalized formula of n different things taken r things at a time is given by n! n Pr = P(n, r) = (n − r ) !

Conditional Permutations Permutations where repetition of items are allowed, distinction between some of the items are ignored or a particular item occurs in every arrangement are called permutation under certain conditions. 1. The number of permutations of n different objects taken r at a time, when a particular object is to be always included in each arrangement, is given by r ⋅ n −1 Pr −1 . 2. The number of permutations of n distinct objects taken r at a time, when a particular object is not taken in any arrangement, is given by n −1 Pr . 3. The number of permutations of n different objects taken r at a time in which two specified objects always occur together is given by 2 !(r − 1) n −2 Pr −2 .

Permutations When all Objects are Not Distinct Here, we discuss the permutations of a given number of objects when all objects are not distinct. The number of mutually distinguishable permutations of n things, taken all at a time, of which p are of one kind and q of another such that p + q = n, is given by: n! p !q ! Now, the following cases arise: 1. The number of permutations of n things, of which p1 are alike of one kind, p2 are alike of second kind and so on such that p1 + p2 +  + pr = n, is given by n! p1 ! p2 !...pr ! 2. The number of permutations of n things, of which p are alike of one kind, q are alike of second kind and remaining are all distinct, is given by n! p !q ! 3. Suppose there are r things to be arranged allowing repetitions. Let p1 , p2 ,..., pr be the integers such that the first object occurs exactly p1 times, the second occurs exactly p2 times and so on. Then

Chapter 09.indd 428

the total number of permutations of these r objects to the above condition is ( p1 + p2 +  + pr )! p1 ! p2 ! p3 !...pr !

Circular Permutations In all the cases discussed so far, we have considered the permutations to be linear, i.e., objects are arranged in a straight line. Now, we will discuss the arrangements along a circle. Such arrangements are called circular permutations. In a circular permutation, there is neither a beginning nor the end. It should be noted that there is no difference between clockwise and counterclockwise arrangement of objects. The number of ways of arranging n distinct objects along a round table is (n − 1)! . The number of ways of arranging n persons along a round table so that no person has the same two neigh1 bours is (n − 1)! . 2

COMBINATIONS Each of different selections made by taking some or all objects, irrespective of the order of arrangements is called combinations. The generalized formula of n different things taken r things at a time is given by n! n Cr = C(n, r) = r ! (n − r ) !

Properties of Combinations Some important properties of combinations are given as follows: 1. For 0 ≤ r ≤ n, n

Cr = n Cn −r

2. If n and r are positive integers such that r ≤ n, then n

Cr + n Cr −1 =

n +1

Cr

3. If n and r are positive integers such that 1 ≤ r ≤ n, then n n Cr = × n −1Cr −1 r 4. If 1 ≤ r ≤ n, then n⋅ n

5. If

n −1

Cr −1 = (n − r + 1) ⋅ n Cr −1

Cx = n Cy , then x = y ⇒ x+y = n

22-08-2017 06:35:33

RECURRENCE RELATION         429

6. If n is an even natural number, then the greatest of the values n C0 , n C1 , n C2 , ..., n Cn is given by n Cn /2 .

The above G(x) is called the generating function for the sequence a0 , a1 , a2 , …

7. If n is an odd natural number, then the greatest of the values n C0 , n C1 , n C2 , ..., n Cn is given by n C(n −1)/2 = n C(n +1)/2 .

Ordinary Generating Function

Conditional Combinations

The ordinary generating function of a sequence an is given by ∞ G (an ; x) = ∑ an xn n =0

1. The number of ways in which (m + n) things can be divided in the two groups containing m and n things, (m + n)! respectively, is given by C(m + n, n) = . m!n! If m = n, then two cases arise: (a) When distinction can be made between the groups, then the required number of ways = (2m)! . (m !)2

(b) When no distinction can be made, then the (2m)! number of ways = . 2 !(m !)2 2. The total number of ways in which a selection can be made out of p + q + r things of which p are of one kind, q are of another kind and remaining r are different, then the required number of ways is given by ( p + 1)(q + 1)2r − 1 3. The number of combinations of n different things taken r at a time, when p particular things are always included, is given by n− p

Cr − p

4. The number of combinations of n different things, taken r at a time, when p particular things are always to be excluded, is given by n− p

The ordinary generating function of a two-dimensional array am,n (where n and m are natural numbers) is G (am, n ; x, y ) =





am, n xm y n

m, n =0

Exponential Generating Function The exponential generating function of a sequence an is given by ∞ xn EG (an ; x) = ∑ an n! n =0 Exponential generating functions are preferred over ordinary generating functions for combinatorial enumeration problems involving labelled objects.

Poisson Generating Function The Poisson generating function of a sequence an is given by ∞ xn PG (an ; x) = ∑ an e−x = e−x EG (an ; x) ! n n =0

RECURRENCE RELATION

Cr

GENERATING FUNCTIONS A generating function is a formal power series in one indeterminate, whose coefficients encode information about a sequence of number an that is indexed by the natural numbers. It can also be defined as a powerful tool for solving counting problems. Generally, in counting problems, we are often interested in counting the number of objects of `size n’, which we denote by an. By varying n, we get different values of an. In this way, we get a sequence of real numbers a0 , a1 , a2 , … Now, from the sequence, we can define a power series which in some sense can be regarded as an infinite degree polynomial. G (x) = a0 + a1x + a2 x2 + 

Chapter 09.indd 429

If an is the probability mass function of a discrete random variable, then its ordinary generating function is called probability generating function.

A recurrence relation is a sequence that gives us a connection between two consecutive terms. The two terms can be given as U n and U n−1 or U n and U n+1 . It defines a sequence once one or more initial terms are given, and each further term of the sequence is defined as a function of the preceding terms. Some of the common types of recurrence relations are logistic map, Fibonacci numbers and binomial coefficients.

Logistic Map Logistic map is of the form xn +1 = rxn (1 − xn ) with a given constant r; given the initial term x0, each subsequent term is determined by this relation. Solving a recurrence relation means obtaining a closed-form solution: a non-recursive function of n.

22-08-2017 06:35:37

430        Combinatory 

Fibonacci Numbers

ASYMPTOTIC ANALYSIS

Fibonacci numbers have a linear, homogeneous recurrence relation with constant coefficients. They are defined using the following recurrence relation: fn = fn −1 + fn −2 with f0 = 0 and f1 = 1. Hence, we can deduce that recurrence yields the equations f2 = f1 + f0 f3 = f2 + f1 f4 = f3 + f2 and so on.

Binomial Coefficients A binomial coefficient is an example of a multi-dimensional recurrence relation which counts the number of ways of selecting i out of a set of n elements. They can be computed by the recurrence relation n −1

 i  =  i −1 +  n

n −1 i



n

Let us consider a function f (n) given by f (n) = n3 + 2n. We want to analyze the properties of this function when (n) becomes very large. As is evident from the function; the term 2n becomes insignificant as compared to n3 as n becomes large. The function f (n) in this case is said to be asymptotically equivalent to n3 as n → ∞. This is written symbolically as f (n) ~ n3. In general, for given functions f and g of a natural number variable n, f ∼ g (as n → ∞)

 0  = n = 1.

With the base cases

Asymptotic analysis is a method of describing the limiting behaviour. In applied mathematics, asymptotic analysis is used to build numerical methods to approximate equation solutions. It finds a number of applications in different branches of science, which includes analysis of algorithms in computer science; statistical mechanics for studying the behaviour of very large physical systems and in accident analysis to identify for example the cause of crash through count modeling with large number of crash counts in a given time and space. One use of asymptotic analysis is evident from the following example.

n

If and only if

f (n ) lim

SUMMATION

n →∞

The operation of adding a sequence of numbers to obtain their sum or total is called summation. For finite sequences, summation always produces a well-defined sum. However, for the summation of an infinite sequence of values (also called series), we often use a limit to define its value. n

∑ f(a)

Summations are represented as be expanded as

and it can

a =1

n

∑ f(a) = f(1) + f(2) + f(3) +  + f(n)

n

∑ a2 = a =1 n

∑a

3

a =1 n

=

n (n + 1)(2n + 1)

Asymptotic expansion of a function f(x) is expressed in terms of a series such that taking any initial partial sum provides an asymptotic formula for f and successive terms provide an increasingly accurate description of the order of growth of f. The partial sums do not necessarily converge. Mathematically, it is expressed as follows. f ∼ g1

Chapter 09.indd 430

f ∼ g1 + g2 and f ∼ g1 +  + gk The requirement that the successive sums improve the approximation may then be expressed as: f − (g1 +  + gk ) = o (gk )

6



n (n + 1) 2



2

∑ i = ni, where i is a constant a =1

=1

This relation is an equivalence relation on the set of functions of n. The equivalence class of f informally consists of all functions g which are approximately equal to f in a relative sense, in the limit.

a =1

Some of the common summation rules that we use are given as follows: n n (n + 1) ∑a = 2 a =1

g (n )

In case the asymptotic expansion does not converge, for any particular value of the argument, there will be a particular partial sum which provides the best approximation and adding additional terms will decrease the accuracy. However, this optimal partial sum will usually have more terms as the argument approaches the limit value.

22-08-2017 06:35:39

SOLVED EXAMPLES         431

SOLVED EXAMPLES 1. In a class of 20 boys and 18 girls, the teacher wants to select one student as the class monitor. In how many ways the teacher can make this selection. Solution:  The teacher can choose a student as class monitor by choosing a boy in 20 ways or by choosing a girl in 18 ways. Hence, the teacher can choose a class monitor in 38 ways. 2. In a class of 20 boys and 18 girls. The teacher wants to select one boy and one girl as class monitor. In how many ways the teacher can make the selection. Solution:  The teacher can choose a boy in 20 ways and a girl in 18 ways. The teacher can select one boy and one girl as class monitor in 20 × 18 = 360 ways. 3. What is the number of different permutations of APPLE? Solution:  We have to choose five letter words from the letters A, P, P, L and E. Hence, total arrangements = 5 P5 = 5 ! . However, letter P is used twice and hence the total 5! number of arrangements = = 5 × 4 × 3 = 60. 2! 4. Find the number of permutations formed by using all the letters of the word `ENGINEERING’ when all E’s come together.

3 men out of 6 men can be selected in 6 C3 ways Similarly, 2 women out of 4 women can be selected in 4 C2 ways Hence, the committee can be formed in 6 C3 × 4 C2 ways 6! 4! = 5 × 4 × 3 × 2 = 120 × 3!×3! 2!×2! 7. If P(n − 1, 3): P(n, 4) = 1:9, then find n. Solution:  We have P(n − 1, 3): P(n, 4) = 1:9 (n − 1)! (n − 1 − 3) ! 1 (n − 1)! (n − 4)! 1 × ∴ = ⇒ = n! n! 9 9 (n − 4) ! ( n − 4) ! (n − 1)! 1 1 1 = ⇒ = ⇒n=9 n! n 9 9 8. If n Pr = 720 and n Cr = 120, then find the value of r. Solution:  We know that n

Cr =

Pr r!

⇒ 120 =

720 r!

n

Solution:  Since we want all the E’s to come together, we treat it as a single letter. ⇒ r! = 6 Therefore, total number of arrangements = 9 ! . Now, 3! = 3 × 2 = 6 However, N, I and G are repeated 3, 2 and 2 times, Hence, r! = 3! ⇒ r = 3. respectively. 9! 9. Thus, total number of arrangements = = 15120 . How many triangles can be formed by joining the 3!×2!×2! vertices of a hexagon? = 15120.

9! 3! ×2!×2! 5. Sonakshi has six friends. In how many ways can she invite one or more of them to dinner? Solution:  The order of inviting friends does not matter. Hence, we use the formula of combination. Sonakshi can invite the friends in 6 C1 ways. Similarly, Sonakshi can invite one or more of them in 6 C1 + 6 C2 + 6 C3 + 6 C4 + 6 C5 + 6 C6 ways

Solution:  Total vertices of a hexagon = 6 Triangle is formed by selecting a group of 3 vertices. Hence, this can be done in 6 C3 ways. Number of triangles = 6 C3 =

6! = 20. 3!3!

10. How many numbers greater than a million can be formed with the digits 2, 3, 0, 3, 4, 2, 3?

= 6 + 15 + 20 + 15 + 6 + 1 = 63 6. How many different committees of 5 members can be formed from 6 men and 4 women on which exactly 3 men and 2 women serve? Solution:  We know that Total members in the committee = 5 (3 men and 2 women)

Chapter 09.indd 431

Solution:  Any number greater than a million will contain all seven digits. Also, 2 and 3 are repeated twice and thrice, respectively. 7! Number of arrangements = = 420. 2! ×3! However, arrangements also include 0 at the ­millionth place.

22-08-2017 06:35:42

432        Combinatory 

These 6 digits can be arranged in

6! = 60 ways. 2!×3!

Hence, the required numbers = 420 − 60 = 360. 11. If f (x) = x3 + 3x + 2, what would f (x) be for x → ∞.

Solution:  f (x) ∼ x3 as the remaining terms are insignificant as x tends to infinity. Also, (x3 + 3x + 2)/x3 = (1 + 3/x2 + 2/x3), which equals 1 as x → ∞

PRACTICE EXERCISE 1. In how many ways can 5 red and 4 white balls be drawn from a bag containing 10 red and 8 white balls? (a)

10

(c)

18

C5 × 8 C4 C9

(b) 8 C5 × 10 C4 (d)

10

(a) 5440 (c) 1680

C9

2. The number of triangles that can be formed by joining the vertices of an octagon is (a) 28 (c) 20

(b) 40 (d) 56

(b) 215 (d) none of these

4. The number of ways to arrange the letters of the word `CHEESE’ is (a) 60 (c) 120

5. In a class of 10 students, there are 3 girls A, B and C. In how many different ways can they be arranged in a row such that no two of the three girls are seated together?

6. If

56

Pr + 6 :

(a) 51 (c) 45 7. If

n

54

(b) 336 × 8! (d) 56 × 8!

(b) 1175 (d) 1100

11. In how many ways can it be done if a particular player is always chosen? (a) 1000 (c) 1001

(a) 520 (c) 770

(b) 1500 (d) 1221

(b) 364 (d) 216

13. If 6 identical coins are arranged in a row, then the number of arrangements in which 4 are heads and 2 are tails are (a) 4 (c) 12

(b) 6 (d) 15

14. A polygon has 44 diagonals. What is the number of sides of the polygon?

(b) 49 (d) 41 then

(b) 3 (d) 9

Common Data for Q. 8 and Q. 9  We have a word `ALLAHABAD’ and we make different words from the letters. 8. In how many of them vowels occupy the even positions? (a) 60 (b) 45 (c) 54 (d) 36

Chapter 09.indd 432

(a) 1250 (c) 1365

Pr + 3 = 30800 : 1 , find the value of r.

Cr −1 = 36, n Cr = 84 and n Cr +1 = 126 ,

find r C2 . (a) 10 (c) 1

Common Data for Q. 10 to Q. 12 We have to choose a cricket team of 11 players out of 15.

12. In how many ways can it be done if a particular player is never chosen?

(b) 64 (d) 6

(a) 336 × 7! (c) 176 × 7!

(b) 7560 (d) 5880

10. In how many ways can it be done if there is no restriction on the selection?

3. In an examination, there are four multiple questions and each question has four choices. The number of ways in which a student can fail to get all answers correct is (a) 216 (c) 81

9. In how many of them both `L’ do not come together?

(a) 8 (c) 10

(b) 9 (d) 11

15. A box contains 5 black and 6 white balls. In how many ways can 6 balls be selected so that there are at least 2 balls of each colour? (a) 425 (c) 388

(b) 525 (d) 510

16. Out of 7 consonants and 4 vowels, how many words of 3 consonants and 2 vowels can be formed? (a) 25000 (c) 25200

(b) 26400 (d) 27400

22-08-2017 06:35:44

ANSWERS         433

Common Data for Q. 17 and Q. 18 We have to form four-letter words using the letters of the word `FAILURE’. 17. In how many ways can it be done if `F’ is included in each other. (a) 180 (c) 480

(b) 360 (d) 540

18. In how many ways can it be done if `F’ is not included in any word. (a) 180 (c) 480 19. If

n

(b) 360 (d) 540

P4 = 20 × n P2 , then what is the value of n?

(a) 7 (c) 3

(b) 5 (d) 1

20. How many different signals can be given using any number of flags from four flags of different colours? (a) 105 (c) 64

(b) 35 (d) 25

21. How many numbers lying between 100 and 1000 can be formed with the digits 1, 2, 3, 4, 5 if the repetition of digits is not allowed? (a) 30 (c) 75

(b) 60 (d) 50

(b) 4220 (d) 3660

25. How many different words can be formed from the letters of the word `MISSISSIPPI’? (a) 45480 (c) 34650

(b) 27600 (d) 69300

26. A man has 7 relatives, 4 of them are ladies and 3 are gentlemen. His wife also has 7 relatives, 3 of them are ladies and 4 are gentlemen. In how many ways can they invite a dinner party of 3 ladies and 3 gentlemen so that there are 3 of the man’s relatives and 3 of the wife’s relatives? (a) 485 (c) 505

(b) 375 (d) 420

27. Find the value of n, if A.P. (a) 7 (c) 3

2n

C1 , 2n C2 and

2n

C3 are in

(b) 7/2 (d) 3/2

28. From 4 mangoes, 5 oranges and 6 apples, how many selections of fruits can be made by taking at least one of them? (a) 210 (c) 209

(b) 254 (d) 201

(a) 120 (c) 660

(b) 40320 (d) 720

30. Five boys and five girls form a line with the boys and girls standing at alternate places. Find the number of ways of making the line.

23. How many words can be formed from the letters of the word `DAUGHTER’ so that the vowels are always together? (a) 2400 (c) 3600

(b) 36000 (d) 40320

29. How many words can be formed from the letters of the word `TRIANGLE’ if each word begins with `T’ and ends with `E’?

22. How many four-letter words can be formed out of the letters of the word `LOGARITHMS’ if repetition of letters is not allowed? (a) 1080 (c) 5040

(a) 3600 (c) 4032

(b) 2150 (d) 4320

(a) 5 × 5 !

(b) (5!)2

(c) (5!)3

(d) 2 × (5!)2

31. If f (n) = n3 + 2n, then one of the following expressions is correct.

24. How many words can be formed from the letters of the word `DAUGHTER’ so that the vowels are not together?

(a) f (n) ∼ n3 (c) f (n) ∼ n (n2 + 2)

(b) f (n) ∼ 2n (d) None of these

ANSWERS 1. (a)

5. (a)

9. (d)

13. (d)

17. (c)

21. (b)

25. (c)

29. (d)

2. (d)

6. (d)

10. (c)

14. (d)

18. (b)

22. (c)

26. (a)

30. (d)

3. (b)

7. (b)

11. (c)

15. (a)

19. (a)

23. (d)

27. (b)

31. (a)

4. (c)

8. (a)

12. (b)

16. (c)

20. (c)

24. (b)

28. (c)

Chapter 09.indd 433

22-08-2017 06:35:45

434        Combinatory 

EXPLANATIONS AND HINTS 1. (a) 5 red balls can be selected from 10 red balls in 10 C5 ways. Similarly, 4 white balls can be selected from 8 white balls in 8 C4 ways. Therefore, number of ways in which 5 red balls and 4 white balls can be selected would be equal to: 10

⇒ 51 − r = 10 ⇒ r = 41 7. (b) We know that n

C5 × 8 C4

n

2. (d) Total vertices required to draw a triangle = 3.



Total vertices of octagon = 8.

n

8×7 ×6 8! = C3 = = = 56. 3×2 5! × 3!

⇒ ⇒

3. (b) Each multiple question has 4 options. Hence, an answer can be chosen in 4 ways.

Hence, the number of ways in which a student can fail to get all the answers correct = 216 − 1 = 215. 4. (c) Since order of letters does not matter, we can form 6 P6 = 6! words from the letters of `CHEESE’. However, letter `E’ is repeated thrice. Therefore, total different words from letters `C, H, 6! S, and E’ = = 12. 3! 5. (a) We know that Total number of boys = 7

r +1 n−r

r + 1 84 2 = = ⇒ (2n − 5r) = 3 n − r 126 3

8

Also there is only one combination when all the combinations are correct.

Cr +1

=

(1)

Now,

Total triangles that can be drawn from octagon

Now, there are 4 questions and 4 options can be chosen in 4 × 4 × 4 × 4 = 256 ways.

Cr



Cr −1 n

Cr

=

r 36 = n − (r − 1) 84

r r 36 3 = ⇒ = n − r + 1 84 n −r +1 7 ⇒ 3n − 10r = − 3

(2)

Multiplying Eq. (1) by 2 and subtracting Eq. (2) from Eq. (1), we get 4n − 3n = 63 − (−3) ⇒ n=9 Substituting the value of n in Eq. (2), we get 27 − 10r = − 3 30 r= =3 10 3! Therefore, r C2 = 3C2 = =3 2 ! 1! 8. (a) There are 9 letters in `ALLAHABAD’ out of which 4 are A’s, 2 are L’s and rest are all distinct. 9! = 7560. 4! 2! There are 4 vowels and all are `A’. There are 4 even places and the vowels can occupy the even places in 1 way. Hence, the total number of words =

Total number of girls = 3 Seven boys can be arranged in a row = 7 P7 = 7 ! ways. Now, we have 8 places to arrange 3 girls = 8 P3 . Hence, the number of arrangements = 8 P3 × 7! = 336 × 7 ! . 6. (d) We know that 56



Pr + 6 :

54

Pr + 3 = 30800 : 1

56 ! 54 ! = 30800 : 1 : (56 − r − 6)! (54 − r − 3)! ⇒ ⇒

56 ! 54 ! = 30800 : 1 : (50 − r)! (51 − r)!

(51 − r)! 56 ! 30800 × = (50 − r)! 54 ! 1

⇒ 56 × 55 × (51 − r) = 30800

Chapter 09.indd 434

Now, 5 letters can be arranged in 5! ways. Also, `L’ is repeated twice. Hence, letters can be arranged 5! in ways. 2! Hence, the total number of words in which vowels 5! 4! occupies the even places = × = 60. 2! 4! 9. (d) Considering both `L’ together and treating them as one letter, and that `A’ repeats 4 times, 8! the letters can be arranged in ways = 1680. 4! Number of words in which both `L’ do not come together

22-08-2017 06:35:50

EXPLANATIONS AND HINTS         435

= Total words − Number of words in which both `L’ come together

Hence, there are 7 C3 × 4C2 ways of arranging 3 consonants and 2 vowels. Now, each group contains 5 letters and can be arranged in 5! ways. Hence, the required number of words = 7! 4! (7 C3 × 4C2 ) ×5! = × ×5! = 25200. 4! × 3! 2! × 2!

= 7560 − 1680 = 5880. 10. (c) The total number of ways of selecting 11 players out of 15 is 15

C11 = 15C4 =

15 × 14 × 13 × 12 = 1365 4 × 3 × 2 ×1

11. (c) If a particular player is always chosen, then 10 players are selected out of the remaining 14 players.

17. (c) We know that there are 7 letters in the word `FAILURE’. Also, `F’ is included in all the 4-letter words. Now, the rest of 3 letters can be selected from the remaining 6 letters in 6 C3 ways.

Now, the 4-letter word can be arranged in 4! ways. 6! Hence, the total number of words = 6 C3 × 4 ! = × 4 ! = 480. 3!×3! 6! 6 C3 × 4 ! = × 4 ! = 480. 3!×3! 12. (b) If a particular player is never chosen, then 11 players are selected out of the remaining 14 players. 18. (b) Now, `F’ is not included. Hence, the 4 letters from the remaining 6 letters can be selected in 6 C4 14 14 Hence, required number of ways = C11 = C3 = 364 . ways. 14 C11 = 14C3 = 364 . Also, the 4-letter word can be arranged in 4! ways. 13. (d) Total coins = 6 6! Hence, the total number of words = 6 C4 × 4 ! = ×4! Now, they can be arranged in 6! ways. 4! ×2! 6! 6 × 4 ! = 360. However, we also know that 4 are heads and 2 are C4 × 4 ! = 4!×2! tails. 6! 19. (a) We know that Hence, coins can be arranged in = 15 ways. 4! × 2! n P4 = 20 × n P2 14. (d) Suppose there are n sides of a polygon. n! n! ⇒ = 20 × We know that the number of diagonals of n-sided (n − 2)! (n − 4)! n(n − 3) ⇒ (n − 4)! × 20 = (n − 2)! polygon is . 2 ⇒ (n − 2) (n − 3) = 20 n(n − 3) = 44 ⇒ n2 − 3n = 88 Now, ⇒ n2 − 5n + 6 = 20 ⇒ n2 − 5n − 14 = 0 2 ⇒ (n − 7) (n + 2) = 0 ⇒ n = 7   (∵ n > 0) ⇒ (n − 11)(n + 8) = 0 ⇒ n = 11   (∵ n > 0)

Hence, remaining number of ways = 14 C10 = 14C4 = 1001.

14

C10 = 14C4 = 1001.

15. (a) The selection of 6 balls consisting of at least 2 balls each from 5 black balls and from 6 white balls can be made in the following way by selecting 2 black balls out of 5 and 4 white balls out of 6. This can be done in 5 C2 × 6C4 ways. Selecting 3 black balls out of 5 and 3 white balls out of 6 = 5 C3 × 6C3 ways. Selecting 4 black balls out of 5 and 2 white balls out of 6 = 5 C4 × 6C2 ways. Hence, total number of arrangements = 5 C2 × 6C4 + 5C3 × 6C3 + 5C4 × 6C2

5

Hence, the total number of signals = 4 P1 + 4 P2 + 4 P3 + 4 P4 = 4 + (4 × 3) + (4!) + 4! = 4 + 12 + 24 + 24 = 64 21. (b) All numbers between 100 and 1000 are three digit numbers.

Now, we have6 to find permutations of five digits 1, 6 C2 × 6C4 + 5C3 × + 55Ctaken 4 × Cthree 2 2, C 3,34, at a time. Hence, the required

5! 6! 5! 6! 5! 6! + + = × × × 3! × 2! 4! × 2! 3! × 2! 3! × 3! 4! × 1! 4! × 2! = 10 × 15 + 10 × 20 + 5 × 15 = 425 16. (c) We can choose 3 consonants out of 7 in 7 C3 ways. Similarly, we can choose 2 vowels out of 4 in 4 C2 ways.

Chapter 09.indd 435

20. (c) Signals can be made using 1, 2, 3 or 4 flags.

number of numbers is 5! 5 P3 = = 5 × 4 × 3 = 60 2! 22. (c) We know that there are 10 letters in the word `LOGARITHMS’ and all are distinct. Hence, the total number of 4-letter words = 10 ! 10 P4 = = 10 × 9 × 8 × 7 = 5040. 6!

10

P4 =

10 ! 6!

22-08-2017 06:35:55

436        Combinatory 

23. (d) There are 8 letters in the word `DAUGHTER’, 3 vowels `A, U, E’ and 5 consonants `D, G, H, T, R’. Now, since vowels come together, we consider them as one letter. Therefore, the 6 letters can be arranged in 6 P6 = 6 ! ways.

(2n − 1)(2n − 2) 3

The vowels can be arranged in 3! ways.

⇒ 4n − 2 = 2 +

Hence, the required number of ways = 6! × 3! = 720 × 6 = 4320.

⇒ 12n − 6 = 6 + 4n2 − 2n − 4n + 2

24. (b) Total number of words using letters of the word `DAUGHTER’ is 8 P8 = 8 ! = 40320. Total number of words in which vowels are together = 4320. Total number of words in which vowels are not together = 40320 − 432 = 36000. 25. (c) Total letters in the word `MISSISSIPPI’ are 11. However, `S’ is repeated 4 times, `I’ is repeated 4 times and `P’ is repeated 2 times. 11! Hence, total number of words = = 34650. 4! × 4! × 2! 26. (a) Let L and G denote ladies and gentlemen, respectively. Now, they need to invite 3 ladies and 3 gentlemen. Also, they need 3 relatives each from the man’s and the wife’s relatives. This can be done in the following ways: •• 3

L from the man’s relatives and 3 G wife’s relatives •• 3 G from the man’s relatives and 3 G wife’s relatives •• 2 L, 1 G from the man’s relatives and from the wife’s relatives. •• 1 L, 2 G from the man’s relatives and from the wife’s relatives.

4 9±5 7 ⇒ = ,1 4 2 Now, n ≠ 1 since 2 C3 is not possible. ∴ n = 7/2. 28. (c) Four mangoes can be selected in 5 ways. This also includes the case when no mango is selected. Similarly, 5 oranges and 6 apples can be dealt in 6 and 7 ways, respectively. Therefore, the total number of ways of selecting a fruit = 210. However, this also includes the case when no fruit is selected.

1 L, 2 G 2 L, 1 G

Therefore, the total number of words which begin with `T’ and end with `E’ = 6! = 720. 30. (d) Five boys can be arranged in a line in 5 P5 = 5! ways. Now, girls are arranged in alternating places to boys. If a girl is placed before a boy then they can be arranged in 5! ways. Similarly, if a girl is placed after a boy then they can be arranged in 5! ways.

= 16 + 324 + 144 + 1 = 485

⇒ 2×

−(−9) ± (−9)2 − 456

29. (d) If `T’ and `E’ are fixed at the first and last spots, then there are 6 letters remaining which can be arranged in 6 P6 ways.

= 4 × 4 + 6 × 3 × 3 × 6 + 4 ×3 × 3 × 4 + 1 × 1

2n

⇒n=

from the

C1 × 4C2 + 4C1 × 3C2 × 3C2 × 4C3 + 3C3 × 3C3

⇒ 2 × 2n C2 = 2n C1 +

⇒ 2n2 − 9n + 7 = 0

Hence, at least one of the fruit is selected in (210 − 1) = 209 ways.

3

27. (b) We know that 2n C1 , 2n C2 and

⇒ 4n2 − 18n + 14 = 0

from the

Therefore, the required number of ways of inviting a dinner party = 4C3 × 4C3 + 4C2 × 3C1 ×

Chapter 09.indd 436

2n(2n − 1) (2n)(2n − 1)(2n − 2) = (2n) + 2 3! n(2n − 1)(2n − 2) ⇒ 4n2 − 2n = 2n + 3 ⇒ 2×

2n

C3 are in A.P.

C3

(2n)! (2n)! (2n)! = + (2n − 2)! 2 ! (2n − 1)! (2n − 3)! 3 !

Total ways of arranging girls = 5! + 5!. Total ways of arranging students = 5!(5! + 5!) = 2 ×(5 !)2 . 31. (a) Concept based.

22-08-2017 06:35:59

SOLVED GATE PREVIOUS YEARS’ QUESTIONS        437

SOLVED GATE PREVIOUS YEARS’ QUESTIONS 1. The minimum number of comparisons required to find the minimum and the maximum of 100 numbers is . (GATE 2014, 2 Marks) Solution:   Number of comparisons for finding 3 minimum and maximum of n numbers = n − 2 2 Here n = 100, so (300/2) − 2 = 148 Ans. 148 2. The number of distinct positive integral factors of 2014 is . (GATE 2014, 2 Marks) Solution:  2014 = 2 × 19 × 53 (product of prime factors) Number of distinct positive integral factors of 2014 = (1 + 1) × (1 + 1) × (1 + 1) = 8 Ans. 8 3. The number of divisors of 2100 is

.

(GATE 2015, 1 Mark) Solution:  Let N = 2100 = 22 + 3 × 52 × 7 (i.e. product of primes) Then the number of division of 2100 is given by (2 + 1)(1 + 1)(2 + 1)(1 + 1) = (3)(2)(3)(2) = 36

Four-digit numbers with first digit as 2: 2222, 2223, 2233, 2333, that is 4. Four-digit numbers with first digit 3: 3333, that is 1. Therefore, total number of four digit numbers = 10 + 4 + 1 = 15. Ans. 15

∑ i=0 i3 = X n

6. Consider the equality lowing choices for X: (i) q (n4)

(ii) q  (n+5) (iii) O(n5)

and the fol(iv) Ω(n3)

(a) Only (i) (b) Only (ii) (c) (i) or (iii) or (iv) but not (ii) (d) (ii) or (iii) or (iv) but not (i) (GATE 2015, 1 Mark) Solution:  X = sum of the cubes of first n natural n2 (n + 1)2 numbers = which is q (n 4 ), O(n5 ) and 4 Ω(n3 ). Ans. (c) 7. Consider the equation (43)x = (y3)8 where x and y are unknown. The number of possible solutions is . (GATE 2015, 1 Mark) Solution:   We have,

Ans. 36 (43)x = (y3)8 4. An unordered list contains n distinct elements. The number of comparisons to find an element in this list that is neither maximum nor minimum is (a) O(n log n) (b) O(n)

(c) O(log n) (d) O(1) (GATE 2015, 1 Mark)

Solution:  Consider first three element of the list, at least one of them will be neither minimum nor maximum. Therefore, the correct answer is option (d). Ans. (d) 5. The number of four-digit numbers having their digits in non-decreasing order (from left to right) constructed by using the digits belonging to the set {1, 2, 3} is . (GATE 2015, 1 Mark) Solution:  Four-digit numbers with first digit as 1: 1111, 1112. 1113, 1122, 1123, 1133, 1222, 1223, 1233, 1333, that is 10.

Chapter 09.indd 437

⇒ 3 + 4x = 3 + 8y ⇒ 4x = 8y ⇒ x = 2y ⇒ x ≥ 5 and y ≤ 7 Thus, five solutions are possible which are (14, 7), (12, 6), (10, 5), (8, 4) and (6, 3). Ans. 5 8. Let an be the number of n-bit strings that do NOT contain two consecutive 1s. Which one of the following is the recurrence relation for an? (a) an = an − 1 + 2an − 2

(b) an = an − 1 + an − 2

(c) an = 2an − 1 + an − 2

(d) an = 2an − 1 + 2an − 2 (GATE 2016, 1 Mark)

Solution:  Let us consider an be the number of nbit strings that do not contain two consecutive 1’s. Also, let us develop a recurrence relation for an. (i) Let us assume 1-bit strings 0, 1: That is, a1 = 2.

22-08-2017 06:36:00

438        Combinatory 

(ii) Let us also assume 2-bit strings 00, 01, 10, 11: In this case, 00, 01, 10 do not have two consecutive 1s. Therefore, a2 = 3.

Substituting r = 3 in the above equation, we get coefficient of x12 in (x3 + x4 + x5 + x6 + ⋅⋅⋅)3 as follows: 3+2

C3 = 5C3 = 5C2 = 10

(iii) Now, let us assume 2-bit strings as shown below:

Ans. 10 00 01 10

0 1 0 1 0 1

11. Consider the recurrence relation a1 = 8, an = 6n2 + 2n + an − 1. Let a99 = K × 104. The value of K is __________. (GATE 2016, 2 Marks) Solution:  The recurrence relation is given by

In this case, only the five strings 000, 001, 010, 100 and 101 do not have two consecutive 1s; therefore, a3 = 5. Therefore, the three numbers a1 = 2, a2 = 3 and a2 = 3 satisfy the following recurrence relation:

an − an − 1 = 6n2 + 2n(1) Let the complementary function be C1 and the particular solution be (An2 + BnC)n

an = an − 1 + an − 2 Ans. (b) 9. The spot speeds (expressed in km/hr) observed at a road section are 66, 62, 45, 79, 32, 51, 56, 60, 53, and 49. The median speed (expressed in km/hr) is     . (Note: Answer with one decimal accuracy.) (GATE 2016, 1 Mark)

On substituting in Eq. (1), we get A = 2; B = 4; C = 2. The solution is an = C1 + 2n3 + 4n2 + 2n It is given that a1 = 8; therefore, 8 = C1 + 8 ⇒ C1 = 0 Also, a99 = 2[(993) + 2(992) + 99]

Solution:  Writing spot speeds in ascending order is

= 2[(100 − 1)3 + 2(100 − 1)2 + (100 − 1)]

32, 45, 49, 51, 53, 56, 60, 62, 66, 78 (total 10 terms)

= 104(198)

Median will be given as (i) Middle term for odd no. of terms. (ii) Average of middle two terms for even no. of terms. Since even number of terms hence median will be 53 + 56 = 54.5 2 Ans. 54.5 10. The coefficient of x12 in (x3 + x4 + x5 + x6 + ⋅⋅⋅)3 is __________. (GATE 2016, 2 Marks) Solution:  We have (x3 + x4 + x5 + x6 + )3 = [x3 (1 + x1 + x2 + )]3 = x9 (1 + x + x2 + )3

That is, a99 = 104(198) ⇒ K = 198 Ans. 198 12. The value of the expression 1399 (mod 17), in the range 0 to 16, is __________. (GATE 2016, 2 Marks) Solution:  By Fermat’s theorem; ap − 1 = d (mod p). It is a fact that 17 is a prime number and it is not a divisor of 13. Therefore, 1316 = 1 (mod p) 1399 = (13)96 × (13)3 = (1316)6 × 2197 = 16 × 2197 (mod 17) By modulo arithmetic, we get

x9 = (1 − x)3

(16 × 2197) mod 17 = 4



= x9 ∑ (3 − 1) + rCr xr

Ans. 4

r =0

Chapter 09.indd 438

22-08-2017 06:36:01

CHAPTER 10

GRAPH THEORY

v1

INTRODUCTION

e3 v5

Graph theory is the study of graphs, which are mathematical structures used to model pairwise relations between objects. A graph may be undirected, i.e., there is no distinction between the two vertices associated with each edge or its edges may be directed from one vertex to another. Graphs are used in many practical problems and can be used to model many types of relations and processes in physical, biological and information systems.

FUNDAMENTAL CONCEPTS OF GRAPH A graph is formed by vertices or nodes which are connected by edges. Mathematically, a graph is a pair of sets given by G = [V (G), E(G)], where V(G) is a finite set of vertices and E(G) is the set of edges formed by pairs of vertices. E(G) is a multi-set, i.e., its elements can occur more than once so that every element has a multiplicity.

Chapter 10.indd 439

e1 e2

e5

e4 v2

v4 v3

Figure 1 |   Graph showing vertices and edges.

In Fig. 1, we have V (G) = {v1 , v2 , v3 , v4 , v5 } for the vertices and E(G) = { (v1 , v2 ),(v2 , v5 ),(v5 , v5 ),(v5 , v4 ),(v5 , v4 )} for the edges.

Common Terminologies Two vertices joined by an edge are called the end vertices of that edge. In Fig. 1, v1 and v2 are the end vertices of e1. Similarly, v2 and v5 are the end vertices of e2 . If two or more edges have the same end vertices, then these edges are called parallel. In Fig. 1, e4 and e5 are

22-08-2017 06:38:00

440     Graph Theory 

parallel edges because they have the same end vertices, v4 and v5. An edge joining a vertex to itself is called a loop. A loop has only one end vertex. In Fig. 1, e3 is a loop with end vertex v5. A graph with no edges is called an empty graph. For an empty graph, E(G) is empty.

Multigraph A multigraph (also called pseudo graphs) also consists of sets V(G) and E(G). However, it may consist of parallel edges and may consist of one or more loops. An example of multigraph is shown in Fig. 3. A multigraph is finite if the sets V(G) and E(G) are finite.

A graph with no vertices is called a null graph. For a null graph, V(G) and E(G) are empty.

v3

Edges are adjacent if they share a common end vertex. Two vertices are adjacent if they are connected by an edge. In Fig. 1, e1 and e2 are adjacent edges and v1 and v2 are adjacent vertices.

v1

v4 v5

v2 A graph which has no parallel edges or loops is called a simple graph. Figure 2 depicts a simple graph.

v3

e2 v1

WALKS, PATHS AND CONNECTIVITY A walk in the graph G = {V (G) , E (G)} consists of alternating vertices and edges of G. A walk is of the form,

e3

e1 e4 v2

Figure 3 |   Multigraph.

v5 vi0 , ej1, vi1,ej2 ,..., ejk , vik

v4

Figure 2 |   Simple graph.

A vertex which is not the end of any edge is called an isolated vertex. In Fig. 1, v3 is the isolated vertex.

Degree of a Vertex Degree of a vertex in a graph is equal to the number of edges which are incident on that vertex or the number of edges which contain that vertex as an endpoint. It is represented as deg(v) or d(v). The vertex is said to be even or odd according to the value of d(x). A vertex whose degree is 1 is called a pendant vertex. Evidently, an edge that has a pendant vertex as an end vertex is a pendant vertex.

A walk starts and ends at a vertex. vi0 is the initial vertex and vik is the final vertex. A walk is open if vi0 ≠ vik and is closed if vi0 = vik . A walk in which any edge is traversed at most once is called a trail. A trail is a path in which any vertex is visited at most once except possibly the initial and terminal vertices when they are the same (i.e., the path is closed). A  closed path is also called a circuit. The number of edges traversed in a path is called the length of a path. A closed path whose vertices are distinct except vi0 = vik is called a cycle. A cycle of length n is called n -cycle.

v2

e1

v1

e2

Also, an isolated vertex has a degree 0.

e7 In Fig. 1, v2 has a degree of 2 because it is the endpoint of two edges, e1 and e2 . Theorem 1. The graph G = {V (G), E(G)}, where V(G) = {v1 , v2 ,..., vn } and E(G) = {e1 , e2 ,..., em }, satisfies n

∑ d (vi ) = 2m i =1

Hence, it can be noted that every graph has an even number of vertices of odd degree.

Chapter 10.indd 440

e6

v3

e8 e3 v5 e5 v4 e4 Figure 4 |   Example of connectivity of graph.

22-08-2017 06:38:03

TYPES OF GRAPHS     441

In Fig. 4, walk v2 , e6 , v5 , e5 , v4 , e4 , v4 , e3 , v3

Theorem 4. If G is a connected graph and k ≥ 2 is the maximum path length, then any two paths in G with length k share at least one common vertex.

is open, and walk v2 , e6 , v5 , e8 , v1, e1, v2

TYPES OF GRAPHS

is closed.

Complete Graph

In Fig. 4, walk v1, e7 , v5 , e8 , v1, e1, v2 , e6 , v5 is a trial. In Fig. 4, walk

A graph in which every vertex is connected to all the other vertices of the graph is called a complete graph. A complete graph of n vertices is given by Kn . Figure 5 gives an example of complete graphs.

v1 , e1 , v2 , e2 , v3 , e3 , v4 , e5 , v5 is a path, and walk v1, e1, v2 , e6 , v5 , e8 , v1 is a closed path or circuit. A graph is connected if all the vertices are connected to each other.

K3

K4

(a)

SUBGRAPH A subgraph of a graph G is a graph whose V(G) is a subset of G and whose adjacency relation is a subset of that of G restricted to this subset.

Figure 5 |   Complete graphs K3 and K4 with v ­ ertices 3 and 4, respectively.

Number of edges, m, in a complete graph Kn with n vertices is given by n (n − 1)

If from a graph G with vertex set V(G), a subset U(G) of V(G) is deleted and all the edges have a vertex in U(G) as an endpoint, then G-U(G) is called vertexdeleted subgraph. If a subset U(G) of E(G) is deleted from G, then G-U(G) denotes the subgraph of G with vertex set V(G) and edge set E(G)-U(G), then G-U(G) is called edge-deleted subgraph.

(b)

m= 2

Regular Graph A regular graph is a graph whose every vertex has the same degree. Figure 6 shows example of regular graphs.

The subgraph G1 of a graph G is a component of G if 1. G1 is connected. 2. Either G1 is trivial or G1 is not trivial, and G1 is the subgraph induced by those edges of G that have one end vertex in G1. Theorem 1. If graph G has a vertex v that is connected to a vertex of the component G1 of G, then v is also a vertex of G1. Theorem 2. Every vertex or edge of G belongs to exactly one component of G. Theorem 3. A graph is circuit less when there is at most one path between any two given vertices and there are no loops.

Chapter 10.indd 441

(a)

(b)

Figure 6 |   (a) 0-regular graph and (b) 1-regular graph.

Bipartite Graph A simple graph G is a bipartite graph if a vertex set V(G) can be partitioned into two non-empty disjoint subsets V1 and V2 such that every edge connects a vertex V1 and a vertex V2 . The partition V = V1 ∪ V2 is called a bipartition of G. Figure 7 shows an example of bipartite graph.

22-08-2017 06:38:07

442     Graph Theory 

A bipartite graph is complete if there is an edge from every vertex in v1 to every vertex in v2, denoted by Km,n where m = | v1| and n = |v2 |. Figure 8 shows an example of complete bipartite graph.

Figure 10 |   Forest. Figure 7 |   Bipartite graph.

A spanning tree of a connected graph is a sub-tree that includes all the vertices of the graph. If T  is a spanning tree of graph G, the G-T gives cospanning tree. The edges of the spanning tree are called branches and the edges of the ­corresponding cospanning tree are called links or chords. Figure 11 shows examples of spanning and cospanning trees.

Figure 8 |   Complete bipartite graph.

The diameter of k1,1 will be one because there are only two vertices and the shortest path between them is the length one. All other bipartite graphs have diameter two because any two points in either m or n will be exactly distance 2 units apart.

(a)

(b)

Tree Graph A tree is a connected acyclic simple graph. A forest is a circuit less graph. A tree is simply a connected forest. The drawing of tree becomes complicated as the number of vertices increase. Figures 9 and 10 show examples of tree graphs.

(c) Figure 11 |   (a) Graph, G, (b) spanning tree of G and (c) cospanning tree.

Theorem 1. If T is a tree with at least two vertices and the longest path in T is given by u0 , u1,..., un , then both u0 and u1 have degree 1. 1 Vertex

2 Vertices

In other words, any tree T with at least two vertices has more than one vertex of degree 1. Theorem 2. If a tree has n vertices, then it has precisely (n - 1) edges. Theorem 3. A connected graph has at least one spanning tree.

3 Vertices

4 Vertices

Figure 9 |   Tree graph.

Chapter 10.indd 442

Trivial Graph A graph with no edges is called a trivial graph.

22-08-2017 06:38:09

MATRIX REPRESENTATION OF GRAPHS     443

CYCLE A non-trivial closed trail in a graph with its origin and internal vertices distinct is called a cycle. A cycle graph with n vertices is denoted by Cn. Every vertex of a cycle has degree 2. The closed trail C = v1, v2 ,..., vn and v1 is cycle, if C has at least one edge and v1, v2 ,..., vn are n distinct vertices. A cycle with k edges is called k-cycle. A k-cycle is even or odd depending on whether k is odd or even. Two cycles with same length are isomorphic. Figure 12 shows a K-cycle of length 4. v1

4. Product of graphs: To define product G1 × G2 of two graphs, consider any two points u = (u1, u2 ) and v = (v1, v2 ) in V = V1 × V2, then u and v are adjacent in G1 × G2 whenever 1. u1 = v1 and u2 is adjacent to v2 or 2. u2 = v2 and u1 is adjacent to v1. 5. Composition of graphs: Suppose we have two graphs G1 = (V1, E1 ) and G2 = (V2 , E2 ). The composition G = G1 G2  is the graph with point vertex V1 × V2 and u = (u1, u2 ) adjacent with v = (v1, v2 ) whenever 1. u1 is adjacent to v1 or 2. u1 = v1 and u2 is adjacent to v2. Figure 14 illustrates composition of graphs.

e4

e1

u2 u2 v2

v2

w2

v4 e2

v1 G1

e3

G2 (u u ) 2, 1

(u2 , v1)

(v2 , u1)

(v2 , v1)

(w2 , u1)

(w2 , v1)

v3

Figure 12 |   Cycle of length 4.

OPERATIONS ON GRAPHS

(v1 , u2) (v1 , v2) (v1 , w2) (b)

(a)

Operations on graphs are performed to obtain new graphs. Some of the commonly used operations are: 1. Union: Suppose we have two graphs G1 = (V1, E1 ) and G2 = (V2 , E2 ). The union will be G1 ∪ G2 = (V1 ∪ V2 , E1 ∪ E2 ). If V1 and V2 are disjoint, their union is referred to as the disjoint union, and denoted by G1 + G2 . 2. Intersection: Suppose we have two graphs G1 = (V1, E1 ) and G2 = (V2 , E2 ). The intersection will be G1 ∩ G2 = (V1 ∩ V2 , E1 ∩ E2 ). 3. Complement: The complement or inverse of a graph G1 is a graph G2 on the same vertices such that two distinct vertices of G2 are adjacent if and only if they are not adjacent in G1. Figure 13 illustrates complement of a graph.

Figure 14 |   (a) Composition G1 G2  and (b) composition G2 G1 .

MATRIX REPRESENTATION OF GRAPHS In this section, we discuss the two methods of matrix representation of a graph, namely, the adjacency matrix and the incidence matrix of the graph.

Adjacent Matrix Let G be a graph with vertex set V(G) = {v1 , v2 ,..., vn }. The adjacency matrix of G is given by A(G) = (aij ), where (aij ) = [EG (vi , vj )] = number of edges joining the vertex vi to vj . If G has no loops, then all entries of the main diagonal will be 0 and if G has no parallel edges, then entries of A(G) are either 0 or 1.

G1

G2

Figure 13 |   Complement of G1 given by G2 .

Chapter 10.indd 443

Theorem 1. Let A be the adjacency matrix of a graph G with vertex set V (G) = {v1 , v2 ,..., vn }. Then the entry in position (i, j) of Ak is the number of different (vi − vj ) walks in G of length k.

22-08-2017 06:38:16

444     Graph Theory 

Theorem 2. Let A be the adjacency matrix of a graph G with vertex set V (G) = {v1 , v2 ,..., vn } and B = (bij ) be the matrix. Hence,

Theorem 1. Let e be a bridge of graph G, then w(G) ≤ w(G − e) ≤ w(G) + 1

B = A + A2 +  + An−1

where w(G) is the number of connected components of G.

Then G is a connected graph if for every pair of distinct indices i and j, we have bij = / 0 , i.e., b has no zero entries of the main diagonal.

Theorem 2. An edge e of a graph G is a bridge if it is not a part of any cycle in G.

The results from Theorem 1 and Theorem 2 are used to check whether a graph is connected or not.

Theorem 3. Let G be a graph with n vertices and m edges, then m ≥ n − w(G)

Incidence Matrix

where w(G) is the number of connected components of G. Let G be a graph with n vertices, V (G) = {v1 , v2 ,..., vn }, and t edges, E(G) = {e1 , e2 ,..., et }. The incidence matrix M (G) is the n × t matrix given by M (G) = (mij ) where mij is number of times the vertex vi is incident with the edge ej. Hence,



 0, if vi is not an end of ej  mij =  1, if vi is an end of non--loop ej   2, if v is an end of loop e i j 

The degree of vi is given by the sum of elements in the ith row of M (G).

Theorem 4. The vertex v is a cut vertex of the connected graph G if and only if there exists two vertices u and w in G, such that (a) v = / u, v = / w and u = / w, but (b) v is on every u − w path. Theorem 5. A non-trivial simple graph has at least two vertices which are not cut vertices.

SPANNING TREES AND ALGORITHMS As already mentioned, a spanning tree of a connected graph G is a sub-tree that includes all the vertices of that graph. A complete tree Kn has n n−2 different spanning trees.

CUTS An edge e is called a cut edge of the graph G if by removing e, the connected component of the graph either remains unchanged or it increases exactly by 1. Cut edge is also called a bridge. Similarly, a vertex v of a graph G is a cut vertex or an articulation vertex of G if the graph (G − V ) consists of a greater number of components than G. A graph is separable if it is not connected or if there exists at least one cut vertex in the graph. Otherwise, graph is non-separable. Figure 15 shows cut-edges.

A graph in which each edge is assigned a weight, w(e), is called a weighted graph. Suppose H is a subgraph of a weighted graph G and has edge set E(H ) = {e1 , e2 ,..., ek }, then w(H ) = w(e1 ) + w(e2 ) +  + w(ek ) A minimal spanning tree (MST) is a spanning tree with weight less than or equal to the weight of every other spanning tree. In the sections that follow, we have discussed two algorithms for finding MST. It is to be noted that a necessary condition for these algorithms is that no weights should be negative.

v e

Kruskal’s Algorithm (c)

(a)

(b)

Figure 15 |   Graphs: (a) G, (b) G − e and (c) G − c.

Chapter 10.indd 444

It is a greedy algorithm in graph theory that finds a minimum spanning tree for a connected weighted graph. The algorithm gives an acyclic subgraph of G and it is a MST of G.

22-08-2017 06:38:19

HAMILTONIAN GRAPHS     445

The following steps are involved in Kruskal’s algorithm: 1. Select an edge of G, say e1 such that w(e1 ) is as small as possible and e1 is not a loop. 2. If edges e1, e2 ,..., ei have been chosen, then choose an edge ei+1 that is not already chosen such that (a) Induced subgraph G[e1 , e2 ,..., ei +1 ] is acyclic. (b)  w(ei +1 ) is as small as possible. 3. If G has n vertices, then stop after (n − 1) edges have been chosen, or else repeat step 2.

EULER TOURS A tour of any graph is a closed walk of that graph in which every edge is encountered at least once. A Euler tour of any graph is a closed walk of that graph in which every edge is encountered exactly once. Any graph G is called Eulerian or Euler if it has a Euler tour. The necessary conditions for the existence of Eulerian circuits are that all vertices in the graph have an even degree.

Prim’s Algorithm It is a greedy algorithm that finds a minimum spanning tree for a connected weighted undirected graph. It finds a subset of the edges that forms a tree that includes every vertex, where the total weight of all the edges in the tree is minimized. The following steps are involved in Prim’s algorithm: 1. Choose any vertex of G, say v1. / v1 2. Choose an edge e1 = v1v2 of G such that v2 = and e1 has the smallest weight among the edges of G incident with v1. 3. If edges e1, e2 ,..., ei have been chosen involving endpoints v1, v2 ,..., vi+1, choose an edge ei+1 = vj vk with vj ∈ {vi ,..., vi +1} and vk ∉ {v1 ,..., vi +1} such that ei+1 has the smallest weight among the edges of G with precisely one end in  v1,..., vi+1  . 4. Stop after n − 1 edges have been chosen. Otherwise repeat step 3.

Theorem 1. A connected graph G has a Euler trail if it has at most two odd vertices, i.e., it has either no vertices of odd degree or exactly two vertices of odd degree.

Königsberg Bridge Problem .. The Konigsberg bridge problem asks if the seven .. bridges of the city of Konigsberg can all be traversed in a single trip without doubling back, with the additional requirement that the trip ends in the same place it began. For the existence of Eulerian trails, it is necessary that zero or two vertices have an odd degree. This means .. the Konigsberg graph is not Eulerian. Therefore, a solution does not exist. Figure 17 shows Euler’s graphical .. representation of the Konigsberg bridge problem.

BINARY TREES A binary tree is a rooted tree that is also an ordered tree, in which every node has at most two children, which are referred to as the left child and the right child. A full binary tree or a perfect tree is a tree in which every node other than the leaves has two children. Figure 16 shows an example of a full binary tree. Binary trees are extensively used in data representation.

Figure 17 |   Euler’s graphical representation of the .. Konigsberg bridge problem.

a

HAMILTONIAN GRAPHS c

b

d

e

f

Figure 16 |   Full binary tree.

Chapter 10.indd 445

g

Hamiltonian path is a path in an undirected or directed graph that visits each vertex exactly once. Such a part is a cycle and is called Hamiltonian cycle. As already mentioned before, a Eulerian cycle uses every edge exactly once but may repeat vertices, whereas

22-08-2017 06:38:22

446     Graph Theory 

GRAPH ISOMORPHISM

a Hamiltonian cycle uses each vertex exactly once (except for the first and the last) but may skip edges.

A simple graph G is called maximal non-Hamiltonian Suppose we have two graphs G1 = {v1 , E1} and G2 = {v2 , E2 } if it is not Hamiltonian but addition of any edge connectG2 = {v2 , E2 }. G1 and G 2 are said to be isomorphic if ing two non-adjacent vertices forms a Hamiltonian graph. Figures 18 (a) and (b), respectively, show Hamiltonian 1. there is a bijection (or one-to-one correspondence) and Eulerian graphs. f from v1 to v2 and 2. there is a bijection g from E1 to E2 that maps each edge (v, u) to (f (v), f (u)). Figures 20 (a) and (b), respectively, show non-isomorphic and isomorphic graphs. a (a)

a

(b)

Figure 18 |   (a) A graph which is Hamiltonian and nonEulerian and (b) a graph which is Eulerian and non-Hamilton.

b

Theorem 1. If G is a simple graph with n vertices n where n ≥ 3, and the degree d(v) ≥ for every vertex 2 v of G, then G is Hamiltonian.

b

e

c

d

e

c

d

(a) A A

D

B

C

CLOSURE OF A GRAPH Suppose G is a simple graph. If we have two ­non-adjacent vertices u1 and v1 in G such that d(u1 ) + d(v1 ) ≥ n in G1, join u1 and v1 by an edge to form the super graph G1. Similarly, if we have two non-adjacent vertices u2 and v2 in G1 such that d(u2 ) + d(v2 ) ≥ n, join u2 and v2 by an edge to form the super graph G2 .

D C

B (b)

Figure 20 |   (a) Non-isomorphic graphs and (b) isomorphic graphs.

Continue these steps until we get the last pair of nonadjacent vertices whose degree sum is at least n. The final super graph obtained is called closure of G or C(G). It should be noted that a simple graph is Hamiltonian if its closure C(G) is Hamiltonian. Figure 19 shows closure of graph.

HOMEOMORPHIC GRAPHS Suppose we have a graph G, we can get a new graph by dividing an edge of G with additional vertices. This is called homeomorphism and the two graphs G and H are called homeomorphic graphs. Two graphs G and H are homeomorphic if they can be made isomorphic by inserting new vertices of degree 2 into their existing edges (Fig. 21).

(a)

(b)

Figure 19 |   (a) Simple graph, G, and (b) closure of graph, C(G).

Chapter 10.indd 446

G

H

Figure 21 |   Homeomorphic graphs G and H.

22-08-2017 06:38:26

GRAPH COLORING     447

PLANAR GRAPHS

COVERING

A planar graph is a graph that can be embedded in a plane, i.e., it can be drawn on a plane in a way that its edges intersect only at their endpoints and no edges cross each other. Figures 22 (a) and (b), respectively, show examples of non-planar and planar graph.

A set of edges E(G) of a graph G is set to cover G if every vertex in G is incident on at least one edge in E(G). A set of edges that covers a graph G is called edge covering.

A

A

D

D

Covering exists for a graph if and only if the graph has no isolated vertex. n Covering of an n-vertex graph will have at least    2  edges, where [x] denotes the smallest integer not less than x. If we denote remaining edges of a graph by (G − E(G)), the set of edges E(G) is a covering if and only if, for every vertex v,

E

degree of vertex in (G − E (G)) ≤ (degree of vertex v in G) B

B

C (a)

C (b)

Figure 22 |   (a) Non-planar graph and (b) planar graph.

MATCHING

A minimal covering for an n-vertex graph can contain no more than n-edges. The number of edges in a minimal covering of the smallest size is called covering number of the graph. Figure 23 shows a graph G and its covering C(G). A

In graph theory, matching is a set of edges without common vertices. Formally, a matching M in G is a set of pairwise non-adjacent edges, i.e., no two edges share a common vertex. In other words, a vertex is matched if it is an endpoint of one of the edges in the matching or else the vertex is unmatched. A maximal matching M of a graph is a matching with the property that if any edge which is not in M is added to M, it is no longer a matching. In other words, M is maximal if it is not a proper subset of any other matching in graph G. The number of edges in a largest maximum matching is called matching number of the graph. A matching which matches as the vertices of the graph is called a perfect matching. In other words, every vertex of the graph is incident to exactly one edge of the matching. Theorem 1. A complete matching of v1 and v2 in a bipartite graph exists, if and only if every subset of v vertices in v1 is collectively adjacent to one or more vertices in v2 for all values of v. Theorem 2. In a bipartite graph, if a positive integer n exists such that degree of every vertex in v1 ≥ 2 ≥ degree of every vertex in v2 , then a complete matching v1 into v2 exists.

Chapter 10.indd 447

D

C G

D

E

E

B

A

A

B

C

B

D

E

C C(G)

Figure 23 |   Graph G and its covering C(G).

INDEPENDENT SET A set of vertices in a graph in which no two sets are adjacent is called an independent set. In other words, it is a set of vertices such that there is no edge connecting any two vertices of that set. The size of an independent set is the number of vertices it contains. An independent set of the largest possible size of a graph G is called a maximum independent set and this size is called independence number of G, which is denoted by α(G).

GRAPH COLORING The assignment of colors to graph elements subject to certain conditions or constraints is called graph coloring. Graph coloring is a special case of graph labeling.

22-08-2017 06:38:28

448     Graph Theory 

Coloring the graph vertices such that no two adjacent vertices have the same color is called vertex coloring. Coloring the graph edges such that no two adjacent edges have the same color is called edge coloring.

Coloring the graph faces such that no two faces that share a boundary have the same color is called face coloring.

SOLVED EXAMPLES 1. Find the number of edges in the graphs: (a) K8 and (b) K14 .

3. Find the complement of the graph given below: 1

Solution:  The number of edges in a complete graph is given by n(n − 1) m= , where n = number of vertices 2 8(7) (a) For K8, m = = 28 2 14(13) (b) For K14 , m = = 91 2

2

3

4

5

2. Explain whether the following graphs are connected or not. A

B

B

E

A Solution:  We have to keep in mind that the two vertices are adjacent in the complement only when they are not originally adjacent.

D C

1 C

D (b)

(a)

2 A

A

3

B

E B

D

C (c)

C

D

5

4

(d) 4. A graph G has the following adjacency matrix. Find whether it is connected.

Solution:  (a) Yes, it is a connected graph because all the vertices are connected to each other. (b) No, it is not a connected graph. A, B and D are connected, and E and C are connected. However, there is no connectivity between A, B and D, and C and E. (c) No, it is not a connected graph because E is not connected to any other vertex. (d) Yes, it is connected because all the vertices are connected to all other vertices of the graph.

Chapter 10.indd 448

0  0 A (G) =  0 0  0 

0 0 0 1 0

1 0 0 0 1

0 1 0 0 1

0  0 1 1 0

Solution:  We have n=5 Hence, B = A + A2 + A3 + A4

22-08-2017 06:38:30

SOLVED EXAMPLES     449

2  1 =  0 0  3 

We find A2 , A3 and A4 using matrix multiplication, 0  0 2 A =  0 0  0 

0 0 0 1 0

1 0 0 0 1

0 1 0 0 1

0  0 1  1 0 1  1 0 1 1

0  0 0  0  0 

1  0 =  0 0  1 

0 1 0 0 1

0 0 2 1 0

0 0 1 2 0

1  0 A3 =  0 0  1 

0 1 0 0 1

0 0 2 1 0

0 0 1 2 0

0  0 0  0  0 

1 2 0 0 3

1  1 0 1 1 0  0 3 3 0

0  0 =  2 1  0 

0 0 1 2 0

2 1 0 0 3

0  0 4 A =  2 1  0 

0 0 1 2 0

2 1 0 0 3

1 2 0 0 3

0  0 3 3 0

0  0 0  0  0 

0 0 0 1 0

1 0 0 0 1

0  0 1 1 0  

0 1 0 0 1

1 2 0 0 3

0 0 5 4 0

3  3 0 0 6

0 0 4 5 0

Now, B = A + A2 + A3 + A 4

0 0 0 1 0

1 0 0 0 1

0 0 0 1 0

0  0 1 1 0

0 1 0 0 1

1 0 0 0 1

0  0 1 1 0

0 1 0 0 1

0  0 =  0 0  0 

0 0 0 1 0

1 0 0 0 1

0 1 0 0 1

0  1   0  0 1 +  0 1  0 0  1

0 1 0 0 1

0 0 2 1 0

0 0 1 2 0

0  0 +  2 1  0 

0 0 1 2 0

2 1 0 0 3

1 2 0 0 3

0  2   0  1 3 +  0 3  0 0  3

1 2 0 0 3

0 0 5 4 0

0 0 4 5 0

3  1 =  3 1  4 

1 3 1 3 4

3 1 7 5 4

1 3 5 7 4

4  4 4 4 8

1  1 0 1 1 3  3 0 0 6

As B has no more zero entry off the main diagonal, the graph is connected.

5. Apply Kruskal’s algorithm to determine a minimal spanning tree of the graph G with 5 vertices. v1 3 4 3

v5

3

3 1

v2

2

2 v4

3

v3

Solution:  Using step 1, choose e = v2v4 as it has minimum weight. v1 v1

3 4 3

v5

3

v2

v5

3 Choose e = v4 v5

2 2 v4

3 3

3

v2

3 1

2

v3

4 3

v4

2 v3

3

v1 v5

v1

4

3 3

3

v5

3

Choose e = v2 v3

3

Choose e = v1 v4

2 Chapter 10.indd 449

1

2 2

v4

v

4

3

v2

3 3 1

v2 2

22-08-2017 06:38:32

v1 v1

3 4 3

v5

3

v2

3 3

v5

3 Choose e = v4 v5

2 2

4 3

v2

3 1

2

2

450     Graph Theory 

v4

v3

3

v4

v3

3

v1 v1

4

3 3

v5

3

v5

3

Choose e = v2 v3

4

3

v2

3

3

Choose e = v1 v4

2

1

2 2

v4

v2

3 1

2

v3

3

v4

3

v3

As vertices are 5, we require 4 edges. Minimal spanning tree of graph is given by v5

v1

v2

3 2 1 v4

2

v3

6. Apply Prim’s algorithm to determine a minimal spanning tree of the graph G in the previous example.

Again, w(v2 , v5 ) = 3, w(v2 , v4 ) = 1 and w(v3 , v4 ) = 3 We choose the minimum, e = (v2 , v4 ) v1

Solution:  Using step 1, we choose vertex v1. Now, edge with the smallest weight incident on v1 is e = v1v3 .

4

3 3

v5

3

v2

3

v1 1

2

3

4

2

3 3 v5

v4

v2

3 2

v3

3

Now, w(v4 , v5 ) = 2 , we choose e = v4 , v5

2 1

v1

v4

v3

3

3

4

Now, we look on the weights, 3

3

v5

w(v1 , v2 ) = 4, w(v1 , v4 ) = 3, w(v1 , v5 ) = 3,

v2

3

w(v3 , v2 ) = 2, w(v3 , v4 ) = 3

2

Now, we choose the minimum, e = (v3 , v2 )

1 v4

2

v3

3

v1 The minimal spanning tree is shown below:

4

3

v1

3

3 v5

v2

3 v5

3 2

1

v2

2

1

2

2 v3

v4

Chapter 10.indd 450

3

v3

v4

22-08-2017 06:38:35

SOLVED EXAMPLES     451

7. Suppose G (V , E ) has 5 vertices. Find the maximum number of m of edges in E if G is a multigraph.

11. Prove that in finite graphs, the number of vertices of odd degree is always even. Solution:  Consider the following two graphs with their degree sequence.

Solution:  As multiple edges are allured, G can have any number of edges and loops, finite or infinite. Hence, no such maximum number m exists. 8. Draw the graph for the following vertex and edge set: V (G) = {A, B, C , D} and E (G) = {{A, B} , {C , A} , {C , D} , {B, C }} E (G) = {{A, B}      , {C , A} , {C , D} , {B, C }} Solution: A

B Let Vodd be the set of all vertices of odd degree and Veven be the set of all vertices of even degree. Now, the sum of all degrees is twice the number of edges. Therefore, we have

C

∑ d (v) = 2e ⇒ ∑ d (v ) = ∑

D

v =V odd

9. Show that every simple finite graph has two vertices of the same degree.





d (v ) = even integer

v =V even

d (v ) + (even integer) = even integer

v =V odd

Solution:  Let us assume that the graph has n ⇒ ∑ d (v ) = even integer vertices. Each of these vertices is connected to v =V odd either of the other vertices, 0, 1, 2,....., n − 1. If any of the vertices is connected to n − 1 vertices, then This is true only when Vodd has even number of it is connected to all the others, so there cannot be elements. Hence, G has even number of odd degree a vertex connected to 0 others. Thus, it is imposvertices. sible to have a graph with n vertices where one 12. Let R be a binary relation on A = {a, b, c, d, e, f , g, h} vertex has degree 0 and another has degree n − 1. A = a , b , c , d, e, f , g, h} represented by the following two com{ Thus, the vertices can have at most n − 1 different ponents. Find the smallest integers m and n such degrees, but since there are n vertices, at least two that m < n and R m = R n . must have the same degree. 10. Find the composition G1 G2 , given that

a1

b1

a2

G1

b2 G2

c

b

c2 d

f

Solution: (a1 , a2)

(a1 , b2)

(a1 , c2)

a

c h

g

Solution:  Here, R = {(a, b ) , (b, c ) , (c, a) , (d, e) , (e, f ) , (f , g ) , (g, h) , (h, d )}

G1 [G2]

Say R1 = {(a, b ) , (b, c ) , (c, a)} and (b1 , a2)

Chapter 10.indd 451

(b1 , b2)

(b1 , c2)

R2 = {(d, e) , (e, f ) , (f , g ) , (g, h) , (h, d )} ∴ R = R1 ∪ R2

22-08-2017 06:38:41

452     Graph Theory 

From the digraph, R =

R

=

R

=

Solution: 

R



and R = R = R ⇒ R = R116 ∪ R216 =

=

R

= R 16

=

R

=

=

(b) Cardinality of minimum edge cut in this graph is 2. The minimum edge cut is the set {(B, C), (C, F )}.

R

⇒ m = 1 and n = 16 13. Let G be a connected, undirected graph. A cut in G is a set of edges whose removal results in G being broken into two or more components which are not connected with each other. The size of a cut is called its cardinality. A min-cut of G is a cut in G of minimum cardinality. Consider the following graph. A

B

(a) S  et (ii) is a cut because on deleting BD, CF and AB, the graph gets disconnected.

As there are n vertices and minimum degree of any vertex is k, and we know that each edge is incident  nk  on two vertices, G has at least   edges.  2 

E

14. Let S be a set of n elements {1, 2, …, n} and G a graph with 2n vertices, where each vertex corresponds to a distinct subset of S. Two vertices are adjacent if the symmetric difference of the corresponding sets has exactly 2 elements.

F

(a) E  very vertex in G has the same degree. What is the degree of a vertex in G?

D

C

(c) If degree < k, then by deleting all the edges incident on this vertex, we would be able to disconnect the graph. Hence, the degree of each vertex on this graph is at least k.

(a) Which one of the following sets of edges is a cut? (i) {(A, B), (E, F), (B, D), (A, E), (A, D)}

(b) How many connected components does G have?

(ii) {(B, D), (C, F), (A, B)}

Solution: 

(b) What is the cardinality of min-cut in this graph?

n(n − 1) A2 D A D (b) G has twoE connected components. (a) E A D (a) E B C (a) B C (a)

(c) Prove that if a connected undirected graph G with n vertices has a min-cut of cardinality K,  nk  then G has at least   edges.  2 

B A A

C D D

(b) (b)

A

D

(b)

B B

C C

B A A

BC B

PRACTICE EXERCISE 1. Find the number of edges in the graph K6 . (a) 12

(b) 15

(c) 16

(d) 30

2. Find the number of edges in the graph K11. (a) 110

(b) 40

(c) 55

(d) 30

(c) (c)

3. Which one of the following graphs is not connected? A

B A

(c)

D

A A

E

E C C

D D

C

D C C

E E

C

E

(d) (d) A

(a) B

C

A

D

B

C

(d)

E E

B B

D D

B

D

(b)

Chapter 10.indd 452

B A

E

22-08-2017 06:38:43

PRACTICE EXERCISE     453

4. Which one of the following options is the complement of the following graph? b

a

(b) V (G) = {A, B, C , D, E } and E (G) = {{A, B} , {C , A} , {C , D} , {B, E } , {D, E }} (c) V (G) = {A, B, C , D, E } and E (G) = {{A, B} , {C , A} , {D, C } , {B, D} , {D, E }}

d

c

(d) V (G) = {A, B, C , E } and E (G) = {{A, B} , {C , A} , {C , D} , {B, D} , {D, E }}

f

e b

a

6. Consider the adjacency matrix of the following graphs. Find which one of the following options is connected.

d c

(a)

f

e

b

a

d c

(b)

f

e

b

a

d c

(c)

f e

0  1 (a) 0 1  1

1 0 0 0 1

0 0 0 1 1

1 0 1 0 0

0 1 1   1 1 0  1   (b) 0 0  1 0 0   0 0 0

0 0 0 1 0

1 0 1 0 0

0  0  0  0  0

0  1 (c) 0 0  0

1 0 1 0 0

0 1 0 1 0

0 0 1 0 0

1 0 0   0 0 1  0   (d) 0 0  0 0 0   0 0 1

0 0 1 0 0

0 0 0 1 0

0  0  0  0  1

7. The number of distinct simple graphs with up to five nodes is

b a d

(a) 15   (b) 10 (c) 31          (d) 9

(d) c f

8. Find the value of G2 G1  if G1 and G2 are given as follows:

e 5. How can you represent the following graph?

a1 A

B a2

b2

c2

b1 C

D G1

(a2, a1)

G2

(a2, b1)

E (a) V (G) = {A, B, C , D} and E (G) = {{A, B} , {C , A} , {C , D} , {B, D}}

(a) (b2, a1)

(a2, a1)

Chapter 10.indd 453

(b) (b , a )

(b2, b1)

(a2, b1)

22-08-2017 06:38:46

(a2, b1)

(a2, a1)

(c) (a) a1) (b2,Theory  454     Graph

(a2, a1)

(b2, b1)

d

c

(a2, b1)

a

b

c

d

e

f

(d) (b) (b2, a1)

(b2, b1)

10. Find the degree of vertex of E. (c2, b1)

(c2, a1)

B

A E

(a1, a2)

(a1, b2)

(a1, c2)

F

C

(c)

(b1, a2)

(b1, b2)

(b1, c2)

(d) None of these

b

(a) 1

(b) 2

(c) 3

(d) 4

11. The minimum number of edges in a connected cyclic graph on n vertices is

9. Find the value of G1 ∪ G2 when

a

D

c

a

b

c

(a) n − 1

(b) n

(c) n + 1

(d) none of these

12. The number of distinct simple graphs with up to three nodes is (a) 15    (b) 10    (c) 7    (d) 9

d

d

e

f 13. A graph is planar if and only if it does not contain

G1

(a) a (a) a (a) a

b

G2 c

b b a

b

a a

b b

(b)

(a) subgraphs homomorphic to k3 and k3.3

c c

(b) subgraphs isomorphic to k5 or k3.3

c

(c) subgraphs isomorphic to k3 or k3.3

c c

(d) subgraphs homomorphic to k3 or k3.3 14. Maximum number of edges in an n-node undirected graph without self-loops is

(b) (b)

(a) n2 d

e

d d a

e e b

a a

b b

d

c

d d a

c c b

c

a a

b b

c c

n (n − 1) (b) 2

(c) n − 1

(d)

n (n + 1) 2

15. Let G be a graph with 100 vertices numbered 1 to 100. Two vertices i and j are adjacent if i − j = 8 or i − j = 12. The number of connected components in G is

(c) (c) (c)

(a) 8

(b) 4

(c) 12

(d) 25

(d) Chapter 10.indd 454

(d)

22-08-2017 06:38:49

EXPLANATIONS AND HINTS     455

(c) G  raph obtained by removing any edge from G is not connected

16. Consider a simple connected graph G with n vertices and n edges (n > 2). Then which of the following statements are TRUE?

(d) None of these

(a) G has no cycles (b) G has at least one cycle

ANSWERS 1. (b)

5. (c)

9. (d)

13. (d)

2. (c)

6. (a)

10. (d)

14. (b)

3. (a)

7. (c)

11. (b)

15. (b)

4. (d)

8. (b)

12. (c)

16. (c)

EXPLANATIONS AND HINTS 1. (b) The number of edges in a complete graph is given by n(n − 1) m= , where n = number of vertices 2 6(5) For K6 , m = = 15. 2 2. (c) The number of edges in a complete graph is given by n(n − 1) m= , where n = number of vertices 2 11(10) For K6 , m = = 55. 2 3. (a) Graph (a) is not a connected graph because E is not connected to any other vertex. All the other graphs are connected because all the vertices are connected to each other. 4. (d) We have to keep in mind that the two vertices are adjacent in the complement only when they are not originally adjacent. b a d c f e 5. (c) The given figure has 5 vertices. Hence, the vertex set V (G) = {A, B, C , D, E } . Hence, options (a) and (d) are incorrect. The edge set E(G) = {{A, B} , {B, D} , {D, C } , {C , A} , {D, E }} .

Chapter 10.indd 455

6. (a) Let us consider option (a), 0  1 A = 0 1  1

1 0 0 0 1

0 0 0 1 1

1  1  1  0  0

1 0 1 0 0

We have n=5 2

Hence, B = A + A + A3 + A4 We find A2 , A3 and A4 using matrix multiplication, 0 1  2 A = 0  1 1  3 1  = 2  0 1   3 3  1 1  33   AA == 22   0 0  1 1   4 4  4 4  = =  1 1   5 5 66 

1 0 0 0 1

0 0 0 1 1

1 0 1 0 0

1 2 1 1 1

2 1 2 0 0

0 1 0 2 2

11 22 11 11 11

22 11 22 00 00

00 11 00 22 22

66 22 22 22 55

33 22 00 44 55

55 22 44 00 11

1  0 1  1 1  0  0  1 0  1 3 1 0  2 3 33   0 0 11  1 1 00   0 0  22   1 1 33   1 1 66  44  55   11 22 

1 0 0 0 1

0 0 0 1 1

1 0 1 0 0

1 1 1  0 0

11 00 00 00 11

00 00 00 11 11

11 00 11 00 00

11 11 11  00  00 

22-08-2017 06:38:52

456     Graph Theory 

4 4  4 A = 1  5 6  17  8  =  11   3  8 

6 2 2 2 5

3 2 0 4 5

5 2 4 0 1

6  0 4  1 5  0  1  1 2  1

1 0 0 0 1

0 0 0 1 1

1 1 1  0 0

1 0 1 0 0

9. (d) G1 ∪ G2 should contain all the edges and vertices that are contained by G1 and G2 .

10 11 7 13 8 6 6 8 6 9 1 3  6 1 9 11 8 3 11 16

a

b

c

d

e

f

Now, 10. (d) The vertex E is connected to A, B, C and D. Hence, the degree of the vertex E is 4.

B = A + A2 + A3 + A 4 0  1 =  0 1  1 

1 0 0 0 1

0 0 0 1 1

1 0 1 0 0

4  4 +  1 5  6 

6 2 2 2 5

3 2 0 4 5

5 2 4 0 1

1  3   1  1 1 +  2 0  0 0  1 6 17   4  8 5 +  11 1  3 2  8

1 2 1 1 1

2 1 2 0 0

0 1 0 2 2

10 11 8 6 6 9 6 1 8 3

3  1 0 2 3 7 13  6 8 1 3 9 11 11 16

11. (b) The minimum number of edges in a connected cyclic graph with n vertices is n. 12. (c) The formula for number of distinct simple graphs is given by 2m − 1, where m = number of nodes. Therefore, number of distinct simple graphs with three nodes = 23 − 1 = 7. 13. (d) A graph is planar if and only if it does not contain subgraphs homomorphic to k3 or k3.3 . 14. (b) The edge count for each node in case of undirected graphs decreases gradually from n, n  −  1,

24 18 16 13 23   14 12 9 9 14 = 14 9 11 6 9  9 9 6 11 14   16 15 9 14 21  

n − 2, …, 1. Hence, the maximum number of edges n (n − 1) in an n-node undirected graph is . 2 15. (b) We are given that i − j = 8 are connected.

As B has no more zero entry off the main diagonal, the graph is connected.

Hence, vertices {1, 9, 17, 25, 33, …} are connected. Similarly,

Similarly, we can prove that the graphs given by adjacency matrices (b), (c) and (d) are not connected.

{2, 10, 18, 26, 34, …}, {3, 11, 19, 27, 35, …},

7. (c) The formula for the number of distinct simple graphs is given by 2m − 1 , where m = number of nodes. Therefore, the number of distinct simple graphs with three nodes = 25 − 1 = 31.

{4, 12, 20, 28, 36, …}, {5, 13, 21, 29, 37, …}, {6, 14, 22, 30, 38, …}, {7, 15, 23, 31, 39, …} and {8, 16, 24, 32, 40, …} are connected. Also, i − j = 12 are connected. Hence, {1, 13, 25, 37, …}, {2, 14, 26, 38, …}, {3, 15, 27, 39, …}, {4, 16, 28, 40, …}, {5, 17, 29, 41, …}, {6, 18, 30, 42, …},

8. (b) (a2, a1)

(a2, b1)

{7, 19, 31, 43, …}, {8, 20, 32, 44, …}, {9, 21, 33, 45, …}, {10, 22, 34, 46, …}, {11, 23, 35, 47, …} and

(b2, a1)

(b2, b1)

{12, 24, 36, 48, …} are connected. Now, we can see that 1, 9, 13 and 21 are connected. Hence, the graph has four connected components.

(c2, a1)

Chapter 10.indd 456

(c2, b1)

16. (c) The statement “Graph obtained by removing any edge from G is not connected” is true.

22-08-2017 06:38:54

SOLVED GATE PREVIOUS YEARS’ QUESTIONS     457

SOLVED GATE PREVIOUS YEARS’ QUESTIONS 1. Let G be an arbitrary graph with n nodes and k components. If a vertex is removed from G, the number of components in the resultant graph must necessarily lie between (a) k and n

(b) k - 1 and k + 1

(c) k - 1 and n - 1

(d) k + 1 and n - k (GATE 2003, 1 Mark)

Solution:  Minimum: It may be possible that the removed vertex doesn’t disconnect its component. Maximum: It may be possible that the removed vertex disconnects all components.  Ans. (a) 2. Consider the following graph:

4. A graph G = (V, E) satisfies ïEï£ 3 ïVï- 6. The min-degree of G is defined as mi n {deg ree(v)} . v ∈V

Therefore, min-degree of G cannot be (a) 3

(b) 4

(c) 5

(d) 6

(GATE 2003, 2 Marks) Solution:  Let the min-degree of G is x, then G has at least v ∗ x/2 edges. v ∗ x/2 − Re(a) (s + a)2 + w 02

t

Figure 1 |   Unit step function. ∞

L [ u(t)] =



u (t) e−st dt =

−∞





∞ 0+

1 = , Re(s) > 0 s

Re(s) > 0

(13)

(14)

(15)

Properties of Laplace Transform

where 0+ = lim (0 + e )

(3)

e →0

Laplace transforms exhibit the following properties:

2. Laplace transform of a unit impulse function as shown in Fig. 2 is given by

1. Linearity: If X1(s) and X2 (s) are, respectively, Laplace transforms of x1(t) and x2 (t), then

δ(t)

L a1x1(t) + a2 x2 (t) = a1X1(s) + a2 X2 (s)

1

t Figure 2 |   Unit impulse function. ∞

L [d (t)] = ∫ d (t)e−st dt = 1 for all s

(4)

−∞ 3. Laplace transform of some other common signals are given as under.

1 a. L [−u(−t)] = Re(s) < 0 s b. L [ tu(t)] = 2 Re(s) > 0 s k! c. L  tk u(t) = (k +1) Re(s) > 0   s 1 d. L e−at u(t) = Re(s) > − Re(a)   (s + a) 1 Re(s) < − Re(a)  e. L −e−at u(−t) =   (s + a)

L  x(t − t0 ) = e−st0 X(s) (16) Both transforms have same region of convergence. 3. Time scaling: If Laplace transform of x(t) is X(s) with a region of convergence equal to R, then Laplace transform of x(at) is given by L[x(at)] =

(5)

1

(6)

(7)

(8)

(9)



(s + a)2

Re(s) > − Re(a)

(10)

s 1 ×   | a |  a 

(17)

The region of convergence of resultant transform would be aR. 4. Shifting in s-domain: If Laplace transform of x(t) is X(s) having a region of convergence R, then L es0 t x(t) = X(s − s0 ) (18)   Region of convergence of resultant transform = R + Re(so ) 5. Time reversal: If Laplace transform of x(t) is X(s) with region of convergence R, then

1

If the regions of convergence of X1(s) and X2 (s) are R1 and R2 , respectively, then the region of convergence of resultant transform R is equal to intersection of R1 and R2 .

2. Time shifting: If Laplace transform of x(t) is X(s), Laplace transform of x(t − t0 ) is given by

0

f. L  te−at u(t) =  

(s + w 02 ) 2

k. L e−at sin w 0 t u(t)   w0 Re(s) > − Re(a) = (s + a)2 + w 02

e−st dt

0+

1 = − e−st s

Chapter 11.indd 466

w0

1



L[x(−t)] = X(−s)

(19)

The transform has region of convergence −R.

8/22/2017 5:04:27 PM

z-TRANSFORM     467

6. Differentiation in time domain: If Laplace transform of x(t) is X(s), then  dx(t)   = sX(s) L (20)  dt  If R is region of convergence of X(s), then region of convergence R ′ of the resultant transform is expressed by R ′ ⊃ R. 7. Differentiation in s-domain: If Laplace of x(t) is X(s) with a region of convergence R, then

dX(s) L[−tx(t)] = ds



(21)

The resultant transform also has region of convergence R. 8. Integration in time domain: If Laplace transform of x(t) is X(s), then t

Proper rational function (m < n): If the poles of X(s) are simple, then partial fraction expansion of X(s) can be written as cn c1 X(s) = + + (25) (s − p1 ) (s − pn ) where coefficients (Ck ) are given by Ck = (s − pk )X(s)| s = pk If the denominator D(s) has multiple roots, then expansion of X(s) consists of terms of the form: lr l1 l1 + + + (s − pi ) (s − pi )2 (s − pi )r



 t  X(s) L  ∫ x(t ) dt  = s −∞ 





(22)

If R is region of convergence for X(s), region of ­convergence R ′ of the resultant transform is expressed by R ′ = R ∩ {Re(s) > 0}

9. Convolution: If X1(s) and X2 (s) are Laplace transforms of x1(t) and x2 (t), respectively, then L[x1(t) ∗ x2 (t)] = X1(s) ⋅ X2 (s)



(23)

If R1 and R2 are regions of convergence of X1(s) and X2 (s), respectively, then region of convergence of resultant transform is expressed by R ′ ⊃ R1 ∩ R2. Convolution in time domain is multiplication in s-domain.

Inverse Laplace Transform



where lr −k =

(26)

1 dk [(s − pi )r X(s)] s = pi k ! dsk

Improper rational function (m ≥ n ): When X(s) is an improper rational function, that is, m ≥ n , X(s) can be written in the following form: X(s) =

N (s) R(s) = Q(s) + D(s) D(s)

X(s) is a polynomial in s with degree (m − n). R(s) is a polynomial in s with degree strictly less than n. The inverse Laplace transform of X(s) can be determined from inverse Laplace transform of Q(s) and R(s)/D(s). The inverse Laplace transforms of Q(s) can be determined by using transform below:

L

dkd(t) k

= sk, k = 1, 2, 3, …

dt

Laplace transform of R(s)/D(s) can be computed by partial fraction method explained earlier. Note that R(s)/ D(s) is a proper rational function.

Inverse Laplace transform of X(s) is given by −1

L



z -TRANSFORM

c + jw

1 X(s) = x(t) = 2pj



st

X(s)e ds

(24)

c − jw

In the above integral, the real (c) is to be selected in such a way that if region of convergence of X(s) is s 1 < Re(s) < s 2, then s 1 < c < s 2 If x(t) = x1(t) + x2 (t) + x3 (t) +  + xn (t) Then, X(s) = X1(s) + X2 (s) + X3 (s) +  + Xn (s)

z-Transform is the discrete time counter part of Laplace transform. z-Transform is used to convert difference equations representing discrete time signals into algebraic equations. Expression of discrete time signals in the form of algebraic equations simplifies analysis of discrete time systems in the same way as the Laplace transform does to the continuous time signals in the case of continuous time systems.

If X(s) is a rational function of the form:

(s − z1 )(s − z2 )… (s − zm ) N (s) X(s) = =k D(s) (s − p1 )(s − p2 )… (s − pn )

Then inverse Laplace transform of the above function can be determined by partial fraction expansion.

Chapter 11.indd 467

The z-transform X(z) of a discrete time signal or sequence is defined by ∞

X(z) =



x[n ]z−n

(27)

n =−∞

8/22/2017 5:04:38 PM

468     TRANSFORM THEORY 

Variable z is generally complex valued and is expressed in polar form by Z = re jΩ  (28) where r and Ω, respectively, are magnitude and angle of z. The z-transform defined by Eq. (27) is a bilateral or two-sided transforms unilateral or one-sided z-transform is defined as follows: X ′(z) =





x[n ]z−n 

z for all z. − z 1

u[n ] ↔

x[n ] ↔ X(z)

(30)

Again as in the case of Laplace transform, a region of convergence is defined in the case of z-transform too. Region of convergence (ROC) in the case of z-transform is the range of value of complex variable z for which the z-transform converges. To illustrate the term region of convergence, let us consider

3. Some other common z-transform pairs are given as under. a. −u[−n − 1] ↔

z for | z | < 1 z −1

x[n ] = a u[n ], where a is real

b. d[n − m ] ↔ z−m for all z except z = 0 (for m > 0) and ∞(for m < 0) c. an u[n ] ↔ −

z for | z | > | a | z −a

d. −an u[−n − 1] ↔



e. nan u[n ] ↔

n =−∞

for | z | > | a | 

(37)

(z − a)2

f. −nan u[−n − 1] ↔

az (z − a)2

∑ (az−1 )n

z 2 − (2 cos Ω0 )z + 1

for | z | > |1|

n =0

i. (sin Ω0 n)u[n ] ↔

n =0

for | z | > |1|

∑ (az−1 ) < ∞ or | az−1 | < 1 For convergence of z-transform, ) < ∞ or | az

−1

(sin Ω0 )z

z − (2r cos Ω0 )z + r 2

This implies that |z | > |a|.

n k. (r sin Ω0 n)u[n ] ↔ (r sin Ω0 )z

z -Transform of Common Sequences 1. z-Transform of a unit impulse sequence d[n ] is given by ∞

X(z) = ∑ d [n ]z−n = z−n = 1 for all z



n =−∞

Chapter 11.indd 468

u[n ] z−n =

1 1−z

= −1

 for | z |

(42)

 for | z | > r

(43)

an , for 0 ≤ n ≤ N − 1  ↔ l.     0, otherwise 1 − az−1

for | z | > 0

(44)

(31)

2. z-Transform of a unit step sequence u[n] is given by ∞

z 2 − (2r cos Ω0 )z + r 2

1 − aN z−N

n =0

d[n ] ↔ 1 for all z.

 (41)

2

Therefore,



2

n =0

X(z) =

z − (2 cos Ω0 )z + 1

z − (r cos Ω0 )z

| | a |  g. (n + 1)an u[n ] ↔   (z − a)   

n



(32)

(29)

n =0 x[n] and X(z) are said to form a transform pair denoted as



Therefore,

z ,| z | > 1 z −1

Properties of z -Transform 1. z-Transform exhibits the property of linearity. If X1(z) and X2 (z), respectively, are z-transforms of X1[n ] and X2 [n ], then

8/22/2017 5:04:47 PM

z-TRANSFORM     469

a1x1[n ] + a2 x2 [n ] ↔ a1X1(z) + a2 X2 (z)



(45)

If R1 and R2 are ROC of X1(z) and X2 (z), respectively, then ROC of a1X1(z) + a2 X2 (z) is given by R ′ ⊃ R1 ∩ R2 . (a1 ) and (a2 ) are arbitrary constants.

6. If X(z) is z-transform of x[n] with ROC = R, following transform pair holds: n



k =−∞



x[k ] ↔

1 1−z

−1

X(z) =

z X(z) (z − 1)

(51)

2. If z-transform of x[n] is X(z), then z-transform of Region of convergence R ′ is given by R ′ = R ⊃ {| z | > 1} x[n − n0 ] is given by z−n 0 X(z) R ′ = R ⊃ {| z | > 1} x[n − n0 ] ↔ z n 0 X(z)

That is,

n

(46)







x[k ] is the discrete time counterpart of inte-

k =−∞







The region of convergence is given by R ′ = R ∩ {0 < | z | < ∞}, where R is region of convergence for x(z). This is the time shifting property. As an example, z-transform of x[n - 1] is given by z−1X(z) with region of convergence given by R ′ = R ∩ {| z | < ∞}. If X(z) is z-transform of x[n] with ROC (R), then z-transform of z0 n x[n ] is given by z z0n x[n ] ↔ x   with ROC R = |z0| R  z0 

e jΩ0 n x[n ] ↔ X(e−jΩ0 z) with R ′ = R

(48)

In this special case, all poles and zeros are rotated by angle Ω0 and the region of convergence remains unchanged. 4. According to the time reversal property of z-transform, if X(z) is z-transform of x[n] with ROC = R, then 1 1 x[−n ] ↔ X   with ROC R ′ = z  R



This



If X1(z) is z-transform of x1[n ] with ROC = R1



X2 (z) is z-transform of x2 [n ] with ROC = R2



Then



And R ′ ⊃ R1 ∩ R2

x1[n ] ∗ x2 [n ] ↔ X1(z) ⋅ X2 (z) 

(52)

Inverse z-Transform

As a special case of this property, following transform holds:



of

(47)

3. A pole (or zero) at z = zk in X(z) moves to z = z0 zk after multiplication of x[n] by z0 n. The region of convergence also gets multiplied by |zo|.

gration in the continuous time domain. 7. According to the convolution property z-transform,

(49)

Inverse z-transform of x(z) is given by x[n ] =

1 2pj



∫ X(z)z

n −1

dz

(53)

c

where C is the counterclockwise contour of integration enclosing origin. As an alternative approach, inverse z-transform of X(z) may be found by expressing x(z) as a sum of functions that have known inverse transforms and using linearity property. For example, if X(z) = X1(z) + X2 (z) +  + Xn (z) then x[n ] = x1[n ] + x2 [n ] +  + xn [n ] where x1[n ] ↔ X1(z)

implies

that a pole or zero at  1  z = zk moves to   after time reversal. As far as  zk  the ROC is concerned, time reversal means that a right-sided sequence becomes a left-sided sequence and vice versa. 5. In the case of multiplication of discrete sequence x[n] by n, the following transform pair holds:

x2 [n ] ↔ X2 (z) . . . . xn [n ] ↔ Xn (z)

z-Transform as Power Series Expansion dX(z) nx[n ] ↔ −z dz



(50)

The region of convergence remains unchanged, i.e., R ′ = R .

Chapter 11.indd 469

z-Transform given by Eq. (27) earlier can also be represented by a power series where the sequence values of x[n] are nothing but the coefficients of z−n.

8/22/2017 5:05:02 PM

470     TRANSFORM THEORY 

Relationship between z-Transform and Laplace Transform

That is, ∞

X(z) =



x[n ]z−n

n =−∞

= ...... + x[−2]z 2 + x[−1]z + x(0)  + x(1)z

−1

+ x[2]z

−2

(54)

+ ...

This approach is particularly useful for finite-length sequences.

z-Transform is related to Laplace transform by following expressions. 1 1. s = ln z (59) T 2. X(s) = X(z)|z = e+Ts (60) where T is the sampling period.

Partial Fraction Expansion

FOURIER TRANSFORM

Partial fraction expansion method can be used to determine inverse z-transform when X(z) is a rational function of z. If X(z) is expressed by

X(z) =

Fourier transform transforms a time domain signal into its frequency domain counterpart. If X(w) is the Fourier transform of x(t), then ∞

(z − z1 )(z − z2 ) (z − zm ) N (z) =k⋅ D(z) (z − p1 )(z − p2 ) (z − pn )

X(w ) =



x(t) =



(55)

X(z) = c0 + ∑ ck

m−n

z  z − pk

q =0



n

∑ bqz q + ∑ ck z − p k =1

z



(57)

k

lr l1 l2 + + + 2 (z − pi ) (z − pi ) (z − pi )r where

lr −k

(62)

−∞

x(t) and X(w ) form a Fourier transform pair written as

X(w ) = X(w ) e jf(w) (63) |X(w )| is known as the magnitude spectrum and f(w ) is called the phase spectrum of X(w). If X(t) is a real signal, we can write ∞

X(−w ) =



x(t) e jwt dt

(64)

−∞

And X(−w ) = X * (w ) We can also write

In the case of multiple order poles, the partial fraction X(z) expansion of will consists of terms of the form z (assuming pole (pi) has multiplicity of r),



X(w )e jwt dw

X(w ) is, in general, complex and is expressed as

(56) k =1 If m > n, then a polynomial of z must be added to the right-hand side of Eq. (57), the order of which is (m − n). In that case, partial fraction expansion has the following form: X(z) =



x(t) ↔ X(w )

X (z ) | z z = pk

Equation (56) may be written as n

1 2p



n c ck = 0 +∑ z k =1 z − pk

Where c0 = X(z) |z = 0 and ck = (z − pk )

(61)

The inverse Fourier transform is defined by

cn c1 c2 X (z ) c 0 = + + + + z z z − p1 z − p2 z − pn



x(t)e−jwt dt

−∞



If n ≥ m and all poles are simple, then



X(z)  1 d  (z − pi )r  = k z  k ! dz  z=p k



| X(−w )| = | X(w )| and f(−w ) = − f(w ) 

(65)

It follows from Eq. (64) that in the case of periodic signals, the amplitude spectrum | X(w )| is an even function and phase spectrum f(w ) is an odd function.

Convergence of Fourier Transforms 

(58) Dirichlet conditions for convergence of Fourier transforms are defined as follows:

i

Chapter 11.indd 470

8/22/2017 5:05:11 PM

FOURIER TRANSFORM     471



1. x(t) is absolutely integrable, i.e.



| x(t)| dt < ∞ .

−∞

6. According to property of duality or symmetry X(t) ↔ 2p x(−w )



2. x(t) has a finite number of maxima and minima within any finite integer. 3. x(t) has a finite number of discontinuities within any finite interval. Each of the discontinuities is finite. Compliance of the above-mentioned Dirichlet conditions guarantees the existence of Fourier transform. It may be noted that if impulse functions are permitted in the transform, signals not satisfying these conditions can have Fourier transform.

7. Differentiation in time domain causes multiplication is frequency domain is dx(t) ↔ j wX(w ) dt



(−jt)x(t) ↔

(73)

t



(66)

x(t ) dt ↔ pX(0)d (w ) +

−∞

1. According to linearity property, if X1(w ) and X2 (w ) are Fourier transform of x1(t) and x2 (t), respectively, then a1x1(t) + a2 x2 (t) ↔ a1X1(w ) + a2 X2 (w )

dX(w ) dw

9. Fourier transform of integration in time domain is given by

Important properties of Fourier transforms, many of which are similar to those of Laplace transforms are briefly described as under.



(72)

8. Differentiation in frequency domain causes multiplication in time domain is



Properties of Fourier Transform

(71)

1 X(w ) jw

(74)

10. Convolution in time domain is multiplication in frequency domain x1(t) ∗ x2 (t) ↔ X1(w ) ⋅ X2 (w )



(75)

(a1 ) and (a2 ) are arbitrary constants. 2. According to the time shifting property, a shift in time causes addition of a linear phase shift in the Fourier spectrum. x(t − t0 ) ↔ e−jwt0 X(w )



e jw 0 t x(t) ↔ X(w − w 0 )

x1(t) ⋅ x2 (t) ↔

(67)

3. Complex modulation in time domain causes frequency shifting in frequency domain, that is,

The above equation is also called time convolution theorem. 11. Convolution in frequency domain is multiplication in time domain this is also known as frequency convolution theorem.

x(t) ↔ X(w ) = A(w ) + jB(w )

(68)

4. Scaling in time domain causes inverse scaling in frequency domain.





1  w  ×  | a |  a 



Chapter 11.indd 471

x(−t) ↔ X(−w )

(70)

(77)

and

X(−w ) = X * (w )

then

xe (t) ↔ A(w )

(78)

and

xo (t) ↔ j B(w )

(79)

(69)

Inverse scaling of both frequency variable (w) and w  1 amplitude scaling of X   by takes place. a |a | According to time reversal property,

(76)

12. If x(t) is real and given by xe (t) + xo (t) where xe (t) and xo (t) are even and odd components of x(t) and if

x(at) ↔

1 X (w ) ∗ X2 (w ) 2p 1



This implies that Fourier transform of an even signal is a real function of w and Fourier transform of an odd signal is a pure imaginary function of w. 13. Bilateral Laplace transform of x(t) can be interpreted as the Fourier transform of x(t) e−s t .

8/22/2017 5:05:21 PM

472     TRANSFORM THEORY 

SOLVED EXAMPLES 1. Determine Laplace transform of x(t) = eat u( − t). therefore X(s) = Solution:  Laplace transform of x(t) is given by ∞



X(s) =

−st

e u(−t) e at

= dt

−∞ 0

=



−∞

eat ⋅ e−st dt =





e−10 e−5s 1 − (s + 2) (s + 2)

1  1 − e−5(s + 2)  with Re(s) > −2  (s + 2) 

1 4. Given that Laplace transform of u(t) = , what is s the Laplace transform of d(t)?

e−t(s−a)dt

−∞ 0 −t(s−a)

1 1 s du(t) =− e Laplace transform of d(t) = Laplace transform of (s − a) −∞ dt du(t) 1 d(t) = Laplace transform of =− for Re(s) < a dt (s − a) Using time differentiation property, we get 2. Determine the Laplace transform and region of 1 d(t) ↔ s ⋅ = 1 convergence of x(t) = e−3t u(t) + e−4t u(−t). s 5. Determine the inverse Laplace transform of Solution:  Laplace transform of s , Re(s) > 0 . 2 1 (s + 4) e −3t u(t) = with Re (s) >−3 (s + 3) Solution:  cos 2t u(t) Laplace transform of s Laplace transform of cos w 0 t = 2 and hence (s + w 02 ) 1 e−4t u(−t) = with Re(s) −3 Therefore, inverse Laplace transform of

x(t) = e−2t [u(t) − u(t − 5)]

1 = e−3t u(t) (s + 3)

Solution:  x(t) can be rewritten as x(t) = e−2t u(t) − e−2t u(t − 5) It is also given that Re(s) < −1 x(t) = e−2t u(t) − e−2t u(t − 5) Therefore, inverse Laplace transform of = e−2t u(t) − e−10 ⋅ e−2(t−5)u(t − 5) Now Laplace transform of e−2t u(t) =

1 s+2

and Laplace transform of e−2(t−5)u(t − 5) = e−5s × e−2(t−5)u(t − 5) = e−5s ×

Chapter 11.indd 472

1 s+2

1 = e−1t u(−t) (s + 1) 2s + 4 Therefore, inverse Laplace transform of 2 s + 4s + 3 is given by 1 s+2

e−3t u(t) + e−t u(−t). 7. Determine the inverse Laplace transform of 2s + 1 , Re(s) > −2. s+2

8/22/2017 5:05:31 PM

SOLVED EXAMPLES     473

Solution:  X(s) =

2s + 1 2(s + 2) − 3 3 = = 2− s+2 s+2 s+2

Inverse Laplace transform of 2 = 2d(t) Inverse Laplace transform of

z  12. Determine the inverse z-transform of X  . a Solution:  This is one of the properties of z-­transform, which says that z zo n x[n ] ↔ X    z0 

3 = 3e−2t u(t) s+2

Therefore, x(t) = 2d (t) − 3e−2t u(t) 8. Determine the z-transform and associated region of convergence of d[n − 3]. Solution:  d[n ] ↔ 1, for all z Applying time shifting property,

13. What is the Fourier transform of an impulse function? Solution:  Fourier transform of ∞

d[n − n0 ] ↔ z−no , | z | > 0 . Therefore, z-transform of d[n − 3] = z−3. −n

9. Determine the z-transform of x[n ] = a

u [−n ].

z Solution:  z-Transform of an u[n ] is equal to z −a for | z | > a. By using time reversal property, we get

z-Transform | z |
|a | z −a

x(t) = e−at for t > 0 x(t) = eat for t < 0

and

0

Therefore, X(w ) =

Therefore, z-transform of

−∞ 0

d z z = , |z | > |a | nan−1u[n ] = da (z − a) (z − a)2

=

(z − a)2

Solution:  x[n ] ↔ X(z) By multiplication property,

0 (a − jw )t

∫−a e

a

dt + ∫ e−(a + jw )t dt 0

1 1 2a = + = 2 a − jw a + jw a + w2

az 11. Determine the inverse z-transform of





eat ⋅ e−jwt dt + ∫ e−at ⋅ e−jwt dt

.

15. Determine the Fourier transform of x(t) = cos w 0 t. Solution:

nx[n ] ↔ −z an u[n ] ↔

Now

z z −a

Therefore, z-transform of d  z  na u[n ] = −z   dz  z − a  (z − a) − z   = az = −z    (z − a)2  (z − a)2   n

And hence the answer.

Chapter 11.indd 473

(

dX(z) dz

((

)

))

1 w 0 t11 jwj−w0jt0wt 0 t −−jwjw0 t0 t cos = +ee ++ee cos w 0 tcos = ww00tetj= 22 2 1 1 jw 0 t 11 1jwjw0 t− jw 01 +t ee−−jwjw0 t0 t = e == +ee e 0 t+ 22 2 22 2 tt 00 eej2wjwpd ()w−−ww00)) e jw 0 t ↔ (↔ w 2−2pd w(0w Now ↔ pd Now Now

and

tt −jw 0 t −−jw 00 aaend nd e↔ e 2jwpd ↔ 2pd pd (w (↔ w 2+ w(0w ) ++ww00))

     Therefore, 1 2pd (w − w 0 ) + 2pd ( w + w 0 ) 2 or cos w 0 t ↔ p d (w − w 0 ) + d (w + w 0 ) cos w 0 t ↔

8/22/2017 5:05:43 PM

 (a + jw )   (a + jw )     Now e−at u(t) ∗ e−at u(t) ∞

=



e−at u(t )e−a(t−t )u(t − t )dz

−∞ t

= e−at ∫ dz = te−at u(t)

474     TRANSFORM THEORY 

0

16. Find inverse Fourier transform of x(w ) =

1 . (a + jw )2

Solution:   1 1   1 1   1 1       X(X w()w=) = = = 2 2  (a + jw )   (a  )  (a (+a j+wj)w )   (a + jw   )   (+a j+wj)w   1 1 ↔↔ ua(ttu)(t) , , e−aaet−a Now Now, Now (a (+a j+wj)w ) Convolution in time domain is equal to multiplication in frequency domain. Therefore,    1  1 1 −at at e −at u(− at    11   → e1−→ ) u(t) ) ∗u ) ∗ ee−tat ue((tt)) ∗ue(−tat     u( t→  (a + j(w   (jaw+  a + ) ( a + j w ) ) ) j w     (a + j w )   (a + jw )  −at −tat at e(t−)at∗uee(− ) ∗u ) u(t) Now Now e−at uNow ue((tt)) ∗ue(−tat ∞



=

∞ =−at

∞ −at a(t −t−)a(t −t ) (tt(− t− ) t )dz u(−tat)e u(t−−au ue(t )e− )tdz

∫= ∫

e

−∞

−∞

=

u(t )e

e

u(t − t )dz

−∞ t

t t at −at at te−at u(−tat at ) = e−dz e =−dz te−= u(t)



=∫ e

0

0

∫ dz = te

u(t)



(c)





x (t) est dt



e−st dt

(b)

17. What is the Fourier transform of a DC signal with unity strength? Solution:  d(t) ↔ 1 By principle of duality, Fourier transform of DC signal of unity strength is given by 2pd (−w ) = 2pd (w ) and hence the answer. 18. Give the statement of frequency convolution theorem. Solution:  According to the frequency convolution theorem, multiplication in time domain is convolution in frequency domain. That is, if X1(w) and X2(w) are Fourier transforms of x1(t) and x2(t), respectively, then according to frequency convolution theorem, x1 (t) ⋅ x2 (t) ↔ X1 (w ) ∗ X2 (w )



x (t) e−st dt



est dt

6. Which one of the following expressions is correct for inverse Laplace transform? c + jw

(a) x (t) =

−∞ ∞

(d)

−∞

1 2pj 1 2p

−∞

2

(a) s2

(b)

(c)



(d)

2

s

s

1 s



(s + a)



c + jw

1 2pj

is

(c) tu (t)

(d) ae

u (t )

4. Laplace transform of x1 (t) convolving with x2 (t) is (a) x1 (s) ∗ x2 (s)

(b) x1 (s) ⋅ x2 (s)

(c) x1 (s) / x2 (s)

(d) None of these

s + 2s + 5 (a) 5e−t cos 2t (c) 5e

sin 2t

c − jw





is 2

(a) X(z) =

t

(d) 5e sin 2t

(c) X (z ) =





x [ n ] z n (b) X(z) =

n =−∞

(b) 5et cos 2t

∑ x [ n ] z n n =0

Chapter 11.indd 474

X (s) e−st dt

(a) Difference equations into algebraic equations (b) Differential equations into algebraic equations (c) Integral equations into algebraic equations (d) Integral-differential equations into algebraic equations 8. Which one of the following expressions is correct for determining z-transform?

10 5. Inverse Laplace transform of



7. z-Transform transforms

(b) e−at u (t) −at

X (s) e−st ds

c − jw

2

(a) te−at u (t)

X (s) est ds

−∞

1 2pj

(d) x (t) =

1 3. Inverse Laplace transform of

X (s) est ds

c − jw

c + jw

(c) x (t) =

1

2





(b) x (t) =

2. Laplace transform of x (t) = t is

−t

(a + jw )2

PRACTICE EXERCISE

1. Which one of the following expressions is correct for determining Laplace transform of x (t) ?

−∞ ∞

1

0

1 1 1 −at therefore u(−tat )↔ therefore , therefore te−at, ute(t), ↔ te u(t()a↔ 2 )2 jw )2 (a + jw )+ (jaw+

(a)

therefore, te−at u(t) ↔



x [ n ] z−n

n =−∞

(d) X (z ) =



∑ x [n ] z−n n =0

8/22/2017 5:05:56 PM

EXPLANATIONS AND HINTS     475

9. z-Transform of x [−n ] is (assuming x (z ) to be the z-transform of x[n ]) (a) −X (z )

(b) X (−z )

12. Laplace transform and z-transform are interrelated by ln z (a) s = ln z (b) s = T (c) s = T ln z (d) s = z

1 (c) X    z 

(d) None of these

13. Frequency convolution theorem can be expressed as (a) x1 (t) ∗ x2 (t) ↔ X1 ( w ) ⋅ X2 (w )

10. If X1 (z ) and X2 (z ) are, respectively, z-transforms of x1 [ n ] and x2 [ n ] then x1 [ n ] ∗ x2 [ n ] is

(b) x1 (t) x2 (t) ↔

1 X (w ) ∗ X2 (w ) 2p 1

(a) X1 (z ) ∗ X2 (z )

(b) X1 (z ) / X2 (z )

(c) x1 (w ) ∗ x2 (w ) ↔ x1 (t) ⋅ x2 (t)

(c) X1 (z ) ⋅ X2 (z )

(d) X1 (z ) + X2 (z )

(d) None of these 14. Fourier transform of an impulse function is

11. If X (z ) is z-transform of x [ n ], then z-transform of

(a) 1 (c) ∞

nx[n ] is (a)

dX (z ) dz

(b) z

15. Fourier transform of e−jw 0 t is

dX (z ) dz

d X (z ) (c) dz 2

(d) −z

(b) 0 (d) 2pd (w )

dX (z ) dz

(a) 2pd (w − w 0 )

(b) pd (w − w 0 )

(c) pd (w + w 0 )

(d) 2pd (w + w 0 )

ANSWERS 1. (b)

4. (b)

7. (a)

10. (c)



13. (b)

2. (c)

5. (c)

8. (b)

11. (d)



14. (a)

3. (a)

6. (a)

9. (c)

12. (b)



15. (d)

EXPLANATIONS AND HINTS 1. (b) This is by definition of Laplace transform. 2. (c) Multiplication in time domain is differentiation in s-domain. If X (s) is Laplace transform of x (t), then dX (s) Laplace transform of tx (t) = − ds Now x (t) = t = tu (t) Laplace transform of u (t) =

1 s

Therefore, Laplace transform of tu (t) =

−d  1  1   = 2 ds  s  s

Therefore, inverse Laplace transform of 1 = te−at u (t) 2 (s + a) 4. (b) Convolution in time domain is multiplication in frequency domain. 10 2×5 5. (c) X (s) = = 2 s + 2s + 5 (s + 1)2 + 22 Inverse Laplace transform of 2 = e−t sin 2t 2 2 (s + 1) + 2

1 s+a

Therefore, inverse Laplace transform of 10 = 5e−t sin 2t 2 s + 2s + 5 6. (a) It is by definition of inverse Laplace transform.

−d  1  1 u (t ) = =  ds  s + a  (s + a)2

7. (a) What Laplace transform does to continuous time signals, z-transform does to the discrete time signals.

3. (a) Laplace transform of e−at u (t) = Therefore, Laplace transform of −at

te

Chapter 11.indd 475

8/22/2017 5:06:13 PM

476     TRANSFORM THEORY 

8. (b) It is by definition of z-transform.

14. (a) Fourier transform of impulse function ∞

9. (c) It is by the time reversal property of z-transform. =

10. (c) It is the convolution property of z-transform.



d (t) e−jwt dt = 1

−∞

11. (d) It is the multiplication property of z-transform. 12. (b) If T is the sampling time interval, then X (s) becomes equal to X (z ) for z = e+sT , which gives 1 s = ln z. T

15. (d) It is the frequency shifting property of Fourier transform, according to which e jw 0 t x (t) ↔ X (w − w 0 ) .

13. (b) It is by the statement of multiplication property, also known as frequency convolution theorem.

SOLVED GATE PREVIOUS YEARS’ QUESTIONS 1. If (L) defines the Laplace transform of a function, L[sin (at)] will be equal to a (a)

Solution:



Laplace transform of u (t − a) =

∫e

(c)

(b)

2

s +a

(d)

2





a

s2 + a2 s

=

s −a



2

u (t − a) dt

o

a

s2 − a2 s

−st

2

(GATE 2003, 2 Marks)



e−st ⋅ 0 dt +∫ e−st ⋅ 1 dt =

o

a

e−as s Ans. (d)

4. In what range should Re (s) remain so that Laplace transform of the function e(

a + 2) t + 5

exists.

Solution: L f (t) =





e−st f (t) dt =





0

e−st sin (at) dt

s2 + a2

(c)

s −w 2



(b)

2

s + w2 w

= e( ) ⋅ e5   5 1 e Laplace transform =    s − (a + 2) Therefore, Laplace transform exists for s > (a + 2).

2



(d)

2

s −w 2



−st

∫e

sin wt dt =

o

Ans. (a)

5. A solution for the differential equation x˙ (t) + 2x(t) =

( )

d (t) with the initial condition x 0− = 0 is (a) e−2t u (t) (b) e2t u (t)

Solution: L sin w t =

w



s + w2 2



Ans. (b) 0, for t < 0 u (t − a) =  1, for t > 0

(a) ae



eas (b) s

e−as (c) a

sX (s) − x (0) + 2X (s) = 1 or

Its Laplace transform is e−as (d) s

(c) e−t u (t) (d) et u (t)

(GATE 2006, 1 Mark) . Solution:  Given that x (t) + 2x(t) = d (t) Taking Laplace transforms on both sides, we get

3. A delayed unit step function is defined as

−as

a +2 t



2

(GATE 2003, 2 Marks) ∞

(GATE 2005, 2 Marks) a + 2) t + 5

w

s s +w s

(d) Re (s) > a + 5

Solution:  e(

Ans. (b)

2. Laplace transform of function sin wt is 2

(c) Re (s) < 2 

a

(a)

(b) Re (s) > a + 7

0

= 

(a) Re (s) > a + 2

X (s) =

1 s+2

Therefore, x (t) = e−2t u (t) 

Ans. (a)

6. If F (s) is the Laplace transform of f(t), then t



(GATE 2004, 2 Marks)

Laplace transform of

∫ f (t ) dt

is

0

Chapter 11.indd 476

8/22/2017 5:06:28 PM

SOLVED GATE PREVIOUS YEARS’ QUESTIONS     477

F (s)

F (s) (a)



(b)

9. Laplace transform of f (x) = cosh (ax) is

− f (0 )

s

s (c) sF (s) − f (0)

(d)





a

F (s) ds

(a)

(GATE 2007, 2 Marks)

t

∫ ∫  ∫ f (t) dt 0 0

n

(b)

s − a2 s 2



(d)

s2 + a2

F (s)

t

s 2

(c)

Solution:  In general, Laplace transform of t

s −a a 2

s2 + a2



(GATE 2009, 2 Marks)

= sn

0

t

Therefore, Laplace transform of

∫ f (t ) dt =

F (s)

Solution:  It is a formula from the list of standard formulas in Laplace transform.  Ans. (b)

s

0



Ans. (a) ∞

7. Evaluate



sin t dt t

1 10. Laplace transform of a function f (t) is 2 . s (s + 1) The function f (t) is

0

(a) p

(b)

p 2

p p (d) 4 8 (GATE 2007, 2 Marks) (c)



(b) t + 1 + e−t

(c) −1 + e−t

(d) 2t + et



(GATE 2010, 2 Marks) Solution: 1

Solution:

s (s + 1)

m As

(a) t − 1 + e−t

A B C + 2 + s s s+1

=

2

L(sin mt) =

= f (t), (say) (s2 + m2 )

As (s + 1) + B (s + 1) + Cs2 =

Therefore,  sin mt  L  =  t 





∫ f(s)ds = ∫ s2 + m2 mds

s

= tan−1

s

s2 (s + 1)



s ms

Solving this, we get A = −1, B = 1, C = 1 1 Therefore,

By definition, we have ∞

−st

∫e

s (s + 1)

=

2

sin mt p s dt = tan−1 2 t m

Therefore, inverse Laplace transform of

0

s   = 0 if m > 0 or p if m < 0. m

s→ 0

Therefore, taking limits as s → 0, we get ∞

sin mt p p ∫ t dt = 2 if m > 0 or − 2 if m < 0. 0 p As m = 1, which is > 0, the answer is . 2  Ans. (b) 1 8. Inverse Laplace transform of 2 is s +s (b) 1 − et

(a) 1 + et 

(c) 1 − e−t

(d) 1 + e−t

(GATE 2009, 1 Mark) Solution: 1 = 2

s +s

1 1 1 = − s (s + 1) s s + 1 1

Therefore, Laplace transform of 2

= 1 − e−t

s +s 

Chapter 11.indd 477

1

−1  

Now lt tan

−1 1 1 + 2 + s s+1 s

Ans. (c)

s (s + 1) 2

= −1 + t + e−t



Ans. (a)

  3s + 1  , if 11. Given L−1  3  2  s + 4s + (k − 3) s  then the value of K is (a) 1

(b) 2



(c) 3

lim f (t) = 1,

t →∞

(d) 4

(GATE 2010, 2 Marks) Solution: lim f (t) dt = lim sF (s)

t →∞

s→ 0

  3s + 1  = 1 or Therefore, lim s   s→ 0  s3 + 4s2 + (K − 3) s      3s + 1  =1 lim  2  s→ 0  s + 4s + K − 3  1 or =1 ⇒K =4 K −3  Ans. (d)

8/22/2017 5:06:45 PM

478     TRANSFORM THEORY 

Common Data Questions 12 and 13:  Given f (t) and g (t) as shown below: f(t)

g(t)

1

1

Hence, inverse Laplace transform of

1 = 1 − e−t s (s + 1)



Ans. (d)

15. Consider the differential equation: d2y

dy + y (t ) = d (t ) dt

+2 dt t 0

t

1

3

0

5

12. g (t) can be expressed as (a) g (t) = f (2t − 3) (c) g (t) = f (2t − 3/2) 

with y (t)





(a) -2

(d) g (t) = f (t/2 − 3/2)

(

)

and

(

(

)



3

)



(GATE 2012, 2 Marks)

dy (t)

e−st ⋅ 0 dt +∫ e−st ⋅ 1 dt +∫ e−st ⋅ 0 dt

0

3

dt

5

dy (t)

 e−st  5 e−5s − e−3s   =  =     s s   3  

dt

e−3s −2s e−3s − 13 = =− 1 − e−2s e s s

(

)

(

= − −2e−t + e−t − te−t  u (t)   = −[−2 + 1 + 0 ] = 1

t = 0+



)

Ans. (d)

16. The function f (t) satisfies the differential equation: d 2f



Ans. (c) 1 14. Inverse Laplace transform of F (s) = is s (s + 1) given by (a) f (t) = sin t

(b) f (t) = e−t sin t

(c) f (t) = e−t

(d) f (t) = 1 − e−t



A (s + 1) + Bs 1 A B = + = s (s + 1) s s +1 s (s + 1)

Solving this, A = 1, B = -1 Therefore,

Chapter 11.indd 478

+f = 0 dt2 and the auxiliary conditions f (0) = 0,

df (0) = 4. dt The Laplace transform of f (t) is given by (a)

(GATE 2012, 2 Marks) Solution: 

1 1 1 = − s (s + 1) s s + 1

(d) 1

Therefore, y (t) = − 2e−t + te−t  u (t)   Differentiating y(t) with respect to t, we get



5

(c) 0

By partial fraction expansion, this becomes   2 1  = −  +   (s + 1) (s + 1)2   

Solution:  Laplace transform of g (t ) =

is y = 0+

Solving this equation, we get − (2s + 3) Y (s) = 2 (s + 1)

)

(

dy dt

=0 t=0

s2Y (s) − sY(0) − Y ′(0) + 2[sY (s) − Y (0)] + Y (s) = 1

1 −5 s e − e−3s s 1 (d) e5s − e3s s (GATE 2010, 2 Marks) (b)

dy dt

Solution:  Taking Laplace transform on both sides of the given differential equation, we get

No other answer satisfies these conditions.  Ans. (d) 13. The Laplace transform of g (t) is 1 3s e − e5 s s e−3s (c) 1 − e−2s s

(b) -1



(GATE 2010, 2 Marks) Solution:  Let us take g(t) = f (t/2 − 3/2)

(a)

= −2 and

The numerical value of

(b) g (t) = f (t/2 − 3)

This gives g (3) = f (0) , which is true g (5) = f (5/2 − 3/2) = f (1) , which is true.

t=0

2

2 s+1

(b)



4 s+1

4

2 (d) 2 s +1 s +1 (GATE 2013, 2 Marks) (c)

2

d 2f +f = 0 dt Taking Laplace transform on both sides, we get Solution:     

s2 F (s) − sf (0) − f ′ (0) + F (s) = 0

8/22/2017 5:07:01 PM

SOLVED GATE PREVIOUS YEARS’ QUESTIONS     479

or s2 F (s) − 0 − 4 + F (s) = 0 or F (s) s2 + 1 = 4   Therefore, F (s) =

s−2

s+2 (a)

(s − 2) + 16



(b)



(d)

(s − 2)2 + 16

2

s−2

4

s+2

(c)

and hence the answer.

2

2

(s + 2)2 + 16

(s + 2) + 16

(s + 1) 

Ans. (c)

17. Let X (s) =

3s + 5



s + 10s + 21 form of a signal x(t). Then, x(0+) is

(a) 0

(GATE 2014, 1 Mark)

be the Laplace trans-

2

(b) 3

Solution:  Let L[f(t)] = F(a), then L[ext f(t)] = F(a − x). Also, putting w = 4, we get

(c) 5 (d) 21 (GATE 2014, 1 Mark)



L[e−2t cos(4t)] =

s−2

s−2

Solution:  We have to find the Laplace transform 3s + 5 of X (s) = 2 s + 10s + 21 x[0+] = lim

s→∞

(s

2

s ⋅ 3s + 5

2

(s − 2)2 + 16 Ans. (b)

20. For the time domain function f(t) = t2, which ONE of the following is the Laplace transform of

)

t

∫ f (t) dt ?

+ 10s + 21

[using initial value theorem]  5 s2 3 +   s  = lim =3 s→∞  10 21  1 +  + 2  s s 

0

3 (a)

1

2

(b)

4



(c)

2

(d) s4

s

 Ans. (b)

2

3

4s

s

(GATE 2014, 1 Mark) Solution:   We have

18. A function f(t) is shown in the figure.

1  F (s) L  ∫ f (t) dt = where F (s) = L (f (t)) s 0   

f(t) 1/2

 2  1   3    2 ⇒ L  ∫ t2dt = s = 4 s 0  s  

T/2 -T/2

=

(s − 2) + 4 2

0

t

-1/2

  ∵ L  t2  = 2    3   s   Ans. (d)

21. For a function g(t), it is given that +∞

The Fourier transform F(w) of f(t) is

∫−∞

(a) real and even function of w (b) real and odd function of w (c) imaginary and odd function of w (d) imaginary and even function of w  (GATE 2014, 1 Mark) Solution:  Since f(t) is odd and real,

g(t)e−jwt dt = we−2w for any real value w. If 2

y(t) = ∫ (a) 0

+∞

t

−∞

g(t)dt, then

∫−∞

j (c) − 2

(b) −j



y(t) dt is

(d)

j 2

(GATE 2014, 2 Marks)

f (t) = −f (−t)

∫

F (w ) is imaginary and odd [symmetry property

of Fourier transform]. Ans. (c)

Solution:  We are given ∞



g (t)⋅ e−jwt dt = we−2w

−∞ ∞

s 19. Laplace transform of cos(wt) is

s +w 2

−2t

Laplace transform of e

Chapter 11.indd 479

cos(4t) is

2

. The 2





g (t) dt = 0

−∞

8/22/2017 5:07:13 PM

480     TRANSFORM THEORY 

(a)

a−b s

(b)

es (a − b) s

(c)

eas − e−bs s

(d)

es(a −b) s

t



y ( t) =

g (t) dt ⇒ y (t) = g (t) ∗ u (t)

−∞

[u(t) in step function] ⇒ Y ( jw ) = G ( jw ) ⋅ U ( jw ) 



Y ( jw ) =



y (t) ⋅ e−jwt dt

−∞ ∞

(GATE 2015, 1 Mark) Solution:  We are given,

 2 ⇒ Y ( j0) = ∫ y (t) dt = we−2w   −∞ 1 w = 0 = = −j j

1 if a ≤ t ≤ b f (t) =  0 otherwise

 1   + pd (w )   jw   



L {f (t)} =

= 22. The unilateral Laplace transform of f(t) is 1 . Which one of the following is the uni2 s + s+1 lateral Laplace transform of g(t) = t ⋅ f(t)?

2s + 1

2

2

(d) (s + s + 1)2

a

b

b

e−st −s

b

=

−1 −bs − s−as ) (e s

a

a

e−as − e−bs = s

(s2 + s + 1)2

s (c)



= 0 + ∫ e−st dt + 0 =

(b)

(s2 + s + 1)2





e−st f (t)dt + ∫ e−st f (t)dt + ∫ e−st f (t)dt

0

−(2s + 1)

−s

f (t)dt

0 a

Ans. (b)

(a)

−st

∫e



Ans. (c)

2

(s + s + 1) 

25. The Laplace transform of f (t) = 2 t/p is s−3/2.

(GATE 2014, 2 Marks)

The Laplace transform of g(t) = 1/pt is

1

(a) 3s−5/2/2

(b) s−1/2

(c) s1/2

(d) s3/2

Solution:  F (s) = s2 + s + 1 d [F (s)] ds (using multiplication by t)

L[ g (t) = t ⋅ f (t)] = −

2s + 1 = 2

(s + s + 1)2 Ans. (d)



(GATE 2015, 1 Mark) Solution:  Given the t f (t) = 2 is s−3 /2 . p Given that g(f ) =

23. The result of the convolution x(-t) * d (-t - t0) is

transform

2 t/p = 2t

f (t) 2t ∞

1  f (t) 1 L{g(t)} = L  {f (t)}ds = ∫ s−3 /2ds =  2t  2 ∫ 2 s s 0



(GATE 2015, 1 Mark) Solution:  x(−t) ∗ d (−t − t 0) = x(−t) ∗ d (t + t 0) = x[−t(t + t 0)] = x(−t − t 0)

of

1 pt

⇒ g(t) =

(a) x(t + t 0) (b) x(t - t 0) (c) x(-t + t 0) (d) x(-t - t 0)

Laplace

=

∞ 1 1  s−3 /2 +1  1 −1 / 2  ] = s−1/2 =  = (−2)[0 − s 2  −3 / 2 + 1 2 s s

Ans. (b)

Ans. (d)

24. The bilateral Laplace transform of a function 1 if a ≤ t ≤ b f (t) =  is 0 otherwise

Chapter 11.indd 480

26. The Laplace transform of ei 5t, where i = −1, is s − 5i (a)

s2 − 25

s + 5i

(b) s2 + 25

8/22/2017 5:07:24 PM

SOLVED GATE PREVIOUS YEARS’ QUESTIONS     481

s − 5i

s + 5i (c)

s − 25



Solution:  X = AX

(d)

2

X(s) = (sI − A)−1 X(0)

2

s + 25



 s −1 −1 1     X(s) =   0 s + 1 0 1 / s  X(s) =   0 

(GATE 2015, 1 Mark) Solution: L(ei5t ) = L(cos 5t + i sin 5t) = L(cos 5t) + iL(sin 5t) s + 5i = 2 s + 25



1  x(t) =    0   1  y(t) = [0 1]   = 0  0  

Ans. (b)

27. Let x(t) = as(t) + s(-t) with s(t) = be-4tu(t), where u(t) is a unit-step function. If the bilateral Laplace transform of x(t) is 16 X(s) = 2 − 4 < Re{s} < 4, then the value of s − 16 b is . 



Ans. (d)

29. Consider a signal defined by e j 10 t x(t) =  0  Its Fourier transform is 2 sin(w − 10) (a) w − 10

(GATE 2015, 2 Marks) Solution:  We are given,

(c)

x(t) = as(t) + s(−t) and s(t) = be−4t u(t) ab s+4 L b be4t u(−t) → s−4

j10

(b) 2e

2 sin w w − 10

j10w

(d) e



x(t) = abe−4t u(t) + be4t u(−t)

for t ≤ 1 for t > 1 sin(w −10) w −10 2 sin w w

(GATE 2015, 2 Marks)

L

abe−4t u(t) →

Solution:  We are given e j10 t for t ≤ 1 x(t) =  0 for t > 1  Applying Fourier transform, we get

Therefore, ab b − s+ 4 s−4  a (s − 4) − (s + 4)  16 = ;−4 a; z
a a  1 1  (c) z > (d) z < min  a , a  a  

Chapter 11.indd 482

Solution:  The solution of Laplace function is called harmonic function and the following properties: The properties are true irrespective of the number of dimensions (one-, two- or three-­dimensions) in which we solve the equation for ∇2f = 0. 1 f(x, y, z) is the average of f values over a spherical surface of radius, R, which is centered at (x, y, z).

Solution:

L [f (t)] =

(GATE 2016, 1 Mark)

(GATE 2015, 2 Marks)

 1  f (x, y, z) =  ∫ fda  4pR 2  Sphere This is for the case of three-dimensional solution. In case of one-dimensional solution, f(x) is the average of f(x + a) and f(x − a). For any a, f (x) =

1 [f(x + a) + f(x − a)] 2

Similarly, for two-dimensional solution, the value f(x, y) is the average of values of f on any circle of radius R centered at the point (x, y).

8/22/2017 5:07:39 PM

SOLVED GATE PREVIOUS YEARS’ QUESTIONS     483

2 Laplace’s equation does not endure local minima or maxima. In general, the extreme values of f must occur at endpoints or boundary. Thus, we conclude that only the property in option (a) is true.

36. The solution of the differential equation, for t > 0, y ′′(t) + 2y ′(t) + y(t) = 0 with initial conditions y(0) = 0 and y ′(0) = 1, is [u(t) denotes the unit step function],

(a) te-t u(t)



(b) (e-t - te-t)u(t)



(c) (-e-t + te-t)u(t)



(d) e-tu(t)

Ans. (a) 2t

34. The Laplace transform of f(t) = e sin (5t)u(t) is 5

(a)

5

s − 4s + 29



(b)

2

2

s +5 

s−2

(c)



5 s+5

(d)

s2 + 4s + 29

(GATE 2016, 1 Mark) Solution:  Using Laplace transforms d2y



(GATE 2016, 1 Mark)

2dy +y = 0 dt dt 2 ′ [s Y (s) − sy(0) − y (0)] + 2[sY (s) − y(0)] + Y (s) = 0 ⇒ +



2

Solution:  For the given function e2t sin(5t)u(t), the standard form of Laplace transform is

⇒ Y (s) =

e sin (bt)u(t)   →



sY (0) + y ′(0) + 2y(0) s2 + 2s + 1

LT

at

As y ′(0) = 1, y(0) = 0 , we have 5

b =



(s − a)2 + b2

=

(s − 2)2 + 52

1

Y (s) = (s + 1)2

5 =



s + 4 − 4s + 25

⇒ y(t) = t ⋅ e−t u(t)

5 =

2

s − 4s + 29 2



Ans. (a)



Ans. (a)

37. If f(t) is a function defined for all t ≥ 0, its Laplace transform F (s) is defined as ∞ st

35. Consider a continuous-time system with input x(t) and output y(t) given by

(a)

∫0

(c)

∫0

∞ −st

e f (t) dt

(b)

∫0

(d)

∫0

∞ ist

y(t) = x(t) cos(t)

e

f (t) dt

∞ − ist

e f (t) dt

e

f (t) dt

This system is 

(a) linear and time-invariant



(b) non-linear and time-invariant



(c) linear and time-varying



(d) non-linear and time-varying

(GATE 2016, 1 Mark) Solution:  By definition of Laplace transform, we have ∞ L [f (t)] = ∫ e−st f (t)dt 0

Ans. (b) 

(GATE 2016, 1 Mark) Solution:  Let there be two inputs Ax1(t) and Bx2(t). Apply x(t) = Ax1(t) + Bx2(t)

38. Laplace transform of cos (w t) is ω s (a) 2 (b) 2 2 s + ω2 s +ω 2

s −ω

y(t) = [Ax1(t) + Bx2(t)] cos(t)



= Ax1(t) cos(t) + Bx2 (t) cos(t)



= y1(t) + y2(t)

ω

s (c)

(d)

2



2

s − ω2 (GATE 2016, 1 Mark) s

Solution:  L(cos ω t) = 2

s + ω2

Thus the function is linear. Let input be time shifted by t0. Then output

y(t) = x(t - t0)cos(t)

y(t) does not shift t0, hence it is time-varying.

Chapter 11.indd 483

Ans. (c).

Ans. (a)

39. Solutions of Laplace’s equation having continuous second-order partial derivatives are called (a) biharmonic functions (b) harmonic functions

8/22/2017 5:07:49 PM

484     TRANSFORM THEORY 

(c) conjugate harmonic functions (d) error functions 

X1(jw)

(GATE 2016, 1 Mark)

1 0.5

Solution:  Laplace’s partial differential equation is written as ∂2u

−1

∂2u +

∂x

0.3

2

0

2 w

1

=0 ∂y

2

X2(jw) Solving this equation, we would get two harmonics that are conjugates to each other. Therefore, option (c) is correct. Ans. (c)

1 0.5 0.3

40. The Laplace transform of the causal periodic square wave of period T shown in the figure below is

−2

f(t)

−1

0

1 w

Which one of the following statements is TRUE?

(a) x1(t) and x2(t) are complex and x1(t)x2(t) is also complex with non-zero imaginary part.



(b) x1(t) and x2(t) are real and x1(t)x2(t) is also real.



(c) x1(t) and x2(t) are complex but x1(t)x2(t) is real.



(d) x1(t) and x2(t) are imaginary but x1(t)x2(t) is real.

1

0

T/2

T

3T/2

2T

t

(a) F (s) =

1 1 + e − sT /2

(b) F (s) =

1 s(1 + e − sT /2 )

(c) F (s) =

1 s(1 − e − sT )

(d) F (s) =

1 1 − e − sT





Solution:  X1(jω) and X2(jω) are not conjugate symmetric. Therefore, x1(t)  × x2(t) are not real. Fourier transform of

(GATE 2016, 2 Marks)

Solution:  given as

Laplace transform for periodic signal is

1 1 − e − sT

∫ f(t)e

− st

dt



  T /2 1 1 (1)e − st dt + 0 = − sT  ∫ sT 1−e  0  1−e

=

1  1 − sT /2 )=   (1 − e (e − e − sT )  s 

 e − st   −s   0

symmetric,

so

Ans. (c)

T /2

  1 − e − sT /2  − sT /2 − sT /2  s ( 1 + e )( 1 − e )  

1 s(1 + e − sT /2 ) Ans. (b)

41. Suppose x1(t) and x2(t) have the Fourier transforms shown as follows.

Chapter 11.indd 484

1 X ( jw ) * X2 ( jw ) 2p 1

0

=

⇒ F (s) =

x1(t) ⋅ x2 (t) →

X1(jω) * X2(jω) is conjugate x1(t) × x2(t) will be real.

T

F (s) =

(GATE 2016, 2 Marks)

42. The output of a continuous-time, linear time-­ invariant system is denoted by T{x(t)} where x(t) is the input signal. A signal z(t) is called eigen-­ signal of the system T, when T{z(t)} = γz(t), where γ is a complex number, in general, and is called an eigenvalue of T. Suppose the impulse response of the system T is real and even. Which of the following statements is TRUE?

(a) cos(t) is an eigen-signal but sin(t) is not.



(b) cos(t) and sin(t) are both eigen-signals but with different eigenvalues.

8/22/2017 5:07:53 PM

SOLVED GATE PREVIOUS YEARS’ QUESTIONS     485



(c) sin(t) is an eigen-signal but cos(t) is not.



(d) cos(t) and sin(t) are both eigen-signals with identical eigenvalues.

Output sin(t) and cos(t) are eigen-signals with same eigenvalues.

Ans. (d) t



(GATE 2016, 2 Marks)

43. The Laplace transform of te is s (a)

Solution:  Since impulse response is real and even, H(jω) will also be real and even. Therefore,

( s − 1)

H(jω0) = H(-jω0)

1



cos(t) input =

e jt + e−jt input 2

L {tet(b) }=

2

1 (s − 1)2

(c)

( s + 1)

(d)

2

s s−1



(GATE 2017, 1 Mark)

Therefore, output Solution:

Chapter 11.indd 485

−jt

H( j1)e + H(−j1)e = H( j1)cos(t) 2 Similarly for sin(t), Output = H(j1)sint jt

=

L {tet } =

1 (s − 1)2 Ans. (b)

8/22/2017 5:07:56 PM

Chapter 11.indd 486

8/22/2017 5:07:56 PM

GENERAL APTITUDE

SECTION A VERBAL ABILITY

VA_Chapter 1.indd 487

22-08-2017 06:40:55

VA_Chapter 1.indd 488

22-08-2017 06:40:55

CHAPTER 1

ENGLISH GRAMMAR

This chapter will help improve your grammatical skills and hence the fluency with which you speak the language. It consists of basic theory of all the topics involved and multiple exercises, from very basic questions to advanced problems. The reader will be confident in solving any grammatical problem after solving the exercises given in the chapter.

ARTICLES An  article is a word  (an adjective to be precise) that combines with a noun to indicate the type of reference being made by the noun. Articles are broadly classified into two categories: 1. `A’ (or `an’) is called indefinite article, because it does not point out any definite or particular thing or person. 2. `The’ is called definite article, because it points out some definite or particular person or thing. Use of `A’ 1. Before a word beginning with a consonant sound: example, a man, a toy, a car, etc.

VA_Chapter 1.indd 489

2. Before a word beginning with the consonant sound of `yu’, though it may begin with a vowel letter: example, a university, a European, a useful thing, etc. 3. Before words with the sound of `wu’, even though it may begin with a vowel letter: example, a onerupee note, a one-legged dog, a one-eyed beast, etc. 4. Before a word beginning with a sounded h: ­example, a horse, a house, a holiday, etc. Use of `An’ 1. Before a word beginning with a vowel sound: ­example, an apple, an octopus, an arrow, etc. 2. Before a word beginning with h which is not pronounced and therefore the beginning sound of the word becomes a vowel sound: example, an hour, an heir, an honest boy, etc. 3. Before individual letters spoken with a vowel sound: example, an M.L.A., an S.P., an M.P., etc. Some common examples of the use of a (or an) are as follows:

1. Give me a mango. 2. I saw an elephant. 3. A dog is a faithful animal. 4. Two of a trade seldom agree.

22-08-2017 06:40:55

490     English Grammar 



5. David won a price of a hundred thousand rupees. 6. Mark always travels by an aeroplane. 7. What a long queue? 8. It is a rainy day.



Example: Alexander the Great Louis the fifth Henry the second

Use of `The’

9. Before a noun when special emphasis is required.

1. When we speak of a particular person or thing, or one already mentioned or one well-known to us.



Example: He is playing with the racquet I gave him. Pass me the cup on the table. Clear the house. 2. When a singular noun or an adjective of quality is meant to represent a whole class. Example: The lion is the kind of beasts. The rich are not always happy. The Chinese have a very high I.Q. 3. Before the names of rivers, seas, oceans, countries, important geographic names, etc. Example: The Ganga The Sahara desert The Highlands 4. Before titles of books. Example: The Gita The Quran The Ramayana 5. Before the names of ships, aeroplanes, well-known buildings and newspaper. Example: The Eiffel tower The Times of India The Red Fort

Example: This is just the opportunity I was looking for. Now is the time to act. He is the undisputed king.

10. Before the name of a nation and sometimes communities. Example: the English the Hindus the Indians 11. Before an adjective in the comparative degree when not more than two persons or things are being compared.

Example: Raj is the taller of the two. Elizabeth is the smarter of the two. Jack is the tougher of the two.

12. Before numeral adjectives showing order.

Example: All students of the second year were on a holiday. All employees on the third floor are lawyers. All the good football teams play in the first division.

13. Before a Proper Noun preceded by a more or less permanent adjective. Example: The late Mr Rajesh Khanna The beautiful Mrs Dawson The clever twins 14. In other fixed phrases.

6. Before superlatives.

Example: He is the oldest man alive. He is the youngest cricketer to score a century. He is the fastest runner in the world.



Example: On the other hand To the contrary All the more

7. Before the names of important events.

Exercise 1.1

Example: The French Revolution The Jallianwala Bagh incident The World War

Complete the following sentences using suitable definite or indefinite article in the blank.

8. Before an epithet attached to a personal proper name.

VA_Chapter 1.indd 490

1. I would like to buy couple of shirts and jeans I just tried. (a, the) 2. It has been almost hour since he left. (an)

22-08-2017 06:40:55

NOUN     491

3. Jerry drew a beautiful picture of mountain, similar to one we went to last summer. (a, the) 4. This man is Einstein of this age. (the) 5. Kshitij had appointment with the doctor. (an) 6. Delhi is capital of India. (the) 7. apple a day, keeps doctor away. (An, the) 8. Americans helped poor kid to cross the road. (The, a) 9. fire broke out in apartment taking lives of ten innocent people. (A, an, the) 10. Nikhil is sweet boy. (a)

NOUN A noun is a word which names a person, place, animal, action, quality, feeling or anything that we can think of. Nouns are classified into five categories: 1. Proper noun: They are used to denote a particular person, place or thing. Example: India, Jack, the Ganga. 2. Common noun: They are used to denote a class of objects. Example: desk, chair, man.





(a) Collective noun:  They are used to denote several persons or things regarded as one group. Example: jury, army, class. (b) Abstract noun:  They are used to denote a quality or action of a person, something which we cannot see or touch.    Example: kindness, honesty, arrogance. (c) Material noun:  They are used to denote a substance of which things are made.    Example: Coal, Gold, Wool.

Use of Nouns in Singular Form Some of the nouns which are always used in singular form are as follows:

1. Furniture 2. Fuel 3. Bread 4. Mathematics, accounts, etc. 5. Words like dozen, score, hundred, etc. when preceded by a numeral. 6. Expressions like a five-year plan, a ten-man jury, etc. Example: The furniture of my house is very costly. Jack picked up a five hundred rupee note. Mathematics is the mother of science.

VA_Chapter 1.indd 491

Use of Nouns in Plural Form Some of the nouns which are always used in plural form are as follows:

1. Army, police, etc. 2. People 3. Scissors 4. Trousers, pants 5. Spectacles 6. Goods

Example: The police have caught the terrorist. Where are my spectacles? David’s trousers are very costly.

Conversion of Nouns from Singular to Plural Ending

Singular

Plural

um-ia

Medium

Media

on-a

Phenomenon

Phenomena

is-es

Hypothesis

Hypotheses

a-ae

Antenna

Antennae

Us-i

Radius

radii

Ex/ix-ices

Matrix

Matrices

o-i

Concerto

Concerti

Collective Nouns Some of the most commonly used collectively nouns are given below: 1. A class of students. 2. An army of soldiers. 3. A choir of singers. 4. A crew of sailors. 5. A band of musicians. 6. A gang of thieves. 7. A group of dancers. 8. A team of players. 9. A troupe of artists/dancers. 10. A staff of employees. 11. A regiment of soldiers. 12. A panel of experts. 13. A flock of tourists. 14. A board of directors. 15. A catch of fish. 16. An army of ants. 17. A flight of birds. 18. A haul of fish. 19. A flock of sheep. 20. A herd of deer/cattle/elephants/goats/buffaloes. 21. A troop of lions.

22-08-2017 06:40:56

492     English Grammar 

22. A pack of wolves. 23. A litter of puppies/kittens. 24. A swarm of bees/ants/rats/flies. 25. A murder of crows. 26. A kennel of dogs. 27. A galaxy of stars. 28. A stack of wood. 29. A fleet of ships. 30. A string of pearls. 31. An album of stamps/autographs/photographs. 32. A library of books. 33. A basket of fruit. 34. A bowl of rice. 35. A pack of cards. 36. A pair of shoes. 37. A bouquet of flowers. 38. A bunch of keys. 39. A range of mountains. 40. A cloud of dust.

Exercise 1.2

Pronouns are classified as follows:

1. Personal 2. Reflexive and Emphatic 3. Demonstrative 4. Indefinite 5. Distributive 6. Reciprocal 7. Relative 8. Interrogative

Personal Pronoun I, we, you, he, etc., are called personal pronouns. They can be used for first person, second person and third person. First person (masculine or feminine) Singular

Plural

Nominative

I

We

Possessive

my, mine

our, ours

Accusative

Me

Us

A.  Point out the nouns in the following sentences. 1. Mount Everest is the highest peak in the world. 2. Cheetah is a very fast animal. 3. Himani and Shivani formed a deep and lasting friendship. 4. Jawaharlal Nehru was the 1st Prime Minister of India. 5. Div saw a large tea garden in Assam.

Second person (masculine or feminine)

B.  Fill in the blanks using suitable nouns.

Third person

1. There is so (many, much) smoke coming out of that building. (much) 2. He threw (a little, some) pebbles in the river. (some) 3. Akul saw (a large amount of, many) cows grazing the field. (many) 4. Nupur uses only (a little, a few) cooking oil in her cooking. (a little) 5. Karan ate a (group, bunch) of grapes today. (bunch)

PRONOUN A pronoun is a word used in place of a noun.

Singular

Plural

Nominative

you

You

Possessive

Your

Yours

Accusative

You

You

Singular

Plural

Masculine Feminine Neutral All Genders Nominative He

she

Possessive

her, hers its

their, theirs

Her

Them

His

Accusative Him

it

it

They

Reflexive and Emphatic Pronoun When `-self’ is added to my, your, him, her, it and `-selves’ to our, your, etc. then such pronouns are called compound personal pronouns. They are called reflexive pronouns when an action done by the subject reflects upon the subject.

Example: Example: She hurt herself. David is a good student. David gets straight A’s in all the subjects. David is a good student. He gets straight A’s in all the subjects.

VA_Chapter 1.indd 492

If compound personal pronouns are used for the sake of emphasis, they are called emphatic pronouns. Example: He himself admitted his guilt.

22-08-2017 06:40:56

PRONOUN     493

Demonstrative Pronoun

Interrogative Pronoun

Pronouns that are used to point out the objects to which they refer are called demonstrative pronouns.

Interrogative pronouns are used when you want to ask a question.

Example:

Example:

Both the vehicles are good, but this is better than that.

What are you both talking about?

This   shirt is better than that shirt.

Who is the culprit?

My views are quite in accordance with those of the University Board.

Which colour did you choose for your car?

Use of Pronouns

Indefinite Pronoun These are pronouns that refer to person or things in a general way, but do not refer to any person or thing in particular. Example: One hardly knows what to do. Somebody has stolen his car. Some are born with a golden spoon.

Distributive Pronoun Each, either, neither are called distributive pronouns because they refer to persons or things one at a time. Example:

1. The pronoun `One’ must be followed by `one’s’. 2. `Everyone’ or `Everybody’ must be followed by `his’. 3. `Let’ is followed by pronoun in the objective case. 4. `But’ and `except’ are followed by pronoun in the objective case. 5. Reflexive pronouns are never used with verbs keep, conceal, quality, spread, rest, stay. 6. `Who’ denotes subject and `whom’ denotes object. 7. `Whose’ is used for persons and `which’ for lifeless objects. 8. `Which’ conveys additional information and `that’ explains a certain thing. 9. `Each other’ is used for two and `one another’ for more than two. 10. When the same person is the subject and object, it is necessary to use reflexive pronouns.

Either of them couldn’t pass the examination. Neither of them were guilty of the crime they are convicted of.

Exercise 1.3

Each of the girls got a chocolate. Fill in the blanks with suitable pronouns.

Relative Pronoun Words like who, whom, which, etc. that refer or relate to some noun are called relative pronouns. The different forms of relative pronouns are: Singular and Plural Nominative

:

who

Genitive

:

whose

Accusative

:

whom

Example: This is the boy who plays well. This is the girl whose work is good. This is the teacher whom all praise.

VA_Chapter 1.indd 493

1. This is the boy (who, which, whose) comes from England. (who) 2. My friends enjoyed (himself, themselves, herself) very much at the movie. (themselves) 3. Mannan, did you do the maths homework (yourself, himself, itself)? (yourself) 4. The man, (who, which, whose) father is a scientist, forgot his phone. (whose) 5. Ram and Shyam haven’t met (ourselves, themselves, each other) for a long time. (each other) 6. What did you do with the money (who, which, whose) I lent you? (which) 7. I bought (me, myself, each other) a new camera. (myself) 8. John is driving car. (he) 9. Susan is a good dancer. is the best in the crew. (she) 10. Neha and Anisha are sisters. fight a lot. (they)

22-08-2017 06:40:56

494     English Grammar 

ADJECTIVE Any word which describes the quality or nature or quantity of a noun, is called an adjective. Example: Arun is a good boy. I asked you to get ten bananas.

5. When two changes happen together, comparative degree is used in both. Example: The closer you get to the sun, the hotter it gets. 6. When comparative degree is used in superlative sense, it is followed by `any other’. Example: Sachin is better than any batsman. (✕) Sachin is better than any other batsman. (✓)

Adjectives can be classified as follows: 1. Adjectives of Quality: They are used to describe the quality of the noun. Example: Large, small, good, bad, etc. 2. Adjectives of Quantity: They are used to describe the quantity (how much) of the noun. Example: Little, some, more, etc. 3. Adjectives of Number: They are used to describe the number of the noun. Example: Ten apples, twenty-five tons of water, etc. 4. Demonstrative Adjectives: They are used to answer the question `Which’. Example: This, that, these, those, etc. 5. Interrogative Adjectives: They are used to ask a question about the noun. Example: What, which, whose, etc.

Use of Adjectives

7. When two or more comparatives are joined by `and’, they must be in the same degree. Example: Jack is one of the most handsome and wisest men of his family. 8. If comparisons are made by using `other’, `than’ is used instead of `but’. Example: He turned out to be no other than my old classmate. Exercise 1.4 Point out the adjectives in the following sentences. 1. Bani got good marks in all the papers. 2. Jack is a better painter than Jill. 3. This is the article in question. 4. Alexander was a great conqueror. 5. Naina ate three apples. 6. Which place did you visit? 7. Pele is one of the best footballers ever. 8. Who was driving the car? 9. He has little knowledge about the accident. 10. Anupam is the best criminal lawyer.

1. The adjectives ending like prior, junior, senior, superior, etc. take `to’ and not `than’ after them. Example: He is senior to me. 2. Some adjectives like unique, ideal, perfect, extreme, complete, infinite, etc. are not compared. Example: It is the most unique pen. (✕) It is a unique pen. (✓)

PREPOSITION A preposition is a word used to relate a noun or a pronoun to some other word in a sentence. Example: David is at the top of his career. You should be home by midnight.

3. Double comparatives and double superlatives must not be used. Example: He is more wiser than his brother. (✕) He is wiser than his brother. (✓) 4. When two adjectives in superlative or comparative degree are used together, the one formed by adding `more’ or `most’ must follow the other adjective. Example: He is more intelligent and wiser than his brother. (✕) He is wiser and more intelligent than his brother. (✓)

VA_Chapter 1.indd 494

Prepositions can be used in the following manner:

1. Preposition of time 2. Preposition of position 3. Preposition of direction 4. Other uses of preposition

Preposition of Time 1. `At’ is used with a definite point of time or with festivals.

22-08-2017 06:40:56

PREPOSITION     495



Example: I will reach there at 6 p.m.

4. `For’ denotes destination. Example: Amit is leaving for Johannesburg tomorrow.

2. `In’ is used with parts of the day, months, years and with the future tense. Example: Rohit plays in the afternoon.

5. `From’ denotes departure point. Example: Nupur’s father is coming back from Chandigarh.

3. `On’ is used with dates and days. Example: Inu was born on 30th October.

6. `Into’ denotes motion towards the inside of something. Example: Prince fell into a hole.

4. `By’ refers to the latest time at which an action will be over. Example: The movie will be over by 9 p.m.

7. `Off’ refers to separation. Example: He took the bandage off his hand.

5. `For’ is used with perfect continuous tense showing the duration of action. Example: I have been living in my house for twenty years. 6. `Since’ is used with the point of time when action begins and continuous. Example: We haven’t met since 5 years.

Other Uses of Preposition 1. `About’ shows nearness. Example: I am about to reach. 2. `After’ refers to sequence. Example: She came just after her husband.

7. `From’ refers to the starting point of action. Example: From rags to riches.

3. `Across’ means from one side. Example: Rahul shouted my name from across the road.

Preposition of Position

4. `Before’ stands for `in front of’. Example: Aman stood before his father and begged for his forgiveness.

1. `At’ refers to an exact point. Example: My cousin stayed at my house. 2. `In’ refers to lager areas. Example: Raj stays in Delhi. 3. `Between’ is used for two persons or two things. Example: Things are fine between Jack and Jill. 4. `Among’ is used for more than two persons and `amongst’ is used before the words which start with a vowel. Example: Charlie is the shortest amongst us. 5. `Above’ is used for higher than, `under’ is used for below. Example: The traveller rested under the tree.

Preposition of Direction 1. `To’ or `towards’ is used to express motion from one place to another. Example: My father walked towards me. 2. `At’ refers to aim on a target. Example: The hunter aimed at the deer and fired. 3. `Against’ shows pressure. Example: The thief pressed the knife against her neck.

VA_Chapter 1.indd 495

5. `Beside’ means `by the side of’. Example: A couple should stand beside each other through tough times. 6. `Besides’ means `in addition to’. Example: I cannot come, besides I am not even interested in the event.

Exercise 1.5 Complete the following sentences by use of the suitable prepositions in the blank.

1. Gautam reached (at, on) 6 pm sharp. (at) 2. Kshitij lives (at, on) my house. (at) 3. My father used to live (at, in) Punch. (in) 4. The racer was quickly reaching (towards, on) the finishing line. (towards) 5. The boy fell (in, into) a very deep hole. (into) 6. Mahatma Gandhi was born (at, on) 2nd October. (on) 7. Naina is the fairest (among, amongst) all of us. (amongst) 8. We haven’t met (since, from) 10 years. (since) 9. We are leaving (for, to) the office. (for) 10. (for, from) rags to riches. (from)

22-08-2017 06:40:56

496     English Grammar 

VERBS



A verb is a word that tells us something about a person or thing that is an action being done by the person/ thing, the action being done on the person/thing or state of being. Verbs are of two types: 1. Transitive Verbs: They are verbs which involve a direct object. Example: The man is reading a book. He is driving a car. He spoke the truth.

Example: The doctor and nurse work together.

3. A singular subject and a plural subject joined by `or’ or `nor’ will take a singular or plural verb depending upon which subject is nearer the verb. Example: Neither Rahul nor his friends are coming. Neither his friends nor Rahul is coming. 4. If two subjects are joined together by `as well as’ or `with’ the verb will act according to the first subject. Example: Students as well as the teacher are playing. Jack with his sons is going for a drive.

In the above examples, reading, driving and spoke are the verbs while book, car and truth are the objects. 2. Intransitive Verbs: They are verbs which do not involve a direct object. Example: The man is reading quickly. He drives very cautiously. He speaks softly.

5. Indefinite pronouns such as someone, somebody, each, nobody, anybody, everybody, everyone, either, neither etc. always take a singular verb.

Example: Somebody disturbs Neena every night. Everybody fails at least once in life.

6. Indefinite pronouns which indicate more than one (several, few, both, many) always take plural verbs.

The infinitive is the base of a verb, often preceded by `to’. When `ing’ are added to verbs, they are called gerunds.

Example: All the brothers are coming together. Both the books required careful reading.

Verbs can be used in active or passive voice. A verbs is said to be in the active voice when its form shows that the person (or thing) denoted by the subject does something. Similarly, a verb is said to be in the passive voice when its form shows that something is done to the person (or thing) denoted by the subject. Example:

7. Collective nouns (groups like army, jury, panel, committee, etc.) take singular verbs when the group works together and take plural verbs when the members of the group work alone.

1. Tom loves Jerry. Jerry is loved by Tom.

(Active) (Passive)

Example: The jury has given the verdict. The jury have argued for three hours. 8. Titles take singular verbs.

2. Who did this? (Active) This was done by whom? (Passive) 3. He kicked the football. The football was kicked by him.

(Active) (Passive)

Use of Verb 1. Singular subjects have singular verb and plural subjects have plural verb.

Example: He writes. They write.

2. Two subjects joined by `and’ will always have a plural verb whereas two singular subjects joined by `or’ or `nor’ will take a singular verb.

VA_Chapter 1.indd 496

Example: Angels and Demons is a very good book. The Avengers is a good movie. 9. If the subject begins with `The number of’ use a singular verb and if the subject begins with `A number of’ use a plural verb. Example: The number of balls is less. A number of balls are missing.

Use of Infinitives 1. Verbs such as learn, remember, promise, swear, refuse, try, care, hope, love etc. are followed by infinitive.

22-08-2017 06:40:56

TENSES     497



Example: The policeman refused to obey the bizarre order of his superior. I would love to join you. 2. Verbs such as order, tell, invite, oblige, allow, instruct, advise, remind, etc. are followed by object and infinitive. Example: Jake allowed his son to play. The newlywed couple invited their neighbours to eat. 3. Verbs or expressions like- will, can, do, must, may, let, etc. are followed by infinitive without `to’. Example: Let the witness be excused. He must finish this work himself. 4. Expressions like would rather, sooner than later, sooner than, rather than, had better, etc. are followed with infinitive without `to’. Example: I would rather travel alone. You had better help him. 5. The infinitive is used after adjectives like delight, angry, glad, excited, etc.

Example: I was delighted to hear your result. I am glad to be back.

6. The verb `know’ is never directly followed by the infinitive. It is followed by a conjunction and then the infinitive. Example: Do you know how to drive a car? I didn’t know how to act until you taught me.

Use of Gerunds 1. When an action is being considered in general sense, gerund is used as subject. Example: Painting is his favourite hobby. 2. Gerund is used as subject in short prohibitions. Example: Drinking and smoking here is prohibited. 3. Verbs such as help, stop, detest, avoid, finish, dread, mind, prevent, dislike, risk, deny, recollect, no good, no use, suggest, etc. are followed by the gerund. Example: Let me know when you finish studying. 4. If there is a sense of dislike, hesitation, risk, etc. in a sentence, use gerund. Example: There is always a risk when driving fast.

VA_Chapter 1.indd 497

Exercise 1.6 Fill in the blanks by choosing the suitable verb. 1. The number of balls (are, is) less.  (is) 2. They (should be, should have) finished the job by now.  (should have) 3. One should not drink and (drive, drove). (drink) 4. The earth (moves, moved) around the sun. (moves) 5. A number of kids (are, is) arriving.  (are) 6. He (has fallen, fell) asleep while drinking. (fell) 7. I (will, know) him from a long time.  (know) 8. He (is driving, was driving) fast when the accident happened.  (was driving) 9. I would love (to join, joining) you.  (to join) 10. Everybody (fails, failed) at least once in life. (fails)

TENSES The changed forms of a verb that indicate time of the action are called tenses of the verb. The word tense comes from the Latin tempus, meaning time. Tenses are of three types which can further be classified as: 1. Past Tense: A verb that refers to something that has passed or already taken place is said to be in the Past Tense. Example: I played.   I was playing.   I had played.   I had been playing. 

(Simple Past) (Past Continuous) (Past Perfect) (Past Perfect Continuous)

2. Present Tense: A verb that refers to the present or something happening at the moment is said to be in the Present Tense. Example: I eat.   I am eating.   I have eaten.   I have been eating.  

(Simple Present) (Present Continuous) (Present Perfect) (Present Perfect Continuous)

3. Future Tense: A verb that refers to the future or something that is going to happen is said to be in the Future Tense. Example: I shall sleep.    (Simple Future) I shall be sleeping.    (Future Continuous) I shall have slept.    (Future Perfect) I shall have been sleeping. (Future Perfect Continuous)

22-08-2017 06:40:56

498     English Grammar 

Use of Tenses 1. When the verb in the principal clause is in the past tense, the verbs of the subordinate clauses should be in the past tense. However, an exception can be made if the latter expresses universal or habitual truth.

Example: She said that he had finished her assignment. He said that he had left from his office.

2. Any tense may be used in the sub-ordinate clause if it gives a comparison by using the word `than’.

Example: Tim preferred sandwich more than he prefers burgers. The teacher likes Priya more than she likes Pooja. 3. Any tense can be when the subordinate clause is in a quotation.

Example: I said, “I am going to be busy today.” I told you, “I am going to Jammu tomorrow.”

2. You (do) what you had to.  (did) 3. Tara did (manage) to persuade her husband for a vacation.  (manage) 4. If it hadn’t been for the bad weather, the plane would have (take) off at 5 p.m. (taken) 5. What goes around, (come) around.  (comes) 6. He preaches as if he (is) a saint.  (were) 7. My father has just finished (read) the ­newspaper.  (reading) 8. I wish there (is) no war.  (was) 9. He (say), “He won’t be able to come to the party.”  (said) 10. The teacher (like) Ram more than she (like) Raj.  (likes)

ADVERBS A word that modifies the meaning of a verb, an adjective or another adverb is called an adverb. Example: He strokes the horse gently.

4. With the phrase `as if’ and `as though’, past tense and plural form must be used.

The strawberries were very sweet. Bolt runs quickly.

Example: He preaches as if he were a saint. He boasts as though he were the best in everything. 5. Words like usually, generally, often, whenever, etc. are used in present indefinite tense.

Adverbs can be classified into the following according to their meaning (though one adverb may belong to more than one class depending upon how and where they are used): 1. Adverbs of Time: They show when.

Example: I usually eat my lunch early. We will go whenever you want to go.

Example: I have heard that song before. Edward is always late for his class.

6. Present perfect tense should be used if the action began in the past and is still continuing in the present.

Example: He has been taking wickets since 5 matches. She has been preparing for the exam since 3 months.

7. Past tense should be used with expressions like suppose that, it is high time, it is time, as if, etc.

Example: It is time you went home. Suppose that I played with you.

2. Adverbs of Frequency: They show how often an action is done. Example: The policeman warned twice before shooting. Barking dogs seldom bite. 3. Adverbs of Places: They show where the action is to be performed or described. Example: The murder took place here. Rose left her family and went away.

Exercise 1.7 Read the following sentences and fill in the blanks using the right tense. 1. I didn’t have any idea what the man (said)

VA_Chapter 1.indd 498

(say).

4. Adverbs of Manner: They show how or in what manner the verb is used. Example: He works diligently. The soldiers fought fiercely.

22-08-2017 06:40:56

PRACTICE EXERCISE     499

5. Adverbs of Degree or Quantity: They show how much or the degree/extent to which the verb is used.

Exercise 1.8

Example: He was too careless. He couldn’t be more precise.



6. Adverbs of Affirmation and Negation: Example: I am certain that he is the killer. I am not sure how long it will take to reach. 7. Adverbs of Reason: Example: He was therefore expelled from school. His efforts were hence futile.

Point out the adverbs in the following sentences. 1. He finished his work quickly. 2. Barking dogs seldom bite. 3. James thought twice before answering. 4. Federer lost because he was too complacent. 5. The policeman was certain that the thief will steal again. 6. Roger was caught stealing and was therefore fired from the office. 7. Bill was late for his assessment. 8. The soldiers fought bravely. 9. Jennifer’s husband left her and went away. 10. Jacob is a shy kid. He hardly has any friends.

PRACTICE EXERCISE Directions for Q. 1 to Q. 15

5. (a) H  e was travelling with his wife, who was also his business partner. (b) Fortune favours the brave. (c) He shall not be forgiven. (d) No error.

Read each sentence and spot whether there is any error in any of the parts. There would be only one sentence which will have an error or none at all and the sentence will have a maximum of one error. 1.

6. (a) I will be studying till late. (b) John bought some mangoes. (c) He spoke as if he was an expert. (d) No error.

2.

(a) Raj is better than any player. (b) A mother always loves her children equally. (c) Juice is sold by the litre. (d) No error. 7.

(a) He didn’t knew what he was doing. (b) I will pick you up at noon. (c) Rahul is an employee. (d) No error.

(a) He was raised among orphans. (b) I will reach home at 7 o’clock. (c) He went to a city where he was born. (d) No error. 8.

3.

(a) T  he person which sat next to me was a sportsperson. (b) The pen which Arun carried was very expensive. (c) He who must not be named has returned. (d) No error.

(a) Where there is a will, there is a way. (b) Dave got me a pair of trouser. (c) It is a beautiful night. (d) No error. 4.

9. (a) The more the merrier. (b) He is much more gentler than his brother. (c) Pass me a pair of scissors.’ (d) No error.

VA_Chapter 1.indd 499

(a) God helps those, who help themselves. (b) He is as old as I am (c) May he rest in peace. (d) No error.

22-08-2017 06:40:56

500     English Grammar 

10.

(c) while I (d) am working. (a) He is wiser than his brother. (b) He is wiser and more intelligent than his brother. (c) He is more intelligent and wiser than his brother. (d) No error.

18. (a) He is (b) an oldest (c) person of (d) the house.

11. (a) We saw a movie (b) We had a good play of cricket. (c) We liked the movie. (d) No error.

19. (a) Leo is (b) more taller (c) than anybody else (d) in the class.

12. (a) What can I do you for? (b) What can I do for you? (c) What shall I do? (d) No error

20. (a) Neither Kshitij (b) nor his friends (c) is coming (d) to the party.

13. (a) These all mangoes are ripe. (b) The oranges were juicy. (c) An apple a day keeps the doctor away. (d) No error.

21. (a) I bought me (b) a new (c) camera for my (d) birthday.

14. (a) Who are you and what do you want? (b) Which kind of a person does that? (c) What kind of a person does that? (d) No error

22. (a) Women in (b) India are more (c) self employed (d) then men.

15. (a) I have hurt my hand. (b) Give me little sugar. (c) Can I borrow some money? (d) No error.

23. (a) He boasts (b) as though (c) he is the best (d) at everything.

Directions for Q. 16 to Q. 30 Each sentence is broken down into four parts. Read each sentence and spot the part which has an error. Ignore errors of punctuation.

24. (a) Divya didn’t (b) knew what (c) she was (d) doing.

16. (a) Every person (b) in the (c) sinking boat (d) have been saved. 17. (a) Somebody disturb (b) me every night

VA_Chapter 1.indd 500

25. (a) The jury (b) have (c) agreed to (d) a verdict.

22-08-2017 06:40:56

PRACTICE EXERCISE     501

(c) There are more than two errors. (d) No errors.

26. (a) I have been (b) living in (c) this house (d) till twenty years. 27. (a) It is the most (b) unique object (c) I have ever (d) seen before.

34. It’s a most unique pen. My best friend gave it to me and didn’t took any money for it. (the, take) (a) There is one error. (b) There are two errors. (c) There are more than two errors. (d) No errors. 35. He is taller then his younger brother but in all his friends, he is shorter. (than, among, shortest)

28. (a) I reached (b) on 6 pm (c) but my brother (d) was late.

(a) There is one error. (b) There are two errors. (c) There are more than two errors. (d) No errors. 36. I loved the dog but my father told me that we won’t keeping it. (won’t be keeping)

29. (a) As of this morning, (b) none of my friends (c) have been able (d) to solve the puzzle.

(a) There is one error. (b) There are two errors. (c) There are more than two errors. (d) No errors.

30. (a) Jim is (b) more faster (c) than his (d) brother Tim.

37. “Will you marry me?” he asked me as we both eat our lunch. (ate) (a) There is one error. (b) There are two errors. (c) There are more than two errors. (d) No errors.

Directions for Q. 31 to Q. 40 Read the following sentences and answer accordingly; avoid mistakes of punctuation. 31. He is more intelligent and wiser than his elder brother. (wiser and more intelligent) (a) There is one error. (b) There are two errors. (c) There are more than two errors. (d) No errors. 32. Hazards of life cannot be negated but they can be quite effortlessly evaded. (a) There is one error. (b) There are two errors. (c) There are more than two errors. (d) No errors. 33. Ram has the most best collection of art. (best) (a) There is one error. (b) There are two errors.

VA_Chapter 1.indd 501

38. A beast is an aggressive creature by nature. It is a hard fact of life and undeniable. (a) There is one error. (b) There are two errors. (c) There are more than two errors. (d) No errors. 39. Manoj’s father is a M.P. He will stand for re-­ election after a year. (an, an) (a) There is one error. (b) There are two errors. (c) There are more than two errors. (d) No errors. 40. The students were advised to follow the instructions to the examiner. (of) (a) There is one error. (b) There are two errors. (c) There are more than two errors. (d) No errors.

22-08-2017 06:40:56

502     English Grammar 

ANSWERS 1. (c)

7. (c)

13. (a)

19. (b)

25. (d)

31. (a)

37. (a)

2. (a)

8. (a)

14. (b)

20. (c)

26. (d)

32. (d)

38. (d)

3. (b)

9. (d)

15. (b)

21. (a)

27. (a)

33. (a)

39. (b)

4. (b)

10. (c)

16. (d)

22. (d)

28. (b)

34. (b)

40. (a)

5. (d)

11. (b)

17. (a)

23. (c)

29. (c)

35. (c)

6. (a)

12. (d)

18. (b)

24. (b)

30. (b)

36. (a)

VA_Chapter 1.indd 502

22-08-2017 06:40:57

CHAPTER 2

SYNONYMS

A synonym is defined as the word having the same or nearly the same meaning as another word in the same language. This chapter provides the reader with the list of some of the important synonyms which are commonly used followed by an exercise towards the end, but first we discuss some of the techniques to help solve the questions related to synonyms.

TIPS TO SOLVE SYNONYM BASED QUESTIONS Questions based on synonyms and antonyms can be pretty difficult to answer if the word is new or unheard of. In such scenarios, there are certain tips or tricks that the student can use in order to increase his chances of getting the correct answer. Some of the common tips are given as follows: 1. Try to define the stem word. Example: tempestuous — related word is temper, severance — related word is sever, etc.

VA_Chapter 2.indd 503

2. Put the word in context. Putting the word in a sentence or a phrase makes it easier to guess the meaning. Example: gratuitous — “gratuitous violence” — means unnecessary, requiem — “requiem for a heavyweight” means rest. 3. When you have a word, try to guess all its synonyms and from the answers see the exact word which has the same meaning and the opposite meaning for antonym. Example: Which of the following is a synonym of abundance? (a) Ampleness (b) Redundant (c) Donation (d) Augmentation Now, we know that the meaning of the word “abundance” is something which is large or sufficient in quantity. Hence, we are looking for a word like affluent or ample. Hence, the correct answer is option (a), ampleness.

22-08-2017 06:41:43

504        Synonyms 

4. Test the word for positive or negative connotations. (a) As a matter of general notation, words that include the concept of going up are usually positive, while those that include the concept of going down are usually negative. (b) Any word that starts with `de’, `dis’ or `anti’ is usually negative. Example: Some of the positive connotations are elevate, ascend, adulation, etc. Some of the negative connotations are decline, suborn, derision, etc.

Accord:

agree, concur, conform, harmonize.

Accustom:

acclimatize, acquaint, adapt, familiarize.

Acrid:

caustic, harsh, irritating, pungent.

Adamant:

determined, firm, resolute, stubborn.

Adept:

dexterous, masterful, expert, genius.

Adhere:

cleave, cling, obey, observe.

Adroit:

clever, cunning, ingenious, quick-witted.

Adversity:

affliction, calamity, difficulty, misfortune.

Aggravate: annoy, increase, infuriate, intensify.

5. Some words can be used as both nouns and verbs. If the given word is used as a verb, then all of the answer choices will also be verbs. This helps the student to quickly determine if the word is being used in a secondary sense, as common words have different meanings, if they are used as verbs, nouns or adjectives. Hence, never overlook the rare meanings of words and always know the part of the speech the word is used in. Example: Some words commonly used as both nouns and verbs are curb, document, table, air, bustle, etc.

Alleviate:

ease, lighten, mitigate, pacify.

Amenable:

agreeable, favourable, manageable, obedient.

Anguish:

discomfort, distress, pain, sorrow.

Arrogant:

disdainful, imperious, pompous, scornful.

Astonish:

amaze, confound, overwhelm, surprise.

Atrocious:

awful, bad, rotten, shocking.

6. Use the following guessing strategies.

Audacious: courageous, daring, dauntless, rash. Austere:

forbidding, grave, grim, harsh.

Averse:

antipathetic, disinclined, hostile, unwilling.

Avid:

avaricious, eager, passionate, zealous.

(a) Eliminate answer choices that have no clear opposite. (b) If two (or more) choices have the same meaning, eliminate both.

B

Example: What is the synonym of Jovial?

Baffle:

confound, confuse, deceive, muddle.

(a) (b) (c) (d)

Banal:

common, conventional, ordinary, plain.

Barren:

depleted, desolate, unfertile, unfruitful.

Berate:

censure, criticise, disapprove, reprimand.

Bastion:

citadel, defence, fortress, mainstay.

Betray:

abandon, deceive, deceit, fool.

Bias:

bent, inclination, predisposition, unfair.

Bitter:

acerb, acrid, harsh, sour.

Miserable Gloomy Cheerful Depressed

Now, we can see that words miserable, gloomy and depressed are similar in meaning. Hence, we eliminate these options and we are left with option (c) which is the right answer, cheerful. A

Blaspheme: abuse, curse, profane, swear.

Abandon:

desert, discard, evacuate, leave.

Blend:

amalgam, combine, mix, union.

Abduct:

kidnap, snatch, seize.

Bliss:

beatitude, gladness, happiness, joy.

Abet:

aid, assist, help, support.

Bluff:

betray, counterfeit, feign, mislead.

Abeyance: adjournment, postponement, intermission, recess.

Bold:

adventurous, daring, dauntless, fearless.

Bother:

annoy, concern, fuss, irritate.

Abide:

accept, bear, endure, reside.

Breach:

aperture, break, estrangement, infringement.

Abstain:

avoid, decline, renounce, refrain.

Brief:

abstract, concise, short, summary.

Abundance: affluence, ampleness, bounty, copiousness.

Brilliant:

accomplished, clever, ingenious, intelligent

Abysmal:

bottomless, boundless, complete, deep.

Brisk:

alert, fast, speedy, swift.

Accede:

accept, admit, comply, concede.

Brood:

agonize, dwell, repine, ruminate.

Acclaim:

applaud, approve, celebrate, hail.

Budget:

allot, cost, plan, total.

VA_Chapter 2.indd 504

22-08-2017 06:41:43

TIPS TO SOLVE SYNONYM BASED QUESTIONS        505

C

Deflate:

chasten, collapse, empty, shrink.

Candid:

fair, honest, truthful, unbiased.

Designate:

label, name, select, title.

Caricature:

cartoon, imitation, mockery, parody.

Detain:

apprehend, constrain, hold, keep.

Casual:

extempore, informal, natural, unplanned.

Disclose:

announce, confess, discover, reveal.

Catastrophe:

affliction, devastation, disaster, fiasco.

Diligent:

active, attentive, busy, hardworking.

Category:

classification, department, division, group.

Dire:

alarming, appalling, cruel, disastrous.

Cease:

culminate, desist, discontinue, stop.

Discreet:

careful, cautious, circumspect, diplomatic.

Chaotic:

deranged, disordered, messy, turbulent.

Dissent:

decline, differ, disagree, refuse.

Chastity:

celibacy, innocence, modesty, purity.

Dodge:

avoid, deceive, duck, sidestep.

Cherish:

embrace, esteem, love, nourish.

Dubious:

doubtful, hesitant, mysterious, uncertain.

Circumvent:

avoid, deceive, elude, escape.

Dwell:

abide, establish, inhabit, settle.

Clan:

band, brotherhood, fraternity, gens.

Dwindle:

abate, decay, diminish, weaken.

Clinch:

assure, conclude, confirm, secure.

Colloquial:

conversational, demotic, familiar, informal.

E

Commemorate: admire, celebrate, honour, salute.

Eager:

anxious, earnest, enthusiastic, keen.

Commotion: agitation, disorder, disturbance, excitement.

Eccentric:

abnormal, bizarre, erratic, idiosyncratic.

Edgy:

anxious, irritable, nervous, touchy.

Compensate:

balance, recompense, repay, satisfy.

Elaborate:

embellish, enhance, extravagant, overdone.

Competent:

able, appropriate, capable, skill.

Elusive:

baffling, puzzling, subtle, tricky.

Complicate:

confuse, entangle, involve, muddle.

Emanate:

arise, derive, radiate, originate.

Conceive:

accept, design, grasp, plan.

Embezzle:

forge, loot, purloin, steal.

Confirmation: acknowledgement, affirmation, proof, testament.

Eminent: distinguished, dominant, prominent, renowned.

Contour:

curve, figure, profile, relief.

Endure:

bear, last, persist, suffer.

Contradict:

challenge, deny, disagree, oppose.

Enrage:

aggravate, incite, irritate, provoke.

Contribution:

augmentation, donation, grant, present.

Essential:

basic, capital, necessary, needful.

Estimate:

evaluation, guess, predict, projection.

Exalt:

advance, dignify, elevate, honour.

Exhaust:

deplete, empty, fatigue, weakened.

Conviction: assurance, certainty, confidence, persuasion. Courteous:

affable, civilized, polite, well-mannered.

Craving:

desire, longing, need, passion. Exhilarated: animated, cheerful, euphoric, zestful.

Credulous: accepting, confident, trustful, unsuspicious.

Expedite:

accelerate, advance, dispatch, facilitate.

Explicit:

absolute, accurate, definite, specific.

Creditable: admirable, commendable, deserving, honourable. Culminate:

climax, close, conclude, finish.

Cynical:

distrustful, sarcastic, scornful, sneering. D

F Fame:

celebrity, honour, prominence, stardom.

Famine:

scarcity, dearth, hunger, starvation.

Fastidious: critical, exacting, particular, picky.

Dare:

challenge, defy, provocation, taunt.

Feeble:

weak, delicate, exhausted, infirm.

Daunt:

alarm, frighten, intimidate, terrify.

Fervour:

intensity, passion, seriousness, sincerity.

Decay:

decline, deterioration, mortification, rot.

Feud:

argument, conflict, dispute, fight.

Decent:

chaste, honourable, noble, pure.

Fierce:

barbarous, brutal, cruel, passionate.

Deed:

achievement, act, action, exploit.

Fishy:

doubtful, dubious, suspect, suspicious.

VA_Chapter 2.indd 505

22-08-2017 06:41:43

506        Synonyms 

Flatter:

charm, compliment, glorify, praise.

Flawless:

faultless, impeccable, perfect, unblemished.

Indifferent: apathetic, disinterested, dispassionate, uncaring.

Foresee:

anticipate, divine, predict, prophesy.

Isolate:

Fretful:

captious, complaining, irritable, peevish.

Frivolous:

childish, foolish, impractical, juvenile.

Frugal:

careful, discreet, prudent, saving.

detach, part, quarantine, remove. J

G

Jargon:

argot, dialect, slang, vocabulary.

Jovial:

affable, genial, merry, pleasant.

Judge:

estimate, intermediary, referee, umpire.

Gaiety:

animation, elation, exhilaration, glee.

Justification: absolution, excuse, reason, reply.

Gaudy:

brilliant, flashy, glaring, showy.

Juvenile:

adolescent, childish, immature, naive.

Gestation: development, evolution, formation, pregnancy.

K

Gloomy:

cloudy, dissolute, dim, dreamy.

Kernel:

centre, core, crux, root.

Goad:

impulsion, incentive, irritation, motivation.

Kindle:

blaze, burn, flare, ignite.

Grubby:

filthy, frowzy, foul, shabby.

Knack:

ability, aptitude, skill, talent.

Guile:

cunning, deceit, deception, treachery.

Knotty:

complex, complicated, hard, problematic.

Gullible:

credulous, foolish, naïve, trusting.

Kudos:

credit, fame, glory, honour. L

H Habitual:

accustomed, common, familiar, regular.

Label:

brand, classify.

Haggle:

bargain, barter, quarrel, wrangle.

Labour:

toil, work.

Haphazard: accidental, arbitrary, chance, random.

Lead:

direct, proceed.

Harass:

annoy, attack, disturb, intimidate.

Lean:

slim, thin.

Hasty:

abrupt, careless, hurried, quick.

Leave:

abandon, desert.

Haughty:

arrogant, egoistic, pretentious, snobbish.

Liberal:

lenient, open-minded.

Holocaust: annihilation, carnage, destruction, devastation.

Limitation: boundary, constraint. Lucid:

clear, understandable.

Hygiene: cleanliness, sanitation, regimen, wholesomeness.

Lucky:

auspicious, fortunate.

Hypocrisy:

deceitfulness, duplicity, falseness, pretence.

M Manage:

I

administer, control.

Manipulate: control, shape.

Ideal:

absolute, complete, optimal, perfection.

Marginal:

borderline, limited.

Idle:

abandoned, barren, lazy, unoccupied.

Match:

agree, correspond.

Ignorant:

inexperienced, stupid, unaware, unintelligent.

Maze:

complexity, labyrinth.

Illogical:

incongruent, irrational, rambling, senseless.

Meditate:

ponder, think.

Illustrious:

brilliant, celebrated, eminent, famous.

Memorial:

commemoration, monument.

Imitate:

copy, duplicate, reflect, xerox.

Mention:

allude, refer.

Immense:

endless, huge, large, mammoth.

Merge:

blend, fuse.

Impartial:

candid, equal, impersonal, unbiased.

Implicate:

accuse, associate, blame, insinuate.

N

Importune: demand, insist, persuade, solicit.

Narrow:

confined, slim, thin, restricted.

Inadvertent: accidental, careless, negligent, unintentional.

Nature:

aspect, character, features, personality.

VA_Chapter 2.indd 506

22-08-2017 06:41:43

TIPS TO SOLVE SYNONYM BASED QUESTIONS        507

Negate:

contradict, countermand, nullify, refute.

S

Negligent: careless, complacent, indifferent, unconcerned.

Sanction: authorization, approval, permission, support.

Negotiate:

bargain, deal, mediate, settle.

Scope:

aim, extent, opportunity, space.

Noble:

aristocratic, distinguished, gentle, imperial.

Section:

area, division, field, portion.

Novice:

amateur, beginner, learner, rookie.

Shallow:

depthless, hollow, superficial, trivial.

Nuisance:

annoyance, offense, problem, trouble.

Shrewd:

careful, calculating, cunning, piercing.

Slight:

insignificant, negligible, slim, small.

O Obedient:

amenable, devoted, faithful, loyal.

Spontaneous: extemporaneous, impulsive, instinctive, unplanned.

Objection:

challenge, disapproval, question, protest.

Stabilize:

balance, maintain, steady, sustain.

Obligatory: binding, compulsory, imperative, required. Observe:

monitor, notice, spot, watch. T

Obvious: conspicuous, definite, straightforward, transparent. Offend:

anger, annoy, irritate, provoke.

Omen:

indication, premonition, sign, warning.

Omit:

cancel, exclude, remove, skip.

Opportune: appropriate, advantageous, auspicious, timely. P Pacify:

appease, assuage, moderate, placate.

Tame:

amenable, domesticated, manageable, trained.

Tangle:

complication, intertwine, mess, twist.

Tendency:

bias, inclination, trend, readiness.

Term:

cycle, duration, period, terminology.

Thrift: carefulness, conservation, prudence, stringiness. Tumult:

agitation, commotion, confusion, uproar.

Turbulent: disordered, disturbance, unstable, violent.

Paramount: chief, leading, principal, superior. Partisan:

biased, dogmatic, partial, prejudiced.

Passive:

inactive, lethargic, lifeless, static.

Permeate:

diffuse, disseminate, infiltrate, penetrate.

U Ubiquitous: everywhere, omnipresent, pervasive, universal. Umbrage:

anger, annoyance, grudge, rage.

Perpetuate: bolster, endure, preserve, secure. Unanimity: accord, agreement, concurrence, unison. Perplex:

astonish, baffle, cheat, deceive.

Persecute:

afflict, harass, torment, torture.

Unequivocal: absolute, apparent, certain, definite.

Q

Unsavoury: distasteful, disagreeable, repulsive, revolting.

Quash:

annihilated, crush, quench, repress.

Queer:

abnormal, anomalous, bizarre, odd.

Quibble:

complaint, criticism, object, protest.

Vain:

arrogant, boastful, haughty, narcissistic.

Quiver:

shake, shiver, tremble, vibration.

Valid:

accurate, authenticate, legitimate, tested.

Variety:

array, collection, diversity, range.

Verify:

attest, certify, prove, test.

R Radiate:

effuse, emit, scatter, transmit.

Radical:

basic, essential, fundamental, natural.

Rank:

arrange, classify, organize, position.

V

W Waggish:

amusing, cheerful, comical, playful.

Recalcitrant: contrary, defiant, opposing, stubborn.

Waive:

abandon, delay, leave, resign.

Receptacle: box, container, holder, repository.

Warden:

administrator, caretaker, curator, guardian.

Reconcile:

adjust, atone, conciliate, pacify.

Wear:

attrition, damage, deterioration, loss.

Regret:

affliction, anguish, dissatisfaction, remorse

Woe:

agony, anguish, grief, suffering.

VA_Chapter 2.indd 507

22-08-2017 06:41:43

508        Synonyms 

Z

X Xenophobia: animosity, antipathy, bias, prejudice.

Zany:

comical, eccentric, funny, foolish.

Zeal: alacrity, determination, eagerness, enthusiasm.

Y Yen:

craving, desire, lust, passion.

Zenith:

acme, climax, height, top.

Yield:

earnings, harvest, income, produce.

Zest:

bite, flavour, relish, taste.

PRACTICE EXERCISE Read the words given below and choose the option that is most nearly same in meaning of the word.

(a) enjoyment (c) suffering

1. Abase (a) incur (c) humiliate

(b) dignify (d) estimate

(b) abhorrence (d) absence

(a) obedience (c) conceive

(b) encourage (d) evade

(a) detest (c) affection

(b) love (d) embarrassed

5. Abnegation (a) cause (c) expectation

(b) selfishness (d) renunciation

(b) forged (d) prove genuine

14. Baffle (a) certain (c) angry

4. Abhor

(b) increase (d) harvest

13. Authenticate (a) original (c) fake

3. Abet

(b) excitement (d) frustrated

12. Augment (a) decrease (c) friendly

2. Aberration (a) deviation (c) dislike

11. Anguish

(b) confuse (d) sure

15. Baleful (a) detest (c) powerful

(b) evil (d) pure

16. Baneful 6. Abridge (a) extend (c) mediate

(b) lengthen (d) shorten

(a) intellectual (c) decisive

(b) thankful (d) poisonous

17. Bawdy 7. Abysmal (a) eternal (c) shallow

(b) bottomless (d) big

(a) flashy (c) indecent

(b) confident (d) show-off

18. Blandishment 8. Acclivity (a) elevation (c) report

(b) decline (d) climate

(b) agreement (d) misdirection

VA_Chapter 2.indd 508

(a) cheat (c) sincere

(b) toy (d) fair

20. Brackish

10. Ample (a) abundant (c) parity

(b) love (d) embarrassed

19. Bluff

9. Accord (a) direction (c) massive

(a) flattery (c) affection

(b) deficiency (d) less

(a) careful (c) chosen

(b) salty (d) tough

22-08-2017 06:41:43

PRACTICE EXERCISE        509

21. Brazen (a) shameless (c) pleasant

35. Defiance (b) quick (d) modest

22. Callous (a) kind (c) sympathetic

(b) apathy (d) insensitive

(b) outspoken (d) anxious

(b) suspicious (d) expected

(b) table (d) bottle

(b) timid (d) pure

(b) fear (d) scold

(b) cautious (d) complacent

(b) loosen (d) thin

(b) arrest (d) confused

(b) hate (d) disorganized

(b) disgrace (d) collide

VA_Chapter 2.indd 509

(a) anxious (c) pious

(b) shocking (d) sociable

(a) debased (c) joyful

(b) depressed (d) embarrassed

(a) liberate (c) remove

(b) poverty (d) avoid

(a) joy (c) affection

(b) enjoyment (d) mystery

(a) apprehend (c) punish

(b) acquit (d) cremate

(a) attack (c) punish

(b) exile (d) exclude

47. Factitious (b) downward slope (d) decline

34. Decorous (a) momentary (c) suppressed

(b) proof (d) recognize

46. Expatriate

33. Declivity (a) trap (c) quarter

(a) efficient (c) mystery

45. Exonerate

32. Decimate (a) search (c) kill

(b) pleasure (d) treasure

44. Enigma

31. Cordial (a) friendly (c) distorted

(a) speed (c) warmth

43. Emancipate

30. Comprehend (a) remove (c) understand

(b) inhabit (d) lessen

42. Elated

29. Coagulate (a) thicken (c) sum

(a) blow (c) spin

41. Egregious

28. Circumspect (a) careless (c) flashy

(b) sadness (d) tidiness

40. Efficacy

27. Chide (a) unite (c) record

(a) wickedness (c) heaviness

39. Ecstasy

26. Chaste (a) loyal (c) curt

(b) deception (d) mountain

38. Dwindle

25. Chalice (a) cup (c) drink

(a) confusion (c) flood 37. Depravity

24. Capricious (a) deviating (c) unpredictable

(b) acceptance (d) strength

36. Deluge

23. Candid (a) vague (c) experienced

(a) resistance (c) obedience

(a) artificial (c) real

(b) true (d) imaginary

48. Feebleminded (b) emotional d) proper

(a) confused (c) stupid

(b) shrewd (d) intelligent

22-08-2017 06:41:43

510        Synonyms 

49. Ferocious (a) angry (c) shoot

63. Imminent (b) punish (d) fierce

50. Finesse (a) smooth (c) final

(b) delicate skill (d) end

(b) useless (d) uncertainty

(b) accept (d) advantage

(b) smooth (d) decorate

(b) introvert (d) extrovert

(b) abolishment (d) attack

(b) foreign (d) angry

(b) impressiveness (d) unimpressive

(b) confused (d) horrible

(b) order (d) anger

(b) confused d) crowd

VA_Chapter 2.indd 510

(a) ingenious (c) indecent

(b) poverty (d) diligent

(a) equality (c) money

(b) property (d) unfairness

(a) perfect (c) joyful

(b) to fall (d) complacent

(a) devilish (c) unsatisfied

(b) complete (d) satanic

(a) meanwhile (c) serious

(b) international (d) interval

(a) resistant (c) unalterable

(b) alterable (d) vocabulary

75. Jaunty (b) aspect (d) opinion

62. Igneous (a) volcanic (c) mighty

(b) cautious (d) ignorant

74. Irrevocable

61. Hue (a) fight (c) argument

(a) lacking caution (c) insensitive

73. Interim

60. Hubris (a) chaos (c) arrogance

(b) improvement (d) imperial

72. Insatiable

59. Havoc (a) disorder (c) passion

(a) impenetrable (c) confused

71. Infallible

58. Gruesome (a) long (c) exhausting

(b) improvement (d) hindrance

70. Inequity

57. Grandeur (a) big (c) path

(a) judgement (c) impeachment

69. Indigence

56. Gibberish (a) nonsense (c) ethical

(b) stern (d) impossible

68. Imprudent

55. Genesis (a) end (c) origin

(a) unreachable (c) perfect

67. Impervious

54. Gaudy (a) flashy (c) ill-mannered

(b) unchangeable (d) immovable

66. Impediment

53. Garnish (a) grand (c) spoil

(a) noisy (c) different 65. Impeccable

52. Gainsay (a) deny (c) profit

(b) danger (d) relaxed

64. Immutable

51. Futile (a) fertile (c) worthy

(a) near at hand (c) important

(a) showy (c) cheerful

(b) arrogant (d) sarcastic

76. Knack (b) powerful (d) anger

(a) irritate (c) knock

(b) talent (d) hit

22-08-2017 06:41:44

PRACTICE EXERCISE        511

77. Larceny (a) money (c) large

91. Pacify (b) great (d) theft

78. Loath (a) soap (c) reluctant

(b) money (d) detest

(b) shining (d) dull

(b) holiday d) foul

(b) adore (d) confused

(b) document (d) urgent

(b) maim (d) suffer

(b) negative (d) alter

(b) inspiration (d) newness

(b) mad (d) futile

(b) offensive (d) irritating

(b) present everywhere (d) all knowing

VA_Chapter 2.indd 511

(a) peak (c) depth

(b) finicky (d) fruit

(a) medical (c) previous

(b) perception (d) pre-planned

(a) vision (c) temporary

(b) spacious (d) pre-empt

(a) eccentric (c) ordinary

(b) casual (d) fight

(a) insatiable (c) quarry

(b) satisfy (d) quit

(a) query (c) quibble

(b) quarrel (d) tremble

103. Rakish (b) outlander (d) common

90. Outspoken (a) arrogant (c) outsider

(b) angry (d) perceive

102. Quiver

89. Outlandish (a) splendid (c) bizarre

(a) challenge (c) disturb

101. Quench

88. Omniscient (a) unknowing (c) omnipresent

(b) theft (d) pungent

100. Queer

87. Obnoxious (a) disastrous (c) preposterous

(a) testify falsely (c) burglary

99. Provisional

86. Nugatory (a) confused (c) worthy

(b) path (d) hell

98. Premeditated

85. Novelty (a) copy (c) stupid

(a) blessings (c) heaven

97. Pinnacle

84. Negate (a) nullify (c) amplify

(b) inactive (d) aggressive

96. Perturb

83. Mutilate (a) pacify (c) mute

(a) pacifist (c) active

95. Perjury

82. Manifest (a) humane (c) evident

(b) equality (d) inequality

94. Perdition

81. Malicious (a) hateful (c) delicious

(a) close (c) double 93. Passive

80. Malady (a) illness (c) condition

(b) windy (d) aggressive

92. Parity

79. Luminous (a) grand (c) luscious

(a) make peace (c) oceanic

(a) speedy (c) rake

(b) rash (d) jaunty

104. Ratify (b) candid (d) introvert

(a) confused (c) confirm

(b) rattled (d) suspicious

22-08-2017 06:41:44

512        Synonyms 

105. Recant (a) remember (c) curse

119. Simulate (b) disavow (d) canny

106. Refute (a) refuge (c) recall

(b) disapprove (d) feud

(b) medical (d) remove

(b) demonstrate (d) remedy

(b) denounce (d) disown

(b) strong (d) dust

(b) fight (d) lucrative

(b) pitiless (d) aggressive

(b) salacious (d) wise

(b) save (d) magnificent

(b) diligent (d) sedate

(b) serendipity (d) insane

VA_Chapter 2.indd 512

(a) counterfeit (c) fight

(b) furious (d) spare

(a) pungent (c) flow

(b) motionless (d) foul

(a) obsolete (c) super

(b) previous (d) replace

(a) singular (c) hypnotic

(b) detailed (d) summary

(a) temporary (c) violent storm

(b) temper (d) preposterous

(a) casual (c) needs

(b) persistence (d) cruelty

131. Tentative (b) casualty (d) obtuse

118. Shrewd (a) angry (c) fish

(b) grief (d) enjoyment

130. Tenacity

117. Severity (a) severance (c) harshness

(a) peace (c) consolation

129. Tempestuous

116. Serenity (a) calmness (c) sane

(b) bark (d) truffle

128. Synoptic

115. Sedulous (a) seldom (c) confused

(a) sniffle (c) snivel

127. Supersede

114. Savour (a) enjoy (c) solitude

(b) slide (d) cut

126. Stagnant

113. Sagacious (a) saga (c) stupid

(a) tongue (c) dirty

125. Spurious

112. Ruthless (a) merciful (c) rusty

(b) slight (d) dexterity

124. Solace

111. Rue (a) feud (c) regret

(a) singular (c) fleet

123. Snuffle

110. Robust (a) force (c) weak

(b) slippery (d) gibberish

122. Slither

109. Repudiate (a) repulse (c) reputed

(a) frivolous (c) careful 121. Sleight

108. Remonstrate (a) objection (c) substrate

(b) imitate (d) thrust

120. Skittish

107. Remedial (a) reparable (c) irreparable

(a) stimulate (c) begin

(a) provisional (c) certain

(b) confirmed (d) brittle

132. Timidity (b) clever (d) foolish

(a) shyness (c) confident

(b) careless (d) boastful

22-08-2017 06:41:44

ANSWERS        513

133. Torrid

142. Waggish

(a) rapid (c) torpedo

(b) passionate (d) violent

(a) random (c) decent

134. Tranquillity (a) strike (c) peace

(b) swiftly (d) mischievous

143. Weary (b) hyper (d) extreme

(a) wearing (c) loose

135. Travail

(b) tired (d) clothes

144. Woe

(a) problematic (c) trail

(b) lazy (d) strenuous work

(a) anger (c) feud

136. Truncate

(b) theft (d) sorrow

145. Wrangle

(a) paste (c) increase

(b) shorten (d) trunk

(a) noisy quarrel (c) intertwined

137. Turpitude (a) arrogance (c) turbulence

146. Wrath (b) guarantee (d) depravity

(a) disappointment (c) curtains

138. Ubiquitous (a) unanimous (c) doubt

(b) involved (d) connected

(b) anger (d) argument

147. Yen (b) omnipresent (d) anonymous

(a) dreams (c) longing

139. Umbrage

(b) aspirations (d) dislikes

148. Yoke

(a) anger (c) remorse

(b) homage (d) supportive

(a) vomit (c) unite

140. Uncanny

(b) separate (d) shout

149. Zeal

(a) strange (c) absolute

(b) clear (d) repulsive

(a) enthusiastic (c) liar

141. Upshot

(b) seal (d) idle

150. Zenith

(a) loud (c) outcome

(b) high (d) upward

(a) wrong (c) low

(b) below (d) top

ANSWERS 1. (c)

10. (a)

19. (a)

28. (b)

37. (a)

46. (b)

55. (c)

64. (b)

2. (a)

11. (c)

20. (b)

29. (a)

38. (d)

47. (a)

56. (a)

65. (c)

3. (b)

12. (b)

21. (a)

30. (c)

39. (b)

48. (c)

57. (b)

66. (d)

4. (a)

13. (d)

22. (d)

31. (a)

40. (a)

49. (d)

58. (d)

67. (a)

5. (d)

14. (b)

23. (b)

32. (c)

41. (b)

50. (b)

59. (a)

68. (a)

6. (d)

15. (b)

24. (b)

33. (b)

42. (c)

51. (b)

60. (c)

69. (b)

7. (b)

16. (d)

25. (a)

34. (d)

43. (a)

52. (a)

61. (b)

70. (d)

8. (a)

17. (c)

26. (d)

35. (a)

44. (d)

53. (d)

62. (a)

71. (a)

9. (b)

18. (a)

27. (d)

36. (c)

45. (b)

54. (a)

63. (a)

72. (c)

VA_Chapter 2.indd 513

22-08-2017 06:41:44

514        Synonyms 

73. (a)

83. (b)

93. (b)

103. (d)

113. (d)

123. (a)

133. (b)

143. (b)

74. (c)

84. (a)

94. (d)

104. (c)

114. (a)

124. (c)

134. (c)

144. (d)

75. (c)

85. (d)

95. (a)

105. (b)

115. (b)

125. (a)

135. (d)

145. (a)

76. (b)

86. (d)

96. (c)

106. (b)

116. (a)

126. (b)

136. (b)

146. (b)

77. (d)

87. (b)

97. (a)

107. (a)

117. (c)

127. (d)

137. (d)

147. (c)

78. (c)

88. (d)

98. (d)

108. (a)

118. (b)

128. (d)

138. (b)

148. (c)

79. (b)

89. (c)

99. (c)

109. (d)

119. (b)

129. (c)

139. (a)

149. (a)

80. (a)

90. (b)

100. (a)

110. (b)

120. (a)

130. (b)

140. (a)

150. (d)

81. (a)

91. (a)

101. (b)

111. (c)

121. (d)

131. (a)

141. (c)

82. (c)

92. (b)

102. (d)

112. (b)

122. (b)

132. (a)

142. (d)

VA_Chapter 2.indd 514

22-08-2017 06:41:44

CHAPTER 3

ANTONYMS

An antonym of any word has a meaning opposite to the word. An antonym is one of a pair of words with opposite meanings. A word may have more than one antonym. This chapter deals with the types of antonyms and a list of some antonym pairs followed by an exercise at the end.

GRADED ANTONYMS Graded antonyms deal with levels of the meaning of the words, like if something is not “good”, it may still not be “bad.” There is a scale involved with some words, and besides good and bad there can be average, fair, excellent, terrible, poor, or satisfactory. Examples include:

1. Fat and skinny 2. Young and old 3. Happy and sad 4. Hard and soft 5. Last and first 6. Foolish and wise 7. Fast and slow 8. Warm and cool

VA_Chapter 3.indd 515

9. Wide and narrow 10. Abundant and scarce 11. Joy and grief 12. Dark and light 13. Dangerous and safe 14. Clever and foolish 15. Early and late 16. Empty and full 17. Smart and dumb 18. Risky and safe 19. Bad and good 20. Pretty and ugly 21. Best and worst 22. Simple and challenging 23. Soft and hard 24. Worried and calm 25. Sane and crazy 26. Rich and poor 27. Cool and hot 28. Wet and dry 29. Late and early 30. Ignorant and educated 31. Big and small 32. Optimistic and pessimistic 33. Excited and bored 34. Dull and interesting

22-08-2017 06:42:17

516        Antonyms 

COMPLEMENTARY ANTONYMS

RELATIONAL ANTONYMS

Complementary antonyms  have a relationship where there is no middle ground. There are only two possibilities, either one or the other.

Relational antonyms are sometimes considered a subcategory of complementary antonyms. With these pairs, for there to be a relationship, both must exist.

Examples include:

Examples are:

1. Man and woman 2. Push and pull 3. Dead and alive 4. Off and on 5. Day and night 6. Absent and present 7. Exit and entrance 8. Sink or float 9. True or false 10. Pass and fail 11. Former and latter 12. Input and output 13. Interior and exterior 14. Exhale and inhale 15. Input and output 16. Occupied and vacant 17. Leave and arrive 18. Pre and post 19. Question and answer 20. Single and married 21. Hired and fired 22. Brother and sister 23. Before and after 24. Crooked and straight 25. Identical and different 26. Natural or artificial 27. Silence or noise 28. Identical or different 29. Yes and no 30. Wet and dry 31. Sharp and dull 32. Raise and lower 33. Fantasy and reality

1. Husband and wife 2. Doctor and patient 3. Buy and sell 4. Predator and prey 5. Above and below 6. Give and receive 7. Teach and learn 8. Instructor and pupil 9. Servant and master 10. Borrow and lend 11. Come and go 12. Toward and away 13. Divisor and dividend 14. Parent and child 15. East and west 16. North and south 17. Seller and buyer 18. Mother and daughter 19. Slave and master 20. Floor and ceiling 21. Front and back 22. Up and down 23. Win and lose 24. Part and whole 25. Offense and defence 26. Behind and ahead 27. Before and after 28. On or off 29. Trap and release 30. Lost and found 31. Left and right 32. Give and get 33. Employer employee

PRACTICE EXERCISE Read the words given below and choose the option that is opposite or most nearly opposite in meaning to the word.

2. Affable

1. Abrogate

3. Affluence

(a) nullify (c)  annul

VA_Chapter 3.indd 516

(b) signify (d) ratify

(a) rude (c) soft

(a) abundance (c) influence

(b) ruddy (d) lovable

(b) poverty (d) consideration

22-08-2017 06:42:17

PRACTICE EXERCISE        517

18. Bestial

4. Agility (a) detest (c) alacrity

(b) solidity (d) stiffness

(b) enough (d) parity

(b) ample (d) shorten

(b) comparable (d) unanimous

(b) comfort (d) irritation

(b) normality (d) desperation

(b) detest (d) dislike

(b) crack (d) volubility

(b) increase (d) harvest

(b) investigate (d) fake

(b) enlighten (d) fight

(b) auspicious (d) evil

(b) bizarre (d) constant

VA_Chapter 3.indd 517

(b) insured (d) expected

(a) kind (c) minute

(b) impressive (d) tolerant

(a) patience (c) understatement

(b) commendation (d) generosity

(a) assurance (c) acerbity

(b) state (d) delay

(a) prime (c) beginning

(b) end (d) complacent

(a) corrupt (c) timid

(b) incorrupt (d) haste

(a) remove (c) understand

(b) arrest (d) misunderstand

31. Cordial

17. Belittle (a) praise (c) disobey

(a) deviating (c) steadfast

30. Comprehend

16. Baroque (a) polished (c) simple

(b) outspoken (d) anxious

29. Chaste

15. Baleful (a) detest (c) powerful

(a) honest (c) deceitful

28. Cessation

14. Baffle (a) confuse (c) angry

(b) apathy (d) insensitive

27. Celerity

13. Authenticate (a) original (c) prove genuine

(a) shy (c) unashamed

26. Castigation

12. Augment (a) decrease (c) friendly

(b) cheerful (d) cheerless

25. Carnal

11. Aphasia (a) pain (c) necessity

(a) shameless (c) happy

24. Capricious

10. Antipathy (a) objection (c) admiration

(b) cheerful (d) wet

23. Candid

9. Anomaly (a) aberration (c) massive

(a) pale (c) sudden

22. Brazen

8. Anguish (a) suffering (c) pain

(b) cheat (d) embarrassed

21. Blithe

7. Analogous (a) incomparable (c) incapable

(a) criticize (c) sincere 20. Bleak

6. Amplify (a) improve (c) increase

(b) clear (d) noble

19. Blandishment

5. Ample (a) cause (c) insufficient

(a) flattery (c) refer

(b) forget (d) criticize

(a) friendly (c) unfriendly

(b) harmony (d) disorganized

22-08-2017 06:42:17

518        Antonyms 

32. Coy (a) weak (c) immodest

46. Equivocal (b) airy (d) old

33. Crux (a) affliction (c) events

(b) spark (d) trivial point

(b) trusting (d) conclusive

(b) cowardly (d) solid

(b) deception (d) orderly

(b) immediate (d) baleful

(b) integrate (d) consecrate

(b) dazzling (d) treasure

(b) confidence (d) timid

(b) contract (d) participate

(b) joyful (d) embarrassed

(b) poverty (d) avoid

VA_Chapter 3.indd 518

(a) effective (c) essential

(b) modern (d) advantage

(a) agreement (c) spoil

(b) fight (d) angry

(a) confuse (c) ill-mannered

(b) senseless (d) sensible

(a) frivolous (c) economical

(b) rude (d) lavish

(a) beginning (c) crux

(b) end (d) big

(a) costly (c) gratitude

(b) complimentary (d) unimpressive

58. Gregarious (b) arrange (d) strengthen

45. Ennui (a) hate (c) enmity

(b) blame (d) hit

57. Gratuitous

44. Enervate (a) sputter (c) scrutinize

(a) pardon (c) fight

56. Genesis

43. Emancipate (a) imprison (c) liberty

(b) hate (d) end

55. Frugal

42. Elated (a) expand (c) depressed

(a) love (c) admit

54. Frivolous

41. Dilate (a) procrastinate (c) conclude

(b) prevail (d) fierce

53. Feud

40. Diffidence (a) efficient (c) hesitancy

(a) blame (c) remove

52. Extraneous

39. Destitute (a) deficient (c) affluent

(b) empty (d) fertile

51. Exonerate

38. Desecrate (a) desist (c) violate

(a) confused (c) frank

50. Execrate

37. Derogatory (a) praising (c) cynical

(b) incorrect (d) correct

49. Exculpate

36. Decorous (a) inappropriate (c) appropriate

(a) artificial (c) bizarre 48. Evasive

35. Dauntless (a) resistance (c) peculiar

(b) certain (d) exclude

47. Erroneous

34. Cynical (a) gallant (c) effortless

(a) mistaken (c) azure

(a) friendly (c) exhausting

(b) affable (d) unfriendly

59. Gullible (b) excitement (d) humility

(a) disorder (c) perceptive

(b) gulp (d) naive

22-08-2017 06:42:17

PRACTICE EXERCISE        519

60. Gusty (a) chaos (c) calm

74. Kindle (b) windy (d) ghastly

61. Hackneyed (a) clich d (c) timely

(b) original (d) shrewd

(b) accidental (d) destruction

(b) danger (d) peace

(b) sad (d) high

(b) economical (d) heterogeneous

(b) guessed (d) factual

(b) improvement (d) deceptive

(b) careless (d) rude

(b) wealth (d) poor

(b) complacent (d) faulty

(b) complacent (d) faded

VA_Chapter 3.indd 519

(a) diminish (c) condition

(b) enlarge (d) fire

(a) friendly (c) delicious

(b) detest (d) confused

(a) destructive (c) menacing

(b) honest (d) dishonest

(a) destruct (c) repair

(b) hit (d) suffer

(a) nullify (c) starry

(b) cold (d) definite

(a) bad (c) stupid

(b) good (d) sinful

86. Obligatory (b) danger (d) courage

73. Jocular (a) curiosity (c) funny

(b) pale (d) dull

85. Nefarious

72. Jeopardy (a) devilish (c) safety

(a) gloomy (c) radiant

84. Nebulous

71. Jaded (a) fresh (c) exhausted

(b) silliness (d) seriousness

83. Mutilate

70. Infallible (a) perfect (c) joyful

(a) amalgam (c) leverage

82. Mendacious

69. Indigence (a) ingenious (c) indecent

(b) sluggish (d) lethal

81. Malicious

68. Imprudent (a) cautious (c) insensitive

(a) active (c) large

80. Magnify

67. Illusive (a) real (c) confused

(b) talent (d) delay

79. Livid

66. Hypothetical (a) ethical (c) imaginary

(a) dormant (c) apparent

78. Levity

65. Hybrid (a) mixture (c) homogeneous

(b) compact (d) wordy

77. Lethargic

64. Hilarity (a) laughter (c) different

(a) short (c) cynical 76. Latent

63. Havoc (a) recent (c) calamity

(b) burn (d) extinguish

75. Laconic

62. Haphazard (a) careful (c) aimless

(a) kindly (c) hold

(a) oblivious (c) nonessential

(b) binding (d) essential

87. Obnoxious (b) serious (d) interval

(a) offensive (c) preposterous

(b) kind (d) irritating

22-08-2017 06:42:17

520        Antonyms 

88. Odium (a) approval (c) shame

102. Protract (b) noise (d) order

89. Outlandish (a) outlaw (c) bizarre

(b) outlander (d) common

(b) modest (d) avid

(b) windy (d) make peace

(b) equality (d) pair

(b) inactive (d) aggressive

(b) path (d) dislike

(b) harmless (d) hurtful

(b) continual (d) standard

(b) potent (d) pleasant

(b) epilogue (d) aria

(b) condemned (d) pure

VA_Chapter 3.indd 520

(a) weak (c) strong

(b) demonstrate (d) heavy

(a) wan (c) pale

(b) deceitful (d) blushing

(a) heartless (c) denounce

(b) compassionate (d) rusty

(a) upset (c) kind

(b) fight (d) perverse

(a) merciful (c) spacious

(b) foolish (d) judicious

(a) save (c) solitude

(b) redeem (d) endanger

114. Scurrilous (b) casual (d) deceitful

101. Propitious (a) induced (c) prosperous

(b) nervous (d) calm

113. Salvage

100. Profane (a) moral (c) define

(a) lazy (c) impatient

112. Sagacious

99. Pristine (a) vision (c) cultivated

(b) clarify (d) authorize

111. Sadistic

98. Prelude (a) presume (c) previous

(a) disagree (c) rational

110. Ruthless

97. Petulant (a) moody (c) angry

(b) stillness (d) solution

109. Ruddy

96. Perpetual (a) brief (c) logical

(a) vibration (c) argument

108. Robust

95. Pernicious (a) baleful (c) suspicious

(b) odd (d) suspicious

107. Restive

94. Penchant (a) fondness (c) pungent

(a) feud (c) normal

106. Ratify

93. Passive (a) pacifist (c) active

(b) rash (d) emergency

105. Quiver

92. Parity (a) close (c) inequality

(a) rewarding (c) premature 104. Queer

91. Pacify (a) agitate (c) oceanic

(b) extract (d) shorten

103. Punitive

90. Overweening (a) arrogant (c) wiener

(a) extend (c) retract

(a) decent (c) active

(b) scabby (d) savage

115. Sedulous (b) unfavourable (d) favourable

(a) active (c) confused

(b) indolent (d) vindictive

22-08-2017 06:42:17

PRACTICE EXERCISE        521

116. Serenity (a) calmness (c) sane

130. Tenacity (b) serendipity (d) disruption

117. Severity (a) sever (c) kindness

(b) casualty (d) asperity

(b) clever (d) condescending

(b) nervous (d) thrust

(b) peace (d) discord

(b) valid (d) misleading

(b) subside (d) cut

(b) artificial (d) real

(b) knowledge (d) closure

(b) furious (d) retreat

(b) motionless (d) scientific

(b) attack (d) complacent

VA_Chapter 3.indd 521

(a) lengthen (c) trump

(b) shorten (d) trunk

(a) clear (c) cloudy

(b) guarantee (d) depravity

(a) unanimous (c) doubt

(b) expert (d) novice

(a) anger (c) happiness

(b) homage (d) displeasure

(a) strange (c) absolute

(b) clear (d) content

(a) loud (c) unstable

(b) high (d) perfect

142. Unilateral (b) punctual (d) summary

129. Tempestuous (a) temporary (c) gentle

(b) lazy (d) seriousness

141. Unfaltering

128. Tardy (a) tired (c) late

(a) problematic (c) spoof

140. Unassuaged

127. Tactful (a) obsolete (c) super

(b) active (d) extreme

139. Umbrage

126. Synthesis (a) amalgam (c) division

(a) temporary (c) lasting

138. Tyro

125. Surreptitious (a) honest (c) secretive

(b) brazen (d) afraid

137. Turbid

124. Surmise (a) surprise (c) guess

(a) rapid (c) torpedo

136. Truncate

123. Superficial (a) analytical (c) superior

(b) careless (d) fail

135. Travesty

122. Subdue (a) moderate (c) incite

(a) prosper (c) confident

134. Transient

121. Specious (a) singular (c) dexterity

(b) thin (d) brittle

133. Timorous

120. Solace (a) homage (c) journey

(a) substantial (c) strenuous 132. Thrive

119. Skittish (a) calm (c) slippery

(b) persistence (d) cruelty

131. Tenuous

118. Shrewd (a) frivolous (c) sharp

(a) diligence (c) weakness

(a) many-sided (c) descent

(b) swiftly (d) one-sided

143. Valour (b) stormy (d) preposterous

(a) wearing (c) loose

(b) cowardice (d) bravery

22-08-2017 06:42:17

522        Antonyms 

148. Wry

144. Withdraw (a) sit (c) endure

(a) straightforward (c) distorted

(b) remove (d) stay

149. Yen

145. Withstand (a) yield (c) endure

(a) dreams (c) unite

(b) theft (d) sit

(b) dislike (d) desire

150. Zeal

146. Wrangle (a) feud (c) peace

(b) aspirations (d) weary

(a) enthusiastic (c) liar

(b) involved (d) connected

(b) seal (d) lethargy

147. Wrath (a) disappointment (c) happiness

(b) anger (d) argument

ANSWERS 1. (d)

20. (b)

39. (c)

58. (d)

77. (a)

96. (a)

115. (b)

134. (c)

2. (a)

21. (d)

40. (b)

59. (c)

78. (d)

97. (d)

116. (d)

135. (d)

3. (b)

22. (a)

41. (b)

60. (c)

79. (c)

98. (b)

117. (c)

136. (a)

4. (d)

23. (c)

42. (c)

61. (b)

80. (a)

99. (c)

118. (a)

137. (a)

5. (c)

24. (c)

43. (a)

62. (a)

81. (a)

100. (a)

119. (a)

138. (b)

6. (d)

25. (a)

44. (d)

63. (d)

82. (b)

101. (b)

120. (d)

139. (c)

7. (a)

26. (b)

45. (b)

64. (b)

83. (c)

102. (d)

121. (b)

140. (d)

8. (b)

27. (b)

46. (b)

65. (c)

84. (d)

103. (a)

122. (c)

141. (c)

9. (b)

28. (c)

47. (d)

66. (d)

85. (b)

104. (c)

123. (a)

142. (a)

10. (c)

29. (a)

48. (c)

67. (a)

86. (c)

105. (b)

124. (b)

143. (b)

11. (d)

30. (d)

49. (a)

68. (a)

87. (b)

106. (a)

125. (a)

144. (d)

12. (a)

31. (c)

50. (a)

69. (b)

88. (a)

107. (d)

126. (c)

145. (a)

13. (d)

32. (c)

51. (b)

70. (d)

89. (d)

108. (a)

127. (d)

146. (c)

14. (b)

33. (d)

52. (c)

71. (a)

90. (b)

109. (c)

128. (b)

147. (c)

15. (b)

34. (b)

53. (a)

72. (c)

91. (a)

110. (b)

129. (c)

148. (a)

16. (c)

35. (b)

54. (d)

73. (b)

92. (c)

111. (c)

130. (c)

149. (b)

17. (a)

36. (a)

55. (d)

74. (d)

93. (c)

112. (b)

131. (a)

150. (d)

18. (d)

37. (a)

56. (b)

75. (d)

94. (d)

113. (d)

132. (d)

19. (a)

38. (d)

57. (a)

76. (c)

95. (b)

114. (a)

133. (b)

VA_Chapter 3.indd 522

22-08-2017 06:42:17

CHAPTER 4

SENTENCE COMPLETION

Sentence completion problems are used to test your vocabulary skills and your reading ability. Each sentence contains one or two blanks. You are then presented with four options which may consist of words or phrases, or four pairs of words or phrases if there are two blanks in the sentence. From these choices, you need to select the most appropriate or correct words or phrases. To make the correct choice, you will need to able to understand the main idea of the sentence, the meaning of the words or phrases and the logical structure of the sentence.

of. In such scenarios, there are certain tips or tricks that the student can use in order to increase his chances of getting the correct answer. Some of the common tips are given as follows:

The sentence completion questions do not have any predefined technique to solve them. However, certain steps can be followed in order to increase the odds in your favour to get the right answer.



(a) Elegance (c) Anger



We can see that the sentence talks about the positive traits of the princess. Also, it is evident enough that the blank is not pertaining to her physical appearance and is a quality of her nature like humility, honesty, kind, etc. Thus, looking at the options, we can choose the answer to be honest.

TIPS TO SOLVE SENTENCE COMPLETION BASED QUESTIONS Questions based on synonyms and antonyms can be pretty difficult to answer if the word is new or unheard

VA_Chapter 4.indd 523

1. Before looking at the answer-choices, the student should think of a word that fits the sentence. Example: The princess was popular and loved throughout her kingdom not only for her physical beauty, but also for her . (b) Honesty (d) Arrogance

2. Look for key words and phrases in the text that tells the student where the sentence is going. If the sentence is continuing along one line

22-08-2017 06:42:50

524     Sentence Completion 









of though, then the student should look for a completion that carries the idea through. If the text changes direction in midstream, then look for a completion that establishes a contrast between ideas. (a) Contrast indicators: These point out how the two things differ. In this type of sentence completion problem, we look for a word that has the opposite meaning (an antonym) of some key word or phrase in the sentence. Some of the common contrast indicators are but, yet, despite, although, however, etc.   Example: Although the warring parties had settled a number of disputes, past experience made them to express optimism that the talks would be a success.  (a) Rash  (b) Ambivalent  (c) Scornful  (d) Reticent   In the sentence, “although” sets up a contrast between what has occurred, success on some issues, and what can be expected to occur – success for the whole talks. Hence, the parties are reluctant to express optimism. The common word “reluctant” is not offered as an answer-choice but we have a synonym which is reticent. (b) Support indicators: These further explain what has already been said. In this type of sentence completion problem, we look for a word that has similar meaning (synonym) of some key word or phrase in the sentence. Some of the common support indicators are and, also, furthermore, likewise, moreover, etc.



  Example: Travis is an opprobrious and speaker, equally caustic toward friend or foe.







  In the sentence, “and” indicates that the missing adjective is similar in meaning to “opprobrious”, which is very negative. Also, vituperative which is the only negative word means abusive. (c) Cause and effect indicators: These words indicate that one thing causes another to occur. Some of the common cause and effect indicators are because, for, thus, hence, therefore, etc.



VA_Chapter 4.indd 524

 (a) Lofty  (b) Vituperative  (c) Retiring  (d) Laudatory



  Example: Because the House has the votes to override a presidential veto, the President has no choice but to .







  Since the House has the votes to pass the bill or motion, the President would be wise to compromise and make the best of the situation.

 (a) Object  (b) Abstain  (c) Capitulate  (d) Compromise

3. Words or phrases in apposition are placed next to each other, and the second word or phrases defines, clarifies or gives evidence to the first word or phrase.

Example: His novels are ; he uses a long ­circumlocution when a direct coupling of a simple subject and verb would be best.



(a) Succinct (c) Vapid



The sentence has no linking words (such as because, although, etc.) Hence, the phrase following the semicolon is in apposition to the missing word, it defines or further clarifies the missing word. Now, writing filled with circumlocutions is aptly described as prolix.

(b) Prolix (d) Risque

4. While reading the sentence, the student should look out for adjectives/adverbs which give the idea of the sentence. After that, find out if the idea of the sentence is positive/negative. If words such as no, non, not, etc., are used then the idea of the sentence is negative. Punctuations can divide the sentence into 2 or 3 parts. If the flow of the first part of the sentence is positive and the second part if negative, then the blank must be negative to even the flow of the sentence. This would solve the sentence completion question without any understanding of the question.

Example: Because he did not want to appear , the junior executive refused to dispute the board’s decision, in spite of his belief that the decision would impair employee morale.



(a) Contentious (c) Solicitous



Options (c) and (d) are gone because they are positive words. Hence, solicitous and steadfast cannot be used. Also, we cannot use indecisive because the clue is “refused to dispute”, which doesn’t work with indecisive. Hence, the best answer is contentious.

(b) Indecisive (d) Steadfast

22-08-2017 06:42:50

PRACTICE EXERCISE     525

the options that are definitely wrong or are eliminated through the positive/negative flow. This helps in increasing the probability of getting the correct answer. It is obvious that the probability of getting a right answer from 2 options is much higher than from 5 options.

5. Be flexible in handling multiple-blank completions. A completion for one blank might make sense in the context of the phrase or sentence in which it appears, yet not work in the context of the passage as a whole.

Example: The attendees at the normally office Christmas party were delighted by the lavish decorations and atmosphere this year.



(a) mundane, exuberant (b) entertaining, ornate (c) litigious, modish (d) monotonous, morose



There seems to be a contrast between how that party was “normally” and how it was “this year”. So, if the party is lavish this year, it probably has been dull or boring in the past. Also, mundane means dull and exuberant is a contrast to it. Thus, that is the correct answer.

6. If all the above techniques fail to obtain the correct answer, then the student can easily eliminate all



Example: Impressed by the doctor’s , the medical students watched intently as he demonstrated the complicated strategy.



(a) Inability (b) Competence (c) Recidivism (d) Importunity



This particular question requires a positive answer since the students are impressed with a certain quality of the doctor. If we consider the four options, we can easily make out that inability cannot be used. Also, recidivism and importunity have no relation to the context in the sentence. Hence, the correct option is competence.

PRACTICE EXERCISE ( a ) (b) ( c ) (d)

Directions for Q. 1 to Q. 15 Each sentence given below consists of four options. The options may be words or phrases. Read each sentence carefully and choose the option which fills in the blank most appropriately.

4. Within hours the tsunami had ended, serving to the fears of thousands of residents who couldn’t evacuate in time.

1. The director is normally lauded for his exciting sci-fi films, but his latest effort was marred by its effects. ( a ) (b) ( c ) (d)

electrifying breathtaking bland emotive

( a ) (b) ( c ) (d)

strength focus hesitancy indolence

3. Although the actress received great reviews during her press conference throughout the country, her much awaited debut film was met with uniformly reviews.

VA_Chapter 4.indd 525

aggravate assuage alleviate annihilate

5. Even after the series of scams, the government had the to talk about the progress of the country in their speeches.

2. Despite his long battle with illness, the boxer showed astonishing in the ring. ( a ) (b) ( c ) (d)

electrifying deprecating appreciative obsequious

( a ) (b) ( c ) (d) 6.

audacity shame morality strength

, he decided to pass on the assignment, his professed support notwithstanding. ( a ) (b) ( c ) (d)

presumably self-indulgently obscurely unexpectedly

22-08-2017 06:42:50

526     Sentence Completion 

7. This is about penetrate. ( a ) (b) ( c ) (d)

a psychological analysis can

the relative distances that as long as  as far as just how far into the subject that

8. Jack hastened to his untied shoelaces.

( c ) gets for others (d) has overcome through it 14. ( a ) (b) ( c ) (d)

when he saw Jade trip over 15.

( a ) (b) ( c ) (d)

assist with support gasp in concern remedy the disaster ease the pain

9. In the world of professional team sports, individual prowess has its place, but ultimately the players are valued chiefly for their qualities. ( a ) (b) ( c ) (d)

singular collaborative dispersive inspirational

10. After a destructive, summer-long drought, during which the crops , the farmers did not know whether to welcome or curse the heave, late-August rains. ( a ) (b) ( c ) (d)

retracted persevered plundered languished

11. Having test-driven this car and finding its performance lacklustre at best, its maker’s sanguine claims are . ( a ) (b) ( c ) (d)

plausible understandable unfounded understated

12. Difficult as it may sometimes be, in all our dealings with both clients and competitors, we must be seen to be above . ( a ) (b) ( c ) (d)

integrity question profit reproach

13. The highest reward for a man’s toil is not what he gets for it but what he . ( a ) becomes by it (b) makes out of it

VA_Chapter 4.indd 526

, the more they remain close to each other. the less the dynamism the more things change the more pronounced the transformation the more the merrier

that in this chaos, the important things are not interfered with. ( a ) (b) ( c ) (d)

it cannot be emphasized enough it is of cardinal important it is important it should be imminently understood

Directions for Q. 16 to Q. 30 Each sentence given below consists of two or three blanks. Read each sentence carefully and choose the option which contains the right combination of answers to fill in the blanks most appropriately. 16. Given the nature of the evidence, the authorities are unlikely to present a case against the defendant. ( a ) (b) ( c ) (d)

superficial, effective rakish, tepid strong, convincing flimsy, convincing

17. It is hard to believe that the highly game of football began many centuries ago as a rowdy without rules, fought cross-country by entire villages. ( a ) (b) ( c ) (d)

complex, s ance structured, brawl unplanned, massacre organized, encounter

18. Ambition is a useful that leads people to greatness, but it can also be force. ( a ) (b) ( c ) (d)

factor, an inspirational indicator, a pulsating motivator, a destructive tenet, a resisting

19. In the workplace, the employees should of the company. ( a ) (b) ( c ) (d)

the

confine to, peccadilloes object to, idiosyncrasies conform to, standards balk at, simplicity

22-08-2017 06:42:51

PRACTICE EXERCISE     527

20. The explorers were almost certainly to failure from the start as the members lacked a sound sense of knowledge. ( a ) (b) ( c ) (d)

doomed, geographical accustomed, general immune, basic destined, abstruse

21. In our country, the challenges are to raise incomes to reduce poverty and to inefficient public sector enterprises. ( a ) (b) ( c ) (d)

rural, restructure farm, liberalise middle-class, privatise workers, take over

unchanged — actors in costume still the stage before audiences who willingly suspend their in order to enter into the `reality’ of events created for them. ( a ) (b) ( c ) (d)

implacable, straddle, interest modest, stalk, logic remarkable, strut, disbelief long, surround, understanding

27. Although the report indicated a rise in obesity, many people, by junk food over nutrition, continue to the problem. ( a ) (b) ( c ) (d)

huge, preferring, avoid disturbing, choosing, compound slow, ignoring, attenuate steep, selecting, abhor

22. The interest generated by the Football World Cup is compared to the way cricket India. ( a ) (b) ( c ) (d)

unusual, grips tepid, inspires milder, fascinates mild, electrifies

23. The ideas that these companies used seem so simple with that their business rivals will now themselves for not thinking of them first. ( a ) (b) ( c ) (d)

passage of time, curse analysis, applaud hindsight, kick technology, hit

24. Since her face was free of , there was no way to if she appreciated what had happened. ( a ) make-up, realise (  b) expression, ascertain ( c ) scars, judge (d) emotion, diagnose 25. The situation in Israel and Palestine is far more than what we understand it to be. While we find those areas unstable, the people who live there are it. ( a ) (b) ( c ) (d)

stable, proud of simple, happy with complex, content by adverse, accustomed to

26. The development of drama over the centuries has been a journey. Yet, much has remained

VA_Chapter 4.indd 527

28. The idea that the Internet is not a place has become ingrained in popular culture. It has become more complicated for authorities to the breaches of privacy that proliferate on a regular basis because of the increasing number of users. The average user should remain the exchange of personal data over the Internet. ( a ) (b) ( c ) (d)

secure, constrain, careful in sensible, castigate, indignant about reliable, relinquish, complacent in trustworthy, stop, sceptical toward

29. A dictionary that provides the of words — that is, the origin and development of their ­meanings — offers proof of a language. Over time, words not only change but sometimes even their meaning. ( a ) (b) ( c ) (d)

etymology, living, reverse taxonomy, faltering, exchange toxicology, nascent, amend etymology, variable, complicate

30. As a result of poor planning and disorganization, the young team attacking the root of the problem. This went to until there was no other recourse left to them. They were obliged to a , last minute solution to the problem. ( a ) (b) ( c ) (d)

accelerated, envision, calculated accelerated, reject, premeditated expedited, implement, measured postponed, implement, desperate

22-08-2017 06:42:51

528     Sentence Completion 

ANSWERS 1. (c)

7. (c)

13. (a)

19. (c)

25. (d)

2. (a)

8. (a)

14. (b)

20. (a)

26. (c)

3. (b)

9. (b)

15. (c)

21. (a)

27. (b)

4. (b)

10. (d)

16. (d)

22. (d)

28. (a)

5. (a)

11. (c)

17. (b)

23. (c)

29. (a)

6. (d)

12. (d)

18. (c)

24. (b)

30. (d)

VA_Chapter 4.indd 528

22-08-2017 06:42:51

CHAPTER 5

VERBAL ANALOGIES

In verbal analogies, the student is given one or more pair(s) of related words and another word without its pair. The answer is the word(s) that has the same relationship to the word as the initial pair(s). Verbal analogies assess the student’s ability to identify the relationship between words and then to apply this verbal analogy. These questions provide excellent training in seeing relationships between concepts. From a practical standpoint, verbal analogies always appear in standardized tests like SAT, GRE and other professional exams. Also, employers may use these word comparisons on personnel and screening tests to determine an applicant’s quickness and verbal acuity.







TYPES OF ANALOGIES



There are many ways of forming a relationship between pairs of words in verbal analogies. Some of the common relationships are given as follows:



1. Things that go together analogies: Some objects are indisputably connected to each other.

VA_Chapter 5.indd 529

These sets of objects are extensively used in modern verbal analogies. Example: bat/ball, salt/pepper, etc. 2. Opposite analogies: As the name suggests, this analogy type contains things that are opposites. This is a common analogy type which is encountered fairly often and pretty straightforward. Example: black/white, big/small, etc. 3. Similar analogies: As the name suggests, this analogy type contains things that are similar. Like opposites analogies, this is a common analogy type which is encountered fairly often and pretty straightforward. Example: cold/icy, hide/conceal, etc. 4. Classification of objects: Objects can be a classification, a group of objects to which they belong. Most objects can even be classified to several different groups. Example: red/colour, grapes/fruit, etc. 5. Objects and group analogies: These are objects which form a specifically named group when several are put together. Example: bird/flock, cow/herd, etc. 6. Object and characteristic analogies: This type of analogy contains an object and an adjective describing the object. In this analogy, a single object can have different analogies depending on the adjectives being used. Example: grass/green, ball/round, etc.

22-08-2017 06:43:19

530     Verbal Analogies 

7. Object and a part of the whole analogies: This type of analogy is sometimes confused with the object and group analogy. The difference lies in the fact that in the object and part of a whole relation the object is not automatically the whole when lots of the objects are brought together. Example: hand/fingers, year/month, etc. 8. Object and function analogies: Some objects have designated functions which are inseparably connected to the concerning object. Example: knife/cut, book/read, etc. 9. Object and location analogies: In this relation objects are designated to their most logical location. However, this is not always strictly defined and the student will have to find the correct answer again by carefully analysing the analogy problem and its possible solutions. Example: plane/hangar, tree/forest. 10. Performer and action analogies: As the name suggests, one of the word is the performer and the other is the action which is performed. These questions are pretty easy to spot and answer. Example: magician/magic, actor/act, etc. 11. Verb tenses analogies: This type of analogy in which two tenses of a verb are analogous to two of the same tenses of another verb. Example: eat/ate, drive/drove, etc. 12. Cause and effect analogies: The similarity in this type of analogies derives from the cause on one side and its indisputably connected effect on the other side. The student should be careful as to not to confuse this type of analogy with the “effort and result”. Example: fire/burn, trip/fall, etc. 13. Effort and result analogies: The difference between this analogy and “cause and effect” is the

fact that for the effort and result connection an actual effort has to be made. Example: paint/painting, write/letter, etc. 14. Things that are in sequence analogies: This type of analogies is also easy to spot as it contains the words which are in direct sequence to each other. Example: Monday/Tuesday, day/night, etc. 15. Degrees of characteristic analogies: This type of analogies can be explained by the following scenario. In the case of warm and hot, one degree higher then warm can be hot and another degree higher can be burning. Similarly, one degree higher than cold can be freezing. Example: tired/ exhausted, cold/freezing, etc.

TIPS TO SOLVE VERBAL ANALOGIES QUESTIONS Some of the common methodologies to solve the questions of verbal analogies are given as follows: 1. Determine the relationship between the first pair of words. 2. Eliminate all the pairs in the answer choices that don’t have the same relationship. The remaining choice is the correct answer. 3. Another strategy is to read the verbal analogies in sentences. 4. Sometimes it is difficult to identify the relationship by just looking at the analogy in the given order and hence sometimes it is helpful to switch the words to obtain a relationship. 5. Consider alternate meaning of words.

PRACTICE EXERCISE 4. If light : blind then

1. If paw : cat then hoof : ? (a) elephant (c) lion

(b) horse (d) lamb

3. If pain : sedative then (a) comfort : stimulant (b) trance : narcotic (c) grief : consolation (d) ache : extraction

VA_Chapter 5.indd 530

(b) language : deaf (d) voice : vibration

5. If after : before then

2. If thrust : spear then (a) mangle : iron (c) scabbard : sword

(a) speech : dumb (c) tongue : sound

(b) bow : arrow (d) fence : epee

(a) first : second (b) primary : secondary (c) contemporary : historic (d) successor : predecessor 6. If distance : mile then (a) liquid : litre (c) weight : scale

(b) bushel : corn (d) fame : television

22-08-2017 06:43:19

ANSWERS     531

17. If extort : obtain then

7. If ten : decimal then (a) seven : septet (c) two : binary

(a) pilfer : steal (b) explode : ignite (c) plagiarize : borrow (d) consider : appeal

(b) four : quartet (d) five : quince

18. If seconds : minute then

8. If army : logistics then (a) soldier : students (c) team : individual

(a) time : clock (b) minute : speed (c) running : athleticism (d) fingers : hand

(b) business : strategy (d) war : guns

9. If gravity : pull then (a) iron : metal (b) north pole : directions (c) magnetism : attractions (d) dust : desert

19. Cup is to coffee as bowl is to (a) soup (c) spoon

20. Optimist is to cheerful as pessimist is to

10. If scribble : write then stammer : ? (a) shout (c) dance

(a) mean (c) helpful

(b) weep (d) speak

(a) Divide : unite (b) Song : music (c) Incongruous : asymmetrical (d) Collate : order

(b) pen (d) paint

12. If ornithologist : birds then anthropologist : ? (a) mankind (c) animals

(b) environment (d) plants

22. If outmanoeuvre : strategy then (a) Bait : lion (b) Lure : decoy (c) Competition : product (d) Entice : tempt

13. If calf : cow then puppy : ? (a) horse (c) buffalo

(b) dog (d) bitch

23. If aphorism : terse then

14. If incandescent : glowing then

(a) Draught : game (c) Eulogy : praise

(a) indefatigable : untiring (b) flash : flame (c) tedious : bore (d) fast : run

(a) Wave : water (c) Incubate : nurture

(b) pessimistic : optimistic (d) malicious : deceitful

(b) Lament : grief (d) Beauty : vanity

25. If moratorium : payment then

16. If medicine : illness then (a) hunger : thirst (c) love : treason

(b) Malign : meagre (d) Holistic : cycle

24. If preen : pride then

15. If indigent : wealthy then (a) healthy : wealthy (c) indigent : rude

(b) petty (d) gloomy

21. If muster : gather then

11. If sculptor : statue then poet : ? (a) canvas (c) verse

(b) dish (d) food

(b) law : anarchy (d) stimulant : sensitivity

(a) Fraud : embezzlement (b) Flush : anger (c) Retort : insolent (d) Reprieve : punishment

ANSWERS 1. (b)

6. (a)

11. (c)

16. (b)

21. (c)

2. (d)

7. (c)

12. (a)

17. (c)

22. (b)

3. (c)

8. (b)

13. (d)

18. (d)

23. (c)

4. (a)

9. (c)

14. (a)

19. (a)

24. (a)

5. (d)

10. (d)

15. (b)

20. (d)

25. (d)

VA_Chapter 5.indd 531

22-08-2017 06:43:19

532     Verbal Analogies 

EXPLANATIONS AND HINTS 1. (b) Paw is foot of Cat. Hoof is foot of Horse. 2. (d) Thrust is the act whereas spear is the object. Hence, the option which comes closest is fence : epee. 3. (c) Here, pain is the problem and sedative is the ­solution. Hence, the option grief : consolation is the correct answer. 4. (a) A person who is blind cannot see the light, and a person who is dumb cannot speak. 5. (d) After and before represent different time spans and hence the answer successor : predecessor. 6. (a) Distance is the object and mile is the quantity of measurement. Similarly, liquid is the object and litre is the quantity of measurement. 7. (c) Decimal number system has a base ten and a binary number system has a base two. 8. (b) Army use logistics to perform their operations. Similarly, business use strategies to perform their operations. 9. (c) Gravity causes the action of pull and similarly, magnetism causes the action of attraction. 10. (d) People scribble while writing and people stammer while speaking. 11. (c) A sculptor creates a statue like a poet creates a verse. 12. (a) Ornithologist is a study of birds and anthropologist is a study of mankind. 13. (d) Cow is the mother of a calf while bitch is the mother of a puppy. 14. (a) Incandescent and glowing are synonyms to each other. Similarly, indefatigable and untiring are synonyms of each other.

VA_Chapter 5.indd 532

15. (b) Indigent and wealthy are antonyms of each other. Similarly, pessimistic and optimistic are antonyms of each other. 16. (b) Medicine is the solution of illness as law is the ­solution or answer to anarchy. 17. (c) To extort something is to obtain something by force. Plagiarism is borrowing material from another writer without giving him/her the ­acknowledgement. 18. (d) Minutes can be divided into seconds. Hence, seconds are a part of an object which is minutes. Similarly, fingers are a part of a hand. 19. (a) Coffee goes into a cup and soup goes into a bowl. Dish and spoon are incorrect since they are utensils. Option (d) that is, food, is too general. Hence, the correct option is bowl. 20. (d) An optimist is a person whose outlook is positive or cheerful. A pessimist is a person whose outlook is negative or gloomy. 21. (c) Muster and gather are both synonyms which mean collect. Incongruous and asymmetrical both are synonyms too which mean harmony. 22. (b) Function and purpose. The purpose of strategy is to outmanoeuvre the adversary or enemy, the purpose of a decoy is to lure and trap the enemy. 23. (c) The chief characteristic of aphorism is terseness pithy, as praise is of eulogy. 24. (a) Preen is an action that demonstrates pride. Lament is an action that signifies grief. 25. (d) Moratorium is a delay in payment whereas reprieve is a delay in punishment.

22-08-2017 06:43:19

CHAPTER 6

WORD GROUPS

The questions on word groups are designed to check the vocabulary of the students and their understanding of word meanings. Hence, the questions focus on relationships between the words and the questions are framed such that the student needs to know the exact meaning of the words given, in order to select the correct answer. Questions on word groups often require the student to have a good knowledge about synonyms, antonyms, dictionary definitions and word pairs. These questions also include words which sound similar but have different meanings. Such words are called homophones. A list of homophones is given as follows: Ad, add Ail, ale Air, heir All, awl Allowed, aloud Altar, alter Arc, ark Ate, eight Auger, augur Aural, oral Away, aweigh

VA_Chapter 6.indd 533

Axel, axle Bail, bale Bait, bate Ball, bawl Band, banned Bard, barred Berth, birth Billed, build Boarder, border Brake, break Call, caul Cast, caste Cede, seed Ceiling, sealing Cell, sell Cent, scent, sent Cereal, serial Chord, cord Coarse, course Cue, queue Currant, current Dam, damn

22-08-2017 06:43:55

534     Word Groups 

Dear, deer Desert, dessert Deviser, divisor Dew, due Die, dye Dual, duel Faint, feint Fair, fare Faun, fawn Feat, feet Flea, flee Flour, flower Forth, fourth Foul, fowl Gamble, gambol Genes, jeans Groan, grown Hail, gale Hair, hare Hall, haul Hart, heart Hear, here Heard, herd Hour, our Idle, idol Knead, need Knight, night Knob, nob Knock, nock Know, no Lac, lack Lade, laid Laps, lapse Leach, leech Lead, led Leak, leek Liar, lyre Loan, lone Loot, lute Made, maid Mail, male Main, mane Maize, maze Mall, maul Manna, manner Mantel, mantle Marshal, martial Meat, meet Medal, meddle

VA_Chapter 6.indd 534

Might, mite Mind, mined Naval, navel Nun, none Ode, owed One, won Packed, pact Pain, pane Pair, pear Pause, paws Pea, pee Peace, piece Peak, peek Pearl, purl Pedal, peddle Plain, plane Practice, practise Principal, principle Profit, prophet Quarts, quartz Rain, reign Raise, rays Rap, wrap Read, reed Real, reel Right, write Ring, wring Role, roll Rood, rude Rough, ruff Sale, sail Satire, satyr Soar, sore Sea, see Seam, seem Shear, sheer Shoe, shoo Slay, sleigh Sole, soul Some, sum Son, sun Sort, sought Stake, steak Storey, story Straight, strait Tale, tail Talk, torque Team, teem Threw, through

22-08-2017 06:43:55

PRACTICE EXERCISE     535

Throne, thrown Tide, tied Tire, tyre Tough, tuff Vain, vane, vein Waist, waste Wait, weight Ware, wear, where

Watt, what Weak, week Weather, whether Which, witch Whine, wine Wood, would Yaw, yore, your Yoke, yolk

PRACTICE EXERCISE 1. Which of the following words does not have a similar meaning to energize? (a) rejuvenate (c) enervate

(b) strengthen (d) uplift

2. Which of the following words mean tuneful or marked with agreement? (a) harmonious (c) exclusive

(b) inclusive (d) lucid

3. Which of the following words mean to accumulate or to gather? (a) expend (c) abjure

(b) deliberate (d) amass

4. Which of the following words mean modest or lack of self-confidence? (a) wanton (c) trite

(b) diffident (d) barren

5. Which of the following words does not have a similar meaning to comprise? (a) compose (c) contain

(b) cover (d) encompass

6. Which of the following words does not have a similar meaning to densely populated? (a) populace (c) populous

(b) crowded (d) packed

7. Which of the following words has/have meaning nearly opposite to pragmatic? (a) exuberant (c) philosophical

(b) impractical (d) realistic

8. Which of the following words has/have meaning nearly opposite to absolve? (a) pardon (c) free

(b) condemn (d) exonerate

9. Which of the following words has/have meaning nearly opposite to exigent?

VA_Chapter 6.indd 535

(a) strenuous (c) easy

(b) light (d) difficult

10. Which of the following words has/have meaning similar to condescend? (a) usurp (c) patronize

(b) respectful (d) contribute

11. Which of the following words has/have meaning similar to contradict? (a) gainsay (c) tarnish

(b) disparage (d) oppose

12. Which of the following words has/have meaning similar to perplex? (a) reiterate (c) dither

(b) affiliate (d) discomfit

13. Which of the following words has/have meaning similar to expedite? (a) facilitate (c) exterminate

(b) disrespect (d) beckon

14. Which of the following words has/have meaning nearly opposite to fecund? (a) abundant (c) unfriendly

(b) barren (d) productive

15. Which of the following words has/have meaning nearly opposite to malicious? (a) vile (c) divine

(b) corrupt (d) malign

16. Which of the following statement is incorrect? (a) (b) (c) (d)

I I I I

read the whole book in one day. have a whole in my trousers. ate whole of the pie. will finish whole of my homework.

17. This is

difficult for me

(a) too (c) both (a) and (b)

(b) to (d) none of the above

22-08-2017 06:43:56

536     Word Groups 

18. Which of the following statement is incorrect? (a) (b) (c) (d)

20. Which of the following statement is incorrect?

He sailed all the way to the shore. I bought this watch during the winter sale. I will buy a watch during the next sale. The boat will sale across the ocean.

19. Can you

(a) (b) (c) (d)

David is a good writer. You did the right thing. When you reach the crossing, turn right. Can you help him right his letter?

me?

(a) here (c) both (a) and (b)

(b) hear (d) none of the above

ANSWERS 1. (c)

5. (b)

9. (c)

13. (a)

17. (a)

2. (a)

6. (a)

10. (c)

14. (b)

18. (d)

3. (d)

7. (b)

11. (d)

15. (c)

19. (b)

4. (b)

8. (b)

12. (d)

16. (b)

20. (d)

EXPLANATIONS AND HINTS 1. (c) Rejuvenate, strengthen and uplift have a ­similar meaning to the word energise. However, enervate means lacking in energy. Hence, the answer is ­enervate. 2. (a) Harmonious means tuneful or free from dis­agreement. 3. (d) Amass means to accommodate or to gather. 4. (b) Diffident means to be shy, modest or having a lack of self-confidence. 5. (b) Contain, compose and encompass are synonyms of comprise. Hence, the answer is cover. 6. (a) Populous, crowded and packed are more or less synonyms of densely populated. However, populace means people living in a particular area. 7. (b) Pragmatic means being realistic or having a realistic approach to something. Hence, the word meaning opposite to it is impractical. 8. (b) Absolve means to free someone from guilt. It is clear that pardon, free and exonerate are synonyms of absolve. Hence, the answer is condemn. 9. (c) Exigent means to be pressing and demanding. Hence, the most appropriate word meaning opposite to it is easy.

11. (d) To contradict is to speak against. Hence, the word similar in meaning is oppose. 12. (d) Perplex means to make somebody feel uncomfortable or uneasy. Hence, the word having a meaning similar to perplex is discomfit. 13. (a) Expedite means to cause an action. Hence, the word having a meaning similar to expedite is ­facilitate. 14. (b) Fecund means to be fertile. Hence, the word having a meaning opposite to fecund is barren. 15. (c) Malicious means to be immoral. Hence, vile, corrupt and malign are synonyms of malicious. The answer is divine. 16. (b) The answer is (b). The correct sentence should be, I have a hole in my trousers. 17. (a) This is too difficult for me. Hence, the answer is too. 18. (d) The boat will sail across the ocean. Hence, the answer is (d). 19. (b) Can you hear me? Hence, the answer is (b). 20. (d) Can you help him write his letter? Hence, the answer is (d).

10. (c) Condescend means to show that one feels superior. Hence, the word similar to it is patronize.

VA_Chapter 6.indd 536

22-08-2017 06:43:56

CHAPTER 7

VERBAL DEDUCTION

Verbal deduction questions are not designed to measure a student’s facility with English. These are designed to test the ability to take a series of facts expressed in words and to understand and manipulate the information to solve a specific problem. Verbal deduction problems usually follow the pattern in which a paragraph is given and then certain conclusions are asked to be made from the information in the paragraph. Another type of problems involves a statement followed by the arguments and the student is asked to choose as to which of the arguments best support the statement.

TIPS AND TRICKS TO SOLVE VERBAL DEDUCTION QUESTIONS Some of the common methodologies to solve the questions of verbal deduction questions are given as follows:

VA_Chapter 7.indd 537

1. Scan for keywords in the questions. Read the comprehension or the directions given and mostly you will be able to find a hint which gives the answer. This is not something which is applicable for every question, but it’s a very handy tool. 2. Make no assumptions. Verbal deduction questions are meant to be answer absolutely literally. Hence, if it isn’t included in the passage then you cannot include it in your decision-making process for the questions. Don’t factor in real-life intelligence that you know proves or disproves a statement. 3. If you are stuck, try starting at the end of the sentence. A great way to unravel a confusing piece of writing is to start at the end of the sentence and work backwards. For long statements that make contradictory points and circular references this can be very useful in decoding their meaning. 4. We can use the process of elimination in which we can start with eliminating the choices which we are sure are not the answer in order to increase the odds of getting the correct answer.

22-08-2017 06:44:22

538     VERBAL deduction 

PRACTICE EXERCISE Directions for Q. 1 to Q. 8 In the questions below are given two or three statements followed by conclusions. Take the given two statements to be true even if they seem to be at variance from commonly known facts. Read the conclusion and then decide which of the given conclusion(s) logically follows from the two given statements. 1. Statement: No women teachers can play. Some women teachers are athletes.

Conclusions: (A) Male athletes can play. (B) Some athletes can play. (a) Only conclusion (A) follows (b) Only conclusion (B) follows (c) Neither (A) nor (B) follows (d) Both (A) and (B) follow

2. Statement: Some blankets are beds. Some pillows are blankets. All beds are pillows.

Conclusions: (A) Some blankets are pillows. (B) Some pillows are beds. (C) Some beds are blankets. (a) (b) (c) (d)

Only conclusion (A) or (B) follows  nly conclusion (A) and either (B) or (C) follows O Only (C) and either (A) or (B) follows All (A), (B) and (C) follow

3. Statement: Some lawyers are fools. Some fools are rich.

Conclusions: (A) Some lawyers are rich. (B) Some rich are doctors. (a) Only conclusion (A) follows (b) Only conclusion (B) follows (c) Neither (A) nor (B) follows (d) Both (A) and (B) follow

4. Statement: All mangoes are golden in colour. No golden-coloured things are cheap.

Conclusions: (A) All mangoes are cheap. (B) G  olden-coloured mangoes are not cheap. (a) Only conclusion (A) follows (b) Only conclusion (B) follows (c) Neither (A) nor (B) follows (d) Both (A) and (B) follow

5. Statement: No bat is ball. No ball is wicket.

Conclusions: (A) No bat is wicket. (B) All wickets are bats. (a) Only conclusion (A) follows (b) Only conclusion (B) follows

VA_Chapter 7.indd 538

(c) Neither (A) nor (B) follows (d) Both (A) and (B) follow 6. Statement: Some swords are sharp. All swords are rusty. Conclusions: (A) Some rusty things are sharp. (B) Some rusty things are not sharp. ( a) (b) ( c ) (d)

Only conclusion (A) follows Only conclusion (B) follows Neither (A) nor (B) follows Both (A) and (B) follow

7. Statement: All fishes are blue in colour. Some fishes are heavy. Conclusions: (A) All heavy fishes are blue in colour. (B)  All light fishes are not blue in colour. (a) (b) (c) (d)

Only conclusion (A) follows Only conclusion (B) follows Neither (A) nor (B) follows Both (A) and (B) follow

8. Statement: All snakes are trees. Some trees are roads. All roads are mountains. Conclusions: (A) Some mountains are snakes. (B) Some roads are snakes. (C) Some mountains are trees. (a) (b) (c) (d)

Only conclusion (A) follows Only conclusion (B) follows Only conclusion (C) follows Conclusions (A), (B) and (C) follows

Directions for Q. 9 to Q. 16 In the questions below is given a statement followed by arguments to support or oppose it. Read the arguments and then decide which of the given argument(s) logically follows from the given statement. 9. Statement: Should India have no military force at all? Arguments: (A) N  o. Other countries in the world would not believe in non-violence. (B) Yes. Many Indians believe in non-violence. (a) (b) (c) (d)

Only argument (A) is true Only argument (B) is true Either (A) or (B) is true Neither (A) nor (B) is true

10. Statement: Should the articles of only deserving authors be allowed to be published? Arguments: (A) Yes. It will save a lot of paper which is in short supply.

22-08-2017 06:44:22

PRACTICE EXERCISE     539



(a) (b) (c) (d)

(B) N  o. It is not possible to draw a line between the deserving and the undeserving. Only argument (A) is true Only argument (B) is true Either (A) or (B) is true Neither (A) nor (B) is true

11. Statement: Should India be a permanent member of the UN’s Security Council?

Arguments: (A) Y  es. India is a country who loves peace and amity.

(a) (b) (c) (d)

(B) N  o. Let us first solve the problems of our own people like poverty, malnutrition etc. Only argument (A) is true Only argument (B) is true Either (A) or (B) is true Neither (A) nor (B) is true

12. Statement: Should airlines immediately stop issuing free passes for all of its employees?

Arguments: (A) N  o. The employees have the highest right to travel free. (B) Y  es. This will help airlines provide better facilities. (e) Only argument (A) is true (f) Only argument (B) is true (g) Either (A) or (B) is true (h) Neither (A) nor (B) is true

13. Statement: Should a total ban be put on trapping wild animals?

Arguments: (A) Y  es. People who trap the animals are making a lot of money. (B) N  o. Bans on hunting and trapping are not effective. (a) (b) (c) (d)

Only argument (A) is true Only argument (B) is true Either (A) or (B) is true Neither (A) nor (B) is true

14. Statement: Should cutting of trees be banned altogether?

Arguments: (A) Y  es. People who trap the animals are making a lot of money. (B) N  o. Bans on hunting and trapping are not effective. (a) (b) (c) (d)

VA_Chapter 7.indd 539

Only argument (A) is true Only argument (B) is true Both (A) and (B) are true Neither (A) nor (B) is true

15. Statement: Should jobs be linked with academic degrees and diplomas? Arguments: (A) No. A very large number of persons with meagre academic qualifications will apply. (B) No. Importance of higher education will be diminished. (a) (b) (c) (d)

Only argument (A) is true Only argument (B) is true Either (A) and (B) is true Neither (A) nor (B) is true

16. Statement: Should judiciary be independent of the executive? Arguments: (A) Yes. This would help curb the unlawful activities of the executives. (B) No. The executive would not be able to take the bold measures. (a) (b) (c) (d)

Only argument (A) is true Only argument (B) is true Either (A) and (B) is true Neither (A) nor (B) is true

17. Shoshana has never received a violation from the Federal Aviation Administration during her 20-year flying career. Shoshana must be a great pilot. Which of the following can be said about the reasoning above? (a) The definitions of the terms create ambiguity. (b) The argument uses circular reasoning. (c) The argument works by analogy. (d) The argument is built upon hidden assumptions. 18. No national productivity measures are available for underground industries that may exist but remain unreported. On the other hand, at least some industries are run entirely by self-employed industrialists are included in national productivity measures. Which of the following facts can be validly concluded from the information given above? (a) T  here are at least some industries run entirely by self-employed industrialists that are not underground industries. (b) There are at least some industries other than those run entirely by self-employed industrialists that are underground industries. (c) No industries that are run entirely by selfemployed industrialists operate underground. (d) There are at least some industries run entirely by self-employed industrialists that are underground industries.

22-08-2017 06:44:22

540     VERBAL deduction 

19. Abate observes that if flight 409 is canceled, then the manager could not possibly arrive in time for the meeting. But the flight was not canceled. Therefore, Abate concludes, the manager will certainly be on time. Eve replies that even if Abate’s premises are true, his argument is fallacious. And therefore, she adds, the manager will not arrive on time after all. Which of the following is the strongest thing that we can properly say about this discussion? (a) E  ve is mistaken in thinking Abate’s argument to be fallacious, and so her own conclusion is unwarranted. (b) Eve is right about Abate’s argument, but nevertheless her own conclusion is unwarranted. (c) Both (a) and (b) (d) None of the above 20. No national productivity measures are available for underground industries that may exist but remain unreported. On the other hand, at least some industries that are run entirely by self-employed industrialists are included in national productivity measures.

shape to cane sugar. Gradually the grains grow larger and the air pockets between the grains get smaller, meaning that the snow slowly becomes denser. After about two winters, the snow turns into firn, an intermediate state between snow and ice. Over time the larger ice crystals become more compressed and even denser, this is known as glacial ice. Glacial ice, because of its density and ice crystals, often takes a bluish or even green hue. 21. Firn is less dense than (a) (b) (c) (d)

 .

Glacial ice, ice crystals Ice crystals, glacial ice Ice, snow Snow, ice

22. The increase in density is caused by the grains becoming smaller. (a) (b) (c) (d)

True False True only up to a point Indeterminate

23. Over time the larger ice crystals become more compressed and even denser, this is known as  .

From the information given above, it can be validly concluded that (a) N  o industries that are run entirely by selfemployed industrialists operate underground. (b) There are at least some industries other than those run entirely by self-employed industrialists that are underground industries. (c) Both (a) and (b). (d) There are at least some industries run entirely by self-employed industrialists that are not underground industries.

, but more dense than

(a) (b) (c) (d)

Ice crystals Glacier ice Snow None of the above

24. Glaciers cannot form where snow does not remain all year round. (a) (b) (c) (d)

True False True for North Pole and false for South Pole Indeterminate

25. Which of the following colour does glacial ice often take?

Directions for Q. 21 to Q. 25 Glaciers begin to form where snow remains yearround and enough of it accumulates to transform into ice. New layers of snow compress the previous layers and this compression forces the icy snow to recrystallize, forming grains similar in size and

(a) (b) (c) (d)

Green Blue Red Green or blue

ANSWERS 1. (c)

6. (a)

11. (a)

16. (a)

21. (c)

2. (d)

7. (a)

12. (d)

17. (d)

22. (b)

3. (c)

8. (c)

13. (d)

18. (a)

23. (b)

4. (b)

9. (d)

14. (c)

19. (b)

24. (a)

5. (c)

10. (b)

15. (b)

20. (d)

25. (d)

VA_Chapter 7.indd 540

22-08-2017 06:44:22

EXPLANATIONS AND HINTS     541

EXPLANATIONS AND HINTS 1. (c) Since one premise is negative, the conclusion must be negative. Hence, neither (A) nor (B) follows. 2. (d) (A) is the converse of (B)’s premise, (B) is the converse of (C)’s premise and (C) is the converse of (A)’s premise and hence, all (A), (B) and (C) follow. 3. (c) Since both the premises are particular, no definite conclusion follows. 4. (b) Clearly, the conclusion must be universal negative and should not contain the middle term. So, it follows that `No mango is cheap’. Thus, (A) does not follow. Also, since all mangoes are golden in colour, we may substitute mangoes with `golden coloured mangoes’. Thus, (B) follows. 5. (c) Since both the premises are negative, no definite conclusions follow. 6. (a) Since one premise is particular, the conclusion must be particular and should not contain the middle term. Hence, (A) follows. Since both the premises are affirmative, the conclusion cannot be negative. Thus, (B) doesn’t follow. 7. (a) Since one premise is particular, the conclusion must be particular and should not contain the middle term. So, it follows that `Some heave things are blue in colour’. Thus, only (A) holds. 8. (c) All snakes are trees. Some trees are roads. Since the middle term is not distributed even once in the premises, no conclusion follows. Some trees are roads. All the roads are mountains. Since one premise is particular, the conclusion must be particular and should not contain the middle term. So, it follows that `Some trees are mountains’. (C) is the converse of this statement and hence, it follows. 9. (d) Clearly, India (or any country for that matter) needs to maintain military force in order to defend itself against the threats of other military powers in the world. Hence, none of the arguments hold strong.

11. (a) Argument (A) is strong since it talks about the characteristics of the India which are related to the statement. However, argument (B) has no connection to the statement and hence it does not hold. 12. (d) Free passes given to the airway employee is a privilege to them and not their right, hence argument (A) does not hold. Also, argument (B) is vague. 13. (d) The ban is necessary only to prevent our environment and wildlife. It has nothing to do with how much money the people who trap the animals make or how effective they are. Hence, both the arguments do not hold. 14. (c) Both the arguments (A) and (B) are strong, valid and are related to the statement. Hence, both the arguments hold. 15. (b) Delinking jobs with degrees will diminish the need for higher education as many people pursue such education for jobs. So, only argument (B) is strong. 16. (a) Clearly, independent judiciary is necessary for impartial judgements so that the executive does not take the wrong measure. Hence, only argument (A) holds. 17. (d) The fact that Shoshana never violated any rules doesn’t mean she was a great pilot. It simply ­confirms the fact that she was disciplined. The argument is built upon hidden assumptions. 18. (a) There are at least some industries run entirely by self-employed industrialists that are not underground industries. This is the only apt conclusion from the passage. 19. (b) According to the above passage only the argument “Eve is right about Abate’s argument, but nevertheless her own conclusion is unwarranted” is true. 20. (d) The only apt conclusion is “there are at least some industries run entirely by self-employed industrialists that are not underground industries.”

10. (b) Argument (A) does not provide a strong enough reason in support of the statement.

21. (c) We know that snow cools into firn and firn cools into ice. Hence, ice is denser than firn and similarly, firn is denser than snow.

Also, it is not possible to analyse the really deserving and not deserving. Hence, argument (B) holds true.

22. (b) The increase in density is caused by the grains becoming larger. Hence, the given statement is false.

VA_Chapter 7.indd 541

22-08-2017 06:44:22

542     VERBAL deduction 

23. (b) As it can be seen from the given passage, `Over time the larger ice crystals become more ­compressed and even denser, this is known as glacier ice.’ 24. (a) Glaciers are only formed where ice is present all year round. Hence, the statement

VA_Chapter 7.indd 542

“Glaciers cannot form where snow does not remain all year round.” is true. 25. (d) It can be deducted from the passage that “glacial ice, because of its density and ice crystals, often takes a bluish or even green hue.” Hence, the answer is option (d).

22-08-2017 06:44:22

GENERAL APTITUDE

SECTION B NUMERICAL ABILITY

Chapter 01-1.indd 543

22-08-2017 06:49:18

Chapter 01-1.indd 544

22-08-2017 06:49:18

UNIT 1: BASIC ARITHMETIC

Chapter 01-1.indd 545

Chapter  1.

Number System

Chapter  2.

Percentage

Chapter  3.

Profit and Loss

Chapter  4.

Simple Interest and Compound Interest

Chapter  5.

Time and Work

Chapter  6.

Average, Mixture and Alligation

Chapter  7.

Ratio, Proportion and Variation

Chapter  8.

Speed, Distance and Time

25-08-2017 01:21:22

Chapter 01-1.indd 546

22-08-2017 06:49:18

CHAPTER 1

NUMBER SYSTEM

NUMBERS

Classification of Numbers

Natural numbers include all positive integers. For example 1, 2, 3, 4, ….

Numbers can be more specifically classified into more categories than just natural numbers as depicted in Table 1.

A natural number larger than unity is a prime number if it does not have other divisors except for itself and unity. For example 2, 3, 5, 7, 11, 13, ….



All other natural numbers that are not prime numbers are composite numbers. For example 4, 6, 8, 9, 10, 12, …. Some important formulas of natural numbers are as follows: n(n + 1) . 2 2. Sum of squares of first n natural numbers, n(n + 1)(2n + 1) ∑ n2 = . 6 1. Sum of first n natural numbers, Sn = ∑ n =

3. Sum of cubes of first n natural numbers,  n(n + 1)  2  n2 (n + 1)2  2  = ∑ n3 =  = [∑ n] .    2 4   

Chapter 01-1.indd 547

Table 1 |   Classification of numbers into  

various categories Class

Symbol

Description

Natural number

N

Natural numbers are defined as non-negative counting numbers: N = {0, 1, 2, 3 …}.

Integer

Z

Integers extend N by including the negative numbers: Z = {…, −3, −2, −1, 0, 1, 2, 3, …}.

Rational number

Q

A rational number is the ratio of two integers such that: Q = {x/y where x, y ∈ Z, y ≠ 0} For example: 3/2, 2.5, etc. (Continued )

22-08-2017 06:49:19

548     Number System 

Table 1 |   (Continued)

Class

Symbol

Irrational number

P

Real number

Imaginary number

Description

AP can be represented as a, a + d, a + 2d, [a + (n − 1)d ].  Here, quantity d is to be added to any chosen term to get the next term of the progression.

Irrational numbers are those which cannot be represented as fractions: P = { 2 , 3 }

Geometric Progressions

R

Real numbers are all the numbers on a number line. It is the union of all the rational numbers and all irrational numbers.

A geometric progression (GP) is a sequence of numbers where each term after the first is found by multiplying the previous one by a fixed, non-zero number called the common ratio. In other words, any term of a GP can be obtained by multiplying the preceding number by the common ratio.

I

An imaginary number is a number whose square is a negative real number, and is denoted by i. Hence, i2 = −1. For example: −3i, 5i, etc. Sometimes, j is used instead of i.

Let a be the first term, n be the number of terms, r be the common ratio, an be the nth term, Sn be the sum of the first n terms and S be the sum of the entire sequence. Then, an = ar(n −1) Sn =

Complex number

C

A complex number consists of two parts, real and imaginary numbers. It can be represented as a + ib. For example: 7 + 5i.

PROGRESSIONS

a(1 − r n ) if r < 1 1− r

a(r n − 1) if r > 1 r −1 r × last term − first term S= r −1

Sn =

GP can be represented as a, ar, ar2, …. Here, a is the first term and quantity r is the common ratio of the ­geometric progression.

Infinite Geometric Progressions When the numbers in a sequence are related to each other either linearly or geometrically, then the sequence is a progression. This section discusses the type of progressions and the formulae used to calculate the sequences.

Arithmetic Progressions An arithmetic progression (AP) is a sequence of numbers such that the difference between the consecutive terms is constant. This constant value is called common difference. In other words, any term of an AP can be obtained by adding common difference to the preceding term. Let a be the first term, n be the number of terms, d be the common difference, an be the nth term, Sn be the sum of first n terms and S be the sum of the entire sequence. Then,

If −1 < r < + 1 or r < 1, then sum of a geometric progression does not increase infinitely; it converges to a particular value. Such a GP is called infinite geometric progression. a Sum of an infinite GP, S∞ = . 1− r

AVERAGES, MEAN, MEDIAN AND MODE A central value around which a group of values shows a tendency to concentrate is called average. Thus, an average is a single value that is in some way indicative of a group of values. The following are five measures of central tendency:

an = a + (n − 1) × d Sn = S=

Chapter 01-1.indd 548

n × [2a + (n − 1) × d ] 2 n ×(first term + last term) 2

1. Arithmetic mean (AM): The most commonly used average is the AM or simply the average. AM of n numbers x1, x2, x3, …, xn is denoted by X and calculated as x + x2 + x3 +  + xn X= 1 n

22-08-2017 06:49:21

SOME ALGEBRAIC FORMULAS     549

n

∑ xi ⇒X=

i =1

n If values x1, x2, x3, …, xn are assigned weights w1, w2, w3, …, wn, respectively, then the weighted arithmetic mean is given as follows:

∑ i=1 wixi Weighted arithmetic mean, xw = n ∑ i=1 wi n

2. Geometric mean (GM): GM of n numbers x1, x2, x3, …, xn is the nth root of their products. It is a type of mean or average that indicates the central tendency or typical value of a set of numbers by using the product of their values. Geometric mean of n numbers x1, x2, x3, …, xn is denoted by

Suppose we have a given set of numbers 19, 33, 22, 40, 28, 15. The median of the set will be calculated by first arranging the values in ascending order, that is, 15, 19, 22, 28, 33, 40. Now, the median will be (22 + 28)/2 = 50/2 = 25.

Relation between AM, GM and HM Consider two numbers a and b. Then, AM = (a + b)/2; GM = ab; HM = 2ab (a + b). Therefore, (GM)2 = AM × HM Also,

∏x  n

1/ n

AM > GM > HM

i

i =1



and calculated as

  n

∏ xi

1/ n

= n x1x2 x3 … x4

i =1

3. Harmonic mean (HM): Harmonic mean is the special case of the power mean. As it tends strongly toward the least elements of the list, it may (compared to the arithmetic mean) mitigate the influence of large outliers and increase the influence of small values. Harmonic mean of n numbers x1, x2, x3, …, xn is calculated as n HM =



n (1 / xi ) i =1

SOME ALGEBRAIC FORMULAS Some of the commonly used algebraic formulae are as follows: 1. (a ± b)2 = a2 ± 2ab + b2 2. (a + b)2 + (a − b)2 = 2(a2 + b2 ) 3. (a + b + c)2 = a2 + b2 + c 2 + 2(ab + bc + ca) 4. (a + b + c + d)2 = a2 + b2 + c 2 + d 2 + 2a(b + c + d) + 2b(c + d) + 2cd 5. (a ± b)3 = a3 ± 3ab(a ± b) ± b3

4. Mode: It is the number that occurs most number of times in a given set of numbers. In a given set of data, if two or more values occur the same number of times, then a unique mode does not exist. For example, suppose we have a given set of numbers 1, 2, 4, 4, 6, 5, 9, 1, 6, 6. The mode of this set is 6, since it occurs the most number of times in the set.

6. a3 ± b3 = (a ± b)3 ∓ 3ab(a ± b)

5. Median: It is the middle value of a group of numbers arranged in an ascending or descending order. If number of values n, in a given set of data is odd, then median is the (n + 1)/2th value. If number of values n, in a given set of data is even, then there will be two middle values say a and b, and hence median is taken as (a + b)/2.

The Remainder Theorem

For example, suppose we have a given set of numbers 31, 29, 42, 18, 50. The median of the set will be calculated by first arranging the values in ascending order, that is, 18, 29, 31, 42, 50. Now, the median will be (5 + 1)/2th value, 5th value, that is, 31.

Chapter 01-1.indd 549

7. (x + a)(x + b) = x2 + (a + b)x + ab 8. (x + a)(x + b)(x + c) = x3 + (a + b + c)x2 + (ab + bc + ca)x + abc

The remainder theorem is useful for evaluating polynomials at a given value of x. It states that when we divide a polynomial f(x) by x − a then the remainder r equals f(a).

The Polynomial Factor Theorem The polynomial factor theorem is a special case of the remainder theorem. It is used to link the factors and zeros of any polynomial.

22-08-2017 06:49:23

550     Number System 

Table 2 |    (Continued)

The factor theorem states that a polynomial f(x) has a factor (x − a) if and only if f(a) = 0 or in other words, a is a root of f(x).

Decimal

Binary

Octal

Hexadecimal

 7

0111

7

7

 8

1000

10

8

 9

1001

11

9

The number of digits used to represent a number in a system is called the base system. The amount of digits is the base of that number system. For example, digital computers use binary base system which represents data using only two digits, viz. 0 and 1.

10

1010

12

A

11

1011

13

B

12

1100

14

C

13

1101

15

D

Some of the most commonly used base systems are given as follows:

14

1110

16

E

15

1111

17

F

BASE SYSTEM

1. Binary  (Base-2): As previously mentioned, binary base system is used extensively in digital computing. In binary system, we use only two digits (0 and 1) to represent the data. 2. Octal  (Base-8): Octal base systems use eight digits (0-7) to represent the data. 3. Decimal  (Base-10): Decimal base systems use ten digits (0-9) to represent the data. We use the decimal system in our day-to-day life for various calculations. 4. Hexadecimal or Hex  (Base-16): Hexadecimal base systems use sixteen digits (0-9 and A, B, C, D, E, F) to represent the data. Hexadecimal system is widely used by computer system designers and programmers. Table 2 gives the conversion table which can be used for the conversions of one base system to another. We have to keep in mind that whenever we are converting from binary to octal, we divide the binary number into groups of 3 and while converting from binary to hexadecimal, we divide the binary number into groups of 4. If the binary number cannot be divided into groups of 3 or 4, then we add sufficient zeros before the M.S.B. (Most Significant Bit). Table 2 |    Conversion table of decimal, binary,

Counting Trailing Zeros Trailing zeros are a sequence of 0s in the decimal representation of a number after which no other digits follow. Trailing zeros to the right of a decimal point do not affect the value of a number and can be removed. They may be useful for indicating the number of significant figures. The number of trailing zeros in a non-zero base b and integer n equals the exponent of the highest power of b that divides n. For example, 15000 has three trailing zeros and is therefore divisible by 1000 but not by 104.

INEQUATIONS A mathematical statement that says that one expression is greater or smaller than another in value is called an inequation or an inequality. Some of the important symbols of inequality are as follows:

1. x ≠ y 2. x > y 3. x < y 4. x ≥ y 5. x ≤ y

for for for for for

x x x x x

is is is is is

not equal to y greater than y less than y greater than or equal to y less than or equal to y

octal and hexadecimal base systems Decimal

Some of the important properties of inequality are as follows:

Binary

Octal

Hexadecimal

0

0000

0

0

1

0001

1

1

2

0010

2

2

3

0011

3

3

1. An inequality will still hold after each side has been increased, diminished, multiplied or divided by the same positive quantity. For example, if a > b and c > 0, then

4

0100

4

4

a+c > b+c

5

0101

5

5

ac > bc

6

0110

6

6

a b > c c

(Continued )

Chapter 01-1.indd 550

22-08-2017 06:49:25

HCF AND LCM OF NUMBERS     551

2. In an inequality, any term may be transposed from one side to the other if its sign is changed. That is, if a - c > b, then a > b + c or c > b − a. 3. If sides of an inequality be multiplied by same negative quantity, then sign of inequality must be reversed. That is, if a > b and c > 0, then ac < bc. 1 1 4. If a > b and a, b ≥ 0, then an > bn and n < n , b or a−n > b−n, if n is a positive quantity. a 5. Square of every real quantity is positive and therefore must be greater than zero, that is, for a ≠ b, (a - b)2 > 0; a2 + b2 > 2ab. 6. If sum of two positive quantities is given, their product is greatest when they are equal; and if product of two positive quantities is given, their sum is least when they are equal. 7. If a, b, c, …, k are n unequal quantities, then (a + b + c +  + k)n > a × b × c × d × × k n 8. If a and b are positive and unequal, then



am bm a+b > 2 2



m

except when m is a positive proper function. If m is a positive integer or any negative quantity, then m m

a b 2

>



a+b 2



m

If m is positive and less than 1, then



am bm a+b < 2 2



m

9. If x is positive and a < b, then a+x a > b+x b If x is positive and a > b, then a+x a < b+x b 10. If x is positive and a > b > x, then a−x a > b−x b If x is positive and x < a < b, then a−x a > b−x b

QUADRATIC EQUATIONS A quadratic equation is any equation having the form ax2 + bx + c = 0 , where x represents an unknown and  a, b and c are constants with a ≠ 0. If a = 0, then the equation is linear, not quadratic. The constants a, b and c are called, respectively, the quadratic coefficient, the linear coefficient and the constant or free term. The solution of a quadratic equation is given as

x=

−b ± b2 − 4ac 2a

EVEN AND ODD NUMBERS All positive integers divisible by 2 are called even numbers, while all positive integers not divisible by 2 are called odd numbers. For example 2, 4, 6, 8, … are even numbers and 1, 3, 5, 7, … are odd numbers. 0 is considered neither even nor odd. Some important properties of even and odd numbers are as follows:

1. Even number + Even number = Even number 2. Odd number + Odd number = Even number 3. Even number + Odd number = Odd number 4. Odd number + Even number = Odd number 5. (Even number) × (Even number) = Even number 6. (Even number) × (Odd number) = Even number 7. (Odd number) × (Even number) = Even number 8. (Odd number) × (Odd number) = Odd number

9. (Even number)Even = Even number 10. (Even number)Odd = Even number 11. (Odd number)Even = Odd number 12. (Odd number)Odd = Odd number

PRIME NUMBERS AND COMPOSITE NUMBERS Prime numbers are perfectly divisible either by 1 or by the number itself, while composite numbers are divisible by at least one more number other than 1 or by that number itself. For example 2, 3, 5, 7, 11, … are prime numbers while 4, 6, 9, 12, 14, … are composite numbers.

11. a2 + b2 + c 2 ≥ ab + bc + ac 12. a2b + b2c + c 2a ≥ 3abc a b c d 13. + + + ≥ 4 b c d a 14. a4 + b 4 + c 4 + d 4 ≥ 4abcd

Chapter 01-1.indd 551

HCF AND LCM OF NUMBERS HCF (highest common factor) of two or more numbers is the greatest number that perfectly divides each of them. HCF is also called GCD (greatest common divisor).

22-08-2017 06:49:27

71 = 7 552     Number System 

72 = 49 LCM (lowest common multiple) of two or more numbers is the least or a lowest number that is exactly divisible by each one of them.

73 = 343 7 4 = 2401 75 = 16807

CYCLICITY Cyclicity of a number is used mainly for the calculation of unit digits.



The unit digits can be 7, 9, 3 or 1 and after every four intervals it repeats itself. 8. Cyclicity of 8 81 = 8 82 = 64

1. Cyclicity of 1 In 1n, unit digit will always be 1.

83 = 512

2. Cyclicity of 2

84 = 4096 1

2 =2

85 = 32768

2

2 =4 23 = 8 24 = 16 25 = 32

The unit digits can be 2, 4, 6 or 8 and after every four intervals it repeats. 3. Cyclicity of 3 31 = 3 32 = 9

The unit digits can be 8, 4, 2 or 6 and after every four intervals it repeats itself. 9. Cyclicity of 9 91 = 9 92 = 81 93 = 729 The unit digits can be 9 or 1 and after every two intervals it repeats itself.

33 = 27 34 = 81

TEST FOR DIVISIBILITY

35 = 243

The unit digits can be 3, 9, 7 or 1 and after every four intervals it repeats. 4. Cyclicity of 4 41 = 4

One whole number is divisible by another if the remainder is zero after dividing. If one whole number is divisible by another number, then the second number is a factor of the first number. Divisibility tests are used to find the factors of large numbers.

42 = 16

Some basic tests for divisibility are as follows:

43 = 64

1. A number is divisible by 2 if the last digit is 0, 2, 4, 6 or 8. 2. A number is divisible by 3 if the sum of the digits is divisible by 3. 3. A number is divisible by 4 if the number formed by the last two digits is divisible by 4. 4. A number is divisible by 5 if the last digit is either 0 or 5. 5. A number is divisible by 6 if it is divisible by 2 and by 3. 6. The test for divisibility for 7 is slightly different. Remove the last digit, double it, subtract it from the truncated original number and continue doing this until only one digit remains. If this is 0 or 7, then the original number is divisible by 7. 7. A number is divisible by 8 if the number formed by the last three digits is divisible by 8. 8. A number is divisible by 9 if the sum of the digits is divisible by 9. 9. A number is divisible by 10 if the last digit is 0.



The unit digits can be 4 or 6 and after every two intervals it repeats. 5. Cyclicity of 5 51 = 5 52 = 25 53 = 125 The unit digit will always be 5. 6. Cyclicity of 6 61 = 6 62 = 36 63 = 216 The unit digit will always be 6. 7. Cyclicity of 7 71 = 7 72 = 49 73 = 343 7 4 = 2401 75 = 16807 Chapter 01-1.indd 552

22-08-2017 06:49:28

SOLVED EXAMPLES     553

SOLVED EXAMPLES 1. What will be the last digit number obtained by multiplying the numbers 81 × 82 × 83 × 84 × 85 ×  86 × 87 × 88 × 89? Solution:  The last digit will be multiplication of  1 × 2 × 3 × 4 × 5 × 6 × 7 × 8 × 9. Since, 5 and 2  are given here, their multiplication will result in zero as last digit. 2. The sum of the digit of a two-digit number is 10, while when the digits are reserved, the number decreases by 54. Find the changed number. Solution:  Let the two-digit number be xy. We know that x + y = 10. Also, when digits are reserved, the number is decreased by 54. Hence, the changed number can be 19, 28, 31 or 46. 91 − 19 = 82 82 − 28 = 54 Hence, the changed number = 28. 3. If GM = 12 and HM = 7.2, then what will be the value of AM? Solution:  We know that GM = AM × HM 2

x2 − 7 x + 10 = (x − 5)(x − 2)

and Therefore,

HCF = (x − 2) 7. Find the LCM of (6x2 + 5x − 4) and (2x2 + 7 x + 3). Solution:  We have (6x2 + 5x − 4) = (2x − 1)(3x + 4) and

(2x2 + 7 x + 3) = (2x + 1)(x + 3)

Therefore, LCM = (4x2 − 1)(3x + 4)(x + 3) 8. What will be the remainder if 37 is divided by 8? Solution:  We know that 37 = 32 × 32 × 32 × 3 When 32 is divided by 8, then remainder is 1. Therefore, remainder will be 1 × 3 = 3. 9. If a = y + z, b = x + z, c = x + y, then what will be the value of a2 + b2 + c 2 − 2ab − 2bc + 2ca. Solution:  We know that

We have GM = 12 HM = 7.2 12 × 12 AM = = 20 7.2 4. What is the least number which on being divided by 12, 15 and 24 will leave in each case a remainder of 5? Solution:  LCM of the three numbers 12, 15 and 124 is 120. Hence, the number that will leave remainder 5 = 120 + 5 = 125. 5. Find the number of divisors of 1420.

a2 + b2 + c 2 − 2ab − 2bc + 2ca = (a − b − c)2 Substituting values of a, b, c, we get (y + z − x − z − x − y)2 = (−2x)2 = 4x2 10. What is the smallest number which when increased by 5 is divisible by 20, 35, 25 and 30? Solution:  LCM of 20, 35, 25 and 30 is 2100. Hence, the number to which when 5 is added becomes divisible by 20, 35, 25 and 30 will be 2100 - 5 = 2095. 11. If A381 is divisible by 11, then what is the value of the smallest natural number A?

Solution:  We know that Solution:  A381 is divisible by 11 if and only if  (A + 8) - (3 + 1) is divisible by 11. So, So the number of divisors = (2 + 1)(1 + 1)(1 + 1) = 3 × 2 × 2 = 12 A + 8 - (3 + 1) = 11 (2 + 1)(1 + 1)(1 + 1) = 3 × 2 × 2 = 12 ⇒A = 11 - 4 = 7 6. What is the HCF of the polynomials (x2 − 5x + 6) 12. Find the unit digit of 2323. and (x2 − 7 x + 10)? Solution:  Here, we know that 2, 4, 8, 6 will repeat Solution:  We have after every four intervals till 320. Next digit will  x2 − 5x + 6 = (x − 3)(x − 2) be 8. So, unit digit of 2323 will be 8.

Chapter 01-1.indd 553

1420 = 142 × 10 = 71 × 2 × 2 × 5 = 22 × 51 × 711

22-08-2017 06:49:31

554     Number System 

13. Solve the inequality 2x2 − x < 1.

17. If x + y + z = 0, then what will be the value of x2 y 2 z2 + + . yz xz xy

Solution:  We have 2x2 − x < 1

Solution:  We are given that x+y+z=0 Therefore,

2x2 − x − 1 < 0

or

(2x + 1)(x − 1) < 0

x3 + y 3 + z 3 = 3xyz

Hence, either 2x + 1 > 0 and x − 1 < 0, or 2x + 1 < 0 and x − 1 > 0. Thus,

Also, x2 y 2 z2 x3 y3 z3 + + = + + yz zx xy xyz xyz xyz

−1 x> and x < 1 2 −1 x< and x > 1 2

or

−1 and x > 1 is not possible. Hence, the 2 −1 ,1 . solution is 2

=

But x
n, then what is the remainder when mn is divided by 6? So the remainder for



⇒ 3y 3 + y 2 − 7 y − 5 = f (y)(3y − 5) 3y 2 + y 2 − 7 y − 5 ⇒ f (y) = (3y − 5)

Chapter 01-1.indd 554





Solution:  Since the remainder when m + n is divided by 12 is 8, we have m + n = 12 p + 8 (1)



Also, since the remainder when m − n is divided by 12 is 6, we have m − n = 12q + 6 (2)

= y 2 + 2y + 1 Thus, the function is (y + 1)2 .



22-08-2017 06:49:37

SOLVED EXAMPLES     555

Here, p and q are integers. Adding Eqs. (1) and Solution:  HCF of 780, 840 and 960 = 60 780 840 960 (2), we get Thus, the total number of pieces = + + = 13 + 14 + 16 = 4 780 840 960 2m = 12p + 12q + 14 60 60 60 + + = 13 + 14 + 16 = 43 60 ⇒ m = 6 p + 6q + 7 = 6 ( p + q + 1) + 1 = 6r60 + 1 60 Therefore, total number of persons required = 43 × 2 = 86 where r is a positive integer = p + q + 1. Now, subtracting Eq. (2) from Eq. (1), we get 2n = (12 p + 8) − (12q + 6) = 12 ( p − q ) + 2 ⇒ n = 6 ( p − q ) + 1 = 6t + 1 where t is an integer = p − q

25. Convert 110010012 to decimal.

Solution:  We are given that 110010012 The number can be represented in decimal = 20 × 1 + 23 × 1 + 26 × 1 + 27 20 × 1 + 23 × 1 + 26 × 1 + 27 × 1 = 1 + 8 + 64 + 128 = 201 26. Convert 1001.00102 to decimal.

Hence, we have

Solution:  We are given that 1001.00102 mn = (6r + 1)(6t + 1) = 36rt + 6r + 6t + 1 = 6 (6rt + r + t) + 1The number can be represented in decimal =   1 + 1 = 6 (6rt + r + t) + 1 by factoring out 6. (6t + 1) = 36rt + 6r + 6t 20 × 1 + 23 × 1 + 2−3 × 1 = 1 + 8 + = 9.125 8 Thus, the remainder is 1. 27. What is the decimal value of the hexadecimal 22. Let X be a four digit number with exactly three number 777? consecutive digits being same and is a multiple of 9.  Solution:  We are given 77716. How many such X’s are possible? In decimal, the number can be represented as follows: Solution:  Let the four digit number be `aaab’ or 7 × 162 + 7 × 161 + 7 × 160 = 1792 + 112 + 7 = 1911 `baaa’. 28. What is the result when a decimal 156 is converted Since the number has to be a multiple of 9, thus  to base 16? 3a + b should either be 9 or 18 or 27. Solution:  We are given 15610. Case I: 3a + b = 9 The decimal value can be converted into binary as Possible cases are: (1116, 6111, 2223, 3222, 3330, follows: 9000) 2 156 Case II: 3a + b = 18 2 78 0 Possible cases are: (3339, 9333, 4446, 6444, 5553, 3555, 6660) 2 39 0 Case III: 3a + b = 27 2 19 1 Possible cases are: (6669, 9666, 8883, 3888, 7776, 2 91 6777, 9990) Hence, total number of cases = 20. 23. N is the smallest number that has 5 factors. How many factors does (N − 1) have?

) ) ) ) ) 2) 4 1 2) 2 0 2) 1 0

Solution:  A number that has 5 factors has to be of the form p4 where p is a prime number. The smallest such number is 24 = 16. Therefore, (N − 1) = 15 The factors of 15 are 1, 3, 5 and 15. Hence, (N − 1) has 4 factors.

1 Hence, the binary conversion is 100111002. Now, from the conversion table, we can convert from binary to hexadecimal. 10011100  9

C

Hence, the hexadecimal value is 9C. 24. Three gold coins of weight 780g, 840g and 960g are cut into small pieces, all of which have the equal weight. Each piece must be heavy as possible. If one such piece is shared by two persons, then how many persons are needed to give all the pieces of gold coins?

Chapter 01-1.indd 555

29. Convert the decimal 151.75 to binary. Solution:  We are given 151.7510. The decimal value can be converted into binary as follows:

22-08-2017 06:49:39

556     Number System 

) 2) 75 1 2) 37 1 2) 18 1 2) 9 0 2) 4 1 2) 2 0 2) 1 0

Also, 0.75 = 2−1 × 1 + 2−2 × 1

2 151

Hence, the binary conversion is 10010111.112. 30. Convert 1111010002 to octal base system. Solution:  We are given that 1111010002. Diving them into pairs of 3 and converting using the conversion table, we get 111101000  7

5

0

Thus, the conversion into octal base system = 7508.

1

PRACTICE EXERCISE 1. What number should be subtracted from x3 + 4x2 − 7 x + 12 if it is to be perfectly divisible by x + 3? (a) 12

(b) 3

(c) 20

8. A ball is dropped from a height of 24 m and it rebounds up to half of the distance it falls. If it continues to fall and rebound in this way, how far will it travel before coming to rest?

(d) 24 (a) 36 m

2. The product of four consecutive even numbers is always divisible by which of the following? (a) 384

(b) 600

(c) 768

(b) 48 m

9. What will be the HCF of y − 4y and 4y(y 3 − 8)? (c) y 2 − 4

(a) y(y + 2) (b) 1

(a) 176

(b) 187

(c) 540

(d) 648

4. What is the harmonic mean of two numbers whose geometric mean and arithmetic mean are 6 and 12, respectively? (a) 2

(b) 3

(c) 5

(d) 9

(a) 3 5 (b) 3

1 1 xy − (b) x y x+y

(c) 3 3

(d) 3 + 2 5

(a) Even (b) Odd (c) Even or odd (d) Even when x is even, and odd when x is odd 7. If 3x + 8y = 30, x and y are natural numbers, then what can be said about x? (a) Even (b) Odd (c) Even or odd (d) Even when y is even, and odd when y is odd

(c)

1 1 − y x

(d) x2 + y 2

11. When x + y + z = 6 and xy + yz + zx = 9, then what will be the value of x3 − y 3 − z 3 − 3xyz ? (b) 15

12. Solve the equality

6. If 12x + 6y = 48, and x and y are natural numbers, then what can be said about y?

Chapter 01-1.indd 556

(a)

. 9+4 5

(d) y − 2

10. If a = x + y, b = x2 − y 2 and c = xy , then what b will be the value of ? ac

(a) 54

3 5. Evaluate 3 80 +

(d) 72 m

3

(d) 864

3. What is the minimum number of square marbles required to tile a floor of length 5 m 78 cm and width 3 m 74 cm?

(c) 64 m

(d) 38

x −1 ≥ 1. x +1

(a) (1, ∞) (c) (−∞, 1) 13. If

(c) 82

(b) (−1, ∞) (d) (−∞, −1)

x−4 ≥ 2, then which of the following is correct? x+6

(a) x ≤ −16 (c) x ≤ 8

(b) x ≥ −16 (d) x ≥ −8

14. If a, b, c, d, x and y are non-zero, unequal intea + bi p gers and = , then what will be the value of c + di q a2 + b 2 ? c2 + d2 (a) p q

(b) p2 q

(c) p2 q 2

(d) 1

22-08-2017 06:49:45

ANSWERS      557

24. The LCM of (16 − x2 ) and (x2 + x − 6) is

15. Find the unit’s digit in 264102 + 264103. (a) 0

(b) 2

(c) 4

(d) 6

(a) (x − 3)(x + 3)(4 − x2 ) (b) 4(4 − x2 )(x + 3)

16. What is the minimum number of square marbles required to tile a floor of length 3 m 84 cm and width 2 m 88 cm?

(c) (16 − x2 )(x2 + x − 6) (d) (4 − x2 )(x − 3) 25. What is the remainder of

(a) 20

(b) 36

(c) 12

(d) 24

17. Average marks of 15 students in a class is 145, maximum marks being 150. If two lowest scores are removed, the average increases by 5. Also, two lowest scores are consecutively multiples of 9. What is the lowest score in the class?

(a) 1

(b) 100

(c) 118

(b) 30

(c) 27

(d) 35

19. x and y are integers and if x2/y3 is an even integer, then which of the following must be an even integer? (a) x − y

(b) y − 1

(c) x2/y4

(a) 5

(c) 1090

(a) 0

(c) 8

(d) 9

(c) 6

(b) 9

(d) 9

(c) 3

29. Find the unit digit of 17 (b) 5

(d) 6

× 27 . 27

(c) 3

(d) 1

30. What is the greatest number of four digits that when divided by any of the numbers 6, 9, 12, 17 leave a remainder of 1? (b) 9487

(c) 9895

(d) 9793

(d) 4660 31. Convert 11001012 into octal base system. (a) 1458

(b) 2

(b) 7

17

(a) 7

21. What is the remainder when 72 × 82 is divided by 6? (a) 4

(c) 7

28. Let N = 1421 × 1423 × 1425. What is the remainder when N is divided by 12?

(a) 9997 (b) 9662

(b) 5

27. If 381A is divisible by 9, find the value of the smallest natural number A.

(d) xy

20. Find the greatest number of four digits which when divided by 15, 20, 28 leaves in each case a remainder 2? (a) 9077

(d) 6

(d) 96

18. A number when divided by a divisor leaves a remainder of 22. When twice the original number is divided by the same divisor, the remainder is 9. What is the value of the divisor? (a) 14

(c) 4

26. When a four digit number is divided by 85 it leaves a remainder of 39. If the same number is divided by 17, the remainder would be? (a) 3

(a) 108

(b) 2

3250 ? 7

(b) 3408

(c) 2578

(d) 1508

(d) 6 32. Convert 10278 into decimal base system.

22. The LCM of two numbers is 12 times their HCF. The sum of HCF and LCM is 403. If one number is 93, then find the other. (a) 134

(b) 130

(c) 128

(a) 535

(b) 342

(c) 255

(d) 555

33. What is the binary equivalent of 19210?

(d) 124

(a) 11001100 (c) 10100000

23. For a number to be divisible by 88, it should be

(b) 11000000 (d) 11000010

34. Convert 74318 into hexadecimal.

(a) divisible by 22 and 8 (b) divisible by 11 and 8 (c) divisible by 11 and thrice by 2 (d) All of the above

(a) D90

(b) 119

(c) F19

(d) A74

35. What is the decimal equivalent of 92116? (a) 1234

(b) 4991

(c) 3433

(d) 2337

ANSWERS 1. (d)

5. (c)

9. (a)

13. (b)

17. (a)

21. (a)

25. (c)

29. (d)

33. (b)

2. (a)

6. (c)

10. (c)

14. (c)

18. (d)

22. (d)

26. (b)

30. (d)

34. (c)

3. (b)

7. (a)

11. (a)

15. (a)

19. (d)

23. (b)

27. (c)

31. (a)

35. (d)

4. (b)

8. (d)

12. (d)

16. (c)

20. (b)

24. (c)

28. (c)

32. (a)



Chapter 01-1.indd 557

22-08-2017 06:49:47

9 − 4 5   ×  9 + 4 5  9 − 4 5  3

558     Number System 

= 3 80 +

EXPLANATIONS AND HINTS

(3 × 9 − 3 × 34 5 )

= 3 80 +

1. (d) To find the number to be subtracted from x3 + 4x2 − 7 x + 12 to make it divisible by x + 3, we need to divide the two and find the remainder. x2 + x − 4 x + 3 x3 + 4x2 − 7 x + 12

)

x3 + 3x2 −

92 − (4 5 )2 27 − 12 5 = 3 16 × 5 + 27 − 12 5 81 − 80

= 3 80 +

= 3 × 4 × 5 + 27 − 12 5 = 12 5 + 27 − 12 8 = 27 = 3 3 6. (c) We know that 12x + 6y = 48 Now, all three terms are even. If 6y = even, then y can be even or odd since any number multiplied with an even number is even. Hence, y can be even or odd.

x2 − 7 x x2 − 3x + − 4x + 12 − 4x − 12 + 24

7. (a) We know that 3x + 8y = 30 3x is even and 8y is also even. Hence, 3x has to be even since addition or subtraction of two even numbers is always even. Hence, x is always even.

Therefore, the number to be subtracted is 24. 2. (a) The product of four consecutive numbers is always divisible by 4! . Now, since we have four even numbers, we have an additional 2 available with each number. It is always divisible by

8. (d) Total distance travelled by ball = 24 + 2(12 + 6 + 3 + 1.5 + …) This is an infinite series, hence total distance will be   12  24 + 2   1 − (1 / 2)    = 24 + 4(12) = 24 + 48 = 72 m

(2 )4 ! = 16(24) = 384 4

3. (b) The marbles are square marbles. The length and width of the floor are 5 m 78 cm and 3 m 74 cm.

9. (a) We have y 3 − 4y = y(y 2 − 4) = y(y + 2)(y − 2)

The HCF of 578 and 374 = 34 Hence, the side of the square = 34 and

4y(y 3 − 8) = 4y(y + 2)(y 2 − 2y + 4)

The number of such square marbles required will be 578 × 374 = 17 × 11 = 187 34 4. (b) We know that GM = AM × HM 2

HCF = y(y + 2) 10. (c) Given that a = x + y, b = x2 − y 2 and c = xy (x + y)(x − y) b x2 − y 2 = = ac xy(x − y) xy(x + y)

We have GM = 6 and AM = 12, hence HM =

6×6 =3 12

5. (c) We have

=

x−y 1 1 = − xy y x

11. (a) We are given that x+y +z = 6

3

xy + yz + zx = 9

3 80 + 9+4 5 9 − 4 5   ×  9 + 4 5  9 − 4 5  3

= 3 80 +

(3 × 9 − 3 × 34 5 ) = 3 80 +

= 3 80 +

92 − (4 5 )2

We know that x3 − y 3 − z 3 − 3xyz = (x + y + z)(x2 + y 2 +z 2 − xy − yz − zx) = 6[(x + y + z)2 − 3(xy + yz + zx)] = 6[36 − 3(9)] = 6[36 − 27 ] = 6 × 9 = 54

27 − 12 5 = 3 16 × 5 + 27 − 12 5 81 − 80

= 3 × 4 × 5 + 27 − 12 5 = 12 5 + 27 − 12 8 = 27 = 3 3 Chapter 01-1.indd 558

22-08-2017 06:49:51

EXPLANATIONS AND HINTS      559

So the total marks = 150 × 13 = 1950

12. (d) We have x −1 ≥1 x +1

Say the two lowest scores are 9x + 9x + 9. Therefore, 9x + 9x + 9 = 2175 − 1950 = 225 ⇒ 18x = 216 ⇒ x = 12

First, we note that x + 1 ≠ 0 , that is, x ≠ −1 and for x ≠ −1, (x + 1)2 > 0. Multiplying both sides by (x + 1)2 , we get

Thus, lowest score = 9 × 12 = 108

(x − 1)(x + 1) ≥ (x + 1)2 = x2 − 1 ≥ x2 + 2x + 1 = −2x ≥ 2 = x ≤ −1; but x ≠ −1

18. (d) Let the original number be a and the divisor be d. Let the quotient of the divisor “a” by “d ” be a “x.” Therefore, = x and remainder = 22. Thus, d a = dx + 22 ⇒ 2a = 2dx + 44

Thus, the solution of the equation is x ∈ (−∞, −1). 13. (b) We have x−4 ≤2 x+6 ⇒ x − 4 ≤ 2x + 12 ⇒ x ≥ −16

Also, given that

“2dx” is perfectly divisible by “d  ” and will not leave a remainder. The remainder of 9 was obtained by 44 by d. When 44 is divided by 35, the remainder is 9. Hence, the divisor is 35.

14. (c) We are given that a + bi p = c + di q

x2 19. (d) If

x2 = y 3 even Hence x2 = even and x is an integer So x = even So, only xy must be even.

and                qb = pd p p ⋅ c and b = ⋅ d q q

Therefore, ( p2 / q 2 ) ⋅ c 2 + ( p2 / q 2 ) ⋅ d 2

a2 + b 2 c2 + d2

=

p2 =

c2 + d2

= even y3

⇒ qa + qbi = pc + pdi Equating real and imaginary parts, we get qa = pc

a=

 2dxd+ 44  leaves a remainder of 9.

q2

20. (b) LCM of 15, 20, 28 is 420. So, the greatest four digit number divisible by 420 is 9660. Hence, required number is 9660 + 2 = 9662. 21. (a) We have (72 × 82 ) = (7 × 8)2 = 562

15. (a) Required unit’s digit = unit’s digit in 4102 + 4103 Now, 42 gives unit digit 6 and 4102 gives unit digit 6.

The number immediately before 56 that is divisible by 6 is 54. Now, writing 562 as (54 + 2)2 , we have

4103 gives unit digit of 4.b

562 = (54 + 2)2

Hence, unit digit in 264102 + 264103 = unit’s digit in (6 + 4) = 0

= 542 + 22 + 2(2)54 = 54[54 + 2(2)] + 22 = 6 × 9[54 + (2 × 2)] + 4

16. (c) The marbles are square marbles. Side of a square should be a factor to both 3 m 84 cm and  2 m 88 cm.

Here, the remainder is 4. HCF of 384 and 288 = 96 Hence the number of square marbles required 384 × 288 = 4 × 3 = 12 marbles 96 × 96

22. (d) We know that LCM = 12 × HCF Also, LCM + HCF = 403 ⇒ 13 HCF = 403 ⇒ HCF = 31

17. (a) Given that average marks of 15 students = 145 Thus, So the total marks = 145 × 15 = 2175 When two lowest marks are removed then average =  150

Chapter 01-1.indd 559

LCM = 372 Also, HCF × LCM = x × 93

22-08-2017 06:49:56

560     Number System 

⇒ 31 × 372 = 93x ⇒x = 124

Hence, we rewrite the integer as 001100101. ⇒ 001100101 

23. (b) For a number to be divisible by 88, it should be divisible by 8 and 11 because 8 and 11 are co-prime numbers whose multiplication gives 88. 24. (c) We have 16 − x2 = (4 − x)(4 + x) and

x2 + x − 6 = (x + 3)(x − 2)

1

4

5

Hence, the number in octal base system is 1458. 32. (a) We are given the octal integer, 10278. To convert it into decimal, we have 1 × 83 + 2 × 8 + 7 = 535

LCM will be (16 − x2 )(x2 + x − 6).

33. (b) Using the long division method, we have

) 2) 96 0 2) 48 0 2) 24 0 2) 12 0 2) 6 0 2) 3 0 2) 1 1

25. (c) We have to find the remainder of

2 192 (32 )125 7 =

(7 + 2)125 2125 = 7 7

(23 )41 × 22 1× 4 ⇒ 7 7 Hence, remainder is 4. ⇒

26. (b) We have n = 85k + 39 Now,

1

n 85k + 39 34 + 5 5 = = 5k + = 5k + 2 + 17 17 17 17

Hence, the binary equivalent of 192 is 11000000. 34. (c) We are given an octal number, 74318.

So the remainder is 5. Converting it into binary, we get 27. (c) Given that 381A is divisible by 9. So, 3 + 8 + 1 + A = 12 + A is divisible by 9.

7 4 3 1 111 100 011 001

Thus, A = 6





1421 × 1423 × 1425 28. (c) Remainder of ⇒ Remainder 12 (5 × 7 × 9) (35 × 9) of = Remainder 12 12 (11 × 9) 99 Remainder of = Remainder =3 12 12

Hence, the binary equivalent to the number = 111100011001. Now, making groups of 4 and converting into hexadecimal, we get 111100011001  F

1

9

Hence, the number in hexadecimal is F1916. 29. (d) Unit digit of 1717 is 7 and unit digit of 2727 is 3. So unit digit of 1717 × 2727 will be 7 × 3 = 21 ⇒ 1

35. (d) We are given the hexadecimal number, 92116. To convert it into decimal, we have

30. (d) LCM of 6, 9, 12, 17 = 612 The greatest number of four digits divisible by 612 is 9792. To get remainder of 1, number should be 9792 + 1 = 9793

9 × 162 + 2 × 16 + 1 = 2337 Hence, the number in decimal is 2337.

31. (a) We are given the binary integer, 1100101. To convert it into octal, we need to be able to form groups of 3.

Chapter 01-1.indd 560

22-08-2017 06:50:00

CHAPTER 2

PERCENTAGE

INTRODUCTION The term percentage is often used to avoid the fractions less than 1. Instead of treating the complete entity as 1, we treat it as 100 and take the ratios accordingly. The term percent means for every hundred. A fraction of any quantity whose denominator is 100 is called percentage of that quantity, and numerator of the fraction is called rate percent. It is denoted by the symbol %.

SOME IMPORTANT FORMULAE List of some of the common formulae used are as follows: 1. To increase the number by a given percentage: Say the number is x and the percentage is y, then the increased number z will be (100 + y) z = x× 100

Chapter 01-2.indd 561

2. To decrease the number by a given percentage: Say the number is x and the percentage is y, then the decreased number z will be z = x×

(100 − y) 100

3. To find the percent increase of a number: Say the final value is x and the initial value is y, then the percent increase z will be

z=

x−y ×100 y

4. To find the percent decrease of a number: Say the final value is x and the initial value is y, then the percent decrease z will be

z=

y −x ×100 y

22-08-2017 06:54:13

562        Percentage 

SOLVED EXAMPLES 1. A quantity x = 240 is increased by 50%. What is the new value? Solution:  It is given that x = 240, so the new value of x will be 50 240 + × 240 = 360 100 2. A quantity x = 100 is first increased by 20% and then the resultant is decreased by 10%. What is the final value? Solution:  It is given that x = 100, so the value of x after 20% increase will be 20 × 100 = 120 100 Therefore, the value of x after 10% decrease will be 100 +

120 −

10 × 120 = 108 100

3. If the value of x increases from 400 to 540, then what is the percent increase? Solution:  It is given that the initial value of x = 400 and the new value of x = 540. So the total change in value = 540 − 400 = 140. Therefore, the total percent increase will be

Total number of girls who failed is 45 × 800 = 360 100 So the total number of students who failed is 1020 and the total number of students who passed is 980. It is given that total students = 2000. So the total percentage of the students who passed will be 980 × 100 = 49% 2000 6. For a square with side of length 4 cm, the numerical value of perimeter is what percent of the numerical value of its area? Solution:  Perimeter of the square = 4 × 4 = 16 cm and the area of the square = 42 = 16 cm2 Now, since the numerical value of the area and perimeter is the same. Perimeter is 100% of the area of the square. 7. A quantity X is 50% more than Y. What percent is Y less than X? Solution:  We have two quantities X and Y and X is 50% more than Y. So

140 × 100 = 35% 400

X = 1.5Y =

4. Gaurav scored 64, 80 and 70 in three out of his four exams. What should be his score in the fourth exam so that his overall average is 73?

Also, Y=

Solution:  It is given that the total marks of Gaurav in the three subjects = 64 + 80 + 70 = 214. Now, if marks in the fourth subject are x, then 214 + x = 73 4 ⇒ x = 73 × 4 − 214 = 292 − 214 ⇒ x = 78 Hence, Gaurav must get 78 marks to get an overall average of 73.

3 Y 2 2 X 3

Therefore, the total percentage of Y of X is (2 / 3)X × 100 = 66.67% X Hence, the total decrease of quantity Y to X = 33.33%. 8. If the price of wheat rises by 5%, then by what percent must Raj reduce his consumption so as not to increase his expenditure? Solution:  Percent reduction in consumption is

5. 1200 boys and 800 girls are examined in a test; 55% of the boys and 45% of the girls fail. What percent of the total students passed? Solution:  Total number of boys who failed is 55 × 1200 = 660 100

Chapter 01-2.indd 562

1005+ 5 ×100 = 211 ×100 = 4.762% 9. The price of soda increases from H10 to H12. If it then decreases by double the total percent increase then what is the new price of the soda?

22-08-2017 06:54:15

SOLVED EXAMPLES        563

Solution:  Percent increase of soda is 2 × 100 = 20% 10 Now, the percent decrease is twice the percent increase. Hence, percent decrease = 40% of the new value. So the final price of the soda will be 12 −

40 × 12 = 12 − 4.8 = 7.2 100

Thus, the final price of soda is H 7.2. 10. If the radius of a sphere is increased by 20%, then what is the percent change in the value of its area? Solution:  If the radius of the sphere initially is r, then the area of the sphere = pr 2 Now, the new radius is 20 r+ × r = 1.2r 100 So the new area = p(1.2r)2 = 1.44r 2p Now, the percent change in value of the area is 0.44pr 2 pr 2

× 100 = 44%

11. A salesman sells two kinds of trousers: cotton and woollen. A pair of cotton trousers is sold at 30% profit and a pair of woollen trousers is sold at 50% profit. The salesman has calculated that if he sells 100% more woollen trousers than cotton trousers, his overall profit will be 45%. However, he ends up selling 50% more cotton trousers than woollen trousers. What will be his overall profit? Solution:  Let the cost price of 1 cotton trouser and 1 woollen trouser be x and y, respectively. Case I: Number of woollen trousers sold is 100% more than cotton trousers. 1.3x + 1.5 × 2 × y = 1.45 (x + 2y ) ⇒ 0.15x = 0.1y ⇒ 3x = 2y Case II: Number of cotton trousers sold is 50% more than woollen trousers. 2y Selling price = 1.3x + 1.5 × 3 ⇒ Selling price = 1.3x + 1.5x = 2.8x

 

2 ⇒ Cost price = x + y = 2x 3 Profit =

Chapter 01-2.indd 563

 2.8x2x− 2x  ×100 = 40%

12. A candidate who gets 20% marks fails by 10 marks but another candidate who gets 42% marks gets 12% more than the passing marks. What are the maximum marks? Solution:  Total pass percentage = 42% − 12% = 30% Let maximum marks be x. 30% of x − 20% of x = 10 10% of x = 10 10 × x = 10 ⇒ x = 100 100 Thus, maximum marks = 100 13. Two persons Raj and Ram started working for a company in similar jobs on January 1, 1991. Raj’s initial monthly salary was H 400, which increases by H 40 after every year. Ram’s initial monthly salary was H 500 which increases by H 20 after every six months. If these arrangements continue till December 31, 2000, find the total salary they received during that period. Solution:  Raj’s salary on January 1, 1991 is H 400 per month. Total increment = H 40 per annum. We can use the formula of sum of arithmetic progression in order to calculate the salary in ten years.

 

10 12 2 (400) + (10 − 1) 40 × = H 69600 2 Ram’s salary as on January 1, 1991 is H 550 and his half yearly increment in his month salary is H 20. His total salary from January 1, 1991 to December 31, 2000 is given by 6 2 (500) + (20 − 1) 20 ×

 202  = H 82800

Total salary of Raj and Ram in the ten-year period = 69600 + 82800 = H 152400 37 % less than the 64 volume of sphere P and the volume of sphere R is 19 % less than that of sphere Q. By what percent 27 is the surface area of sphere R less than the surface area of sphere P?

14. The volume of the sphere Q is

Solution:  Let the volume of sphere P be x. 37 27 Therefore, volume of sphere Q = x − x = x 64 64 The volume of R =

  2764 x = 18 x

27 19 x− 64 27

22-08-2017 06:54:18

564        Percentage 

Thus, decrease in profit % = 27 1 Ratio of volume of P:Q:R = x : x : x ⇒ 6416000 : 27 : 8− 14000 2000 64 8 × 100 = × 100 = 12.5% 16000 16000 3 3 3 Ratio of radius of P:Q:R = 64 : 27 : 8 = 4 : 3 : 2





The surface area of P:Q:R will be in the ratio of 16 : 9 : 4 Surface area of R is less than the surface area of P, therefore 16y − 4y = 12y where y is a constant. 12y × 100 = 75% 16y Thus, surface area of sphere R is less than the surface area of sphere P by 75%. Now,





− 14000 2000 1600016000  ×100 = 16000  ×10

17. The cost of raw material of a product increases by 30%, the manufacturing cost increases by 20% and the selling price of the product increases by 60%. The raw material and the manufacturing cost, originally, formed 40% and 60% of the total cost, respectively. If the original profit % was one-fourth the original manufacturing cost, then what is the approximation new profit percentage? Solution:  Let total initial cost of the product be x. Manufacturing cost = 0.6x

15. A textile manufacturing firm employees 50 looms. Raw materials cost = 0.4x 0.6x It makes fabrics for a branded company. The aggreAlso, original selling price = x + = 1.15x gate sales value of the output of the 50 looms is 4 H 500000 and the monthly manufacturing expense 30 New raw material cost = 0.4x + × 0.4x = 0.52x is H 150000. Assume that each loom contributes 100 equally to the sales and manufacturing expenses. 20 Monthly establishment charges are H 75000. If one New manufacturing cost = 0.6x + × 0.6x = 0.72x 20 100 loom breaks down and remains idle for one 0.6xmonth, + × 0.6x = 0.72x 100 what is the decrease in profit? New cost of the product = 0.52x + 0.72x = 1.24x 60 Solution:  Total profit = 500000 − (150000 + 75000) = New selling price = 1.15x + (1.15x) = 1.84x = H 275000 100 0.6x Since, such loom contributes equally to sales and New profit percentage = × 100 = 48.39% 1.24x manufacturing expenses. But the monthly charges are fixed at H 75000. 18. The marks scored in History by P, Q, R and S If one loom breaks down, sales and expenses will form a geometric progression in that order. If the decrease. 275     49   49  marks scored by R were % less than the sum − × − 75000 150000 New profit = 500000 × 9    50 50 of the marks scored by P and Q, then marks scored     49   49  − 150000 × − 75000 = H 268000   by S were what percent more than the marks scored 50   50  Decrease in profit = 275000 − 268000 = H 7000 by Q? (Assuming nobody scored negative marks.)

   

 

 



 

 

 

16. An electronic company has 14 machines of equal efficiency in its factory. The annual manufacturing costs are H 42000 and establishment charges are H 12000. The annual output of the company is H 70000. The annual output and manufacturing costs are directly proportional to the number of machines. The shareholders get 12.5% profit, which is directly proportional to the annual output of the company. If 7.14% machines remain closed throughout the year, then what would be the percentage decrease in the amount of profit of the shareholders?

Solution:  Let the marks scored by P, Q, R and S be a, ax, ax2 and ax3, respectively. 275 We are given that marks scored by R were % 9 less than the marks scored by P and Q. So, if marks scored by P and Q were 36 then those scored by R is 25. Marks scored by P and Q = a + ax = 36 Marks obtained by R = ax2 = 25

(a + ax) 36 = ax

Solution:  Initial profit = 70000 − 42000 − 12000 = 16000 0000 − 42000 − 12000 = 16000 If one of the machines is closed throughout the year, 13 then change in profit = × (70000 − 42000) = 14000 14 13 × (70000 − 42000) = 14000 14

 



Chapter 01-2.indd 564

2

25



1+ x = x

2

36 ⇒ 36x2 − 25x − 25 = 0 25

5 . 4 5a 25a 125a ,R = . Hence, P = a, Q = and S = 4 16 64 Solving the quadratic equation, we get x =

22-08-2017 06:54:22

SOLVED EXAMPLES        565

Now, percentage difference between S and Q =





 

125a/64 − 5a/4 45 × 100 = × 100 = 56.25% 5a/4 80

The value of the order placed = 3510 × 8 = 28080 21. A test has 20 questions. If Peter gets 80% correct, how many questions did Peter missed?

Thus, S scored 56.25% more than Q. 19. Mannan starts a month with provisions expected to last for the entire month. After few days, it is discovered that the provisions will, in fact, be shortened by 12 days and it is calculated that if the stock of provisions left is immediately tripled, it will be possible to exactly make up for the shortfall. If the stock of provisions left is doubled instead of being tripled and simultaneously the strength of Mannan is decreased by 25%, then by how many days will the provisions fall short? Solution:  At the moment the shortfall is discovered, let there be x days of provision left. Now, 3x − x = 2x extra days of provisions last for the 12 additional days. ⇒ 3x lasts for 18 days. But if the provisions are only doubled and the 3 th strength becomes , then the provisions will 4 last for



12 ×

4 = 16 days. 3

Thus, the provisions will fall short by 18 − 16 = 2 days. 20. In the period from January to March, an electronics company sold 3150 units of television, having started with a beginning inventory of 2520 units and ending with an inventory of 2880 units. What was the value of order placed by the company during the three-month period? (Profits are 25% of cost price, uniformly.) Solution:  Units ordered = Units sold + Ending inventory − Beginning inventory ⇒ 3150 + 2880 − 2520 = 3510

20 − 16 = 4. Thus, Peter missed 4 questions. 22. 24 students in a class took an algebra test. If 18 students passed the test, what percent do not pass? Solution:  Number of students who did pass = 24 − 16 = 6 6 Percent of people who did not pass = ×100 = 25% 24 Therefore, 25% of students did not pass. 23. Rajeev buys good worth H 6650. He gets a rebate of 6% on it. After getting the rebate, he pays sales tax @ 10%. Find the amount he will have to pay for the goods. 6 Solution:  Rebate = × 6650 = H 399 100 10 10 Sales tax = × (6650 − 399) = × 6251 = 100 100 H 625.1





Thus, final amount = H (6251 + 625.10) = H 6876.10 24. The population of a town increased from 175000 to 262500 in a decade. What is the average percent increase of population per year? Solution:  Increase in 10 years = (262500 − 175000) = 87500 87500 × 100% = 50% 175000 50 Thus, required average =   = 5% 10 Increase % =

Total sales of television = 900 + 1800 + 6300 + 1050 + 2100 + 7350 + 1200 + 2400 + 8400 = H 31500 31500 Sales price per unit of television = = 10 3150 Profits are 25% of the cost price.

25. Three candidates contested an election and received 1136, 7636 and 11628 votes, respectively. What percentage of the total votes did the winning candidate get?

Sales price = Cost price + profits = Cost profit + 0.25 ×Cost price = 1.25 × Cost price 10 Cost price per unit of television = =8 1.25

Solution:  Total number of votes pulled = (1136 + 7636 + 11628) = 20400 11628 Therefore, required percentage = ×100 = 57% 20400



Chapter 01-2.indd 565

Solution:  The number of correct answers is 80% 80 of 20 or × 20 = 16. 100 Since the test has 20 questions and he got 16 answers correct, the number of questions he missed is







22-08-2017 06:54:24

566        Percentage 

PRACTICE EXERCISE 1. Adam’s scores in five math tests are 99, 85, 88, 94 and 93. What does he need to score in the next test for his final average in all the tests to be at least 92? (a) 90 (c) 92

(b) 91 (d) 93

2. At a football game with 5400 fans, 60% of the fans are rooting for the home team. If 810 of the home team fans are students, what percent of the home team fans are not students? (a) 75 (c) 80

(b) 25 (d) 70

3. A batsman scored 120 runs which included 3 boundaries and 8 sixes. What percent of his total score did he make by running between the wickets? (a) 45.5 (c) 50

(b) 55 (d) 62

4. In a certain year, California produced 70% and South Carolina produced 10% of all fresh peach crops in the United States. If all the other states combined, produce 240 million pounds of fresh peach crop that year, how many million pounds of peach crop did South Carolina produce? (a) 240 million pounds (b) 120 million pounds (c) 100 million pounds (d) 360 million pounds 5. A scientist is studying the change in the population of tigers for a certain area. She observes a 25% increase in the population of tigers for that area. If the new population is 45 tigers, what was the previous population? (a) 40 (c) 30

(b) 36 (d) 28

6. 1100 boys and 700 girls are examined in a test; 42% of the boys and 30% of the girls pass. What percentage of the total students failed? (a) 62.67 (c) 55

(b) 58 (d) 37.33

7. The price of gasoline at a certain station has been increased from H 72 to H 76. If it is decreased by half the percent of the percent increase, what is the new price? (a) 74.12 (c) 74

(b) 72.68 (d) 73.89

8. 10% of the voters did not cast their vote in an election between two candidates. 10% of the votes polled were found invalid. The successful candidate got 54% of the valid votes and won by a majority

Chapter 01-2.indd 566

of 1620 votes. What was the total number of voters enrolled on the voters list? (a) 32000 (c) 25000

(b) 35000 (d) 23000

9. While purchasing item A costing H 400, Jack had to pay the sales tax at 8% and on item B costing H 6400, the sales tax was 10%. What percent of the sales tax Jack had to pay, taking the two items together on an average? (a) 7.65% (c) 11.12%

(b) 9.88% (d) 8.32%

10. A student has to obtain 33% of the total marks to pass. He got 125 marks and failed by 40 marks. What are the maximum marks a student can get? (a) 1000 (c) 500

(b) 300 (d) 800

11. Difference of two numbers is 1400. If 10% of the larger number is equal to 15% of the smaller number, then what are the two numbers? (a) 2100, 1400 (c) 8400, 5600

(b) 4200, 2800 (d) 5600, 4200

12. For a sphere of radius 20 cm, the numerical value of surface area is what percent of the numerical value of its volume? (a) 33.3% (c) 30%

(b) 15% (d) 20%

13. What is the value of (x% of y) + (y% of x)? (a) 20% of x/y (c) 2% of xy

(b) 2% of x/y (d) 20% of xy

14. At an election, where there are only two candidates, a candidate who gets 40% of the votes is rejected by a majority of 640 votes. What is the total number of votes recorded assuming that there was no void vote? (a) 3200 (c) 6400

(b) 2800 (d) 4000

15. If the price of onion rises by 25%, then by what percent Amita must reduce her consumption of onions so as not to increase her expenditure? (a) 22.5% (c) 50/3%

(b) 25% (d) 20%

16. A quantity X is 25% more than Y. What percent is Y less than X? (a) 15% (c) 20%

(b) 17.5% (d) 25%

22-08-2017 06:54:25

PRACTICE EXERCISE        567

17. A quantity A is increased successively by 20%, 30%, 50%, 20% to give X and 20%, 50%, 30%, 20% to give Y. What percent is X of Y? (a) 0% (c) 100%

(b) 50% (d) 200%

18. A quantity X is increased by 20% and the resultant is then increased by 30%. The same quantity is then first increased by 30% and the resultant is then increased by 20%. Say the results are Y1 and Y2 respectively. What percent is Y1 of Y2?

8 years of age which is 48. What is the total number of students in the school? (a) 100 (c) 80

(b) 120 (d) 150

26. Two numbers A and B are such that the sum of 5% of A and 4% of B is two-third of the sum of 6% of A and 8% of B. What is the ratio of A:B? (a) 2:3 (c) 3:4

(b) 1:1 (d) 4:3

3 5 instead of . 5 3 What is the percent error in the calculation?

27. A student multiplied a number by (a) 100% (c) 0%

(b) 200% (d) 50%

19. The length and breadth of a room in the shape of a cuboid is increased by 10% each and the height is decreased by 20%. What is the percent change in the volume of the cuboid? (a) 0% (c) 20%

(b) 3.2% (d) 32%

20. The base and height of a right triangle are increased by 10% and 20%, respectively. What is the percent change in the area of the triangle? (a) 10% (c) 30%

21. Two students appeared at an examination. One of them secured 9 marks more than the other and his marks was 56% of the sum of their marks. What marks was obtained by them? (b) 41, 32 (d) 43, 34

22. A fruit seller had some apples. He sells 40% apples and still has 420 apples. How many apples did he originally have? (a) 588 (c) 672

(b) 600 (d) 700

23. What percent of numbers from 1 to 70 have 1 or 9 in the unit’s digit? (a) 1

(b) 14

(c) 20

(d) 21

24. If A = x% of y and B = y% of x, then which one of the following options is true? (a) A is smaller than B (b) A is greater than B (c) If x is smaller than y, then A is greater than B (d) A is the same as B 25. In a certain school, 20% of students are below 8 years of age. The number of students above 2 8 years of age is of the number of students of 3

Chapter 01-2.indd 567

(b) 44% (d) 64%

28. In an election between two candidates, one got 55% of the total valid votes, 20% of the votes were invalid. If the total number of votes was 7500, then what is the number of valid votes that the other candidate got? (a) 2700 (c) 3000

(b) 2900 (d) 3100

29. If 20% of a = b, then b% of 20 is the same as:

(b) 15% (d) 32%

(a) 39, 30 (c) 42, 33

(a) 34% (c) 54%

(a) 4% of a (c) 20% of a

(b) 5% of a (d) None of these

30. Gauri went to the stationers and bought things worth H25, out of which 30 paise went on sales tax on taxable purchases. If the tax rate was 6%, then what was the cost of the tax free items? (a) H 15 (c) H 20

(b) H 19.70 (d) H 22.75

31. Two tailors X and Y are paid a total of H 550 per week by their employer. If X is paid 120 percent of the sum paid to Y, how much is Y paid per week? (a) H 200 (b) H 250 32. If

 xx +− yy  = 34

(c) H 300 (d) H 400 and x ≠ 0, then approximately

what percentage of x + 3y is x − 3y? (a) 20% (c) 40%

(b) 30% (d) 50%

33. Intech Pvt. Ltd. generated a revenue of H 1250 in 2006. This was 12.5% of its gross revenue. In 2007, the gross revenue grew by H 2500. What is the percentage increase in the revenue in 2007? (a) 12.5% (c) 25%

(b) 20% (d) 50%

22-08-2017 06:54:25

568        Percentage 

34. If the price of petrol increases by 25%, by how much must a consumer cut down his consumption so that his expenditure on petrol remains constant? (a) 16.67% (b) 20%

(c) 25%

(d) 33.33%

35. 30% of the men are more than 25 years old and 80% of the men are less than or equal to 50 years old. 20% of all men play football. If 20% of the men above the age of 50 play football, what percent of the football players are less than or equal to 50 years? (a) 15%

(b) 20%

(c) 80%

(d) 70%

36. How much flower nectar must be processed to yield 1 kg of honey, if nectar contains 50% water, and the honey obtained from this nectar contains 15% water? (a) 1.7 kg (b) 1.9 kg

(c) 2.5 kg

(d) 3.33 kg

37. In an election contested by two parties, A secured 12% of the votes more than B. If B got 132000 votes, then by how many votes did it lose the election? (a) 300000 (c) 24000

(b) 168000 (d) 36000

38. A shepherd has 1 million sheep at the beginning of year 2000. The numbers grow by x% (x > 0) during the year. Due to widespread illness next year, many of his sheep die. The sheep population decreases by y% during 2001 and at the beginning of 2002 the shepherd finds that he is left with 1 million sheep. Which one of the following options is correct? (a) x > y (b) y > x (c) x = y (d) None of these 39. If the price of petrol increases by 25% and Raj intends to spend only an additional 15% on petrol, by how much % will he reduce the quantity of petrol purchased? (a) 13.5% (b)12%

(c) 10%

(d) 8%

40. A batsman scored 110 runs which included 3 boundaries and 8 sixes. What percent of his total score did he make by running between the wickets? 5 (a) 45% (b) 45  % 11 6 (c) 54  % (d) 55% 11

ANSWERS 1. (d)

6. (a)

11. (b)

16. (c)

21. (c)

26. (d)

31. (b)

36. (a)

2. (a)

7. (d)

12. (b)

17. (c)

22. (d)

27. (d)

32. (c)

37. (d)

3. (c)

8. (c)

13. (c)

18. (a)

23. (c)

28. (a)

33. (c)

38. (a)

4. (b)

9. (b)

14. (a)

19. (b)

24. (d)

29. (a)

34. (b)

39. (d)

5. (b)

10. (c)

15. (d)

20. (d)

25. (a)

30. (b)

35. (c)

40. (b)

EXPLANATIONS AND HINTS 1. (d) Consider the score in the sixth math test to be x. It is given that the final average should be at least 92. So we get 99 + 85 + 88 + 94 + 93 + x = 92 6 ⇒ 459 + x = 92 × 6 ⇒x = 552 − 459 = 93 So the marks in the sixth test should be 93. 2. (a) It is given that the total number of fans are 5400. So the total number of home fans will be 60 × 5400 = 3240 100 Total fans who are students are 810, so the number of fans that are not students will be 2430 × 100 = 75% 3240

Chapter 01-2.indd 568

3. (c) It is given that the total runs scored = 120 and the total runs scored using boundaries and sixes = 3 × 4 + 8 × 6 = 60 So the total runs scored by running between the wickets = 120 − 60 = 60 Therefore, the total percent of runs scored by running are 60 × 100 = 50 120 4. (b) It is given that the total worth of production of crops of all other states is 240 million pounds. So this production constitutes = 100 − 70 − 10 = 20%. Therefore, the worth of total production is 240 ×

100 = 1200 million pounds 20

22-08-2017 06:54:27

⇒ Therefore, the production of South Carolina is 10 × 1200 = 120 million pounds 100 5. (b) It is given that the new population of tigers = 45 and the percent increase = 25% If the original population was x, then 25 x = 45 100 5 ⇒ x = 45 4 ⇒ x = 36











8 90 90 x = 1620 × 100 100 100 1620 × 100 × 100 × 100 ⇒x= = 25000 8 × 90 × 90 ⇒

Therefore, the total number of voters on the voters list is 25000. 9. (b) It is given that the sales tax paid by Jack on item A is 8% of � 400, then

x+

8 × 400 = H 32 100

So the original population was 36. 6. (a) The total number of boys who passed is 42 × 1100 = 462 100 The total number of girls who passed is 30 × 700 = 210 100 So the total number of students who passed is 672 and the total number of students is 1800. Therefore, the percentage of students who passed is 672 × 100 = 37.33% 1800 Hence, the percentage of students who failed is 62.67%. 7. (d) It is given that the rise in the price of gasoline is H 4. So the percent rise in the price of gasoline will be 4 100 50 × 100 = = % 72 8 9 Now, the percent decrease is half of the percent increase. Therefore, the percent decrease is

Sales tax paid by Jack on item B is 10% of � 6400, then 10 × 6400 = H 640 100 Therefore, the total sales tax is � 672 and the percent of sales tax will be 672 672 × 100 = = 9.88% 6800 68 10. (c) It is given that the total marks the student got are 125 and the minimum marks required to pass are 165 If the maximum marks are x, therefore, 33 × x = 165 100 100 ⇒ x = 165 × = 500 33 11. (b) Say the larger number is x and the smaller number is y. We know that x − y = 1400. Also, 10 15 x= y 100 100 ⇒x = 1.5 y

1 50 25 × = % 2 9 9

Putting this value in the first equation 1.5y − y = 1400

New price of gasoline will be = 76 −



54 90 90 46 90 90 AND HINTS       1620 569 × × x EXPLANATIONS − × x = 100 100 100 100 100 100

 25100/9 × 76 = 76 − 2.11 = 73.89

8. (c) Let the total number of voters be x. Then, the votes polled are 90% of x.

⇒0.5y = 1400 ⇒y = 2800 ⇒x − 2800 = 1400 ⇒x = 4200 Thus, the two numbers are 4200 and 2800.

So the valid votes = 90% of (90% of x) Therefore,

12. (b) Given that radius of sphere = 20 cm 54% of [90% of (90% of x)] − 46% of [90% of (90% of x)] = 1620 ⇒









54 90 90 46 90 90 × × x − × x = 1620 100 100 100 100 100 100



Surface area of sphere will be 4pr 2 = 4 × p × 400 = 1600p cm2



8 90 90 × x = 1620 100 100 100 1620 × 100 × 100 × 100 ⇒x= = 25000 8 × 90 × 90 ⇒

Chapter 01-2.indd 569

22-08-2017 06:54:29

570        Percentage 

Volume of sphere will be

4 3 pr 3

4 = p × 8000 3 32000p cm3 = 3 So the percentage of the surface area to volume 1600 p = × 100 (3200 p / 3) 3 = × 100 20 = 15% 13. (c) x% of y =

 100x × y y% of x =



y ×x 100



17. (c) Say we have a quantity A, then X = (1.2)(1.3)(1.5)(1.2)A and

Y = (1.2)(1.5)(1.3)(1.2)A

Now, (1.2)(1.3)(1.5)(1.2) A X = =1 (1.2)(1.5)(1.3)(1.2) A Y

18. (a) Say we have a quantity X. After incrementing the quantity by 20%, we get 1.2X. Now, Y1 =

 100x × y + 100y × x =

xy xy 2xy + = 100 100 100

14. (a) The total number of votes of the rejected candidate is 40% and the total number of votes of the winner candidate is 60%. The difference between the votes is 20%. Also, the difference between the votes = 640 Say, total number of votes are x, then 20 × x = 640 100 100 ⇒ x = 640 × = 3200 20 Thus, total number of votes is 3200. 15. (d) Percent reduction in the consumption = 25 × 100 100 + 25 1 = × 100 = 20% 5



16. (c) We have two quantities X and Y and X is 25% more than Y. Hence X = 1.25Y =

5 Y 4

Also, 4 Y= X 5 Therefore, total percentage of Y of X will be

30 ×(1.2X ) + (1.2X ) 100

⇒Y1 = 1.56X After incrementing the quantity by 30%, we get 1.3X. Now Y2 =

= 2% of xy

Chapter 01-2.indd 570

Hence, the total decrease of quantity Y to X is 20%.

Hence, X is 100% of Y.

Hence,



(4 / 5)X × 100 = 80% X

20 ×(1.3X ) + (1.3X ) 100

⇒Y2 = 1.56X Hence, Y1 is 100% of Y2. 19. (b) Say length of cuboid be x, breadth be y and height be z The volume of the cuboid = x ⋅ y ⋅ z Now, new length of cuboid = 10% increase of x = 1.1x And new breadth of cuboid = 10% increase of y = 1.1y And new height of cuboid = 20% decrease of z = 0.8z Now, decreased volume = (1.1x)(1.1y)(0.8z) = 0.968xyz Hence, percent decrease in volume of the cuboid = 3.2% 20. (d) Say base of triangle be x and height of triangle be y. 1 The area of the triangle = x ⋅ y 2 Now, new base of triangle = 10% increase of x = 1.1x New height of triangle = 20% increase of y = 1.2y 1 (1.1x)(1.2y) = 1.32xy 2 Hence, the percent increase in the area of the triangle = 32% Now, increased area =

22-08-2017 06:54:32

EXPLANATIONS AND HINTS        571

21. (c) Let their marks be (x + 9) and x.

80 × 7500 = 6000 100 Therefore, valid votes polled by other candidate = 45 × 6000 = 45 × 60 = 2700 100

28. (a) Number of valid votes = Then, 56 (x + 9 + x ) 100 ⇒ 25 (x + 9) = 14 (2x + 9) x+9 =

⇒ 3x = 99 ⇒ x = 33

29. (a) We are given that 20 1 b= a⇒b= a 100 5

Hence, their marks are 42 and 33. 22. (d) Suppose originally he had x apples. Then, (100 − 40)

Therefore, b% of 20 = × x = 420 ⇒ x =

100

 42060×100100  = ×70020 = 100 a × 100 × 20 = 100 a = 4% of a. b

20

4

30. (b) Let the amount taxable purchases be H x.

23. (c) Total numbers from 1 to 70 which have 1 or 9 in the unit’s digit = 14 (1, 9, 11, 19, 21, 29, 31, 39, 41, 49, 51, 59, 61, 69) Total numbers from 1 to 70 = 70 14 Required percentage = × 100 = 20% 70



1

1 20 4 a× × 20 = a= 100b × 20 = 100 100 100

Then, 30 30 100 ⇒ x+ × =5  1006 × x = 100 100 6 



Therefore, cost of tax free items = 25 − (5 + 0.30) = H 19.70

xy  100x × y = 100 y xy B = y% of x =  × x = 100 100

24. (d) A = x% of y =

31. (b) Let the sum paid to Y per week be H z. Then,

Thus, A = B.

z+

120 z = 550 100

25. (a) Let the number of students be x. Then, ⇒

Number of students above 8 years of age = 80% of x. Therefore, 80 2 80 × x = 48 + × 48 ⇒ x = 80 ⇒ x = 100 100 3 100



32. (c) Dividing the numerator and the denominator of the given equation, we get

26. (d) According to the data given,







5 4 2 6 8 A+ B A+ B= 100 100 3 100 100 A B A B + = + ⇒ 20 25 25 75 ⇒

  x +1 y x −1 y



Required ratio = 4:3. 27. (d) Let the number be x. 5 3 16 Then, error = x − x = x 3 5 15 16x 3 Error% = × × 100 = 64% 15 5x





x − 3y × 100 = x + 3y

  x −3 y x +3 y

=

4 3

x 3x 4x +3 = −4 ⇒ = 7 y y y

Now, required percentage =

1 1 A= B 100 75 4 A ⇒ = 3 B ⇒

Chapter 01-2.indd 571



x+y 4 = ⇒ x−y 3



 201 − 251 A =  745 − 251 B





11 550 × 5 z = 550 ⇒ z = = 250 5 11





x − 3y × 100 = x + 3y

  x −3 y x +3 y

× 100

Putting value of

x , we get y

77 −+ 33  ×100 = 40%

22-08-2017 06:54:35

× 100

572        Percentage 

If Party A got 56% of the votes, then Party B got 44% of the total votes.

33. (c) We are given that Intech Pvt. Ltd. generated a revenue of H 1350 in 2006. Also, this was 12.5% of the gross revenue. Hence, if total revenue by end of 2007 is x, then 12.5 × x = 1250 ⇒ x = 10000 100 Thus, the growth is



Say total votes are y. Then, 44 100  ×y

13200044×100 

= 300000

votes



2500 × 100 = 25% 10000

The margin by which Party B lost the election = 12 × 300000 = 36000 100

34. (b) Let the price of petrol be H 100 per litre. Let the user use 1 litre of petrol. Therefore, his expense on petrol = 100 × 1 = H 100

= 132000 ⇒ y =

38. (a) Let the value of x be 10%. Therefore, the number of sheep in the herd at the beginning of year 2001 will be 1 million + 10% of 1 million = 1.1 million.

Now, the price of petrol increases by 25%. Therefore, new price of petrol = H 125 It is given that the expenditure remains constant. Hence, he will be spending only H 100 on petrol.

In 2001, the numbers decrease by y% and at the end of the year the number of sheep in the herd = 1 million.

Let x be the number of litres of petrol he will use at the new price.

Thus, 0.1 million sheep have died in 2001. Hence,

10..11 ×100 = 9.09% decrease during 2001.

Thus, 100 4 125 × x = 100 ⇒ x = = = 0.8 litres. 125 5 He has cut down his petrol consumption by 0.2 litres, i.e. 20% reduction.

Thus, x > y.

35. (c) It is given that 20% of the men are above the age of 50 years. 20% of these men play football.

39. (d) Let the price of 1 litre of petrol be x and let Aman initially buy y litres of petrol. Therefore, he has spent H xy on petrol initially.

Thus, 20% of 20% or 4% of the total men are football players above the age of 50 years.

When the price of petrol increases by 25%, the new price per litre of petrol is 1.25x.

Thus, 16% of the men are football players below the age of 50 years.

Aman intends to increase the amount he spends on petrol by 15%. 15 xy = 1.15xy Thus, he is willing to spend xy + 100

Therefore, the percent of men who are football play16 ers and below the age of 50 = × 100 = 80% 20

 

Let the new quantity of petrol that he can get be z.

36. (a) Flower nectar contains 50% of non-water part.

Then, 1.25x × z = 1.15xy ⇒ z =

In honey, this non-water part constitutes 85%. Therefore, amount of flower nectar is x. 50 85 85 ×x ⇒ ×1 ⇒ x = = 1.7 kg 100 100 50 37. (d) Let the percentage of total votes secured by Party A be x%.

As the new quantity that he can buy is 0.92y, he gets 0.08y lesser than what he used to get earlier. Thus, total reduction = 8%. 40. Runs scored by boundaries = 12

Runs scored by sixes = 48



Runs scored by running between wickets

As there are only two parties contesting in the election, the total sum of the votes secured by the two parties should be up to 100%. Thus,









x + x − 12 = 100 ⇒ 2x = 112 ⇒ x = 56%



Then, the percentage of total votes secured by Party B = (x − 12) %

Chapter 01-2.indd 572

1.15y = 0.92y 1.25

= 110 − (12 + 48)

= 50 50 5 Percentage = × 100 = 45  % 110 11

22-08-2017 06:54:37

CHAPTER 3

PROFIT AND LOSS

INTRODUCTION The price for which the item is bought is called its cost price (CP). The price for which the item is sold is called its selling price (SP). If the selling price is greater than the cost price, then the difference between the selling price and cost price is called profit. If the cost price is greater than the selling price, then the difference between the cost price and selling price is called loss. The price on the label is called the marked price (MP). The reduction made on the marked price of an item is called discount. When no discount is given, selling price is the same as marked price.

Chapter 01-3.indd 573

SOME IMPORTANT FORMULAE Some of the common formulae used are as follows: 1. To calculate profit: If the cost price is CP and the selling price is SP, then the profit (if SP > CP) will be Profit = SP − CP Profit Profit% = ×100 CP 2. To calculate loss: If the cost price is CP and the selling price is SP, then the loss (if CP > SP) will be Loss = CP − SP Loss Loss% = ×100 CP

22-08-2017 06:55:29

574     Profit and Loss 

3. To calculate SP: If the cost price is CP, marked price is MP and we know the profit percentage or loss percentage, then the selling price SP will be

MARGIN AND MARKUP Margin and markup are two very important terms which are used nowadays. However, they are often misunderstood and their meanings are interchanged.

100 + Profit% SP = × CP 100 100 − Loss% × CP SP = 100 SP = MP − Discount

Margin is defined as the percentage difference between the selling price and the profit. It is calculated by finding the net profit as a percentage of the revenue.

4. To calculate CP: If the selling price is SP and we know the profit percentage or loss percentage, then the selling price CP will be CP = SP ×

Profit margin =

Net profit Selling price

100 + Profit% 

In other words, profit margin is the selling price that is turned into profit.



On the other hand, markup or profit percentage is the percentage difference between the actual cost and the profit. It is calculated by finding the profit as a percentage of cost price.

100

100 CP = SP × 100 − Loss%



5. To calculate Discount%: If the marked price is MP and we know the discount, then the discount% will be Discount% =

Discount ×100 MP

Profit percentage =

Net profit Cost price

In other words, profit percentage is the percentage of cost price that one gets a profit on top of cost price.

SOLVED EXAMPLES 1. Ali bought a car for H 320000 and sold it for H 240000. What is the loss percentage? Solution:  Given that cost price of the car = H 320000 And selling price of the car = H 240000 So the loss = H 80000 Therefore, Loss% =

80000 × 100 = 25% 320000

2. A shopkeeper sold 20 books and used the money to buy 24 books. What is the profit percentage? Solution:  Let the cost price of one book be H 1. Then, SP of 20 books = CP of 24 books So, SP of 20 books = H 24 24 Hence, SP of 1 book = 20 So the profit =

24 4 −1 = 20 20

Therefore, Profit% =

4 / 20 × 100 = 25% 1

3. Nikhil sold two laptops for H 9900 each. On one laptop, he gained 10% and on the other laptop he

Chapter 01-3.indd 574

lost 10%. What is the loss or gain percentage in the whole transaction? Solution:  Given that selling price of the two laptops = H 19800 If the cost price of the first laptop (when gain is 10%) be x, then 9900 − x × 100 x ⇒ x = 99000 − 10x ⇒ 11x = 99000 ⇒ x = 9000

10 =

If the cost price of the second laptop (when loss is 10%) be y, then y − 9900 × 100 x ⇒ x = 10x − 99000 ⇒ 9x = 99000 ⇒ x = 11000

10 =

Total selling price = H 20000 Total cost price = H 19800 So the loss = H 200

22-08-2017 06:55:31

SOLVED EXAMPLES     575

So the profit = H 545

Therefore, Loss% =

200 × 100 = 1% 20000

4. Shikha bought 150 plates for H 3000 and spent H 500 on transportation. After paying an amount of H 1000 on their redecoration, she sold the plates at H 40 each. Find the total profit and profit percent. Solution:  Total cost price of 150 plates = H (3000 + 500 + 1000) = H 4500 Total selling price of plates = 150 × 40 = H 6000 So the total profit = H (6000 - 4500) = H 1500

So the profit% will be 545 × 100 = 13.625% 4000 8. The marked price of a frame is H 1600, which is 20% above the cost price. It is sold at a discount of 8% on the marked price. Find the profit percent. Solution:  It is given that the marked price of the frame = H 1600 Cost price =

1800 × 100 = 1500 120

Therefore, Selling price will be 1500 Profit% = × 100 = 33.3% 4500 5. A student bought a book from the bookstore for H 525. If the list price of the book was H 600, then what was the percentage of discount offered?

1800 −

So the profit = H 156 So the profit% will be 156 × 100 = 10.4% 1500

Solution:  Given that price of the book = H 525 List price = H 600

9. A store owner sold a bed for H 9020 and made a profit of 10%. What would be the profit percentage if he had sold the watch for H 9225?

So the discount = H 75 Therefore Discount% =

8 × 1800 = A1656 100

75 × 100 = 12.5% 600

6. Find the single discount equivalent to two successive discounts of 40% and 50%. Solution:  Let the marked price be H 100. After the first discount, price = H (100 - 40) = H 60.

Solution:  Selling price of the bed = H 9020 Profit = 10% So the cost price of the bed = 9020 ×

100 = A 8200 110

Now, if the selling price = H 9225 Profit = H 1025 So the profit% will be

After the second discount, price will be 50 60 − × 60 = H 60 - 30 = H 30 100 Total discount = H (100 - 30) = H 70 Thus, the single discount% = 70% 7. A company allows a discount of 10% on its product. Find the profit% if the cost price is H 4000 and listed or marked price is H 5050.

10. A florist allows a certain discount on the list price of his bouquet and still manages to get a profit of 15%. If the net cost price of the item is H 1200 and list price is H 2300, then what is the discount percent?

Solution:  It is given that the marked price of the product = H 5050

Solution:  It is given that the cost price = H 1200 115 So the selling price = 1200 × = A1380 100 List price = H 2300

And discount = 10%

So the discount = H (2300−1380) = H 920

So the selling price will be 5050 −

10 × 5050 = 4545 = H 4545 100

Now, the cost price = H 4000

Chapter 01-3.indd 575

1025 × 100 = 12.5% 8200

So the discount% will be 920 × 100 = 40% 2300 11. A trader mixes 26 kg of sugar at H 20 per kg with 30 kg of sugar of other variety at H 36 per kg and

22-08-2017 06:55:33

576     Profit and Loss 

16. 100 apples are bought at the rate of H 350 and sold at the rate of H 48 per dozen. What is the profit% or loss%? Solution:  Cost price of 56 kg sugar = (26 × 20 + 30 × 36) = (520 + 1080) = 350 = H 3.50 Solution:  Cost price of 1 apple = (26 × 20 + 30 × 36) = (520 + 1080) = H 1600 100 Selling price of 56 kg sugar = (56 × 30) = H 1680 48 = H 4. Selling price of 1 apple = 80 12 × 100 = 5% Therefore, profit% = 1600 0.50 100 2 Therefore, gain% = × 100 = = 14 % 12. A shopkeeper sells one transistor for H 840 at a gain 3.50 7 7 of 20% and another for H 960 at a loss of 4%. What 17. A vendor bought 6 sweets for a rupee. How many is the total gain or loss%? for a rupee must he sell to gain 20%? Solution:  Cost price of the first transistor = 100 Solution:  Cost price of 6 sweets = H 1 × 840 = H 700 120 6 120 100 Selling price of 6 sweets = × 1 = H  Cost price of the second transistor = × 960 = 5 100 96 6 H 1000 For H  , sweets sold = 6 5 So, total cost price = (700 + 1000) = H 1700 5 =5 For H 1, sweets sold = 6 × Total selling price = (840 + 960) = H 1800 6 100 15 18. The cost price of 20 articles is the same as the × 100 = 5 % Thus, gain% = selling price of x articles. If the profit is 25%, then 1700 17 what is the value of x? 13. A shopkeeper expects a gain of 22.5% on his cost price. If in a week, his sale was of H 392, what was Solution:  Let cost price of each article be H 1. his profit? Cost price of x articles = H x 100 × 392 = H 320 Solution:  Cost price = Selling price of x articles = H 20 122.5 sells the mixture at � 30 per kg. What is the total profit%?

 





 



















 







Therefore, profit = (392 − 320) = H 72

Profit = H (20 − x) Thus,

14. Sameer purchased 20 dozens of toys at the rate of H 375 per dozen. He sold each one of them at the rate of H 33. What was his profit%? 375 Solution:  Cost price of 1 toy = = H 31.25 12 Selling price of 1 toy = H 33

 

So, gain = (33 − 31.25) = H 1.75

 31.25 ×100 = 5.6% 1.75

Therefore, gain% =

15. The percentage profit earned by selling an article for H 1920 is equal to the percentage loss incurred by selling the same article for H 1280. At what price should the article be sold to make 25% profit?

1920 − x x − 1280 × 100 = × 100 x x ⇒ 1920 − x = x − 1280 ⇒ 2x = 3200 ⇒ x = 1600

Chapter 01-3.indd 576



20 − x × 100 = 25 x ⇒ 2000 − 100x = 25x ⇒ 125x = 2000 ⇒ x = 16

19. If the cost of an item is H 50 and it is sold for H 62.5, what is the margin? Solution:  We are given cost price = H 50 Selling price = H 62.5 Hence, profit = H 12.5 12.5 Therefore, margin = × 100 = 20% 62.5 20. If the selling price of an item is 25% more than the cost price, what is the markup?

Solution:  Let cost price be x. Thus,

Thus, required selling price =



125 × 1600 = H 2000 100

Solution:  Let the cost price be x. Then, Selling price = x +

25 x = 1.25x 100

Profit = 0.25x Markup =

0.25x × 100 = 25% x

22-08-2017 06:55:36

PRACTICE EXERCISE     577

21. A dealer purchased a washing machine for H 7660. He allows a discount of 12% on its marked price and still gains 10%. What is the marked price of the machine? Solution:  Cost price of the machine = H 7660, and profit% = 10%





100 + 10 × 7600 = H 8426 100 Let the marked price be x. Therefore, selling price =

12 3x x= 100 25 Therefore, the selling price = marked price 3x 22x = ­discount = x − 25 25 Also, selling price = H 8426 Then, the discount =









22x 25 = 8426 ⇒ x = 8426 × = 9575 25 22 Thus, marked price of the washing machine is H 9575. 22. How much percent above the cost price should a shopkeeper mark his goods so that after allowing a discount of 25% on the market price, he gains 20%?

Therefore, selling price = marked price - discount



= x−



x 3x = 4 4

Now, 3x 120 × 4 = 120 ⇒ x = = 160 4 3 Therefore, marked price = H 160. Hence, the marked price is 60% above the cost price. 1 23. An agent receives 3 % commission. What does his 2 sales amount to if his commission is H 70? Solution:  Let his sales be x. The commission of 1 H 70 is calculated as 3 % of sales. 2 1 ∴ 3 % of x = 70 2 3.5 7000 × x = 70 ⇒ x = = 2000 100 3. 5 Hence, his sales amount to H 2000. 24. The marked price of a cooler is H 1250 and the shopkeeper allows a discount of 6% on it. What is the selling price of the cooler? Solution:  We are given that marked price = H 1250 and discount% = 6%.

Solution:  Let the cost price be H 100. Gain required = 20%

Discount =

Therefore, selling price = H 120

6 × 1250 = H 75 100

Selling price = marked price - discount = 1250 - 75 = H 1175

Let the marked price be x. 25 x Then, discount = x= 100 4

Thus, the selling price of the fan is H 1175.

PRACTICE EXERCISE 1. Rohan bought a cycle for H 7000 and sold it at H 6790. What was the total loss%? (a) 12%

(b) 3%

(c) 30%

(d) 13%

2. A man bought 100 glasses for H 8500. He broke 20 glasses while carrying them. What should be the selling price of each of the remaining glasses in order to have an overall profit of 4%? (a) 110.5

(b) 100

(c) 121.5

(d) 115

3. A shopkeeper sold 40 chairs and used the money to buy 32 chairs. What is the profit% or loss%?

(a) H 37500 (b) H 32500 (c) H 30750 (d) H 30250 5. Arun sold two tables for H 3000 each. On one table he gained 20% and on the other table he lost 20%. What is the loss or gain% in the whole transaction? (a) 2% profit (c) 4% profit

6. A man sells an item at 10% above its cost price. If he had bought it at 10% less and sold it at the price of H 4 less, he would have made a profit of 20%. What is the cost price of the item? (a) H 216 (b) H 200

(a) 20% profit (c) 20% loss

(b) 10% loss (d) 10% profit

4. Rachel bought a piano for H 30000 and sold it at a profit of 2.5%. What is the selling price of the piano?

Chapter 01-3.indd 577

(b) 2% loss (d) 4% loss

(c) H 400

(d) H 350

7. If the cost price of 20 bags is equal to the selling price of 25 bags, what is the loss%? (a) 40%

(b) 25%

(c) 20%

(d) 10%

22-08-2017 06:55:38

578     Profit and Loss 

8. Mark bought paper sheets for H 7000 and spent H 100 on transportation. After paying an amount H 750, he had made 310 boxes, which he sold at H 26.50 each. Find the total profit and profit%. (a) H 365, 7.35% (c) H 1215, 17.36%

(b) H 465, 6% (d) H 365, 4.65%

9. A computer salesman offers an 8% discount on his computer. If the marked price on the computer is H 32000, what is the selling price of the computer? (a) H 30000 (b) H 26780 (c) H 28000 (d) H 29440 10. Shruti bought a book worth H 465 after getting a discount of 7%. What was the list price of the book? (a) H 472 (b) H 500

(c) H 535

(d) H 550

11. A bed was listed at H 32000. However, due to the lack of demand, it was sold at H 28000. What percent of discount is offered? (a) 12.5% (b) 6%

(c) 16%

(d) 8%

12. A company allows a discount of 15% on its product and still make a profit of 10%. Find the cost price if the listed or marked price is H 32700. (a) H 25268.18 (c) H 26460.00

(b) H 27894.22 (d) H 28244.10

13. After allowing a discount of 8%, there was still a gain of 5%. At what percent above the cost price was the marked price?

(a) 25%

(b) 30%

(c) 40%

(b) 14.5%

(c) 7.5%

(d) 12.75%

15. Find the single discount equivalent to two successive discounts of 5%, 10% and 20% (a) 15%

(b) 17.5%

(c) 35%

(d) 31.6%

16. A shopkeeper allows successive discounts of 12% and 8%. What is the net selling price of the item whose marked price is H 5800? (a) H 4640 (b) H 4225.56 (c) H 5215

(d) H 4695.68

17. A vendor allows successive discounts of 10% and 5%. If the vendor still made a profit of 20%, then what is the net cost price of the item whose marked price is H 2000? (a) H 1710 (b) H 1700 (c) H 1425 (d) H 1550 18. Ritesh allows a certain discount on the list price of an item and still manages to get a profit of 20%. If the net cost price of the item is H 6400 and list price is H 10240, then what is the discount%?

Chapter 01-3.indd 578

(d) 27.5%

(a) 6%

(b) 6.6%

(c) 7%

(d) 7.7%

20. Akhil sold his old phone for H 1980 and had a loss of 10%. What would be the selling price of the phone if the loss% was 32.5%? (a) H 1420 (b) H 1550 (c) H 1640 (d) H 1485 21. If the selling price of a product is doubled, then the profit triples. What is the profit%? (a) 66.5% (b) 100%

(c) 75%

(d) 125%

22. In a certain store, the profit is 320% of the cost. If the cost is increased by 25% but the selling price remains constant, approximately what percentage of the selling price is the profit? (a) 35%

(b) 70%

(c) 100%

(d) 200%

23. When a plot is sold for H 18700, the owner loses 15%. At what price must that plot be sold in order to gain 15%? (a) H 25800 (b) H 22500 (c) H 25300 (d) H 21200 24. On selling 17 bells at H 720, there is a loss equal to the cost price of 5 bells. What is the cost price of one bell? (a) H 60

(b) H 50

(c) H 55

(d) H 45

25. Some articles were bought at 6 articles for H 5 and sold at 5 articles for H 6. What is the profit%? (b) 33.3%

(c) 35%

(d) 44%

(d) 20%

14. Find the single discount equivalent to two successive discounts of 10% and 5%. (a) 15%

(c) 30%

19. Manik sold his watch for H 5040 and made a profit of 12%. What would be the profit% if he had sold the watch for H 4797?

(a) 30% (a) 50%

(b) 20%

Direction for Q26-Q28: Each of the questions given below consists of a question and two or three statements. You have to decide whether the data provided in the statement(s) is/are sufficient to answer the given question. 26. A man mixes two types of salt (A and B) and sells the mixture at the rate of H 17 per kg. Find his total profit%. I.  The rate of A is H 20 per kg. II.  The rate of B is H 13 per kg. (a) I alone is sufficient while II alone is not sufficient to answer. (b) II alone is sufficient while I alone is not sufficient to answer. (c) Both I and II are not sufficient to answer. (d) Both I and II are necessary to answer. 27. By selling a product with 20% profit, how much profit was earned?  I. The difference between cost price and selling price is H 40. II.  The selling price is 120% of the cost price.

22-08-2017 06:55:38

EXPLANATIONS AND HINTS     579

(a) I alone is sufficient while II alone is not sufficient to answer. (b) II alone is sufficient while I alone is not sufficient to answer. (c) Both I and II are not sufficient to answer. (d) Both I and II are necessary to answer.

(c) Only I and III

(d) All I, II and III

29. A soda for H 12 has a discount of 20% on it due to lack of demand. What is the selling price of the soda? (a) H 10

(b) H 9.40

(c) H 9.80

(d) H 9.60

28. By selling a chair, what is the profit% achieved? I. 5% discount is given on list price. II. If discount is not given, 20% profit is achieved. III.  The cost profit of the articles is H 5000. (a) Only I and II

30. A customer is allowed a discount of 10% on the purchase of a vase. The discount is H 7.40. What is the marked price of the vase? (a) H 50

(b) H 56

(c) H 64

(d) H 74

(b) Only II and III

ANSWERS 1.  (b)

4.  (c)

7.  (c)

10.  (b)

13.  (c)

16.  (d)

19.  (b)

22.  (b)

25.  (d)

28.  (a)

2.  (a)

5.  (d)

8.  (d)

11.  (a)

14.  (b)

17.  (c)

20.  (d)

23.  (c)

26.  (c)

29.  (d)

3.  (c)

6.  (b)

9.  (d)

12.  (a)

15.  (d)

18.  (a)

21.  (b)

24.  (a)

27.  (a)

30.  (d)

EXPLANATIONS AND HINTS 1. (b) Given that the total cost price of the scooter for Rohan = H 7000

Therefore, Loss% =

Selling price of the scooter = H 6790 So the loss = 7000 − 6790 = H 210

8 / 40 × 100 = 20% 1

4. (c) Given that the cost price of piano = H 30000

Therefore, Loss% =

Total profit percentage that Rachel got = 2.5%

210 × 100 = 3% 7000

Hence, if the profit is x, then

2. (a) Given that the total glasses the man bought = 100 2. 5 = Cost price of 100 glasses = H 8500

⇒ x = 2.5 × 300 = 750

Glasses broken while carrying = 20 We know that overall profit should be 4%. Now, the total selling price will be



8500



100 + 4 = 85 × 104 = A 8840 100

8840 Selling price of each remaining glass = 80 = H 110.5 3. (c) Let the cost price of one chair be H 1. Then, the selling price of 40 chairs = Cost price of 32 chairs So, the selling price of 40 chairs = H 32 Selling price of 1 chair =

32 40

x × 100 30000

Therefore, the total profit = H 750 Total selling price of the piano = 30000 + 750 = H 30750 5. (d) Given that the selling price of Table A = H 3000 Profit = 20% So the cost price of Table A = 3000 ×

100 = A 2500 120

Similarly, selling price of Table B = H 3000 Loss = 20% 100 = A 3750 80 So the total cost price on both the tables = H 6250 So the cost price of Table B = 3000 ×

Total selling price on both the tables = H 6000 Therefore,

Loss = 1 −

Chapter 01-3.indd 579

32 8 = 40 40

Loss% =

250 × 100 = 4% 6250

22-08-2017 06:55:40

580     Profit and Loss 

6. (b) If the cost price of the item is x, selling price of the item is 10 11 +1 = A x 100 10  



 10 9 Now, if the cost price = x −  100 x = 10 x and the selling price =

11 x−4 10

10. (b) It is given that the price of the book = H 465 If the list price is x and discount is 7%, then Discount × 100 x

7=

⇒ Discount = Price = List price - Discount

Then, profit% = 20%

465 = x −

7 x 100

⇒ 465 =

93x 100

Also, Profit ×100 = Profit% CP 20 × 9 10 x ⇒ Profit = = 9 50 x 100 Comparing both equations of selling prices, 9 9 11 x+ x= x−4 10 50 10 2 9 ⇒ x− x = 4 10 50 1 ⇒ x=4 50 ⇒ x = H 200

7 x 100

⇒ x = 500 Therefore, the marked or list price of the book = H 500 11. (a) It is given that the marked price of bed = H 32000 Selling price = H 28000 Discount = H 4000 Therefore, Discount% =

4000 × 100 = 12.5% 32000

7. (c) Let the cost price of one bag = H 1 So the cost price of 25 bags = H 25 And the cost price of 20 bags = H 20 Now, we know that the selling price of 25 bags = Cost price of 20 bags = H 20 So the loss = CP - SP = H 25 - 20 = H 5 Loss% will be 5 × 100 = 20 % 25

12. (a) It is given that the marked price of the product = H 32700 Discount = 15% Discount offered =

So the selling price of the product = H 32700 - 4905 = H 27795 If the cost price is x and profit% = 10%, then

8. (d) Given that the total cost price to make the boxes by Mark = H 7850 Total number of boxes made = 310 Total selling price = 310 × 26.5= H 8215 So the profit = H 365 Therefore, the profit% will be 365 × 100 = 4.65% 7850 9. (d) It is given that the marked price of computer = H 32000 Discount = 8% So,

8 × 32000 = H 2560 100 Selling price of the computer = H 29440 Discount =

Chapter 01-3.indd 580

15 × 32700 = A 4905 100

10 =

Profit × 100 x

⇒ Profit = x / 10 ⇒

x = 27795 − x 10



11x = 27795 10

⇒x=

277950 = 25268.18 11

Therefore, the cost price of the product = H 25268.18 13. (c) Let the cost price be H 100. Then gain = 5% of 100 = H 5 Selling price = H 105 Now, if the marked price be x. Then, discount will be

22-08-2017 06:55:43

EXPLANATIONS AND HINTS     581

25 % of x =

25 1 x= x 100 4

5 After second discount, price = 1800 − × 1800 100 = 1710 Hence, net selling price = H 1710

Selling price will be 1 3 x− x = x 4 4

Now, the total profit% = 20% If the cost price be x, profit will be (1710 - x). Therefore,

Therefore, 3 x = 105 4 105 × 4 ⇒x= = A 140 3

20 =

1710 − x × 100 x

⇒ x = (1710 − x) × 5 ⇒ 6x = 8550

Marked price = H 140

⇒ x = 1425

If the cost price is H 100 and the marked price is H 140, the percentage increase will be 40%.

Hence, the cost price = H 1425 18. (a) Cost price of the item = H 6400

14. (b) Let the marked price be H 100 After the first discount, price = H (100 - 10) = H 90 5 After the second discount, price = 90 − × 90 100 = H 90 - 4.5 = H 85.5

Now, profit percent = 20% Therefore, if profit is x, then x × 100 6400

20 =

Total discount = H (10 + 4.5) = H 14.5

x = 20 × 64 x = 1280

Thus, the single discount% = 14.5%

Selling price of the item = H 7680

15. (d) Let the marked price be H 100 After the first discount, price = H (100 - 5) = H 95

Now, list price of the item = H 10240

10 After the second discount, price = 95 − × 95 = 100    H 95 - 9.5 = H 85.5 20 After third discount, price = 85.5 − × 85.5 100 = 85.5 − 17.1 = 68.4

Thus, if the discount% is y, then

So the total discount = H (100 − 68.4) = H 31.6

price



= A 5800 −

256000 10240

19. (b) Selling price of the watch = H 5040

First discount = 12% the

⇒y=

10240 − 7680 × 100 10240

⇒ y = 25%

16. (d) Marked price = H 5800

So

y=

after

the

first

discount



12 × 5800 = H 5104 100

Profit% = 12% If the cost price be x, then 5040 − x × 100 x x = 4500

12 = Second discount = 8% So

the



price

= A 5104 −

after



8 × 5104 100

the

second

discount

= H (5104 - 408.32) =

H 4695.68

Hence, cost price of the watch will be H 4500. Now, if watch is sold at H 4797, then 4797 − 4500 × 100 4500 297 = × 100 = 6.6% 4500

Profit% =

Thus, the net selling price = H 4695.68 17. (c) Marked price of the item = H 2000 Successive discount offered are of 10% and 5%. 10 After first discount, price = 2000 − × 2000 100 = 1800

Chapter 01-3.indd 581

20. (d) Selling price of the phone = H 1980 Loss% = 10% If the cost price be x, then

22-08-2017 06:55:47

582     Profit and Loss 

10 =

x − 1980 × 100 x

⇒ x = 2200 Hence, cost price of the phone = H 2200 Now, if the phone is sold at a loss of 32.5% and y is the new loss, then y 32.5 = × 100 2200 ⇒ y = 715

Hence, cost price of 12 bells = selling price of 17 bells = H 720. 720 Cost price of 1 ball = = H 60. 12 25. (d) Let the number of articles bought is 30 (= 5 × 6). Cost price of 30 articles = H

Then, 3 (y − x) = (2y − x) ⇒ y = 2x Profit = (y − x) = (2x − x) = H x

 x ×100 = 100% x

Profit% =

6

 25 ×100 = 44%

26. (c) The ratio in which A and B are mixed is not given. Hence, both I and II together cannot give the answer. 27. (a) We are given that profit% = 20% According to statement I, profit = H 40 (i.e. the difference between selling price and cost price) Hence, I gives the answer, but II does not.

22. (b) Let cost price = H 100. Then, profit = H 320 and selling price = H 420.

28. (a) According to statement I, if the list price is H x. 95 19 ×x = x 100 20

Now, increased cost price = H 125.

Then, selling price =

We are given that the selling price remains the same. Hence, selling price = H 420

According to statement II, gain = 20%.

Profit = (420 − 125) = H 295

If selling price is H x, then

Thus, required percentage =

´ 100  70%  295 420

23. (c) We are given that selling price of a plot is H 18700 and loss% is 15%.

Cost price =

100 5x ×x = 120 6

Now, we need to find the selling price such that profit is 15%. Let the selling price be x. x=

100 + 15 × 22000 = 115 × 220 = 25300 100

Hence, selling price should be H 25300 in order to gain 15%. 24. (a) We are given that the difference of cost price of 17 bells and selling price of 17 bells is equal to the cost price of 5 bells.

and gain% =

5x



57 x − 50x 7x = 60 60

 60 × 5x ×100 = 14% 7x

100 − Loss% SP = × CP 100 100 − 15 ⇒ 18700 = × CP 100 18700 × 100 ⇒ CP = = 22000 85

 20 − 6  =  19x

Therefore, gain =

We know that,

Chapter 01-3.indd 582

 5 × 30 = H 36

11

Thus, profit% =

21. (b) Let C.P. be x and S.P. be y.

5

Selling price of 30 articles = H

Selling price of the phone for 32.5% loss = 2200 − 715 = H 1485

 6 × 30 = H 25

6

Hence, I and II only give the answer. 29. (d) The rate of discount on the soda = 20% 20 × 12 = H 2.40 The discount on the soda = 100 The selling price of the soda = 12 − 2.4 = H 9.60 30. (d) Let the marked price of the vase be H x. We are given that discount is H 7.40. Thus, 10 × x = 7.40 100 ⇒ x = 74 Hence, the marked price is H 74.

22-08-2017 06:55:50

CHAPTER 4

SIMPLE INTEREST AND COMPOUND INTEREST

INTRODUCTION When a sum of money is loaned from party A to B, the initial amount of sum is called principal (P). The total amount of time for which the money is ­borrowed is called period (N ). The extra amount of money paid back after the period is called interest (I ).

SOME IMPORTANT FORMULAE Some of the common formulae used are as follows: 1. To calculate simple interest: If the original sum is P, rate of interest is R and time period is N, then the simple interest SI and amount A will be SI =

The rate at which the interest is calculated on the principal is called rate of interest (R).

P ×N ×R 100

A = P + SI The total amount of money paid after the time period is called amount (A). When interest is paid as it falls due, it is called simple interest (SI ). The interest throughout the loan period is charged on the principal borrowed. When the money is lent at compound interest (CI ), the interest per annum or any fixed amount of time period is added to the principal amount and the interest for the next year is calculated from the new principal value. The process is repeated until the amount for the last period is found.

Chapter 01-4.indd 583

2. To calculate compound interest: If the original sum is P, rate of interest is R and N is the time period multiplied by the number of times the interest is compounded per year, then the amount A and compound interest CI will be



A = P 1+



N

R 100

CI = A − P

22-08-2017 06:56:39

584     Simple Interest and Compound Interest 

SOLVED EXAMPLES 1. What is the amount Rajesh will receive on a sum of H 28000 after 2 years at 6.5% interest per annum? Solution:  Given that principle = H 28000 Time period = 2 years Rate of interest = 6.5% 28000 × 6.5 × 2 So the simple interest = = 280 × 13 100 = H3640 So the total amount Rajesh will receive after 2 years = H (28000 + 3640) = H 31640 2. A sum of H 12000 amounted to H 13380 in 2 years on a certain rate of interest. What is the rate of interest? Solution:  Let the rate of interest be x Given that amount = H 13380 Sum = H 12000 So the total interest on the sum = H (13380 − 12000) = H 1380

If sum of money is x, then x×4×5 100 ⇒ x = 4450 890 =

Thus, the sum of money = H 4450 If the interest was compounded annually, then



Amount = 4450 1 + 4

= 4450

= H 5409.003

5. What will be the amount of H 20000 for 3 years compounded annually, the rate of interest being 2% for the first year, 5% for the second year and 10% for the third year? Solution:  Given that sum = H 20000 R1 = 2% p.a. R2 = 5% p.a.

Now, 12000 × x × 2 = 240x 100 1380 ⇒x= = 5.75 240

R3 = 10% p.a.

1380 =

Hence, the rate of interest on the amount = 5.75% 3. What will be the ratio of simple interest earned by a certain amount at the same rate of interest for 2 years and that for 6 years? Solution:  We know that the simple interest after Sum × Rate × 2 2 years = 100 Simple interest of same amount after 6 years = Sum × Rate × 6 = 100 Hence, 2 1 Sum × Rate × 2 Sum × Rate × 6 Ratio = ÷ = = 100 100 6 3 Ratio = 1 : 3 4. If the simple interest on a certain sum is H 890 after 4 years at a rate of 5% per annum. If the interest was compounded annually for the same time period at a same rate, what would be the amount? Solution:  It is given that the simple interest on the sum = H 890

Chapter 01-4.indd 584

105 100 



4

5 100



A = 20000 1 + = 20000 ×

 

 

2 5 10 × 1+ × 1+ 100 100 100



102 105 110 102×105×110 ×2 × × = 100 100 100 100

= 23562 Hence, the total amount = H 23562 6. What is the difference between the compound interests on H 8000 for 1 year at 10% per annum compounded yearly and half-yearly? Solution:  It is given that the sum of money = H 8000 If compounded annually then n = 1, so the compound interest will be



CI = 8000 1 +



1

10 − 8000 = H 800 100

If compounded annually then n = 2, then



 − 8000 2

10 100

CI = 8000 1 +

 

2

= 8000

11 − 8000 = (80 × 121) − 8000 10

= 9680 − 8000 = H 1680 Difference between the compound interest = H (1680 − 800) = H 880

22-08-2017 06:56:42

SOLVED EXAMPLES     585

7. If a sum of money becomes 5/3 of itself in 2 years at a certain rate of simple interest, then what is the rate of interest? Solution:  Let the principle sum be x, then 5 Amount = x 3 Simple interest will be 5 2 x−x = x 3 3

Hence, the rate of interest is (5/2)% and time period is 5 years.

Time period = 2 years Now, the rate will be SI × 100 (2/3)x × 100 100 = = = 33.33% 3 P ×T x×2 8. A sum of money placed at a compound interest doubles itself in 4 years. In how many years will it amount to 4 times itself? Solution:  Let the sum be x, then, we have



R 2x = x 1 + 100



⇒ 1+



4



10. If the sum of H 16000 is divided into two parts. Interest (simple) of one part is calculated at a rate of 10% for 2 years and interest (compound) of the other part is calculated with the same rate for same duration. If the total interest was H 3270, then calculate the two parts of the original sum. Solution:  If the part of simple interest is x and part of compound interest is (16000 − x) So the simple interest will be x × 10 × 2 x = 100 5 Compound interest will be



R = 21/4 100



2

(16000 − x) 1 +

10 100

− (16000 − x)

Total interest will be

Also,



4x = x 1 +

R 100



T

⇒ 4 = (21/ 4 )T ⇒ 4 = 2T/4 ⇒ 22 = 2T/4 ⇒

T =2 4

⇒T =8 Hence, the amount becomes 4 times of itself in 8 years. 9. The simple interest on a sum of money is 1/8 of the sum. If the number of years is twice the rate percent per annum, then what is the time period and rate of interest? Solution:  If the sum of money is x, then simple 1 interest is x. 8

Chapter 01-4.indd 585

Let the rate of interest be y and time period be 2y, then x × y × y /2 1 x= 8 100 100 ⇒ y2 = 16 10 5 ⇒y= = 4 2





2

20x 10 + (16000 − x) 1 + − (16000 − x) = 3270 100 100   121 20x + (16000 − x)  ⇒ − 1  = 3270 100   100  21  20x  ⇒ + (16000 − x)   = 3270 100 100    21  20x  = 3270 + (16000 − x)  ⇒  100  100  ⇒ 20x + 336000 − 21x = 327000 ⇒ x = 9000





   

Hence, the part on which the simple interest is calculated is H 9000 and the part on which compound interest is calculated is H 7000. 11. What will be the ratio of interest earned by certain amount at the same rate of interest for an year compounded yearly and half-yearly? Solution:  Let the principal be P, rate of interest be R% and time period be 1 year.

22-08-2017 06:56:45

586     Simple Interest and Compound Interest 

Compound interest for T when compounded yearly R 1 = P 1 + −P 100





Compound interest for T when compounded half2 R yearly = P 1 + −P 100





15. A man took loan from a bank at the rate of 12% p.a. simple interest. After 3 years he had to pay H 5400 interest only for the period. What is the principal amount borrowed by him? Solution:  We are given the values of time, ­interest and rate. Hence, we can calculate the principal amount using the formula,

Required ratio

 

 

 

1

Principal =

 

1

R R P 1+ −P 1+ −1 100 100 = = R 2 R 2 P 1+ −P 1+ −1 100 100

=

=

R 100 



100R

10000 + R + 200R − 10000 10000 2



= 2

R + 200R

100 R + 200

16. Mr. Batra invested an amount of H 13900 divided in two different schemes A and B at the simple rate of 14% p.a. and 11% p.a. respectively. If the total amount of simple interest earned in 2 years be H 3508, what was the amount invested in scheme B? Solution:  Let the sum invested in scheme A be H x and that in scheme B be H (13900 − x). Then,

(13900 − x) × 11 × 2

12. A person borrows H 5000 for 2 years at 4% p.a. simple interest. He immediately lends to another 1 person at 6 p.a. for 2 years. Find his gain in the 4 transaction per year. Solution:  Total interest when person borrows 25 2 money = 5000 × × = H 625 4 100 Total interest when person lends money to another 5000 × 4 × 2 person = = H 400 100 Gain in 2 years = 625 − 400 = H 225 225 Thus, gain in 1 year = H = H 112.50 2









 

13. A sum of money amounts to H 9800 after 5 years and H 12005 after 8 years at the same rate of simple interest. What is the rate of interest per annum? Solution:  Simple Interest for 3 years = H (12005 − 9800) = H 2205 2205 Simple Interest for 5 years = H  × 5 = H 3675 3 Thus, principal = H (9800 − 3675) = H 6125 100 × 3675 Required rate = = 12% 6125 × 5









14. A sum of H 12500 amounts to H 15500 in 4 years at the rate of simple interest. What is the rate of interest? Solution:  Simple interest for 4 years = H (15500 − 12500) = H 3000 100 × 3000 Rate of interest = = 6% 12500 × 4



Chapter 01-4.indd 586

= H 15000 10012××5400 3 



14 × 2 x ×100 +

 = 3508

100 ⇒ 28x − 22x = 350800 − (13900 × 22) ⇒ 6x = 45000 ⇒ x = 7500

So, sum invested in scheme B = H (13900 − 7500) = H 6400 17. Reena took a loan of H 1200 with simple interest for as many years as the rate of interest. If she paid H 432 as interest at the end of the loan period, what was the rate of interest? Solution:  Let rate of interest be x% and time be x years. Then, ×x×x 1200100  = 432 ⇒ x

2

=

432 12

⇒ x = 36 = 6 Since, time cannot be a negative quantity we do not consider −6. Hence, the rate of interest = 6% p.a. and time = 6 years. 18. How much time will it take for an amount of H 450 to yield H 81 as interest at 4.5% per annum of simple interest? Solution:  We are given the values of principal, interest and rate. Hence, we can calculate the principal amount using the formula, Time =

100 × 81 = 4 years 450 × 4 . 5

22-08-2017 06:56:47

SOLVED EXAMPLES     587

Solution:  Let the sum be x. Then,

19. There is 60% increase in an amount in 6 years at simple interest. What will be the compound interest of H 12,000 after 3 years at the same rate?

2    x 1 + 10 − x = 525   100  



Solution:  Let P be x. Then, S.I. = 0.6x.





100 × 0.6x R= = 10%p.a. x×6

 11 2  525 × 100 ⇒ x − 1 = 525 ⇒ x = = 2500  10  21  

 

Now, the principal amount to be compounded = H 12000, time = 3 years and rate = 10% p.a.  3   10 331   C.I. = 12000 ×  1 + − 1 = 12000 ×   100 1000     = 3972 Hence, the total compound interest = H 3972









Hence, simple interest =

 

107 ⇒ 100

x



  2 x 15000 × x × 2   − 15000 − = 96 15000 × 1 + 100 100    





= 34347 ⇒ x = 2 2

Thus, the time period is 2 years. ⇒ x2 = 21. What is the difference between the compound 1 interests on H 5000 for 1 years at 4% per annum 2 compounded yearly and half-yearly?







  26 51  = H 5304  = 5000 × × 25 50 



1 ×4 4 2 1+ × 1+ 100 100













 5000 × 1 + 2  100 



963× 2 = 64 ⇒ x = 8







  >2 x



3

  = 5000 × 5051 × 5051 × 5051

3

  = 5000 × 5051 × 5051 × 5051 = H 5306.04 Difference = H (5306.04 − 5304) = H 2.04

22. The compound interest on a certain sum of 2 years at 10% per annum is H 525. What is the simple interest on the same sum for double the time at half the rate percent per annum?

Chapter 01-4.indd 587



Solution:  Say the number of years is x. 26 51 Thus, × according to the data given 25 50 x 20 P 1+ > 2P 100

Compound interest when interest is compounded  2 half-yearly = 5000 × 1 + 100 



24. What is the least number of complete years in which a sum of money put out at 20% compound interest will be more than doubled?

Solution:  Compound interest when interest is com  1  ×4  4   2 × 1+ pounded yearly = 5000 × 1 +  = 5000 × 100 100  





 (100 + x)2 − 10000 − (200x)  = 96 ⇒ 15000   10000    

 





  2 x 2x   −1− ⇒ 15000  1 +  = 96 100 100    

x

107 34347 11449 = = = 100 30000 10000

×5×4 2500100  = H 500

Solution:  Let the rate of interest be x.

Then,





23. The difference between compound interest and simple interest on an amount of H 15,000 for 2 years is H 96. What is the rate of interest per annum?

Solution:  Amount = H (30000 + 4347) = H 34347 Let the time be x years.

7 100



∴ Sum = H 2500

20. The compound interest on H  30000 at 7% per annum is H 4347. What is the total time period?

30000 1 +



Now,

6 5

65 × 65 × 65 × 65 > 2

Thus x = 4 years. 25. Simple interest on a certain sum of money for 3  years at 8% per annum is half the compound interest on H 4000 for 2 years at 10% per annum. What is the sum placed on simple interest?  Solution:  The compound interest can be calculated using the following formula,

22-08-2017 06:56:49

588     Simple Interest and Compound Interest 

C.I.   2 10   − 4000  4000 × 1 + 100    





  2 10 11 11   − 4000 = 4000 × × − 4000 =  4000 × 1 + 100 10 10     11 11 = 4000 × × − 4000 = H 840 10 10













Let the principal amount be x and simple interest 1 = × 840 = H 420 2 420 × 100 Thus, sum = H  = H 1750 3×8









PRACTICE EXERCISE 1. A sum of money at simple interest amounts to H 815 in 3 years and to H 854 in 4 years. What is the rate of interest? (a) 5.59% (c) 7.36%

(b) 6.35% (d) 3.67%

2. Ted took a loan of H 1600 with simple interest for as many years as the rate of interest. If he paid H 400 as interest at the end of the loan period, what was the rate of interest? (a) 3.5% (c) 5%

(b) 6% (d) 8%

3. Mr. Gupta invested an amount of H 12000 divided in two different schemes A and B at the simple interest rate of 14% per annum and 11% per annum respectively. If the total amount of simple interest earned in 2 years be H 3015, what was the amount invested in Scheme B? (a) H 5750 (c) H 6250

(b) H 4655 (d) H 3650

4. A sum of H 800 is lent in the beginning of a year at a certain rate of interest. After 8 months, a sum of H 360 more is lent but at the rate twice the former. At the end of the year, H 31.20 is earned as interest from both the loans. What was the original rate of interest? (a) 7.5%

(b) 3.5%

(c) 6%

(d) 3%

5. Ankita lent H 4250 to Bhina for 2 years and H 3100 to Akhil for 4 years on a simple interest and at the same rate of interest and received H 1985.5 in all from both of them as interest. The rate of interest per annum is (a) 7.75% (c) 5%

(b) 9.5% (d) 10%

(b) 2:3

(c) 1:2

(d) 3:5

7. A person borrows H 5000 for 2 years at 4% per annum simple interest. He immediately lends it to

Chapter 01-4.indd 588

(a) H 425 (c) H 225

(b) H 365 (d) H 300

8. What will be the amount of H 14700 for 3 years compounded annually, the rate of interest being 8% for the first year, 16% for the second year and 20% for the third year? (a) H 22099.39 (c) H 19504.72

(b) H 26577 (d) H 43893.96

9. If the simple interest on a certain sum is H 200 after 2 years at a rate of 4% per annum. If the interest was compounded annually for the same time period at a same rate, what would be the amount? (a) H 2704 (c) H 2500

(b) H 4650 (d) H 3377

10. The difference between simple and compound interests compounded annually on a certain sum of money for 2 years at 4% per annum is H 20. The sum (in H) is (a) H 5000 (c) H 10000

(b) H 7500 (d) H 12500

11. What is the difference between the compound interests on H 6200 for 2 years at 8% per annum compounded yearly and half-yearly? (a) H 1375.75 (c) H 1203.35

(b) H 1544.32 (d) H 1155.65

12. At what rate of compound interest per annum will a sum of H 1200 become H 1348.32 in 2 years? (a) 8% (c) 6%

6. What will be the ratio of simple interest earned by a certain amount at the same rate of interest for 6 years and that for 9 years? (a) 3:2

another person at 6.25% per annum for 2 years. What is his gain in the transaction per year?

(b) 4% (d) 5.5%

13. Simple interest on a certain sum of money for 3  years at 8% per annum is half the compound interest on H 4000 for 2 years at 10% per annum. The sum placed on simple interest is (a) H 1750 (c) H 4840

(b) H 1500 (d) H 1275

22-08-2017 06:56:49

PRACTICE EXERCISE     589

14. If a sum of money becomes 7/4 of itself in 3 years at a certain rate of simple interest, then what is the rate of interest? (a) 12% (c) 25%

(b) 15% (d) 27%

(b) II alone sufficient while I alone not sufficient to answer. (c) Both I and II are not sufficient to answer. (d) Both I and II are necessary to answer. 22. What is the sum which earned interest? I.   The total simple interest was H 7000 after 7 years. II. The total of sum and simple interest was double of the sum after 5 years.  

15. A sum of money placed at compound interest doubles itself in 3 years. In how many years will it amount to 16 times itself? (a) 8 years (c) 24 years

(b) 12 years (d) 32 years

16. A sum of H 160000 is deposited by Arun in a bank for 3 years. If the rate of interest provided by the bank is 8% per annum, then what amount will Arun get after 3 years? (a) H 198400 (c) H 208200

23. What is the rate of simple interest?

(b) H 180000 (d) H 188600

I.  The total interest earned was H 4000. II.  The sum was invested for 4 years.

17. After how many years will a sum of H 12500 become H 17500 at the rate of 10% per annum? (a) 1 years (c) 3 years

(b) 2 years (d) 4 years

18. The simple interest on a sum of money is 1/9 of the sum. If the number of years is numerically equal to the rate percent per annum, then what is the rate percent of interest? (a) 10/3% (c) 6%

(b) H 20000 (d) H 25000

20. If a sum of money becomes 8 times itself in 3 years at compound interest, what is the rate of interest? (a) 25%

(b) 50%

(c) 75%

(d) 100%

Directions for Q21—Q24:  Each of the questions given below consists of a question and two or three statements. You have to decide whether the data provided in the statement(s) is/are sufficient to answer the given question.

24. What percentage of simple interest per annum did Amy pay to Derek?

(a) I alone sufficient while II alone not sufficient to answer. (b) II alone sufficient while I alone not sufficient to answer. (c) Both I and II are not sufficient to answer. (d) Both I and II are necessary to answer. Directions for Q25—Q28:  Each of the questions given below consists of a question and two or three statements. You have to decide whether the data provided in the statement(s) is/are sufficient to answer the given question. 25. What will be the compound interest earned on an amount of H 5000 in 2 years?  

I.  The interest rate is 10% p.a. II.  The sum earned simple interest in 10 years.

I. The simple interest on the same amount at the same rate of interest in 5 years is H 2000. II. The compound interest and the simple interest earned in one year is the same. III. The amount became more than double on compound interest in 10 years.

(a) I alone sufficient while II alone not sufficient to answer.

(a) I only (c) II and III only

21. The simple interest on a sum of money is H 50. What is the sum?

Chapter 01-4.indd 589

(a) I alone sufficient while II alone not sufficient to answer. (b) II alone sufficient while I alone not sufficient to answer. (c) Both I and II are not sufficient to answer. (d) Both I and II are necessary to answer.

I.   Amy borrowed H 8000 from Derek for four years. II. Amy returned H 8800 to Derek at the end of two years and settled the loan.

(b) 9% (d) 12%

19. Anya finds that an increase in the rate of interest 7 1 from 4 % to 5 % per annum increases her yearly 8 8 income by H 50. What is her initial investment? (a) H 10000 (c) H 15000

(a) I alone sufficient while II alone not sufficient to answer. (b) II alone sufficient while I alone not sufficient to answer. (c) Both I and II are not sufficient to answer. (d) Both I and II are necessary to answer.

(b) I and II only (d) I or II or III

22-08-2017 06:56:49

590     Simple Interest and Compound Interest 

26. What is the compound interest earned at the end of 3 years?

(a) H 8600 (c) H 8620

(b) H 8820 (d) H 8750



I. Simple interest earned on that amount at the same rate and for the same period is H 4500. II.  The rate of interest is 10% p.a. III. Compound interest for 3 years is more than the simple interest for that period by H 465.

33. What is the effective annual rate of interest corresponding to nominal rate of 6% p.a. payable half-yearly?

(a) I and II only (b)  II and III only (c) Either II or III only (d)  Any two of the three

34. A sum fetched a total simple interest of H 4016.25 at the rate of 9% p.a. in 5 years. What is the sum?

(a) 6.06% (c) 6.08%

(a) H 4462.5 (c) H 8900

(b) 6.07% (d) 6.09%

(b) H 8032.5 (d) H 8925

27. What is the rate of compound interest? I.  The principal was invested for 4 years. II.  The earned interest was H 1491. (a) I alone sufficient while II alone not sufficient to answer. (b)  II alone sufficient while I alone not sufficient to answer. (c) Either I or II alone sufficient to answer. (d)  Both I and II are not sufficient to answer. 28. What will be the compounded amount? I. H 200 was borrowed for 192 months at 6% compounded annually. II.  H 200 was borrowed for 16 years at 6%. (a) I alone sufficient while II alone not sufficient to answer. (b)  II alone sufficient while I alone not sufficient to answer. (c) Either I or II alone sufficient to answer. (d)  Both I and II are not sufficient to answer. 29. A bank offers 5% compound interest calculated on half-yearly basis. A customer deposits H 16000 each on 1st January and 1st July of a year. At the end of the year, what amount would he have gained by way of interest? (a) H 120 (c) H 122

(b) H 121 (d) H 123

30. The difference between simple and compound interests compounded annually on a certain sum of money for 2 years at 4% per annum is Re. 1. What is the sum? (a) H 625 (c) H 640

(b) 630 (d) H 650

31. What will be the compound interest on a sum of H 25000 after 3 years at the rate of 12 p.a.? (a) H 10123.20 (c) H 9720

(b) H 10483.20 (d) H 9000.30

32. Albert invested an amount of H 8000 in a fixed deposit scheme for 2 years at compound interest rate 5% p.a. How much amount will Albert get on maturity of the fixed deposit?

Chapter 01-4.indd 590

35. A person claims to be lending money at simple interest, but he includes the interest every six months for calculating the principal. If he is charging an interest of 10%, then what is the rate of interest? (a) 10% (c) 10.5%

(b) 10.25% (d) 11%

36. A lent H 5000 to B for 2 years and H 3000 to C for 4 years on simple interest at the same rate of interest and received H 2200 in all from both of them as interest. What is the rate of interest per annum? (a) 5% (c) 10%

(b) 7% (d) 12%

37. A sum of H 725 is lent in the beginning of a year at a certain rate of interest. After 8 months, a sum of H 362.50 more is lent but at the rate twice the former. At the end of the year, H 33.5 is earned as interest from both the loans. What was the original rate of interest? (a) 3.46% (c) 5%

(b) 4.5% (d) 6%

38. A sum of money at simple interest amounts to be H 815 in 3 years and to H 854 in 4 years. What is the sum? (a) H 650 (c) H 698

(b) H 690 (d) H 700

Directions for Q39 to Q40:  Each of the questions given below consists of a question and two or three statements. You have to decide whether the data provided in the statement(s) is/are sufficient to answer the given question. 39. A man borrowed a sum of money on compound interest. What will be the amount to be repaid if he is repaying the entire amount at the end of 2 years?   I.  The rate of interest is 5% p.a. II. Simple interest fetched on the same amount in one year is H 600.

22-08-2017 06:56:50

EXPLANATIONS AND HINTS     591

III. The amount borrowed is 10 times the simple interest in 2 years.

40. What is the principal sum?   I.  The sum amounts to H 690 in 3 years at S.I. II.  The sum amounts to H 750 in 5 years at S.I. III.  The rate of interest is 5% p.a.

(a) I or II only (b)  II and either I or III only (c) III only (d)  All I, II and III are required

(a) I and II only (c) I and III only

(b) II and III only (d) Any two of the three

ANSWERS 1. (a)

7. (c)

13. (a)

19. (b)

25. (a)

31. (a)

37. (a)

2. (c)

8. (a)

14. (c)

20. (d)

26. (c)

32. (b)

38. (c)

3. (a)

9. (a)

15. (b)

21. (d)

27. (d)

33. (d)

39. (b)

4. (d)

10. (d)

16. (a)

22. (d)

28. (c)

34. (d)

40. (d)

5. (b)

11. (c)

17. (d)

23. (c)

29. (b)

35. (b)

6. (b)

12. (c)

18. (a)

24. (d)

30. (a)

36. (c)

EXPLANATIONS AND HINTS 1. (a) Let the rate of interest be x%. So the simple interest after 3 years will be Sum × x × 3 100

⇒ x = 5.59% Thus, the rate of interest = 5.59% 2. (c) Let the rate of interest be x.

Amount = Sum + Simple interest

Given that the sum taken by Ted = H 1600

Thus, the amount after three years will be

Simple interest on that sum = H 400

Sum +

Sum × x × 3 100

Also, the rate of interest = Time period. Hence, 400 =

So



815 = Sum 1 +

3x 100



⇒ x2 = 25 ⇒  x = 5

Similarly, the amount after 4 years will be Sum × x × 4 100 4x ⇒ 854 = Sum + 1 + 100



If the amount invested in scheme A be x. Thus, the amount invested in scheme B = 12000 − x Simple interest from scheme A will be 14 × x × 2 100

Dividing the two equations, we get 815 (1 + 3x/100) = 854 (1 + 4x/100)



⇒ 815 1 + ⇒ 815 + ⇒

Chapter 01-4.indd 591





4x 3x = 854 1 + 100 100



3260x 2562x = 854 + 100 100

698 x = 39 100

Thus, the rate of interest on the amount = 5% 3. (a) Total amount invested by Mr. Gupta = H 12000

Sum +



1600 × x × x 100

Simple interest from scheme B will be 11 × (12000 − x) × 2 100 Thus, the total simple interest will be 14 × x × 2 11 × (1200 − x) × 2 + 100 100

22-08-2017 06:56:52

592     Simple Interest and Compound Interest 

We know that the total simple interest = H 3015, thus, 14 × x × 2 11 × (1200 − x) × 2 + = 3015 100 100 28x 22x + 2640 − = 3015 ⇒ 100 100 6x = 375 ⇒ 100 375 × 100 ⇒x= = 6250 6 Therefore, amount invested in scheme B = H (12000 − 6250) = H 5750 4. (d) If the original rate be x, therefore, the new rate of interest = 2x

6. (b) Simple interest after 6 years will be Sum × Rate of interest×6 100 Simple interest of same amount after 9 years Sum × Rate of interest× 9 100 So the ratio =

6 2 = 9 3

7. (c) Given that the sum of money borrowed = H 5000 500 × 2 × 4 So the total interest on 4% per annum = 100 = H 400 Total amount the person has to return = H (5000 + 400) = H 5400

Simple interest from the original amount will be Now, the person lend the money at 6.25% 800 × x × 1 100 Simple interest from the new amount will be 350 × 2x × 1 100 × 3

So the interest on the money will be 5000 × 6.25 × 2 = H 625 100 So the total amount be required = H 5625 And gain = 5625 − 5400 = H 225

Total simple interest will be 8x +

240x = 10.4x 100

Also, we know that total simple interest = H 31.2

8. (a) Given that sum = H 14700 R1 = 8% p.a. R2 = 16% p.a.

Therefore, R3 = 20% p.a. 10.4x = 31.2 ⇒x = 3 Thus, the original rate of interest = 3% 5. (b) Given that the total money lent by Akhil to Bhina = H 4250 Let the rate of interest be x. So the simple interest on the money lent to Bhina will be 4250 × x × 2 = H 85x 100 Total money lent to Chaman = H 3100 So the simple interest on the money lent to Chaman will be 3100 × x × 4 = H 124x 100



A = 14700 1 + = 14700 ×

 

 

8 16 20 × 1+ × 1+ 100 100 100

108 116 120 147 × 108 × 116 × 6 × × = 100 100 100 5 × 100

= H 22099.39 9. (a) Given that the simple interest on the sum = H 200 If the sum of money is x, then 200 =

x×2×4 100

x = 2500 Thus, the sum of money = H 2500 If the interest was compounded annually, then

Total simple interest = H 1985.5, so 124x + 85x = 1985.5



Amount = 2500 1 +

1985.5 ⇒x= = 9.5 209

Chapter 01-4.indd 592



2

4 100

  = H 2704 2

= 2500 Therefore, the rate of interest = 9.5%.



104 100

22-08-2017 06:56:55

EXPLANATIONS AND HINTS     593



10. (d) Let the sum be x. Simple interest on the sum will be x×2×4 8 x = 100 100





2

4 100



2

x 100

x ⇒ x = 0.06 × 100 = 6 100 Thus, the rate of interest = 6% ⇒ 1.06 = 1 +

Compound interest on the sum will be x 1+

⇒ 1.1236 = 1 +

104 100  x−x 2

−x =

13. (a) Let the sum of money be x and the sum which is compounded is H 4000, so the compound interest will be



Difference between compound interest and simple interest = H 20



2

400 1 +

10 − 4000 100

 

2

Thus, 104 100  x − x − 1008 x = 20 2

 

2



104 108 x− x = 20 100 100

⇒x

− 10800 1081610000  = 20

⇒x=

20 × 10000 = 12500 16

The sum of money = H 12500 11. (c) We know that the sum of money = H 6200

11 − 4000 = 40 × 121 − 4000 10 = 4840 − 4000 = H 840 1 Now, the simple interest = × 840 = H 420 2 So 420 × 100 x×3×8 420 = ⇒x= = 1750 100 24 So the original sum of money = H 1750 = 4000

14. (c) Let the principle sum be x 7 So the amount = x 4 7 3 Simple interest = x − x = 4 4 Time period = 3 years

If compounded annually then n = 2, so





2

Compound interest ==6200 6200 1 +

8 − −6200 6200 100

Now, the rate will be SI × 100 (3/4)x × 100 100 = = = 25% 4 P ×T x×3

= H 1031.68 If compounded annually then n = 4, so the compound interest will be



15. (b) Let the sum be x and rate be R. Then, using the formula for compound interest, we have







⇒ 1+

Difference between the compound interest = H (2235.03 − 1031.68) = H 1203.35



3

2x = x 1 +

8 4 6200 1 + − 6200 = H 2235.03 100

R 100



R = 21/3 100

Also,



12. (c) Let the rate of interest be x.

16x = x 1 + Given that the sum = H 1200 Amount = H 1348.32

⇒ 16 = (21/3 )T

Time period = 2 years

⇒ 16 = 2T /3

Using the formula of compound interest, we have

⇒ 24 = 2 T / 3





Chapter 01-4.indd 593



1348.32 x = 1+ 1200 100



2

1348.32 = 1200 1 +

x 100



2



T

R 100

T =4 3 ⇒ T = 12 Hence, the amount becomes 16 times of itself in 12 years. ⇒

22-08-2017 06:56:59

594     Simple Interest and Compound Interest 



16. (a) Simple interest will be

⇒ 1+

160000 × 8 × 3 = 1600 × 24 = H 38400 100 Amount = Principle + Simple interest

R =1 100 ⇒ R = 100 ⇒

Amount = H 160000 + 38400 = H 198400 17. (d) Given that the principle sum = H 12500 Amount = H 17500



R =2 100

Hence, the rate of interest on the amount compounded annually is 100%. 21. (d) We are given that S.I. = H 50

Simple interest = H 5000 Now, simple interest is given by

According to statement I, rate = 10% p.a.

12500 × 10 × T 100 500 ⇒T = =4 125 Time period in which the sum of H 12500 will amount to H 17500 is 4 years.

According to statement II, time = 10 years. 100 × S.I. 100 × 50 Thus, sum = = = H 50 T×R 10 × 10

5000 =

18. (a) Let the sum of money be x. 1 Then, the simple interest is x 9 Let the rate of interest and time period be y. Then using the formula of simple interest, we get 1 x× y × y x= 9 100 100 2 ⇒y = 9 10 ⇒y= 3 10 Hence, the rate of interest is % 3 19. (b) Let the sum of money be x. Then, x × (41/8) x × (39/8) − = 50 100 100 ⇒



 



Thus, I and II together give the answer. 22. (d) Let the sum be H x. According to statement I, simple interest = H 7000 and time = 7 years According to statement II, sum + S.I. for 5 years = 2 × Sum ⇒ Sum = S.I. for 5 years. Now, S.I. for 7 years = H 7000. 7000 ∴ S.I. for 1 year = = H 1000 7 Thus, I and II both are needed to get the answer. 23. (c) We know that, R =

100P ××TS.I.

Now, according to statement I, simple interest = H 4000 According to statement II, time = 4 years But principal is unknown and we cannot find the value of rate of interest. Hence, the given data is insufficient to get r. 24. (d) Let the rate be r% p.a. According to statement I, principal = H 8000 and time = 4 years.

2 x = 5000 8

⇒ x = 5000 × 4 = 20000 Hence, the initial investment is H 20000.

According to statement II, simple interest = (8800 − 8000) = H 800 100 × S.I. 100 × 800 1 ∴R= = = 2 % p.a. 8000 × 4 P× T 2



 



20. (d) Let the sum of money be x Thus, I and II both are needed to get the answer. Then, the amount = 8x Hence, using the formula of compound interest we get





⇒ 1+

Chapter 01-4.indd 594



3

8x = x 1 +



R 100

3

R = 23 100

25. (a) We are given that principal = H 5000 and time = 2 years. According to statement I, simple interest on H 5000 in 5 years is H 2000. 5000 × R × 5 = 2000 ⇒ R = 8 100 Thus, only I gives the answer.

22-08-2017 06:57:02

EXPLANATIONS AND HINTS     595





26. (c) According to statement I, simple interest for x×4×2 2x Simple interest = = 3 years = H 4500 100 25 According to statement II, rate of interest = 10% p.a. 51x 2x ∴ − = 1 ⇒ x = 625 According to statement III, Compound interest − 625 25 Simple interest = H 465 Hence, sum = H 625. Clearly, using I and III, we get compound interest = H (465 + 4500). 31. (a) We are given, principal = H 25000, time = Hence, II is redundant. 3 years and rate = 12% p.a. 100 × 4500  3 Also, from I and II, we get a sum = = 15000 12   28 28 28   10 × 3 = 25000 × 1 + Thus, amount   = 25000 × × ×  100 × 4500 100   25 25 25   = 15000   10 × 3   3 Now, Compound interest on H 15000 at 10% p.a. for 12   28 28 28  = 25000 × 1 +  = 25000 × × ×  = H 35123.20 3 years can be obtained. Hence, III is redundant. 100   25 25 25    Therefore, either II or III is redundant.  Thus, C.I. = H (35123.20 − 25000) = H 10123.20 27. (d) Let Principal = H P and Rate = R% p.a. Then, 32. (b) We are given, principal = H 8000, time = 2 years  4 R   and rate = 5% p.a. Amount = H P 1 +  100    2   5   21 21   Thus, amount =  8000 × 1 + Thus,  =  8000 × ×  100   20 20       4 4    R R 2     21 21  C.I. = P  1 + − 1 ⇒ P  1 + −51 = 1491 × 1 +   =  8000 × ×  = H 8820 100   = 8000 100 100   20 20       It is clear that it does not give the answer. 33. (d) Let the principal amount be H 100.



















=





=

 2 5 5  = 1600 × 1 + + 1600 × 1 + 2 × 100 2 × 100    41  41 41 × = 1600 × + 1600 ×   40  40 40



    



  2 4 676 51   − x = x−x = x x 1 + 100 625 625    

Chapter 01-4.indd 595

.25 401625 = = H 8925 100 ×9 4016  ×5 45 

35. (b) Let the sum be H 100. Then, Simple interest for first 6 months =

100100×10×2×1 = H 5 



105 × 10 × 1 = Simple interest for last 6 months = 100 × 2 H 5.25 Thus, the amount at the end of 1 year = H (100 + 5 + 5.25) = H 110.25 Thus, the effective rate = 110.25 − 100 = 10.25% 36. (c) Let the rate be x% p.a. × x× 4 500 ×100x × 2 + 3000100  = 2200

Compound interest =



Sum =

Then,

30. (a) Let the sum be H x. Then,





Thus,

 1600 × 41 × 81   = H 3321 =   40 × 40 Therefore, compound interest = H (3321 − 3200) = H 121



Amount on that principal for 1 year when com 2 3   = H 106.09 H106.09 pounded half-yearly = H 100 × 1 + = 100     Thus, effective rate = (106.09 − 100) = 6.09% 34. (d) We are given total simple interest = H 4016.25, rate = 9% p.a. and time = 5 years.

29. (b) We are given, principal = H 1600, time = 1 year compounded half yearly and rate = 5% p.a. Amount















28. (c) According to statement I, amount  16  6   200 × 1 + H   100     According to statement II, amount  16  6   H 200 × 1 +  100     Thus, I as well as II gives the answer.















⇒ 100x + 120x = 2200 ⇒ x =

2200 = 10 220

Hence, the rate = 10%.

22-08-2017 06:57:05

596     Simple Interest and Compound Interest 

37. (a) Let the original rate be x. Thus, new rate = 2x.

40. (d) According to statement I, amount = H 690 and time = 3 years.

Also, original rate is for 1 years and the new rate is 1 for 4 months = years. 3 Thus,

According to statement II, amount = 750 and time = 5 years.



 



725 × x × 1 362.5 × 2x × 1 + = 33.5 100 100 × 3 ⇒ ( 2175 + 725 ) x = 33.5 × 100 × 3 ⇒ ( 2900 ) x = 10050 ⇒ x =

10050 = 3.46 2900

Hence, original rate = 3.46%. 38. (c) Simple interest for 1 year = H (854 − 815) = H 39

According to statement III, rate of interest = 5% p.a. From I and II, if sum is x and rate is r. We have, x×r×3 x+ = 690 ⇒ x (100 + 3r ) = 69000  100

(i)

Similarly, x+

x×r×5 = 750 ⇒ x (100 + 5r ) = 75000  100

(ii)

Simple interest for 3 years = H (39 × 3) = H 117 Thus, sum = H (815 − 117) = H 698. 39. (b) According to statement I, rate = 5% p.a. According to statement II, S.I. for 1 year = H 600. According to statement III, sum = 10 × (S.I. for 2 years). Now, I and II give the sum. For this sum, the compound interest and hence the amount can be obtained. Also, II gives Simple interest for 2 years = H (600 × 2) = H 1200. Now, from III, sum = H (10 × 1200) = H 12000.





100 × 1200 = 5% p.a. 2 × 12000 Hence, I is redundant and I or III redundant.

Chapter 01-4.indd 596

(100 + 5r ) 75000 = ⇒ 6900 + 345r (100 + 3r ) 69000 = 7500 + 225r ⇒ 120r = 600 ⇒r=5 x (100 + 5 × 5) = 75000 ⇒ x (125) = 75000 ⇒ x = 600 Thus, the sum is H 600.

Thus, III is redundant.

Thus, rate =

Dividing Eq.(ii) by Eq.(i), we get

From I and III, or II and III, we have the value of rate and we can put in the formula of the amount to calculate the sum. Hence, we can use any of the two statements in order to calculate the sum.

22-08-2017 06:57:06

CHAPTER 5

TIME AND WORK

INTRODUCTION Work is defined as the amount of job or task completed. Problems on work are based on the application of the concept of ratio of time and speed. In physics, a force is said to do work if, when it acts on a body, there is a displacement of the point of application in the direction of force. However, when we talk about work in time, speed and distance problems, then an analogy between time, speed, distance and work exists. Work ≡ 1 ≡ Distance Hence, rate at which the work is done = speed Number of days required to do the work = time Also, problems on pipes and cisterns are closely related to problems on work. Filling or emptying a cistern can be considered as work done. A pipe is called an inlet if it fills the cistern, and it is called an outlet if it empties the cistern.

Chapter 01-5.indd 597



IMPORTANT FORMULAS AND CONCEPTS

Some important formulas and concepts of work are as follows: 1. A man can do a piece of work in a number of days, then in 1 day (1/a)th of the work is done. Similarly, if a man does (1/a)th of a work in 1 day, then he can complete the work in a days. 2. If A is a times as good a workman as B, then he will take (1/a)th of the time taken by B to do the same work. 3. If A and B can do a piece of work in x and y days, respectively, the amount of time they take to finish the work together will be xy/(x + y) days, and the total amount of work done by them in 1 day will be [xy/(x + y)]th of the total work. 4. If number of men to do a job is changed in the ratio a:b, then time required to do the work will be in the ratio b:a, assuming amount of work done by each of them in the given time is the same.

22-08-2017 06:57:57

598     Time and Work 

5. If two men A and B together can finish a job in x days, and if A working alone takes a days more than A and B working together, and B working alone takes b days more than A and B working together, then x = ab 6. Sometimes, two operations or work are being performed which are opposite in nature. A tank being filled by a pipe and being emptied by another one or a worker throwing rocks into the truck and another working picking those up and throwing them back are examples of work of opposite nature. In such situations, for the purpose of calculations one value is taken as positive and the other is taken with a sign negative.



Usually for pipes and cisterns problems, the pipe which drains the container is said to do negative work. 7. Efficiency is basically defined by the formula Input ×100. However, in the problems encountered Output



in this chapter, you will come across efficiency used to define the work capacity of entity with respect to another. Thus, if A is 50% as efficient as B, then A has half the work capacity as B. Hence, if we are given the capacity of one of the two, we can calculate the capacity of the other.

SOLVED EXAMPLES 1. A worker A can finish work in 3 days and B can finish work in 9 days. How long will it take to finish the work if they both worked together? Solution:  We know that 1 A’s 1 day’s work = 3 1 B’s 1 day’s work = 9

(i)

1x + 3y =

1  30

(ii)



Solving the two equations, we get



1 1 3 +1 4 + = = 3 9 9 9 x=

2. A and B can do a piece of work in 10 and 20 days, respectively. In how many days can A do the work if he is assisted by B on every third day? Solution:  We know that 1 1 A’s 2 days’ work = ×2 = 10 5





10 + 20  = 20 1

(A + B)’s 1 day’s work =

1

3

 20 + 5  = 20 3

1

7

7 work is done in 3 days. 20

20 Therefore, whole work will be done in 3 × days = 7 60 days 7 3. If 5 men and 3 women can do a piece of work in 6 days while 1 man and 3 women can do the same in 30 days, then what is the time taken by 3 men and 4 women in doing the same type of work?

Chapter 01-5.indd 598

1  6



Thus, the time taken by A and B to complete the 9 work together = days 4

Thus,

5x + 3y =

(A + B)’s 1 day’s work =

Work done in 3 days =

Solution:  Let 1 man’s 1 day’s work be x and 1 woman’s 1 day’s work be y. Then,

1 ;y=0 30

 30 + 0 = 10 3

(3 men + 4 women)’s 1 day’s work =

1

Thus, 3 men and 4 women can do the work in 10 days. 4. Suppose A is twice as good as workman as B. If A and B together can finish the work in 30 days, then how long will they take to finish the work alone? Solution:  Let A takes x days to finish the work alone and B takes 2x days to finish the work alone. So, A and B take 30 days to finish the work together. 1 (A + B)’s 1 day’s work = 30 Then, 1 1 1 + = x 2x 30 3 1 = 2x 30 ⇒ x = 45 ⇒

Hence, A can finish work alone in 45 days and B can finish work alone in 90 days.

22-08-2017 06:58:00

SOLVED EXAMPLES     599

5. If 100 workers can finish a work in 50 days, then how many days does it take for 30 workers to finish the work? Solution:  Given that 100 workers can finish the work in 50 days. 1 So the total work by 100 workers in 1 day = 50 1 Total work done by 25 workers in 1 day = 200 So the time taken by 25 workers to complete the work = 200 days

8. A large tank can be filled using two pipes A and B in 3 h and 5 h, respectively. How long will it take to fill the entire tank if both the pipes are used to fill half of the tank and the rest of the half by A alone? Solution:  We know that 1 A’s 1 h work = 3 1 B’s 1 h work = 5 Now, (A and B)’s 1 h work =

6. Worker A alone can complete a work in 16 days and B alone in 12 days. Starting with A, they work on alternate days. How many days will it take for the total work to be completed?

A and B together fill the tank in 15/8 h. A and B together fill half of the tank in 15/16 h. A can fill the tank alone in 3 h. Hence, A can fill half of the tank in 3/2 h.

Solution:  We know that

16 + 12  = 48 1

(A + B)’s 2 days’ work =

1

So the remaining work =

7

9. There are two inlet pipes and an outlet pipe. The efficiency of one of the inlet pipes is double than that of the other inlet pipe. The efficiency of the outlet pipe is half that of the lesser-efficient inlet pipe. The empty tank gets filled in 8 h when all the pipes are opened. How many hours will it take to fill the empty tank when the most efficient inlet pipe is plugged and the rest kept opened?

1 8

Work done by A on 13th day = So the remaining work =

Thus, the total time taken to fill the tank = 15 3 39 + = h 16 2 16

7

 48 × 6 = 8 7

Work done in 12 days =

1 16

1 1 1 − = 8 16 16

On 14th day, work is done by B.

Solution:  Let the number of hours taken by the outlet and two inlet pipes be 4x, 2x and x respectively. So, in part of tank filled in 1 h will be

1 12 1 3 work is done by B in day 16 4

Now, B’s 1 day’s work =

1 1 1 1 + − = x 2x 4x 12

Therefore, the total time taken = 13

3 days 4

7. Two pipes A and B can fill a tank in 8 and 12 min, respectively. Both the pipes are opened together, but after 3 min, pipe A is turned off. What is the total time required to fill the tank? Solution:  We know that





Total time taken to fill the remaining part = The tank will be full in 4 +

Chapter 01-5.indd 599





5 1 = 4x 8

⇒ x = 10 Hence, inlet pipe with higher efficiency fills tank in 10 h. Now, the second inlet piped is plugged. Thus, the desired pipes fill 1 1 1 − = 2x 4x 4x

1 1 5 Part filled in 4 min = 3 + = 8 12 8 3 Remaining part = 8 1 Part filled by B in 1 min = 12



1 1 8 + = 3 5 15

Time taken = 4x = 4 × 10 = 40 h

3 9 × 12 = 8 2

9 minutes = 8 min 30 s 2

10. There are three kinds of workers A, B and C in the ratio of 3:2:1. There are 20 workers of type A, 30 of type B and 36 of type C in a factory. Their weekly wages amount of H 780 is divided in the ratio of work done by the workers. What will be the wages of 15 workers of type A, 21 workers of type B and 30 workers of type C for 2 weeks?

22-08-2017 06:58:03

600     Time and Work 

Solution:  We know that Ratio of wages of A, B, C = (3 × 20):(2 × 30): (1 × 36) = 5:5:3

13. One pipe can fill a tank three times as fast as another pipe. If together the two pipes can fill the tank in 36 minutes, then how long will it take for the slower pipe to fill the tank alone?

So the total wages of 20 workers of type A = 5 A = × 780 = H 300 13 And wages of 1 worker of type A = H 15

Solution:  Let the slower pipe alone fill the tank in x minutes. Then, the faster pipe will fill it in x/3 minutes.

Similarly, wages of 30 workers of type B = 5 B= × 780 = H 300 30 And wages of 1 worker of type B = H 10 Similarly, wages of 36 workers of 3 C = × 780 = H 180 13 And wages of 1 worker of type C = H 5

type

1 3 1 4 1 ⇒ = ⇒ x = 144 minutes. + = x x 36 x 36 14. Kanan and Biswa are working on an assignment. Kanan takes 6 hours to type 32 pages on a computer while Biswa takes 5 hours to type 40 pages. How much time will they take working together on two different computers to type an assignment of 110 pages. Solution:  Number of pages typed by Kanan in 32 16 1 hour = = 6 3 Number of pages typed by Biswa in 1 hour = 40 =8 5 Number of pages typed by both in one hour = 16 40 +8 = 3 3

Therefore, total weekly wages of 15 workers of type A, 21 workers of type B and 30 workers of type C 15 × 15 + 21 × 10 + 30 × 5 = H 585 Thus, total wages for 2 weeks = H 1170 11. Two pipes A and B together can fill a cistern in 4 hours. Had they been opened separately, then B would have taken 6 hours more than A to fill the cistern. How much time will be taken by A to fill the cistern separately? Solution:  Let the cistern be filled by pipe A alone in x hours. Then, pipe B will fill it in (x + 6) hours. 1 1 1 x+6+x 1 + = ⇒ = x ( x + 6) 4 x( x + 6 ) 4 ⇒ x2 − 2x − 24 = 0 ⇒ ( x − 6 )( x + 4 ) = 0 ⇒ x = 6 (Since, we ignore negative values) 12. Two pipes A and B can fill a tank in 15 minutes and 20 minutes respectively. Both the pipes are opened together but after 4 minutes, pipe A is turned off. What is the total time required to fill the tank? 1 1 7 + = Solution:  Part filled in 4 minutes = 4 15 20 15 7 8 = Part remaining = 1 − 15 15 1 Part filled by B in 1 minute = 20 1 8 ∴ : :: 1 : x 20 15 8 ⇒x= × 1 × 20 = 10 min 40 sec 15













Thus, the tank will be full in (4 min + 10 min 40 sec) = 14 min 40 sec.

Chapter 01-5.indd 600





Time taken by both to type 110 pages =

110 × 40  = 8 4 hours 3

1

15. A, B and C can complete a piece of work in 24, 6 and 12 days respectively. How long will they take to complete the same work together? 1 1 1 7 Solution:  (A + B + C)’s 1 day’s work = + + = 24 6 12 24 1 1 1 7 + + = 24 6 12 24 24 3 Thus, they complete the entire work in =3 7 7 days









16. A can lay railway track between two given stations in 16 days and B can do the same job in 12 days. With help of C, they did the job in 4 days only. How long does it take for C to do the job alone? 1 Solution:  (A + B + C)’s 1 day’s work = 4 1 A’s 1 day’s work = 16 1 B’s 1 day’s work = 12 1 1 1 5 + = Therefore, C’s 1 day’s work = − 4 16 12 48



So, C can do the work alone in



48 days. 5

22-08-2017 06:58:05

PRACTICE EXERCISE     601

17. A is thrice as good as workman as B and therefore is able to finish a job in 60 days less than B. How long will it take them to finish when working together?

Solution:  Let the number of fill pipes be x. Therefore, total number of waste pipes = 8 − x.

Solution:  Ratio of times taken by A and B = 1:3 If the time difference is 2 days, then B takes 3 days. 3 × 60 = 90 If difference is 60 days, then B takes 2 days.

1 th Therefore, each of the fill pipes will fill of the 8 tank in an hour. x th Hence, x fill pipes will fill of the tank in an 8

So, A takes 30 days to do the work.

hour. Similarly, each of the waste pipes will drain





1 30 1 B’s 1 day’s work = 90

Each of the fill pipes can fill the tank in 8 hours.





the full tank in 6 hours or hour.

A’s 1 day’s work =

 30 + 90  = 90 = 45 1

(A + B)’s 1 day’s work =

1

4



th

1 6

of the tank in an

Therefore, (8 − x) waste pipes will drain of the tank in an hour.



8− x 6



th

2

45 1 Therefore, A and B together can do it in = 22 2 2 days. 18. A tank is fitted with 8 pipes, some of them that fill the tank and others that are waste pipes meant to empty the tank. Each of the pipes that fill the tank can fill it in 8 hours, while each of those that empty the tank can empty it in 6 hours. If all the pipes are kept open when the tank is full, it will take exactly 6 hours for the tank to empty. How many of these are fill pipes?

Now, when all the pipes are open, it takes the tank

6  1

6 hours to drain. Hence, drained in an hour.

th

of the tank gets

(Amount of water filled by fill pipes − amount of 1 th water drained by waste pipes in 1 hour) = of 6 the tank gets drained in an hour.



1 6x − 64 + 8x 1 x 8−x − =− ⇒ =− 8 6 6 48 6 ⇒ 14x − 64 = −8 ⇒ x = 4

PRACTICE EXERCISE 1. A can do a work in 10 days and B in 15 days. If they work on it together for 5 days, then what fraction of the work is left? (a) 2/3 (c) 1/2

(b) 5/6 (d) 1/6

2. X can lay a railway track between two given stations in 15 days and Y can do the same job in 10 days. With the help of Z, they did the job in 5 days only. How long does it take for Z to complete the job alone? (a) 1/6 (c) 5/6

(b) 1/30 (d) 9/10

3. A, B and C can do a piece of work in 20, 30 and 60 days, respectively. In how many days can A do the work if he is assisted by B and C on every third day? (a) 12 days (c) 18 days

(b) 15 days (d) 16 days

4. If 6 men and 8 boys can do a piece of work in 10 days while 26 men and 48 boys can do the same in 2 days,

Chapter 01-5.indd 601

then what is the time taken by 15 men and 20 boys in doing the same type of work? (a) 2 days (c) 4 days

(b) 5 days (d) 6 days

5. Peter does 75% of work in 12 days. He then calls Charlie for help and they both complete the rest of the work in 3 days. How many days would Charlie have taken to complete the work alone? (a) 18 (c) 72

(b) 24 (d) 48

6. If A is twice as good as workman as B and therefore is able to finish a job in 40 days less than B, how many days will it take to finish the same job if A and B work together? 1 2 2 (c) 26 3 (a) 28

(b) 40 (d) 22

22-08-2017 06:58:07

602     Time and Work 

7. Pipe A can fill a cistern in 36 min and pipe B in 48 min. If both the pipes are opened together, when should pipe B be closed so that the cistern may be full in 24 min? (a) 16 min (c) 12 min

(b) 10 min (d) 15 min

8. Pipes A and B can fill a tank in 4 and 8 h, respectively. Pipe C can empty it in 16 h. If all the three pipes are opened together, then how long will it take to fill the tank? (a) 4.5 h (c) 6 h

(b) 2 h (d) 3.2 h

9. Two pipes A and B can fill a tank in 4 h. If only pipe A is open, then it would take 4 h longer to fill the tank. How much longer would it take if only pipe B is open? (a) 7 h (c) 8 h

(b) 4 h (d) 5 h

10. A pump can fill a tank with water in 1.5 h. However, due to a leak it took 2.5 h to fill the tank. How long will the leak take to drain all the water in the tank? (a) 15/4 h (c) 15/2 h

(b) 14/9 h (d) 12/5 h

11. There are two inlet pipes and an outlet pipe. The efficiency of one of the inlet pipes is double than that of the other inlet pipe. The efficiency of the outlet pipe is half that of the lesser-efficient inlet pipe. The empty tank gets filled in 12 h when all the pipes are opened. How many hours will it take to fill the empty tank when the lesser-efficient inlet pipe is plugged and the rest kept opened?

B can fill the remaining part in 10 h. What is the number of hours taken by C alone to fill the tank? (a) 16/3 h (c) 15 h

15. A large tank can be filled using two pipes A and B in 2 and 4 h, respectively. How long will it take to fill the entire tank if both the pipes are used to fill half of the tank alone? (a) 2 h (c) 3.5 h

(b) 25 h (d) 12 h

12. Two pipes A and B can fill a tank in 12 and 16 min, respectively. Both the pipes are opened together, but after 4 min, pipe A is turned off. What is the total time required to fill the tank? (a) 8 min (c) 10 min 40 s

(b) 6 min 40 s (d) 12 min 30 s

13. Three taps A, B and C can fill a tank in 12, 15 and 20 h, respectively. If A is open all the time and B and C are open for 1 h each alternatively, how long will it take to fill the tank? (a) 8.5 h (c) 7 h

(b) 10 h (d) 5 h

14. Three pipes A, B and C can fill a tank in 8 h. After working together for 2 h, C is closed and A and

Chapter 01-5.indd 602

(b) 5 h (d) 3 h

16. Pipes A, B and C are attached to a tank and each can act as either an inlet or an outlet pipe. They take 6, 12 and 18 h to fill an empty tank or empty the full tank. In the first hour, pipes A and B work as inlet and C works as outlet. In the second hour, pipes B and C work as inlet and pipe A as outlet. In the third hour, A and C work as inlet and pipe B as outlet. What part of the tank will be filled in 3 h? (a) 11/18 (c) 17/20

(b) 5/28 (d) 5/9

17. If Karan can do 1/4 of total work in 3 days and Arjun can do 1/6 of the same work in 4 days, how much will Karan get if both work together and are paid H 300 in total? (a) H 150 (c) H 100

(b) H 200 (d) H 225

18. If A and B can complete a work in 8 days, B and C in 12 days, C and A in 24 days, then how long will it take for A, B and C to complete the work if working together? (a) 8 days

(a) 20 h (c) 15 h

(b) 20 h (d) 11 h

(c) 9 days

(b) 6 days 2 (d) 7 days 3

19. A sum of money is sufficient to pay A’s wages for 20 days and B’s wages for 30 days. For how many days is the same amount of money sufficient to pay both A and B together? (a) 15 2 (c) 21 3

(b) 12 (d) 14

20. Worker A alone can do a piece of work in 6 days and B alone in 8 days. A and B undertook to do it for H 4000. With the help of worker C, they completed the work in 3 days. How much money will be given to C ? (a) H 600 (c) H 400

(b) H 350 (d) H 500

22-08-2017 06:58:07

PRACTICE EXERCISE     603

Directions for Q21—Q24: Each of these questions is followed by multiple statements. You have to study the question and all the statements given to decide whether any information provided in the statement(s) is redundant and can be dispensed with while answering the given question. 21. In how many days can 10 women finish a job? I. 10 men can complete the work in 6 days. II. 10 men and 10 women together can complete 24 the work in days 7 III. If 10 men work for 3 days and thereafter 10 women replace them, the remaining work is completed in 4 days. (a) Any two of the three (b) I and II only (c) II and III only (d) I and III only 22. How many workers are required for completing the construction work in 10 days? I. 20% of the work can be completed by 8 workers in 8 days. II. 20 workers can complete the work in 16 days. III. O  ne-eighth of the work can be completed by 8 workers in 5 days. (a) I only (b) II and III only (c) I and III only (d) I or II or III 23. A and B together can complete a task in 7 days. B alone can do it in 20 days. What part of the work was carried out by A? I. A completed the job alone after A and B worked together for 5 days. II. Part of the work done by A could have been done by B and C together in 6 days. (a) I alone is sufficient while II alone is insufficient. (b) II alone is sufficient while I alone is insufficient. (c) Both I and II alone are sufficient. (d) Both I and II are necessary to answer. 24. How long will machine Y, working alone, take to produce x candles? I. Machine X produces x candles in 5 minutes. II. Machine X and Machine Y working at the same time produce x candles in 2 minutes. (a) I alone is sufficient while II alone is insufficient. (b) II alone is sufficient while I alone is insufficient. (c) Both I and II alone are sufficient. (d) Both I and II are necessary to answer. 25. A and B can do a work in 8 days, B and C can do the same work in 12 days. A, B and C together can finish it in 6 days. How long will it take for A and C to do it together?

Chapter 01-5.indd 603

(a) 4 days (c) 8 days

(b) 6 days (d) 12 days

26. A and B together can do a piece of work in 30 days. A having worked for 16 days, B finishes the remaining work alone in 44 days. In how many days shall B finish the whole work alone? (a) 30 days (c) 60 days

(b) 40 days (d) 70 days

3 4 times as efficient as B. How long does it take for A to do it alone? 1 (a) 9 days (b) 11 days 3 15 1 (c) days (d) 17 days 2 3

27. A and B can do a job together in 7 days. A is 1

28. A can finish a work in 24 days, B in 9 days and C in 12 days. B and C start the work but are forced to leave after 3 days. How long did it take for A to complete the remaining work? (a) 10 days

(b) 8 days 21 (d) days 2

(c) 4 days

29. X can do a piece of work in 40 days. He works at it for 8 days and then Y finished it in 16 days. How long will they together take to complete the work? 1 (a) 13 days (b) 15 days 3 (c) 20 days (d) 26 days 30. Three pipes A, B and C are connected to a tank. These pipes can fill the tank separately in 5 hr, 10 hr and 15 hr respectively. When all the three pipes were opened simultaneously, it was observed 3 th that pipes A and B were supplying water at 4 of their normal rates for the 1st hour after which they supplied water at normal rate. Pipe C sup2 rd plied water at of its normal rate for first 3 2 hour, after which it supplied at its normal rate. In how much time the tank would be filled?





(a) 1.05 hours (c) 3.05 hours

(b) 2.05 hours (d) 4.05 hours

31. There are 12 pipes that are connected to a tank. Some of them are fill pipes and the others are drain pipes. Each of the fill pipes can fill the tank in 8 hours and each of the drain pipes can drain the tank completely in 6 hours. If all the fill pipes and drain pipes are kept open, an empty tank gets

22-08-2017 06:58:08

604     Time and Work 

filled in 24 hours. How many of the 12 pipes are fill pipes? (a) 8 (c) 7

(b) 6 (d) 5

32. A cylindrical overhead tank is filled by two pumps P1 and P2. P1 can fill the tank in 8 hr while P2 can fill the tank in 12 hr. There is a pipe P3 which can empty the tank in 8 hr. Both the pumps are opened simultaneously. The supervisor of the tank, before going out on a work, sets a timer to open P3 when the tank is half filled so that the tank is exactly filled up by the time he is back. Due to technical fault P3 opens when the tank is one-third filled. If the supervisor comes back as per the plan what percent of the tank is still empty? (a) 17.5% (c) 12%

(b) 15% (d) 10%

1 min2 utes and 45 minutes respectively. Both pipes are opened. How long is B turned after if the cistern will be filled in just half an hour?

33. Two pipes A and B can fill a cistern in 37

(a) 5 min (c) 10 min

(b) 9 min (d) 15 min

34. Two pipes can fill a tank in 20 and 24 minutes respectively and a waste pipe can empty 3 gallons per minute. All the three pipes working together can fill the tank in 15 minutes. What is the capacity of the tank? (a) 60 gallons (c) 120 gallons

(b) 100 gallons (d) 180 gallons

35. A tank is filled in 5 hours by three pipes A, B and C. The pipe C is twice as fast as B and B is twice as fast as A. How much time will pipe A alone take to fill the tank?

(a) 20 hours (c) 35 hours

(b) 25 hours (d) Indeterminate

36. Three pipes A, B and C can fill a tank in 6 hours. After working at it together for 2 hours, C is closed and A and B can fill the remaining part in 7 hours. How long does it take for C alone to fill the tank? (a) 8 (c) 12

(b) 10 (d) 14

37. A can finish a work in 18 days and B can do the same work in 15 days. B worked for 10 days and left the job. In how many days, A alone can finish the remaining work? (a) 6 (c) 5

(b) 5.5 (d) 4

38. A and B can together finish a work 30 days. They worked together for 20 days and then B left. After another 20 days, A finished the remaining work. In how many days A alone can finish the work? (a) 50 (c) 65

(b) 60 (d) 70

39. P can complete a work in 12 days working 8 hours a day. Q can complete the same work in 8 days working 10 hours a day. If both P and Q work together, working 8 hours a day, in how many days can they complete the work? 5 6 (a) 5 (b) 5 11 11 5 6 (c) 6 (d) 6 11 11 40. A is 30% more efficient than B. How much time will they, working together, take to complete a job which A alone could have done in 23 days? (a) 8 days (c) 12 days

(b) 11 days (d) 13 days

ANSWERS 1. (d)

6. (c)

11. (a)

16. (a)

21. (a)

26. (c)

31. (c)

36. (d)

2. (b)

7. (a)

12. (c)

17. (b)

22. (d)

27. (b)

32. (d)

37. (a)

3. (b)

8. (d)

13. (c)

18. (a)

23. (a)

28. (a)

33. (b)

38. (b)

4. (c)

9. (b)

14. (b)

19. (b)

24. (d)

29. (a)

34. (c)

39. (a)

5. (d)

10. (a)

15. (d)

20. (d)

25. (c)

30. (c)

35. (c)

40. (d)

Chapter 01-5.indd 604

22-08-2017 06:58:09

EXPLANATIONS AND HINTS     605

EXPLANATIONS AND HINTS 1. (d) We know that A’s 1 day’s work = And B’s 1 day’s work =

1 10

5. (d) Given that Peter does 75% of work in 12 days.

1 15

10 + 15  = 6 1

So (A + B)’s 1 day’s work =

So the total work will be completed by Peter in 4 12 × = 16 days 3

1

1

Therefore, (A + B)’s 5 days’ work = 5 ×



Thus, the remaining work = 1 −





3 amount of work is done by Peter and 4 Charlie in 3 days. Now, 1 −

1 5 = 6 6



5 1 = 6 6

Total work would be completed by Peter and Charlie together in 3 × 4 = 12 days 1 Peter and Charlie’s one day’s work = 12 1 1 1 − = Charlie’s 1 day’s work = 12 16 48

2. (b) We know that (X + Y + Z)’s 1 day’s work 1 = 5 1 X’s 1 day’s work = 15 Charlie alone would do the work in 48 days. 1 Y’s 1 day’s work = 10 6. (c) Given that the ratio of times taken by A and 1 1 1 1 1 B = 1:2 Therefore, Z ’s 1 day’s work = − + = − 5 15 10 5 6 1 1 1 1 1 1 If A takes 1 day to finish work, then B takes 2 days = − + = − = to finish the same work. 5 6 5 15 10 30













So Z will take 30 days to complete the job.

 1 1 1 6 1 (A + B + C)’s 1 day’s work =  + + = = 20 30 60 60 10 6 1

3. (b) We know that A’s 2 days’ work =







So the difference in time taken by A and B = 1 day

1 1 ×2 = 20 10

1 1 1 = + + = = 20 30 60 60 10

10 + 10  = 5 1

Work done in 3 days = Thus,

1

1

1 work is done in 3 days. 5

Therefore, whole work will be done in 3 × 5 days = 15 days 4. (c) Let 1 man’s 1 day’s work be x and 1 boy’s 1 day’s work be y. Then, 1 6x + 8y =  (1) 10 1 26x + 48y =  (2) 2 Solving the two equations, we get

If difference is 1 day then B takes 2 days. Thus, if difference is 40 days then B will take 2 × 40 = 80 days So, A takes 40 days to do the work. 1 A’s 1 day’s work = 40 1 B’s 1 day’s work = 80 1 1 3 (A + B)’s 1 day’s work = + = 40 80 80 Thus, A and B together complete the work in 80 2 = 26 3 3 7. (a) Let pipe B be turned off after x min.

 36 + 48  = 144 1

1

7x

So the part filled in x min = x Part filled in 24 − x min =

24 − x 36

Hence, x=

1 1 ;y= 100 200

100 + 200  = 4 15

(15 men + 20 boys) 1 day’s work =

20

1

Thus, 15 men and 20 boys can do the work in 4 days.

Chapter 01-5.indd 605

7x 24 − x + =1 144 36 ⇒

7 x + 96 − 4x =1 144 x = 16 min

22-08-2017 06:58:13

606     Time and Work 



1 1 8. (d) We know that the part filled in 1 h = + 4 8 3 = 8 1 Total part emptied in 1 h = 16 3 1 5 So the part filled in 1 h = − = 8 16 16 16 Therefore, tank will be full in h = 3.2 h 5







So the total time taken to fill remaining part 5 20 = = × 16 = 12 3 20 The tank will be full in 4 + min = 10 min 40 s 3





13. (c) We know that (A + B)’s 1 h work 1 1 9 3 = + = = = 12 15 60 20





12 + 20  = 60 = 15 1

9. (b) We know that part filled if both A and B are 1 open = 4 1 Part filled if only A is open = 8 1 1 1 Part filled if only B is open = − = 4 8 8 Thus, B alone would take 8 h to fill the tank. Hence, B alone will take (8 − 4) = 4 h longer to fill. 10. (a) Work done by the leak in 1 h =





2 2 4 − = 3 5 15

15 Thus, leak alone will empty tank in h. 4 11. (a) Let the number of hours taken by the outlet and the two inlet pipes be 4x, 2x and x respectively. So, the part of tank filled in 1 h will be

And (A + C )’s 1 h work =

8

2

 20 + 15  = 60 3

So the part filled in 2 h =

1

2

17

And the part filled in 6 h =

17 17 ×3 = 60 20

So the remaining part = 1 −

17 3 = 20 20

The remaining part is filled by (A + B) in 1 h. Thus, the total time taken to fill the tank =6+1=7h 14. (b) We know that (A + B + C )’s 1 h work = Total part filled in 2 h = Remaining part = 1 −

1 8

1 4

1 3 = 4 4

Now, C is closed. 1 1 1 1 + − = x 2x 4x 12 ⇒

5 1 = 4x 12

Thus, (A + B)’s 10 h work = (A + B)’s 1 h work =

3 40

⇒ x = 15 Hence, C ’s 1 h work = Hence, inlet pipe with higher efficiency fills tank in 15 h.

3 4

1 3 2 1 − = = 8 40 40 20

C alone can fill the tank in 20 h. Now, second inlet pipe is plugged. 1 1 3 Thus, the desired pipes fill − = part of x 4x 4x the tank 4x 4 × 15 Time taken = = = 20 h 3 3 12. (c) We know that the part filled in 4 min 1 1 7 =4 + = = 12 16 12 5 So the remaining part = 12 1 Part filled by B in 1 min = 16



Chapter 01-5.indd 606



15. (d) We know that A’s 1 h work = And B’s 1 h work =

1 2

1 4

1 of total tank in 1 h. 2 1 And B will fill of total tank in 1 × 2 = 2 h. 2 Thus, total time taken to fill the tank = 2 + 1 h =3h Now, A will fill

16. (a) In the cycle of 3 h, pipes A, B and C are working as inlet pipes for 2 h each and they work as outlet pipes for an hour each.

22-08-2017 06:58:17

EXPLANATIONS AND HINTS     607

So, part of tank filled in 3 h = 2 ×



 





 6 + 12 + 18 −  6 + 12 + 18⇒ 1 man’s 1 day’s work = 60 1

1

1

1

 

1

1

1

According to statement II,



1 1 1 6 +3 +2 6 +3 +2 1 1 1 11 = 2× + + − + + = 2× − = 6 12 18 18 18 6 12 18 18

10 × 7  men + 10 × 7  women can complete 24

17. (b) We know that whole work is done by Karan in = 3 × 4 = 12 days

24

the work in 1 day.

 7  men’s 1 day work +  7  women’s 1 day 240

Whole work is done by Arjun in = 4 × 6 = 24 days



Karan’s wages : Arjun’s wages = Karan’s 1 day’s work : Arjun’s 1 day’s work

work = 1 ⇒

1 1 Thus, : = 2 :1 12 24 2 So Karan’s share = × 300 = H 200 3

240

 7  × 60 men’s 1 day work +  7  women’s 240

1

240

1 day work = 1 ⇒

1 18. (a) We know that (A + B)’s 1 day’s work = 8 1 And (B + C )’s 1 day’s work = 12 1 And (C + A)’s 1 day’s work = 24

 7  women’s 1 day work = 1 − 7  = 7 240

⇒ 10 women’s 1 day’s work =

4

3

7 × 240 ×10 = 8 3

7

1

So, 10 women can finish the work in 8 days. 1 1 woman’s one day work = 80 1 1 1 1 1 6 1 + + = =According to statement III, So (A + B + C )’s 1 day’s work = 2 8 12 24 2 24 8 (10 men’s work for 3 days) + (10 women’s work for 1 1 1 1 1 6 1 = + + = = 4 days) = 1 2 8 12 24 2 24 8 ⇒ 30 men’s 1 day’s work + 40 women’s 1 day’s Thus, A, B and C together can complete work in work = 1 8 days. Hence, I and II, or I and III or II and III give us the answer. 19. (b) Let the total money be H x. x So A’s 1 day’s wages = H 22. (d) According to statement I, 20 x 1/5 of the work can be completed by (8 × 8) workAnd B’s 1 day’s wages = H ers in 1 day. 30 xx xx xx Then, (8 × 8 × 5) workers can complete work in ++ == Thus, (A + B)’s 1 day’s wages = H H 2020 3030 1212 1 day.









 

 





8×8×5 workers can complete work in 10 days = 10 1 1 1 1 7 32 1workers in 10 days. = − = 20. (d) We know that C ’s 1 day’s work = − + 3 6 8 3 24 According 24 to statement II, 1 7 1 1 1 1 = − + = − = 20 workers can complete work in 16 days. 3 24 24 3 6 8 1 1 1 (20 × 16) workers can complete work in 1 day. A’s wages :B’s wages :C ’s wages= : : = 4 :3 :1 6 8 24 20 × 16 Then, workers can complete work in 10 1 10 Thus, C ’s share = × 4000 = H 500 8 days = 32 workers in 10 days. So the money is sufficient to pay the wages of both for 12 days.











21. (a) According to statement I, (10 × 6) men can complete the work in 1 day.

Chapter 01-5.indd 607



According to statement III, 8 workers can complete 1/8 work in 5 days.

22-08-2017 06:58:21

608     Time and Work 

8 workers can complete the entire work in 40 days. (8 × 4) workers can complete the entire work in 10 days = 32 workers in 10 days.

16x + 44y = 1  Eq.(2) − 16 × Eq. (i),

Thus, any one of the three statements gives us the answer. 1 20 1 (A + B)’s one day work = 7 According to statement I,

y=

B’s 1 day’s work =

23. (a) B’s one day work =

(ii)



1 60

1 60

Hence, B alone can finish the whole work in 60 days. 27. (b) (A’s 1 day’s work) : (B’s 1 day’s work) =

(A + B)’s 5 day’s work =



Work remaining = 1 −

5 7

7 :1= 7 : 4 4



5 2 = 7 7

Let A’s and B’s 1 day’s work be 7x and 4x respectively.

2 work was carried out by A. 7 Statement II is irrelevant. Hence,

Then, 7 x + 4x =

24. (d) According to statement I, x Machine X produces candles in 1 min. 5 According to statement II, x Machine X and Y produce candles in 1 min. 2 x x 3x − = From statement I and II, Y produces 2 5 10 candles in 1 min.





 3x × x 10

Hence, x candles will be produced by Y in

10 min 3 Thus, statements I and II are both necessary to get the answer. min =

25. (c) (A + B + C)’s 1 day’s work = (A + B)’s 1 day’s work =

1 1 ⇒x= 7 77

77 × 7 = 11 1

Thus, A’s 1 day’s work =

1

A alone can complete the work in 11 days.

 9 + 12  = 36 1

28. (a) (B + C)’s 1 day’s work =

1

7 7 ×3 = 36 12

Work done by B and C in 3 days = Work remaining =

5 12

A completes the entire work in 24 days. Hence, A completes





5 5 work in × 24 = 10 days. 12 12

 40 × 8 = 5 1

1 6

29. (a) Work done by X in 8 days =

1 8

Work remaining = In 16 days, Y does

1 (B + C)’s 1 day’s work = 12

7

1

4 5

4 work. Hence, Y’s 1 day’s work = 5

4 1 1 × = 1 5 5 316 120 1 1 1 = − = = Thus, (A + C)’s 1 day’s work = 2 × − + 1 1 3 3 24 24 8 6 8 12 (X + Y)’s 1 day’s work = + = 1 5 3 1 1 1 1 40 20 20 2× − + = − = = 3 24 24 8 6 8 12 Total time taken to complete work by X and Y = Hence, A and C will together to the work in 8 days. 20 1 = 13 days 3 3 26. (c) Let A’s 1 day’s work = x and B’s 1 day’s work = y



 

 





 

 

Then, according to the given data x+y =

Chapter 01-5.indd 608

1  30

(i)







30. (c) The part of the tank filled by A and B in first 3 1 1 1 1 21 two hours = × + + + = 4 5 10 5 10 40



 



22-08-2017 06:58:25

EXPLANATIONS AND HINTS     609



The part of tank filled by C in first two hours =



1 1 33 = = 10% Part of the tank which is empty = 1 − + 10 3 60 1 1 33 1− + = = 10% 10 3 60

2 1 4 2× × = 3 15 45



 

139 Remaining part = 360



33. (b) Let B be turned off after x minutes. Then,

11 In 1 hour, all the three pipes together will fill = 30 Hence, the time taken to fill the remaining tank = 139 30 = × = 1.053 hour 360 11 Thus, the total time taken to fill the remaining tank = 3.05 hours.

Part filled by (A + B) in x min + Part filled by A in (30 − x) min = 1 2 1 2 ∴ x  +  + (30 − x) × =1  75 45  75 11x (60 − 2x) + =1 225 75 ⇒ 11x + 180 − 6x = 225 ⇒ x = 9 ⇒

31. (c) Let there be `n’ fill pipes attached to the tank. 34. (c) Work done by the waste pipe in 1 minute = Therefore, there will be (12 − n ) drain pipes attached to the tank. Each pipe fills the tank in



1 8 hours. Thus, each of the fill pipes will fill 8 the tank in an hour. Hence, n fill pipes will fill hour.



 

 

1 1 1 1 11 1 − + = − = − 15 20 24 15 120 40



[−ve sign

th

of

means emptying] Volume of



n of the tank in an 8

1 part = 3 gallons. 40

Volume of whole = (3 × 40) gallons = 120 gallons.

Each drain pipe will drain the tank in 6 hours.



1 Therefore, each of the drain pipes will drain 6 of the tank in an hour. Hence, (12 − n ) drain pipes will drain the tank in an hour.



th

12 − n of 6



When all these 12 pipes are kept open, it takes 24 hours for an empty tank to overflow. Therefore,

 24  1

in an hour

of the tank gets filled. Hence, 36. (d) Total part filled in 2 hours =

24 32. (d) P1 and P2 can fill the tank in 5 12 It takes hrs to fulfil half the tank. For remain5 ing half of the tank P3 will open and this will take

 5 + 6 hrs. 12

6 hours. Supervisor has gone out for

3 1

rd

tank will fill in

 5  hr only  60  42

Chapter 01-5.indd 609

33

1 2 4 1 + + = x x x 5 7 1 ⇒ = ⇒ x = 35 x 5 ∴

Thus, pipe A takes 35 hours to fill the tank.

th

n 12 − n 1 − = 8 6 24 3n − 4 (12 − n ) 1 ⇒ = ⇒ 7n − 48 = 1 24 24 ⇒n=7

Now,

35. (c) Let pipe A take x hours to fill the tank. x x Then, pipes B and C will take and hours, 2 4 respectively, to fill the tank.

 8 5



Remaining part = 1 −



2 1 = 3 3

(A + B)’s 7 hour’s work =

2 3

(A + B)’s 1 hour’s work =

2 2 = 3×7 21

C’s 1 hour’s work = (A + B + C)’s 1 hour’s work −

 6 − 21 = 14 1

(A + B)’s 1 hour’s work =

th

part of the tank will fill.

2

1

Hence, C alone can fill the tank in 14 hours.

15 ×10 = 3 1

hr. In remaining

2 1 = 6 3

37. (a) B’s 10 day’s work =



Work remaining = 1 −

2



2 1 = 3 3

22-08-2017 06:58:29

610     Time and Work 

Now,

Therefore, ∴

So, both P and Q will finish the work in hours.









1 1 work is done by A in 18 × = 6 days 3 3

38. (b) (A + B)’s 20 day’s work =



1 2 × 20 = 30 3



∴ Number of days of 8 hours each = 480 1 60 5 × = = 5 days 11 8 11 11



2 1 Remaining work = 1 − = 3 3 1 Now, work is done by A in 20 days. Therefore, 3 the whole work will be done by A in (20 × 3) = 60 days.



Then, 10 : 13 :: 23 : x ⇒ x =

39. (a) P can complete the work in (12 × 8) hours = 96 hours Q can complete the work in (8 × 10) hours = 80 hours 1 1 P’s 1 hour’s work = and Q’s 1 hour’s work = 96 80 1 1 11 Hence, (P + Q)’s 1 hour’s work = + = 96 80 480

B’s 1 day’s work =

10 299

Chapter 01-5.indd 610

480

1

60

5

Suppose B takes x days to do the work.

1 23



 11 × 8  = 11 = 5 11

40. (d) Ratio of times taken by A and B = 100 : 130 = 10 : 13.

A’s 1 day’s work =



 11  480

1 work is done by A in 1 day. 18

23 × 13 299 = 10 10



 23 + 299  = 299 = 13 1

(A + B)’s 1 day’s work =



10

23

1

Thus, A and B together can complete the work together in 13 days.

22-08-2017 06:58:31

CHAPTER 6

AVERAGE, MIXTURE AND ALLIGATION

AVERAGE The term average has a broad meaning and can be used in many ways. Average of a set of values is defined as the sum of all the values divided by the number of values. It is an effective way to represent a group of values by a single value. Mathematically, Average =

Sum of all values Total number of values

Some important points when working with problems concerning average are as follows: 1. If each value in the group is increased/decreased by a constant k, then the average of the group also increases/decreases by the same constant k. 2. If each value in the group is multiplied/divided by a constant k, then the average of the group is also multiplied/divided by the same constant k. 3. The average value can never be smaller than the smallest value of the group and can never be larger than the largest value of the group. Hence, the average value always lies between the smallest and the largest value of the group.

Chapter 01-6.indd 611

Weighted Average In some problems, individual frequencies of each value in the group are given to us. In that case, the formula given before does not apply. This average is known as weighted average. Mathematically, Average =

n1x1 + n2 x2 + n3 x3 +  + nk xk n1 + n2 + n3 +  + nk

where x1, x2, x3, …, xk are the corresponding values in the list and n1, n2, n3, …, nk are the frequencies of the corresponding values in the list.

MIXTURE AND ALLIGATION When two different substances are mixed together, it is called simple mixture. When two or more simple mixtures are mixed together to form another mixture, it is called compound mixture. According to the alligation rule, if two solutions are mixed in the ratio x:y and the concentration of the two solutions is cx and cy, respectively, then the ratio of

22-08-2017 06:59:09

612     Average, Mixture and Alligation 

their quantities is inversely proportional to the ­difference in their concentration from the mean concentration. Mathematically,

CP of cheaper (X)

cy − c x = y c − cx

CP of costlier (Y)

Mean price (M)

where x ⋅ cx + y ⋅ cy c= x+y

X-M

Figure 1 represents the pictorial representation of the concept of alligation for better understanding.

M-Y

Figure 1 |   Flowchart of solution using alligation rule.

SOLVED EXAMPLES 1. Marks of a student in six subjects are 97, 92, 85, 80, 92 and 79. What are the average marks obtained by the student? Solution:  Given that the total number of subjects = 6 Total marks in six subjects = 97 + 92 + 85 + 80 + 92 + 79 = 525 Average marks of the student in six subjects = 525 = 87.5 6 2. Two kilograms of wheat costing H 20 per kilogram and 6 kg of wheat costing H 25 per kilogram are mixed together. What is the net cost of the mixture?

Now, cost price of the mixture is only due to the cost of milk. We know that the cost price of milk = H 9 per litre 1000 Thus, for H 8, the quantity of milk = ×8 = 9 888.89 mL Total water in the mixture = 1000 − 888.89 = 111.11 mL 4. The average marks of a class of 30 students are 82. If one student’s marks from another section are added, then the average becomes 81. What are the marks of the student from another section?

Solution:  Let the marks of the added student be x. Then Solution:  Net cost of the mixture is 30 × 82 + x = 81 1× 20 + 3× 24 1× 20 + 3× 24 2 and 31 (because 2 and 6 are (because in the ratio 1:3)6 are in the ratio 1:3) 1+ 3 1+ 3 ⇒ 2460 + x = 2511 20 + 72 20 + 72 = = ⇒ x = 2511 − 2460 = 51 4 4 92 92 Hence, the marks of the added student are 51. = = 4 4 5. In what ratio must Tim mix two varieties of sugar = 23 = 23 worth H 20 per kilogram and H 32 per kilogram so Hence, net cost = H 23 per kilogram that by selling the mixture at H 36 per kilogram he may gain 20%? 3. A man buys milk at H 9 per litre and after adding water, sells it at H 10 per litre, thereby making a Solution:  It is given that the selling price of the profit of 25%. How much water is in the mixture? mixture = H 36 per kilogram Gain = 20% Solution:  Given that the selling price of the mixture = H 10 per litre 100 Cost price of the mixture = × 10 = H 8 per litre 125

Chapter 01-6.indd 612

100 × 36 = H 30 120 If the amount of sugar worth H 20 be x So the cost price of the mixture =

22-08-2017 06:59:11

SOLVED EXAMPLES     613

Thus, the amount of sugar worth H 32 be (1000 − x), thus 20 × x 32 (1000 − x) + = 30 1000 1000 ⇒ 20x + 32000 − 32x = 30000 ⇒ 12x = 2000 500 ⇒x= 3 Quantities of both the sugar = 500/3 g and 2500/3 g, so the ratio will be Ratio =

500 / 3 1 = =1:5 2500 / 3 5

6. In a mixture of 30 litre, the ratio of milk and water is 3:2. If this ratio is to be 1:2, then what is the quantity of water to be further added? Solution:  Given that the total quantity of the mixture = 30 litre Ratio of milk and water = 3:2 Total quantity of milk will be 3×

30 = 18 liter 3+2

Total quantity of water will be 2×

30 = 12 liter 3+2

8. The speed of a train from A to B was 40 km/h while when coming back from B to A, its speed was 60 km/h. What is the average speed during the whole journey? Solution:  Let the distance between A and B be x x So the time taken from A to B = 40 x And the time taken from B to A = 60 Total distance covered = 2x Now the average speed will be 2x (x / 40) + (x / 60) 2x × 40 × 60 = 40x + 60x 4800x = = 48 100x Hence, the average speed over the journey = 48 km/h 9. Average of five numbers is 40. The last number is replaced and the average becomes 39. If the last number before being replaced was 42, then what is the new last number? Solution:  Let the sum of four numbers be x and the replaced number be y. Now, initially the average of the five numbers is given by x + 42 = 40 5 ⇒ x = 200 − 42 = 158

Now, if x litre of water is added. New quantity of mixture = 30 + x Milk = 18 litre

Hence, the sum of the first four numbers = 158

Now, quantity of water = 12 + x, now

Now, using the above value, after replacing the number 42 with y, we get

18 1 = 12 + x 2 ⇒ 36 = 12 + x ⇒ x = 24 Thus, the quantity of water to be added = 24 litre 7. The average age of three people is 36. If their ages are in the ratio 3  :  2  :  4, what are the ages of the three persons? Solution:  Let the ages be 3x, 2x and 4x. Then, 3x + 2x + 4x = 36 3 ⇒ 9x = 108 ⇒ x = 12

158 + y = 39 5 ⇒ y = 195 − 158 = 37 The last number after replacing is 37. 10. In a class, 5 students got 19 marks, 4 students got 17, 7 students got 15, 3 students got 14 and 1 student got 20. What are the average marks of the class? Solution:  We know that total students in the class = 5 + 4 + 7 + 3 + 1 = 20 Average marks of all the students in the class will be 5 × 19 + 4 × 17 + 7 × 15 + 3 × 14 + 1 × 20 20 =

Hence, the three ages are 39, 24 and 48.

Chapter 01-6.indd 613

95 + 68 + 105 + 42 + 20 330 = = 16.5 20 20

22-08-2017 06:59:14

614     Average, Mixture and Alligation 

11. A vessel is filled with liquid, 3 parts of which are water and 5 parts syrup. How much of the mixture must be drawn off and replaced with water so that the mixture may be half water and half syrup? Solution:  Suppose the vessel initially contains 8 litres of liquid. Let x litres of this liquid be replaced with water. Quantity of water in new mixture = 3 − 3x + x 8 litres Quantity of syrup in new mixture = 5 − 5x litres 8   Since the quantities of water and syrup is same,









Let x be the quantity of pulses costing H 15 per kg and y be the quantity of pulses costing H 20 per kg Thus, putting the values given to us, x 20 − 16.5 3.5 7 = = = y 16.5 − 15 1.5 3 14. A servant stole juice from the jar which was 40% concentration and replaced it with juice with concentration 16%. The jar only had juice of concentration 24%. How much of the juice did the servant steal? Solution:  Using alligation rule,

3 − 38x + x = 5 − 58x

cy − c x 40 − 24 16 2 = = = = y c − cx 24 − 16 8 1

⇒ 5x + 24 = 40 − 5x ⇒ 10x = 16 ⇒ x =

8 5





Hence, part of the mixture replaced = 8 × 1 = 1 5 8 5 12. A 20 litre mixture of milk and water contains milk and water in the ratio 3: 2. 10 litres of the mixture is removed and replaced with pure milk and the operation is repeated once more. At the end of the two removals and replacement, what is the ratio of milk and water in the resultant mixture? Solution:  Ratio of milk and water in the mixture initially = 3:2 Say, quantity of milk is 3x and quantity of water is 2x. Hence, 3x + 2x = 20 ⇒ x = 4 Thus, in the beginning we have 12 litres of milk and 8 litres of water. After the first removal, we have 10 litres of solution containing 6 litres of milk and 4 litres of water. We then add 10 litres of pure milk in the solution. The new ratio of milk and water in the solution is 16:4 or 4:1. After the second removal, we have 10 litres of solution containing 8 litres of milk and 2 litres of water. We then add 10 litres of pure milk in the solution again. The new ratio of milk and water in the solution is 18:2 or 9:1. 13. In what ratio must a grocer mix two varieties of pulses costing H 15 and H 20 per kg respectively so as to get a mixture worth H 16.50 kg? Solution:  According to alligation rule, cy − c x = y c − cx

Chapter 01-6.indd 614

Where x is the quantity of juice with 16% concentration and y is the quantity of juice with 40% concentration. Thus, 1/3 of the juice was left. Hence, the servant took 2/3 of the original juice. 15. In what ratio must water be mixed with milk to 2 gain 16 % on selling the mixture at cost price? 3 Solution:  Let C.P. of 1 litre milk be H x. S.P. of 1 litre of mixture = H x 2 Gain = 16 % 3 Thus, C.P. of 1 litre of mixture = 100 × 3 x = 6 x 350 7 Using alligation rule,



6 x− x x 7 = 1 : 6 = 1: 6 = 6 7 7 y x−0 7



[C.P. of water = 0]

Hence, ratio of water and milk = 1:6. 16. The average weight of 16 boys in a class is 50.25 kg and that of the remaining 8 boys is 45.15 kg. Find the average weights of all the boys in the class. Solution:  Using the formula of weighted average, we can get the required average =

50.25 ×1616 ++ 458 .15 × 8 = 804 +24361.2 .2 = 48.55 1165 24  Thus the average weight of all the boys in the class is 48.55 kg.

22-08-2017 06:59:16

PRACTICE EXERCISE     615

17. A student’s marks were wrongly entered as 83 instead of 63. Due to that the average marks for the class got increased by half. How many students are there in the class?

Solution:  Total quantity of petrol consumed in 4000 4000 4000 76700 3 years =  litres + + = 7. 5 8 8. 5 51



 



Total amount spent = 3 × 4000 = H 12000 Solution:  Let the total number of students in the class be x. Total increase in marks = x 2 Thus, x x = (83 − 63) ⇒ = 20 ⇒ x = 40 2 2 18. The captain of a football team of 11 members is 26 years old and the wicket keeper is 3 years older. If the ages of these two are excluded, the average age of the remaining players is one year less than the average age of the whole team. What is the average age of the team?

Thus, average cost spent on petrol over the three 12000 6120 years = = H 7.98 per litre = 76700/51 767



20. The average monthly income of P and Q is H 5050. The average monthly income of Q and R is H 6250 and the average monthly income of P and R is H 5200. What is the monthly income of P? Solution:  Let x, y and z represent the respective monthly incomes of P, Q and R. Thus, we can write

Solution:  Let the average age of the whole team be x years. Thus, 11x − ( 26 + 29 ) = 9( x − 1) ⇒ 11x − 9x = 46 ⇒ 2x = 46 ⇒ x = 23



x + y = (5050 × 2) = 10100

(1)

y + z = (6250 × 2) = 12500

(2)

x + z = (5200 × 2) = 10400

(3)

Adding Eqs. (1), (2) and (3), we get

Hence, average age of the team is 23 years. 19. A person buys petrol at H 7.50, H 8 and H 8.50 per litre for three successive years. What approximately is the average cost per litre of petrol if he spends H 4000 each year?

2 (x + y + z ) = 33000 ⇒ x + y + z = 16500

(4)

Subtracting Eq. (2) from (4), we get x = 4000. Hence, P’s monthly income = H 4000.

PRACTICE EXERCISE 1. Marks of 10 students in English are 46, 48, 47, 44, 40, 49, 39, 42, 44 and 41. What are the average marks obtained in English? (a) 44

(b) 45

(c) 43.5

(d) 46.4

2. Average weight of 5 people is 64.4. If the weight of four people is 64, 66, 58 and 72, then what is the weight of the fifth person? (a) 65

(b) 60

(c) 62

(d) 64

3. The average marks of students of class 5A in a subject are 38 and of class 5B are 42.5. If the number of students in class 5A is 25 and 5B is 20, then what are the average marks of all the students taken together? (a) 42

(b) 40

(c) 39.5

(d) 41.5

4. Five kilogram of flour costing H 6 per kilogram and 10 kg of flour costing H 9 per kilogram are mixed together. What is the net cost of the mixture? (a) H 7.2 per kilogram (c) H 10 per kilogram

Chapter 01-6.indd 615

(b) H 6.5 per kilogram (d) H 8 per kilogram

5. A man buys milk at H 5 per litre and after adding water, sells it at H 6 per litre, thereby making a 1 profit of 33 %. What is the proportion of milk to 3 water in the mixture? (a) 1:9

(b) 9:10

(c) 9:1

(d) 4:5

6. The average weight of a group of 20 boys is 60 kg. If the weight of another boy is added then the average weight becomes 58. What is the weight of the boy? (a) 39 kg (b) 41 kg

(c) 35.5 kg (d) 48 kg

7. The average temperature from Monday to Friday was recorded to be 12.5°C. If the average temperature of Monday and Tuesday was 14°C and the temperature on Wednesday and Thursday was 10°C and 11°C, respectively, then what was the temperature on Friday? (a) 12°C (c) 11.4°C

(b) 13.5°C (d) 14°C

22-08-2017 06:59:17

616     Average, Mixture and Alligation 

8. In what ratio must a grocer mix two varieties of salt worth H 40 per kilogram and H 48 per kilogram so that by selling the mixture at H 50.6 per kilogram he may gain 10%? (a) 1:2

(b) 1:3

(c) 2:3

(d) 4:1

9. In a mixture of 48 litre, the ratio of milk and water is 2:1. If this ratio is to be 1:3, then what is the quantity of water to be further added? (a) 96 litre (b) 72 litre (c) 60 litre

(d) 80 litre

10. The average of 10 results is 20 and that of 20 more results is 10. What is the average for all the results taken together? (a) 12.5

(b) 14.75

(c) 13.33

(d) 12

11. Average of four numbers is 8. If the second and fourth number is same, first number is twice the second number and third number is twice the first number, then what is the smallest number? (a) 8

(b) 6

(c) 4

(d) 2

12. The ratio of the number of boys and girls in a college is 5:4. If the percentage increase in the number of boys and girls be 20% and 25%, respectively, what will be the ratio of the total number of students before to the total number of students after increase? (a) 9:11

(b) 5:11

(c) 4:9

(d) 8:9

13. Jack has a collection of 280 balls. If 35% of them are red, 70 of them are yellow and the rest of the balls are black, then what is the ratio of red balls to black balls? (a) 7:8

(b) 3:4

(c) 5:6

(d) 2:5

14. The average age of three people is 52. If their ages are in the ratio 2:5:6, which of the following is not the age of any of the three persons? (a) 60

(b) 64

(c) 24

(d) 72

15. The speed of a train from A to B is 50 km/h while when coming back from B to A, its speed is 75 km/h. What is the average speed during the whole journey? (a) 65 km/h (c) 60 km/h

(b) 62.5 km/h (d) 67.5 km/h

16. A lab assistant mixes Zn and Cl in the ratio of 1:2 in order to produce ZnCl2. If the total amount of ZnCl2 produced is 50.7 g, what is the amount of Zn added? (a) 21.3 g (b) 25.35 g (c) 30.8 g

(d) 16.9 g

17. Average of 5 numbers is 24. The last number is replaced and the average becomes 26. If the last number before being replaced was 25, then what is the new last number? (a) 30

Chapter 01-6.indd 616

(b) 35

(c) 24

(d) 28

18. A tank contains 100 litre of pure milk. After every 10 min, 10 litre of solution is replaced with water. What will be the remaining amount of milk in the solution after 40 min? (a) 65.61 litre (c) 72.25 litre

(b) 58.65 litre (d) 62.45 litre

19. A person made a profit of 20% on 25% of the total quantity sold and on the rest he makes a profit of 10%. What was the net profit percentage in the whole transaction? (a) 15%

(b) 12.5%

(c) 25%

(d) 17.5%

20. A man bought 5 apples worth H 5 each, 10 bananas worth H 2 each, 3 oranges worth H 4 each and 7 kiwis worth H 12 each. What is the average of the total amount of fruits bought? (a) H 7

(b) H 6.54

(c) H 4.35

(d) H 5.36

21. If the average marks of three batches of 55, 60 and 45 students, respectively, are 50, 55, 60, then what are the average marks of all the students? (a) 53.33 (c) 54.68

(b) 55 (d) None of these

22. The average of 20 numbers is zero. Of them, at the most, how many may be greater than zero? (a) 0

(b) 1

(c) 10

(d) 19

23. The average age of husband, wife and their child 3 years ago was 27 years and that of wife and the child 5 years ago was 20 years. What is the present age of the husband? (a) 35 years (c) 50 years

(b) 40 years (d) 60 years

Directions for Q24 to Q25: Each of the questions given below consists of a question and two or three statements. You have to decide whether the data provided in the statement(s) is/are sufficient to answer the given question. 24. The average age of P, Q, R and S is 30 years. How old is R? I.  The sum of ages of P and R is 60 years. II.  S is 10 years younger than R. (a) I alone sufficient while II alone not sufficient to answer. (b) II alone sufficient while I alone not sufficient to answer. (c) Both I and II are not sufficient to answer. (d) Both I and II are necessary to answer. 25. In a cricket team, the average age of eleven players in 28 years. What is the age of the captain? I. The captain is eleven years older than the youngest player.

22-08-2017 06:59:17

EXPLANATIONS AND HINTS     617

II. The average age of 10 players, other than the captain is 27.3 years. III. Leaving aside the captain and the youngest player, the average ages of three groups of three players each are 25 years, 28 years and 30 years respectively.

26. A can contains a mixture of two liquids A and B is the ratio 7:5. When 9 litres of mixture are drawn off and the can is filled with B, the ratio of A and B becomes 7:9. How many litres of liquid A was contained by the can initially? (b) 20

(c) 21

(d) 25

27. Tea packs worth H 126 per kg and H 135 per kg are mixed with a third variety in the ratio 1:1:2. If the mixture is worth H 153 per kg, what will the price of the third variety per kg?

(b) H 170 (d) H 180

28. Find the ratio in which sugar at H 7.20 per kg be mixed with rice at H 5.70 a kg to produce a mixture worth H 6.30 a kg. (a) 1:3

(a) All I, II and III (b) II and III only (c) II only or I and III only (d) Any two of the three

(a) 10

(a) H 175.50 (c) H 169.50

(b) 2:3

(c) 3:4

(d) 4:5

29. How many kilogram of sugar costing H 9 per kg must be mixed with 27 kg of sugar costing H 7 per kg so that there may be a gain of 10% by selling the mixture at H 9.24 per kg? (a) 63 kg (b) 54 kg

(c) 42 kg

(d) 36 kg

30. A container contains 40 litres of milk. From this container 4 litres of milk was taken out and replaced by water. This process was repeated further two times. How much milk is now contained by the container? (a) 26.34 litres (c) 28 litres

(b) 27.36 litres (d) 29.16 litres

ANSWERS 1. (a)

5. (c)

9. (d)

13. (a)

17. (b)

21. (c)

25. (c)

29. (a)

2. (c)

6. (a)

10. (c)

14. (b)

18. (a)

22. (d)

26. (c)

30. (d)

3. (b)

7. (b)

11. (c)

15. (c)

19. (b)

23. (b)

27. (a)

4. (d)

8. (b)

12. (a)

16. (d)

20. (d)

24. (c)

28. (b)

EXPLANATIONS AND HINTS 1. (a) Given that the total number of students = 10 Total marks of the students in English = 46 + 48 + 47 + 44 + 40 + 49 + 39 + 42 + 44 + 41 = 440 440 Average marks of 10 students in English =  = 44 10 2. (c) Given that the average weight of 5 people = 64.4 Let the weight of the fifth person be x. Then, 64 + 66 + 58 + 72 + x = 64.4 5 ⇒ 260 + x = 322 ⇒ x = 62 3. (b) Given that the total students in class 5A = 25 and total students in class 5B = 20 The students are in the ratio of 25:20 = 5:4 Hence, average marks of all students taken together will be 5 × 38 + 4 × 42.5 360 = = 40 5+4 9

Chapter 01-6.indd 617

4. (d) Net cost of the mixture is 1 × 6 + 2 × 9 24 = =8 1+2 3 Hence, the net cost = H 8 per kilogram 5. (c) Given that the selling price of the mixture = H 6 per litre. So the cost price of the mixture will be 100 × 6 = H 4.5 per liter 133.33 Now, the cost price of the mixture is only due to the cost of milk We know that the cost price of milk = H 5 per litre 1000 Thus, for H 4.5, we would get milk = × 4.5 = 5   900 mL The mixture contains 900 mL milk and 100 mL water Thus, the ratio of milk to water = 900:100 = 9:1

22-08-2017 06:59:18

618     Average, Mixture and Alligation 

6. (a) Let the weight of the boy be x. Then, 20 × 60 + x = 58 21 ⇒ 1200 + x = 59 × 21 ⇒ x = 1239 − 1200 = 39

Milk = 32 litre Now quantity of water = 16 + x Now, 32 1 = 16 + x 3 ⇒ 3 × 32 = 16 + x ⇒ x = 96 − 16 = 80

Hence, the weight of the boy = 39 kg 7. (b) Let the temperature on Friday be x.

Thus, the quantity of water to be added = 80 litre Now, the average temperature of Monday and Tuesday = 14°C

10. (c) The average is given by 10 × 20 + 20 × 10 10 + 20 400 = = 13.33 30

Hence, the total temperature of Monday and Tuesday = 28°C Temperature recorded on Wednesday = 10°C Temperature recorded on Thursday = 11°C Now, average of temperature from Monday to Friday can be given as 28 + 10 + 11 + x = 12.5 5 ⇒ 49 + x = 12.5 × 5 ⇒ x = 62.5 − 49 = 13.5 Hence, the temperature on Friday = 13.5°C 8. (b) Given that the selling price of the mixture = H 50.6 per kilogram Gain = 10% 100 × 50.6 = H 46 110 If the amount of salt worth H 40 be x Cost price of the mixture =

The amount of salt worth H 48 be (1000 − x) Thus,

Third number = 4x Fourth number = x Average of four numbers = 8 Average is given by 2x + x + 4x + x =8 4 ⇒ 8x = 32 ⇒ x= 4 Hence, the smallest number = 4 12. (a) Given that the ratio of boys to girls = 5:4 Increase in the number of boys = 20%

40 × x 48 (1000 − x) + = 46 1000 1000 ⇒ 40x + 48000 − 48x = 46000 ⇒ 8x = 2000 ⇒ x = 250 Quantities of both the salts = 250 g and 750 g Ratio =

250 1 = =1:3 750 3

9. (d) Given that the total quantity of mixture = 48 litre Ratio of milk and water = 2:1 48 = 32 liter 2 +1 48 Total quantity of water = 1 × = 16 liter 2 +1 Now, if x litre of water is added. Then the new quantity of mixture = 48 + x Total quantity of the milk = 2 ×

Chapter 01-6.indd 618

11. (c) If the second number is x. The first number = 2x

Increase in the number of girls= 25% New ratio of boys to girls will be 5+

20 25 ×5 : 4 + 4× = 6:5 100 100

If x is a constant such that total students before = 5x + 4x = 9x Similarly, total students after increment = 6x + 5x = 11x Ratio of students before and after the increase = 9x : 11x = 9:11 13. (a) Given that the total balls that Jack has = 280 and the total yellow balls = 70 70 Total percentage of yellow balls = × 100 = 25% 280 Total red balls = 35% Total black balls = 100 − (25 + 35)% = 40% Ratio of red to black balls = 35/40 = 7/8 = 7:8

22-08-2017 06:59:21

EXPLANATIONS AND HINTS     619

14. (b) Let the ages be 2x, 5x and 6x. Then, 2x + 5x + 6x = 52 3 ⇒ 13x = 156 ⇒ x = 12

The solution has milk and water in the ratio of 9:1. Hence, the remaining amount of milk = 81 litre Remaining amount of water = 19 litre

Hence, the three ages are 24, 60 and 72.

Now, solution has milk and water in the ratio of 81:19.

So the answer is 64.

Now, after 30 min, 10 litre of the solution is replaced with water.

15. (c) Let the distance between A and B be x x Time taken from A to B = 50 x Time taken from B to A = 75 Total distance covered = 2x Now, average speed will be 2x 2x × 50 × 75 = (x / 50) + (x / 75) 75x + 50x 7500x = = 60 125x Hence, the average speed over the journey will be 60 km/h. 16. (d) Given that the ratio of zinc and chloride = 1:2 Total amount of ZnCl2 produced = 50.7 g If Zn added be x and Cl added be 2x Thus, the total amount = x + 2x = 50.7 ⇒ 3x = 50.7 50.7 ⇒x= = 16.9 3 Total amount of Zn mixed = 16.9 g 17. (b) Let the sum of four numbers be x and the replaced number be y. Now, initially the average of the five numbers is given by x + 25 = 24 5 ⇒ x = 120 − 25 = 95 Hence, the sum of the first four numbers = 95 Now, using the above value, after replacing the number 25 with y, we get 95 + y = 26 5 ⇒ y = 130 − 95 = 35 The last number after replacing is 35. 18. (a) Given that after 10 min, 10 litre of milk is replaced with water. Hence, in the solution, milk = 90 litre and water = 10 litre

Chapter 01-6.indd 619

Now, after 20 min, 10 litre of the solution is replaced with water.

The solution has milk and water in the ratio of 81:19. Hence, milk taken out from the solution = 8.1 litre Water taken out from the solution = 1.9 litre Remaining amount of milk = 72.9 litre Remaining amount of water = 27.1 litre After 40 min, 10 litre of solution will have 7.29 litre milk and 2.71 litre water and is replaced with water. Remaining amount of milk = 72.9 − 7.29 = 65.61 litre 19. (b) If the total quantity is x, then Profit made on 25% of the total quantity = 20 x x × = 100 4 20 Profit made on 75% of the total quantity = 10 3x 3x × = 100 4 40 So the total profit x 3x 5x + = 20 40 40 Total profit percentage = 12.5% 20. (d) Total cost of apples = H 5 × 5 = H 25 Total cost of bananas = H 2 × 10 = H 20 Total cost of oranges = H 4 × 3 = H 12 Total cost of kiwis = H 11 × 7 = H 77 Total cost of all the fruits = H 77 + 12 + 20 + 25 = H 134 Total fruits = 5 + 3 + 10 + 7 = 25 134 Average cost of all the items = = H 5.36 25 21. (c) According to formula of weighted average, 55 × 50 + 60 × 55 + 45 × 60 required average = 55 + 60 + 45 2750 + 3300 + 2700 8750 = = = 54.68 160 160









22. (d) Average of 20 numbers = 0. Therefore, sum of 20 numbers (0 × 20) = 0. It is possible that 19 of these numbers are positive with sum = x and the 20th number be −x.

22-08-2017 06:59:23

21 7x −   7 4 ∴ = 9 5x − 154 + 9

620     Average, Mixture and Alligation 

28x − 21 7 = ⇒ 252x − 189 = 140x + 147 20x + 21 9 ⇒ 112x = 336 ⇒x=3 Hence, the can contained 21 litres of A.

23. (b) Sum of the present ages of husband, wife and child = (27 × 3 + 3 × 3) years = 90 years



Sum of the present ages of wife and child = (20 × 2 + 5 × 2) years = 50 years Thus, husband’s present age = (90 − 50) = 40 years. 24. (c) P + Q + R + S = (30 × 4) ⇒ P + Q + R + S = 120 I.  P + R = 60 II.  S = (R − 10)

(i) (ii) (iii)

From Eqs. (i), (ii) and (iii), we cannot find R. Thus the data is insufficient.

27. (a) We know that first and second varieties are mixed in equal proportions. 126 + 135 = H130.5 Thus, average price = 2 So, their mixture is formed by mixing two varieties, on at H 130.50 per kg and the other at H X in the ratio 1:1.



25. (c) Total age of 11 players = (28 × 11) years = 308 years. I.  C = Y + 11 ⇒ C − Y = 11 (i) II. Total age of 10 players (excluding captain) = (27.3 × 10) years = 273 years Thus, age of captain = (308 − 273) years = 35 years Thus, C = 35

(ii)



Hence, by using alligation formula, x − 153 = 1 ⇒ x = 175.5 22.5 28. (b) Using alligation rule, x 6.3 − 5.7 0.6 2 = = = y 7.2 − 6.3 0.9 3 29. (a) S.P. of 1 kg of mixture = H 9.24

Putting value from Eq. (ii) in (i), we get Gain = 10%

Y = 35 − 11 = 24

× 9.24 = H 8.40 100 110

∴ C.P. of 1 kg of mixture = H 

III. Total age of 9 players = (25 × 3) + (28 × 3) + (30 × 3) (25 × 3) + (28 × 3) + (30 × 3) = 249 years  

Using the formula for alligation, x 8. 4 − 7 1.4 7 = = = y 9 − 8. 4 0.6 3

Thus, C + Y = (308 − 249) years = 59 years (iii) From Eq. (i) and (iii), we get C = 35

Let z kg of sugar of 1st kind be mixed with 27 kg of 2nd kind.

Thus, II alone gives the answer. Also, I and III together give the answer.

Then, 7 : 3 = z : 27

26. (c) Suppose the can initially contains 7x and 5x of mixtures A and B, respectively.



 





 



⇒z=

7 ×327 = 63 kg

7 21 Quantity of A in mixture left = 7 x − × 9 = 7 x − 30. 12 4 (d) Container contains 40 litres of milk. After remov21 litres. 7 ing 4 litres of milk by water, we have a mixture of 7x − × 9 = 7x − 4 12 36 litres milk and 4 litres of water. The ratio of milk 5 15 and water is 9:1. Hence, when 4 litres of mixture Quantity of B in mixture left = 5x − × 9 = 5x − 12 4 is removed again, 3.6 litres of milk and 0.4 litres of 15 litres. 5 water is removed. Now, remaining mixture will have 5x − × 9 = 5x − 4 12 (36 − 3.6) = 32.4 litres of milk and 7.6 litres of water. When we remove 4 litres of mixture again, 3.24 21 litres of milk and 0.76 litres of water is replaced by 7x − 7 4 4 litres of water. ∴ = 9 15 Hence, the final mixture contains (32.4 − 3.24) = 29.16 5x − +9 4 (32.4 − 3.24) = 29.16 litres. 28x − 21 7 ⇒ = ⇒ 252x − 189 = 140x + 147 20x + 21 9 ⇒ 112x = 336 ⇒x=3

 

 



 







Chapter 01-6.indd 620





22-08-2017 06:59:25

CHAPTER 7

RATIO, PROPORTION AND VARIATION

RATIO A ratio is a fraction of two numbers of the same kind. It is used to tell how many times a given number is in comparison to the other number or what part a given number is in comparison to the other number. Ratio of two numbers, a and b, is denoted by a:b and a is calculated by . b In the above expression, numerator “a” is called antecedent and denominator “b” is called consequent. If a c e = = b d f Then, each ratio will be a+c +e b+d+f The same principle can also be applied after multiplying numerator and denominator of any fraction by the same number.

Chapter 01-7.indd 621

If a c e = = b d f Then each ratio will be

 pb

pan + qc n + ren n

n

+ qd + rf

n



1/ n

where p, q, r and n may have any values except that they must not all be zeros. The compound ratio of r1 and r2, where r1 = a : b and r2 = x : y is defined as r1r2 = ax : by. Hence, compound ratios are simply a multiplication of two ratios. While comparing two numbers in terms of ratio: 1. Both the numbers or quantities should be of the same kind. For example, you cannot calculate the ratio between rupees and apples. 2. Units of measurements of numbers or quantities must be same. For example, you cannot calculate the ratio between H 20 and 200 paise. You can either convert H 20 into 2000 paise or convert 200 paise into H 2 to calculate the correct ratio.

22-08-2017 07:00:14

622     Ratio, Proportion and Variation 

3. Ratio of any quantity is always dimensionless and unitless. It is a pure number. 4. The ratio remains unchanged if both the antecedent and consequent are multiplied and divided by the same number.

2. Alternendo: Ratios of antecedents and consequents of two equal ratios are equal. Thus, if a c = b d

Then,

Till now we have just focused on ratios when they are equal. However, ratios sometimes do not follow equality. For unequal ratios, we have a c ≠ b d For unequal ratios, the two important points to be considered are as follows: 1. The sense of an inequality remains unchained if both members of the ratio are multiplied (or divided) by the same positive number. a b If a > b and k > 0, then ka > kb and > . k k 2. The sense of an inequality is reversed if both members of the ratio are multiplied (or divided) by the same negative number. a b If a > b and k < 0, then ka < kb and < . k k

PROPORTION When two ratios are equal to each other, then the four terms constituting the ratios are said to be in proportion. If a:b = c:d, then a, b, c and d are in proportion. The terms a and d are called extremes, while b and c are called means. When c = b / c, then a, b and c are said to be in continued proportion and b is called the geometric mean or mean proportional between a and c. Also, b2 = a × c

a b = c d 3. Componendo: It is the result we get after adding 1 to both sides of the equality. Thus, if a c = b d Then, a+b c +d = b d 4. Dividendo: It is the result we get after subtracting 1 from both sides of the equality. Thus, if a c = b d

Then a−b c −d = b d

5. Componendo—Dividendo: It is the result we get by dividing results of componendo by dividendo. Thus, if a c = b d Then a+b c +d = a−b c −d

VARIATION Variation means change of a quantity with respect to another quantity. It is the relationship between the change of two or more quantities. Now, quantities can vary broadly in the following ways:

Then, b = a×c Some properties of ratios and proportions are as follows: 1. Invertendo: Inverse ratios of two equal ratios are equal. Thus, if a c = b d Then, b d = a c

Chapter 01-7.indd 622

1. If a is directly proportional to b then, the nature of change is similar in the values of a and b. Hence, if a becomes twice its value then b also becomes twice its value. a ∝ b 2. If a is indirectly proportional to b, then the nature of change is inverse in the value of a and b. Hence, if a becomes twice its value, then b becomes half its value. 1 a ∝ b

22-08-2017 07:00:18

SOLVED EXAMPLES     623

SOLVED EXAMPLES 1. Two numbers x and y are 20% and 40% more than a number z. What is the ratio of the two numbers? Solution:  We have three numbers x, y and z. Also, 6 z 5 7 y = z + 40% of z = z 5 6 / 5z \x : y = 7 / 5z

Solution:  Given that the ratio of boys to girls = 7:5 Increase in the number of boys = 20% Increase in the number of girls = 40% So the new ratio of boys to girls will be

x = z + 20% of z =

⇒ x:y =

6 =6:7 7

2. A sum of money is to be distributed among A, B, C and D in the proportion of 3:2:1:4. If A gets H 1500, what is D’s share? Solution:  Given that A:B:C:D = 3:2:1:4 Now, we know that A’s share = H 1500 So A:D = 3:4 Now, if A = H 1500, then D=

4 × 1500 = 2000 3

Hence, D’s share = H 2000 3. If a sum of money is to be distributed among A, B, C and D in the proportion of 1:7:4:2. If B gets H 900 more than C, what is D’s share?

7+

20 25 ×7 : 5 + 5× = 6 : 5 = 9 : 11 100 100

5. If x:13::121:143, then what is the value of x? Solution:  It is given that x 121 = 13 143 121 × 13 ⇒x= 143 ⇒ x = 11 6. A sum of three numbers is 84. The ratio of the first and the second number is 5:3 and the ratio of the second and the third number is 3:4. What are the three numbers? Solution:  If x is a constant such that 5x + 3x + 4x = 84 ⇒ 12x = 84 ⇒x=7 First number = 5 × 7 = 35 Second number = 3 × 7 = 21 Third number = 4 × 7 = 28 So, the three numbers are 35, 21 and 28.

Solution:  Given that A:B:C:D = 1:7:4:2 Let B get x amount of the total. Then, 4x C= 7 Also, B − C = 900 Hence, 4x x− = 900 7 3x ⇒ = 900 7 ⇒ x = 300 × 7 = 2100 Thus, B’s share = H 2100 2 Total share of D = × 2100 = H 600 7 4. The ratio of the number of boys and girls in a college is 7:5. If the percentage increase in the number of boys and girls be 20% and 40%, respectively, what will be the new ratio?

Chapter 01-7.indd 623

7. The ratio of sale of shirts to trousers of a store is 4:3 and the ratio of sale of tie to trouser is 4:5. What is the ratio of sale of shirts to tie? Solution:  Given that the ratio of sold shirts to trousers = 4:3 Ratio of sold tie to trousers = 4:5 Ratio of sold trousers to tie = 5:4 4 5 5 Ratio of sold shirts to tie = × = = 5 : 3 3 4 3 8. What is the third proportional to 8 and 12? Solution:  Let the third proportional be x. Then, 8:12::12:x 8 12 = ⇒ x 12 x=

12 × 12 = 3 × 6 = 18 8

Thus, the third proportional is 18.

22-08-2017 07:00:21

624     Ratio, Proportion and Variation 

9. What ratio is equal to the ratio 83:162.5?

Solution:  Let the two numbers be 3x and 5x. Then, 3x − 9 12 = 5x − 9 23 ⇒ 23 (3x − 9) = 12 (5x − 9)

Solution:  Given that the ratio is 83:162.5. 3 3

4 2.5

Now, the ratio can be rewritten as [(2) ] : [(2) ] ⇒ 29 : 210 = 1 : 2

⇒ 9x = 99 ⇒ x = 11

Thus, the ratio equal to 83:161.5 = 1:2

Thus, the smaller number is (5 × 11) = 55. 10. A car running at a certain speed increases its speed to 4/3 times and reaches its destination 15 min earlier. How long will it take to reach the destination with the original speed?

14. The salaries A, B, C are in the ratio 2:3:5. If the increments of 15%, 10% and 20% are allowed, respectively, in their salaries, then what will be new ratio of their salaries?

Solution:  Let the original speed be x km/h. So the new speed = 4/3x

Solution:  Let A = 2x, B = 3x and C = 5x.

If the time taken originally be t1 and with the new speed be t2, then 4 t1 × x = t2 × x 3 4 ⇒ t1 = t2 × 3 3 ⇒ t2 = t1 × 4

A’s new salary =

100 × 2x =

23x 10

B’s new salary =

100 × 3x =

33x 10

C’s new salary =

100 × 5x = 6x

115

110

120

Also, t1 − t2 = 15 min. Hence

Thus, new ratio =

3 t1 − t1 × = 15 4 1 ⇒ t1 = 15 4 ⇒ t1 = 60

 10



23x 33x : : 6x = 23 : 33 : 6 10

15. The ratio of the number of boys and girls in a college is 7:8. If the percentage increase in the number of boys and girls be 20% and 10%, respectively, what will be the new ratio? Solution:  Originally, let the number of boys and girls in the college be 7x and 8x, respectively.

Hence, the time taken originally = 60 min 11. A metal bar ten feet long weighs 128 kg. What is the weight of a similar bar that is two feet four inches long?









120 × 7x Number of boys and girls after increase is 100 110 × 8x and 100 Solution:  Length of the second bar = 2 feet 120 110 28 7 × 7x : × 8x = 21 : 22 Hence, the required ratio = 4 inches = = feet 100 100 120 110 12 3 × 7x : × 8x = 21 : 22 100 Say, weight of the bar is x kg. Thus, 100 16. If H 736 was divided into three parts, proportional 7/3 10 7 128 448 1 2 3 ⇒x= × = 29.87 kg = = to : : , then what is the first part? 128 x 3 10 15 2 3 4 1 2 3 12. What is the fourth proportional to 5, 8 and 15? Solution:  Given ratio = : : = 6 : 8 : 9. 2 3 4 Solution:  Let the fourth proportional to 5, 8, 15 6 6 be x. = 736 × = H 192 The first part = 736 × 6+8+9 23 Then, 5:8:15:x 17. In a mixture 60 litres, the ratio of milk and water ⇒ 5x = (8 × 15) 2:1. If this ratio is to be 1:2, then what quantity of ⇒ x = 24 water is to be further added? 2 13. Two numbers are in the ratio 3:5. If 9 is subtracted Solution:  Quantity of milk = 60 × = 40 litres 3 from each, the new numbers are in the ratio 12:23. Quantity of water in it = (60 − 40) = 20 litres What is the larger number?















Chapter 01-7.indd 624



22-08-2017 07:00:24

PRACTICE EXERCISE     625

New ratio = 1:2 Let quantity of water to be added further be x litres. Then, ratio of milk and water = (40 : 20 + x)

19. The sum of three numbers is 98. If the ratio of the first to second is 2:3 and that of the second to the third is 5:8, then what is the second number? Solution:  Let the three parts be x, y and z. Then,

Now,







3 24 3 x : y = 2 : 3 and y : z = 5 : 8 = 5 × : 8 × = 3 : 40 1 = 5 5 5 20 + x 2 3 24 3 y : z = 5 : 8 = 5× : 8× = 3 : ⇒ 20 + x = 80 ⇒ x = 60 5 5 5 24 Therefore, quantity of water to be added = 60 litres. ⇒x:y:z =2:3: = 10 : 15 : 24 5 18. A sum money is to be distributed among A, B, 15 ⇒ y = 98 × = 30 C, D in the proportion of 5:2:4:3. If C gets H 1000 49 more than D, what is B’s share? 20. If 0.75:x::5:8, then what is the value of x? Solution:  Let the shares of A, B, C and D be H 5x,













H 2x, H 4x and H 3x, respectively. Then,



Solution:  We are given that 0.75 : x :: 5 : 8. Hence,

( x × 5 ) = ( 0.75 × 8 )

4x − 3x = 1000 ⇒ x = 1000

⇒x=

Thus, B’s share = H 2x = 2 × 1000 = H 2000 19. Two numbers are, respectively, 20% and 50% more than a third number. What is the ratio of the two numbers? Solution:  Let the third number be x. Then, first number =

Second number =





120x 6x = 100 5

150x 3x = 100 2

Therefore, ratio of first two numbers =

6x 3x : = 12x : 15x = 4 : 5 5 2



 5  = 1.2 6

21. If 40% of a number is equal to two-third of another number, what is the ratio of first number to the second number? 2 Solution:  Let 40% of A = B 3 Then, 40A 2B = 100 3 ⇒



6x 3x : = 12x : 15x = 4 : 5 5 2





A 2A 2B 2 5 5 ⇒ = × = = B 5 3 3 2 3

∴ A: B = 5 : 3

PRACTICE EXERCISE 1. Two numbers x and y are 25% and 50% more than a number z. What is the ratio of the two numbers? (a) 1/2 (c) 2/3

(b) 4/5 (d) 5/6

2. A sum of money is to be distributed among A, B, C and D in the proportion of 2:3:5:4. If C gets H 3000, what is A’s share? (a) H 600 (c) H 1500

(b) H 1200 (d) H 750

3. If a sum of money is to be distributed among A, B, C and D in the proportion of 3:1:4:6. If A gets H 1500, what is the total sum of money? (a) H 6500 (c) H 5000

Chapter 01-7.indd 625

(b) H 7000 (d) H 8000

4. If a sum of money is to be distributed among A, B, C and D in the proportion of 1:3:4:2. If C gets H 1000 more than D, what is B’s share? (a) H 1500 (c) H 1000

(b) H 2000 (d) H 3000

5. A car running at a certain speed increases its speed to 5/4 times and reaches its destination 20 min earlier. How long will it take to reach the destination with the original speed? (a) 120 min (c) 100 min

(b) 80 min (d) 60 min

6. A car running at a certain speed increases its speed to 2/3 times and reaches its destination 45 min earlier. How long will it take to reach the destination with the original speed?

22-08-2017 07:00:26

626     Ratio, Proportion and Variation 

(a) 120 min (c) 60 min

(b) 90 min (d) 75 min

7. Two quantities A and B vary directly. If initially A = 20 and B = 50, then what is the value of B if A = 24? (a) 96 (c) 60

(b) 72 (d) 80

8. The ratio of the number of boys and girls in a college is 5:4. If the percentage increase in the number of boys and girls be 20% and 25%, respectively, what will be the new ratio? (a) 6:5 (c) 1:1

(b) 5:3 (d) 6:7

allowed, respectively, in their salaries, then what will be the new ratio of their salaries? (a) 4:7:11 (c) 3:4:5

(b) 10:13:17 (d) 69:96:125

16. A, B and C invested money in the ratio 2:3:5 and the total time of their investments is 2:3:1. What is the ratio of their profits? (a) 2:6:11 (c) 3:6:14

(b) 2:5:7 (d) 4:12:15

17. What is the third proportional to 0.12 and 0.24? (a) 0.48 (c) 0.36

(b) 0.42 (d) 0.60

18. What ratio is equal to the ratio 44.5:82? 9. The ratio of the two numbers is 1:3 and the LCM of the numbers is 39. What are the two numbers? (a) 4 and 12 (c) 13 and 39

(b) 11 and 33 (d) 5 and 15

(a) 4:1 (c) 8:1

(b) 10:1 (d) 6:1

19. What fraction bears the same ratio to 13/27 that 11/13 does to 5/27?

10. If x:42::11:6, then what is the value of x? (a) 7 (c) 77

(b) 66 (d) 11/6

11. A sum of three numbers is 88. The ratio of the first and second number is 1:3 and the ratio of the second and third number is 3:4. What are the three numbers? (a) 10, 30, 40 (c) 24, 28, 32

(b) 12, 36, 48 (d) 11, 33, 44

12. If the ratio of the amount of money with A, B and 1 C is 3:2:8. If they get an increment of 33 %, 50% 3 and 25%, respectively, what is the new ratio of the money A, B and C have after increment? (a) 3:2:8 (c) 4:6:7

(b) 4:3:10 (d) 2:3:5

13. A fruit shop sold apples and oranges in the ratio of 5:2, respectively. The ratio of oranges to bananas sold is 5:3. What is the ratio of apples to bananas sold? (a) 25:3 (c) 5:3

(b) 25:6 (d) 2:3

14. A and B started a shop with initial investments in the ratio 5:7. If after one year their profits were in the ratio 1:2 and the period for A’s investment was 7 months, then what was the time period for which B invested the money? (a) 12 months (c) 14 months

(b) 9 months (d) 11 months

15. The salaries of three co-workers are in the ratio 3:4:5. If the increments of 15%, 20% and 25% are

Chapter 01-7.indd 626

(a) 11/27 (c) 13/11

(b) 11/5 (d) 5/13

20. If ratio of two natural numbers a and b is x and that of b and a is y, then what is the value of x + y? (a) x + y ≤ 2 (c) x + y ≥ 1

(b) x + y ≥ 0 (d) x + y ≥ 2

4 21. A and B together have H 1210. If  of A’s 15 2 amount is equal to    of B’s amount, how much 5 amount does B have? (a) H 460 (c) H 550

(b) H 484 (d) H 664

22. Seats for Electronics, Computers and Mechanical in an engineering college are in the ratio 5:7:8. There is a proposal to increase these seats by 40%, 50% and 75%, respectively. What will be the ratio of increased seats? (a) 2:3:4 (c) 6:8:9

(b) 6:7:8 (d) None of these

23. Salaries of Ravi and Sumit are in the ratio 2:3. If the salary of each is increased by H 4000, the new ratio becomes 40:57. What is Sumit’s salary? (a) 17000 (c) 25500

(b) 20000 (d) 38000

24. In a bag, there are coins of 25 p, 10 p and 5 p in the ratio of 1:2:3. If there is H 30 in all, how many 5 p coins are there? (a) 50 (c) 150

(b) 100 (d) 200

22-08-2017 07:00:27

EXPLANATIONS AND HINTS     627

25. The marks scored by a student in three subjects are in the ratio of 4:5:6. If the candidate scored an overall aggregate of 60% of the sum of the maximum marks and the maximum marks in all three subjects is the same, in how many subjects did he score more than 60%?  (a) 1 (c) 3

(b) 2 (d) None of the subject

26. The ratio of boys to girls in a class is 5:3. The class has 16 more boys than girls. How many girls are there in the class?  (a) 6 (c) 24

(b) 16 (d) 34

27. The ratio of marks obtained by Akshay and Shikhar is 6:5. If the combined average of their percentage is 68.75 and their sum of the marks is 275, find the total marks for which exam was conducted. (a) 150 (c) 400

28. The proportion of milk and water in 3 samples is 2:1, 3:2 and 5:3. A mixture comprising of equal quantities of all 3 samples is made. What is the proportion of milk and water in the mixture? (a) 2:1 (c) 99.61

(b) 5:1 (d) 227:133

29. A picture measuring 3.5″ high by 5″ wide is to be enlarged so that the width is now  9 inches. How tall will the picture be? (a) 6.3 inches (c) 5.5 inches

(b) 5.7 inches (d) 7 inches

30. The monthly salaries of Tejasvini and Sahay are in the ratio of 4:7. If each receives an increase of H 25 in the salary, the ratio is altered to 3:5. What is the value of their respective salaries? (a) 120 and 210 (c) 180 and 300

(b) 200 (d) 600

(b) 80 and 140 (d) 200 and 350

ANSWERS 1. (d)

6. (b)

11. (d)

16. (d)

21. (b)

26. (c)

2. (b)

7. (c)

12. (b)

17. (a)

22. (a)

27. (b)

3. (b)

8. (a)

13. (b)

18. (c)

23. (d)

28. (d)

4. (a)

9. (c)

14. (a)

19. (b)

24. (c)

29. (a)

5. (c)

10. (c)

15. (d)

20. (d)

25. (a)

30. (d)

EXPLANATIONS AND HINTS 1. (d) We have three numbers x, y and z. Also, 5 z 4 6 y = z + 50% of z = z 4 5/4 z x:y = 6/4 z 5 ⇒ x:y = 6 x = z + 25% of z =

2. (b) Given that A:B:C:D = 2:3:5:4. Now, we know that C = H 3000 Therefore, A:C = 2:5 Now, if

Chapter 01-7.indd 627

C = H 3000

Then 2 × 3000 = 1200 5 Thus, A’s share = H 1200 A=

3. (b) Given that A:B:C:D = 3:1:4:6 Now, we know that A = H 1500. So 1 × 1500 = 500 3 4 C = × 1500 = 2000 3 6 D = × 1500 = 3000 3 Total sum of money = H (1500 + 500 + 2000 + 3000) = H 7000 B=

22-08-2017 07:00:28

628     Ratio, Proportion and Variation 

4. (a) Given that A:B:C:D = 1:3:4:2 Let C get x amount of the total, then, x D= 2 Also, C − D = 1000 x x − = 1000 2 x = 2 × 1000 = 2000 Thus, C = H 2000 3 B = × 2000 = 1500 4 So the total share of B = H 1500

7. (c) Since the values of A1 and B1 vary directly with respect to A2 and B2, respectively. Then, A1 A = 2 B1 B2 ⇒

⇒ B2 =

Increase in the number of boys = 20% Increase in the number of girls = 25% So the new ratio of boys to girls 5+

Them the new speed = 5/4x

6. (b) Let the original speed be x km/h New speed = 2/3x If the time taken originally be t1 and with the new speed be t2 2 t1 × x = t2 × x 3 2 t1 = t2 × 3 3 t2 = t1 × 2 Also, t2 − t1= 45 min 3 t1 × − t1 = 45 2 1 ⇒ t1 = 45 2 ⇒ t1 = 90 Hence, time taken originally = 90 min

Chapter 01-7.indd 628

24 × 50 = 60 20

8. (a) Given that the ratio of boys and girls = 5:4

5. (c) Let the original speed be x km/h

If the time taken originally be t1 and with the new speed be t2 5 t1 × x = t2 × x 4 5 t1 = t2 × 4 4 t2 = t1 × 5 Also, t1 − t2 = 20 min 4 t1 − t1 × = 20 5 1 t = 20 5 1 t1 = 100 Hence, the time taken originally = 100 min

20 24 = B2 50

20 25 ×5 : 4 + 4× 100 100

= 5 + 1 : 4 + 1 = 6:5 9. (c) Ratio of the two numbers = 1:3 If the two numbers are x and 3x. Hence, x × 3x = H.C.F × 39 Now, H.C.F of the two numbers is x. So x × 3x = x × 39 ⇒ 3x2 = 39x ⇒ x = 13 Hence, the two numbers are 13 and 39 10. (c) Given that x 11 = 42 6 11 × 42 ⇒x= 6 ⇒x = 77 11. (d) If x is a constant such that 1x + 3x + 4x = 88 ⇒ 8x = 88 ⇒ x = 11 First number = 11 Second number = 3 × 11 = 33 Third number = 4 × 11 = 44 So the three numbers are 11, 33 and 44 12. (b) Given that the ratio of A, B and C = 3:2:8 1 Increment of A, B, C = 33 %, 50% and 25%, 3 respectively

22-08-2017 07:00:31

EXPLANATIONS AND HINTS     629

New ratio after the increment will be 1 33 50 25 3 + 3× 3 : 2 + 2× : 8 + 8× 100 100 100 = 3 + 1 : 2 + 1 : 8 + 2 = 4:3:10 13. (b) Given that the ratio of sold apples to oranges = 5:2

18. (c) Given that the ratio is 44.5:82 Now, the ratio can be rewritten as [(2)2 ]4.5 : [(2)3 ]2 ⇒ 29 : 26 = 23 = 8 Thus, the ratio equal to 44.5:82 = 8:1 19. (b) If the required fraction is x. Then,

Ratio of sold oranges to bananas = 5:3 5 5 25 Ratio of sold apples to bananas = × = = 25 : 6 2 3 6

x: ⇒

(x/13) (11/13) 11 13 27 = ⇒x= × × 27 (5/27) 13 27 5

14. (a) Let the investments of A and B, respectively, be 3x and 5x.

⇒x=

If the time period for B’s investments is y months 3x × 10 1 = 5x × y 2 ⇒ y = 3×2×2 ⇒ y = 12

Ratio of salaries after increment will be 15 20 25 : 4 + 4× : 5 + 5× 100 100 100 9 16 25 = 3+ :4+ :5+ = 69:96:125 20 20 20 3 + 3×

16. (d) Given that the ratio of money invested by A, B and C be 2:3:5. Hence, let the money invested by A, B and C be 2x, 3x and 5x, respectively.

11 5

Thus, the required fraction = 11/5 a b b y= a a b ⇒ x+y = + b a

20. (d) We have,

x=

Hence, the time period for B’s investments is 12 months 15. (d) Given that the ratio of salaries of co-workers = 3:4:5

a2 + b 2 ab Since the two numbers are natural numbers, the lowest value can be if a = b = 1. In that case, =

x+y =

21. (b) We are given that 4 2 A= B 15 5 ⇒A=

And let the time of investment by A, B and C be 2y, 4y and 3y, respectively.

= (2x)(2y):(3x)(4y):(5x)(3y) = 4:12:15 17. (a) Let the third proportional be x, then,

(1)2 + (1)2 =2 1×1

Thus, the value of x + y ≥ 2.

Ratio of time of investment = 2:4:3

Ratio of profit of A: profit of B: profit of C

13 11 5 :: : 27 13 27

5 × 4  B = 2 B 2

15

3

A 3 = ⇒ A:B = 3 : 2 B 2 2 Thus, B’s share = H 1210 × = H 484 5 ⇒





22. (a) Let the number of seats of Electronics, Computers and Mechanical be 5x, 7x and 8x, respectively.

0.12:0.24::0.24:x ⇒

0.12 0.24 = x 0.24

0.24 × 0.24 0.12 = 0.24 × 2 = 0.48 Thus, the third proportional is 0.48. ⇒x=

Chapter 01-7.indd 629

Number of increased seats = (140% of 5x), (150% of 7x) and (175% of 8x). ⇒

100 × 5x, 100 × 7x, 100 × 8x 140

⇒ 7 x,

150

175

21x , 14x 2

22-08-2017 07:00:35

630     Ratio, Proportion and Variation 

Therefore, the required ratio = 7 x :

i.e., 5x − 3x = 16 ⇒ x=8

21x : 14x 2

⇒ 14x : 21x : 28x ⇒2:3:4

Number of girls = 3x = 3 × 8 = 24

23. (d) Let the original salaries of Ravi and Sumit be H 2x and H 3x, respectively.

27. (b) Let Akshay’s marks be 6x and Shikhar’s marks be 5x. Therefore, the sum of the marks = 11x Total marks = 275 Thus, 11x = 275 ⇒ x = 25

Then,

Akshay’s marks = 6x = 6 × 25 = 150

2x + 4000 40 = 3x + 4000 57 ⇒ 57 (2x + 4000) = 40 (3x + 4000)

Shikhar’s marks = 5x = 5 × 25 = 125

⇒ 6x = 68000 ⇒ 3x = 34000 Sumit’s present salary = (3x + 4000) = H (34000 + 4000) = H 38000



Therefore, the combined average of their marks = (150 + 125) = 137.5 2 If total marks is 100, then their combined average of their percentage is 68.75.

24. (c) Let the number of 25 p, 10 p and 5 p coins be Hence, if their combined average of their percentage x, 2x and 3x, respectively. 137.5 25x 20x 15x 60x is 137.5 then the total marks = × 100 = 200 Then sum of their values = H + + = 68.75 100 100 100 100 25x 20x 15x 60x + + =H 28. (d) Proportion of milk in 3 samples is 2/3, 3/5, 5/8. 100 100 100 100 Proportion of water in the 3 samples is 1/3, 2/5, 3/8. Thus,







60x 30 × 100 = 30 ⇒ x = = 50 100 60

We are given that equal quantities are taken. 2 3 5 227 + + = 3 5 8 120 1 2 3 133 Total proportion of water = + + = 3 5 8 120 Total proportion of milk =

Hence, the number of 5 p coins = (3 × 50) = 150. 25. (a) Let the maximum marks in each of the three subjects be 100. Maximum marks attainable = 300 Therefore, the candidate scored 60% of 300 marks = 180 marks. Let the marks scored in the three subjects be 4x, 5x and 6x. Then, 4x + 5x + 6x = 180  ⇒ 15x = 180 or x = 12. Therefore, marks scored by the candidate in the three subjects are 4 × 12 = 48 5 × 12 = 60 6 × 12 = 72 Hence, the candidate has scored more than 60% in one subject. 26. (c) Let the number of boys in the class be 5x and the number of girls in the class be 3x. The class has 16 more boys, i.e., the difference between the number of boys and girls is 16

Chapter 01-7.indd 630

Proportion of milk and water in the solution is 227:133. 29. (a) The initial ratio of height to width is 3.5 to 5 inches. Let the height to be calculated be x inches. Hence, 3.5 x = ⇒ x = 6. 3 5 9 30. (d) Let the salaries of Tejasvini and Sahay be 4x and 7x. Therefore, 4x + 25 3 = 7 x + 25 5 ⇒ 5 (4x + 25) = 3 (7 x + 25) ⇒ x = 125 − 75 = 50 Therefore, their salaries are 4 × 50 = 200 and 7 × 50 = 350.

22-08-2017 07:00:39

CHAPTER 8

SPEED, DISTANCE AND TIME

INTRODUCTION

SOME IMPORTANT FORMULAS

Speed of an object is defined as the distance covered by that object per unit time. It is a scalar quantity. The unit of speed is m/s.

List of some of the common formulas used for speed, distance and time are as follows:

Speed =

Distance Time

The average speed of an object in an interval of time is the distance traveled by the object divided by the duration of the interval. Average speed =

Total distance traveled Total time taken

Relative speed of an object A, with a reference to object B, is equal to (sA − sB) m/s if A and B are traveling in the same direction, and (sA + sB) m/s if A and B are traveling in the opposite direction. Here, sA is the speed of object A and sB is the speed of object B.

Chapter 01-8.indd 631

1. If a body covers the distance d1 and d2 meters at a speed of s1 and s2 m/s, respectively, in time t1 and t2, then the total time taken T will be T = t1 + t2 =

d1 d2 + s1 s2

Total distance covered D will be D = d1 + d2 = s1t + s2 t2

2. While traveling a certain distance d, if a man changes his speed in the ratio x:y, then ratio of time taken becomes y:x. 3. If a certain distance d is covered at a speed of a m/s from A to B, and the same distance is covered again from B to A at b m/s, then average speed sav during the whole journey is given by

22-08-2017 07:01:20

632     Speed, Distance and Time 

Sav =

they meet is same as the time taken by the moving object to cover “a + b” meters with its own speed.

2ab m/s a+b

Also, if t1 and t2 are the time taken to travel from A to B, and B to A, respectively, then distance d from A to B is given by d = (t1 + t2 )

ab a+b

d = (t1 − t2 )

ab a−b

d = (a − b)

 a + b  s − s B t =  A  a + b   sB − sA

t1t2 t1 − t2

4. If a body travels a distance d from A to B with speed a m/s in time t1, and travels back from B to A with x/y of the usual speed a m/s, then change in time taken to cover the same distance is given by   y  − 1 × t1 ; for y > x Change in time =  x   1 − y × t1 ; for x > y  x

 

7. If two objects (such as a train or car) of length a and b meters move in the same direction at sA and sB m/s, then time taken t to cross each other from the time they meet is

 

5. Time taken by a moving object a meters long in passing a stationary object of negligible length from the time they meet is same as the time taken by the moving object to cover “a” meters with its own speed. 6. Time taken by a moving object a meters long in passing a stationary object b meters long from the time

if a > b if b > a

8. If two objects (such as a train or car) of length a and b meters move in the opposite direction at sA and sB m/s, respectively, then time t to cross each other from the time they meet is a+b t= sA + sB 9. If speed of a boat in still water is x m/s and speed of the stream is y m/s, then Speed of boat while traveling upstream su = (x + y) m/s 10. Speed of boat while traveling downstream sd = (x − y) m/s. In a circular track, the speed of boat in still water, s = 1 × (su + sd) m/s 2 length of the track is given by 2p r where r is the radius of the circular track. 11. In a circular race, the total length of the race is given by 2p rn where n is the number of times the racer covers the track.

SOLVED EXAMPLES 1. A policeman goes after a thief who has a 1000 m start. The policeman runs 1 km in 5 min and the thief 1 km in 8 min. How far did the thief go before he was overtaken?

2. A man rows upstream 18 km and downstream 30 km taking 3 h each time. What is the velocity of the current? Solution:  We know that,

Solution:  We know that

18 =3 1 Speed upstream Speed of the policeman = km/min 5 30 1 =3 and Speed of the thief = km/min Speed downstream 8 1 1 3 3000 500 Policeman gains = − = km/min = m/min = Thus, m/min speed upstream = (18/3) km/h = 6 km/h 5 8 40 18 3 and speed downstream = (30/3) km/h = 10 km/h 500 1 3 3000 = km/min = m/min = m/min 1 1 3 8 40 18 Speed of the river = (10 − 6) = × 4 = 2 km/h 1000 2 2 To gain 1000 m, time required = min = 6 min 500 3 3. A train leaves from Delhi to Jalandhar at 6 am 1 3 Hence, the thief has gone ahead by 6 × = km or 750 m and another train leaves from Jalandhar at 7 am. 1 3 8 4 6 × = km or 750 m The speed of the train from Jalandhar is 1/3 times 8 4

Chapter 01-8.indd 632

22-08-2017 07:01:23

SOLVED EXAMPLES     633

faster than the train leaving from Delhi. If the ­distance between Delhi and Jalandhar is roughly 360 km and both trains meet at 8:30 am, then what is the speed of the two trains?

5. A train running with the speed of 90 km/h crosses a platform in 15 s. If the length of the train is 200 m, what is the length of the platform? Solution:  Let the length of the platform be x meters. We know that the length of the train = 200 m

Solution:  Let the speed of the train from Delhi is x km/h. Hence, speed of the train from Jalandhar Time taken = 15 s 4x km/h = 5 3 Speed of the train = 90 km/h = 90 km/h = 90 × m/s = 25 m/s 18 Distance traveled by the train from Delhi till 5 90 km/h = 90 × m/s = 25 m/s 5 18 8:30 am = x × 2 Relative speed = 25 m/s (since platform is stationary) Distance traveled by the train from Jalandhar till Hence, 4x 3 8:30 am = × = 2x 200 + x 3 2 = 25 15 Now, total distance = 360 km ⇒ x = 375 − 200 = 175 m Hence, 6. A sailor covers a distance in 5 h downstream and 5 x + 2x = 360 covers the same distance in 3 h upstream. If the 2 speed of the river is 4 m/s, then what is the speed 9 of the boat? ⇒ x = 360 2 Solution:  Let the speed of the boat in still water ⇒ x = 80 is x m/s. Speed of train from Delhi = 80 km/h Speed of the boat while going upstream = (x − 4) m/s 4 × 80 Speed of boat while going downstream = (x + 4) m/s Speed of train from Jalandhar = km/h = 106.67 km/h 3 4 × 80 Since distance remains the same in both cases, = km/h = 106.67 km/h 3 (x + 4) × 3 = (x − 4) × 5 4. A train traveling at 60 km/h passes a cyclist going ⇒ 3x + 12 = 5x − 20 in the same direction in 8 s. If the cyclist would ⇒ x = 16 have been in the opposite direction, the train would have passed him in 4 s. What is the length of the The speed of the boat in still water = 16 m/s. train? 7. A train increases its speed by 10% and reaches Solution:  Let the speed of the cyclist is x km/h. 4 min early. What was the original time the train We know that took to reach the destination? Length = Relative speed × Time = (60 − x) × 8 Similarly, length = (60 + x) × 4 Solving the above equations, we get (60 − x) × 8 = (60 + x) × 4 ⇒ 60 = 3x ⇒ x = 20

Solution:  Let the initial speed be x. 11x So the new speed = 10 10y Now, if the initial time was y, time becomes . 11 Hence, 10y y− =4 11 ⇒ y = 44 Thus, the initial time taken to reach the destination is 44 min.

Thus, the speed of the cyclist = 20 km/h Length of the train = (60 − 20) × length of the train = 88.89 m.

Chapter 01-8.indd 633

5 × 8. Thus, the 18

8. Rachyita goes to office at a speed of 7 km/h and returns to her home at a speed of 4 km/h. If she takes 22 h in total, what is the distance between her office and her home?

22-08-2017 07:01:25

634     Speed, Distance and Time 

Solution:  Let the distance between office and home be x km. Then x x + = 22 7 4 11x ⇒ = 22 28 ⇒ x = 56 km

From Southampton to London, Time taken = 1 hour and 30 minutes = Average speed =

3 hours 2

90 = 60 mph 3/2

12. Jane drives at an average speed of 45 mph on a journey of 175 km. How long does the journey take? Solution:  We are given that,

Total distance = 56 km Distance = 175 km 9. Without stoppage, a train travels at an average speed of 60 km/h and with stoppages it covers the same distance at an average speed of 48 km/h. How many stoppages are there if each stoppage is of exactly 3 min? Solution:  The average speeds of the trains are 60 km/h and 48 km/h. To cover a distance of 48 km, the slower train will take 60 min.

Average speed = 45 mph = 45 × 1.61 = 72.45 kmph Time =

Distance 175 = = 2.415 hours Speed 72.45  

13. A man covered a certain distance at some speed. Had he moved 3 kmph faster, he would have taken 40 minute less. If he had moved 2 kmph slower, he would have taken 40 minutes more. What is the distance? Solution:  Let the distance be x km and speed = y kmph.

The faster train will cover that distance in 48 min. Hence, for the faster train to cover the same distance as the slower train, total time of stoppage = 12 min.

Then,

Now, since each stoppage is of 3 min, total number of stoppages = 4.



x x 40 − = ⇒ 2y ( y + 3 ) = 9x  y y + 3 60

10. A train requires 3 s to pass a pole while it requires 8 s to cross a stationary train which is 150 meters long. What is the speed of the train?



x x 40 − = ⇒ y (y − 2) = 3x  y −2 y 60

Solution:  Let the length of moving train be x, and speed of train be y. To cross a pole Length of train Speed of train x 3= y

(i)

(ii)

On dividing Eq. (i) by Eq. (ii), we get 2y (y + 3) y (y − 2)

=

9x ⇒ 2 (y + 3) = 3 (y − 2) ⇒ y = 12 3x

Putting this value in Eq. (ii), we get

Time =

x + 250 y 150 8 = 3+ y y = 30 km/h

Also, to cross a stationary train, 8 =

Hence, speed of the moving train = 30 km/h. 11. Aishling drives from Plymouth to Southampton, a distance of 160 miles, in 4 hours. She then drives from Southampton to London, a distance of 90 miles, in 1 hour and 30 minutes. Determine her average speed of her each journey. Solution:  From Plymouth to Southampton, 160 Average speed = = 40 mph 4

Chapter 01-8.indd 634

12 (12 - 2) = 3x ⇒ x = 40 14. A farmer travelled a distance of 61 km in 9 hours. He travelled partly on foot at the rate of 4 km/ hr and partly on bicycle at 9 km/hr. What is the distance travelled on foot? Solution:  Let the distance travelled on foot be x km. Then, distance travelled on bicycle = (61 − x) km So, x (61 − x) + = 9 ⇒ 9x + 4 (61 − x) = 9 × 36 4 9 ⇔ 5x = 80 ⇒ x = 16 km 15. It takes eight hours for a 600 km journey, if 120 km is done by train and the rest by car. It takes 20 ­minutes more, if 200 km is done by train and the rest by car. What is the ratio of the speed of the train to that of the cars?

22-08-2017 07:01:27

SOLVED EXAMPLES     635

Solution:  Let the speed of the train be x km/hr and that of the car be y km/hr.

Solution:  Let the duration of the flight be x hrs. Then,

Then,

600 600 − = 200 x x + (1 / 2) (i)



120 480 1 4 1  + =8⇒ + = x y x y 15

(ii)



200 400 25 1 2 1 ⇒ + =  + = x y 3 x y 24

600 1200 − = 200 x 2x + 1 ⇒ x (2x + 1) = 3 ⇒

⇒ 2x2 + x − 3 = 0 ⇒ (2x + 3)(x − 1) = 0

Eq. (i) − (ii), we get

x = 1 hr   [neglecting the negative value of x] 4 2 1 1 − = − y y 15 24 2 8−5 2 3 ⇒ = ⇒ = ⇒ y = 80 y 120 y 120 Substituting value of y in Eq. (i), we get 1 4 1 1 1 1 4−3 1 1 ⇒ = − ⇒ = + = = x 80 15 x 15 20 60 x 60 ⇒ x = 60

Thus, the duration of the flight = 1 hr. 19. A man completes a journey in 10 hours. He travels first half of the journey at the rate of 21 km/hr and second half at the rate of 24 km/hr. What is the total length of the journey? Solution:  We are given that total time of the journey = 10 hours. Let the distance be x km. Then,

(1 / 2) x

(1 / 2) x +

Therefore, x = 60 and y = 80. 21

= 10 ⇒

24

x x + = 20 21 24

Thus, ratio of speeds = 60:80 = 3:4. 16. Hatem is travelling on his cycle and has calculated to reach point A at 2 P.M. if he travels at 10 kmph, he will reach there at 12 noon if he travels at 15 kmph. At what speed must he travel to reach A at 1 P.M? Solution:  Let the distance travelled by x km. Then, x x − = 2 ⇒ 3x − 2x = 60 ⇒ x = 60 km 10 15 60 Time taken to travel 60 km at 10 km/hr = hrs 10 = 6 hrs.

 

⇒ 15x = 168 × 20 ⇒ x =

 

17. In covering a distance of 30 km, Abhay takes 2 hours more than Sameer. If Abhay doubles his speed, then he would take 1 hour less than Sameer. What is Abhay’s speed?

168 × 20 15



x = 224 km Thus, the total length of the journey = 224 km. 20. The ratio between the speeds of two trains is 7:8. If the second train runs 400 km in 4 hours, then what is the speed of the first train? Solution:  Let the speed of the two trains be 7x and 8x km/hr. Then, 400 8x = = 100 4

 

So, Hatem started 6 hours before 2 p.m., i.e. at 8 a.m. 60 Therefore, required speed = = 12 kmph 5



⇒x=

 8  = 12.5 100

Therefore, speed of first train = (7 × 12.5) = 87.5 km/hr. 21. A man on tour travels first 160 km at 64 km/hr and the next 160 km at 80 km/hr. What is the average speed for the first 320 km of the tour?

Solution:  Let Abhay’s speed be x kmph. Solution:  Total time taken for the entire journey Then,

 64 + 80  = 2 hrs 160

30 30 − = 3 ⇒ 6x = 30 ⇒ x = 5 km/hr. x 2x 18. In a flight of 600 km, an aircraft was slowed down due to bad weather. Its average speed for the trip was reduced by 200 km/hr and the time of flight increased by 30 minutes. What is the duration of the flight?

Chapter 01-8.indd 635

=

160

9



Therefore, average speed = 320 ×



2 = 71.11 km/hr. 9  

5 of its actual speed covers 7 42 km in 1 hr 40 min 48 sec. What is the actual speed of the car?

22. A car travelling with 

22-08-2017 07:01:29

636     Speed, Distance and Time 

Solution:  We are given that time taken = 1 hr 40 min 48 sec = 1 hr 40

126 4 min = hr. 75 5

Solution:  Let speed of the car be x kmph. Then, the speed of the train =

 

150x 3 = x kmph. 100 2

Let the actual speed be x km/hr. Thus, Then, 75 75 125 75 50 5 − = ⇒ − = x (3 / 2) x 10 × 60 x x 24

5 126 42 × 7 × 75 = 42 ⇒ x = = 35 km/hr x× 7 75 5 × 126





⇒x= 23. Excluding stoppages, the speed of a bus is 54 kmph and including stoppages, it is 45 kmph. For how many minutes does the bus stop per hour? Solution:  Due to stoppages, it covers (54 − 45) km less in one hour.



25 × 24 = 120 5



Hence, the speed of the car is 120 kmph. 25. If a person walks at 14 km/hr instead of 10 km/hr, he would have walked 20 km more.  What is the actual distance travelled by him?

Hence, the bus covers 9 km less.

 54  = 6 9

Time taken to cover 9 km = 1 × 60 = 10 min. 6

Solution:  Let the actual distance travelled by x km.

1 hr =

24. A train can travel 50% faster than a car. Both start from point A at the same time and reach point B 75 km away from A at the same time. On the way, however, the train lost about 12.5 minutes while stopping at the stations. What is the speed of the car?

Then, x x + 20 = ⇒ 14x = 10x + 200 10 14 ⇒ 4x = 200 ⇒ x = 50 km Hence, the actual distance travelled is 50 km.

PRACTICE EXERCISE 1. A car covers first 300 m at a speed of 54 km/h and the next 500 m at a speed of 72 km/h. What is the average speed of the car? (a) 50 km/h (c) 60 km/h

(b) 47 km/h (d) 64 km/h

2. A policeman goes after a thief who has a 500 m start. The policeman runs 1 km in 6 min and the thief 1 km in 9 min. How far did the thief go before he was overtaken? (a) 1 km (c) 1 14 km

(b) 1 12 m (d) 2 km

3. A man rows upstream 13 km and downstream 28 km, taking 5 h each time. What is the velocity of the current? (a) 1.5 km/h (c) 3.6 km/h

(b) 1.2 km/h (d) 2 km/h

4. A train overtakes two villagers walking at a rate of 3 km/h and 4 km/h, respectively, in the same

Chapter 01-8.indd 636

direction and completely passes them in 8 and 10 s. Assuming that the train is traveling at a uniform speed, what is the speed of the train in km/h? (a) 8 (c) 15

(b) 12.5 (d) 10

5. In the previous question, what is the length of the train? (a) 50 m (c) 20.50 m

(b) 11.11 m (d) 12 m

6. A train leaves from Delhi to Jammu at 7 pm and another train leaves from Jammu at 8 pm. The speed of the train from Delhi is 60 km/h and the speed of the train from Jammu is 1/4 times faster. If the distance between Delhi and Jammu is roughly 600 km, then at what distance from Delhi will the two trains meet? (a) 350 km (c) 324 km

(b) 276 km (d) 288 km

22-08-2017 07:01:31

PRACTICE EXERCISE     637

3 times faster than Jeremy. If Jack 4 gives Jeremy a start of 90 m, how far must the winning post be so that it may be a dead heat?

7. Jack runs 1

(a) 250 m (c) 175 m

(b) 140 m (d) 210 m

8. A train traveling at 40 km/h passes a runner going in the same direction in 6 s. If the runner would have been in the opposite direction, the train would have passed him in 4 s. What is the length of the train? (a) 72.5 m (c) 60 m

(b) 55 m (d) 67 m

9. Two men A and B run a 4-km race on a circular course of 1/2 km. If their speeds are in the ratio of 5:4, how often does the winner pass the other? (a) Once (c) Thrice

(b) Twice (d) Four times

10. A train running with the speed of 72 km/h crosses a platform in 20 s. If the length of the train is 100 m, what is the length of the platform? (a) 200 m (c) 400 m

(b) 300 m (d) 500 m

11. A sailor covers a distance in 6 h downstream and covers the same distance in 3 h upstream. If the speed of the river is 5 m/s, then what is the speed of the boat? (a) 10 m/s (c) 15 m/s

(b) 12 m/s (d) 18 m/s

12. A train increases its speed by 25% and reaches 5 min early. What was the original time the train took to reach the destination? (a) 30 min (c) 40 min

(b) 25 min (d) 15 min

13. A car travels from A to B at x1 km/h, travels back from B to A at x2 km/h and again goes back from A to B at x2 km/h. What is the average speed of the car? 2x1x2 (a) x1 + 2x2 (c)

3x1x2 x1 + 2x2

2x1x2 (b) 2x1 + x2 (d)

3x1x2 2x1 + x2

14. Charlie reaches his college 6 min late if he walks at a speed of 10 km/h. Next time he doubles his speed but finds that he is still 2 min late. What is the distance between Charlie’s college and house?

Chapter 01-8.indd 637

(a) 2.5 km (c) 7.5 km

(b) 5 km (d) 10 km

15. Rachyita goes to office at a speed of 5 km/h and returns to her home at a speed of 3 km/h. If she takes 8 h in total, what is the distance between her office and her home? (a) 15 km (c) 7.5 km

(b) 20 km (d) 10 km

16. A boat rows 16 km upstream and 30 km downstream taking 5 h each time. What was the speed of the boat in still water? (a) 6 km/h (c) 5 km/h

(b) 8 km/h (d) 2 km/h

17. Two trains are traveling in the same direction at 60 km/h and 30 km/h, respectively. The faster train crosses a man in the slower train in 9 s. What is the length of the faster train? (a) 50 km (c) 75 km

(b) 60 km (d) 100 km

18. Without stoppage, a train travels at an average speed of 60 km/h, and with stoppages it covers the same distance at an average speed of 40 km/h. How many stoppages are there if each stoppage is of exactly 2 min? (a) 8 (c) 5

(b) 10 (d) 6

19. A train requires 5 s to pass a pole while it requires 12 s to cross a stationary train which is 350 m long. What is the speed of the train? (a) 50 km/h (c) 35 km/h

(b) 85 km/h (d) 65 km/h

20. A train decreases its speed by 20% and reaches 4  min late. What was the original time the train took to reach the destination? (a) 30 min (c) 12 min

(b) 24 min (d) 16 min

21. A person crosses a 600 m long street in 5 minutes. What is his speed in kmph? (a) 3.6 (c) 8.4

(b) 7.2 (d) 10

22. An aeroplane covers a certain distance at a speed of 240 kmph in 5 hours. To cover the same distance 2 in 1 hours, what must be its speed? 3 (a) 300 kmph (c) 600 kmph

(b) 360 kmph (d) 720 kmph

22-08-2017 07:01:32

638     Speed, Distance and Time 

Directions for Q23 to Q26: Each of the questions given below consists of a question and two or three statements. You have to decide whether the data provided in the statement(s) is/are sufficient to answer the given question. 23. Two towns are connected by railway. What is the distance between them? I. The speed of the mail train is 12 km/h more than that of an express train. II. A mail train takes 40 minutes less than an express train to cover the distance. (a) I alone is sufficient while II alone is not ­sufficient to answer. (b) II alone is sufficient while I alone is not ­sufficient to answer. (c) Either I or II alone sufficient to answer. (d) Both I and II are not sufficient to answer. 24. The towns A, B and C are on a straight line. Town C is between A and B. The distance from A to B is 100 km, How far is A from C? I. The distance from A to B is 25% more than the distance from C to B. II. The distance from A to C is 1/4 of the distance C to B. (a) I alone is sufficient while II alone is not sufficient to answer. (b) II alone is sufficient while I alone is not sufficient to answer. (c) Either I or II alone sufficient to answer. (d) Both I and II are not sufficient to answer. 25. Two cars pass each other in opposite direction. How long would they take to be 500 km apart?

are two conveyor belt, one for carrying parts from P to point Q and another for carrying parts from R to point Q. P, Q and R in that order are in a straight line. Sometimes, the belt serves to transport cart, which can themselves move with respect to the belts. The two belts move at a speed of 0.5 m/s and the cart move at a uniform speed of 2m/s with respect to the belts. A cart goes from point P to R and back to P taking a total of 64 s. Find the distance PR in meters. Assume that the time taken by the cart to turn back at R is negligible? (a) 64 (c) 54

(b) 60 (d) 48

28. A, B and C start simultaneously from X to Y. A reaches Y, turns back and meet B at a distance of 11 km from Y. B reached Y, turns back and meet C at a distance of 9 km from Y. If the ratio of the speeds of A and C is 3:2, what is the distance between X and Y? (a) 99 m (c) 120 m

(b) 100 m (d) 142 m

29. Two friends A and B run around a circular track of length 510 metres, starting from the same point, simultaneously and in the same direction. A who runs faster laps B in the middle of the 5th round. If A and B were to run a 3 km race long race, how much start, in terms of distance, should A give B so that they finish the race in a dead heat? (a) 545.45 m (c) 666.67 m

(b) 600 m (d) 857.14 m

I. The sum of their speeds is 135 kmph. II. The difference of their speed is 25 kmph. (a) I alone is sufficient while II alone is not sufficient to answer. (b) II alone is sufficient while I alone is not sufficient to answer. (c) Either I or II alone sufficient to answer. (d) Both I and II are necessary to answer.

30. Three athletes A, B and C run a race, B finished 24 meters ahead of C and 36 m ahead of A, while C finished 16 m ahead of A. If each athlete runs the entire distance at their respective constant speeds, what is the length of the race?

26. How much time did X take to reach the destination?

31. From a point P, on the surface of radius 3cm, two cockroaches A and B started moving along two different circular paths, each having the maximum possible radius, on the surface of the sphere, that lie in the two different planes which are inclined at an angle of 45 degree to each other. If A and B takes 18 sec and 6 sec, respectively, to complete one revolution along their respective circular paths, then after how much time will they meet again, after they start from P?

I. The ratio between the speed of X and Y is 3:4. II. Y takes 36 minutes to reach the same destination. (a) I alone is sufficient while II alone is not sufficient to answer. (b) II alone is sufficient while I alone is not sufficient to answer. (c) Either I or II alone sufficient to answer. (d) Both I and II are necessary to answer. 27. In an industry, the raw materials and the finished goods are transported on the conveyor belt. There

Chapter 01-8.indd 638

(a) 108 m (c) 80 m

(a) 27 sec (c) 18 sec

(b) 90 m (d) 96 m

(b) 24 sec (d) 9 sec

22-08-2017 07:01:32

PRACTICE EXERCISE     639

32. Arpit and Gaurav leave points x and y towards y and x, respectively, simultaneously and travel in the same route. After meeting each other on the way, Arpit takes 4 hours to reach her destination, while Gaurav takes 9 hours to reach his destination. If the speed of Arpit is 48 km/hr, what is the speed of Gaurav? (a) 72 mph (c) 20 mph

(b) 32 mph (d) None of these

33. A man driving his bike at 24 kmph reaches his office 5 minutes late. Had he driven 25% faster on an average he would have reached 4 minutes earlier than the scheduled time. How far is his office? (a) 24 km (c) 72 km

(b) 18 km (d) 36 km

34. Two motorists Anil and Sunil are practicing with two different sports cars on the circular racing track, for the car racing tournament to be held next month. Both Anil and Sunil start from the same point on the circular track. Anil completes one round of the track in 1 min and Sunil takes 2 min to complete a round. While Anil maintains speed for all the rounds, Sunil halves his speed after the completion of each round. How many times Anil and Sunil will meet between 6th round and 9th round of Sunil (6th and 9th round is excluded)? Assume that the speed of Sunil remains steady throughout each round and changes only after the completion of that round. (a) 382 (c) 260

(b) 347 (d) 189

35. A river flows at 12 km/h. A boy who can row at 25 m/s in still water had to cross it in the least 18 possible time. The distance covered by the boy is how many times the width of the river? (a) 2.1 (c) 2.6

(b) 2.3 (d) 2.9

36. In a 3600 m race around a circular track of length 400 m, the faster runner and the slowest runner meet at the end of the fourth minute, for the first time after the start of the race. All the runners maintain uniform speed throughout the race. If the faster runner runs at thrice the speed of the slowest runner. Find the time taken by the faster runner to finish the race. (a) 36 minutes (c) 16 minutes

(b) 24 minutes (d) 12 minutes

37. If the wheel of a bicycle makes 560 revolutions in travelling 1.1 km, what is its radius?

Chapter 01-8.indd 639

(a) 31.25 cm (c) 35.15 cm

(b) 37.75 cm (d) 11.25 cm

38. A ship develops a leak 12 km from the shore. Despite the leak, the ship is able to move towards the shore at a speed of 8 km/hr. However, the ship can stay afloat only for 20 minutes. If a rescue vessel were to leave from the shore towards the ship and it takes 4 minutes to evacuate the crew and passengers of the ship, what should be the minimum speed of the rescue vessel in order to be able to successfully rescue the people aboard the ship? (a) 53 km/hr (c) 28 km/hr

(b) 37 km/hr (d) 44 km/hr

39. When an object is dropped, the number of feet N 1 that it falls is given by the formula N = gt2 , 2 where t is the time in seconds from the time it was dropped and g is 32.2. If it takes 5 seconds for the object to reach the ground, how many feet does it fall during the last 2 seconds? (a) 64.4 (c) 161.0

(b) 96.6 (d) 257.6

40. By walking at 3/4th of his usual speed, a man reaches office 20 minutes later than usual. What is his usual time? (a) 30 minutes (c) 70 minutes

(b) 60 minutes (d) 50 minutes

41. In a 500m race Dishu beats Abhishek by 100 m or 5 seconds. In another race on the same track at the same speeds. Abhishek and Prashant start at one end while Dishu starts at the opposite end. How many metres would Abhishek have covered, by the time Dishu meets Prashant given that Dishu’s speed is 10 m/sec more than that of Prashant. (a) 200 m (c) 250 m

(b) 225 m (d) 275 m

42. The speed of a motor boat itself is 20 km/h and the rate of flow of the river is 4 km/h. Moving with the stream the boat went 120 km. What distance will the boat cover during the same time going against the stream? (a) 80 km (c) 60 km

(b) 180 km (d) 100 km

43. Akhil covers a part of the journey at 20 kmph and the balance at 70 kmph taking total of 8 hours to cover the distance of 400 km. How many hours has he been driving at 20 kmph? (a) 2 hours (b) 3 hours 20 minutes (c) 4 hours 40 minutes (d) 3 hours 12 minutes

22-08-2017 07:01:32

640     Speed, Distance and Time 

44. I travel the first part of my journey at 40 kmph and the second part at 60 kmph and cover the total distance of 240 km to my destination in 5 hours. How long did the first part of my journey last? (a) 4 hours (c) 3 hours

(b) 2 hours (d) 2 hours 30 minutes

45. Two men are walking towards each other alongside a railway track. A freight train overtakes one of them in 20 seconds and exactly 10 minutes later meets the other man coming from the opposite direction. The train passes this man is 18 seconds. Assume the velocities are constant throughout. How long after the train has passed the second man will the men meet? (a) 89.7 minutes (c) 90.3 seconds

(b) 90 minutes (d) 91 seconds

46. P and Q walk from A to B, a distance of 27 km at 5 km/hr and 7 km/hr, respectively. Q reaches B and immediately turns back meeting P at T. What is the distance from A to T? (a) 25 km (c) 24 km

(b) 22.5 km (d) 20 km

47. A bus without stopping travels at an average speed of 60 km/hr and with stoppages at an average speed of 40 km/hr. What is the total time taken by the bus for stoppages on a route of length 300 km?

(a) 4 hours (c) 2.5 hours

(b) 3 hours (d) 3.5 hours

48. Distance between the points is P. A man with the speed of s and reaches the other point late by 2  hours. Calculated the speed with which the man has to travel to reach in time, where t is the actual time. s × (t + 1) (a)

(t + 2) s × ( t + 2)

s× t ( t + 2)



(b)



(d) s ×

(c) t

t (t + 1)

49. The speed of a bus during the second hour of its journey is twice that in the first hour. Also, its speed during the third hour is two-third the sum of its speeds in the first two hours. Had the bus travelled for three hours at the speed of the first hour, it would have travelled 120 km less. Find the average speed of the bus for the first three hours. (a) 100 kmph (c) 70 kmph

(b) 80 kmph (d) 60 kmph

50. P beats Q by 5 seconds in a race of 1000 m and Q beats R by 5 meters in a race of 100 m. By how many seconds does P beat R in a race of 1000 m? (a) 5 sec (c) 10 sec

(b) 7 sec (d) Cannot be determined

ANSWERS 1. (d)

6. (c)

11. (c)

16. (c)

21. (b)

26. (d)

31. (d)

36. (b)

41. (c)

46. (b)

2. (a)

7. (d)

12. (b)

17. (c)

22. (d)

27. (b)

32. (c)

37. (a)

42. (a)

47. (c)

3. (a)

8. (c)

13. (d)

18. (b)

23. (d)

28. (a)

33. (b)

38. (b)

43. (d)

48. (c)

4. (a)

9. (a)

14. (d)

19. (a)

24. (c)

29. (c)

34. (a)

39. (d)

44. (c)

49. (a)

5. (b)

10. (b)

15. (a)

20. (d)

25. (a)

30. (d)

35. (c)

40. (b)

45. (b)

50. (d)

EXPLANATIONS AND HINTS 300 300 1. (d) Time taken to cover first 300 m = = = 20 s Total distance = 800 m 54 ×(5 / 18) 15 800 18 1600 300 300 × = = 64 km/h So the Average speed = = = = 20 s 45 5 25 54 ×(5 / 18) 15 500 2. (a) We know that the speed of the policeman Time taken to cover next 500 m = 1 72 ×(5 / 18) = km/min 6 500 1 = = 25 s Speed of the thief = km/min 20 9 So the total time = 45 s

Chapter 01-8.indd 640

22-08-2017 07:01:34

EXPLANATIONS AND HINTS     641

Policeman gains =

1 1 1 1000 500 Then, − = km/min = m/min = m/min 6 9 18 18 9

60 × x + 75 × (x − 1) = 600

500 1 1000 km/min = m/min = m/min 9 8 18

⇒ 60x + 75x = 675

500 To gain 500 m, time required = min = 9 min 500 / 9 1 Hence, the thief has gone ahead by 9 × = 1 km 9 3. (a)

⇒ 125x = 675 ⇒x= Hence, they meet after

27 h 5

27 h. 5

We know that, 13 =5 Speed upstream and

28 =5 Speed downstream

13 Thus, speed upstream = km/h and speed down5 28 stream = km/h 5 1 28 13 1 15 Speed of the river = − = × = 1.5 km/h 2 5 5 2 5





4. (a) In each case, train has to travel its own length to pass each man. Let the speed of the train is x km/h, then Length = Relative speed × Time 8 = (x − 3) × 60 × 60 10 Also, length = (x − 4) × 60 × 60 Now, length in both cases remains the same. So (x − 3) ×

So the total distance from Delhi when they meet 27 = × 60 = 27 × 12 = 324 km 5 3 7. (d) Given that Jack is 1 times faster than Jeremy. 4 Speeds of Jack and Jeremy are in the ratio 3 1 :1= 7 : 4 4 In a race of 7 m, A gains 3 m over B. 7 So to gain 90 m, race must be of 90 × = 210 m 3 8. (c) Let the speed of the runner be x km/h. Length = Relative speed × Time = (40 − x) × 6 Similarly, Length = (40 + x) × 4 Solving the above equations, we get (40 − x) × 6 = (40 + x) × 4 ⇒ 80 = 10 x

8 10 = (x − 4) × 60 × 60 60 × 60

⇒ 4x − 12 = 5x − 20 ⇒ x = 8 km/h Thus, the speed of the train is 8 km/h. 5. (b) As already calculated, the speed of the train is 8 km/h. Also, Length = Relative speed × Time 8 = (8 − 3) × km 60 × 60 5×8 = × 1000 m = 11.11 m 60 × 60 So the length of the train = 11.11 m

⇒x=8 Thus, the speed of the runner = 8 km/h Length =

36 × 6 × 1000 6000 = m = 60 m 3600 100

Thus, the length of the train = 60 m. 9. (a) Given that A runs 5 rounds and B runs 4 rounds. A passes B every time in 5 ×

1 5 = km 2 2

Hence, A passes B only once. 10. (b) Let the length of the platform is x m. We know that the length of the train = 100 m Time taken = 20 s

6. (c) Let the two trains meet after x h.

Chapter 01-8.indd 641

5 m/s = 20 m/s 18

Speed of the train from Delhi = 60 km/h

Speed of the train =72 km/h = 72 ×

Speed of the train from Jammu = 75 km/h

Relative speed = 20 m/s (since platform is stationary)

22-08-2017 07:01:38

642     Speed, Distance and Time 

Hence,

Subtracting Eq. (2) from Eq. (1), we get 100 + x = 20 20 ⇒ x = 400 − 100 = 300 m

x x 6 2 − = − 10 20 60 60 4 4 ⇒x= × 20 = km 60 3

11. (c) Let the speed of the boat in still water is x m/s. Speed of the boat while going upstream = (x − 5) m/s Speed of the boat while going downstream = (x + 5) m/s Since distance remains same in both the cases, we have (x + 5) × 6 = (x − 5) × 12 ⇒ x + 5 = 2x − 10 ⇒ x = 15

Total distance = 10 km 15. (a) Let the distance between office and home is x km. Then x x + =8 5 3 8x ⇒ =8 15 ⇒ x = 15 km Therefore, total distance = 15 km

So the speed of the boat in still water = 15 m/s 16. (c) Let the speed of the boat in still water be x km/h and speed of the stream be y km/h.

12. (b) Let the initial speed be x. New speed =

5x 4

4y Now, if the initial time was y, time becomes . 5 Hence, 4y y− =5 5 ⇒ y = 25

(i)



12 = 4 ⇒ x−y = 3  x−y

(ii)



28 = 4⇒ x+y =7 x+y Adding the above two equations, we get 2x = 10 ⇒x=5

Thus, the initial time taken to reach the destination = 25 min

Hence, the speed of boat in still water = 5 km/h 13. (d) We know that 17. (c) Let x is the length of the faster train. Total distance Average speed = Total time Let the distance from A to B = y

Now, relative speed = 60 km/h − 30 km/h = 30 km/h 25 = m/s 3

Total distance = 3y

Time = 9 s

Total time =

So,

y y y + + x1 x2 x2

So the average speed =

=

Time =

3y (y x1 ) + (y x2 ) + (y x2 )

⇒9=

x 25 3

3x1x2 3 = (1 x1 ) + (2 x2 ) 2x1 + x2

⇒x=

25 × 9 = 75 m 3

14. (d) Let distance from house to college be x km. Hence 6 x = t+  (i) 10 60 Also,

Chapter 01-8.indd 642

Distance Relative speed

2 x = t+  20 60

So the length of the faster train = 75 m 18. (b) Given that the average speeds of the trains are 60 km/h and 40 km/h. To cover a distance of 40 km, the slower train will take 60 min.

(ii) The faster train will cover that distance in 40 min.

22-08-2017 07:01:41

EXPLANATIONS AND HINTS     643

Hence, for the faster train to cover the same distance as the slower train, total time of stoppage = 20 min. Now, since each stoppage is of 2 min, total number of stoppages = 10.

23. (b) Let the distance between the two stations be x km. I. Then, speed of the mail train = (y + 12) km/hr II.

x x 40 − = y (y + 12) 60

Thus, even I and II together do not give x. 19. (a) Let the length of the moving train be x and speed be y. To cross a pole, Length of train Speed of train x ⇒5= y

I. Distance from A to B = 125% of C to B.

Time =

125 × (100 − x) 100 100 × 100 ⇒ (100 − x) = = 80 125 ⇒ x = 20 ⇒ 100 =

Also, to cross a stationary train, 12 =

x + 350 y

⇒ 12 = 5 +

Thus, distance from A to C is 20 km. Hence, I alone can give the answer. 1 II. Distance from A to C = (Distance from C to B) 4 1 x = (100 − x) ⇒ 5x = 100 ⇒ x = 20 4

350 y

⇒ y = 50 km/h Hence, the speed of the moving train is 50 km/h.

4x 5

Now, if the initial time was y, time becomes

Thus, distance between A to C = 20 km. Hence, II alone can give the answer.

20. (d) Let the initial speed be x. So the new speed =

24. (c) Let distance from A to C be x. Then distance from C to B = (100 − x).

25. (a) I. Relative speed = 135 km/hr 5y 4

Hence, 5y −y = 4 4 y ⇒ =4 4 ⇒ y = 16 Thus, the initial time taken to reach the destination is 16 min. 21. (b) Time taken by a person = 5 minutes = 300 seconds

 

600 Speed of the person = = 2 m/s 300

500 hrs. 135 II. does not give the relative speed. Thus, time taken =

Thus, I alone gives the answer and II is irrelevant. 26. (d) Since ratio of speed of X:Y is 3:4, then ratio of time will be 4:3. I. If Y takes 3 min, then X takes 4 min.

 3 × 36 min 4

II. If Y takes 36 min, then X takes = 48 min.

Thus, I and II together give the answer. 27. (b) Let the speed of the belts be a and that of the trolley be b. Let PQ = x and QR = y.

Converting speed from m/s into km/h, we get



= 2×



18 = 7.2 km/h 5

22. (d) Distance = (240 × 5) = 1200 km We know that, speed = distance/time 1200 Speed = = 720 km/h 5/3

Chapter 01-8.indd 643

x y + a +b b−a y x Time taken for trolley to cover RP = + a +b b−a  1 1   = 64 s + Total time = (x + y )   a + b b − a  Time taken for cart to cover PR =

 22 − 0.52   × 64 = 60 Hence, x + y =    2 × 2 

22-08-2017 07:01:44

644     Speed, Distance and Time 

28. (a) Let the distance between X and Y be d km In the first instance distance travelled by A = d + 11 In the first instance distance travelled by B = d − 11 Also, the time taken by both is same. ⇒

d + 11 d − 11 = A B ⇒



A d + 11  = B d − 11

(i)

In the second instance, distance travelled by B = d + 9 while distance travelled by C = d — 9 B d+9 ⇒  = C d−9



(ii)

31. (d) Both the circular paths have the maximum possible radius, hence both have a radius of 3 cm each. Irrespective of the angle between the planes of their circular paths, the two cockroaches will meet again, at the point Q only, which is at a diametrically opposite end of P. A reaches point Q after 9 seconds. On the other hand, B would have completed 3/2 of his revolution and it will also reach point Q after 9 seconds. 32. (c) Arpit and Gaurav travel for the same amount of time till the time they meet between x and y. So, the distance covered by them will be the same as the ratio of their speeds. Let the time that they have taken to meet each other be x hours from the time they have started.

From Eq. (1) and (2), we get A A B 3 d + 11 d + 9 3 = × = ⇒ = × C B C 2 d − 11 d − 9 2

d 2 + 20d + 99) 3 ( = ⇒ (d2 − 20d + 99) 2 ⇒ 2 (d 2 + 20d + 99) = 3 (d 2 − 20d + 99) ⇒ d 2 − 100d + 99 = 0 ⇒ (d − 1)(d − 99) = 0 Hence, d = 1 or 99. Since, d = 1 is not possible since distance travelled by B cannot be negative, we get d = 99. 29. (c) A and B run around a circular track. A laps B in the middle of the 5th lap, i.e. when A has run four and a half laps, he has covered a distance which is 1 lap greater than that covered by B’s. 9 7 Therefore, when A runs laps, B runs laps. 2 2 This is same as saying when A runs 9 laps, B runs 7 laps, i.e. in a race that is 9 laps long, A can give B a start of 2 laps. So, if the race is of 3000 m long, then A can give B 2 × 3000 = 666.67 m a start of 9



Therefore, to cover the entire distance, Arpit would take x + 4 hours and Gaurav would take x + 9 hours. Ratio of time taken (Arpit:Gaurav) = x + 4 : x + 9 Hence, ratio of speeds of Arpit and Gaurav = x+4 x+9:x+4 =1: x+9 By the time Arpit and Gaurav meet, Arpit would have travelled = 48x km. After meeting, Gaurav takes 9 hours to cover this distance. 48x Hence, Gaurav’s speed = km/h. 9 But we know that the ratio of Arpit and Gaurav x+4 speeds are 1 : x+9 Therefore, 48 :

48x x+4 x x+4 :: 1 : ⇒ = 9 x+9 x+9 9

⇒ x2 + 9x = 9x + 36 ⇒ x2 = 36 ⇒x=6 48x 48 × 6 = = 32 Hence, speed of Gaurav = 9 9 kmph 20 mph. 33. (b) Let x km be the distance between his house and office.

30. (d) Let the length of the race be d. When B finished the race, A and C would have run (d − 36) and (d − 24) m, respectively. When C finishes the race, A would have run (d − 16) m. The ratio of speeds of C and A is given by d − 24 d = ⇒ d = 96 m d − 36 d − 16

Chapter 01-8.indd 644

Thus, while travelling at 24 kmph, he would take x hours. 24 While travelling at 25% faster speed, i.e. 24 + 25% 1 pf 24 = 24 × = 30 kmph, time taken would be 4 x hours. 30



22-08-2017 07:01:47

EXPLANATIONS AND HINTS     645

We are given that, time difference = 5 min late + 4 min early = 9 min. x x 6x 9 ⇒ − =9⇒ = ⇒ x = 9 × 2 = 18 24 30 24 × 30 60 34. (a) Time taken by Sunil for 1st round = 2min nd

2 3rd 4th 5th 6th 7th 8th

The distance covered in one revolution = circumference of the wheel. Circumference = 2p r = 2 ×

22 55 22 ×r ⇒ = 2× ×r 7 28 7

r = 31.25 cm

round = 4min round = 8 min round = 16 min round = 32 min round = 64 min round = 128 min round = 256 min

38. (b) Distance between the rescue vessel and the ship, which is 12 km has to be covered in 16 ­minutes. (The ship can stay afloat only 20 minutes and it takes 4 minutes to evacuate the people aboard the ship). Therefore, the two vessels should move towards each other at a speed of

We are given that, Anil tales one minute for every round. He meets 127 times in 7th and 255 times in 8th round. Thus, total time Anil and Sunil meet = 127 + 255 = 382.

12 12 × 60 = = 45 km/h  16  16    60  The ship is moving at a speed of 8 km/h. Therefore, the rescue vessel should move at a speed = 45 − 8 = 37 km/h.

35. (c) Speed of the river = 12 kmph. Speed of the boy = 5 kmph



1 Let the time taken by the boy to cross the river in × 32.2 × 52 = 16.1 × 25 = 402.5 39. (d) In 5 seconds, the object travels = 2 1 still water = t. × 32.2 × 52 = 16.1 × 25 = 402.5 2 Width of the river = 5t 1 In first 3 seconds, the object travels = × 32.2 × 32 = 16.1 × 9 = 144.9 The boy takes the least time when he is travel2 1 ling directly across the river. But the river current × 32.2 × 32 = 16.1 × 9 = 144.9 2 pushes him in a direction perpendicular to the flow. Hence, in the last 2 seconds it travelled = Distance travelling along the river = 12t 402.5 − 144.9 = 257.6 feet. Effective distance travelled by the boy = 40. (b) If a man is travelling with 3/4 of his usual 2 2 (12t) + (5t) = 13t speed, then he takes 4/3 of his usual time to cover the same distance. Distance covered by the boy 13t ⇒ = = 2.6 4 1 Width of the river 5t Hence, extra time taken = − 1 = 3 3 36. (b) We are given that the faster runner is thrice as We are given that, 1/3 of time is 20 minutes. fast as the slowest runner. Hence, the faster runner would have completed three rounds by the time Usual time = 20 × 3 = 60 minutes the slowest runner completes one round. 100 Also, their first meeting takes place after the fast 41. (c) Abhishek’s speed = = 20 m/s 5 est runner takes 4 min to complete one and the half





round. Thus,



 3600 × 4 = 24 minutes to finish Hence, he takes  600  the race.

Time taken by Abhishek to cover 500 m = 500 = 25 seconds 20 500 Dishu’s speed = = 25 m/s 20 Prashant’s speed = 15 m/s

37. (a) The distance covered by the wheel in 560 revolutions = 1100 m 1100 55 = Hence, the distance covered per revolution = 560 28 1100 55 meters = 560 28

Time taken by Dishu to meet Prashant in 500 m 500 race in opposite direction = = 12.5 s (15 + 25)

3 400 × = 600 m 2

Chapter 01-8.indd 645

Distance covered by Abhishek = 12.5 ×(20) = 250 m

22-08-2017 07:01:51

646     Speed, Distance and Time 

42. (a) Let the distance to be covered by the boat when it is travelling against the stream be x. The boat goes down the river at a speed = 20 + 4 = 24 km/h

Since, they have met at the same time, they would have travelled for the same time. Hence,

and up the river at a speed = 20 − 4 = 16 km/h. Since the time taken is same, 120 x = 24 16 Therefore, x = 80 km. 43. (d) Let x be the number of hours he travels at 20 kmph. Distance covered in this x hours = 20x km. He would have therefore, travelled the rest (8 − x) hours at 70 kmph.

x (54 − x) = ⇒ x = 22.5 km 5 7

47. (c) Let r = running time of the train, s = stopping time of the train and d = total distance travelled by train. We have, d d = 60, = 40 r r+s (r + s) 3 s 1 ⇒ = ⇒ = 2 2 r r As, d = 300 km

300 = 60 ⇒ r = 5 hrs and s = 2.5 hours. Distance covered in this time = (8 − x) × 70 = (560 − 70x) r (8 − x) × 70 = (560 − 70x) km 48. (c) Assuming x as the actual speed. The total distance travelled = 20x + 560 − 70x = 400 ⇒ 160 = 50x ⇒x=

 50  160

Then, p p = t+2 = t and x s So,

x × t = s × (t + 2)

Thus, x = 3.2 hours = 3 hours and 12 minutes.

⇒x=

t

44. (c) We know that the total time of journey = 5 hours. Let x hours be the time that I travelled at 40 kmph. Therefore, 5 − x hours would be time that I travelled at 60 kmph. Hence, I would have covered in 5 hours =

(x × 40) + (5 − x) 60 = 240 ⇒ 300 − 20x = 240 ⇒ x = 3 hours 45. (b) Let the length of the train be L, x be the speed of the first man, y be the speed of the second man and z be the speed of the train. Thus, we can write 1 1 20 = and 18 = (z − x) (z + y ) ⇒ z = 10x + 9y

s × (t + 2)

49. (a) Let the speed for the first hour be s kmph. Speed during the second hour = 2s kmph 2 Speed in the third hour = × (3s) = 2s kmph 3 Distance travelled = 5s km Had it travelled at s kmph, distance travelled would have been equal to 3s kmph. We are given that, 5s − 3s = 120 ⇒ 2s = 120 ⇒ s = 60 Thus, average speed for the first three hours = 5s 5 × 60 = = 100 kmph 3 3

Distance between the two men = 600(z + y) m

50. (d) If P takes t seconds to run 1000 m, then Q 600 (z + y ) − 600 (z + y ) 600 (9x + 9y ) = 90 Time = = takes (t + 5) seconds. (x + y ) (x + y ) ) − 600 (z + y ) 600 (9x + 9y ) = 90 minutes = In (t + 5) s, Q runs 100 m whereas R runs only x + y) (x + y ) 950 m. 46. (b) Let the distance be x km from A. 1000 × (t + 5) sec R runs 1000 m in So total distance travelled by P = x at a speed of 950 5 km/h. P takes t seconds to cover 1000 m, while R takes Total distance travelled by Q = 27 + (27 − x) = 20t 100 + sec. (54 − x) at a speed of 7 km/h. 19 95 (54 − x) x . Total time taken by P = and by Q = Thus, from the given data we cannot determine t. 5 7



Chapter 01-8.indd 646



22-08-2017 07:01:56

UNIT 2: ALGEBRA

Chapter 02-1.indd 647

Chapter  1.

Permutation and Combination

Chapter  2.

Progression

Chapter  3.

Probability

Chapter  4.

Set Theory

Chapter  5.

Surds, Indices and Logarithms

25-08-2017 01:22:26

Chapter 02-1.indd 648

22-08-2017 07:02:34

CHAPTER 1

PERMUTATION AND COMBINATION

PERMUTATION The notion of permutation is used with several slightly different meanings, all related to the act of  permuting (rearranging) objects or values. Informally, a permutation of a set of objects is an arrangement of those objects in a particular order. When doing permutation, the order of the items is important. For example, permutations of three items a, b, c are ab, ba, ac, ca, bc and cb. The number of permutations of n things taking r at a time is denoted by nPr, and the expression is n

Pr =

n! (n + r)!

y of the things are alike of one kind and z are rest of the things that are alike, is given by n

Pr =

n! x!y !z !

3. The number of ways of arranging n distinct objects along a round table is given by (n − 1)! 4. The number of ways of arranging n persons along a round table so that no person has the same two neighbors is given by 1 (n − 1)! 2

Some other formulae on permutations are as follows: 1. The number of permutations of n different things taken r at a time is given by n

Pr = n r

2. The number of permutations of n things taken all together, when x of the things are alike of one kind,

Chapter 02-1.indd 649

COMBINATION A combination is a way of selecting several things out of a larger group, where (unlike permutations) order does not matter. For example, combinations of three items a, b, c are ab, ac and bc.

22-08-2017 07:02:36

650     Permutation and Combination 

Number of combinations of n things taking r at a time is denoted by nCr, and the expression is n

Cr =

n! r ! (n − r)!

Some other formulae on combinations are as follows: 1. The number of ways in which a + b things can be divided into two groups containing a and b things, respectively, is

subset containing n2 elements from the remaining n − n1 elements and so on until no elements remain. This procedure yields a partition of S into r subsets, with the pth subset containing exactly np elements. Now we wish to count the number of such partitions. n! We know that there are ways to form the n ! (n − n1 ) ! first subset. Hence, we can n ( − n1 −  − np−1 ) ! np ! (n − n1 −  − np ) !

(a + b)! a +b Cb = a !b ! 2. The number of ways in which a selection can be made out of a + b + c things of which a are alike of one kind, b are alike of another kind and remaining c are different is given by

formulate

that

there

are

ways to form the pth subset.

Using the counting principle, the total number of partin! . tions is then given by n1 ! n2 !… nr ! This expression is called a multinomial coefficient.

(a + 1)(b + 1)2c − 1

COUNTING

3. The number of ways of arranging n distinct objects along a round table is given by (n − 1)! 4. The number of ways of arranging n persons along a round table so that no person has the same two neighbors is given by 1 (n − 1)! 2

There are two fundamental principles of counting, which are principle of addition and principle of multiplication. The two principles form the base of permutations and combinations.

Fundamental Principle of Addition If there are two tasks such that they can be performed independently in m and n ways, respectively, then either of the two jobs can be performed in (m + n) ways.

Partitions A combination is nothing but portioning a set into two subsets, one containing k objects and the other containing the remaining n − k objects. Hence, in general the set S = {1, 2, …, n} can be partitioned into r subsets. Consider the following iterative situation leading to partition of S. We first select a subset of n1 elements from S. Having chosen the first subset, we select second

Fundamental Principle of Multiplication If there are two tasks such that one of them can be ­completed in m ways and when it has been completed the second task can be completed in n ways, then the two tasks in succession can be completed in (m × n) ways.

SOLVED EXAMPLES 1. How many words can be formed out of the letters of the word “UNITED” taking all the letters at a time considering no letter is to be repeated? Solution:  The word “UNITED” consists of 6 different letters. Hence the required number of  permutations = 6 P6 = 6! = 6 × 5 × 4 × 3 × 2 × 1 = = 720 720

Chapter 02-1.indd 650

2. From a group of 8 men and 4 women, 4 persons are to be selected to form a committee so that at least 2 women are there on the committee. In how many ways can it be done? Solution:  We may have (2 men and 2 women) or (1 man and 3 women) or (0 man and 4 women).

22-08-2017 07:02:37

SOLVED EXAMPLES     651

Therefore, the required number of ways is ( C2 × C2 ) + ( C1 × C3 ) + ( C4 ) 8

4



8

4

4



8×7 4×3! × + (8 × 4) + (1) 2 4 = 168 + 33 + 1 = 201 =

3. How many words can be formed from the letters of the word “COMMUTE” taking all the letters at a time considering no letter is to be repeated and the first and last letter of the words are “M”? Solution:  The word “COMMUTE” has two M’s. The first and the last letter should be M. The remaining letters are C, O, U, T, E. So, the number of permutations is P5 = 5 × 4 × 3 × 2 ×1 = 120

5

4. In how many ways 6 prizes can be given to  10 students when a student may receive at most two prizes? Solution:  The first prize can be given in 10 ways as any student can receive it. Also, since a student may receive at most two prizes, the second prize can be given in 10 ways. Now, the third prize can be given in 9 ways since a student cannot get more than two prizes. Similarly, the fourth prize can be  given in 9 ways. The fifth and sixth prize can  be given in 8 ways each. Hence, the total number of ways in which the prizes can be given to 10 students when no student can get more than two prizes = 10 × 10 × 9 × 9 × 8 × 8 = 518400

7. Mary has 4 bananas, 6 oranges and 5 apples. How many selections of fruits can be made by selecting at least one of them? Solution:  Five apples can be selected in 6 different ways, that is, we can choose 1, 2, 3, 4, 5 or none of the apple. Similarly, 4 bananas can be selected in 5 different ways and 6 oranges can be selected in  7 different ways. Therefore, Number of ways = 6 × 5 × 7 = 210 This also includes the case in which none of the fruits is selected. Rejecting this case, we get required number of ways = 210 − 1 = 209. 8. In how many ways can the word “APOCALYPSE” be arranged such that all the vowels come together? Solution:  The word “APOCALYPSE” has 10 letters. Now, all the vowels are supposed to come together, we consider all the vowels “A, O, A, E” as one. Hence, number of ways this can be arranged in (7 !/ 2 !) (since 4 vowels are treated as 1, the number of arrangements become 7!, and because “P” is repeated twice). Also, the vowels can be arranged in 4! number of ways within themselves but the vowel “A” is repeated twice. Hence, the total number of arrangements of the word “APOCALYPSE” such that all the vowels come together is 7 ! 4! × = 7 ! × 3! 2! 2!

5. How many natural numbers less than 1000 can be formed with the digits 1, 2, 3, 4 and 5, considering repetition of digits is not allowed? Solution:  The numbers can be of 1, 2 or 3 digits. The numbers can be formed using the digits 1, 2, 3, 4, 5. So the total number of natural numbers less than 1000 using 1, 2, 3, 4, 5 is

= 5040 × 6 = 30240 9. If

n −1

P3 : nP4 = 1 : 6, then what is the value of n?

Solution:  We have = 5 + 5×4 + 5×4×3 = 5 + 20 + 60 = 85 6. In how many ways can 5 beads of different colors form a necklace? Solution:  Here, we are to form circular permutations of 5 things taken all at a time. This can be done in (5 − 1)! ways. But in case of a necklace, clockwise and anti-clockwise, permutations are same. Hence, the required number of ways is 1 1 1 (5 − 1)! = 4 ! = × 24 2 2 2 = 12

Chapter 02-1.indd 651

n −1

P3 : nP4 = 1 : 6



(n − 1)! (n − 4)! 1 = × (n − 1 − 3)! (n)! 6



(n − 1)! (n − 4)! 1 = × (n − 4)! (n)! 6



(n − 1)! 1 = n! 6



(n − 1)! 1 = n ⋅ (n − 1)! 6

⇒n = 6

22-08-2017 07:02:39

652     Permutation and Combination 

10. How many triangles can be formed by joining the vertices of an octagon? Solution:  Total octagon = 8.

number

of

vertices

in

ways. Thus, the total number of triangles that can be formed is

an  8

C3 =

8! 8×7 ×6 = = 56 3 !× 5 ! 3×2

We know that a triangle is formed by joining three vertices. Hence, this can be done in 8C3 number of

PRACTICE EXERCISE 1. How many numbers are there between 100 and 1000 in which all the digits are distinct? (a) 1000 (c) 729

(b) 810 (d) 648

2. How many words can be formed out of the letters of the word “UNITED” taking all the letters at a time considering no letter is to be repeated? (a) 120 (c) 216

(b) 720 (d) 600

3. From a group of 6 men and 4 women, four persons are to be selected to form a committee so that at least 2 men are there on the committee. In how many ways can it be done? (a) 172 (c) 185

(b) 190 (d) 216

4. How many words can be formed from the letters of the word “APPLE” taking all the letters at a time considering no letter is to be repeated and the first and last letter of the words are “P”? (a) 6 (c) 36

(b) 120 (d) 12

5. Out of 6 consonants and 3 vowels, how many words can be formed considering exactly 3 consonants and exactly 2 vowels? (a) 7200 (c) 60

(b) 3600 (d) 120

6. Six candidates are to be examined, 2 in Electronics and the remaining in different disciplines. In how many ways can they be seated in a row so that the two examinees in Electronics may not sit together? (a) 240 (c) 360

Chapter 02-1.indd 652

(b) 480 (d) 720

7. How many words can be formed with the letters of the word “SQUARE” considering all the words start with “S” and end with “E”? (a) 720 (c) 72

(b) 15 (d) 24

8. In how many ways 4 prizes can be given to 5 students when any student may receive any number of prizes? (a) 320 (c) 625

(b) 120 (d) 5

9. How many natural numbers less than 1000 can be formed with the digits 1, 2, 3, 4 and 5, considering repetition of digits is allowed? (a) 200 (c) 155

(b) 125 (d) 120

10. Simon has 5 apples, 3 bananas and 6 oranges. In how many ways can they be arranged in a row? (a) 90 (c) 172880

(b) 168168 (d) 272800

11. In how many ways can 6 beads of different colors form a necklace? (a) 60 (c) 120

(b) 100 (d) 720

12. If nC3 = nC5, then what is the value of n? (a) 10 (c) 6 13. If

n

(b) 12 (d) 8

Cr : nCr +1 = 1 : 2 and

n

Cr +1 : nCr + 2 = 2 : 3,

then what is the value of n and r? (a) 9, 2 (c) 14, 4

(b) 11, 3 (d) 17, 5

22-08-2017 07:02:40

EXPLANATIONS AND HINTS     653

14. Roger has 4 dark chocolates, 3 milk chocolates and 5 caramel chocolates. How many selections of chocolates can be made by selecting at least one of them? (a) 119 (c) 150

17. In how many ways can the word “CONSTANTINE” be arranged such that all the consonants come together? (a) 25200 (c) 50400

(b) 149 (d) 120

15. A man has 7 relatives, 3 men and 4 women and his wife also has 7 relatives, 4 men and 3 women. In how many ways can they invite a dinner party  of 4 women and 4 men so that there are 3 of man’s relatives and 3 of the wife’s relatives? (a) 365 (c) 625

(b) 465 (d) 485

16. In how many ways can the word “CONSTANTINE” be arranged such that all the vowels come together? (a) 64260 (c) 80640

(b) 42500 (d) 604800

18. If 2n +1Pn −1 : of n?

2n −1

Pn = 3 : 5, then what is the value

(a) 4 (c) 2

(b) 3 (d) 1

19. If 10P5 + 4 ⋅ 10P6 = k ⋅ 10P5, then what is the value of k? (a) 10 (c) 5

(b) 21 (d) 3

20. How many triangles can be formed by joining the vertices of a pentagon?

(b) 967680 (d) 72000

(a) 20

(b) 15

(c) 12

(d) 10

ANSWERS 1.  (d)

 4.  (a)



7.  (d)

10.  (b)

  13.  (c)

  16.  (c)

   19.  (b)

2.  (b)

 5.  (a)



8.  (c)

11.  (a)

  14.  (a)

  17.  (c)

   20.  (d)

3.  (c)

 6.  (b)



12.  (d)

  15.  (d)

  18.  (a)

9.  (c)

EXPLANATIONS AND HINTS 1. (d) Total numbers between 100 and 1000 considering digits are distinct

3. (c) We may have (2 men and 2 women) or (3 men and 1 women) or (4 men). Therefore, the required number of ways is

Total distinct hundredth digit = 9 (Since we cannot use 0) Total distinct tenth digit = 9 (since we cannot use the digit used at hundredth digit but can use 0) Similarly, total distinct ones digit = 8. 9 × 9 × 8 = 648 2. (b) The word “UNITED” consists of 6 different letters. Hence the required number of permutations is 6

Chapter 02-1.indd 653

P6 = 6 ! = 6 × 5 × 4 × 3 × 2 × 1 = 720

(6 C2 ×4 C2 ) + (6 C3 ×4 C1 ) + (6 C4 )



 

 

6×5 4×3 6×5×4 6×5 × + ×4 + 2 2 2 3×2 = 90 + 80 + 15 = 185 =



4. (a) The word “APPLE” has two P’s. The first and the last letter should be P. The remaining letters are A, L, E. So the number of permutations = 3 × 2 ×1 = 6 .

22-08-2017 07:02:41

654     Permutation and Combination 

5. (a) Number of ways of selecting 3 consonants out of 6 and 2 vowels out of 3 are 6

C3 ×3 C2 =

6× 5× 4 × 3 = 60 3× 2

11. (a) Here, we are to form circular permutations of 6 things taken all at a time. This can be done in (6 − 1)! ways. But in the case of a necklace, clockwise and anti-clockwise permutations are same. Hence, the required number of ways is

Number of groups, each having 3 consonants and  2 vowels = 60. Each group contains 5 letters.

1 1 1 (6 − 1)! = 5 ! = × 120 = 60 2 2 2

Number of ways of arranging 5 letters among themselves = 5 ! = 5 × 4 × 3 × 2 × 1 = 120

12. (d) We have

Therefore, required number of ways = 60 × 120 = 7200

n

⇒ C(n, n − 3) = C(n, 5) Since, C (n, r) = C (n, n − r) ⇒ n− 3 = 5 ⇒n=8

6. (b) The total number of ways in which 6 candidates can sit = 6 P6 = 720. When two candidates of Electronics sit together, we can consider them as one candidate. Now, total candidates become 5. They can be seated in 5 P5 ways. Two Electronic students can arrange themselves in 2! ways. Therefore, number of ways in which Electronics students sit together = 120 × 2 = 240. Hence, number of ways in which two candidates in Electronics do not sit together = 720 − 240 = 480. 7. (d) The word “SQUARE” has 6 different letters. Since, the first letter of every word is “S” and last letter of every word is “E.” So the total numbers of ways the rest of the four words can be arranged = 4 × 3 × 2 × 1 = 24. 8. (c) First prize can be given in 5 ways. Since, a  student can receive any number of prizes.

13. (c) We have n

Cr :n Cr +1 = 1 : 2 n



n

9. (c) The numbers can be of 1, 2 or 3 digits. The numbers can be formed using the digits 1, 2, 3, 4, 5.  The total number of natural numbers less than 1000 using 1, 2, 3, 4, 5 is 5 + 5 ×5 + 5 ×5 ×5 = 5 + 25 +125 = 155 10. (b) Total number of fruits = 14. Five are of one kind (apples), three are of second kind (bananas) and six are of third kind (oranges). Hence 14 ! 14 × 13 × 12 × 11 × 10 × 9 × 8 × 7 × 6 ! = (5 × 4 × 3 × 2) × (3 × 2) × 6 ! 5! ×3! ×6! = 168168

Chapter 02-1.indd 654

Cr

=

Cr +1

1 2

(r + 1)!(n − r − 1)! 1 n! = × r !(n − r)! n! 2 (r + 1) 1 = ⇒ n−r 2 ⇒ 2r + 2 = n − r  n − 3r = 2 ⇒

n

(i)

Cr +1 : n Cr + 2 = 2 : 3 n

The number of ways, the 4 prizes can be distributed = 5 × 5 × 5 × 5 = 625.

C3 = n C5



n

Cr +1 Cr + 2

=

2 3

(r + 2)!(n − r − 2) ! n! × n! (r + 1)!(n − r − 1)! (r + 2) 2 ⇒ = (n − r − 1) 3 ⇒ 3r + 6 = 2n − 2r − 2  2n − 5r = 8

(ii)

Solving Eqs. (1) and (2), we get 2(2 + 3r) − 5r = 8 ⇒ 4 + 6r − 5r = 8 ⇒r=4 n = 2 + 3(4) = 14 Therefore, value of n and r = 14, 4.

22-08-2017 07:02:43

EXPLANATIONS AND HINTS     655

14. (a) Four dark chocolates can be selected in 5 different ways, that is, we can choose 1, 2, 3, 4 or none of the dark chocolate. Similarly 3 milk chocolates can be selected in 4 different ways and 5 caramel chocolates can be selected in 6 different ways. Therefore, the number of ways 6 × 5 × 4 = 120 This also includes the case in which none of the chocolates is selected. Rejecting this case, we get required number of ways = 120 − 1 = 119. 15. (d) Let M and W denote men and women, respectively. Under the given restrictions of the problem, the different possibilities are (a) 3W from man’s relatives and 3M from wife’s relatives. (b) 2W, 1M from man’s relatives and 1W, 2M from wife’s relatives. (c) 1W, 2 M from man’s relatives and 2W, 1M from wife’s relatives. (d) 3M from man’s relatives and 3 W from wife’s relatives.

17. (c) The word “CONSTANTINE” has 11 letters. Now, since all the vowels are supposed to come together, we consider all the vowels “C, N, S, T, N, T, N” as one letter. Hence, number of ways this can be arranged is 5!. Also, number of ways in which the consonants can be arranged within themselves will be 7! 2! ×3! Hence, the total number of arrangements of the word “CONSTANTINE” such that all the consonants come together will be 7! ×5! 2! ×3! = 7 ! × 10 = 50400 18. (a) We have 2n −1

2n +1

Pn −1 :

Pn = 3 : 5

= 4 × 4 + 6 × 3 × 3 × 6 + 4 × 3 × 3 × 4 +1×1

(2n + 1)! (n − 1)! 3 = × (n + 2)! (2n − 1)! 5 (n − 1)! (2n + 1)(2n)(2n − 1)! 3 ⇒ × = (n + 2)(n + 1)(n)(n − 1)! (2n − 1)! 5 2(2n + 1)! 3 ⇒ = (n + 2)(n + 1) 5 ⇒ 10(2n + 1) = 3(n + 2)(n + 1)

= 16 + 324 + 144 + 1 = 485

⇒ 20n + 10 = 3n2 + 9n + 6

Therefore, the required number of ways of inviting a dinner party 4

C3 × 4C3 + 4C2 × 3C1 × 3C1 × 4C2 + 4C1 × 3C2 × 3C2 × 4C1 + 3C3 × 3C3

16. (c) The word “CONSTANTINE” has 11 letters. Now, since all the vowels are supposed to come together, we consider all the vowels “O, A, I, E” as one.

⇒ 3n2 − 11n − 4 = 0 ⇒ (n − 4)(3n + 1) = 0 ⇒n=4 19. (b) We have

Hence, the number of ways this can be arranged is 8! 2! ×3! (since 4 vowels are treated as 1, the number of arrangements become 8!, and because “N” is repeated thrice and “T” is repeated twice.)

10

P5 + 4 ⋅ 10P6 = k ⋅ 10P5

Now, P5 + 4 ⋅ 10P6

10

10 ! 10 ! +4⋅ 5! 4! 1  21  1  = 10 !  +  = 10 !   5 ! 3 !   120  =

Also, the vowels can be arranged in 4! number of ways within themselves. Hence, the total number of arrangements of the word “CONSTANTINE” such that all the vowels come together will be 8! ×4! 2! ×3! = 8 ! × 2 = 80640

Chapter 02-1.indd 655

 21  = 10 !    5 !  10 ! = 21 × 5!

22-08-2017 07:02:44

656     Permutation and Combination 

Equating the above expression with right hand side, we get 10 ! 10 ! 21 × = k× 5! 5! Therefore, k = 21

ways. Thus, the total number of triangles that can 5 be formed is C3 5! 3 !× 2 ! = 5 × 2 = 10 =

20. (d) Total number of vertices in a pentagon = 5 We know that a triangle is formed by joining three vertices. Hence, this can be done in 5C3 number of

Chapter 02-1.indd 656

22-08-2017 07:02:45

CHAPTER 2

PROGRESSION

ARITHMETIC PROGRESSION

GEOMETRIC PROGRESSION

An  arithmetic progression  (AP)  is a  sequence  of ­numbers such that the difference between the consecutive terms is constant. This constant value is called common difference. In other words, any term of an arithmetic progression can be obtained by adding the common difference to the preceding term.

A  geometric progression  (GP) is a  sequence  of  numbers where each term after the first is found by multiplying the previous one by a fixed, non-zero number called the common ratio. In other words, any term of a geometric progression can be obtained by multiplying preceding number by the common ratio.

Let a be the first term, n be the number of terms,  d be the common difference, an be the nth term, Sn be the sum of first n terms and S be the sum of the entire sequence. Then,

Let a be the first term, n be the number of terms, r be the common ratio, an be nth term, Sn be the sum of first n terms. Then,

an = a + (n − 1) × d n × [2a + (n − 1) × d ] 2 n S = × (First term + Last term) 2

Sn =

Arithmetic progression can be represented as a, a + d,  a + 2d, …, [a + (n − 1)d]. Here, quantity d is to be added to any chosen term to get the next term of the progression.

Chapter 02.2.indd 657

an = ar(n −1) Sn =

a(1 − r n ) if r < 1 1− r

a(r n − 1) if r > 1 r −1 r × (Last term − Firsst term) S= Geometric progression can rbe − 1represented as a, ar,  ar2, …, arn. Here, a is the first term and quantity r is the common ratio of the geometric progression. Sn =

22-08-2017 07:03:23

658        pROGRESSION 

Infinite Geometric Progression If −1 < r < +1 or r < 1, then the sum of a geometric progression does not increase infinitely; it converges to a particular value. Such a GP is called infinite geometric progression. Mathematically, the sum of an infinite geometric progression is

products. It is a type of  mean  or  average, which indicates the central tendency or typical value of a set of numbers by using the product of their values. Geometric mean of n numbers x1, x2, x3, …, xn is denoted by

(∏

)

1/ n n x and i i =1

∏ 

1/ n

n

S∞

a = 1− r

xi

= n x1x2 x3  x4

i =1

HARMONIC SERIES 1. Arithmetic mean (AM): The most commonly used average is the arithmetic mean or simply the average. Arithmetic mean of n numbers x1, x2, x3, …, xn is denoted by X and calculated as X=

calculated as

3. Harmonic mean (HM): Harmonic mean is the special case of the power mean. As it tends strongly toward the least elements of the list, it may (compared to the arithmetic mean) mitigate the influence of large outliers and increase the influence of small values. Harmonic mean of n numbers x1, x2, x3, …, xn is calculated as n HM =

x1 + x2 + x3 +  + xn n

∑ i=1 xi ⇒X=



n (1/xi ) i =1

n

RELATION BETWEEN AM, GM AND HM

n If values x1, x2, x3, …, xn are the assigned weights w1, w2, w3, …, wn, respectively, then the weighted arithmetic mean is given as

∑ i=1 wixi n ∑ i=1 wi

Consider two numbers a and b. Arithmetic mean = (a + b) / 2; Geometric mean = ab;  Harmonic mean = 2ab / (a + b)

n

xw =

Therefore, (GM)2 = AM × HM

2. Geometric mean (GM): Geometric mean of n numbers x1, x2, x3, …, xn is nth root of their

Also, Arithmetic mean > Geometric mean > Harmonic mean

SOLVED EXAMPLES 1. What is the tenth term of an arithmetic progression whose fifth term is 19 and common difference d is 4? Solution:  We know that an = a + (n − 1) × d ⇒ 19 = a + (5 − 1) × 4 ⇒a=3 Now,

Therefore, subtracting the above equations, we get −76d = 76 ⇒ d = −1 Substituting this value of d, we get 86 = a + (10 − 1) × (−1) ⇒ a = 86 + 9 = 95 Now, 50th term of the AP is given as

a10 = 3 + (10 − 1) × 4 = 3 + (9) × 4 = 39 2. If in an AP, tenth term is 86 and 86th term is 10, then what will be the 50th term?

a50 = 95 + (50 − 1)(−1) = 95 − 49 = 46 3. Sum of the first 21 terms of an AP is 1071. What is the first term if the common difference is 6?

Solution:  We know that 86 = a + (10 − 1) × d 10 = a + (86 − 1) × d

Chapter 02.2.indd 658

Solution:  We know that Sn =

n × [2a + (n − 1) × d ] 2

22-08-2017 07:03:26

SOLVED EXAMPLES        659

Now,

Considering L.H.S. 2 2 2 (b − c ) + (c − a) + (d − b) = (ar − ar2 )

21 × [2a + (21 − 1) × 6 ] 2 1071 × 2 ⇒ 2a = − 120 21 ⇒ a = −18/2 = −9

2

1071 =

(

+ ar2 − a  = a2  r − r 2 

(

4. Find out the 10th term of a geometric progression and the sum if a1 = 35 and the common ratio, r = 2.

a6 = 35 × (2)

= 35 × (2) = 35 × 512 = 17920

Hence, the 10th term of a geometric sequence is 17920.

sn =

(

10

1−2

) = 35 (−1023) = 35805

5

−1

a + ar = 2 ⇒ a (1 + r ) = 2 

(

is

35 512

35 140 560 , , ,... 512 512 512

or

(1)

)

8. Find three numbers in GP whose sum is 14 and product is 64. Solution:  Let the numbers be

Using Eq. (1) and (2), we get

)

2 1 + r 2 = 20 ⇒ 1 + r 2 = 10

of the numbers =

⇒ r 2 = 9 ⇒ r = ±3

a , a and ar. Product r

a × a × ar = 64 r

a3 = 64 ⇒ a = 4

Substituting r = ±3 in Eq. (1),

Sum of then numbers = a 1 + a + ar = a + 1 + r = 14 r r



If r = 3, then a = 1/2 If r = −3, then a = −1. The geometric progression is −1, 3, −9, 27, ... 6. If a, b, c and d are in GP then prove that

(b − c ) + (c − a) + (d − b) = (a − d ) . 2

70 35 = 1024 512

If r = −4, then we get a = − Therefore, the GP 35 140 560 − ,− ,− ,... 512 512 512

a + ar + ar 2 + ar 3 = 20a (1 + r ) 1 + r 2 = 20  (2)

2

Solution:  We are given that a, b, c and d are in GP Thus, b = ar, c = ar 2 , d = ar 3 .

Chapter 02.2.indd 659

17920 ⇒ r 4 = 256 ⇒ r = ±4 70

5

Solution:  Consider the GP a, ar, ar , … We are given,

2

2

a (4) = 70 ⇒ a =

2

2

)

Solution:  We are given that the sixth and tenth terms are 70 and 17920. Thus,

ar

5. The sum of the first two terms of a G.P is 2 and the sum of the first four terms is 20. Find the GP

(

2

Substituting r = 4, we get

35 1 − 2 s10 =

)

7. The sixth and the tenth term of a GP are 70 and 17920, respectively. Find the GP

=

)

1− r

) (

ar 9

Series of the sequence,

(

2

2

2

9

a1 1 − r n

2

= (a − d ) = R.H.S.

that gives the nth term to find a10 as follows: 10 −1

2

) + (r2 − 1) + (r3 − r) 

(

(

an = a1 × r n −1

2

= a2 r 6 − 2r 3 + 1 = ar3 − a = a − ar3

Solution:  Use the formula,

) + (ar3 − ar)

(



)

2 1 + r + r 2 = 7 r ⇒ 2r 2 − 5r + 2 = 0 ⇒r=

1 ,2 2

If r = 2, the numbers are 2, 4, 8. 1 If r = , the numbers are 8, 4, 2. 2

22-08-2017 07:03:29

660        pROGRESSION 

9. Find out the 7th term of a geometric progression and the sum if a1 = 79 and the common ratio  r = 2.

6 1 1 1 1 1 1 + + + + + 8 9 6 11 10 5 6 = 7.560 H= 0.7936

So harmonic mean =

Solution:  Using the formula, an = a1 × r n −1 a7 = a1 × (2)7 −1 = 79 × (2)6 = 79 × 64 = 5056 The 7

th

term of the GP sequence is 5056.

Series of the sequence,

13. Which term of the AP 3, 15, 27, 39, … will be 132 more than its 54th term? Solution:  We are given an arithmetic series as 3, 15, 27, 39, … Here, a = 3, d = 15 − 3 = 12 Since, an = ak = (n − k) d

a (1 − r n ) Sn = 1 1− r

an − a54 = (n − 54) d ⇒ 132 = 12n − (54 × 12)

79 (1 − 27 ) 79 (−127) S7 = = = 10033 1−2 −1 10. Find out the 5th term of a geometric progression and the sum if a1 = 46 and the common ratio  r = 2. Solution:  Using the formula,

⇒ 12n = 132 + 648 = 780 ⇒ n = 65 Thus, the 65th term of the AP will be 132 more than its 54th term. 14. Find the 31st term of an AP whose 11th term is 38 and the 16th term is 73. Solution:  Let a be the 1st term and d be the common difference. We are given that,

an = a1 × r n −1 5−1

a5 = 46 × (2)

= 46 × (2) = 46 × 16 = 736 4

The 5th term of a geometric progression is 736.

S5 =

a1(1 − r n ) 1− r

a + (10 × 7) = 38 ⇒ a = 38 − 70 = −32 ⇒ a31 = a + 30d = −32 + (30 × 7) = −32 + 210 = 178 Hence, 31st term is 178.

Solution:  Given that, ai = 5, a2 = 6, a3 = 8, a4 = 12,  a5 = 2. Here n = 5. Geometric mean formula is

 

(ii)

Now, substituting d = 7 in Eq. (1), we get

46 (1 − 25) 46 (−31) = = 1426 1−2 −1

5, 6, 8, 12, 2

n

a16 = a + 15d = 73  a + 10d − 1 − 15d = 38 − 73 ⇒ −5d = −35 ⇒d=7

11. What is the geometric mean of the following set of data

∏ xi

(i)

Subtracting Eq. (2) from (1), we get

Sum of the sequence,

Sn =

a11 = a + 10d = 38 

1/n

= n a1a2a3  an = 5 5 × 6 × 8 × 12 × 2

15. Is −150 a term of the series 11, 8, 5, 2, …? Solution:  Here, a = 11, d = 8 − 11 = −3. Let an = −150 Therefore, a + (n − 1) d = −150 ⇒ 11 + (n − 1)(−3) = −150

i=1

= 5 5760 = 5.65 ⇒ −3 (n − 1) = −161 ⇒ n − 1 = 12. Find the harmonic mean of the following data: 8, 9, 6, 11, 10, 5 Solution:  Given data: {8, 9, 6, 11, 10, 5}

Chapter 02.2.indd 660

161 3

161 is not an integral number, −150 is not 3 a term of the given series. Since,

22-08-2017 07:03:31

PRACTICE EXERCISE        661

PRACTICE EXERCISE 1. What is the sum of all the odd numbers up to 100? (a) 1250 (c) 3200

(b) 2500 (d) 2700

2. Sum of the first four and twelve terms of an AP is 26 and 222, respectively. What is the sum of the first 15 terms of the AP? (a) 345 (c) 400

(b) 295 (d) 370

3. An AP consists of 50 terms of which third term is 12 and the last term is 106. Find the 29th term.

(a) 4:1 (c) 1:2

11. How many terms are there in an arithmetic sequence whose first and fifth terms are −12 and 0, respectively, and the sum of terms is 135? (a) 8 (c) 20

(c)

2 ⋅ 310



20 ⋅ 310 + 1 2 ⋅ 310

(a) 540 (c) 720

(b) 620 (d) 480



(b) 2

(d)

19 ⋅ 310 + 1

3

7. The second term of a GP is 25/4 and the eighth term is 16/625, find the first term of the GP? 125 16 (a) (b) 8 5 16 25 (c) (d) 9 4

(b)

(1 + x )

(a) 3, 6, 9 (c) 5, 9, 13

(b) 2, 4, 8 (d) 2, 8, 12

9. Insert three terms between 2 and 18 if they are  in AP. (a) 5, 10, 15 (c) 4, 8, 12

3

(d)

Chapter 02.2.indd 661

(1 − x2 )2

15. What is the harmonic mean of two numbers whose geometric mean and arithmetic mean are 6 and 12, respectively? (a) 5

(b) 2

(c) 9

(d) 3

16. If GM = 12 and HM = 7.2, then what will be the value of AM? (b) 15

(c) 9

(d) 12.5

17. Arithmetic mean of the two numbers is 25 and geometric mean is 7. What are the two numbers? (a) 2 and 10 (c) 1 and 49

(b) 25 and 25 (d) 2 and 49/2

18. For what value of x, the following numbers are in geometric progression?

(b) 6, 9, 12 (d) 6, 10, 14

10. What is the ratio of common difference d1 and d2 of two arithmetic progressions if respective nth terms are in the ratio of 2n + 3 : n − 1?

(1 − x2 )

3 (c)

(a) 20 8. Insert three terms between 1 and 16 if they are  in GP.

(d) 4

3 2 2

(1 + x ) (b) 0, 5, 10 (d) 1, 5, 9

2 ⋅ 310

(c) 3

2

(a) 3, 5, 7 (c) 2, 5, 8

2 ⋅ 310

14. What is the sum to infinity of the series, 3 + 6x2 + 9x4 + 12x6 +  given that x < 1. (a)

6. Three numbers whose sum is 15 are in AP If 8, 6 and 4 be added to them, respectively, then these are in GP. What are the three numbers?

21 ⋅ 310 + 1

13. If the ratio of the sum of the first eight terms of a GP to the sum of the first four terms of the same GP is 9, what is the common ratio of that GP? (a) 1

5. What is the maximum sum of the series 60, 56, 52, 48, …?

(b)

310 + 1

(b) 76 (d) 52

4 8 4. Find the sum of the following infinite series 2, , ,  3 9 16 ,… 27 (a) 8 (b) 6 (c) 4 (d) 2

(b) 15 (d) 10

12. What is the sum up to 10 terms of the series 2/3, 8/9, 26/27, 80/81, …? (a)

(a) 58 (c) 64

(b) 2:1 (d) 1:4

2 8 , x, 3 27 4 (a) ± 9

8 (b) ± 9

(c) ±

16 9

(d) ±

2 9

22-08-2017 07:03:35

662        pROGRESSION 

19. What is the sum of the following infinite series? 3 5 3 5 − + + + 4 42 43 44 (a) 8/15 (b) 7/15 (c) 7/13 (d) 8/17

20. The third term of a geometric progression is 2. What is the product of the first five  terms? (a) 4 (c) 16

(b) 8 (d) 32

ANSWERS    1.  (b)

   4.  (b)

  7.  (b)

  10.  (b)

  13.  (c)

  16.  (a)

  19.  (b)

   2.  (a)

   5.  (d)

  8.  (b)

  11.  (b)

  14.  (d)

  17.  (c)

  20.  (d)

   3.  (c)

  6.  (a)

  9.  (d)

  12.  (d)

  15.  (d)

  18.  (a)

EXPLANATIONS AND HINTS 1. (b) Given numbers are 1, 3, 5, …, 99. This sequence is an AP with

a + 2d = 12 a + 49d = 106

a = 1, d = 2

Solving the above two equations, we get

If the number of terms be n, then 1 + (n − 1)2 = 99 ⇒ n = 50 n (first term + last term) 2 50 = (1 + 99) = 2500 2

Sum =

a = 8, d = 2 Now, the 29th term of the sequence will be a29 = 8 + 28 × 2 = 8 + 56 = 64 4. (b) We know that a = 2 and r = (2 / 3) < 1, then S∞ =

2. (a) Sum of the first four terms = 26 Sum of the first 12 terms = 222. Therefore, 4 (2a + 3d) = 26 2 ⇒ 2a + 3d = 13

2 2 = =6 (1 − 2) / 3 (1 − 2) / 3

5. (d) First term of the series, a = 60 and the common difference, d = −4. Now, maximum sum will be till the time we have positive numbers in the series. Hence, 0 = 60 + (n − 1)(−4)

Similarly, 12 (2a + 11d) = 222 2 ⇒ 2a + 11d = 37

⇒ 60 − 4n + 4 = 0 ⇒ n = 16 Thus, to obtain the maximum sum, we need to calculate the sum till the 16th term. Hence,

Solving the above equations, we get a = 2, d = 3 Sum of the first 15 terms will be 15 [2 × 2 + (14) × 3] 2 = 15(2 + 7 × 3) = 15(23) = 345 3. (c) If a is the first term and d the common difference, then

Chapter 02.2.indd 662

16 [2 × 60 + (16 − 1)(−4)] 2 = 16 (60 − 30) = 16 (30)

S=

= 480 6. (a) Let the three numbers in AP be a − d, a, a + d.  Hence Sum = (a − d) + a + (a + d) = 15 3a = 15 ⇒ a = 5.

22-08-2017 07:03:36

EXPLANATIONS AND HINTS        663

If n = 2, then the second term is 2(2) + 3 = 7.

Also, (a − d + 8), (a + 6), (a + d + 4) are in GP

Hence, d1 = 7 − 5 = 2

5 − d + 8, 5 + 6, 5 + d + 4 are in GP 13 − d, 11, 9 + d are in GP

Now

(13 − d)(9 + d) are in GP

If n = 1 in n − 11 series, first term is 1 − 11 = −10

(13 − d)(9 + d) = (11)

If n = 2 in n − 11 series, second term is 2 − 11 = −9

2

d − 4d + 4 = 0 2

Hence,

(d − 2) = 0   d = 2 2

Hence, the numbers are 5 − 2 = 3, 5 and 5 + 2 = 7

d2 = −9 − (−10) = 1   ⇒ d1 : d2 = 2 : 1 11. (b) We know that the first term = −12 and the fifth term = 0, so 0 = −12 + (4)d d=3

7. (b) Let a be the first term and r be the common ratio of a given GP, then ar 2−1 = ar 8−1

25 4 16 = 625

Now, the sum of sequence = 135, therefore, n [2 × (−12) + (n − 1)3] 2

135 =

⇒ 270 = 3n2 − 27 n

Dividing the above two equations, we get ⇒ 3n2 − 27 n − 270 = 0

ar7 16 4 = × ar 625 25 ⇒r=

24 × 22

5 ×5 2 ⇒r= 5 4

=

2

Solving the above quadratic equation for n, we get

 2 5

−(−9) ± 81 + 360 2 9 ± 21 = 2

6

Substituting value of r in any of the above equations 2 25 a⋅ = 5 4

n=

Since, the number of terms cannot be negative, hence, n=

125 ⇒a= 8 8. (b) It is given that first term, a = 1 and fifth term = 16. Hence ar4 = 16 ⇒1 × r4 = 16 ⇒r=2 Thus, three terms between 1 and 16 are 2, 4 and 8. 9. (d) We know that a = 2 and a5 = 18. Hence 18 = 2 + 4d ⇒ d = 4 Thus, the three terms between 2 and 18 are 6, 10 and 14.

2 8 26 80 + +  up to  + + 3 9 27 81 10 terms. Series can be expressed as

12. (d) Given series is

1 − 3  + 1 − 9  + 1 − 27  + 1 − 81 +  1

= 10 −

1

3 + 3 1

1 2

1

1 +

1 +

3

3

3

4

1

+ +

1 310



1  1 − (1 / 3)10  = 10 −  3  1 − (1 / 3)  = 10 −

10. (b) The ratio of nth terms is

Chapter 02.2.indd 663

9 + 21 = 15 2

310 − 1 2 × 310

20 × 310 − 310 + 1

2n + 3 n − 11

=

If n = 1 in 2n + 3 series, then the first term is 2(1) + 3 = 5.

=

2 × 310 19 × 310 + 1 2 × 310

22-08-2017 07:03:38

664        pROGRESSION 

17. (c) If the two numbers are x and y, then

13. (c) The sum of first n terms is given as ar n − a r −1

x+y 2 ⇒ x + y = 50 25 =

The sum of first eight terms is and a(r 8 − 1) r −1

xy = 7 ⇒ xy = 49

Similarly, the sum of first four terms is Now,

a(r 4 − 1) r −1 [a(r 8 − 1)] / (r − 1) [a(r 4 − 1)] / (r − 1)

y(50 − y) = 49

= 82 ⇒

⇒ y 2 − 50y + 49 = 0 ⇒ (y − 49)(y − 1) = 0 ⇒ y = 1, 49

r8 − 1 r4 − 1

= 82

⇒ r 4 + 1 = 82

Hence, the two numbers are 1 and 49. ⇒ r4 = 81     ⇒ r = 3 18. (a) It is given that 14. (d) Given series is

2 8 , x, 3 27

f (x) = 3 + 6x2 + 9x4 + 12x6 +  ⇒ x2f (x) = 3x2 + 6x4 + 9x6 + 12x8 + 

are in GP, then

⇒ f (x) = x′ f (x) = (3 + 3x + 3x + 3x + ) 2

4

6

8 / 27 x = x 2/3

= 3 (1 + x2 + x4 + x6 + )

1 − x  1

f (x)(1 − x ) = 3 2



1 − x 

2

1

⇒ f (x) = 3

[since

2

2

16 81 4 ⇒x=± 9 ⇒ x2 =

x < 1]

3 =

(1 − x2 )2

19. (b) Sum of the series is given by

4 + 4 3

15. (d) We know that GM = AM × HM 2

3

3 +

3

5

4

=

GM = 6 and AM = 12 Hence, HM =

6×6 =3 12

16. (a) We know that GM2 = AM × HM

5 2

5 +

5 +

4

4

46



+ + ∞

2

3/4

and it is given that

 4

+ + ∞ −

1 − (1/4)2



5/4

1 − (1/4)2

=

3 16 5 16 × − × 4 15 16 15

=

4 1 7 − = 5 3 15

20. (d) If a is the first term of the GP and r is the common ratio. Then, the first five terms of the GP are given by a, ar, ar2, ar3 and ar4. So the product of five terms will be

and it is given that GM = 12 and HM = 7.2 Hence, AM =

Chapter 02.2.indd 664

12 × 12 = 20 7.2

a × ar × ar 2 × ar3 × ar 4 = (ar2 )5 Now, ar2 = 2, thus the product of first five terms = (2)5 = 32.

22-08-2017 07:03:40

CHAPTER 3

PROBABILITY

INTRODUCTION Probability is used to measure the degree of certainty or uncertainty of the occurrence of events. Probability (P) can be mathematically given as P =

Number of favourable outcomes Total outcomes

Hence, probability can also be defined as the ratio of number of favorable outcomes of an event to the total possible outcomes of the same event.





SOME BASIC CONCEPTS OF PROBABILITY 1. An experiment which, when repeated under identical conditions, does not produce the same outcome every time is called a random experiment. Example: Tossing a die. 2. An experiment which, when repeated under identical conditions, produces the same outcome every time is called a deterministic experiment. Example:

Chapter 02-3.indd 665



If a stone is dropped from a window, then the stone will go down. 3. All the possible outcomes of a random experiment are called events and the set representing all the events of a random experiment is called sample space. Example: Tossing a die is an event and {1, 2, 3, 4, 5, 6} is its sample space. 4. An event that can never occur or the probability of its occurrence is zero, is called an impossible event. Example: The probability of getting a number greater than 6 when a die is tossed once. 5. If the occurrence of one event means the nonoccurrence of another event, then the set of those events is called mutually exclusive events. Example: If a coin is tossed one, it can either result in heads or tails but not both. 6. A set of events that includes all the possible outcomes of the sample space is said to be an exhaustive set of events. Example: If we have the event of tossing a die, then A = {1, 2, 3} and B = {4, 5, 6} are said to be exhaustive set of events. 7. If two events have equal probability of occurrence, then the two events are said to be equally likely events. Example: The probability of getting heads

22-08-2017 07:04:29

666        probability 

when a coin is tossed once is the same as the probability of getting tails. 8. If the occurrence of one event has no effect on the occurrence or probability of another event, then the two events are called independent events. Example: We can flip a coin and gets a head and flip a second coin and get a tails. Thus, the two coins do not influence each other. 9. The occurrence of an event A such that event B has already occurred is denoted by P(A|B) and is called conditional probability. Example: Event of drawing two kings from a deck of cards.

1. If A, B and C are events of a random experiment, then P (A ∪ B) = P (A) + P (B) − P (A ∩ B) P (A ∪ B ∪ C ) = P (A) + P (B) + P (C ) − P (A ∩ B) −P (B ∩ C ) − P (C ∩ A) + P (A ∩ B ∩ C ) 2. If A and B are two events of a random experiment, then P (A ∩ B) = P (B) − P (A ∩ B) P (A ∩ B) = P (A) − P (A ∩ B) 3. For any two events A and B, P (A ∩ B) ≤ P (A) ≤ P (A ∪ B) ≤ P (A) + P (B)

SOME IMPORTANT THEOREMS

4. If A and B are two events of a random experiment, then

To solve the algebraic problems of probability, following are some important and commonly used theorems:

P (A) + P (B) − 2P (A ∪ B) = P (A ∪ B) − P (A ∩ B)

SOLVED EXAMPLES 1. A book contains 50 pages. A page is chosen at random. What is the chance that the sum of the digits on the page is equal to 6? Solution:  The pages whose sum of the digits equal to 6 are 6, 15, 24, 33, 42. Thus, there are five favorable outcomes out of a possible 50 outcomes. So Probability =

5 = 0.1 50

2. A bag contains 4 red balls and 3 black balls and a man draws 2 balls at random. What is the probability of both the balls being red? Solution:  Total number of balls is 4 + 3 = 7 So the number of ways in which 2 balls can be 7 drawn = C2 And the number of ways in which 2 red balls can be drawn = 4 C2 So the probability of both the balls being red is

=

4

C2

7

C2

4!× 5! 4! 2 = = 2!× 7 ! 7 × 6 × 2 7

Solution:  Three digits numbers using 1, 2, 3, 4, 5 and 6 can be formed in 6 × 5 × 4 ways. So the total number of ways will be 120. If the number is even, then the last digit can be selected in two ways; therefore, the number of ways in which even number can be selected is 3 × 5 × 4 = 60. 60 1 = . So the probability of number being even = 120 2 4. A fair dice is rolled twice. What is the probability that the sum of the results is at least 9? Solution:  The total number of outcomes when one fair dice is rolled is 36. So the number of favorable outcomes will be {(3, 6),(4, 5),(4, 6),(5, 4),(5, 5),(5, 6), (6, 3),(6, 4),(6, 5),(6, 6)} So the probability of sum of the results to be at 10 5 = least 9 = . 36 18 5. In a biased coin, head occurs four times as frequently as tail occurs. If the coin is tossed 4 times, what is the probability of getting three heads? Solution:  We know that

3. Three digit numbers are formed by using the digits 1, 2, 3, 4, 5 and 6 without repeating any digit. What is the probability that a chosen number is an odd number?

Chapter 02-3.indd 666

P (H ) = 4P (T ) P (H) + P (T) = 1 ⇒ 4P (T) + P (T) = 1

22-08-2017 07:04:31

SOLVED EXAMPLES        667

⇒ 5P(T) = 1 ⇒ P(T) =

1 5

So the probability that the second card is spades 13 = 51 After 2 cards are drawn, cards left = 50

Therefore, and the total cards of clubs = 13. 4 P(H) = 5 Since the coin is tossed 4 times, three heads may occur if {(H H H T),(H H T H),(H T H H),(T H H H)}.

13 50 1 13 13 169 = So the required probability = × × 4 51 50 10200 So the probability that third card is clubs =

Therefore, required probability is 4 4 4 1 4 4 1 4 4 1 × × × + × × × + × 5 5 5 5 5 5 5 5 5 5 4 4 1 4 4 4 × × + × × × 5 5 5 5 5 5  4 4 4 1  256 = 4 ×  × × ×  =  5 5 5 5  625 6. A positive integer is chosen at random from 1 to 100. What is the probability that the integer chosen is either a multiple of 3 or a multiple of 5?

8. From a bag containing 6 vegetables and 4 fruits, an item is chosen at random. If out of 6 vegetables in the bag 2 are cabbages, 4 are ladyfingers, and out of 4 fruits, 3 are apples and 1 is orange, then what is the probability that the item chosen at random will be an apple? Solution:  We know that total items = 10. Say P(A) be the probability of choosing a fruit, then P (A) =

4 2 = 10 5

Now, out of 4 fruits, total number of apples = 3 Solution:  Say E1 is the event when the chosen integer is multiple of 3 and E2 is the event when the chosen integer is multiple of 5. Hence P (E1 ) =

20 1 33 = and P (E2 ) = 100 5 100

Also, E1 ∩ E2 is an event such that the numbers are divisible by 3 and 5. Hence P (E1 ∩ E2 ) =

6 3 = 100 50

Say P(B) be the probability of choosing an apple if the bag contains only fruits and no vegetables P (B ) =

Now, if P(C ) is the probability of choosing an apple if bag contains all the items, then P (C ) = P (A) × P (B) P (C ) =

Thus, the required probability will be P (E1 ) + P (E2 ) − P (E1 ∩ E2 ) 33 1 2 + − 100 5 50 33 + 20 − 4 49 = = 100 100 =

7. From a well-shuffled deck of 52 cards, three cards are drawn at random without replacing them. What is the probability that the first card is of hearts, second card is of spades and third is of clubs? Solution:  We know that total cards = 52 So the total cards of hearts = 13 Hence, the probability that the first card is hearts 13 = 52 After 1 card is drawn, cards left = 51 And the total cards of spades = 13

Chapter 02-3.indd 667

3 4

2 3 3 × = 5 4 10

9. The letters of the word “MESSAGE” are placed in a row at random. What is the probability that the two “S” come together? Solution:  The word “MESSAGE” contains 7 letters and “E” and “S” are repeated twice. Hence, 7! Total arrangements = 2! × 2! Now, taking both the “S” as one, total number of 6! arrangements = 2! Probability that the two “S” come together is 6 !/2 ! 6! × 2! 2 = = 7 !/2 ! × 2 ! 7! 7 10. Calculate the value of P (A ∪ B), if P(A) = 1/3, P(B) = 1/4 and P (A ∩ B) = 1/36.

22-08-2017 07:04:34

668        probability 

Solution:  We know that P (A ∪ B) = P (A) + P (B) − P (A ∩ B) 2 1 1 8 + 5− 1 = + − = n (S ) = 5 4 20 20 12 3 = = 20 5

Solution:  Total cards in the deck = 52

(52 × 51) = 1326 If sample space is S, then n (S ) = 52C2 = (52 × 51) 2 ×1 52 C2 = = 1326 2 ×1 Say E = event of getting 2 kings out of 4. Thus, n (E ) = 4C2 =

11. A bag contains 6 blacks and 8 white balls. One ball is drawn at random. What is the probability that the ball drawn is white?

(4 × 3) 2 ×1

=6

n (E ) 6 1 The required probability = P(E) = = = . 221 n (S ) 1326 n E ( ) 1 6 Solution:  Let number of balls = (6 + 8) = 14 = = . 221 n (S ) 1326 Number of white balls = 8 8 4 16. Two dice are tossed. What is the probability that = P (drawing a white ball) = 14 7 the total is a prime number? 12. One card is drawn at random from a pack of 52 cards. What is the probability that the card drawn is Solution:  Clearly, n(S) = (6 × 6) = 36 a face card (Jack, Queen and King only) or an ace? Say E = Event that the sum is a prime number Solution:  We know that there are 52 cards, out of Then, E = {(1, 1), (1, 2), (1, 4), (1, 6), (2, 1), (2, 3), which there are 12 face and 4 ace cards. (2, 5), (3, 2), (3, 4), (4, 1), (4, 3), (5, 2), (5, 6), (6, 1), (6, 5)} 16 4 = Required probability = Therefore, 52 13 n (E ) = 15 13. Two cards are drawn together from a pack of 52 cards. What is the probability that one is a spade n (E ) 15 5 = = Hence, required probability = P (E ) = and one is a heart? 36 12 n S ( ) n (E ) 15 5 Solution:  Total cards in a deck = 52 P (E ) = n (S ) = 36 = 12 Total spades = 13 17. In a lottery, there are 10 prizes and 25 blanks. A 13 Probability of drawing a spades = lottery is drawn at random. What is the probabil52 ity of getting a prize? Total hearts = 13 Probability of drawing a heart after a spade = 13 13 169 × = 52 51 2652

Solution:  Sample space = 10 + 25 = 35 Favorable outcome = 10 Hence, probability of getting a prize =

Also, if we draw a heart first and then a spade is 13 13 169 the same = × = 52 51 2652 Thus, the required probability that one is a spade 169 169 13 + = and one is a heart = 2652 2652 102 14. A card is drawn from a pack of 52 cards. What is the probability of getting a queen of club or a king of heart? Solution:  Total cards in a deck = 52. Probability of getting a queen of club or a king of 2 1 = . heart = 52 26 15. From a pack of 52 cards, two cards are drawn together at random. What is the probability of both the cards being kings?

Chapter 02-3.indd 668

10 2 = 35 7

18. In a class, there are 15 boys and 10 girls. Three students are selected at random. What is the probability that 1 girl and 2 boys are selected? Solution:  Let S be the sample space and E be the even of selecting 1 girl and 2 boys. Then, n(S) = number of ways of selecting 3 students out (25 × 24 × 23) = 2300 of 25 = 25C3 = 3 × 2 ×1 

n(E) =

( 10C1 × 15C2 ) = 10 × 

Therefore, P (E ) =

n (E ) n (S )

=

(15 × 14)

= 1050 2 × 1 

1050 21 = 2300 46

22-08-2017 07:04:37

SOLVED EXAMPLES        669

19. Two dice are thrown simultaneously. What is the probability of getting two numbers whose product is even? Solution:  When two dices are thrown simultaneously then the simultaneous sample space can be defined as n (S ) = (6 × 6) = 36 Let event E can be defined as getting two numbers who product is even. Then, E = {(1, 2), (1, 4), (1, 6), (2, 1), (2, 2), (2, 3), (2, 4), (2, 5), (2, 5) (2, 6), (3, 2), (3, 4), (3, 6), (4, 1), (4, 2), (4, 3), (4, 4), (4, 5) (4, 6), (5, 2), (5, 4), (5, 6), (6, 1), (6, 2), (6, 3), (6, 4), (6, 5) (6, 6)} Thus, n(E) = 27. n (E ) 27 3 Hence, required probability P (E ) = = = 36 4 n (S ) n (E ) 27 3 P (E ) = = = 4 n (S ) 36 20. Three unbiased coins are tossed. What is the probability of getting at most two heads? Solution:  The sample space when three unbiased coins are tossed = {TTT, TTH, THT, HTT, THH, HTH, HHT, HHH} Thus, n(S) = 8 Let E be the event of getting at most two heads. Then E = {TTT, TTH, THT, HTT, THH, HTH, HHT} n(E) = 7 Therefore, required probability, P (E ) =

n (E ) n (S )

=

7 8

21. What is the probability of getting a sum of 9 from two throws of a dice? Solution:  When two dices are thrown simultaneously then the simultaneous sample space can be defined as n (s) = (6 × 6) = 36 Let E be the even of getting a sum of 9 = {(3, 6), (4, 5), (5, 4), (6, 3)}

Solution:  Here, sample space S = {1, 2, 3 …, 19, 20} Let E be the event of getting a multiple of 3 or 5 = {3, 6, 9, 12, 15, 18, 5, 10, 20} Thus, n(E) = 9 E )) = Therefore, required probability,PP((E 

22. Tickets numbered 1 to 20 are mixed up and then a ticket is drawn at random. What is the probability that the ticket drawn has a number which is a multiple of 3 or 5?

Chapter 02-3.indd 669

n (S )

=

9 20

23. Debra was born between October 6th and 10th (6th and 10th excluding). Her year of birth is also known. What is the probability of Debra being born on a Saturday? Solution:  Since the year of birth is known, the birthday being on Saturday can have a zero probability. Also, since between 6th and 10th, there are three days, i.e. 7th, 8th and 9th. Therefore, probability of birthday falling on Saturday can be 1/3. 24. Amit, Sumit and Pramit go to a seaside town to spend a vacation there and on the first day everybody decides to visit different tourist locations. After breakfast, each of them boards a different tourist vehicle from the nearest bus-depot. After three hours, Sumit who had gone to a famous beach, calls on the mobile of Pramit and claims that he has observed a shark in the waters. Pramit learns from the local guide that at that time of the year, only eight sea-creatures (including a shark) are observable and the probability of observing any creature is equal. However, Amit and Pramit later recall during their discussion that Sumit has a reputation for not telling the truth five out of six times. What is the probability that Sumit actually observed a shark in the waters? Solution:  The probability that Sumit actually sees a shark, given that he claimed to have seen one. ⇒ ⇒

P ( He actually sees the shark and reports truth ) P ( He claims of seeing a shark ) P ( Sees the shark ) × P ( Reports truth ) P ( Sees the shark ) × P ( reports truth ) + P ( Doesn’t see ) × P ( reports false )

Thus, n(E) = 4.

n (E ) 4 1 Therefore, required probability, P (E ) = = = 36 9 n (S ) n (E ) 1 4 P (E ) = = = 9 n (S ) 36

n (E )

 8× 6  1

=

1

=

 8 × 1. 6  +  8 × 6  1

7

5

1 36

25. A bag contains 10 balls numbered from 0 to 9. The balls are such that the person picking a ball out of the bag is equally likely to pick anyone of them.

22-08-2017 07:04:38

670        probability 

A person picked a ball and replaced it in the bag after noting its number. He repeated this process 2 more times. What is the probability that the ball picked first is numbered higher than the ball picked second and the ball picked second is numbered higher than the ball picked third?

There are 14 such two-digit numbers that give a remainder of 3 when divided by the sum of the digits.

Solution:  Let the number of the ball picked first = x, second = y and third = z.

27. An eight-digit telephone number consists of exactly two zeroes. One of the digits is repeated thrice. Remaining three digits are all distinct. If the first three digits (from left to right) are 987, then what is the probability of having only one 9, one 8 and one 7 in the telephone number?

The three numbers x, y and z are distinct. Three distinct balls can be picked in (10 × 9 × 8) ways. The order of x, y and z can be as follows: x > y > z; x > z > y; y > z > x; y > x > z; z > x > y; z > y > x; They will occur equal number of times. Thus, the number of ways in which (x > y > z) = 1 × (10 × 9 × 8) = 120 6 120 3 Hence, required probability = = 10 × 10 × 10 25



26. Rachyita thought of a two-digit number and divided the number by the sum of the digits of the number. She found that the remainder is 3. Mehek also thought of a two-digit number and divided the number by the sum of the digits of the number. She also found that the remainder is 3. Find the probability that the two digit number thought by Rachyita and Mehek is same? Solution:  Let the two digit number that Rachyita thought be xy, where x and y are single digit numbers. Therefore, 10x + y = k (x + y ) + 3

Probability that Mehek thought of the same 1 number as Rachyita = . 14

Solution:  Let us consider two possible cases. Case 1: There is only one 9, one 8 and one 7 in the number. Hence, there has to be one digit from {1, 2, 3, 4, 5, 6} repeated thrice. Total number of ways in which such a number can 5! = 60 exist = 6C1 × 3! × 2! Case 2: One of the three digits {9, 8, 7} is repeated thrice. Hence, there will be one digit from {1, 2, 3, 4, 5, 6}. Total number of ways in which such a number can 5! exist = 3C1 × 6C1 × = 540 3 !× 2 ! Total possible telephone numbers = 60 + 540 = 600 60 1 = Probability = 600 10 28. If Raman picks a number from 1 to 6 and the operator rolls three dice. If the number Raman picked comes up on all three dice, the operator pays Raman H 3. If it comes up on two dice, Raman is paid H 2 and if it comes up on just one dice, then Raman is paid H 1. Only if the number Raman picked does not come up at all, then Raman pays the operator H 1. What is the probability that Raman will win money playing in this game is?

Solution:  If Raman picks up a number out of six numbers, then the only case in which he will lose 10x + y − 3 money is if none of the three dice shows the picked k= number on the top surface. x+y 5 5 5 125 Required probability of losing the game = × × = Also y + x > 3 6 6 6 216 5 5 5 125 Possible values of x and y for which k is a natural × × = 6 6 6 216 number are: 125 91 = = 0.42 Probability of winning the game = 1 − (x = 1, y = 5), (x = 2, y = 3), (x = 3, y = {1, 3, 5, 125 91 216 216 9}), (x = 4, y = 7), (x = 5, y = {1, 2, 9}), (x == 1 −6, = = 0.42 216 216 y = 0), 29. An Insurance company issues standard, preferred (x = 7, y = {3, 5, 8}), (x = 8, y = 0), (x = 9, y = 4) and ultra-preferred policies. Among the company’s where k is a natural number.



Chapter 02-3.indd 670



22-08-2017 07:04:39

PRACTICE EXERCISE        671

policy holders of a certain age, 50% are standard with the probability of 0.01 dying in the next year, 30% are preferred with a probability of 0.008 of dying in the next year and 20% are ultra-preferred with a probability of 0.007 of dying in the next year. If a policy holder of that age dies in the next year, what is the probability of the decreased being a preferred policy holder?

5! =5 4! 5! = 20 99986 — number of combinations = 3! 99995 — number of combinations =

99977 — number of combinations =

99887 — number of combinations =

5! = 10 3! × 2! 5! = 30 2! × 2!

Solution:  The probability of holders dying = 0.01% for standard, 0.008% for preferred and 5! =5 98888 — number of combinations = 0.007% for ultra-preferred. 4! The expected number of deaths among all the policy Total number of combinations = 70 holders of the given age, say X, during the next Now for a 5 digit number of form (abcde) to be 50 × 0.01 30 × 0.008 20 × 0.007 0.88 year = T × + + =T× divisible by 11 100 100 100 100 (a + c + e) − (b + d ) = 11 0 × 0.008 20 × 0.007 0.88 + =T× (a + c + e) + (b + d ) = 41 100 100 100 Thus, where X = Total number of policy holder of age X. ( a + c + e ) = 26, ( b + d ) = 15 If any of these policy holders (who die during the next year) is picked at random, the probHence, (a, c, e) = (9, 9, 8) and (b, d ) = (8, 7 )  (i) ability that he is a preferred policy holder =

  





30 × 0.008 × T 100 0.88 × T 100





 

Or (a, c, e) = (9, 9, 8) and (b, d ) = (9, 6)  24 3 = = = 0.2727 88 11

(ii)

Using Eq. (i), we can construct

3! × 2 ! = 6 numbers 2!

30. Sum of the digits of a 5 digit number is 41. What is the probability that such a number is divisible by 11?

Using Eq. (ii), we can construct

3! × 2 ! = 6 numbers 2!

Solution:  In order to get the sum equal to 41, the following 5-digit combination exists:

Hence, required probability =

Number of favorable cases = 12. 12 6 = 70 35

PRACTICE EXERCISE 1. A book contains 100 pages. A page is chosen at random. What is the chance that the sum of the digits on the page is equal to 8? (a) 0.08

(b) 0.09

(c) 0.90

(d) 0.10

2. From a class of 11 boys and 9 girls, a group of 5  students is selected in such a way that every group of 5 students is equally likely to be selected. What is the probability that there are exactly three girls in the selected group? (a) 472/555 (c) 308/323

(b) 195/644 (d) 924/949

3. A bag contains 5 red balls and 6 black balls and a man draws 4 balls at random. What is the probability of these being all black?

Chapter 02-3.indd 671

(a) 1/22 (c) 15/254

(b) 6/125 (d) 4/11

4. Three digit numbers are formed by using the digits 1, 2, 3, 4 and 5 without repeating any digit. What is the probability that a chosen number is an even number? (a) 2/5

(b) 3/5

(c) 3/16

(d) 3/4

5. Three people aim at a target and their respective probabilities of hitting the target are 2/3, 3/8 and 5/7. What is the probability that at least two of them hit the target? (a) 107/168 (c) 17/56

(b) 5/28 (d) 5/84

22-08-2017 07:04:41

672        probability 

6. Two fair dice are rolled once. What is the probability that the sum of the results is at least 8? (a) 1/9

(b) 5/12

(c) 7/12

(d) 17/36

7. A pair of coins is tossed once. What is the probability of having exactly one head? (a) 1/4

(b) 2/3

(c) 3/4

(d) 1/2

8. A positive integer is chosen at random from 1 to 100. What is the probability that the integer chosen is either a multiple of 5 or a multiple of 8? (a) 8/25

(b) 6/11

(c) 3/10

(d) 1/50

9. In a biased coin, head occurs three times as frequently as tail occurs. If the coin is tossed 3 times, what is the probability of getting two heads? (a) 9/64

(b) 1/16

(c) 3/64

(d) 27/64

10. A fair dice is rolled twice. What is the probability that at least one of the two results is divisible by 2? (a) 1/2

(b) 3/4

(c) 2/5

(d) 1/4

11. From a well-shuffled deck of 52 cards, a card is drawn at random. What is the chance that it is a face card (i.e., Jack, Queen and King)? (a) 10/13 (b) 6/13

(c) 3/13

(d) 5/26

12. From a well-shuffled deck of 52 cards, three cards are drawn at random without replacing them. What is the probability that all the three cards are of hearts? (a) 11/850 (b) 9/13

(c) 1/64

(d) 3/64

13. From a well-shuffled deck of 52 cards, three cards are drawn at random and the cards are replaced as they are drawn, what is the probability that all the three cards are of spades? (a) 3/64

(b) 3/4

(c) 1/4

(d) 1/64

14. If A and B are two possible events such that P (A ∩ B) = 0.6 and P(A) = 0.3, then what is the value of P(B) such that A and B are mutually exclusive events? (a) 0.4

(b) 0.3

(c) 0.6

(d) 0.5

15. The probability that at least one of A and B occurs is 0.6. If A and B occur simultaneously with probability 0.2. What is the value of P (A) + P (B) ? (a) 1

(b) 0.8

(c) 0.6

(d) 1.2

16. From a bag containing 6 balls and 4 squares, an item is chosen at random. If the bag has 3 red balls, 3 black balls, 2 red squares and 2 black squares, then what is the probability that the item chosen at random will be a red ball? (a) 3/5

Chapter 02-3.indd 672

(b) 2/5

(c) 3/10

(d) 1/20

17. Calculate the value of P (A ∪ B), if P(A) = 1/3, P(B) = 1/4 and P (A ∩ B) = 1/36. (a) 1/9

(b) 5/9

(c) 11/10

(d) 2/3

18. An integer is chosen at random from 1 to 50. What is the probability that the integer selected is a multiple of 2, 3 and 5? (a) 3/5

(b) 31/50

(c) 7/10

(d) 19/50

19. From a full deck of cards, the face cards are removed and four cards are drawn at random. What is the probability that all the four cards are of different suit? (a) 1000/9139 (c) 2197/124950

(b) 1/64 (d) 1995/124950

20. The letters of the word “ADDRESS” are placed in a row at random. What is the probability that the two vowels come together? (a) 1/6

(b) 2/3

(c) 1/7

(d) 2/7

21. In a game there are 70 people in which 40 are boys and 30 are girls, out of which 10 people are selected at random. One from the total group, thus selected is selected as a leader at random. What is the probability that the person, chosen as the leader is a boy? (a) 4/7

(b) 4/9

(c) 5/7

(d) 2/7

22. A number is selected at random from first thirty natural numbers. What is the chance that it is multiple of either 3 or 13? (a) 17/30 (b) 2/5

(c) 11/30

(d) 4/15

23. A bag contains 2 red, 3 green and 2 blue balls are to be drawn randomly. What is the probability that the balls drawn contain no blue ball? (a) 5/7

(b) 10/21

(c) 2/7

(d) 11/21

24. Four boys and three girls stand in queue for an interview. What is the probability that they stand in alternate positions? (a) 1/35

(b) 1/34

(c) 1/17

(d) 1/68

25. If the probability that X will live 15 years is 7/8 and that Y will live 15 years is 9/10, then what is the probability that both will live for 15 years? (a) 1/20

(b) 1/5

(c) 63/80

(d) 5/16

26. There are three events A, B and C, one of which must and only can happen. If the odds are 8:3 against A, 5:2 against B, then what are the odds against C? (a) 13:7

(b) 3:2

(c) 43:34

(d) 43:77

22-08-2017 07:04:42

ANSWERS        673

27. A 5-digit number is formed by the digits 1, 2, 3, 4 and 5 without repetition. What is the probability that the number formed is a multiple of 4?

34. Six dice are tossed together. What is the probability of getting different faces in all of the dice?

(a) 1/2

(b) 1/3

(c) 1/4

(d) 1/5

6

28. A and B play a game where each is asked to select a number from 1 to 5. If the two numbers match, both of them win a prize. What is the probability that they will not win a prize in a single trial? (b) 24/25

(c) 2/25

(c) 23/67

(d) 37/64

30. Abhi has 9 pairs of dark blue socks and 9 pairs of black socks. He keeps them all in the same bag. If he picks out three socks at random, then what is the probability that he will get a matching pair? (a) 1

(b)

2 × 9C2 × 9C1 C3

(c)

C3 × C1 9



(d) None of these

18

C3

31. There are 2 positive integers x and y. What is the probability that x + y is odd? (a) 1/4

(b) 1/3

(c) 1/2

(d) 1/5

32. What is the probability that a two digit number selected at random will be a multiple of 3 and not a multiple of 5? (a) 4/45

(b) 2/45

(c) 1/15

(d) 4/15

33. A bag contains 4 blue, 5 white and 6 green balls. Two balls are drawn at random. What is the probability that both the balls are blue? (a) 2/35

(b) 1/17

6

(c)

6!

6

(d)

5

66

1 (a)

1

(b)

5

6

6!

6!

(c)

6

6

(d) 66

5

(c) 1/15

36. There are 12 prizes and 24 blanks in a lottery. If John has taken a lottery, what is the probability for him to get a prize? (a) 4/5

(b) 1/3

(c) 3/4

(d) 1/2

37. Four different objects 1,2,3,4 are distributed at random in four places marked 1,2,3,4. What is the probability that none of the objects occupy the place corresponding to its number? (a) 17/24 (b) 3/8

(c) 1/2

(d) 5/8

38. An anti-aircraft gun can fire four shots at a time. If the probabilities of the first, second, third and the last shot hitting the enemy aircraft are 0.7, 0.6, 0.5 and 0.4, what is the probability that four shots aimed at an enemy aircraft will bring the aircraft down?

18 9

6!

6

(d) 23/25

29. The probability that an arrow fired from a point will hit the target is 1/4. Three such arrows are fired simultaneously towards the target from that very point. What is the probability that the target will be hit? (a) 19/64 (b) 23/64

(b)

5

35. Six dice are tossed together. What is the probability of getting the same face in all the dice?

6 (a) 1/25

1

1 (a)

(a) 0.084

(b) 0.916

(c) 0.036

(d) 0.964

39. The letters B, G, I, N and R are rearranged to form a word. Then what is the probability that the word is “BRING”? 1 1 1 1 (a) (b) (c) (d) 4 120 24 76 5 40. I forgot the last digit of a 7-digit telephone number. If 1 randomly dials the final 3 digits after correctly dialing the first four, then what is the chance of dialing the correct number? (a)

(d) 2/21

1 1001

(b)

1 990

(c)

1 999

(d)

1 1000

ANSWERS 1. (b)

8. (c)

15. (d)

22. (b)

29. (d)

36. (b)

2. (c)

9. (d)

16. (c)

23. (b)

30. (a)

37. (c)

3. (a)

10. (b)

17. (b)

24. (a)

31. (c)

38. (d)

4. (a)

11. (c)

18. (b)

25. (c)

32. (d)

39. (c)

5. (a)

12. (a)

19. (a)

26. (c)

33. (a)

40. (d)

6. (b)

13. (d)

20. (d)

27. (d)

34. (d)

7. (d)

14. (b)

21. (a)

28. (b)

35. (a)

Chapter 02-3.indd 673

22-08-2017 07:04:44

674        probability 

EXPLANATIONS AND HINTS 1. (b) The pages whose sum of the digits equal to 8 are 8, 17, 26, 35, 53, 62, 71 and 80.

P (A) = 2/3; P (A) = 1/3 P (B) = 3/8; P (B) = 5/8

Thus, there are 9 favorable outcomes. So, Probability =

9 = 0.09 100

2. (c) The number of ways in which five students can be selected from 20 students is 20

C4 =

20 ! = 4845 16 ! × 4 !

Number of ways in which three girls can be selected from nine girls 9! = 84 3!6! Number of ways in which 2 boys can be selected from 11 boys 9

11

C3 =

C2 =

11 ! = 55 2! × 9!

P (C ) = 5/7; P (C ) = 2/7 Event (A ∩ B ∩ C ) ∪ (A ∩ B ∩ C ) ∪ (A ∩ B ∩ C ) denotes that exactly two persons hit the target, and A ∩ B ∩ C is the event that all the three persons hit the target. Hence P (A ∩ B ∩ C ) = P (A) ⋅ P (B) ⋅ P (C ) =

2 3 2 1 × × = P (A ∩ B ∩ C ) 3 8 7 14 = P (A) ⋅ P (B) ⋅ P (C ) =

P (A ∩ B ∩ C ) = P (A) ⋅ P (B) ⋅ P (C ) =

Favorable outcome = 55 × 84. So Probability =

55 × 84 924 308 = = 4845 969 323

3. (a) We know the total number of balls = 5 + 6 = 11

And the number of ways in which 4 black balls can be drawn = 6 C4

P (A ∩ B ∩ C ) + P (A ∩ B ∩ C ) + P (A ∩ B ∩ C ) + P (A ∩ B ∩ C ) =

C4

11

C4

4! × 7 ! 6! × 2! × 4! 11 ! 6! 6× 5× 4× 3× 2 1 = = = 2 ! × 11 × 10 × 9 × 8 11 × 10 × 9 × 8 × 2 22

2 3 5 5 × × = 3 8 7 28

Required probability will be

Probability of all 4 balls being black will be 6

1 3 5 5 × × = 3 8 7 56

P (A ∩ B ∩ C ) = P (A) ⋅ P (B) ⋅ P (C ) =

So the number of ways in which 4 balls can be drawn = 11 C4

2 5 5 25 × × = 3 8 7 84

1 25 5 5 107 × × × = 14 84 56 28 168

6. (b) We know that the total number of outcomes when two fair dice are rolled = 36

=

4. (a) Three digits numbers using 1, 2, 3, 4 and 5 can be formed in 5 × 4 × 3 ways. So the total number of ways = 60 If the number is even, then the last digit can be selected in two ways. Therefore the number of ways in which even number can be selected = 2 × 4 × 3 = 24 24 2 So the probability of number being even = = 60 5 5. (a) Let A, B and C be the events that the targets are hit, therefore,

Chapter 02-3.indd 674

Number of favorable outcomes will be

{(2, 6),(3, 5),(3, 6),(4, 4),(4, 5),(4, 6),(5, 3),(5, 4), (5, 5),(5, 6),(6, 2),(6, 33 ),(6, 4),(6, 5),(6, 6)} So the probability of the sum of results to be at 15 5 = least 8 = 36 12 7. (d) Total number of outcomes when two coins are tossed = {(H, H),(H, T),(T,T),(T,H)} And favorable outcomes = {(H, T),(T,H)} So the probability of having exactly one head 2 1 = = 4 2

22-08-2017 07:04:48

EXPLANATIONS AND HINTS        675

8. (c) Say E1 is the event that chosen integer is multiple of 5 and E2 is the event that chosen integer is multiple of 8. Hence P (E1 ) =

20 1 12 3 = and P (E2 ) = = 100 5 100 25

12. (a) We know that total cards = 52 And total cards of hearts = 13 13 52

So the probability that the first card is hearts = After 1 card is drawn, cards left = 51 And total cards of hearts = 12

Also, E1 ∩ E2 is an event such that the numbers are divisible by 5 and 8. So 2 1 P (E1 ∩ E2 ) = = 100 50 Thus, the required probability will be P (E1 ) + P (E2 ) − P (E1 ∩ E2 ) 1 3 1 + − 5 25 50 10 + 6 − 1 15 3 = = = 50 50 10

So the probability that the second card is hearts = After 2 cards are drawn, cards left = 50 And total cards of hearts = 11 So the probability that the third card is hearts =

11 50

So the required probability will be 13 12 11 33 11 × × = = 52 51 50 2550 850

=

13. (d) We know that total number of cards = 52 And total number of cards that are spades = 13 13 52 Since, the cards are replaced, the probability of drawing a spade everytime a card is drawn remains the same. Hence the required probability will be So the probability of drawn card to be of spades =

9. (d) We know that P (H) = 3P (T) P (H) + P (T) = 1

and

3P (T) + P (T) = 1

4P(T) = 1 P(T) =

12 51

13 13 13 1 × × = 52 52 52 64

1 4

14. (b) If A and B are mutually exclusive events, then

Therefore,

P (A ∩ B) = 0 ⇒ P (A ∪ B) = P (A) + P (B) ⇒ 0.6 = 0.3 + P (B) ⇒ P (B) = 0.3

3 P(H) = 4 Since the coin is tossed three times, two heads may occur if {(H H T),(H T H),(T H H)}. Therefore, the required probability will be

15. (d) We know that 3 3 1 3 1 3 1 3 3 × × + × × + × × 4 4 4 4 4 4 4 4 4 9 9 9 27 = + + = 64 64 64 64 10. (b) When a fair dice is rolled twice, the total outcomes are 36. If one of the two results is divisible by 2, then total outcomes are {(1, 2), (1, 4), (1, 6), (2, 1), (2, 2), (2, 3), (2, 4), (2, 5), (2, 6), (3, 2), (3, 4), (3, 6), (4, 1), (4, 2), (4, 3), (4, 4), (4, 5), (4, 6), (5, 2), (5, 4), (5, 6), (6, 1), (6, 2), (6, 3), (6, 4), (6, 5), (6, 6)}

P (A ∪ B) = 0.6 and P (A ∩ B) = 0.2 Now P (A) + P (B) = 1 − P (A) + 1 − P (B) = 2 − [P (A) + P (B)] = 2 − [P (A) + P (B) − P (A ∩ B)] − P (A ∩ B) = 2 − P (A ∪ B) − P (A ∩ B) = 2 − 0.6 − 0.2 = 1.2 16. (c) We know that total items = 10

So the total favorable outcomes = 27 27 3 And the required probability = = 36 4 11. (c) We know that the total number of cards = 52

Say P(A) be the probability of choosing a ball, then 6 3 P (A) = = 10 5 Now, out of 6 balls, total number of red balls = 3

Total face cards in a deck = 12 So the probability that the card drawn is a face card =

Chapter 02-3.indd 675

12 3 = 52 13

Say P(B) be the probability of choosing a red ball if the bag contains only balls and no squares 3 1 P (B) = = 6 2

22-08-2017 07:04:51

676        probability 

Now, if P(C) is the probability of choosing a red ball if bag contains all the items, then P (C ) = P (A) × P (B) 3 1 3 P (C ) = × = 5 2 10

P(C) = Probability of drawing a card from any of the remaining two suit P(D) = Probability of drawing a card from the remaining suit P(E) = Probability that all the four cards are of the different suit

17. (b) We know that P (A ∪ B) = P (A) + P (B) − P (A ∩ B) 1 1 1 7 1 = + − = − 3 4 36 12 36 20 5 = = 36 9

Therefore, 40 40 30 10 P (B) = = 39 13 20 10 P (C ) = = 38 19 10 P (D) = 37 P (A) =

18. (b) We know that the total numbers = 50 Say, A = Event of getting multiple of 2, B = Event of getting multiple of 3 and C = Event of getting multiple of 5. Then, A = {2, 4, 6,…, 48, 50} B = {3, 6, 9,…, 45, 48} C = {5, 10, 15,…, 45, 50} A ∩ B = {6, 12, 18,…, 42, 48} B ∩ C = {15, 30, 45} A ∩ C = {10, 20, 30, 40, 50} A ∩B ∩ C = {30} 25 P (A) = 50 16 P (B) = 50 10 P (C ) = 50 P (A ∩ B) =

8 5 3 , P (A ∩ C ) = , P (B ∩ C ) = , 50 50 50 1 P (A ∩ B ∩ C ) = 50

P (A ∪ B ∪ C ) = P (A) + P (B) + P (B) − P (A ∩ B) − P (A ∩ C ) − P (B ∩ C ) + P (A ∩ B ∩ C ) =

25 11 10 8 5 3 1 31 + + − − − + = 50 50 50 50 50 50 50 50

19. (a) We know that there are 3 face cards in every suit. Hence, 12 cards are removed from the deck. Remaining cards in the deck = 52 − 12 = 40. Let, P(A) = Probability of drawing a card from any of the four suit P(B) = Probability of drawing a card from any of the remaining three suit

Chapter 02-3.indd 676

Now, P (E ) = P (A) × P (B) × P (C ) × P (D) ⇒ P (E ) =

40 10 10 10 1000 × × × = 40 13 19 37 9139

20. (d) The word “ADDRESS” has seven letters and can be arranged in 7! number of ways. Hence, the total outcome = 7! Now, “A” and “E” can be arranged in 6! ways in the row and 2! ways within themselves. Thus, the probability of vowels coming together will be 6! × 2! 2 = 7! 7 21. (a) The total groups contains boys and girls in the ratio 4:3. If the leader is chosen at random from the selection, the probability of him being a boy = 4/(4 + 3)= 4/7 22. (b) The probability that the number is a multiple 10 . of 3 is 30 Similarly, the probability that the number is a 2 . multiple of 13 is 30 Neither 3 nor 13 has common multiple from 1 to 30. Hence, these events are mutually exclusive events. Thus, the chance that the selected number is a 10 + 2 12 2 = = . multiple of 3 or 13 = 30 30 5 23. (b) 2 balls can be drawn in the following ways: 1 red, 1 green or 2 red or 2 green.

22-08-2017 07:04:54

EXPLANATIONS AND HINTS        677

C1 × C1 3

7

C2





C1 × 3C1



28. (b) Total number of ways in which both of them 6 1 3 10 can select a number each = 5 × 5 = 25 + + = 7 7 7 21 21 21 21 C2 C2 C2 1 1 2 3 = Probability that they win the prize = 1 × C2 C2 6 1 3 10 25 25 +7 +7 = + + = 21 21 21 21 C2 C2 24 1 Probability that they do not win a prize = 1 − = 25 25 24. (a) Total number of possible arrangements for 24 1 = 1− = 4 boys and 3 girls in a queue = 7! 25 25 When they occupy alternate position the arrange 29. (d) Probability of an arrow not hitting the target ment would be like: B G B G B G B = 3/4 Required probability =

2

2

+

C2

3

+

C2

=

Thus, total number of possible arrangements for boys = (4 × 3 × 2) Total number of possible arrangements for girls = (3 × 2) (4 × 3 × 2 × 3 × 2) 1 Required probability = = 7! 35 25. (c) We are given that probability that X will live 15 years = 7/8

Probability that none of the 3 arrow will hit the 3 3 27 = target = 4 64



Probability that the target will hit at least once = 27 37 1− = 64 64 30. (a) If he draws any combination of 3 socks, he will definitely have the matching pair of either colour. Hence, the probability is 1.

Also, we know that probability that Y will lie 15 years = 9/10 63 31. (c) Here we have four cases: 7 9 = Probability that X and Y will live 15 years = × 8 10 80 7 9 63 Case 1: x is even, y is even. = × = 8 10 80 Case 2: x is odd, y is even. 26. (c) According to the question, P (A ′) P (A ) Also,

=

P (B ′) P (B )

=

8 3 8 and P (A ′) = , P (A ) = 3 11 11 5 2

⇒ P (B ) =

5 2 and P (B ′) = 7 7

Now, out of A, B and C, one and only one can happen. P (A) + P (B ) + P (C ) = 1

Case 3: x is even, y is odd. Case 4: x is odd, y is odd. Out of these four cases, in cases 2 and 3. Thus, 2 1 required probability = = 4 2 32. (d) There are a total of 90 two digit numbers. Every third number will be divisible by 3. Therefore, there are 30 of those numbers that are divisible by 3. Of these 30 numbers, the numbers that are divisible by 5 are those that are multiples of 15, i.e. numbers that are divisible by both 3 and 5. There are 6 such numbers 15, 30, 45, 60 and 90.

P (C ) =

34 77

P (C ′) = 1 − P (C ) = Thus, odds against C =

43 77

P (C ) 43 = P (C ′) 34

27. (d) Total number of 5 digits number = 5! = 120. Now to be a multiple of 4, the last two digits of the number have to be divisible by 4, i.e. they must be 12, 24, 32 or 52. Corresponding to each of these ways there are 3! = 6, i.e. 6 ways of filling the remaining 3 places. ( 4 × 6) 1 The required probability = = 120 5

Chapter 02-3.indd 677

Hence, the numbers that are divisible by 3 and not by 5 = 30 − 6 = 24 24 out of the 90 numbers are divisible by 3 and not 24 4 by 5. The required probability = = 90 15 33. (a) Let E be the event that both the balls are blue. n(E) = number of ways in which 2 balls can be drawn from 4 blue balls = 4C2 n(S) = number of ways in which 2 balls can be drawn from 15 balls = 15C2

 

 

4 3 × 4 n (E ) C2 12 2 2 1 = 15 = = = P(E) = n (S ) 210 35 C2 15 14 × 2 1

22-08-2017 07:04:57

678        probability 

34. (d) Total number of outcomes possible when a die is tossed = 6 (since, any one face out of the 6 faces is possible) Hence, total number of outcomes possible when 6 dice are thrown, n(S) = 66 n(E) = number of ways of getting different faces in all the dice = the number of arrangements of 6 numbers 1, 2, 3, 4, 5, 6 by taking all at a time = 6! P (E ) =

n (E ) n (S )

(2, 1, 3, 4), (2, 1, 4, 3), (2, 3, 4, 1), (2, 4, 1, 4), (2, 4, 1, 3), (2, 4, 3, 1). Out of these, three are not acceptable to us [(2, 1, 3, 4), (2, 3, 1, 4), (2, 4, 3, 1)] are not acceptable because 3 and 4 occupy the correct positions. 3 1 Hence required probability = = 6 2 38. (d) The enemy aircraft will be brought down even if one of the four shots hits the aircraft.

6! =

The opposite of this situation is that none of the four shots hit the aircraft.

66

35. (a) Total number of outcomes possible when a die is tossed = 6 (since, any one face out of the 6 faces is possible)

The probability that none of the four shots hit the aircraft is given by = ( 1 − 0.7 )( 1 − 0.6 )( 1 − 0.5 )

Hence, total number of outcomes possible when 6 dice are thrown, n(S) = 66

= 0.3 × 0.4 × 0.5 × 0.6 = 0.036

n(E) = number of ways of getting same face in all the dice = 6C1 = 6

Thus, the probability that at least one of the four hits the aircraft = 1 − 0.036 = 0.964

P (E ) =

n (E ) n (S )

6 =

1 =

66

65

39. (c) Total number of words that can be made out of the letters B, G, I, N, R are 5! = 120 Favorable outcome = 1

36. (b) Total number of prizes, n(E) = 12 Hence, the required probability = Total number of possible outcomes, n(S) = 12 + 24 = 36 n (E ) 12 1 = = Thus, required probability, P (E ) = n (S ) 36 3 37. (c) Let a particular number, say 2, occupies ­position 1.

40. (d) It is given that last three digits are randomly dialled. Then, each of the digits can be selected out of 10 digits in 10 ways. Hence, the required probability =

Then all possible arrangements are given as:

Chapter 02-3.indd 678

1 120

  = 1000 1 10

3

1

22-08-2017 07:04:58

CHAPTER 4

SET THEORY

INTRODUCTION

A set is described by listing elements, separated by commas, within braces {}. For example, a set of even numbers can be described as {2, 4, 6, …}.

­ iagrams, the universal set U is represented by points d within a rectangle and its subsets are represented by points in closed curves (usually circles) within the rectangle. If a set A is a subset of B, then the circle representing A is drawn inside the circle representing B. If A and B are not equal but they have some common elements, then to represent A and B we draw two intersecting circles. Two disjoint sets are represented by two non-intersecting circles.

A set is said to be empty or null or void set, if it has no elements. Two sets A and B are said to be equal sets if every element of A is a member of B, and vice versa.

OPERATION ON SETS

A set is a collection of distinct objects, considered as single entity of size equal to the number of distinct objects.

If every element of A is an element of B, then A is called a subset of B. A set that contains all sets under consideration is called the universal set and is denoted by U.

1. Union of sets: A ∪ B U A

B

Venn Diagrams The diagrams drawn to represent sets are called Venn— Euler diagrams or simply Venn diagrams. In Venn

Chapter 02.4.indd 679

22-08-2017 07:05:46

680     Set Theory 

2. Intersection of sets: A ∩ B

1. n(A) = a + c 2. n(B) = b + c

A

B

U 3. Things belonging to exactly one attribute = a + b 4. n (A ∩ B) = c 5. Things belonging to none of the attributes = n 6. n (A ∪ B) = a + b + c

3. Difference of sets: A − B or B − A B−A

A−B

U

U A

B

A

VENN DIAGRAM WITH THREE ATTRIBUTES

B In this section, we discuss the Venn diagram with two attributes and give the generalized formulae. Figure 2 shows Venn diagram representation of three sets.

4. Complement of a set: A ′ B

A a

e g

U d A

b f

C C None = n Figure 2 |   Venn diagram representation of three sets A, B and C.

VENN DIAGRAM WITH TWO ATTRIBUTES

1. n(A) = a + e + g + d 2. n(B) = b + f + e + g

In this section, we discuss the Venn diagram with two attributes and give the generalized formulae. Figure 1 shows Venn diagram representation of two sets.

3. n(C ) = c + d + g + f 4. Things belonging to exactly one attribute = a + b + c

A

B a

c

b

5. n (A ∩ B) = e + g 6. n (B ∩ C ) = g + f 7. n (A ∩ C ) = d + g

None = n Figure 1 |   Venn diagram representation of two sets A and B.

Chapter 02.4.indd 680

8. n (A ∩ B ∩ C ) = g 9. Things belonging to none of the attributes = n

22-08-2017 07:05:50

SOLVED EXAMPLES     681

SOLVED EXAMPLES Direction (Q1−Q5): Solve the examples on the basis of the following diagram:

Car 34% 11%

10%

Bike 38%

22%

1% Y

X 21

10%

0 10 3 0 14 0

5% 32%

Scooter 48%

None 10%

Z Say x is (car ∩ bike), y is (car ∩ scooter) and z is (bike ∩ scooter). Therefore,

Note: X represents even numbers, Y represents odd numbers and Z represents prime numbers in the sample. 1. How many even numbers are there which are not prime?

31 = 10 + x + 1 + y 38 = 22 + x + 1 + z 48 = 32 + y + 1 + z Solving the above equations for x, y and z, we get

Solution:  Total even numbers which are not prime are the numbers belonging to X only = 21.

x = 10% y = 10% z = 5%

2. How many odd numbers are there which are prime? Solution:  Total odd numbers, which are prime, are the numbers belonging to Y ∩ Z = 14. 3. How many even numbers are prime? Solution:  Total even prime numbers are numbers belonging to X ∩ Z = 3. 4. How many numbers are present which are not prime? Solution:  Total numbers which are not prime = (X only) + (Y only) = 21 + 10 = 31.

Also, total houses surveyed =

100 × 40 = 400. 10

6. How many houses have bikes only? Solution:  Total houses with bikes only = 22% Total houses = 400 22 Houses with bikes only = × 400 = 88 100 7. How many houses have exactly two vehicles? Solution:  Total houses with more than one vehicle = 25% Total houses with three vehicles = 1%

5. How many total odd numbers are present?

Total houses with exactly two vehicles = 24%

Solution:  Total odd numbers = (Y only) + (Y ∩ Z ) =

24 × 400 = 96 100

= 10 + 14 = 24. 8. How many houses have only cars? Direction (Q6—Q10): In a survey conducted for a society, 10% of the houses have just car, 22% have just bike, 32% have just scooter, 1% have a car, bike and scooter, and 10% have none of the three things. So, 40 houses do not have any vehicle.

Chapter 02.4.indd 681

Solution:  Total houses with cars only = 10% =

10 × 400 = 40 100

22-08-2017 07:05:52

682     Set Theory 

9. How many houses have only one vehicle?

12. By which letter, the married people who live in joint family but not are school teachers are represented?

Solution:  Total houses with only cars, bikes or scooters = 10% + 32% + 22% = 64% 64 = × 400 = 256 100 10. How many houses do not have a scooter? Solution:  Total houses which do not have a scooter = 10% + 10% + 22% = 42% =

42 × 400 = 168 100

Direction (Q11—Q13): The following Venn diagram depicts people living in a building who are married, who live in joint family and who are school teacher. Study the chard carefully and answer the following.

Solution:  We have to find the intersection of just married people and who live in joint family. This is represented by S. 13. The people who live in joint family but are neither married nor teachers are represented by which letter? Solution:  We have the find the letter which represents only the people who live in joint family, represented by T. Direction (Q14—Q15): The following Venn diagram depicts people who are educated, backward and employed. Study the diagram carefully and answer the following.

Educated people

T S

8 6

U R

Married people

P

People who live in joint family

11 3 7

Q 17

5

Backward people

Employed people

School teachers 14. How many educated people are employed? 11. By which letter, the married teachers who live in joint family are represented? Solution:  In this question, we basically need to find the intersection of all the three sets. Hence, it is represented by Q.

Solution:  Number of educated people who are employed = 3 + 6 = 9. 15. How many backward people are educated? Solution:  Number of backward people who are educated = 11 + 3 = 14.

PRACTICE EXERCISE Direction (Q1—Q5): During the cultural day of a college, 225 students participated. Out of them, 100 participated in art, 105 participated in music, 95 participated in dance, 20 participated in none of these three, 45 participated in exactly two of the above three events and 20 students participated in all three events. 1. If 45 students participated in only art, then how many students participated in music and dance? (a) 10 (c) 20

(b) 15 (d) 25

2. If 20 students participated in both art and dance but not music, then how many students participated in music only?

Chapter 02.4.indd 682

(a) 55 (c) 65

(b) 60 (d) 70

3. If total students participated in dance only are 40, then how many students participated in both art and music? (a) 5 (c) 15

(b) 10 (d) 20

4. If 50 students participated in music but not dance, find the number of students who participated in art or music? (a) 110 (c) 140

(b) 130 (d) 120

22-08-2017 07:05:54

PRACTICE EXERCISE     683

5. How many students participated in art or music or dance only? (a) 110 (c) 150

(b) 130 (d) 170

Direction (Q6—Q10): In a party there were 60  people who took tea and 40 people who took coffee. If there were 80 people who attended the party then answer the following questions. 6. What is the maximum number of people who took none of these two? (a) 20 (c) 10

(b) 40 (d) 30

7. What is the maximum possible number of people who took at least one drink? (a) 10 (c) 80

(b) 60 (d) 70

8. What is the minimum possible number who took both the drinks? (a) 40 (c) 20

(b) 10 (d) 30

9. What is the minimum possible number of people who took none of these two drinks? (a) 10 (c) 30

(b) 20 (d) 0

10. What is the maximum possible number of people who took exactly one of the two drinks? (a) 20 (c) 60

(b) 40 (d) 70

Direction (Q11—Q15): Say we have three quantities A, B and C. Total value of A is 40, B is 32, C is 50, A ∩ B = 4 , A ∩ C = 5 , B ∩ C = 7 and A ∩ B ∩ C = 2 . Use the given data to answer the following questions. 11. What is the values of only A? (a) 29 (c) 35

(b) 31 (d) 27

12. What is the value of only B? (a) 19 (c) 29

(b) 21 (d) 17

13. What is the value of only C? (a) 18 (c) 30

(b) 22 (d) 36

15. How many values are present in at least two quantities? (a) 22 (c) 18

(b) 15 (d) 19

Direction (Q16—Q20): A survey was conducted among 300 people to find readership of three newspapers: Times of India, Hindustan Times and Delhi Times. It is known that 100 people read at least two of these newspapers, 230 people read Times of India or Delhi Times, 180 people read exactly one, 80 read neither Times of India nor Hindustan Times, 130 read the Times of India or Hindustan Times but not Delhi Times. 16. How many people read at least one of the other two newspapers along the Delhi Times? (a) 50 (c) 60

(b) 80 (d) 70

17. How many read only Hindustan Times or only Delhi Times? (a) 80 (c) 135

(b) 110 (d) 150

18. How many people read Times of India only? (a) 35 (c) 65

(b) 50 (d) 70

19. How many people read exactly one newspaper among the three? (a) 180 (c) 200

(b) 220 (d) 160

20. How many people read none of these three newspapers? (a) 10 (c) 40

(b) 30 (d) 20

Direction (Q21—Q25): Each of these questions given below contains three elements. These elements may or may not have some inter linkage. Each group of elements may fit into one of these diagrams at (A), (B), (C) or (D). You have to indicate the group of elements which correctly fits into the diagrams. 21. Which of the following diagrams indicates the best relation between travelers, train and bus? (a)



(b)

(c)



(d)

14. How many values are present in only one quantity? (a) 122 (c) 96

Chapter 02.4.indd 683

(b) 84 (d) 78

22-08-2017 07:05:55

684     Set Theory 

22. Which of the following diagrams indicates the best relation between profit, dividend and bonus?

24. Which of the following diagrams indicates the best relation between factory, product and machinery?

(a)



(b)

(a)

(c)



(d)

(c)

23. Which of the following diagrams indicates the best relation between women, mothers and engineers?



(b) (d)

25. Which of the following diagrams indicates the best relation between author, lawyer and singer?

(a)



(b)

(a)



(b)

(c)



(d)

(c)



(d)

ANSWERS 1. (a)

6. (a)

11. (a)

16. (d)

21. (c)

2. (b)

7. (c)

12. (a)

17. (c)

22. (b)

3. (b)

8. (c)

13. (d)

18. (b)

23. (a)

4. (d)

9. (d)

14. (b)

19. (a)

24. (d)

5. (c)

10. (c)

15. (c)

20. (d)

25. (b)

EXPLANATIONS AND HINTS Solution to Q1—Q5:

Students who participated in music and dance together = 10.

Art 100

Music 105 x a

b

x + 20 + z = 45 ⇒ x + z = 25 b + x + z + 20 = 105 ⇒ b + 25 + 20 = 105

20 z

y Dance 95

c

2. (b) Total students who participated in art and dance = 20. Thus, y = 20

None 20

⇒ b = 60 Students who participated in music only = 60.

We know that, x + y + z = 45 a + x + y + 20 = 100 b + x + z + 20 = 105 y + z + c + 20 = 95 1. (a) Total students who participated in art only = 45. Thus, a = 45 45 + x + y + 20 = 100 ⇒ x + y = 35 ⇒ z = 45 − 35 = 10.

Chapter 02.4.indd 684

3. (b) Total students who participated in dance only = 40. Thus, C = 40 y + z + 40 + 20 = 95 ⇒ y + z = 35 5 ⇒ x + y + z = 45 ⇒ x + 35 = 45 ⇒ x = 10 Students who participated in both art and music = 10.

22-08-2017 07:05:58

EXPLANATIONS AND HINTS     685

4. (d) We have x + b = 50. Hence, x + b + z + 20 = 105 ⇒ z + 50 + 20 = 105 ⇒ z = 35 x + y + z = 45 ⇒ x + y = 10 a + x + y + 20 = 100 ⇒ a = 70 a + x + b = 70 + 50 = 120 Thus, total students who participated in art or music = 120. 5. (c) We have x + y + z = 45. Hence, 2(x + y + z) = 90 a + b + c + 2(x + y + z) + 60 = 300 ⇒ a + b + c + 90 + 60 = 300 ⇒ a + b + c = 150 Thus, total students who participated in only art, music or dance = 150. Solution to Q6—Q10: 6. (a) Total people who took tea = 60. That is, at least 60 people took tea or coffee. Assuming that whomsoever had coffee, had tea also, we get maximum number of people who did not have anything = 80 − 60 = 20.

Solution to Q11—Q15:

A 40

B 32

4 y

x 2 5

7 z

C 50 11. (a) Value of only A ⇒ 40 = (x + 4 + 2 + 5) ⇒ x = 40 − 11 = 29 . 12. (a) Value of only B ⇒ 32 = (y + 4 + 2 + 7) ⇒ y = 32 − 13 = 19. 13. (d) Value of C ⇒ 50 = (z + 5 + 2 + 7) ⇒ z = 50 − 14 = 36. 14. (d) Values present only in one quantity = 29 + 19 + 39 = 84. 15. (c) Values present in at least two quantities = A∩B+A∩C +B∩C +A∩B∩C = 4 + 5 + 7 + 2 = 18 Solution to Q16—Q25: T.O.I.

H.T. e

b

a g 7. (c) For number of people who took at least one drink to be maximum, the number of people who did not take even a single drink have to be minimum = 0. Thus, the number of people who took at least one drink = 80.

d

f h c D.T.

a + b + c + d + e + f + g + h = 300 e + d + g + f = 100

8. (c) People who took coffee = 40. People who took tea = 60. Total people who attended the party = 100. Thus, minimum number of people who took both = 120 − 100 = 20.

a + b + c = 180 ⇒ h = 20 a + b + c + e + f + g = 230 b + h + 230 = 300 b = 50

9. (d) Minimum possible number of people who did not take any drink means maximum possible number of people who took at least one drink = 80.

c = 80 a = 50

Hence, minimum possible number of people who did not take any drinks = 0. 10. (c) To maximize the number of only one drink taken try to minimize both and none. Maximum number of only one drink taken = (60 − 20) + (40 − 20) = 60.

Chapter 02.4.indd 685

a + e + b = 130 ⇒ e = 30 e + f + g = 230 − (a + b + c) = 230 − 180 = 50 d = 100 − 50 = 50

22-08-2017 07:06:03

686     Set Theory 

16. (d) As already calculated, e = 30. Hence,

20. (d) Total people who read none of the three newspaper = h = 20.

d + g + e + f = 100 Total people who read at least one newspaper along with Delhi Times = d + g + f

21. (c) Bus and train are different from each other but some travelers travel by bus and some travel by train.

= 100 − 30 = 70 17. (c) Total people who read only Hindustan Times or Delhi Times = b + c = 50 + 80 = 135

22. (b) Bonus and dividend are different from each other but both these are parts of profit. 23. (a) All mothers are women and some mothers and some women may be engineers.

18. (b) Total people who read Times of India only = a = 50.

24. (d) Product and machinery are different from each other but both are found in factory.

19. (a) Total people who read exactly one newspaper = a + b + c = 180.

25. (b) All the three professions are different.

Chapter 02.4.indd 686

22-08-2017 07:06:04

CHAPTER 5

SURDS, INDICES AND LOGARITHMS

SURDS

1. xm × xn = xm + n xm

= xm −n

2. Irrational numbers that are of the form n x are called surds of order `n’. Some important rules of surds are as follows:

xn 3. (xm )n = xmn

1. n x  = (x1/n )n = x

4. (x × y)n = xn × y n

2. n xy = n x ⋅ n y

5.

n

xy

n

3. n

x = y

4. n x  5. a

b c

m

n

x

n

y

xn = yn

6. xm = xn 7. x−n =

n

= xm

if

m=n

1 xn

x = abc x = x(1/abc )

8. x0 = 1, x is any number

INDICES

LOGARITHM

A surd expressed in the form xn is called fractional index. Some important rules of indices are as follows:

The logarithm of a number is the exponent to which the base must be raised to produce that number.

Chapter 02-5.indd 687

22-08-2017 07:07:06

688     Surds, Indices and Logarithms 

If ax = b, then x is called logarithm of a number `b’ to the base `a’. It is represented as logab = x. Some of the important rules of logarithms are as follows: 1. log x (a × b) = log x a + log x b 2. log x

ab = log (a) − log (b) x

1 log x (a) y

4. log xy (a) = 5. log x a =

log n a log n x

6. log x a =

1 loga x

x

or log x a × loga x = 1

7. log x 1 = 0 where x is any natural number

3. log x (ab ) = b log x a

8. xlog x a = a

SOLVED EXAMPLES 5− 2

1. Simplify 42x−1 ⋅ 16 = 1.

5. Simplify

. 5+ 2

Solution:  We have 42x−1 ⋅ 16 = 1

Solution:  We have

⇒ 2(2x−1) ⋅ 24 = 1 ⇒2 =2 ⇒ 8x − 4 = 0 1 ⇒x= 2 (8x−4)

5− 2

0

5+ 2 After multiplying numerator and denominator with ( 5 − 2 ), we get ( 5 − 2) 5− 2 × 5− 2 ( 5 + 2)

2. Simplify 3(3x+1) = 81. Solution:  We have 3(3x+1) = 81 ⇒3 =3 ⇒ 3x + 1 = 4 ⇒ 3x = 3 ⇒x=1 (3x+1)

4

( 5 − 2 )2 =

( 5) − ( 2) 2

2

=

5 + 2 − 2 10 7 − 2 10 = 5−2 3

a2 + 1 + a2 − 1 6. Simplify

3

3. Which is greater 3 or

5

5?

Solution:  We have

Solution:  We have 3

3 = 31/3

. a2 + 1 − a2 − 1

and

5

a2 + 1 + a2 − 1

5 = 51/5

a2 + 1 − a2 − 1

L.C.M. of 3 and 5 = 15. Thus, (31/3 )15 = 35 = 243

After multiplying numerator and denominator (51/5 )15 = 53 = 125 Hence, 3 > 5 . 3

with a2 + 1 + a2 − 1, we get

5

2

4. Which is greater 3 or

3



4?



Solution:  We have 2

3 = 31/2

and

3

4 = 41/3

L.C.M. of 2 and 3 = 6. Thus,

2



2

a2 − 1

(a2 + 1) + (a2 − 1) + 2 a4 − 1 2

=

2a2 + 2 a4 − 1 = a2 + a 4 − 1 2

(41/3 )6 = 42 = 16

Chapter 02-5.indd 688

 

a2 + 1 −

=

(31/2 )6 = 33 = 27

Hence, 2 3 > 3 4.



2

a2 + 1 + a2 − 1

22-08-2017 07:07:12

SOLVED EXAMPLES     689

7. If 2x−4 +

After comparing the powers, we get

1 x −2

, then what is the value of x?

16 Solution:  We have



(2)x−4 =

1 1 1 − =+ x y z

1 ⇒

(16)x−2

⇒ (2)x−4 = (16)−(x−2)

1 1 1 + + =0 x y z

11. What is the logarithm of 64 to the base 4?

⇒ (2)x−4 = (2)−4(x−2)

Solution:  log 4 64 = log 4 (4)3

Since, the base is same, we can equate powers ⇒ x − 4 = − 4x + 8 ⇒ 5x = 12 12 ⇒x= 5

= 3 log 4 4 = 3 × 1 = 3 12. If log3 x = 6 , then what is the value of x? Solution:  We have

8. What is the value of x, if 3 16 = 42x  ?

log3 x = 6 ⇒ x = 36 = 729

Solution:  We have 3

13. What is the value of 43 +log 4 2?

16 = 42x

⇒ 161/3 = 42x ⇒4

2/3

Solution:  We have

2x

=4

After equating the powers of bases, we have 2 2x = 3 ⇒ x = 1/3

43 + log 4 2 = 43 × 4log 4 2 = 64 × 2 = 128 14. What is the value of log 4 256 − log3 9 ?   Solution:  Now,

6+ 2 9. Simplify

6− 2

log 4 256 − log3 9

.

= log 4 (4)4 − log3 (3)2

Solution:  We have 6+ 2 6− 2 After multiplying numerator and denominator by ( 6 + 2 ), we get

=4−2=2 1 15. What is the value of x, if log27 x = ? 3 Solution:  We have

6+ 2 6− 2

6+ 2

×

( 6 + 2 )2 =

( 6 ) − ( 2) 2

2

8+4 3 4

=

log27 x =

6+ 2 =

6 + 2 + 2 12 6−2

= 2+ 3

10. If 2−x = 3−y = 6z, then what is the value Solution:  Let 2−x = 3−y = 6z = k −1/x

Thus, 2 = k

−1/y

,3=k

1/z

⇒ x = 271/3 ⇒  x = 3 1 16. What is the value of x, if log x 5 = ? 4 Solution:  We have log x 5 =

1 1 ⇒ log x (5)1/2 = 4 4

,6=k

Now, 2 × 3 = 6. Hence k−1/x ⋅ k−1/y = k1/z

Chapter 02-5.indd 689

 x1 + y1 + z1  ?

1 3

⇒ x1/4 = 51/2 ⇒ x = 52 = 25

22-08-2017 07:07:17

690     Surds, Indices and Logarithms 

17. What is the value of log16 x + log 8 x + log 4 x − log2 x = 1 x + log 8 x + log 4 x − log2 x = 1?

Squaring Eq. (1), we get



1 x2

Solution:  Now,

+x



1 2 2

= 32 ⇒ x + x−1 + 2 = 9 

Squaring Eq. (2), we get

1 1 1 log2 x + log2 x + log2 x − log2 x = 1 4 3 2

(x + x−1 )

2

3 log2 x + 4 log2 x + 6 log2 x − 12 log2 x =1 12 1 ⇒ log2 x = 1 12

= 72 ⇒ x2 + x−2 + 2 = 49



⇒ log2 x = 12

⇒ x2 + x−2 = 47 22. If log12 3 = x then find the value of log of x.

⇒ x = 122 = 144

Solution:  We can rewrite log

8in terms 3

8 as 3

18. What is the value of x if log2 16 + log2 64 − log3 9 + log3 x = 9 g2 16 + log2 64 − log3 9 + log3 x = 9?

log

8= 3

log12 8

log2 16 + log2 64 − log3 9 + log3 x = 9

=

⇒ 4 + 6 − 9 = log3 (9/x) 9 ⇒ 1 = log3 (9/x) ⇒ = 13 x ⇒x=9 x2 y2 z2 + log + log ? yz zx xy



x2 y2 z2 x2 y 2 z 2 + log + log = log × × yz zx xy yz zx xy



xx yy zz  = log(1) = 0 2 2 2

= log

1

log12 3 2



 



=3



log12 (12/3) log12 22 =3 log12 3 log12 3

=3



log12 12 − log12 3 1− x =3 log12 3 x

 



 

x3 + 2x2 y + y 3 xy (x + 3y )

23. Find the value of x = 2 − 1. y

Solution: 

log12 23

log12 2 3 log12 2 =6 log12 3 1/2 log12 3

⇒ log2 (2)4 + log2 (2)6 − [log3 (9/x)] = 9

19. What is the value of log

=

log12 3

Solution:  We have

log

(2)

⇒ x + x−1 = 7

log16 x + log 8 x + log 4 x − log2 x = 1 ⇒



Solution:  We need x3 + 2x2 y + y 3 . xy (x + 3y )

to

find



given that

the

value

of

2 2 2

1 20. What is the value of x, if log 3 9x = 7 ? 2

x  y =

3

3

2

x + 2x y + y xy (x + 3y )

3

1 15 x=7 = 2 2

⇒ x = (3

(

2/3 15/2

)

=

21. If



+x

1 2

= 3, then find the value of x 2 + x −2.

  x  x  + 3  y  y   

( =

2 − 1) + 2 ( 2 − 1) + 1 3

(

⇒ x = 3 = 243 5

1 x2

+1

  

Solution:  We have log 3 9

xy

2

+2

2

2 − 1) ( 2 − 1) + 3  

2 − 1)(3 − 2 2 ) + 2 (3 − 2 2 )

(3 − 2 2 ) + 3 (

2 − 1)

Solution:  We are given that,

Chapter 02-5.indd 690

1 x2



+x

1 2

5 2 −7 + 6−4 2 +1 = =3

(1)

2 =1

= 2

2

22-08-2017 07:07:23

PRACTICE EXERCISE     691

5− 3 24. If x =

25. Solve the equation xlog x Solution:  We are given,

5+ 3 Solution:  We are given that, x=

=

=

5− 3

x x

5+ 3

xlog x

5− 3 5+ 3

−1

= 10 ×

5− 3 5− 3

=

(

= 10−1.

x x

, find the value of 8x − x2 .

5 − 3)



2

= −1 ⇒

( 5) − ( 3) 2

2

3 −log x log x 2

 32 − log x log x

3 2 = −1 ⇒ (log x) − log x − 1 = 0 2

5 + 3 − 2 15 = 4 − 15 2

⇒ 2 (log x) − 3 log x − 2 = 0 2

⇒ (2 log x + 1)(log x − 2) = 0

Putting value of x in 8x − x2 , we get

⇒ log x = −

8 (4 − 15 ) − (4 − 15 ) = 32 − 8 15 − 16 2

⇒x=

− 15 + 8 15 = 32 − 31 = 1

1 or log x = 2 2

1 or 100 10

PRACTICE EXERCISE 1. What is the value of ( 3 27 )2 ? (a) 3 (c) 16

9

(a) −1 (c) −3

(b) 9 (d) 27

(c) 2yz/(y − z)

3. What is the value of 16 ⋅ 3 8x = 8? 10. Simplify

162 × 43

= 16 ?

(b) 3 (d) 6

(b) 5 (d) 2 −1

(a) xyz (c) 0

3

−1

3 −1

x , if x, y

(b) x−1y−1z (d) 1

7. What is the value of x, if 2x + 4 − 2x + 2 = 3? (a) −2 (c) −4

Chapter 02-5.indd 691

(b) +2 (d) +4

(b) a2 + a4

(c) a2 − a4

(d) a2 − a4 − 1

(a) 3 (c) 1/3

(b) −3 (d) −1/3

12. What is the value of x, if log 4 x = 5/2?

6. What is the value of x y ⋅ y z ⋅ z and z are real numbers? 3

(a) a2 + a4 − 1

11. What is the value of log216 6?

5. What is the value of x, if ( 3 )7 × 34 = 3x × 33/2? (a) 6 (c) 4

. a2 + 1 + a2 − 1

42 × 83 × 2x

(a) 2 (c) 5

(b) yz/(y − z) 2yz (d) y +z

a2 + 1 − a2 − 1

(b) 4 (d) 8

4. What is the value of x, if

?

(b) 1 (d) 3

(a) yz/(y + z)

(b) 2 (d) 0

(a) 2 (c) 6

x −3

9. What is the value of x if ax = b y = c z and a2 = bc ?

2. What is the value of x, if 6x = 36? (a) 1 (c) 4

1

8. What is the value of x, if 3x + 3 =

(a) 8 (c) 64

(b) 32 (d) 128

13. What is the value of x, if log 8 x + log 4 x = 5? (a) 64 (c) 25

(b) 125 (d) 16

14. What is the value of log2 (log3 6561)? (a) 3 (c) 5

(b) 4 (d) 2

22-08-2017 07:07:26

692     Surds, Indices and Logarithms 

13   1 1 1 23. Solve the equation log x 8 + log 8 x = .  + + 15. What is the value of  6  1  (log a bc) + 1 (logb ca) + 1 (log c ab) + (a)  2, 4 (b) 16, 2 1 1 1 ? + + (c) 16 2 , 4 (d) 16 2 , 4 2 bc) + 1 (logb ca) + 1 (log c ab) + 1  (a) 3 (c) 3/2

24. Solve the simultaneous equations for x and y:

(b) 2 (d) 1

log2 xy = 5  and  log 8 x = log 4 y 16. What is the value of x, if log2 [log 4 (log3 x)] = 0? (a) 81 (c) 64

(a) x = 4, y = 8 (c) x = 8, y = 4

(b) 9 (d) 16

25. If log10 2 = 0.3010, then what is the value of log10 80 ?

17. What is the value of log16 64 + log 8 16? (a) 5/6 (c) 17/6

(b) x = 2, y = 3 (d) x = 4, y = 3

(b) 10/3 (d) 12

(a) 1.6020 (c) 3.9030

(b) 1.9030 (d) 4.2210 1 1 + 26. Find the value of . y − x) (x − y ) 1 + a( 1+a (a) 0 (b) 1/2 (c) 1 (d) ax + y

18. If log x + log y = log(x − y), then what is the value of y? x +1 (a) x/x − 1 (b) x x −1 7.5 2.5 (c) x/x + 1 (d) (25) × (5) x = 5 x. 27. Find the value of x in the equation 7.5 2.5 1.5 (25) × (5) (125) = 5 x. 19. If log 4 (x2 − 1) − log 4 (x + 1) = 3, then what is the 1.5 (125) value of x? (a) 13 (b) 16 (c) 17.5 (d) 20 (a) 63 (b) 65 (c) 125

28. Given that x = 100.48, y = 100.70 and xz = y 2, then what is the approximate value of z?

(d) 81

20. What is the value of 16log 4 5? (a) 32 (c) 64

(

(a) 1.45 (c) 3.7

(b) 125 (d) 25

)

21. If log5 52x +1 − 20 5 = 2x, then what is the value of x? (a) 3/4 (c) 1/2

29. What is the value of

1 (a) ab (c) 10

log1 60 + log1 60 + log1 60? 3

(a) 0 (c) 5

(b) 1/4 (d) 3/2

22. Find the value of the expression

(b) 1.88 (d) 2.9

loga (ab )

1 +

logb (ab )

.

4

5

(b) 1 (d) 60

30. If a and b are whole numbers such that mn = 121, n +1 then what is the value of (m − 1) ?

(b) a/b (d) 1

(a) 1 (c) 121

(b) 10 (d) 1000

ANSWERS 1. (b)

6. (d)

11. (c)

16. (a)

21. (a)

26. (c)

2. (c)

7. (a)

12. (b)

17. (c)

22. (d)

27. (a)

3. (d)

8. (b)

13. (a)

18. (c)

23. (c)

28. (d)

4. (c)

9. (d)

14. (a)

19. (b)

24. (c)

29. (b)

5. (a)

10. (d)

15. (d)

20. (d)

25. (b)

30. (d)

Chapter 02-5.indd 692

22-08-2017 07:07:30

EXPLANATIONS AND HINTS     693

EXPLANATIONS AND HINTS 1. (b) We have

7. (a) We have 2

3

2

2x + 4 − 2x + 2 = 3

( 27 ) = (3) = 9

⇒ 2x + 2 (22 − 1) = 3

2. (c) We have

⇒ 2x + 2 = 1 = 20

6x = 36 ⇒ 6x/2 = 62 x ⇒ =2 ⇒ x=4 2 3. (d) We have

After equating higher powers, we get x+2 = 0 ⇒ x = −2 8. (b) We have

16 ⋅ 3 x = 8

1

3x + 3 =

⇒ 4⋅3 x = 8

9

⇒3x =2

⇒3

⇒x=2 =8

x +3

3

x −3

1 =

2 x −6

⇒ 3x + 3 = 3−(2x−6)

3

After equating higher powers, we get

4. (c) We have

x + 3 = − 2 x + 6 ⇒ 3x = 3 ⇒ x = 1

42 × 83 × 2x = 16

162 × 43 2 ×2 ×2 4



9

9. (d) Let ax = b y = c z = k x

= 24

28 × 26

( 4 + 9 + x − 8 −6)

⇒2

Then, a = k1/x

=2

4

b = k1/y

After equating the powers, we get 4+9+x−8−6 = 4 ⇒ x = 14 − 9 = 5

c = k1/z Also, a2 = bc. Thus, k2/x = k1/y ⋅ k1/z

5. (a) We have

⇒ k2/x = k(1/y )+(1/z )

( 3) × 3 = 3 × 3 7

x

4

3/2

After equating higher powers, we get ⇒ (3)7 /2 × 34 = 3x × 33/2 2 1 1 = + x y z 2 2yz y +z ⇒ = ⇒x= x yz y +z

⇒ (3)7 /2 + 4 = 3x + 3/2 After equating higher powers, we get 7/2 + 4 = x + 3/2 4 ⇒ x = +4 2 ⇒x=6 6. (d) We have 3

a2 + 1 − a2 − 1 a2 + 1 + a2 − 1 After multiplying numerator and denominator

x−1y ⋅ 3 y−1z ⋅ z−1x

a2 + 1 − a2 − 1

3

= (x−1y)1/3 ⋅ (y−1z)1/3 ⋅ (z−1x)1/3 = [(x−1y)(y−1z)(z−1x)]1/3 1/3

= (1)

Chapter 02-5.indd 693

10. (d) We have

= 1

with

, we get a2 + 1 − a2 − 1 a2 + 1 − a2 − 1 a2 + 1 + a2 − 1

×

a2 + 1 − a2 − 1 a2 + 1 − a2 − 1

22-08-2017 07:07:34

694     Surds, Indices and Logarithms 





2

a2 + 1 − a2 − 1

15. (d) We have 1 1 1 + + loga bc + loga a logb ca + logb b logc ab + logc c

=



 − 2

a2 + 1



2

a2 − 1

(a + 1) + (a − 1) − 2 a4 − 1 2

=

2

a2 + 1 − a2 + 1 2a − 2 a − 1 = a2 − a 4 − 1 2 2

=

4

=

1 1 1 + + loga (abc) logb (abc) logc (abc)

= logabc a + logabc b + logabc c = logabc (abc) = 1 16. (a) We have

11. (c) We have log2 [log 4 (log3 x)] = 0

log216 6 = x

⇒ log 4 (log3 x) = 1

Then, (216)x = 6, hence,

⇒ log3 x = 41

⇒ (63 )x = 6 ⇒ 63x = 61

⇒ x = 34 = 81 17. (c) We have

After equating higher powers, we get

log16 64 + log 8 16

3x = 1

= log 42 43 + log23 24

⇒ x = 1/3

3 4 log 4 4 + log2 2 2 3 3 4 = + 2 3 9 + 8 17 = = 6 6 =

12. (b) We have log 4 x =

5 2

⇒ x = 45/2 = 45 = 25 ⇒ x = 32

18. (c) We have log x + log y = log(x − y)

13. (a) We have

log(xy) = log(x − y) xy = x − y y(x + 1) = x x y= x +1

log 8 x + log 4 x = 5 ⇒

log x log x + =5 log 8 log 4



log x log x + =5 3 log 2 2 log 2



2 log x + 3 log x = 5 ⇒ 5 log x = 30 log 2 6 log 2

19. (b) We have log 4 (x2 − 1) − log 4 (x + 1) = 3

⇒ log x = 6 log 2 ⇒ log x = log 26 ⇒ x = 64 14. (a) We have log2 (log3 6561) = log2 [log3 (3)8 ] = log2 (8) = log 2 (2)3 = 3

Chapter 02-5.indd 694

⇒ log 4

(x2 − 1) =3 (x + 1)

⇒ log 4

x2 − 1 = log 4 64 x +1

(x + 1)(x − 1) x2 − 1 = 64 ⇒ = 64 x +1 x +1 ⇒ x − 1 = 64 ⇒ x = 65 ⇒

22-08-2017 07:07:38

EXPLANATIONS AND HINTS     695

20. (d) We have

24. (c) We are given two equations: 16

log 4 5

log2 xy = 5 and log 8 x = log 4 y

Now, we know that a

log a x

log2 xy = 5 ⇒ xy = 25 = 32



(i)

=x log 8 x = log 4 y

Hence, 16

log 4 5

2 log 4 5

=4

log 4 25



log2 x log2 y log2 x log 2 y = ⇒ = 3 log2 8 log2 4 log2 2 log 2 22



1 1 log2 x = log2 y 3 2

=4 = 25 21. (a) We have,

(

)

log5 52x +1 − 20 5 = 2x

1

⇒ 52x +1 − 20 5 = 52x

( ) ⇒ 4 (52x ) = 20 ⇒ 52x =

⇒ x = y 2

( )

5 ⇒⇒ 52x = 5 5

⇒ 2x =

1

3

⇒ 5 52x − 52x = 20 5

3 52

1

1

⇒ log2 x 3 = log2 y 2 ⇒ x 3 = y 2

Substituting Eq. (2) in Eq. (1), we get

 (y ) = 32 ⇒ 3 y2

3 3 ⇒x= 2 4

22. (d) We have to find the value of

(ii)

1 1 + loga (ab ) logb (ab )

5 y2

3

= 32 ⇒ y

2 = (32)5

=4

3

x = y 2 = (4)2 = 8

Now, let x = loga (ab ) and y = logb (ab ) . Therefore, 25. (b) We can rewrite log10 80 as follows:

ax = ab and b y = ab. 1

log10 80 = log10 (8 × 10) = log10 8 + log10 10

1

Hence, a = (ab )x and b = (ab )y . ⇒ ab

1 = (ab )x

( )

= log10 23 + 1 = 3 log10 2 + 1 = (3 × 0.3010) + 1

1 (ab)y

= 1.9030

1 1 + = (ab )x y

1

When the bases are same, the powers can be equated. 1 1 1 1 Thus, + = 1 ⇒ + =1 x y loga (ab ) logb (ab )

26. (c) We follows:

can

rewrite

1+a

1

1

(x − y )

+

1+a

13 6 1 13 ⇒ + log 8 x = log 8 x 6

=

1+a

2

⇒ (2 log 8 x − 3)(3 log 8 x − 2) = 0 ⇒ log 8 x = 3

2

⇒ x = 16 2 , 4

Chapter 02-5.indd 695

3 2

2 3

( ) ( )

⇒ x = 8 2 , 8 3 ⇒ x = 23

, 23

y − x)

as

1 +

1 + aa  1 + aa  x

y

y

x

ax

+

=

7.5 2.5 (25) × (5)

⇒ 6 (log 8 x) − 13 (log 8 x) + 6 = 0

1 + a(

(ay + ax ) = 1 (ay + ax ) (ay + ax ) (ay + ax ) ay

=

+

1

(y − x)

23. (c) We are given, log x 8 + log 8 x =

1

(x − y )

27. (a) We have

(125)

= 5x

1.5

3 2 , 2 3

Then,

(52 ) × (5)2.5 = 5x ⇒ 515 × 52.5 = 5x 1.5 54.5 (53 ) 7.5

22-08-2017 07:07:42

696     Surds, Indices and Logarithms 

Since, the base is same, we can equate the powers.

(5 ) × (5)2.5 = 5x ⇒ x = 15 + 2.5 − 4.5 = 13 1.5 (53 ) 2 7.5

29. (b) We are given that log60 3 + log60 4 + log60 5 = log60 (3 × 4 × 5) = log60 60 =1

28. (d) We are given that, 30. (d) We know that, (0.48z )

(2 × 0.70)

⇒ 10 = 10 = 10 ⇒ 0.48z = 1.4 140 35 ⇒z= = = 2.9 48 12

Chapter 02-5.indd 696

1.4

112 = 121 Putting a = 11 and b = 2, we get

(11 − 1)

2 +1

= (10) = 103 = 1000 (3)

22-08-2017 07:07:43

UNIT 3: REASONING AND DATA INTERPRETATION

Chapter 03-1.indd 697

Chapter  1.

Cubes and Dices

Chapter  2.

Line Graph

Chapter  3.

Tables

Chapter  4.

Blood Relationship

Chapter  5.

Bar Diagram

Chapter  6.

Pie Chart

Chapter  7.

Puzzles

Chapter  8.

Analytical Reasoning

25-08-2017 01:23:34

Chapter 03-1.indd 698

22-08-2017 07:08:31

CHAPTER 1

CUBES AND DICES

CUBES A cuboid is any three-dimensional figure with length, breadth and height. If all the three sides of a cuboid are same, then the figure is called a cube. Figure 1 shows a Rubik’s cube which is a cube made of 27 smaller cubes.

In a cube:

1. There are 8 corners/vertices. 2. There are 6 faces, all equal in area. 3. There are 12 edges, all equal in area. 4. It should be noted that Number of edges = Number of vertices + Number of face − 2

Some important tips to solve questions based on cubes are as follows:

Figure 1 |   Rubik’s cube.

Chapter 03-1.indd 699

1. If we paint a cube of the dimension n × n × n in any one colour and cut it to get n3 cubelets, then the number of cubes with only one face painted = (n − 2)2 × 6. 2. If we paint a cube of the dimension n × n × n in any one colour and cut it to get n3 cubelets, then the number of cubes with two faces painted = (n − 2) × 12. 3. If we paint a cube of the dimensions n × n × n in any one colour and cut it to get n3 cubelets, then the number of cubes with three faces painted = (n − 2)3 .

22-08-2017 07:08:33

700     Cubes and Dices 

4. If we cut a bigger cube into n3 identical cubelets, using minimum number of cuts, we need a total of 3(n − 1) cuts, such that (n − 1) cuts parallel to each of these faces which are joining to corner. 5. If number of cuts are not multiple of three then cube can never be cut into identical cubes but still it can be cut into maximum number of identical cuboidal pieces. To maximize such number of pieces we need to split the number of cuts into three parts which are closest.

Figure 2 |   Symmetric dices.

When sum of numbers on opposite pair of faces is different, then the dice is called an asymmetric dice. Figure 3  shows asymmetrical dices.

DICES Dices are small cubical structures with numbers 1 to 6 (or dots) marked on each face. When sum of number of opposite pair of faces is same, then the dice is called a symmetric dice. Hence, for a symmetrical dice, the faces opposite to each other will be 1→6, 2→5, 3→4. Figure 2 shows symmetrical dices.

Figure 3 |   Asymmetric dices.

SOLVED EXAMPLES Direction for Q1—Q5:  If a cube is cut into n3 ­identical cubelets using minimum number of cuts, after painting  all faces of cubes with white colour, then answer the  following questions. 1. What is the maximum number of cuts required? Solution:  There are a minimum of (n − 1) equidistant cuts parallel to each of three faces which are joining the corner. Hence, total number of cuts required is 3(n −1). 2. How many cubelets will have exactly two faces painted? Solution:  Cubes with exactly two faces painted =  12(n − 2), where n is the number of parts into which side is divided. 3. How many cubelets will have exactly one face painted? Solution:  Cubes with exactly one face pained =  6(n −2)2, where n is the number of parts into which side is divided.

Hence, total number of cubelets with at most two faces painted = n3 − 8 5. How many cubelets will have at least one face painted? Solution:  Total number of cubelets with at least one face painted = Total number of cubes — Number of cubes with no-face painted. Number of cubes with no face painted = (n − 2)3 Hence, total number of cubelets with at least one face painted = n3 − (n − 2)3 Three adjacent faces of one cube are painted in red, one adjacent pair of faces is painted in black and remaining faces are painted in white. The cube is then cut into 216 identical cubelets. Direction for Q6—Q10:  A cube is cut into 216 cubelets = 6 × 6 × 6 We can draw the pattern of painting as shown in the following figure: Red

4. How many cubelets will have at most two faces painted? Red (R)

Black (B)

Black

Red

Solution:  Total number of cubelets with at most two faces painted = Total number of cubes — Number of cubes with three faces painted. Total number of cubes = n3 Number of cubes with three faces painted = 8

Chapter 03-1.indd 700

White (W)

Now, let us analyze the colour combination.

22-08-2017 07:08:34

SOLVED EXAMPLES     701

Corners: RRR = 1, BBR = 1 RRB = 2, BBW = 1 RRW = 1, RWB = 2

11. Observe the dots on a dice (1—6 dots) in the following figures. How many dots are contained on the face opposite to that containing 4 dots?

Edges: RR = 3, RW = 2 RB = 3, BW = 2 BB = 1 (a)

Faces: R = 3 B=2 W =1 6. How many cubelets have all the three colours on them? Solution:  The small pieces will be formed from the corners. The cubelets having all three colours (red, black and white) are 2. 7. How many cubelets have exactly two colours on them? Solution:  The cubelets with exactly two colours found at corners = 5 Total edges having different colours on either side = 8

(b)

Solution:  We can see from Figs. (a) and (b) that the side of the dice with 5 dots remains stationary. Hence, when we shift the dice such that we move the side with 1 dot, we observe that one side has  2 dots and the other side has 4 dots. Since these two faces are opposite to each other, 2 dots are contained on the face opposite to that containing 4 dots. 12. Three different positions of a dice are shown in the following figures. How many dots lie opposite  2 dots?

Therefore, total cubelets = 8(6 − 2) + 5 = 37 8. How many cubelets have exactly one colour on them? Solution:  The cubelets having exactly one colour at corner (RRR) = 1 At the three edges each of RR categories = 3(6 − 2) = 12 At one edge of BB category = 1(6 − 2) = 4 At middle of each of the six faces = 6(6 − 2)2 = 96 Hence, total cubelets with exactly one colour = 1 + 12 + 4 + 96 = 113 9. How many cubelets do not have red colour on them?

(a)

(b)

(c)

Solution:  From Figs. (b) and (c), we conclude that 1, 6, 3 and 4 dots lie adjacent to 5 dots. Therefore, 2 dots must lie opposite 5 dots. Conversely, 5 dots must lie opposite 2 dots. 13. A dice is thrown four times and its four different positions are shown in the following figures. What is the number on the face opposite the face showing 2?

Solution:  The number of cubelets with no red colour = Total cubes — (Total cubelets from one red surface + Total cubelets from two red surface +  Total cubelets from three red surfaces) = 216 − (36  + 30 + 25) = 1256 1 2 3 = 216 − (36 + 30 + 25) = 125 4 4 4 2 2 5 3 3 10. How many cubelets have black or white colour on them but not red colour on them? (a) (c) (d) (b) Solution:  The number of cubelets with no red colour = 125 Solution:  From Figs. (a), (b) and (d), we conclude  The number of cubelets with black or white but that 6, 4, 3 and 1 lie adjacent to 2. Hence, 5 must not red = 125 − (6 − 2)3 = 125 − 64 = 61 lie opposite 2.

Chapter 03-1.indd 701

22-08-2017 07:08:36

702     Cubes and Dices 

14. Two positions of a cube with its surfaces numbered are shown in the following figures. When the surface 4 touches the bottom, which surface will be on the top?

2. 64 cubes each having two adjacent blue faces and one red and other green on their opposite faces, while red on the rest. They are then mixed up. Using this information, answer the following questions.

1

1 2

3

16. How many cubes have at least two-coloured red faces each? 6

5

(a)

Solution:  According to two schemes, both types of cubes are such who have at least two redcoloured faces each. Therefore, the total number of the required cubes is 128.

(b)

Solution:  In these 2 positions, one common face with number 1 is in the same position. Also, we can see that 2, 3, 5 and 6 are adjacent to 1. Thus, 1 and 4 are opposite. Therefore, when surface 4 touches the bottom,  1 will be on the top.

17. What is the total number of red faces? Solution:  Number of red faces among first 64 cubes = 128 Number of red faces among second 64 cubes = 192 Thus, the total number of red faces = 128 + 192 = 320 18. How many cubes have two adjacent blue faces each?

15. A dice is numbered from 1 to 6 in different ways. If 2 is opposite to 3 and adjacent to 4 and 6, then the statement “1 is adjacent to 2 and 3” is true or false? Solution:  If 2 is opposite to 3, then 1 cannot lie opposite to either of the two numbers, i.e. 2 or 3.

Solution:  According to the second scheme, 64 cubes have two adjacent blue faces. 19. How many cubes have only one red face each? Solution:  None of the schemes have only one face red.

Hence, 1 is necessarily adjacent to both 2 and 3. Thus, the statement is true.

20. Which two colours have the same number of faces?

Direction for Q16—Q20: There are 128 cubes which are coloured according to two schemes, viz.

Solution:  First 64 cubes are such that each of whose two faces are green and second 64 cubes are such that each of whose two faces are blue.

1. 64 cubes each having two adjacent red faces and one yellow and other blue on their opposite faces while green on the rest.

Therefore, green and blue colours have the same number of faces.

PRACTICE EXERCISE Direction for Q1—Q5: A cuboid is divided into 192 identical cubelets. This is done by making minimum number of cuts possible. All cuts are parallel to some of the faces. But before doing so the cube is painted with green colour on one set of opposite faces. But on the other set of opposite faces and red on remaining their part of opposite faces. 1. What is the maximum number of cubelets possible which are coloured with green colour only? (a) 48

(b) 64

(c) 72

5. What is the number of cubelets possible which are painted with no colour? (a) 16 (c) 36

(b) 24 (d) 48

Direction for Q6—Q10: A cube is painted and then divided cut into 336 smaller but identical pieces by making the minimum number of cuts possible. All cuts are parallel to some faces.

(d) 96 6. What is the minimum number of cuts required?

2. What is the minimum number of cuts being made? (a) 15

(b) 18

(c) 21

(d) 12

3. What is the minimum number of cubelets possible which are painted with green and blue colours? (a) 8

(b) 16

(c) 24

(d) 32

4. What is the number of cubelets which are painted with exactly three colours? (a) 2

Chapter 03-1.indd 702

(b) 4

(c) 8

(d) 16

(a) 21 (c) 24

(b) 18 (d) None of the above

7. How many smaller pieces have exactly three painted faces? (a) 8

(b) 12

(c) 20

(d) 24

8. How many smaller pieces have at least two faces painted? (a) 60

(b) 64

(c) 66

(d) 68

22-08-2017 07:08:36

EXPLANATIONS AND HINTS     703

When 2 is at the bottom, what number will be at the top?

9. How many smaller pieces have at most one face painted? (a) 234

(b) 248

(c) 264

(a) 4

(d) 268

10. How many smaller pieces have no face painted? (a) 108

(b) 120

(c) 124

(b) 6

(c) 1

(d) 5

17. A dice is numbered from 1 to 6 in different ways. If 1 is opposite to 5 and 2 is opposite to 3, then

(d) 144

Direction for Q11—Q15: A wooden cube of side 4 cm  has been painted with different colours. The two opposite surfaces are painted red, the other two with green colour. Out of the remaining two surfaces, one is painted white and the other is painted black. Now the cube is cut into 64 equal cubes.

(a) 4 is adjacent to 3 and 6 (b) 2 is adjacent to 4 and 6 (c) 4 is adjacent to 5 and 6 (d) 6 is adjacent to 3 and 4 18. Three positions (a, b and c) of a dice are shown in the following figures:

11. How many cubes have only green colour? (a) 8

(b) 12

(c) 16

(d) 20

12. How many cubes have only two colours, red and black? (a) 2

(b) 4

(c) 6

(d) 12

13. How many cubes have only black colour? (a)

(a) 16

(b) 8

(c) 4

(b)

(c)

(d) 2 Which number lies opposite to 6?

14. How many cubes do not have any coloured face? (a) 2 (a) 8

(b) 4

(c) 16

(b) 2

(c) 8

(c) 1

(d) 5

Direction for Q19—Q20: A dice is prepared in the following manner: 1.  1 should be between 2 and 3. 2.  2 should be opposite to 3. 3.  4 should lie between 5 and 6. 4.  5 and 6 should lie opposite to each other. 5.  4 should lie face down.

15. How many cubes have three colours red, green and white? (a) 12

(b) 4

(d) 12

(d) 4

16. Two positions (a and b) of a dice are shown in the following figures:

19. The face opposite to 1 is (a) 2

(b) 4

(c) 5

(d) 6

20. The faces adjacent to 3 are (a)

(a) 1, 2, 5 and 6 (c) 1, 2, 4 and 5

(b)

(b) 2, 4, 5 and 6 (d) 1, 4, 5 and 6

ANSWERS 1. (a)

3. (b)

5. (d)

7. (a)

9. (d)

11. (a)

13. (c)

15. (d)

17. (b)

19. (b)

2. (a)

4. (c)

6. (b)

8. (d)

10. (b)

12. (b)

14. (a)

16. (c)

18. (c)

20. (d)

EXPLANATIONS AND HINTS 1. (a) Cuboid has to be painted with green colour on set of opposite faces of 6 × 8.

2. (a) The number of cuts when 192 is factorized into three closest factors. Hence, 192 = 4 × 6 × 8 Hence, number of green coloured cubelets = Therefore, we have two opposite faces of 4 × 6, 6 × 8 and 4 × 8. (6 − 2) × (8 − 2) = 24 4 × 6 , 6 × 8 and 4 × 8. Now there are two such faces hence maximum  Now, the minimum number of cuts = (4 − 1) + (6 − 1) + (8 − 1) total number of only green coloured cubelets = = (4 − 1) + (6 − 1) + (8 − 1) = 3 + 5 + 7 = 15 24 × 2 = 48

Chapter 03-1.indd 703

22-08-2017 07:08:38

704     Cubes and Dices 

3. (b) To minimize the number of cubelets with blue and green, blue should be selected on (4 × 6) or (6 × 8) and similarly green should be coloured on (6 × 8) or (4 × 6) cubelets, respectively, from the edges where green and blue faces will be meeting = 4 + 4 + 4 + 4 = 16 4. (c) Only the cubelets from corners of cuboid will be painted with 3 colours.

Total cubes = 43 = 64. Number of cubes with no face painted = (n − 2)3 = (4 − 2)3 = 8 (n − 2) = (4 − 2)3 = 8 3

Number of cubes with only black colour = (n − 2)2 = (4 − 2)2 = 4 (n − 2) = (4 − 2)2 = 4 2

Number of cubes with only green colour = 2(n − 2)2 = 2×4 = 8

Hence, 8 cubelets will be coloured in 3 colours.

Number of cubes with red and black colour = 2(n − 2) = 4 5. (d) Number of unpainted cubelets = (4 − 2)×(6 − 2)×(8 − 2) = 2 × 4 × 6 = 48 Number of cubes with red, green and white colour = 4 = (4 − 2)×(6 − 2)×(8 − 2) = 2 × 4 × 6 = 48 11. (a) Total cubes with only green colour = 8 Solution to Q6—Q10:  Number of identical pieces,  12. (b) Total cubes with only two colours, red and black = 4 336 = 8 × 7 × 6 Hence, we need 7 cuts in Z-direction, 6 cuts in Y-direction and 5 cuts in X-direction.

13. (c) Total cubes with only black colour = 4 14. (a) Total cubes with no colour = 8

Hence, cube is cut into 8 parts in Z-direction,  7 parts in Y-direction and 6 parts in X-direction.

15. (d) Total cubes with three colours (red, green and white) = 4 6. (b) Minimum number of cuts = (8 − 1) + (7 − 1) + (6 − 1) = 18 16. (c) Number 3 is common to both Figs. (a) and (b). (8 − 1) + (7 − 1) + (6 − 1) = 18 Now, let us rotate the dice in Fig. (b) in a way such 3 7. (a) All pieces from the corners of cube = 2 = 8 that 3 is unmoved. 8. (d) Total cubelets with three faces painted are Moreover, the numbers 5 and 2 come on opposite corner faces = 8. sides of 6 and 1, respectively. Hence, when 2 is at top. Total cubelets with two faces painted = 4[(8 − 2) + (7 − 2) + (6the − 2bottom, )] = 4(6 +1 5is+at4)the = 60 4[(8 − 2) + (7 − 2) + (6 − 2)] = 4(6 + 5 + 4) = 60 17. (b) If 1 is opposite to 5 and 2 is opposite to 3, then 4 is opposite to 6. Therefore, 2 cannot lie opposite Hence, required cubelets = 60 + 8 = 68 to any of the two numbers, 4 or 6. Hence, 2 lies 9. (d) Total cubelets with no face painted = (8 − 2)(7 − 2)(6 − adjacent 2) = 6 × 5to × 44 = 1206. and (8 − 2)(7 − 2)(6 − 2) = 6 × 5 × 4 = 120 18. (c) From Figs. (a) and (b), we conclude that 1, 5, Total cubelets with only one face painted 6 and 3 lie adjacent to 4. 2[(8 − 2)(7 − 2) + (6 − 2)(7 − 2) + (6 − 2)(8 − 2)] Therefore, 2 must lie adjacent to 4. From Figs. (b) = 2(6 × 5 + 5 × 4 + 4 × 6) = 148 and (c), we conclude that 4, 3, 2 and 5 lie adjacent Hence, total cubelets with no face painted = 120 + 48 = 268 to 6. Thus, 1 must lie opposite to 6. = 120 + 48 = 268 Solution to Q19 and Q20:  Consider the ­following 10. (b) Total cubelets with no face painted figure: (8 − 2)(7 − 2)(6 − 2) = 6 × 5 × 4 = 120 6 1 Solution to Q11—Q15:  Consider the following figure: Red (Top and bottom) 3 2 2

5 3

White (back) 4 19. (b) The face opposite to 1 will be 4. Black (front) Green (left and right)

Chapter 03-1.indd 704

20. (d) We can conclude that 3 and 2 will be opposite. Hence, 1, 4, 5, 6 are adjacent to 3.

22-08-2017 07:08:41

CHAPTER 2

LINE GRAPH

INTRODUCTION

4. Capability utilization.

A line graph (or line chart) is a graph which displays information as a series of data points connected by straight line segments. A line graph shows a particular data that change at equal intervals of time. Line graph is the simplest way to represent data. Single set or multiple sets of data can be shown in a graph. Figure 1 shows a line graph representing average daily temperature for two states, A and B. Usually, we can analyze the following things using line graphs:

Temperature (°F)

Capacity utilization = 85 80 75 70 65 60 55 50 0

State A State B Mon

1. Increase in profit in absolute terms or in percentage terms. 2. Average annual growth rate. 3. Average profit.

Chapter 03-2.indd 705

Total production ×100 Total capacity

Tue

Wed Days

Thu

Fri

Figure 1 |   Line graph representing average daily temperature for two states.

23-08-2017 05:36:37

706     Line Graph 

SOLVED EXAMPLES

Solution:  We know that capacity utilization is given by Total production Capacity utilization = × 100 Total capacity 120 = × 100 150 = 80%

Direction for Q4 and Q5: Consider the following graph: 30 Consumption in 1000 tones

1. The total capacity of a Hyundai i10 plant is 150 cars per day. In the month of April 2009, the plant manufactured at the rate of 120 cars per day. Find the capacity utilization in the month of April.

25 Metal 20 Plastic 15 10

500

500 480

400 300 250 200 100

150

2001 2002 2003 2004 Years Balance sheet of XYZ Corporation 2. What was the percent increase in profit in 2001—02 and 2003—04? Solution:  Percent increase in profit in 2001—2002 250 − 150 × 100 = 66.67% = 150 Present increase in profit in 2003—04 500 − 480 × 100 = 4.16% = 480 3. What was the average annual growth rate for XYZ Corporation? Solution:  Average annual growth rate Increase in profit for duration 100 = × Base years profit Number of years 500 − 150 100 = × 150 3 350 100 = × = 66.67% 150 3

Chapter 03-2.indd 706

0 2006 2007 2008 2009 2010 2011 Years The above graph is showing consumption of metals versus plastics in the given years (2006—11) for car manufacturing. 4. Which item (and for which year) shows the highest percentage change in consumption over the previous year? Solution:  Percentage change in consumption is highest for metals in 2008. 5. Find out the number of years for which the consumption of metals was less than the consumption of plastic over the given time period. Solution:  For 2006, 2007, 2010 and 2011, the consumption of metals was less than plastics. Hence, for four years the consumption of metals was less than plastics. Direction for Q6—Q8: Consider the following graph: Percent profit earned by a company 80 70

70

65

60 Profit %

Profit (in millions)

Direction for Q2 and Q3: Consider the following graph:

5

60

55

50

45 40

40 30 20 10 0 1995 1996 1997 1998 1999 2000 Years

22-08-2017 07:09:17

PRACTICE EXERCISE     707

Direction for Q9 and Q10: Consider the ­following graph: 410

Solution:  Let the expenditure in 1996 be x.

400

Also, let the incomes in 1996 and 1999 be a and b, respectively.

390 Marks

6. If the expenditures in 1996 and 1999 are equal, then what is the approximate ratio of the income in 1996 and 1999, respectively?

Then, for the year 1996, we have



a−x 155x ×100 = 55 ⇒ a = x 100



b−x 170x ×100 = 70 ⇒ b = x 100

(1)

405

400

385

380 370 370 360 365

360 350

(2)

340 330

Apr,01 Jun,01 Aug,01 Oct,01 Dec,01 Feb,02 Periodical exams

From Eqs. (1) and (2), we get a (155x / 100) 155 = = ≈ 9 : 10 b (170x / 100) 170

The above graph shows marks obtained by a student in six periodicals held in every two months during the year in the session 2001—2002. In addition, the maximum total marks in each periodical exam are 500.

7. What is the average profit earned for the given years?

9. What is the percentage of marks obtained by the student in the periodical exams of August, 01 and Solution:  Average profit for the given years October, 01 taken together? 370 + 385 755 40 + 55 + 45 + 65 + 70 + 60 335 × 100 = × 100 = 75 Solution:  Required percentage = = = = 55.83 500 + 500 1000 6 6 370 + 385 755 = × 100 = × 100 = 75.5% 500 + 500 1000 8. If the income in 1998 was H 264 crores, what was 10. What are the average marks obtained by the student the expenditure in 1998? in all the periodical exams during the last session? Solution:  Let the expenditure is 1998 be H  x crores. Solution:  Average marks obtained in all the periodical 264 − x 264 × 100 360 + 365 + 370 + 385 + 400 + 405 2285 × 100 = 65 ⇒ x = = 160 exams = = = 380.83 x 165 6 6 360 + 365 + 370 + 385 + 400 + 405 2285 Expenditure in 1998 = H 160 crores. = = = 380.83 6 6

















PRACTICE EXERCISE Direction for Q1—Q5: The following graph depicts the consumer price index (CPI) of 2000—2001.

1. Which month showed the highest absolute difference in the consumer price index over the previous month?

370 (a) March (c) May

360

(b) April (d) July

2. For how many months was the CPI greater than 350?

350 340

(a) 1 (c) 3

(b) 2 (d) 4

330 3. Which month showed the highest percentage differences in the CPI over the previous month?

320 Jan Feb Mar Apr May June July Consider the graph and answer the following questions.

Chapter 03-2.indd 707

(a) March (c) May

(b) April (d) July

22-08-2017 07:09:18

708     Line Graph 

4. The difference in the number of months in which there was an increase in the CPI and the number of months in which there was a decrease was

Direction for Q11—Q15: The following graph depicts export from three companies over the years.

140

5. In how many months was there a decrease in the CPI over the previous month? (a) 1 (c) 3

Company X Company Y Company Z

150

(a) 1     (b) 2     (c) 3     (d) 4

130

(b) 2 (d) 4

120 110

Direction for Q6—Q10: The following graph shows the annual profit of two companies for six years.

Company B

Amount (in cr.)

Company A

100 90 80 70

60

Profit %

60 50

50

40

40 30

30 20 1993 1994 1995 1996 1997 1998 1999 Years

20 10

Consider the graph and answer the following questions.

0 1995 1996 1997 1998 1999 2000 Years

11. For which of the following pairs of years, the total exports from the three companies together are equal?

Consider the graph and answer the following questions. 6. If the expenditure of company B in year 1998 was H 200 crore, what was its income? (a) H 360 crore (c) H 240 crore

(b) H 300 crore (d) H 120 crore

7. If the income of company A in year 2000 was H 600 crore, what was its expenditure? (a) H 210 crore (c) H 340 crore

(b) H 275 crore (d) H 375 crore

8. If the income of the two companies were equal in 1996, what was the ratio of their expenditures? (a) 26:27 (c) 1:2

(b) 100:67 (d) 5:27

9. If the income of company B in 1996 was H 200 crore, what was its profit in 1997? (a) H 21.5 crore (c) H 153 crore

(b) H 46.15 crore (d) Cannot be determined

10. What is the percent increase in the present profit for company B from year 1998 to 1999? (a) 25% (c) 75%

Chapter 03-2.indd 708

(b) 50% (d) 90%

(a) 1995 and 1998 (c) 1997 and 1998

(b) 1996 and 1998 (d) 1995 and 1996

12. Average annual exports during the given period for company Y is approximately what percent of the average annual export for company Z? (a) 93.33% (c) 89.64%

(b) 91.21% (d) 87.12%

13. In which year was the difference between the exports from companies X and Y the minimum? (a) 1995 (c) 1997

(b) 1996 (d) 1998

14. What was the difference between the average exports of the three companies in 1993 and average exports in 1998? (a) H 20 crore (c) H 25.55 crore

(b) H 22.17 crore (d) H 17.5 crore

15. In how many given years were the exports from company Z more than the average annual exports over the given years? (a) 1 (c) 3

(b) 2 (d) 4

22-08-2017 07:09:18

EXPLANATIONS AND HINTS     709

Direction for Q16—Q20: The following graph depicts the number of students who joined and left the school in the beginning of year for six years, from 1996 to 2001.

17. For which year, the percentage rise/fall in the number of students who left the school compared to the previous year is maximum? (a) 1997 (c) 1999

Number of students

Initial strength of school in 1995 = 3000

(b) 1998 (d) 2000

600 18. How many students were studying in the school during 1999?

500 400

(a) 2950 (c) 3100

300

(b) 3000 (d) 3150

200 19. The number of students studying in the school in 1998 was what percent of the number of students studying in the school in 2001?

100 0 1996 1997 1998 1999 2000 2001 Years

(a) 92.13% (c) 96.88%

Number of students who left the school Number of students who joined the school Consider the graph and answer the following questions. 16. By approximately what percent, the strength of school increased/decreased from 1997 to 1998?

(b) 93.75% (d) 97.25%

20. What is the ratio of the least number of students who joined the school to the maximum number of students who left the school in any of the years during the given period?

(a) 1.2%   (b) 1.7%   (c) 2.1%   (d) 2.4%

(a) 7:9    (b) 4:5    (c) 3:4    (d) 2:3

ANSWERS 1. (b)

4. (a)

7. (d)

10. (c)

13. (b)

16. (b)

19. (b)

2. (b)

5. (b)

8. (a)

11. (d)

14. (a)

17. (a)

20. (d)

3. (b)

6. (c)

9. (d)

12. (a)

15. (d)

18. (d)

EXPLANATIONS AND HINTS 1. (b) It is visible from the graph given that April showed the highest absolute difference in CPI over the previous month.

8. (a) Ratio of the expenditures of companies A and 100 130 × = 26 : 27 B in 1996 = 135 100

2. (b) For April and July, the CPI was greater than 350.

9. (d) We can calculate the profit for 1996. However, we do not know the income for 1997 and hence the data are insufficient to calculate the profit in 1997.

3. (b) It is visible from the graph given that April showed the highest percentage difference in the CPI over the previous month. 4. (a) The CPI increased in three months: April, June and July The CPI decreased in two months: March and May Hence, the difference = 3 − 2 = 1

10. (c) Total profit percent in 1998 = 20% Total profit percent in 1999 = 35% Total percent increase for company B from 1998 to 35 − 20 × 100 = 75% 1999 = 20

11. (d) Total export of companies X, Y, Z in 1995 = H(40 + 60 + 120) c 5. (b) The CPI decreased in March and May. Hence, 1995 = H (40 + 60 + 120) crore = H 220 crore there was a decrease in 2 months. 120 = H240 croreTotal export of companies X, Y, Z in 1996 = 6. (c) Income of company B in 1998 = 200 × 120 100 H (70 + 60 + 90)crore = H 220 crore 1998 = 200 × = H240 crore 100 100 Total export of companies X, Y, Z in 1997 = = H 375 crore 7. (d) Expenditure of company A in 2000 = 600 × 100 H (100 + 80 + 60)crore = H 240 crore 160 2000 = 600 × = H 375 crore 160

Chapter 03-2.indd 709

22-08-2017 07:09:19

710     Line Graph 

Total export of companies X, Y, Z in 1998 = H (50 + 100 + 80) crore = H 230 crore

•  In

1998: Number of students left = 400 and number of students joined = 450. •  In 1999: Number of students left = 350 and Hence, export of companies X, Y, Z was same in number of students joined = 500. 1995 and 1996. •  In 2000: Number of students left = 450 and 1 560 12. (a) Average annual exports of company Y = × (80 + 40 + 60 + 60 + 80 +of100 + 140) joined = = 80 number students 7 7 = 400. 1 560 •  In 2001: Number of students left = 450 and Y = × (80 + 40 + 60 + 60 + 80 + 100 + 140) = = 80 7 7 number of students joined = 550. Therefore, the number of students studying in the 1 Average annual exports of company Z = ×(60 + 90 + 120school + 90 +in60various + 80 +years 100) is as follows: 7 1 600 •  In 1995 = 3000 (given). Z = ×(60 + 90 + 120 + 90 + 60 + 80 + 100) = 7 7 •  In 1996 = 3000 − 250 + 350 = 3100. •  In 1997 = 3100 − 450 + 300 = 2950.   80 •  In 1998 = 2950 − 400 + 450 = 3000. Therefore, r equired percentage =  × 100    (600/7) •  In 1999 = 3000 − 350 + 500 = 3150. •  In 2000 = 3150 − 450 + 400 = 3100. = 93.33% •  In 2001 = 3100 − 450 + 550 = 3200. 13. (b) Difference between the exports of company X Therefore, percentage increase in the strength of the and Y in 1995 = H (60 − 40) crore = H 20 crore  (3000 − 2950)  × 100 = 1.69%  1.7% school from 1997 to 1998 =    Difference between the exports of company and 2950  (3000X    − 2950)  Y in 1996 = H (70 − 60) cr ore = H 10 = crore × 100 = 1.69%  1.7%   2950   Difference between the exports of company X and 17. (a) The percentage rise/fall in the number of stuY in 1997 = H (100 − 80) cr ore = H 20 crore dents who left the school (compared to the previous year) during various years is: Difference between the exports of company X and Y in 1998 = H (100 − 50) cr ore = H 50 crore Hence, the difference in exports of company X and company Y is minimum in 1996.

 (450 − 250)  × 100 % = 80% rise For 1997 =   250    (450 − 400)  × 100 % = 11.11% fall For 1998 =   450    (400 − 350)  × 100 % = 12.5% fall For 1999 =   400    (450 − 350)  × 100 % = 28.57% rise For 2000 =   350    (450 − 450)  × 100 % = 0% rise For 2001 =   450   Clearly, the maximum percentage rise/fall is for 1997.

14. (a) Average exports of three companies X, Y, Z in 1 170 1993 = ×(30 + 80 + 60) = H crore 3 3 Average exports of three companies X, Y, Z in 230 1 crore 1998 = ×(50 + 100 + 80) = H 3 3   170  60  230 Difference =  − = H 20 crore = 3 3   3   15. (d) Average annual exports of company Z during 1 the given period is ×(60 + 90 + 120 + 90 + 60 + 80 + 100 ) (d) The number of students studying in the school 18. 7 in 1999 = 3000 − 350 + 500 = 3150 600 = H 85.71 crore 0 + 90 + 120 + 90 + 60 + 80 + 100) = 19. (b) Total students studying in the school in 1998 = 7 3000 It is visible from the graph given that the exports Total students studying in the school in 2001 = 3200 of company Z are more than the average annual 3000 × 100 = 93.75% Required percentage = exports of company Z during 1994, 1995, 1996 and 1999, that is, 4 years. 3200 1994, 1995, 1996 and 1999, that is, 4 years. 20. (d) Least number of students who joined in any year = 300 16. (b) From the graph, it can be noted that

   

•  In

1996: Number of students left = 250 and number of students joined = 350. •  In 1997: Number of students left = 450 and number of students joined = 300.

Chapter 03-2.indd 710

Maximum number of students who left in any year = 450 300 2 = Required ratio = 450 3

22-08-2017 07:09:23

CHAPTER 3

TABLES

INTRODUCTION

Tables differ significantly in variety, structure, flexibility, ­notation, representation and use.

A table is used to represent data in rows and ­columns. Tables are arguably the most common way of arranging a given data. Evidently, tables are used almost ­everywhere, in print media, books, presentation slides, etc.

Tables are advantageous to us as they can arrange a large amount of data easily and without any ­complexity. However, locating pattern or visual observations are difficult.

SOLVED EXAMPLES Direction for Q1—Q5: The following table shows sales of cars of five companies over the years (in thousand). Company

Years 2000

2001

2002

2003

2004

Honda

20

21

50

35

75

Hyundai

29

31

23

46

42

Maruti

31

29

27

22

16

Tata

33

14

33

37

48

Skoda

15

17

32

39

47

Total

128

112

165

179

228

Consider the table and answer the following questions.

Chapter 03-3.indd 711

22-08-2017 07:09:57

712        TABLES 

Total sales of Tata = 33 + 14 + 33 + 37 + 48 = 165000 Total sales of Skoda = 15 + 17 + 32 + 39 + 47 = 150000 Hence, Honda had the highest sales from 2000 to 2004.

1. For which company did the amount of sales of cars increase continuously over the years? Solution:  For Skoda, we can see that the sales of each year are always greater than the previous year. 2. For which company did the amount of sales of cars decrease continuously over the years?

Direction for Q6—Q10: The following table shows total revenue of five companies over the years (in $ millions).

Solution:  For Maruti, we can see that the sales of each year are always lesser than the previous year.

Company 3. In 2001, which company had the biggest increase in sales from 2000? Solution:  Three companies had more sales in 2001 than 2000. 21 − 20 × 100 = 5% Percentage increase for Honda = 20 Percentage increase for Hyundai = 31 − 29 × 100 = 6.9% 29 Percentage increase for Skoda =

31 − 29 × 100 = 6.9% 29

A B C D E

Years 2000

2001

2002

2003

20 10 17 6 18

17 12 15 16 30

8 16 4 3 10

18 20 2 9 11

Consider the table and answer the following questions. 17 − 15 ×100 = 13. 33 % 6. What was the total revenue of company D from 15 2000 to 2003?

17 − 15 ×100 = 13. 33% 15 Hence, Skoda had the biggest percentage increase in sales of 2001. 4. In which year did Maruti have the largest percentage decrease in sales?

Solution:  Total revenue of D = 6 + 16 + 3 + 9 = $ 34 million 7. What was the revenue of all the companies in the year 2002?

Solution:  Total revenue is 2002 = 8 + 16 + 4 + 3 + Solution:  Percentage decrease in sales in 2001 = 10 = $ 41 million 31 − 29 ×100 = 6.45% 8. From 2000 to 2003, in which company was there a 31 continuous increase in revenue? 29 − 27 ×100 = 6.9% Percentage decrease in sales in 2002 = Solution:  Company B had a continuous increase 29 29 − 27 in revenue from 2000 to 2003. ×100 = 6.9% 29 9. In which two years did company B and D have the 27 − 22 ×100 = 18.52same Percentage decrease in sales in 2003 = % amount of revenue? 27 27 − 22 Solution:  In 2000 and 2001, revenue of company ×100 = 18.52% 27 B = 10 + 12 = $ 22 million 22 − 16 ×100 = 27.27In%2000 and 2001, revenue of company D = 6 + 16 = Percentage decrease in sales in 2004 = 22 $ 22 million 22 − 16 ×100 = 27.27% Hence, in 2000 and 2001, the sum of revenue of B 22 and D is same. Hence, in 2004, the decrease in sales for Maruti was the highest. 10. What was the percentage decrease in total revenue from 2001 to 2003? 5. Which company sold the most number of cars in the five years? Solution:  Total sales of Honda = 20 + 21 + 50 + 35 + 75 = 201000 Total sales of Hyundai = 29 + 31 +23 + 46 + 42 = 171000 Total sales of Maruti = 31 + 29 + 27 + 22 + 16 = 125000

Chapter 03-3.indd 712

Solution:  Total revenue in 2001 = 17 + 12 + 15 + 16 + 30 = $ 90 million Total revenue in 2003 = 18 + 20 + 2 + 9 + 11 = $ 60 million Percentage decrease in revenue from 2001 to 2003 = 90 − 60 × 100 = 33.33% 90

22-08-2017 07:09:59

PRACTICE EXERCISE        713

PRACTICE EXERCISE Direction for Q1—Q5: The following table depicts countries foreign trade (in crores) from 1990—1991 to 1995—1996.

2. In which year the ratio of imports to exports was maximum? (a) 1993—94 (c) 1991—92

Year

Export

Import

Deficit

1990—1991

  4661

  8248

3587

1991—1992

  5790

11606

5816

1992—1993

  8019

13750

5731

1993—1994

  6756

14110

7354

1994—1995

10420

18300

7880

1995—1996

12550

20010

7460

Consider the table and answer the following questions. 1. Which of the following showed an increase every year? (a) Exports (c) Deficit

(b) 1992—93 (d) 1990—91

3. The percentage increase in exports over previous year was maximum in which year? (a) 1991—1992 (c) 1994—1995

(b) 1992—1993 (d) 1995—1996

4. What was the total trade deficit for the last three years? (a) H 22694 crore (c) H 18980 crore

(b) H 21745 crore (d) H 24407 crore

5. In which year was the difference between imports and exports maximum? (a) 1990—1991 (c) 1992—1993

(b) Imports (d) All of the above

(b) 1991—1992 (d) 1994—1995

Direction for Q6—Q10: The following table depicts productions of main crops in India (in ­million tons). Crops

Years 2000—01

2001—02

2002—03

2003—04

2004—05

Pulses

20.5

24.6

28.2

23.5

22.4

Oats

32.4

40.8

52.4

42.4

34.6

Rice

80.4

88.2

90.8

92.6

86.4

Sugarcane

140.8

152.2

172.5

160.3

150.2

Wheat

130.2

146.8

158.4

141.6

138.4

Consider the table and answer the following questions.

6. What was the total amount of crops produced in 2002—03? (a) 479.5 (b) 510.7 (c) 502.3 (d) 468.4

million million million million

tons tons tons tons

7. What was the total production of rice? (a) 410.2 million tons (b) 422.5 million tons (c) 438.4 million tons (d) 472. 6 million tons 8. Production of which type of crop was always increasing in the given years?

Chapter 03-3.indd 713

(a) Pulses (c) Sugarcane

(b) Oats (d) None

9. What was the average production of oats in the given years? (a) 40.52 (b) 39.65 (c) 42.25 (d) 37.28

million million million million

tons tons tons tons

10. Production of wheat was what percentage of total production in 2002—03? (a) 29.71% (c) 34.75%

(b) 32.06% (d) 31.53%

22-08-2017 07:09:59

714        TABLES 

Direction for Q11—Q15: The following table consists of average number of people receiving certain assistance and the cost required for that assistance. Category of assistance

Average number receiving help

Total cost of help (in crores, H)

Cost paid by Centre (in crores, H)

 

1995

1996

1995

1996

1995

1996

A

36097

38263

38.4

34.8

18.4

17.4

B

  6632

5972

5.0

3.2

2.6

1.6

C

32545

31804

76.4

59.4

13.0

10.0

D

13992

11782

26.4

42.6

6.6

10.6

E

21275

228795

216.6

242.8

55.0

62.6

Consider the table and answer the following questions.  (c) H 2.1 crore less than that in 1995 (d) H 1.8 crore more than that in 1995

11. The category receiving the least percentage help from the centre (in the entire data) is (a) Category (b) Category (c) Category (d) Category

B in 1995 C in 1996 B in 1996 D in 1995

14. Assuming that 50% of the persons receiving category B help in 1995 were adults caring for minor children, but the city’s contribution towards maintaining these adults was 40% of the total contribution to B program in 1995, average amount paid by the city for each adult per year in 1995 is nearly

12. The difference between the average costs paid by the centre during 1995 and 1996 is (a) H 66 lakh (c) H 132 lakh

(a) H 5900 (c) H 7500

(b) H 13.2 crore (d) H 13.2 lakh

(b) H 6000 (d) H 3000

15. Monthly costs to the city of category D during 1995 and 1996 bear a ratio

13. Monthly cost of the city receiving E category assistance in 1996 is nearly  (a) H 1.8 crore less than that in 1995 (b) H 2.1 crore more than that in 1995

(a) 2:3 (c) 3:2

(b) 5:3 (d) 3:5

Direction for Q16—Q20: The following table depicts the number of candidates that appeared and qualified in a competitive examination from different states over the years. State

Year 1997

1998

1999

2000

2001

Appeared Qualified

Appeared Qualified

Appeared Qualified

Appeared Qualified

Appeared Qualified

M

5200

720

8500

980

7400

850

6800

775

9500

1125

N

7500

840

9200

1050

8450

920

P

6400

780

8800

1020

7800

890

9200

980

8800

1020

8750

1010

9750

1250

Q

8100

950

9500

1240

8700

980

9700

1200

8950

995

R

7800

870

7600

940

9800

1350

7600

945

7990

885

Consider the table and answer the following questions. 16. Total number of candidates qualified from all stages together in 1997 is approximately what percentage of the total number of candidates qualified from all the states together in 1998? (a) 80% (c) 75%

Chapter 03-3.indd 714

17. What is the average number of candidates who appeared from state Q during the given years? (a) 8700 (c) 8990

(b) 8760 (d) 8920

(b) 77% (d) 73%

22-08-2017 07:09:59

EXPLANATIONS AND HINTS        715

(a) 10.87% (c) 12.35%

18. In which of the given years, the number of candidates that appeared from state P has maximum percentage of qualified candidates? (a) 1997 (c) 1999

(b) 11.49% (d) 12.54%

20. What is the percentage of candidates that qualified over the candidates that appeared from state N during all the years together?

(b) 1998 (d) 2001

19. Combining the states P and Q together in 1998, what is the percentage of the qualified candidates to that of the appeared candidates?

(a) 12.36% (c) 11.47%

(b) 12.16% (d) 11.15%

ANSWERS 1.  (b)

5.  (d)

  9.  (a)

  13.  (b)

  17.  (c)

2.  (a)

6.  (c)

10.  (d)

  14.  (b)

  18.  (d)

3.  (c)

7.  (c)

11.  (b)

  15.  (d)

  19.  (c)

4.  (a)

8.  (d)

12.  (c)

  16.  (a)

  20.  (d)

EXPLANATIONS AND HINTS 1. (b) As it is visible from the table, imports showed an increase every year. 2. (a) Ratio of import to export in 1990 —1991 = 8248 = 1.77 4661 Ratio of import to export in 1991—1992 = 11606 = 2.00 5790 Ratio of import to export in 1992—1993 = 13750 = 1.71 8019 Ratio of import to export in 1993—1994 = 14110 = 2.09 6756 Hence, the ratio was maximum for 1993—94. 3. (c) Percentage increase in 1991—1992 = 5790 − 4661 × 100 = 24.22% 4661 Percentage increase in 1992—1993 = 8019 − 5790 × 100 = 38.497% 5790 Percentage increase in 1994—1995 = 10420 − 6756 × 100 = 54.23% 6756 Percentage increase in 1995—1996 = 12550 − 10420 × 100 = 20.44% 10420

Chapter 03-3.indd 715

Hence, the maximum percentage increase was in 1994—1995. 4. (a) Total trade deficit for the last three years = (7354 + 7880 + 7460) = H22694 crore 5. (d) As it is visible from the table, the maximum trade deficit was in 1994—1995. 6. (c) Total crops production in 2002—03 = 28.2 + 52.4 + 90.8 + 172.5 + 158.4 = 502.3 million tons 7. (c) Total production of rice = 80.4 + 88.2 + 90.8 + 92.6 + 86.4 = 438.4 million tons 8. (d) As it is visible from the table, production of all caps decreases at least once in five years. 9. (a) The average production of oats is the given 32.4 + 40.8 + 52.4 + 42.4 + 34.6 years = 5 202.6 = = 40.52 million tons 5 10. (d) Total production of wheat in 2002—03 = 158.4 million tons Total production of all crops in 2002—03 = 502.3 million tons 158 × 100 = 31.53% Total percentage = 502.3 2.6 × 100 = 52% 11. (b) Percentage for category B in 1995 = 2.6 5.0 = × 100 = 52% 5.0

 

 

22-08-2017 07:10:01

716        TABLES 

Percentage for category C in 1996 = =

1059..04  ×100 = 16.8% Percentage for category B in 1996 =

=

 13..62  ×100 = 50% Percentage for category B in 1995 =

=

55.0 × 100 = 25.4%  216 .6 

1059..04  ×100 = 16.8%

16. (a) Total candidates who qualified from all states in 1997 = 720 + 840 + 780 + 950 + 870 = 4160 Total candidates who qualified from all states in 1998 = 980 + 1050 + 1020 + 1240 + 940 = 5230

 13..62  ×100 = 50%

4160 20000000 × 100 ==6031 %  H6000 79.54.36 80% 5230 3316 

Required percentage ==

55.0 × 100 = 25.4%  216 .6 

Thus, the least percentage of help from Center is for category C in 1996.

12. (c) The average cost in 1995 = 18.4 + 2.6 + 13.0 + 6.6 + 55.0 95.6 = =H19.12 crore 5 5 The average cost in 1996 = 17.4 + 1.6 + 10.0 + 10.6 + 62.6 102.2 = = H20.44 crore 5 5 + 10.6 + 62.6 102.2 = = H20.44 crore 5

17. (c) Total students who appeared from state Q in the given five years = 8100 + 9500 + 8700 + 9700 + 8950 = 44950 44950 Therefore, required average = = 8990 5 18. (d) The percentage of qualified candidates to appeared candidates from state P during different years is:

Difference = 20.44 − 19.12 = H 1.32 crore = H 132 lakh − 19.12 = H 1.32 crore = H 132 lakh 13. (b) Monthly cost of city receiving E category assis216.6 tance in 1995 = = 18.05 12 Monthly cost of city receiving E category assis242.8 tance in 1996 = = 20.23 12 Difference = 20.23 − 18.05 = 2.183 crore  H 2.1 crore

For 1997:

780  6400  ×100 = 12.19%

For 1998:

× 100 = 11.59% 1020 6400

For 1999:

890 × 100 = 11.41% 7800

For 2000:

× 100 = 11.54% 1010 8750

For 2001:

× 100 = 12.82% 1250 9750

Therefore, the maximum percentage is for the year 2001.

19. (c) Total candidates who qualified in 1998 for P and Q = 1020 + 1240 = 2260 14. (b) 50% of persons receiving B category help Total candidates who applied in 1998 for P and Q = during 1995 = 3316 40 8800 + 9500 = 18300 City’s contribution to maintenance = × 5 = H 2 crore 2260 100 Therefore, required percentage = × 100 % = 12.35% 40 = × 5 = H 2 crore 18300 2260 100 20000000 = = 6031 ×.36 = 12.35% 100%H6000 Therefore, average amount paid = 3316 18300 20000000 20. (d) Total candidates who qualified from state N = = = 6031.36  H6000 3316 840 + 1050 + 920 + 980 + 1020 = 4810 26.4 Total candidates who appeared from state N = 15. (d) Monthly costs of category D in 1995 = 7500 + 9200 + 8450 + 9200 + 8800 = 43150 12 4810 42.6 Therefore, required percentage = × 100 = 11.15% Monthly costs of category D in 1996 = 43150 12 4810 Therefore, required ratio = 26.4:42.6 = 3:5. = × 100 = 11.15% 43150





Chapter 03-3.indd 716













22-08-2017 07:10:04

CHAPTER 4

BLOOD RELATIONSHIP

Table 1 |   Description of relation

INTRODUCTION

Description

Relation

Mother’s/Father’s brother

Uncle

Mother’s/Father’s sister

Aunt

Mother’s/Father’s mother

Grandmother

1. First stage: Grandparents − Grandfather, grandmother, grand uncle and grand aunt.

Mother’s/Father’s father

Grandfather

Grandmother’s/Grandfather’s brother

Grand uncle

2. Second stage: Parents and in-laws − Father, mother, uncle, aunt, father-in-law and motherin-law.

Grandmother’s/Grandfather’s sister

Grand aunt

Children’s children

Grandchildren

Uncle or Aunt’s son/daughter

Cousins

Daughter’s husband

Son-in-law

Son’s wife

Daughter-in-law

Sister’s/Brother’s daughter

Niece

GATE comprises of various questions on blood relations. The easiest and non-confusing way to solve these questions is to draw a family tree diagram and increase the levels in the hierarchy as follows:

3. Third stage: Siblings, spouse and in-laws − Brother, sister, cousin, wife, husband, brotherin-law and sister-in-law. 4. Fourth stage: Children and in-laws  −  Son, daughter, niece, nephew, son-in-law and daughter-in-law.

Sister’s/Brother’s son

Nephew

5. Fifth stage: Grandchildren − Grandson and grand­ daughter.

Wife’s/Husband’s brother or sister’s husband

Brother-in-law

Some of the common relationships are given in Table 1.

Wife’s/Husband’s sister or brother’s wife Sister-in-law

Chapter 03-4.indd 717

22-08-2017 07:10:35

718     Blood Relationship 

STANDARD CODING TECHNIQUE 1. Proper nouns should always be denoted by capital letters. Example: Mark and Rachel can be coded as Mark − M, Rachel − R. 2. In questions where the gender is of importance to solving it, you can denote females by underlining the notation used. Example: Eve is the only girl student in her course. Eve − E. 3. When defining a relationship, always put the elder generation on top of the younger generation. Hence, grandparents come above parents and parents come above children. Example: A is grandfather of B. B is mother of two boys C and D. This can be coded as follows:

4. When defining spousal relationship, use →. Conventionally, write the male on the left and the female on the right. Example: Aman is Prerna’s husband. A → P. 5. When defining a relationship between siblings, use `..’. Example: Sana and Shreya are sisters. S..SH Anupam is Kittu’s brother. A..K 6. When the gender of a person is not confirmed, write it in a box. Example: A is the only son of B. B A

A B C   D

SOLVED EXAMPLES 1. Leena and Meena are Rahul’s wives. Shalini is Meena’s stepdaughter. How is Leena related to Shalini? Solution:  Shalini is Meena’s stepdaughter. Hence, Shalini is the daughter of the other wife of Rahul, so Shalini is the daughter of Leena or Leena is Shalini’s mother. 2. Pointing to a man in a photograph a man said to a woman “His mother is the only daughter of your father.” How is the woman related to the man in the photograph? Solution:  The only daughter of the woman’s father is the woman herself, and hence the man in the photograph is her son. Therefore, the woman is the mother of the man in the photograph. 3. A party consists of grandmother, father, mother, four sons and their wives, and one son and two daughters of each of the sons. How many females are there? Solution:  Grandmother is one female, mother is another, wives of four sons are another 4 and two daughters of each son make it 8 more females. Hence, total females = 1 + 1 + 4 + 8 = 14 4. Introducing Marcelle to guests, Aman said, “Her father is the only son of my father.” How is Marcelle related to Aman?

Chapter 03-4.indd 718

Solution:  According to Aman, Marcelle’s father is the only son of Aman’s father. Hence, Aman is Marcelle’s father. Therefore, Marcelle is the daughter of Aman. Direction for Q5 and Q6: A + B means A is the ­daughter of B. A × B means A is the son of B. A − B means A is the wife of B. 5. What is the meaning of P × Q − S ? Solution:  P × Q − S means P is the son of Q who is the wife of S. 6. If Z × T − S × U + P, then what is the relationship between U and Z? Solution:  Z × T − S × U means Z is the son of T who is the wife of S who is the son of U, that is Z is the son of S who is the son of U, that is Z is the grandson of U or U is the grandmother of Z. Direction for Q7−Q10: A is married to B and L is A’s brother-in-law. A has two daughters. I is the cousin brother of J and is the brother of K. E and F are B’s son-in-laws. E has two daughters and one son, F has one son and one daughter. G and H are two daughters of C. K and A share a granddaughter and a grandfather relationship. D is also the member of this family. 7. How is C related to I?

22-08-2017 07:10:36

PRACTICE EXERCISE     719

8. How is L related to D?

Direction for Q11−Q12: A + B means A is the daughter of B.

9. How is E related to C?

A + B means A is the daughter of B.

10. How is G related to B?

A × B means A is the brother of B.

Solution:  Now, A − B..L

11. What is the meaning of P + Q × R ?

A has two daughters, D1 and D2 .

Solution:  P + Q × R means that P is the daughter of Q who is the brother of R.

I is K’s brother and cousin of J. Hence, I..K, J E and F are husbands of D1 and D2 . Therefore,

12. If P × Q + R, then what is the relation between P and R?

E − D1 ..D2 − F Now, E has two daughters and one son,

Solution:  P × Q + R means P is the brother of Q who is the daughter of R. Hence, P is the son of R.

E − D1 D1′ ..D2′ ..s1′ and D2 − F has one son and one daughter

Direction for Q13−Q15: Read the information carefully and answer the following questions.

F − D2



1. A family consists of 6 members P, Q, R, X, Y, Z.

D3′ ..s′2



2. Q is the son of R but R is not the mother of Q.



3. P and R is a married couple.



4. Y is the brother of R, X is the daughter of P.



5. Z is the brother of P.

G and H are daughters of C, so C is wife of E because only E has two daughters. Now, the complete tree can be formed as follows: A − B..L

13. How is Q related to X? Solution:  From the information given above, we can draw the following tree:

E − C .. D − F ¯

¯



H .. G ..J, I.. K 7. C is the sister of D who is I’s mother. Hence, C is the aunt of I. 8. D is daughter of B, L is B’s brother. Thus, L is D’s uncle. 9. E and C have two daughters and E is the son-inlaw of A and B. C is daughter of A and B. Thus, E is husband of C. 10. We know that G is daughter of C. Also, C is daughter of B.

Y..R → P..Z Q..X

Thus, Q is the brother of X. 14. How is Y related to P? Solution:  Y’s brother is P’s husband. Hence, Y is P’s brother in law. 15. How many females are there in the family? Solution:  There are two females in the family, P and X.

Hence, G is B’s granddaughter.

PRACTICE EXERCISE Direction for Q1−Q5: Read the following information carefully and answer the questions. All the six members of a family P, Q, R, S, T and U are travelling together. Q is the son of R but R is not the mother of Q, and P and R are a married couple. T is the brother of R. S is the daughter of P. U is the brother of Q.

Chapter 03-4.indd 719

1. Who is the mother of Q? (a) S (b) U (c) T (d) P 2. How many children does P have? (a) One (c) Three

(b) Two (d) Four

22-08-2017 07:10:39

720     Blood Relationship 

3. How is U related to S? (a) Father (c) Uncle

(b) Brother (d) Indeterminate

Direction for Q11−Q13: P is the son of Q. Q’s sister has a son S and daughter T. U is the maternal uncle of S. 11. How many nephew(s) does U have?

4. Who is the wife of T? (a) P (c) Q

(b) U (d) Indeterminate

(a) Zero (c) Two

(b) One (d) Three

12. How is P related to S? 5. How many male members are there in the family? (a) 1 (c) 3

(b) 2 (d) 4

(a) Cousin (c) Uncle

(b) Nephew (d) Brother

13. How is T related to U? 6. Looking at the portrait of a man, Tinku said, “His mother is the wife of my father’s son. I have no brothers or sisters.” At whose portrait was Tinku looking? (a) His child (c) His grandson

(b) His father (d) His brother

7. If A + B → A is the mother of B A − B → A is the brother of B A%B → A is the father of B A × B → A is the sister of B then which of the following shows P is the maternal uncle of Q? (a) P − M + N × Q (c) Q − S%P

(b) Q − N + M × P (d) P + S × N − Q

8. If A is the brother of B, B is the sister of C and C is the father of D, how is D related to A? (a) Brother (c) Uncle

(b) Sister (d) Indeterminate

9. Pointing to a gentleman, Deepak said “His only brother is the father of my daughter’s father.” How is the gentleman related to Deepak? (a) Brother (c) Uncle

(b) Father (d) Grandfather

10. If A + B means A is the brother of B A − B means A is the sister of B

 (a)

Sister (b) Daughter  (c) Niece (d) Wife Direction for Q14−Q16: A + B means A is the daughter of B. A − B means A is the husband of B. A × B means A is the brother of B. 14. For P + Q − R, which of the following is true?  (a) (b)  (c) (d)

R R R R

is is is is

the the the the

mother of P sister-in-law of P aunt of P mother-in-law of P

15. For P × Q + R, which of the following is true?  (a) (b)  (c) (d)

P P P P

is is is is

the the the the

brother of R uncle of R son of R father of R

16. For P + Q × R, which of the following is true?  (a) (b)  (c) (d)

P P P P

is is is is

the niece of R the daughter of R cousin of R the daughter-in-law of R

Direction for Q17−Q20: A family consists of six members P, Q, R, X, Y and Z. Q is the son of R but R is not the mother of Q. P and R are married couple. Y is the brother of R, X is the daughter of P. Z is the brother of P.

and A × B means A is the father of B, 17. Who is the brother-in-law of R? then which of the following means that C is the son of M? M − N×C + F (b) M × N − C + F  (c) F − C + N × M (d) N + M − F × C

(a) P (c) Y

(b) Z (d) X

 (a)

Chapter 03-4.indd 720

18. How many children does P have? (a) One (c) Three

(b) Two (d) Four

22-08-2017 07:10:41

EXPLANATIONS AND HINTS     721

19. How many female members are there in the family? (a) One (c) Three

20. Which is a pair of brothers? (a) P and X (c) Q and X

(b) Two (d) Four

(b) P and Z (d) R and Y

ANSWERS 1.  (d)

5.  (d)

  9.  (c)

  13.  (c)

  17.  (b)

2.  (c)

6.  (a)

10.  (b)

  14.  (a)

  18.  (b)

3.  (b)

7.  (a)

11.  (c)

  15.  (c)

  19.  (b)

4.  (d)

8.  (d)

12.  (a)

  16.  (a)

  20.  (d)

EXPLANATIONS AND HINTS Solution to Q1−Q5:

Hence, if P is the maternal uncle of Q, then P − M + N × Q.

Q is son of R R is not the mother, R is the father of Q. P and R are married couple, hence, P − R T is R’s brother, hence T..R − P

8. (d) If D is male, the answer is nephew. And if D is female, then the answer is niece. However, the sex of D is not known, the relation of D and A cannot be determined.

S is P’s daughter. U is brother of Q, hence Q.. U

9. (c) Father of my daughter’s father is Deepak’s father.

The tree can be formed as follows: Brother of Deepak’s father is Deepak’s uncle. T..R − P Q.. U..S

10. (b) M is the father of N → M × N N is the sister of C → N − C

1. (d) The mother of Q is the wife of R which is P.

C is the brother of F → C + F

2. (c) P has three children, Q, U and S.

Thus, M is the father of C if M × N − C + F.

3. (b) U is the brother of S. 4. (d) There is no mention of T’s wife. Hence, it is indeterminate. 5. (d) There are four male members in the family, T, R, Q and U. 6. (a) Tinku has no brothers or sisters, so Tinku is single child. Wife of father’s son is Tinku’s wife. Tinku’s wife is the mother of the person whose portrait it is. So portrait is of Tinku’s child. 7. (a) P is the brother of M → P − M M is the mother of N → M + N N is the sister of Q → N × Q

Chapter 03-4.indd 721

Solution to Q11−Q13:  From the data given in the question, we can form the tree as follows: Q.. R .. U | | P S.. T 11. (c) U has two nephews, a son of R and a son of Q. 12. (a) P is cousin brother of S. 13. (c) T is the daughter of the sister of U. Hence, T is the niece of U. 14. (a) P + Q − R means P is the daughter of Q who is the husband of R, i.e. R is P’s mother. 15. (c) P × Q + R means P is the brother of Q who is the daughter of R, i.e. P is the son of R.

22-08-2017 07:10:43

722     Blood Relationship 

16. (a) P + Q × R, means P is the daughter of Q who is the brother of R, i.e. P is the niece of R.

17. (b) Brother-in-law of R is brother of P, i.e. Z.

Solution to Q17−Q20:  From the data given we can form the family tree,

19. (b) There are two female members, P and X.

Y..R − P..Z Q..X

Chapter 03-4.indd 722

18. (b) P has two children Q and X.

20. (d) R and Y are the only pair of brothers.

22-08-2017 07:10:44

CHAPTER 5

BAR DIAGRAM

INTRODUCTION A bar diagram or bar graph is a chart with rectangular bars in which the lengths of bars are in proportion to the values of the entities they represent. A vertical bar chart is called a column bar chart (Fig. 1a) and a horizontal bar chart is called a row bar chart (Fig. 1b). Bar Diagram depicting average climate of a city from 50°C January to May

Revenue generated by 4 companies in 2008

Bar diagrams are often used to provide visual presentation of categorical data and are considered very reliable. Bar graphs can also be used for comparison of complex data. Also, a bar chart is very useful for displaying discrete data values. Multiple sets of interrelated data can be represented using a multiple bar diagram. It is used to compare more than one quantity. The technique of a simple bar diagram is used to draw the diagram. However, different quantities are represented using different shapes, sizes, colours, patterns, etc. Figure 2 shows a regular multiple bar diagram.

A

Profit (in million dollars)

Temperature

40°C 30°C B 20°C C 10°C D

Jan Feb Mar Apr May (a)

50 100 150 200 250 300 Revenue (in $ million) (b)

Figure 1 |   Bar diagram: (a) Column bar chart and (b) row bar chart.

Chapter 03-5.indd 723

80 70 60 50 40 30 20 10

Company A Company B

2000

2001

2002 Year

2003

Figure 2 |   Multiple bar diagram.

23-08-2017 05:38:11

724     Bar Diagram 

A compound bar diagram is used to combine or compare two or more types of information in one chart. The bars are stacked on top of one another. However, different quantities are represented using different shapes, sizes, colours, patterns, etc. Figure 3 shows a regular compound bar diagram. Total sales of cars (× 1000)

9 8 7 6 5 4 3 2 1

2000 2001 2002 2003 2004

100% Production in percentage

Total production of cars (× 1000)

90 80 70 60 50 40 30 20 10

A bar chart may be drawn on a percentage basis. To obtain a percentage bar diagram, bars of length equal to 100 for each class are drawn and subdivided in the proportion of the percentage of their component. Figure 4 shows a regular percentage bar diagram.

Wheat Rice

80% 60% 40% 20%

2000 2001 2002 2003 2004

0 2001

SUVs Sedan

2002 2003 Years

2004

Figure 4 |   Percentage bar diagram.

Figure 3 |   Compound bar diagram.

SOLVED EXAMPLES Direction for Q1-Q4: Study the following bar diagram carefully and answer the questions.

Total revenue (in million $) of a company from 1990 to 1995

80 75

Average revenue for 1993 and 1994 = $ 57.5 million

65 + 50 = 2

Hence, average revenue of 1993 and 1994 was same as 1990 and 1995.

70 65

2. In which year(s), the percentage increase in production was maximum from the previous year?

60 60 50

50 45

Solution:  Percentage increase from 1990 to 1991 is

40

60 − 40 × 100 40 20 = × 100 = 50% 40

40 30 20 10

Also, percentage increase from 1994 to 1995 is 1990 1991 1992 1993 1994 1995

1. Which pair of years had the same average revenue as that of 1990 and 1995? Solution:  Total revenue in 1990 = $ 40 million Total revenue in 1995 = $ 75 million 40 + 75 Average revenue in 1990 and 1995 = = 2 $ 57.5 million Total revenue in 1993 = $ 65 million

Chapter 03-5.indd 724

Total revenue in 1994 = $ 50 million

75 − 50 × 100 = 50% 50 Thus, percentage increase was maximum in 1991 and 1995. 3. What was the percentage drop in the revenue from 1991 to 1992? Solution:  Total revenue in 1991 = $ 60 million Total revenue in 1992 = $ 45 million

22-08-2017 07:11:25

SOLVED EXAMPLES     725

Percentage drop was

7. By what amount had the expenditure increased over the period 2000-2004?

60 − 45 × 100 60 15 = × 100 = 25% 60

Solution:  Total expenditure in 2000 = H 5100 Total expenditure in 2004 = H 8800 Total increase in expenditure from 2000-2004 was 8800 − 5100 = H 3700

4. What was the difference between revenue in 1995 and 1990? Solution:  Total revenue in 1995 = $ 75 million Total revenue in 1990 = $ 40 million

Direction for Q8-Q10: Study the following bar diagram carefully and answer the questions.

Difference in revenue = 75 − 40 = $ 30 million Direction for Q5-Q7: Study the following bar diagram carefully and answer the questions.

Business population as percent of total population 35%

A

38%

B

91.8% 82.4%

Total salary

6000

5400 5800

5300 5100

8000

9400 8800

8500 8000

7500 6900

10000

30%

C

Total expenditure

32%

D

0

57.3%

42.9%

20 40 60 80 100 Total population (in Lakhs)

4000

8. What is total business population of A? Solution:  Total population of A = 91.8 lakh Total business population = 35%

2000

2000

2001

2002

2003

2004

5. What was the percentage increase in the “Total Salary” in 2002 as compared to 2000?

=

35 × 91.8 lakh = 32.13 lakh 100

9. What place has the highest business population? Solution:  Business population of A =

Solution:  Total salary in 2002 = 7500 Total salary in 2000 = 5300 Percentage increase was 7500 − 5300 × 100 = 29.33% 7500 6. If profit = total salary - expenditure, then in 2003 what percentage of total salary was the profit made? Solution:  Total salary in 2003 = 8500 Total expenditure in 2003 = 8000

35 × 91.8 = 32.13 lakh 100

35 × 91.8 = 32.13 lakh 100 38 × 82.4 = 31.31 lakh Business population of B = 100 30 Business population of C = ×57.3 =17.19 lakh 100 32 Business population of D =  × 42.9 =13.728 lakh 100 Hence, A has the highest business population. 10. What is the difference between business populations of C and D?

Total profit = 500 500 Profit % = × 100 = 5.88% 8500

Chapter 03-5.indd 725

30 Solution:  Business population of C =  × 57.3 = 17.19 lakh 100 30 × 57.3 = 17.19 lakh 100

22-08-2017 07:11:26

726     Bar Diagram 

 245 / 3  Required percentage =  × 100 = 87.5%  280 / 3   

32 Business population of D =  × 49.9 =13.728 lakh 100 Difference between business population of C and D = 17.19 - 13.728 = 3.462 lakh

Sales (in thousand numbers)

Direction for Q11-Q15: Study the following bar diagram carefully and answer the questions. 120 110 100 90 80 70 60 50 40 30 20 10 0

14. Total sale of branch F for both the years is what percent of the total sales of branch C for both the years? Solution:  Total sales of branch F = (70 + 80) ×1000 Total sales of branch F = (95 + 110) ×1000

2000 2001

 70 + 80  Required percentage =  × 100 = 73.17%  95 + 110  15. What is the average sale of all the branches (in thousands) for the year 2000? Solution:  Average sales of all the six branches (in thousands) for the year 2000 =

A

B

C D Branches

E

F

Direction for Q16-Q20: Study the following bar diagram carefully and answer the questions.

11. What is the ratio of the total sales of branch B for both years to the total sales of branch D for both years?

110

Solution:  Total sales of branch B = (75 + 65) × × 1000 = 140000 Total sales of branch D = (85 + 95) × 1000 = 180000 140 7 = 180 9

12. What is the total sale of branches A, C and E together for both the years (in thousands)?

100 A 90 B 80 C 70 Percent

Required ratio =

1 × (80 + 75 + 95 + 85 + 75 + 70) = 80 6

60

D

50

E

40

F

Solution:  Total sales of branches A, C and E for both the years (in thousands)

30

= (80 + 105) + (95 + 110) + (75 + 95)

20 10

= 560 0

13. What percent of the average sales of branches A, B and C in 2001 is the average sales of branches A, C and F in 2000? Solution:  Average sales (in thousands) of branches 1 245 A, C and F in 2000 = × (80 + 95 + 70) = 3 3 Average sales (in thousands) of branches A, B and C in 2001 1 280 = × (105 + 65 + 110) = 3 3

Chapter 03-5.indd 726

2000 2001 Total no. of cars Total no. of cars produced = 350000 produced = 440000

16. What is the difference in the number of E-type cars produced in 2000 and that produced in 2001? Solution:  Total number of E-type cars produced (60 − 40) × 440000 in 2001 = = 88000 100

22-08-2017 07:11:29

SOLVED EXAMPLES     727

Total number of E-type cars produced in 2000 =

(45 − 25) × 350000 = 70000 100 Required difference = 88000 − 77000 = 11000

B = (95 − 85)% of 440000 =

10 × 440000 = 44000 100

A = (100 − 95)% of 440000 =

5 × 440000 = 22000 100

17. If the percentage of production of F-type cars in 2001 were the same as that in 2000, then the number of F-type cars produced in 2001 would have been?

Total number of cars of model B, E and F manufactured in 2000 = (52500 + 70000 + 87500) = 210000

Solution:  We are given that the percentage production of F-type cars in 2001 = percentage production of F-type cars in 2000 = 25%

19. If 85% of the C-type cars produced in each year were sold by the company, how many C-type cars remain unsold?

Then, number of F-type cars produced in 2001 = 25% of 4,40,000 = 110000 18. Total number of cars of model B, E and F manufactured in 2000 is? Solution:  In 2000, total number of cars produced = 350000 25 × 350000 = 87500 F = (25 − 0)% of 350000 = 100 20 × 350000 = 70000 E = (45 − 25)% of 350000 = 100 20 × 350000 = 70000 D = (65 − 45)% of 350000 = 100 C = (75 − 65)% of 350000 =

10 × 350000 = 35000 100

15 × 350000 = 52500 B = (90 − 75)% of 350000 = 100 A = (100 − 90)% of 350000 =

10 × 350000 = 35000 100

In 2001, total number of cars produced = 440000 F = (40 − 0)% of 440000 =

40 × 440000 = 176000 100

20 × 440000 = 88000 E = (60 − 40)% of 440000 = 100

Chapter 03-5.indd 727

Solution:  Number of C-type cars which remain 15 × 35000 = 5250 unsold in 2000 = 100 Number of C-type cars which remained unsold in 15 2001 = × 44000 = 6600 100 Therefore, total number of C-type cars which remained unsold = 5250 + 6600 = 11850 20. For which model was the percentage rise/fall in production from 2000 to 2001 minimum? Solution:  The percentage change in production from 2000 to 2001 for various models is:  (176000 − 87500)  × 100 = 101.14% rise F =   105000    (88000 − 70000)  × 100 = 25.71% rise E =   70000    (70000 − 66000)  × 100 = 5.71% fall D =   70000    (44000 − 35000)  × 100 = 25.71% rise C =   35000    (52500 − 44000)  × 100 = 16.19% fall B =   52500  

D = (75 − 60)% of 440000 =

15 × 440000 = 66000 100

 (35000 − 22000)  × 100 = 37.14% fall A =   35000  

C = (85 − 75)% of 440000 =

10 × 440000 = 44000 100

Therefore, minimum change is observed in model D.

22-08-2017 07:11:32

728     Bar Diagram 

PRACTICE EXERCISE Percentage of Boys & Girls in Attendance at Dance Class

Direction for Q1-Q5: Study the following graph carefully and answer the questions.

Boys Girls

Percentage

Production of wheat by a state (1000 tons)

80 70 65 60 60 50

50 45 40

100 90 80 70 60 50 40 30 20 10

(Total = 1244) 75%

(1300) 67%

(1000) 56% 44%

(1600) 51% 49% 33%

25%

40 0 2007 30

25

2008

2009

2010

6. How many boys attended dance class in 2007? (a) 311

20

(b) 404

(c) 300

(d) 364

7. Which year had the largest number of boys attending dance school?

10

2000

2001

2002

2003

2004

2005

(a) 2007

(b) 2008

(c) 2009

(d) 2010

8. Which year had the lowest number of girls attending dance school? 1. What was the difference in the production of wheat between 2002 and 2004? (a) 10000 tons (c) 20000 tons

(b) 15000 tons (d) 5000 tons

2. What was the percentage increase in production of wheat from 2001 to 2002? (a) 25% (c) 75%

(b) 50% (d) 100%

3. What was the percentage decrease in production of wheat from 2002 to 2003? (a) 10% (c) 25%

(b) 20% (d) 33.3%

(a) 2007

(b) 2008

(c) 2009

(d) 2010

9. In which year(s) shown did approximately the same numbers of girls and boys attend the camp? (a) 2008 (c) 2007 and 2008

(b) 2009 (d) 2008 and 2010

10. In how many years was the number of girls more than the average number of girls? (a) 1 year (c) 3 year

(b) 2 year (d) Indeterminate

Direction for Q11-Q15: Study the following diagram carefully and answer the questions. Demand

4. The average production of wheat in 2001 and 2004 was equal to the average of which two years? (a) 2000 and 2001 (c) 2002 and 2003

(b) 2001 and 2003 (d) 2003 and 2005

Production 3500

3300 3000

3000 2500

2700

2500

5. In how many given years was the production of wheat more than average production of all years?

2200 2000

1800 1500

1500

1200

(a) 3 (c) 1

(b) 2 (d) 4

1000

1000 600 500

Direction for Q6-Q10: Study the following graph carefully and answer the questions:

Chapter 03-5.indd 728

I

II

III IV Organizations

V

22-08-2017 07:11:32

EXPLANATIONS AND HINTS     729

11. What is the difference between average demand Total production of cars 90 (× 1000) and average production of organizations I, II and 80 IV taken together? 70

(a) 400

(b) 200

(c) 500 60 (d) 800

12. Demand of organization II is what 50 percent of orga40 nization I? 30

(a) 10%

(b) 15%

(c) 25% 20 (d) 20%

13. The production of organization II10 is how many 2000IV? 2001 2002 2003 2004 times that of production of organization (a) 2/3

(b) 2

(c) 3

(b) 4:1

(c) 3:2

(d) 3/2

production of sedans in 2002?

(b) 33.33% (c) 20%

Total production of cars (× 1000)

(a) 13500 (c) 15000

2000 2001 2002 2003 2004

(b) 17500 (d) 14500

18. How many SUVs were produced from 2000 to 2004?

Total sales of(a) cars175000 (× 1000)

9 8 7 6 5 4 3 2 1

(b) 10% (d) 8%

17. How many sedans were sold from 2000 to 2004?

(d) 30%

Direction for Q16-Q20: Study the following bar diagram carefully and answer the questions. 90 80 70 60 50 40 30 20 10

(a) 16% (c) 25%

(d) 1:4

15. Production of organization III is what percent of organization I? (a) 15%

2000 2001 2002 2003 2004

SUVs Sedan 16. Sales of session in 2002 were what percentage of

14. What is the ratio of demand and production of organization V? (a) 2:3

Total sales of cars (× 1000)

9 8 7 6 5 4 3 2 1

(c) 120000

(b) 145000 (d) 155000

19. What was the ratio of total SUVs produced in 2001 to the total cars produced? (a) 2:5 (c) 3:5

(b) 5:2 (d) 4:7

20. What percent of total cars produced from 2000 to 2004 were sold? 2000 2001 2002 2003 2004

(a) 15.5% (c) 10.1%

SUVs Sedan

(b) 17.3% (d) 12.2%

ANSWERS 1.  (d)

 4.  (c)



7.  (c)

10.  (b)

  13.  (a)

  16.  (a)

   19.  (c)

2.  (b)

 5.  (a)



8.  (c)

11.  (a)

  14.  (c)

  17.  (d)

   20.  (d)

3.  (c)

 6.  (a)



12.  (d)

  15.  (b)

  18.  (b)

9.  (b)

EXPLANATIONS AND HINTS 1. (d) Production of wheat in 2002 = 60000 tons

3. (c) Productions in 2002 = 60000 tons

Production of wheat in 2004 = 65000 tons

Productions in 2003 = 45000 tons

Difference between production of wheat = 5000 tons

Total decrease = 15000 tons

2. (b) Total production of wheat in 2001 = 40000 tons

Total percentage decrease =

Total increase = (60000 - 40000) tons = 20000 tons Total percentage increase =

Chapter 03-5.indd 729

20000 × 100 = 50% 40000

15000 × 100 = 25% 60000

4. (c) Total productions of wheat in 2001 and 2004 = 40 + 65 = 105000 tons

22-08-2017 07:11:33

730     Bar Diagram 

Total productions of wheat in 2002 and 2003 = 40 + 65 = 105000 tons Hence, average production in same as 2001 and 2004 in 2002 and 2003. 5. (a) Total production of wheat from 2000 to 2005 = 25 + 40 + 60 + 45 + 65 + 50 = 285000 tons 285000 Average production per year = = 47500 tons 6 Therefore, the production is less than the average for 3 years. 6. (a) Total students in 2007 = 1244 25 Total boys = × 1244 = 311 100 7. (c) Total boys who attended dance school in 2007 were 25 × 1244 = 311 100 Total boys who attended dance school in 2008 were 44 × 1000 = 440 100 Total boys who attended dance school in 2009 were 57 × 1600 = 816 100

Average number of girls = 787 Hence, girls are more than the average for 2 years. 11. (a) Average demand of organizations I, II and IV is 3000 + 600 + 1200 = 1600 3 Average production of organizations I, II and IV is 1500 + 1800 + 2700 = 2000 3 Difference between average production and average demand = 400 12. (d) Demand of organization I = 3000 Demand of organization II = 600 600 Total percentage = × 100 = 20% 3000 13. (a) Production of organization II = 1800 Production of organization IV = 2700 1800 2 = 2700 3 Hence, production of organization II is 2/3 times organization IV. Now,

14. (c) Demands of organization V = 3300 Productions of organization V = 2200 Ratio of demands to production = 3300 : 2200 = 3 : 2

Total boys who attended dance school in 2010 = 15. (b) Production of organization III = 1000 33 × 1300 = 429 100 Hence, maximum number of boys who attended dance school were 816 in 2009. 8. (c) Total girls who attended dance school in 2007 were 75 × 1244 = 933 100 Total girls who attended dance school in 2008 were 56 × 1000 = 560 100 Total girls who attended dance school in 2009 = 49 × 1600 = 784 100 Total girls who attended dance school in 2010 = 67 × 1300 = 871 100 Hence, the lowest number of girls who attended dance school was 784 in 2009. 9. (b) In 2009, the difference between percentage of boys and girls was approximately same. 10. (b) Total number of girls = 933 + 560 + 784 + 871 = 3148

Chapter 03-5.indd 730

Production of organization I = 1500 Total percentage =

500 × 100 = 33.33% 1500

16. (a) Sales of sedans in 2002 = 4000 Production of sedans in 2002 = 25000 Now, total percentage =

4000 × 100 = 16% 25000

17. (d) Total sales of sedans from 2000 to 2004 = (1 + 1 + 4 + 4.5 +4) × 1000 = 14.5 × 1000 = 14500 18. (b) Total production of SUVs from 2000 to 2004 = (25 + 30 + 35 + 25 + 30) × 1000 = 145 × 1000 = 145000 19. (c) Total cars produced in 2002 = 50000 Total SUVs produced in 2002 = 30000 Ratio of SUVs produced to the cars produced in 30000 2002 = = 3:5 50000 20. (d) Total cars produced = (30 + 50 + 60 + 75 + 80) × 1000 = 295000 Total cars sold = (6 + 6.5 + 7.5 + 8 + 8) × 1000 = 36000 36000 Total percent of cars sold = × 100 = 12.2% 295000

22-08-2017 07:11:36

CHAPTER 6

PIE CHART

INTRODUCTION A pie chart is a circular chart divided into sectors which represent numerical proportions. In a pie chart, the arc length of each sector is proportional to the quantity it represents. The pie chart is also called a circle graph. Pie charts are widely used to represent numbers and figures in the corporate world meetings and presentations.

TYPES OF PIE CHARTS 3-D Pie Chart A 3-D pie cake, or perspective pie cake, is used to give the chart a three-dimensional look. The third dimension is often used for aesthetic reasons and does not improve the reading of the chart in any way. Figure 1 ­represents a basic 3-D pie chart. French 25%

In a pie chart,

German 100% = 360° 1% = 3.6°

30% 14% 31% Italian

 

18 ° = 5 Spanish 5 Conversely, 1° = % 15

Chapter 03-6.indd 731

Figure 1 |   Pie chart representing the percentage of students taking a certain foreign language.

23-08-2017 05:40:54

732     Pie Chart 

Doughnut Chart A doughnut chart is like an ordinary pie chart with the exception of a blank center. Moreover, we can use it to display multiple statistics at once. The blank center can be used to display additional data. Figure 2 represents a basic doughnut chart.

distance from the center of the circle but have different angles, whereas for the latter the sectors have equal angles and differ rather in how far each sector extends from the center of the circle. The polar area diagram is used to plot cyclic phenomena. Figure 4 represents a basic polar area chart. 254

24

1 2

23

3 3

2005

22

4 2

2006

21

10.9%

5

2007

1 20

6

10.3% 2008 8.9% 4.7%

31.0% 43.1%

0 19

7

2009

5.4% 5.1%

8

18 17

9 16

10

38.0%

15

11 14

12

13

Figure 4 |   Polar area chart.

42.6%  Figure 2 |   Pie chart representing sale of two products over 5 years.

Exploded Pie Chart A chart with one or more sectors separated from the rest of the disk is known as an exploded pie chart. This effect is used to highlight either a sector or segments of the chart with small proportions. Figure 3 represents a basic exploded pie chart.

Ring Chart/Multilevel Pie Chart A ring chart is used to visualize hierarchical data depicted by concentric circles. The circle in the center represents the root node, with the hierarchy moving outward from the center. A segment of the inner circle has a hierarchical relationship to those segments of the outer circle which lie within the angular sweep of the inner segment. Figure 5 represents a basic ring chart. Class, gender, age, and survival breakdown No

Yes

Yes

Yes

No 25%

No Yes

Adult Adult

No le

ma Fe

ild

Wheat Rice

Adult

Ch

Male

Fe ma le

42%

Adult

First Male Yes Adult Third Male Second Female Adult Yes No Crew

Oats Male

34%  Figure 3 |   Pie chart representing the percentage of food items consumed for a certain year.

Adult Yes

No

Polar Area Chart The only difference between a normal pie chart and a polar area chart is that for the former, the sectors have the same

Chapter 03-6.indd 732

Figure 5 |   Ring chart.

23-08-2017 05:40:55

SOLVED EXAMPLES     733

SOLVED EXAMPLES Direction for Q1—Q3: The data of section-wise distribution of 1080 student is shown in the following figure.

A 90°

Direction for Q4 and Q5: The data of colours of shirts (total shirts = 5000) sold in a showroom in the years 2000 and 2010 are shown in the following figures.

B 90° Red 25%

C 54°

D 126°

White 8%

Black 32%

Blue 35%

Red 26%

White 5%

Black 21%

Blue 48%

Consider the figure and answer the following questions. 2000 1. How many percent of students are more in section D than that in A? Solution:  Section D = 126° Section A = 90° 126 − 90 ⇒ × 100 90 36 ⇒ × 100 = 40% 90

2010

Consider the figures and answer the following questions. 4. If there are 1302 black shirts sold in 2010, then what was the total number of white shirts sold in 2000 and 2010 combined? Solution:  Let the total shirts sold in 2010 are x. So

2. What is the percentage composition of students in various sections? Solution:  We know 360° = 100% ⇒ 1° =

5 % 18

5 = 25% 18 5 Section B = 90 × = 25% 18 5 Section C = 54 × = 15% 18 5 Section D = 126 × = 35% 18 Section A = 90 ×

3. What is the number of students in each section? Solution:  We know Total students = 1080 So 100% or 360° = 1080 ⇒ 1° = 3 Section Section Section Section

Chapter 03-6.indd 733

A has 3 × 90 = 270 students B has 3 × 90 = 270 students C has 3 × 54 = 162 students D has 3 × 126 = 378 students

21 × x = 1302 100 ⇒ x = 6200 Therefore, total number of shirts sold in 2010 = 6200 5 × 6200 = 310 Total number of white shirts sold in 2010 = 5 100 = × 6200 = 310 100 Total number of shirts sold in 2000 = 5000 8 × 5000 = 400 Total number of white shirts sold in 2000 = 8 100 = × 5000 = 400 100 So the total number of white shirts sold in 2000 and 2010 = 710 5. If there were 1302 black shirts sold in 2010, and 25% of blue shirts were light blue, then how many dark blue shirts were sold? 1302 Solution:  Total number of shirts sold in 2010 = × 100 = 6200 1302 21 = × 100 = 6200 21 48 Total number if blue shirts sold = 48% of 6200 = × 6200 = 2976 48 100 = × 6200 = 2976 100 Total number of dark blue shirts sold = 75% of 75 2976 = × 2976 = 2232 100

22-08-2017 07:12:08

734     Pie Chart 

Direction for Q6—Q8: The data of percentage distribution of household expenditure of a family per month is shown in the following figure.

Direction for Q9 and Q10: Figure A shows the number of tourists in the following countries and Figure B shows age-wise traffic of the tourists.

15% (miscellaneous) Electricity 10.4%

U.S.A. 40% Food 35%

Rent 19.6%

U.K. 30%

15% 15%

Clothing 20%

Australia Canada

Consider the figure and answer the following questions.

(a) 30–40 years

6. What is the ratio between the money spent on clothing and food? 15% Solution:  Money spent on clothing = 20% Money spent on food = 35% Ratio =

20 4 = 35 7

or 4:7

7. If total expenditure of family is H 45000 per month, then what is the total expenditure on food and rent?

Below 30 years 50%

10%

40–50 years

Above 50 years 25%

(b) Solution:  Total expenditure = H 45000 per month 9. What is the ratio of number of tourists in USA to 35 Total expenditure on food = 35% of 45000 = × 45000 = H15750 the number of tourists above 50 years? 100 35 = × 45000 = H15750 Solution:  Total tourists in USA = 40% 100 19.6 Total tourists above 50 years = 25% Total expenditure on rent =19.6% of 45000 = × 45000 = H 8820 100 19.6 40 8 = × 45000 = H 8820 Ratio = = =8:5 100 25 5 Total expenditure on food and rent = H (15750 + 10. If the number of tourists that went to UK were 8820) = H 24570 360000, then how many of the total tourists are 8. If the total expenditure of family is H 45000 per between 40 and 50 years? month, then what is the difference between the expenditure of clothing and electricity? Solution:  Total tourists in UK = 30% If total number of tourists are x, then, Solution:  Total expenditure = H 45000 30 20 × x = 360000 ⇒ x = 1200000 × 45000 = H 9000 Expenditure on clothing = 100 100 10.4 Now, total tourists between 40 and 50 years × 45000 = H 4680 Expenditure on electricity = 100 10 Difference between expenditure on clothing and × 1200000 = 120000 100 electricity = H 9000 − 4680 = H 4320

Chapter 03-6.indd 734

22-08-2017 07:12:10

SOLVED EXAMPLES     735

Direction for Q11—Q15: Distribution of students at graduate and post graduate levels in seven institutes (M, N, P, Q, R, S and T). Total number of students of graduate level = 27300 N 12%

P 12%

M 17% T 15%

14. What is the ratio between the number of students studying at postgraduate level from institutes S and the number of students studying at graduate level from institute Q?

Q 13%

Solution:  Number of students studying at post21 × 24700 = 5187 graduate level from S = 100 Number of students studying at graduate level 13 × 27300 = 3549 from Q = 100 5187 19 = Required ratio = 3549 13

R 17%

S 14%

Total number of students of post graduate level = 24700

M 8%

N 15%

T 17%

P 12%

15. What is the ratio between the number of students studying at postgraduate and graduate levels from institute S?

Q 13%

Solution:  Number of postgraduate students from 21 × 24700 = 5187 institute S = 100 Number of graduate students from institute S = 14 = × 27300 = 3822 100 5187 19 = Required ratio = 3822 14

R 14%

S 21%

Students studying at postgraduate level from 12 × 24700 = 2964 P = 100 Required number = 3705 + 2964 = 6669

11. What is the total number of graduate and postgraduate level students in institute R?

Direction for Q16—Q20: Sources of funds to be arranged by NHAI for Phase II projects (in crores).

Solution:  Required number = (17% of 27300) +



 



17 14 × 27300 + × 24700 100 100 = 4641 + 3458 = 8099 (14% of 24700) =

12. How many students of institutes of M and S are studying at graduate level?

External assistance 11,486 Market borrowing 29,952

SPVS 5252 Toll 4910

Annuity Solution:  Students of institute M at graduate 6000 17 level = × 27300 = 4641 100 14 × 27300 = 3822 Students of institute S at graduate level = 100 14 = × 27300 = 3822 100 16. 20% of the funds are to be arranged through which Total number of students at graduate level in instisource? tutes M and S = 4641 + 3822 = 8463 Solution:  Total funds = 29952 + 11486 + 5252 + 13. What is the total number of students studying at 4910 + 6000 = 57600 postgraduate level from institutes N and P? 20% of the total funds to be arranged = H (20% of 57600) = H 11520 crores ≈ H11486 crores Solution:  Students studying at postgraduate level 15 Hence, 20% of the total funds are arranged through × 24700 = 3705 from N = External Assistance. 100

Chapter 03-6.indd 735

22-08-2017 07:12:11

736     Pie Chart 

17. If the Toll is to be collected through an outsourced agency by allowing a maximum 10% commission, how much amount should be permitted to be collected by the outsourced agency so that the project is supported with H 4910 crores? Solution:  Amount permitted = (Funds required from Toll for projects of Phase II) + (10% of these funds)  10  = 4910 +  × 4910 = 4910 + 491 100  = H 5401 crores

Required ratio =

4910 1  29952 6

19. If NHAI could receive a total of H 9695 crores as External Assistance, by what percent (approximately) should it increase the Market Borrowing to arrange for the shortage of funds? Solution:  Shortage of funds arranged through External Assistance = 11486 − 9695 = H 1791 crores Thus, increase required in Market Borrowing = H 1791 crores  1791  Percentage increase required =  × 100 %  6%  29952 

18. What is the approximate ratio of funds to be arranged through Toll and that through Market Borrowing?

20. What is the central angle corresponding to Market Borrowing?

Solution:  Total funds from Toll = H 4910 crores Total funds from Market Borrowing = H 29952 crores

Solution:  Central angle corresponding to Market  29952  Borrowing =  × 360° = 187.2°  57600 

PRACTICE EXERCISE Direction for Q1—Q4: The following pie chart shows the items sold by a store.

Direction for Q5—Q8: The following pie chart shows number of marks scored by a student in an examination. Here the total number of marks is 540.

Shorts 12% Coats 18%

Shirts 35%

Physics 90°

Vests 15%

Maths 90°

45°

Jeans 20% Computer

65° 70° English Chemistry

Consider the figure and answer the following questions. Consider the figure and answer the following questions. 1. What is the central angle of the sector for the number of jeans sold? (a) 56°

(b) 72°

(c) 64°

5. By how many marks, the marks in Maths and Chemistry exceed the marks scored in Physics and Computer?

(d) 92° (a) 42

2. If the total number of vests sold is 4500, then how many coats were sold? (a) 5400

(b) 5600

(c) 3000

(d) 6000

3. What is the ratio of number of jeans sold to number of shorts sold? (a) 5 : 3

(b) 3 : 4

(c) 10 : 3

(d) 5 : 4

4. How many more number of jeans is sold compared to vests if the total number of sales of all items is 30000? (a) 4500

Chapter 03-6.indd 736

(b) 6000

(c) 1000

(d) 1500

(b) 55

(c) 37.5

(d) 30

6. What percentage of marks was scored in English? (a) 22%

(b) 18.06% (c) 24.25% (d) 16.55%

7. In which subject did the student get 105 marks? (a) Maths (c) Computer

(b) Physics (d) Chemistry

8. What is percentage of the marks obtained in Physics, Computer and Maths of the total marks? (a) 50%

(b) 42.5%

(c) 62.5%

(d) 80%

22-08-2017 07:12:12

ANSWERS     737

Direction for Q9—Q12: The following pie chart shows the percentage distribution of money spent by a family in August, 2013.

13. What was the total number of Honda or Hyundai cars sold in 2008? (a) 4220 (c) 4730

Miscellaneous

14. If total 2150 Maruti cars were sold in 2008, then how many Skoda cars were sold?

20%

Food 25%

(b) 5100 (d) 4400

Transport 5%

(a) 860 (c) 640

Clothing Savings 10% 15% Housing Education 15% 10%

(b) 1240 (d) 720

15. In 2012, if 800 Hyundai cars were sold, then what is the difference between the sale of Hyundai in 2008 and 2012? (a) 2100 (c) 1600

(b) 1780 (d) 1850

Consider the figure and answer the following questions. 9. If the total amount spent was H 120000, then what was the total amount spent on “housing”?

16. If 1600 Hyundai cars were sold in 2012, then how many cars were sold in total?

(a) H 18000 (b) H 21000 (c) H 15000 (d) H 24500 10. What is the ratio of the total amount of money spent on clothing to that on food? (a) 5 : 2

(b) 2 : 5

(c) 3 : 4

(d) 4 : 3

(a) 2000 (c) 4000

17. If 1600 Hyundai cars were sold in 2012, then how many Honda cars were sold? (a) 1500 (c) 1400

11. How much of the total money was used for savings? (a) 36°

(b) 48°

(c) 52°

(a) H 13000 (b) H 26000 (c) H 20000 (d) H 40000

18. If 1600 Hyundai cars were sold in 2012, then how many cars were sold in 2010 and 2012 combined? (a) 11800 (c) 13600

Direction for Q13—Q20: The following pie chart depicts sales of cars in a city. Here the total number of cars sold is 8600.

(b) 12200 (d) 14500

19. If sales of Tata in 2008 were 4/3 times the sales of Tata in 2012, then how many Tata cars were sold in 2008 and 2012?

Maruti 15%

Honda 25%

(b) 1450 (d) 1300

(d) 54°

12. If total money spent on transportation was H 13000, then how much money was spent on education?

Maruti 25%

(b) 3000 (d) 5000

Honda 28%

(a) 1320 (c) 1505

13%

(b) 1410 (d) 2010

10% Skoda

10%

Hyundai 30%

Hyundai 32%

Skoda

Tata

12%

20. If sales of Tata in 2008 were 4/3 times the sales of Tata in 2012, then what were the total numbers of cars sold in 2012?

Tata 2008

2012

(a) 4700 (c) 5000

Consider the figure and answer the following questions.

(b) 5500 (d) 5375

ANSWERS 1. (b)

3. (a)

5. (c)

7. (d)

9. (a)

11. (d)

13. (c)

15. (b)

17. (c)

19. (c)

2. (a)

4. (d)

6. (b)

8. (c)

10. (b)

12. (b)

14. (a)

16. (d)

18. (c)

20. (d)

Chapter 03-6.indd 737

22-08-2017 07:12:13

738     Pie Chart 

EXPLANATIONS AND HINTS 1. (b) Central angle of sector for jeans 20 × 360 = 72° 100

10. (b) Total money spent on clothing = 10% Total money spent on food = 25% 10 2 Ratio of money spent clothing to food = = = 2 : 5 25 5 2. (a) Total number of vests = 4500 18 ° 11. (b) Total money spent on saving = 15% = 15× = 54° Let the total items sold are x. so 5 18 ° = 15% = 15× = 54° 15 5 x = 4500 100 12. (b) Total money spent on transportation = H 13000 If total money spent is H x. Then, x = 30000 5 Total number of coats sold × x = 13000 100 18 ⇒ x = H260000 × 30000 = 5400 10 100 × 260000 = H26000 Total money spent on education is 10 100 × 260000 = H26000 3. (a) Total percent of jeans sold = 20% 100 Total percent of shorts sold = 12% 13. (c) Total cars sold in 2008 = 8600 55 20 5 × 8600 = 4730 Total Honda or Hyundai sold = 55% = = Ratio = =5:3 55 100 12 3 = 55% = × 8600 = 4730 100 4. (d) Total number of jeans sold = 20% 14. (a) Total Maruti cars sold = 25% or 2150 10 Total number vests sold = 15% Total Skoda cars sold = 10% or 2150 × = 860 25 Thus, 5% of more number of jeans are sold. 30 So the total items sold = 30000 15. (b) Total Hyundai cars sold in 2008 = × 8600 = 2580 Hence, total difference between sales of jeans and 100 30 = × = 8600 2580 vests is 100 5 Total Hyundai cars sold in 2012 = 800 × 30000 = 1500. Difference between sales in 2008 and 2012 = 1780 100 160 5. (c) Marks scored in Maths and Chemistry = × 540 = 16. 240 (b) Total Hyundai cars sold in 2012 = 1600 Let the total cars sold be x. Thus, 160 360 = × 540 = 240 100 360 135 x = 1600 × = 5000 Marks scored in Physics and Computer = × 540 = 202.5 32 360 135 = × 540 = 202.5 17. (c) Total Hyundai cars sold in 2012 = 32% or 1600 360 28 Excess marks in Maths and Chemistry than Physics Total Honda cars sold in 2012 = 28% = 1600 × = 1400 28 32 and Computer = 37.5 = 28% = 1600 × = 1400 32 6. (b) We have 18. (c) Total Hyundai cars sold in 2012 = 1600 If total cars sold are x. Then 360° = 100% 32 100 × x = 1600 ⇒ 1° = % 100 360 100 100 ⇒ x = 1600 × = 5000 Now, the marks in English = 65° = 65× = 18.06% 32 360 Total cars in 2008 and 2012 = 5000 + 8600 = 13600 7. (d) Let the degree is x. So 10 19. (c) Total Tata cars sold in 2008 = × 8600 = 860 x 100 × 540 = 105 3 360 Total Tata cars sold in 2012 = × 8600 = 645 2 4 ⇒ x = 105 × = 70° Total Tata cars sold = 1505 3 10 Hence, the subject is Chemistry. 20. (d) Total Tata cars sold in 2008 = × 8600 = 860 100 8. (c) Total marks in Physics, Computer and Maths = 3 Total Tata cars sold in 2012 = × 860 = 645 100 4 = 225° = 225 × = 62.5% 360 If total cars sold in 2012 be x. Then, 9. (a) Total money spent = H 120000 12 × x = 645 15 100 Money spent on housing = 15% = × 120000 = H 18000 100 100 15 ⇒ x = 645 × = 5375 = 15% = × 120000 = H 18000 12 100





 

Chapter 03-6.indd 738

 

22-08-2017 07:12:16

CHAPTER 7

PUZZLES

INTRODUCTION

TYPES OF PUZZLES

A puzzle is a problem to test the reasoning ability of the student. In a puzzle, one is required to put pieces together, in a logical way, in order to arrive at the desired correct solution.

Puzzles not only come in different shapes and sizes, there are different types of puzzles too. Some common categories of puzzles are:

Puzzles are not only used for games and entertainment but in some cases, their solutions may be a significant contribution to mathematical research.



1. Number puzzles 2. Logical puzzles 3. Picture puzzles 4. Connect the dots 5. Spot the difference

SOLVED EXAMPLES 1. In a ten-digit number, first represents number of “1” present in the number, second digit represents number of “2” in the number. Similarly, third digit represents number of “3” present in the number and so on till the ninth digit, which ­represents number of “9” present in the number. However, the last digit represents number of “0” present in the number. What is the ten-digit number?

Chapter 03-7.indd 739

Solution:  We start with the smallest possible tendigit number is 1000000009. Now, since last digit represents number of zeros, it should be 9. Hence, the number becomes 1000000009. Since, we have only 8 zeros so the last digit should be 8. Hence, the number becomes 1000000008. Since, number of “8” present in the above case is one, next modification will be 1000000108.

22-08-2017 07:12:56

740        Puzzles 

Now, the number of “0” is only seven. Hence, the number is modified to 1000000107.

A

B

F

I

Since, the number of “7” present is one so next modification will be 10000001007. E

H

K

Now, since the number of “1” present is two, the next modification will be 2000001007. Number of “2” present is to be one so the next modification will be 2100001007. Here, since number of zeros present is six, the next modification will be 2100001006.

D

C

G

J

Total triangles not covered in the diagram are ∆ACF, ∆BGI, ∆DBG, ∆CFJ

Now, since number of “6” present is 1, so the next modification will be 2100010006. The number 2100010006 satisfies all the condition so this is the desired result.

Hence, total number of triangles in the figure = 24 + 4 = 28 4. Find the total number of triangles in the following figure.

2. Fill in the number that completes the following sequence: 3, 5, 7, 11, 13, 17, 19, x Solution:  It is evident that the sequence contains prime numbers. Hence, the number x is 23. 3. Find the total number of triangles in the given figure.

Solution:  Let us assume that the smallest side of a triangle to be 1 unit. Number of triangles with side 1 unit = 1 + 5 + 5 + 1 = 12 Number of triangles with side 2 units = 1 + 2 (facing upward) + 1 + 2 (facing downward) = 6 Number of triangles with side 3 units = 1(facing upward) + 1 (facing downward) = 2

Solution:  We can see that the figure contains three identical squares. Hence, let us calculate the triangles in one of these squares. A B

E

D

C

Total triangles in the square are ∆ABE, ∆BCE, ∆DEC, ∆AED, ∆ABC, ∆BCD, ∆ACD, ∆DAB Hence, there are 8 triangles in 1 square. Therefore, there will be 24 triangles in 3 squares. Also, consider the diagram again,

Chapter 03-7.indd 740

Thus, total number of triangles = 12 + 6 + 2 = 20 5. Three people checked into a hotel. They pay $30 to the manager and go to their room. The manager suddenly remembers that the room rates are for $25 for that night and gives $5 to the bellboy to return to the people. However, the bellboy keeps $2 and gives $1 to each person. Now, each person paid $9, totaling $27 and the bellboy kept $2, totaling $29. Where is the missing $1? Solution:  Originally, they paid $30, of which they got $1 and hence they have only paid $27. Of this $27, $25 went to the manager for the room and $2 went to the bellboy. Hence, the figures add up. 6. Four parallel horizontal lines are interesting perpendicularly to four vertical parallel lines. Find the number of rectangular formed after such intersection.

22-08-2017 07:12:57

SOLVED EXAMPLES        741

Three lines can have three intersecting points. x1 x2 x3 x4 y1 y2 y3 y4

Four lines can have six intersecting points.

Solution:  Let x1-x4 represent four parallel ­vertical lines and y1-y4 represent four parallel horizontal lines. To form a rectangle we need to select any two ­horizontal parallel lines which interest any two vertical parallel lines. Thus, C2 × 4C2 = 6 × 6 = 36

4

Hence, 36 rectangles can be formed. 7. There are four holes numbered 1, 2, 3 and 4. The unique properties of these holes are that number of mice become double, triple and four times after entering into hole number 2, 3 and 4, respectively, while it remains the same if they enter hole 1. One cat is running to eat some mice. To save themselves, the mice enter in first hole and come out, 24 of them are eaten by the cat. They, respectively, entered into second, third and fourth holes and come with double, triple and four times in number. After exit from each hole, 24 mice are eaten up by the cat each time. At the end, there is no mouse left. Find the initial number of mouse before entering into the first hole. Solution:  We know that 24 mice exit from fourth hole which were all eaten up by the cat and hence no mouse was left. Hence, 6 mice entered the fourth hole. Now, 24 were eaten after they exited from third hole by the cat and 6 remained. Thus, 30 mice came out of the third hole. Thus, before entering into third hole there were 10 mice. 34 mice came out of the second hole, since 24 mice were again eaten by the cat. Therefore, 17 mice entered the second hole. Again, 41 mice came out of first hole and hence, 41 mice entered into first hole.

9. A cake is to be cut into different pieces. How many maximum pieces are possible if four vertical cuts are made given that no horizontal cut is allowed? Solution:

Total pieces = 1 + 1 = 2

Total pieces = 1 + 1 + 2 = 4

8. Four lines are intersecting each other. Find how many intersecting points are possible. Solution:  Two lines have almost one intersecting point.

Chapter 03-7.indd 741

22-08-2017 07:12:59

742        Puzzles 

Total pieces = 1 + 1 + 2 + 3 = 7

10. Flag of a nation consists of six different vertical strips. How many such flags can be possible if we use six colours to fill the strips such that no two consecutive strips will have same colours? Solution:  First strips can be filled with any six colours. Second strip can be filled with any five colours except the colour filled in strip one. Similarly, successive strips can also be filled in the same manner, that is, 5 ways. Hence, number of possible flags = 6 × 5 × 5 × 5 × 5 × 5 = 18750 = 6 × 5 × 5 × 5 × 5 × 5 = 18750

Total pieces = 1 + 1 + 2 + 3 + 4 = 11

PRACTICE EXERCISE Direction for Q1-Q3: Disha brought some sweets on her 22nd birthday. She offered one less than the half of total number sweets in the gurdwara near her house. She also gave one sweet each to 3 beggars sitting on the stairs of gurdwara on the way back to home, she stopped a big group of poor children and gave them half of what was left with her. After reaching home she shared the remaining two pieces of sweets with her younger brother. 1. How many sweets did she originally have? (a) 8

(b) 10

(c) 12

(d) 14

2. How many sweets did she offer in the gurdwara? (a) 3

(b) 4

(c) 5

(d) 6

3. How many sweets did she give to poor children? (a) 1

(b) 2

(c) 30

(c) 26

(d) 28

6. Which day comes three days after the day which comes two days after the day which comes immediately after the day which comes two days after Monday? (a) Monday (c) Wednesday

(b) Tuesday (d) Thursday

7. Suppose you are in a dark room with a candle, a wood stove and a gas lamp. You only have one match, so what do you light first? (a) Candle (c) Gas lamp

(b) Wood stove (d) Match

(b) Mary (d) None of these

5. How many triangles are there in the following picture?

Chapter 03-7.indd 742

(b) 24

(d) 42

4. Mary’s mom has four children. The second child is called April, third child is called May and the fourth child is called June. What is the name of the first child? (a) March (c) July

(a) 20

Direction for Q8-Q10:  Study the pyramid of the letters given below and answer the following questions.

22-08-2017 07:12:59

PRACTICE EXERCISE        743

I met a man with seven wives.

B

Each wife had seven sacks. M

T

R

G

C

S

N

P

Y

Q

H

K

E

A

I

F

U

W

X

O

V

Z

Each sack had seven cats. Each cat has seven kits. Kits, cats, sacks and wives, J

D

How many people were going to St. Ives? (a) 345

(b) 9

(c) 2

(d) 1

8. Which letter is missing in the pyramid? 15. What is the value of x? (a) L

(b) F

(c) I

(d) P x

9. If letters were to be studied vertically, which two letters happen to be neighbors that occur in alphabetical order?

9

25 36

4 (a) WX

(b) PR

(c) UV

(d) ST

10. If all the horizontal lines were to be studied separately which neighbors in the alphabetical order are the farthest? (a) F and U (c) C and S

(b) Q and Y (d) H and Q

6

(a) 4

3

2

5

3

5

1

1

1

2

8

3

3

7

2

8

4

3

9

x

3

(b) 6

1

(a) Akul

(c) 7

(b) 128

(b) 16

64

49

(c) 4

(d) 17

16. Who started with the lowest amount?

(d) 9

12. How many squares are there in a chessboard? (a) 204

(a) 15

81

Direction for Q16-Q18:  Akul, Kshitij, Nakul and Dhruv are four brothers playing a game where the loser doubles the money of each of the other players by giving them from his share at that point of time. They played four games and each brother lost one game in alphabetical order. At the end of the fourth game, each brother had T 64.

11. Find the value of x. 4

1

(c) 64

(d) 32

(b) Kshitij (c) Nakul

(d) Dhruv

17. How many rupees did Kshitij start with? (a) 64

(b) 136

(c) 68

(d) 72

18. What was the amount left with Nakul at the end of the second round?

13. Which number fits the empty circle? (a) 72

9

1

5

4

8

3

2

(b) 144

(c) 132

(d) 136

Direction for Q19-Q20: When zero was not invented, people could not multiply numbers. Thus, mathematics had only 9 digits (1-9) and after that came 11, 12 and so on. Find the answer to following operations in mathematics used then.

7 19. What is the value of 7 + 11 + 3?

(a) 5

(b) 6

(c) 9

(d) 2

14. Read the following phrase and answer the question: As I was going to St. Ives,

Chapter 03-7.indd 743

(a) 22

(b) 23

(c) 24

(d) 25

20. What is the value of 21 + 29 + 1? (a) 49

(b) 50

(c) 51

(d) 52

22-08-2017 07:12:59

744        Puzzles 

ANSWERS 1. (c)

5. (d)

9. (d)

13. (b)

17. (c)

2. (c)

6. (b)

10. (c)

14. (d)

18. (b)

3. (b)

7. (d)

11. (b)

15. (b)

19. (a)

4. (b)

8. (a)

12. (a)

16. (d)

20. (d)

EXPLANATIONS AND HINTS Since, x = 12, total sweets given to poor children =

1. (c) Total sweets given to the beggars = 3 If x is the total number of sweets that Disha originally had, then total sweets offered in the x x gurdwara = x − −1 = +1 2 2









1 12 −2 = 2 2 2 4. (b) Mary’s mother’s first child was Mary herself. 5. (d)

Total sweets left with her after giving to the x x beggars = + 1 − 3 = − 2 2 2

1 2

Total sweets left with her after giving to poor 1 x x children = − 2 = −1 2 2 4

3

Total number of sweets left with her after giving her brother = 2

4





5 Hence,

6 x −1 = 2 4 ⇒ x = 3 × 4 = 12

Number of non-overlapping triangles between 1 and 2 = 3 Number of non-overlapping triangles between 2 and 3 = 4

Thus, Disha originally had 12 sweets.

Number of non-overlapping triangles between 3 and 4 = 6

x 2. (c) Total sweets offered in gurdwara = − 1 2 We calculated that x = 12 Hence, total sweets offered in gurdwara =

12 −1 = 5 2



1 x 3. (b) Total sweets given to poor children = −2 2 2

Chapter 03-7.indd 744

Number of non-overlapping triangles between 4 and 5 = 4



Number of non-overlapping triangles between 5 and 6 = 3 Number of overlapping triangles = 8 Total number of triangles = 3 + 4 + 6 + 4 + 3 + 8 = 28

22-08-2017 07:13:01

EXPLANATIONS AND HINTS        745

6. (b) Let the day be x. Now, x comes 3 days after the day which comes 2 days after the day which comes immediately after the day which comes 2 days after Monday.

Then, the total number of squares with side 1 cm = 8 × 8 = 64 Total number of squares with side 2 cm = 7 × 7 = 49 Total number of squares with side 3 cm = 6 × 6 = 36

Hence, x comes after 3 + 2 + 1 + 2 = 8 days from Monday.

Total number of squares with side 4 cm = 5 × 5 = 25

Hence, x = Tuesday

Total number of squares with side 6 cm = 3 × 3 = 9

7. (d) You will first light the match since you cannot light anything without the match. 8. (a) L is missing in the pyramid. 9. (d) PR does not occur together in an alphabetical order. WX and UV are not vertical neighbors in a given pyramid. ST occurs together in an alphabetical order, and also S and T are vertical neighbors in a given pyramid.

Total number of squares with side 5 cm = 4 × 4 = 16 Total number of squares with side 7 cm = 2 × 2 = 4 Total number of squares with side 8 cm = 1 × 1 = 1 Hence, total number of squares on the chessboard = 64 + 49 + 36 + 25 + 16 + 9 + 4 + 1 = 204 13. (b) The numbers in each row and column add up 15. Hence, the second column should also add up to 15. Therefore, 1 + 8 + x = 15 ⇒ x = 6

10. (c)  F and U are separated by 14 letters in an alpha­ betical order, while Q and Y are separated by 7 letters. C and S are separated by 15 letters and H and Q are separated by 8 letters.

Thus, the circle should be filled with 6. 14. (d) The phrase clearly mentions, “As I was going to St. Ives.”

Thus, C and S are the farthest from each other among given pairs.

Hence, only “I” was going. The correct answer is 1.

11. (b) Looking at the diagram in rows, the central number equals half the sum of the numbers in the other circles to the left and right of the center.

15. (b) Starting from Eq. (1) and moving clockwise around the triangle, numbers follow the sequence of square numbers. Hence, between 9 and 25 (i.e., square of 3 and 5, respectively), we will add 16.

Hence, Thus, x = 16. x=

9+3 =6 2

12. (a)

16. (a) The four brothers play in such a way that the loser doubles the money of the other players and they lose in an alphabetical order. Thus, working backwards we get

Round 4 Round 3 Round 2 Round 1 Initially

Akul Kshitij Nakul Dhruv 64 64 64 64 32 32 32 160 16 16 144 80 8 136 72 40 132 68 36 20

Thus, Dhruv started with the lowest amount. 17. (c) Kshitij started with T 68. Say the minimum value of the side of square is 1 unit.

Chapter 03-7.indd 745

18. (b) At the end of the second round, Nakul was left with T 144.

22-08-2017 07:13:03

746        Puzzles 

19. (a) We have to calculate 7 + 11 + 3 Now, 7 + 3 = 10.

20. (d) We have to calculate 21 + 29 + 1 Now, 29 + 1 = 30

However, since 0 was not invented, so Since, 0 is not invented, 29 + 1 = 30 7 + 3 = 11 11 + 11 = 22

Chapter 03-7.indd 746

⇒ 31 + 21 = 52

22-08-2017 07:13:04

CHAPTER 8

ANALYTICAL REASONING

INTRODUCTION

own content. Beyond an understanding of the meanings of the words used, no particular experience or knowledge is required for analytical reasoning.

Analytical reasoning represents judgments made on statements that are based on the virtue of the statement’s

SOLVED EXAMPLES Direction for Q1–Q5: Read the passage and answer the following questions: A, B, C and D are four friends living together in a flat and they have an agreement that whatever edible comes they will share equally among themselves. One day A’s uncle came to him and gave a box of chocolates. Since nobody was around, A divided that chocolates in four equal parts and ate his share after which he put the rest in the box. As he was closing the box, B walked in and took the box. He again divided remaining chocolates in four equal parts. A and B ate one part each and kept the remaining chocolates in the box. Suddenly C appeared

Chapter 03-8.indd 747

and snatched the box. He again divided the chocolates in four equal parts; the three of them ate one part each and kept the remaining chocolates in the box. Later when D came, he again divided the chocolates in four equal parts and all four ate their share, respectively. In total D ate three chocolates.

1. How many chocolates did C eat in total? 2. How many chocolates did B eat in total? 3. How many chocolates did A eat in total? 4. How many chocolates were given to A by his uncle? 5. How many chocolates did A eat the first time?

22-08-2017 07:13:33

748     Analytical Reasoning 

Solution:  We know that D ate Before D ate the chocolates, 3 × 4 = 12 chocolates. Before C ate the chocolates, 12 × 3 + 12 = 48 chocolates. Before B ate the chocolates, 24 × 2 + 48 = 96 chocolates.

three chocolates. there should be there should be

A

×

×



×

×

there should be

B

×



×

×

×

C

×

×

×

D

×

×

×

E



×

×

×

×

Before A ate the chocolates, there should be 96 × 4 = 128 chocolates. 3 At the beginning, we have 128 chocolates. A

B

C

Months → January February March April May Course ↓

D

Remaining Lecturer → Course ↓

P

Q

R

S

T

A

×



×

×

×

B



×

×

×

×

C

×

×

×

2. B ate 39 chocolates in total.

D

×

×

×

3. A ate 71 chocolates in total.

E

×

×



×

×

A’s share

32

96

B’s share

24

24

C’s share

12

12

12

D’s share

 3

 3

 3

3

Total

71

39

15

3

48 12  0

1. C ate 15 chocolates in total.

4. 128 chocolates were given to A by his uncle. 5. A ate 32 chocolates the first time. Direction for Q6-Q10: Read the passage and answer the following questions: Five courses: A, B, C, D and E each of one month duration are to be taught from January to May one after the other though not necessarily in the same order by lecturers P, Q, R, S and T. P teaches course B but not in the month of April or May. Q teaches course A in the month of March. R teaches in the month of January but does not teach course C or D. 6. What course is taught by S?



  6. Either C or D.   7. Q’s course A immediately follows course B.   8. Course E is taught in the month of January.   9. Q taught course A in the month of March.  10. In February, course B was taught. Course B is taught by lecture P. Direction for Q11-Q14: Read the passage and answer the following questions: Each of the five friends A, B, C, D and E have different likings, viz., reading, playing, travelling, singing and writing.

7. Which lecturer’s course immediately follows course B? 8. Which course is taught in the month of January?

B has liking for singing whereas C does not like reading or writing. E likes to play, while A has liking for writing.

9. When was Q’s course taught? 11. What does D like? 10. Which lecturer taught in February? 12. What does C like? Solution:  Let us consider the conditions given in the passage.

Chapter 03-8.indd 748

1. P teaches course B but not in the month of April or May. 2.  Q teaches course A in the month of March. 3. R teaches in the month of January but does not teach courses C or D. Now, we make a tabular chart.

13. If reading, singing and writing are categorized as indoor likings whereas travelling and playing are categorized as outdoor likings, then which friends have indoor likings? 14. How many friends have outdoor likings? Solution:  From the passage, we can tabulate the following data:

22-08-2017 07:13:34

PRACTICE EXERCISE     749

Liking → Read Play Travel Friends ↓

15. How is T related to M? Sing

Write 16. Which are the correct pairs of married couples?

A

×

×

×

×



17. What is the profession of L’s wife?

×

×

×



×

18. Who is an architect?

B C

×

×



×

×

D



×

×

×

×

E

×



×

×

×

19. What is the profession of M’s father?



20. Who are the two unmarried persons? Solution:  Professor is the grandfather of R therefore, R will be in third generation and professor will be in the first generation. Now it is given that Doctor is married to lawyer and lawyer is T’s daughter-in-law. From here it is clear that one of the married couples is Doctor-Lawyer and is in the second generation. T is the wife of the professor in the first generation. From the information (2) and (4), we conclude that married couples are L - T and N - S. Hence, we can tabulate the result as follows:

11.  D likes to read. 12.  C likes to travel. 13.  A, B and D have indoor likings. 14.  C and E have outdoor likings. Direction for Q15-Q20: Read the passage and answer the following questions: In a family of six, there are three men L, M and N and three women R, S and T. The six persons are Architect, Lawyer, CA, Professor, Doctor and Engineer by profession but not in the same order.  There are two married couples and two unmarried persons.   N is not R’s husband.  The Doctor is married to a Lawyer. R’s grandfather is a Professor.  M is neither L’s son, nor an Architect or a Professor.   The Lawyer is T’s daughter-in-law.   N is T’s son and the Engineer’s father.   L is married to the CA.



Grandparents

1st generation

L-Professor

T-C.A.

Parents

2nd generation N-Doctor

S-Lawyer

Children

3rd generation R-Architect

M-Engineer

15. We know that T is a female and T is a grandparent of M. Hence, T is M’s grandmother. 16.  L-T and N-S are the two married couples. 17.  L’s wife T is a C.A. 18.  R is the architect. 19.  M’s father is N and he is a doctor. 20.  R and M are the two unmarried couples.

PRACTICE EXERCISE Direction for Q1-Q4: Read the information given below and answer the following questions. There is a family of six persons A, B, C, D, E and F. Their professions are Engineer, Doctor, Teacher, Salesman, Manager and Lawyer. There are two married couples in the family. The Manager is the grandfather of F, who is an Engineer. C, the Salesman, is married to the lady Teacher. B is the mother of F and E. The doctor, D is married to the Manager. 1. How many male members are there in the family? (a) Two (c) Four

Chapter 03-8.indd 749

(b) Three (d) Data inadequate

2. What is the profession of A? (a) Lawyer (c) Manager

(b) Salesman (d) Teacher

3. Who are the two married couples in the family? (a) AB and DC (c) AE and CD

(b) CF and DE (d) AD and BC

4. How is A related to E? (a) Father (c) Mother

(b) Grandfather (d) Wife

Direction for Q5-Q9: Read the information given below and answer the following questions.

22-08-2017 07:13:34

750     Analytical Reasoning 

A, B, C, D, E, E, G and H are eight friends. Three of them play cricket and table tennis each and two of them play football. Each one of them has a different height. The tallest does not play football and the shortest does not play cricket. F is taller than A and D but shorter than H and B. E, who does not play cricket, is taller than B and is second to the tallest. G is shorter than D but taller than A. H, who is fourth from the top, plays table tennis with D. G does not play either cricket or football, B does not play football. 5. Which of the following pairs of friends play football? (a) EF (b) EA (c) HF (d) Data inadequate 6. What is F’s position from the top when they are arranged in descending order of their height? (a) Fifth (b) Sixth (c) Seventh (d) None of these

(a) Yellow-Red (c) Red-Yellow

(b) Green-Black (d) Yellow-Green

12. Which of the following is one of the married couples? (a) CD (c) AC

(b) DA (d) Cannot be determined

13. Which of the following is true about F? (a) Sister of B (c) Daughter of C

(b) Brother of B (d) None of the above

14. How many male members are there in the family? (a) Two (c) Four

(b) Three (d) None of these

Direction for Q15-Q17: Read the information given below and answer the following questions. Aman planted some plants in his lawn but in a certain fixed pattern: In most of the rows there is neither Rose nor Marigold.

7. Who is the tallest? (a) A (b) B (c) C (d) H

There are two more rows of Orchids than Tulips and two more rows of Rose than Orchids. There are four more rows of Rose than Tulips.

8. Who is the shortest? There are not as many rows of Lily as Fireball. (a) A (b) G (c) D (d) F

There is one less Marigold row than Rose. There is just one row of Tulips.

9. Which of the following group of friends play cricket? (a) C-A-E (b) C-B-F (c) C-B-A (d) None of these Direction for Q10-Q14: Read the information given below and answer the following questions. There is a family of six members A, B, C, D, E and F. There are two married couples in the family and the family members represent three generations. Each member has a distinct choice of a colour among Green, Yellow, Black, Red, White and Pink. No lady member like either Green or White.

The maximum number of rows he planted is six. 15. How many rows of Rose he planted? (a) Two (c) Four

(b) Five (d) Cannot be determined

16. What is the sum of the rows of Orchids and Marigold he planted? (a) 7 (c) 9

(b) 5 (d) Cannot be determined

17. How many rows of Fireball did he plant? (a) 2 (c) 6

(b) 5 (d) Cannot be determined

C, who likes Black colour, is the daughter-in-law of E. B is the brother of F and son of D and like Pink.

Direction for Q18-Q20: Read the following passage carefully and answer the following questions.

A is the grandmother of F and F does not like Red. The husband has a choice for Green colour, but his wife likes Yellow. 10. Which of the following is the colour preference of A? (a) Red (b) Yellow (c) Either Yellow or Red (d) None of these 11. Which of the following is the colour combination of one of the couples?

Chapter 03-8.indd 750

Three kids, Sam, Mona and Tanya went on a picnic with their dog Skipper and carried with them few chocolates, without counting. They rested under a tree and slept for a while. After sometime, Sam woke up, gave one chocolate from the total to the Skipper and distributed the remaining into three equal parts, ate his share and slept. After some time, Mona woke up, gave one chocolate to the Skipper and distributed the remaining into three equal parts, ate her

22-08-2017 07:13:34

EXPLANATIONS AND HINTS     751

share and slept. After sometime Tanya woke up and repeated the same. A little later, all of them woke up together, gave one chocolate from the total to Skipper and divided the remaining chocolates among them and each ate their share. However, we know that no chocolate was broken and there were less than 150 chocolates.

19. In what ratio did Sam and Tanya eat the chocolates? (a) 11:6 (c) 26:11

(b) 6:11 (d) 7:26

20. How many more chocolates did Tanya eat from Skipper?

18. How many chocolates were there in the beginning? (a) 66 (c) 118

(a) 18 (c) 22

(b) 84 (d) 79

(b) 12 (d) 14

ANSWERS 1. (d)

3. (d)

5. (b)

7. (c)

9. (b)

11. (d)

13. (b)

15. (b)

17. (c)

19. (a)

2. (c)

4. (b)

6. (a)

8. (a)

10. (b)

12. (a)

14. (b)

16. (a)

18. (d)

20. (d)

EXPLANATIONS AND HINTS Solutions to Q1-Q4:  From the information given in the question it is clear that there is a family tree of three generations. 1st Generation

A (Manager)

D (Doctor)

2nd Generation

C (Salesman)

B (Teacher)

3rd Generation

F (Engineer)

E (Lawyer)

Now, the heights of H and B, and A and D are not given. E does not play Cricket, hence E → Cr and is taller than B and also second to the tallest. So E’s position is (2). G is shorter than D but taller than A. Hence, D G A H is fourth from top so its position is H → (4)

Also, from the information, it is clear that A-D and B-C are married couples whereas sex of E and F cannot be determined from the information given. 1. (d) Since the sex of E and F cannot be determined. Therefore, we cannot find the total number of male members from the given data.

H plays TT with D so, H → TT D → TT G does not play Cricket or Football, hence

2. (c) A is the Manager.

G → TT Now, we can conclude that

3. (d) The two married couples are A-D and B-C. 4. (b) A is the grandfather of E. Solutions to Q5-Q9:  We know that three play Cricket, three play TT and two play Football.

Order of Height

Also, each of them has a different height, say (1) being the tallest and (8) being the shortest.

(1)

C



×

(2)

E

×



(3)

B



(4)

H

(5)

F

(6)

D



(7)

G



(8)

A

Also, tallest does not play Football and shortest does not play Cricket.

Friend

Cricket Football

Therefore, (1) Fb (8) Cr Now, F is taller than A and D but shorter than H and B. Therefore, H, B F A, D

Chapter 03-8.indd 751

Table tennis

✓ ✓

×



22-08-2017 07:13:35

752     Analytical Reasoning 

5. (b) E and A play Football.

17. (c) Total rows of Fireball planted = 6

6. (a) F is fifth from the top.

Solutions to Q18-Q20:  Let x be the number of chocolates in the beginning.

7. (c) C is the tallest. 8. (a) A is the shortest. 9. (b) C-B-F play cricket. Solutions to Q10-Q14:  From the information given to us, we can make the following table: 1st EAGrandparents Generation Green Male Female Yellow DCMale Female Black

Parents

2nd Generation Red

Children

3rd FBGeneration White Male Female

Pink

When Sam wakes up, total chocolates left for 2 others = (x − 1) 3 When Mona wakes up, total chocolates left for  4x − 10 2 2 others =  (x − 1) − 1 =  3  3 9 Similarly, when Tanya wakes up, total chocolates   2  4x − 10 8x − 38  − 1 = left =   3 9 27   8x − 38 Then, − 1 was divided into three equal 27 parts.





⇒ 8x − 65 is divisible by 81. 10. (b) A prefers Yellow.

Thus, 8x − 65 = 81n where n is an integer.

11. (d) Green-Yellow is the colour combination of one of the couples.

So, 81n + 65 should be divisible by 8. Thus, 81n + 65 is an even number and 81n is odd.

12. (a) CD is one of the married couples.

Now, putting values n = 1, 3, 5… we get n = 7 and 15.

13. (b) Since no lady member likes either Green or White. Hence F will be a male and brother of B.

However, n = 15 is not possible since number of chocolates is less than 150.

14. (b) There are three male members, E, D and F. Solutions to Q15-Q17:  Say number of rows of Orchids, Tulips, Roses and Marigold is O, T, R and M.

Thus, n = 7 is the accepted value. (81 × 7) + 65 Number of chocolates initially = = 79 8 Now, we can tabulate our results as follows:

Then,

O = T + 2

(1)



R = O + 2

(2)



R = M + 1

(3)

and

T=1

Solving Eqs. (1)—(3), we get O = 3, R = 5 and M = 4 Also, we know that there are not as many rows of Lily as Fireball.

Sam

Mona

Tanya

Skipper

First

26

26

26

1

Second

17

17

17

1

Third

11

11

11

1

Fourth

 7

 7

 7

1

Total = 79

33

24

18

4

18. (d) There were 79 chocolates initially.

Lily < Fireball 19. (a) Sam ate a total of 33 chocolates. So, Tanya ate a total of 18 chocolates. Lily = 2, Fireball = 6 Ratio = 33:18 = 11:6 15. (b) Total rows of Roses planted = 5 20. (d) Tanya ate a total of 18 chocolates. 16. (a) Total rows of Orchids = 3 Skipper ate a total of 4 chocolates. Total rows of Marigold = 4 Therefore, sum of rows of Orchids and Marigold planted = 7

Chapter 03-8.indd 752

Hence, total chocolates that Tanya ate more than Skipper = 14

22-08-2017 07:13:36

GENERAL APTITUDE

SOLVED GATE PREVIOUS YEARS’ QUESTIONS 1. Which of the following options is the closest in meaning to the word below? Circuitous (a) Cyclic (c) Confusing

(b) Indirect (d) Crooked (GATE 2010, 1 Mark)

Solution:  Circuitous means deviating from a straight course ⇒ indirect (a) Cyclic means recurring in cycle (b) Indirect means not leading by straight line (c) Confusing means lacking clarity (d) Crooked means for shapes (irregular in shape) Ans. (a) 2. The question below consists of a pair of related words followed by four pairs of words. Select the pair that best expresses the relation in the original pair. Unemployed: Worker (a) Fallow : Land (c) Wit : Jester

(b) Unaware : Sleeper (d) Renovated : House (GATE 2010, 1 Mark)

Solution:  Unemployed: Worker. Here one is opposite to other. (a) Fallow: Land ⇒ Fallow means undeveloped land (b) Unaware: Sleeper ⇒ Both are same (c) Wit: Jester ⇒ Wit means ability to make jokes and jester means a joker (d) Renovated: House ⇒ Renovate means to make better and house can be renovated Ans. (a) 3. Choose the most appropriate word from the options given below to complete the following sentence: If we manage to our natural resources, we would leave a better planet for our children.

GATE_GA-Questions.indd 753

(a) uphold (c) cherish

(b) restrain (d) conserve (GATE 2010, 1 Mark)

Solution: (a) Uphold means cause to remain ⇒ not appropriate (b) Restrain means keep under control ⇒ not appropriate (c) Cherish means be fond of ⇒ not related (d) Conserve means keep in safety and protect from harm, decay, loss, or destruction ⇒ most appropriate Ans. (d) 4. Choose the most appropriate word from the options given below to complete the following sentence: His rather casual remarks on politics his lack of seriousness about the subject. (a) masked (b) belied (c) betrayed (d) suppressed (GATE 2010, 1 Mark) Solution: (a) Masked means hide under a false appearance ⇒ opposite (b) Belied means be in contradiction with ⇒ not appropriate (c) Betrayed means reveal unintentionally ⇒ most appropriate (d) Suppressed means to put down by force or authority ⇒ irrelevant Ans. (c) 5. 25 persons are in a room. 15 of them play hockey, 17 of them play football and 10 of them play both hockey and football. Then the number of persons playing neither hockey nor football is (a) 2 (c) 13

(b) 17 (d) 3 (GATE 2010, 1 Mark)

25-08-17 1:44:56 PM

754     GENERAL APTITUDE

Solution:  Using the set theory formula, we have n(A): Number of people who play hockey = 15

Solution:  7 and 6 added is becoming 5 means the given two numbers are added on base 8. (137)8

n(B): Number of people who play football =17 n(A ∩ B) : Persons who play both hockey and football = 10

+(276)8 (435)8

n(A ∪ B) : Persons who play either hockey or football or both

Hence, we have to add the another two given set of numbers also on base 8.

Using the formula, we have

(731)8

n(A ∪ B) = n(A) + n(B) − n(A ∩ B)

+(672)8 (1623)8

n(A ∪ B) = 15 + 17 − 10 = 22 Thus people who play neither hockey not football = 25 - 22 = 3

Hence, the overall problem was based on identifying base, which was 8, and adding number on base 8. Ans. (c)



Ans. (d)

6. Modern warfare has changed from large scale clashes of armies to suppression of civilian populations. Chemical agents that do their work silently appear to be suited to such warfare; and regretfully, there exist people in military establishments who think that chemical agents are useful tools for their cause. Which of the following statements best sums up the meaning of the above passage? (a) Modern warfare has resulted in civil strife. (b) Chemical agents are useful in modern warfare. (c) Use of chemical agents in warfare would be undesirable. (d) People in military establishments like to use chemical agents in war. (GATE 2010, 2 Marks)

8. 5 skilled workers can build a wall in 20 days; 8 semiskilled workers can build a wall in 25 days; 10 unskilled workers can build a wall in 30 days. If a team has 2 skilled, 6 semiskilled and 5 unskilled workers, how long will it take to build the wall? (a) 20 days (c) 16 days

(b) 18 days (d) 15 days (GATE 2010, 2 Marks)

Solution:  Let one day work of skilled semi-skilled and unskilled worker be a, b, c units, respectively. 5a × 20 = 8b + 25 = 10c × 30 = Total units of work 100a = 200b = 300c a = 2b = 3c a a ⇒ b = and c = 3 2

Solution: (a) Modern warfare has resulted in civil strife: There is no direct consequence of warfare given, so it is not appropriate.

Given that 2 skilled, 6 semi-skilled and 5 unskilled workers are working. Consider that they finish the work in `x’ days. (2a + 6b + 5c)x = 5a + 20 = Total units of work

(b) Chemical agents are useful in modern warfare: Passage does not say whether chemical agents are useful or not, so it is not appropriate.

  2a + 3a + 5 a x = 5a × 20  3 

(c) Use of chemical agents in warfare would be undesirable: Given that people in military think these are useful, undesirable is wrong.

20a x = 5a × 20 3

(d) People in military establishments like to use chemical agents in war. Correct choice as last statement tells that military people think that chemical agents are useful tools for their cause (work silently in warfare). Ans. (d)

x = 15 days Ans. (d) 9. Given digits 2, 2, 3, 3, 3, 4, 4, 4, 4 how many distinct 4 digit numbers greater than 3000 can be formed? (a) 50 (c) 52

(b) 51 (d) 54 (GATE 2010, 2 Marks)

7. If 137 + 276 = 435 how much is 731 + 672? (a) 534 (c) 1623

(b) 1403 (d) 1531 (GATE 2010, 2 Marks)

GATE_GA-Questions.indd 754

We have to make 4 digit numbers, so the number should start with 3 or 4, two cases possible; Case 1: Thousands digit is 3. Now other three digits may be any from 2, 2, 3, 3, 4, 4, 4, 4.

8/22/2017 5:34:19 PM

SOLVED GATE PREVIOUS YEARS’ QUESTIONS     755

(a) Using 2, 2, 3 3! = 3 numbers are possible 2! (b) Using 2, 2, 4 3! = 3 numbers are possible 2! (c) Using 2, 3, 3 3! = 3 numbers are possible 2! (d) Using 2, 3, 4 3! = 6 numbers are possible (e) Using 2, 4, 4 3! = 3 numbers are possible 2! (f) Using 3, 3, 4

(g) Using 3, 3, 4 3! = 3 numbers are possible 2! (h) Using 3, 4, 4 3! = 3 numbers are possible 2! (i) Using 4, 4, 4 3! = 1 number is possible 3! Total 4-digit numbers in Case 2 = 3 + 3 + 3 + 3 + 3 + 3 + 1 + 3 + 3 + 1 = 26 Thus total 4 digits numbers using Case 1 and Case 2 = 25 + 26 = 51 Ans. (b)

3! = 3 numbers are possible 2! (g) Using 3, 4, 4

10. Hair (H), Gita (G), Irfan (I) and Saira (S) are siblings (i.e. brothers and sisters). All were born on 1st January. The age difference between any two successive siblings (that is born one after another) is less than 3 years. Given the following facts:

3! = 3 numbers are possible 2! (h) Using 4, 4, 4 3! = 1 number is possible 3!

1. Hari’s age + Gita’s age > Irfan’s age + Saira’s age. 2. The age difference between Gita and Saira is 1 year. However, Gita is not the oldest and Saira is not the youngest. 3. There are no twins.

Total 4-digit numbers in Case 1 = 3 + 3 + 3 + 3 + 3 + 3 + 3 + 3 + 1 = 25

In what order were they born (oldest first)?

Case 2: Thousands digit is 4. Now other three digits may be any from 2, 2, 3, 3, 3, 4, 4, 4.

(a) HSIG (b) SGHI

(c) IGSH (d) IHSG (GATE 2010, 2 Marks)

(a) Using 2, 2, 3 3! = 3 numbers are possible 2! (b) Using 2, 2, 4 3! = 3 numbers are possible 2! (c) Using 2, 3, 3 3! = 3 numbers are possible 2! (d) Using 2, 3, 4 3! = 3 numbers are possible 2! (e) Using 2, 4, 4 3! = 3 numbers are possible 2! (f) Using 3, 3, 3 3! = 1 number is possible 3!

GATE_GA-Questions.indd 755

Solution:  Suppose: Hari’s age: H, Gita’s age: G, Saira’s age: S, Irfan’s age: 1 •• H + G > I + S •• Using statement (2) both G - S = I or S - G = I; G can’t be oldest and S can’t be youngest. •• There are no twins; thus using statement (2) either GS or SG is possible. (a) HSIG: Not possible as there is between S and G which is not possible using statement (3). (b) SGHI: SG order is possible, S > G > H; > I and G + H > S + I (possible) because if {S = G + I; and G = H + I and H = I + 2 then G + (I + 2) > (G + I) + 1} (c) IGSH: According to this I > G and S > H; thus adding these both inequalities we get I + S > G + H which is possible of statement (2). Thus not possible. (d) IHSG: According to this I > H and S > G; Thus adding both inequalities I + S > H which is opposite of statement (2). Thus not possible. Ans. (b)

8/22/2017 5:34:24 PM

756     GENERAL APTITUDE

11. Choose the word from the options given below that is most nearly opposite in meaning to the given word: Amalgamate (a) merge (c) collect

(b) split (d) separable (GATE 2011, 1 Mark)

15. Choose the most appropriate word from the options given below to complete the following sentence. If you are trying to make a strong impression on your audience, you cannot do so by being understated, tentative or (a) hyperbolic (c) argumentative

(b) restrained (d) indifferent (GATE 2011, 1 Mark)

Solution:  The answer is split. Ans. (b)

Solution:  The answer is `restrained'. Ans. (b)

12. Which of the following options is the closest, in the meaning to the word below: Inexplicable (a) Incomprehensible (c) Inextricable

(b) Indelible (d) Infallible (GATE 2011, 1 Mark)

Solution:  The answer is incomprehensible. Ans. (a) 13. If log(P  ) = (1/2)log(Q) = (1/3)log(R  ), then which of the following options is TRUE? (a) P  2 = Q3R2 (c) Q2 = R3P

Solution:  log(P ) =

(b) Q2 = PR (d) R = P  2Q2 (GATE 2011, 1 Mark) 1 1 log(Q) = log(R) 2 3

⇒ log(P) = log(Q)1/2

= log(R)1/3 = K

⇒ P = (Q)1/2 = (R)1/3 = K ⇒ P = K, Q = K2, R = K3

(i)

16. The question below consists of a pair of related words followed by four pairs of words. Select the pair that best expresses the relation in the original pair: Gladiator : Arena (a) dancer : stage (b) commuter : train (c) teacher : classroom (d) lawyer : courtroom (GATE 2011, 1 Mark) Solution:  Like gladiator fights in an arena as people watch, a dancer performs on a stage as audience watch his performance. Hence, dancer : stage best expresses the relation gladiator : arena. Ans. (a) 17. There are two candidates P and Q in an election. During the campaign 40% of the voters promised to vote for P, and rest for Q. However, on the day of election 15% of the voters went back on their promise to vote for P and instead voted for Q. 25% of the voters went back on their promise to vote for Q and instead voted for P. Suppose, P lost by 2 votes, then what was the total number of voters? (a) 100 (c) 90

(b) 110 (d) 95 (GATE 2011, 1 Mark)

Only option (b), Q2 = PR satisfies

Solution:  Let there be overall x candidates. Initially, those who decided to vote for P and Q are 0.4x and 0.6x, respectively.

From Eq. (i), we have Q2 = (K2)2 = K4 PR = K * K3 = K4 Here, Q2 = PR holds true. Ans. (b) 14. Choose the most appropriate word(s) from the options given below to complete the following sentence.

On the day of election 15% of 0.4x = 0.06x went from P’s side to Q’s side and 25% of 0.6x = 0.15x went from Q’s side to P’s side. Now, after transfer P has 0.4x - 0.06x + 0.15x = 0.49x and after transfer Q has

In contemplated Singapore for my vacation but decided against it.

0.6x - 0.15x + 0.06x = 0.51x Given, in the question that P lost by 2 votes

(a) to visit (c) visiting

(b) having to visit (d) for a visit (GATE 2011, 1 Mark)

Therefore, total number of votes are 100.

Solution:  The answer is `visiting'. Ans. (c)

GATE_GA-Questions.indd 756

Q - P = 2 votes 0.51x - 0.49x = 0.02x = 2 votes Ans. (a)

8/22/2017 5:34:25 PM

SOLVED GATE PREVIOUS YEARS’ QUESTIONS     757

18. Choose the most appropriate word from the options given below to complete the following sentence: It was her view that the country’s problems had been by foreign technocrats, so that to invite them to come back would be counter-productive. (a) identified (c) exacerbated

(b) ascertained (d) analysed (GATE 2011, 1 Mark)

Solution:  The given sentence says that inviting foreign technocrats back would be counter- productive. Hence, we can conclude that the foreign technocrats are the reason for the country’s problems. Therefore, the best answer from the options is exacerbated. Ans. (c) 19. Choose the word from the options given below that is most nearly opposite in meaning to the given word: Frequency (a) periodicity (c) gradualness

(b) rarity (d) persistency (GATE 2011, 1 Mark)

Solution:  The term frequency is associated with how quickly the change is occurring. The word opposite to it is gradualness which refers to slow development of a process. Ans. (c) 20. Choose the most appropriate word from the options given below to complete the following sentence: Under ethical guidelines recently adopted by the Indian Medical Association, human genes are to be manipulated only to correct diseases for which treatments are unsatisfactory. (a) similar (c) uncommon

(b) most (d) available (GATE 2011, 1 Mark)

The mixture now has 9 liters of spirit and 1 liter of water. Thus, when 1 liter of mixture is removed, we remove 0.9 liter of spirit and 0.1 liter of water. Now, the mixture has 8.1 liters of spirit and 1.9 liters of water. When 1 liter of mixture is removed, we remove 0.81 liter of spirit and 0.19 liter of water and replace it with 1 liter of water. Therefore, the mixture now contains 7.29 liters. Ans. (d) 22. Few school curricula include a unit on how to deal with bereavement and grief, and yet all students at some point in their lives suffer from losses through death and parting. Based on the above passage which topic would not be included in a unit on bereavement? (a) how to write a letter of condolence (b) what emotional stages are passed through in the healing process (c) what the leading causes of death are (d) how to give support to a grieving friend (GATE 2011, 2 Marks) Solution:  Option (c) would not be included. Ans. (c) 23. The variable cost (V) of manufacturing a product varies according to the equation V = 4q, where q is the quantity produced. The fixed cost (F) of production of same product reduces with q according to the equation F = 100/q. How many units should be produced to minimize the total cost (V + F)? (a) 5 (c) 7

(b) 4 (d) 6 (GATE 2011, 2 Marks)

Solution:  Total cost can be given as TC = 4q +

100 q

For cost to be minimum, we have Solution:  Uncommon Ans. (c) 21. A container originally contains 10 litres of pure spirit. From this container 1 litre of spirit is replaced with 1 litre of water subsequently, 1 litre of the mixture is again replaced with 1 litre of water and this processes is repeated one more time. How much spirit is now left in the container? (a) 7.58 litres (c) 7 litres

(b) 7.84 litres (d) 7.29 litres (GATE 2011, 2 Marks)

Solution:  We have 10 liters of pure spirit. Now 1 liter is replaced with water.

GATE_GA-Questions.indd 757

d  100  =0 4q + dq  q  ⇒ q2 = 25 or q = 5 d  100   >0 4q + dq  q q = 5 Hence, total cost is minimum at q = 5 Ans. (a) 24. P, Q, R and S are four types of dangerous microbes recently found in a human habitat. The area of each circle with its diameter printed in brackets

8/22/2017 5:34:27 PM

758     GENERAL APTITUDE

Toxicity (miligrams of microbe required to destory half of the body mass in kilograms)

represents the growth of a single microbe surviving human immunity system within 24 hours of entering the body. The danger to human beings varies proportionately with the toxicity, potency and growth attributed to a microbe shown in figure below:

1000 P(50 mm) 800

Hence, DS is maximum and most dangerous among them. Ans. (d) 25. A transporter receives the same number of orders each day. Currently, he has some pending orders (backlog) to be shipped. If he uses 7 trucks, then at the end of the 4th day he can clear all the orders. Alternately, if he uses only 3 trucks, then all the orders are cleared at the end of the 10th day. What is the minimum number of trucks required so that there will be no pending order at the end of the 5th day?

600 (a) 4

Q(40 mm) R(30 mm)

400

(c) 6 (d) 7 (GATE 2011, 2 Marks)

S(20 mm)

200 0 0.2

(b) 5

0.4 0.6 0.8 1 (Probability that microbe will overcome human immunity system)

Solution:  Let y be the backlog with transporter and x be the number of orders each day. So, as per conditions given in question 4x + y = 28 (i)

10x + y = 30

(ii)

Solving Eqs. (i) and (ii), A pharmaceutical company is contemplating the development of a vaccine against the most dangerous microbe. Which microbe should the company target in its first attempt? (a) P (c) R

(b) Q (d) S (GATE 2011, 2 Marks)

Solution:  We can understand that the danger of a microbe to human being will be directly proportional to potency and growth. At the same time, it will be inversely proportional to toxicity defined (more dangerous will a microbe be if lesser of its milligram is required). So level of danger is proP ↑ ×G ↑ portional to where, P, G and T are the T ↓ potency, growth and toxicity as defined in question. KPG Di = T

So,

(i)

where K is constant of proportionality. So level of danger of S will be maximum and is given by

DP =

0.4 × p (50mm)2 = 3.927 800

DQ =

0.5 × p (40mm) = 4.189 600

DR =

0.4 × p (30mm)2 = 3.77 300

2

0.8 × p (20mm)2 DS = = 5.026 200

GATE_GA-Questions.indd 758

x=

1 80 and y = 3 3

Since, we need to find out number of trucks so that no pending order will be there at the end of 5th day. 5x + y = n × 5 So we need to find n, where n is the number of trucks required. 5x + y n= = 5

85 1 80 5× + 3 3 = 3 5 5

Hence, 5.66 truck will be required. As number of trucks have to be natural number, 6 trucks will be required. Ans. (c) 26. The horse has played a little known but very important role in the field of medicine. Horses were injected with toxins of diseases until their blood built up immunities. Then a serum was made from their blood. Serums to fight with diphtheria and tetanus were developed this way. It can be inferred from the passage that horses were (a) given immunity to disease (b) generally quite immune to disease (c) given medicines to fight toxins (d) given diphtheria and tetanus serums (GATE 2011, 2 Marks) Solution:  It can be inferred that the horses were given medicines to fight toxins. Ans. (c)

8/22/2017 5:34:29 PM

SOLVED GATE PREVIOUS YEARS’ QUESTIONS     759

120

Solution:  We are given that R took 1/3rd of toffees initially. Therefore, total number of toffees have to be multiple of 3. There is only one option (c) as 48 which is multiple of 3. Ans. (c)

90

29. Given that f(y) = |y|/y, and q is any non-zero real numerator, the value of |f(q) - f(−1)| is

Fuel consumption (kilometers per liter)

27. The fuel consumed by a motorcycle during a journey while travelling at various speeds is indicated in the graph below

(a) 0

(b) −1

(c) 1 (d) 2 (GATE 2011, 2 Marks)

60 y

Solution:  f( y) =

30

y

As we know,

0 0

15

30

45 60 75 Speed (kilometers per hour)

y =y

90

−y



y ≥ 0  y < 0

(1)

Therefore, from Eq. (1), we have The distances covered during four laps of the journey are listed in the table below:

f( y) =

y = 1    if y ≥ 0 y

f( y) =

−y = −1  if y < 0 y

Distance (kilometer)

Average speed (kilometer per hour)

P

15

15

Q

75

45

Ans. (d)

R

40

75

S

10

10

30. The sum of n terms of the series 4 + 44 + 444 + … is

Lap

Hence, |f(q) - f(−q) = |1 - (−1)| = 2

From the given data, we can conclude that the fuel consumed per kilometer was least during the lap (a) P

(b) Q

(c) R (d) S (GATE 2011, 2 Marks)

Solution:  Fuel consumed per kilometer will we least, when mileage (kilometers per litres) mentioned on y axis of graph will be maximum irrespective of number of kilometers travelled. From the graph we can observe that mileage (kilometers per litres) is maximum when vehicle is driven at 45 kilometers per hour. Thus, fuel consumption will be lowest for lap Q.

(a) (4/81) [10n + 1 − 9n - 1] (b) (4/81) [10n − 1 − 9n - 1] (c) (4/81) [10n + 1 − 9n - 10] (d) (4/81) [10n − 9n - 10] (GATE 2011, 2 Marks) Solution:  4 + 44 + 444 +… If we see first term of series = 4 Hence, sum up to 1st term is also = 4 Put n = 1 (first term), only option (c) satisfies. 4  n +1 4 − 9n − 10 = [101+1 − 9 × 1 − 10 ] 10    81 81

Ans. (b) = 28. Three friends, R, S and T shared toffee from a bowl. R took 1/3rd of the toffees, but returned four to the bowl. S took 1/4th of what was left but returned three toffees to the bowl. T took half of the remainder but returned two back into the bowl. If the bowl had 17 toffees left, how many toffees were originally there in the bowl? (a) 38

GATE_GA-Questions.indd 759

(b) 31

(c) 48 (d) 41 (GATE 2011, 2 Marks)

4 × 81 = 4 81

Hence, option (c) is correct answer. (Note: Always solve these questions by putting values.) Alternative Solution 4 + 44 + 444 + … =

4 [9 + 99 + 999 + …] 9

8/22/2017 5:34:32 PM

760     GENERAL APTITUDE

=

4 (10 − 1) + (102 − 1) + (103 − 1) + …  9 

4 (10 + 102 + 103 + … + 10n ) − (1 + 1 + 1)…  9  4  n +1 = − 9n − 10 10  81  Ans. (c) =

31. The cost function for a product in a firm is given by 5q2, where q is the amount of production. The firm can sell the product at a market price of 50 per unit. The number of units to be produced by the firm such that the profit is maximized is (a) 5

(b) 10

(c) 15 (d) 25 (GATE 2012, 1 Mark)

Solution:  P = 50q - 5q2

⇒ 50 - 10q = 0

34. Choose the grammatically INCORRECT sentence: (a) They gave us the money back less than service charges to Three Hundred rupees. (b) This country’s expenditure is not less than that of Bangladesh. (c) The committee initially asked for a funding of Fifty Lakh rupees, but later settled for a lesser sum. (d) This country’s expenditure on educational reforms is very less. (GATE 2012, 1 Mark)

35. Choose the most appropriate alternative from the options given below to complete the following sentence: Suresh’s dog is the one was hurt in the stampede.

q=5 = −10 which is negative

dq 2

Ans. (a)

Solution:  The grammatically incorrect sentence is “They gave us the money back less than service charges to Three Hundred rupees.” Ans. (a)

dP For max/min, =0 dq

d 2P

Solution:  Diminish is the correct answer. Mitigate means to reduce, to lessen etc. So, only the word diminish is close. Rest all choices have no link with the given word.

q =5

Therefore, maximum profit will happen at q = 5. Ans. (a)

(c) who (d) whom (GATE 2012, 1 Mark)

32. Choose the most appropriate alternative from the options given below to complete the following sentence:

Solution:  Suresh’s dog is the one that was hurt in the stampede. That is used with restrictive clauses.

Despite several the mission succeeded in its attempt to resolve the conflict.

Ans. (a)

(a) attempts (c) meetings

(b) setbacks (d) delegations (GATE 2012, 1 Mark)

Solution:  Setbacks is the correct answer. Despite several setbacks the mission succeeded in its attempt to resolve the conflict. The word despite indicates that there has to be a contrast in the sentence, use of the word setbacks in the blank indicates that despite many problems the mission was successful. Ans. (b) 33. Which one of the following options is the closest in meaning to the word given below? Mitigate (a) Diminish (c) Dedicate

GATE_GA-Questions.indd 760

(b) Divulge (d) Denote (GATE 2012, 1 Mark)

(a) that

(b) which

36. Choose the most appropriate alternative from the options given below to complete the following sentence: If the tired soldier wanted to lie down, he the mattress out on the balcony. (a) should take (c) should have taken

(b) shall take (d) will have taken (GATE 2012, 1 Mark)

Solution:  If the tired soldier wanted to lie down, he should have taken the mattress out on the balcony. As the sentence is in past tense, options (a), (b) and (d) are incorrect. Ans. (c) 37. If (1.001)1259 = 3.52, (1.001)2052 = 7.85, then (1.001)3321 (a) 2.23

(b) 4.33

(c) 11.37 (d) 27.64 (GATE 2012, 1 Mark)

8/22/2017 5:34:34 PM

SOLVED GATE PREVIOUS YEARS’ QUESTIONS     761

Solution:  (1.001)1259 = 3.52 and (1.001)2062 = 7.85 (1.001)1259 × (1.001)2062 = (1.001)3321 3.52 × 7.85 = (1.001)3321 (as am × an = am+n)

No other word fits in the blank. Beggary is word that is related to extreme poverty (Beggar). Nomenclature refers to the art of naming.

38. One of the parts (A, B, C, D) in the sentence given below contains an ERROR. Which one of the following is INCORRECT?

Ans. (d) 41. Wanted Temporary, Part-time persons for the post of Field Interviewer to conduct personal interviews to collect and collate economic data. Requirements; High School-pass, must be available for Day, Evening and Saturday work, Transportation paid, expenses reimbursed.

I requested that he should be given the driving test today instead of tomorrow.

Which one of the following is the best inference from the above advertisement?

(a) requested that (c) the driving test

(a) Gender-discriminatory (b) Xenophobic (c) Not designed to make the post attractive (d) Not gender-discriminatory (GATE 2012, 2 Marks)

Hence (1.001)3321 = 27.632 ≈ 27.64 Ans. (d)

(b) should be given (d) instead of tomorrow (GATE 2012, 1 Mark)

Solution:  The incorrect part of the sentence is should be given. Ans. (b) 39. Which one of the following options is the closest in meaning to the word given below?

Option (b), xenophobic is one who has fear of foreigners, no link with the given argument.

Latitude (a) Eligibility (c) Coercion

Solution:  Option (a) cannot be considered since there is no gender discrimination mentioned in the argument.

(b) Freedom (d) Meticulousness (GATE 2012, 1 Mark)

Solution:  Latitude refers to freedom of action, freedom of expression from restrictions etc. For example, he allowed his children a fair amount of latitude. Coercion refers to force which is an opposite of the word latitude. Meticulousness refers to being extremely careful and conscientious. Ans. (b) 40. Choose the most appropriate word from the options given below to complete the following sentence:

Option (c), it is wrong to say that the profile has not been designed to make the post attractive, since there are certain features which have been added to make the profile lucrative (which are given towards the end of the advertisement, like Transportation paid, expenses reimbursed). Hence, the best discriminatory.

inference

(a) beggary (c) jealously

Solution:  y = 2x - 0.1x2

GATE_GA-Questions.indd 761

gender-

42. A political party orders an arch for the entrance to the ground in which the annual conventions is being held. The profile of the arch follows the equation y = 2x - 0.1x2 where y is the height of the arch in meters. The maximum possible height of the arch is (a) 8 meters (c) 12 meters

Solution:  Given the seriousness of the situation that he had to face, his nonchalance was impressive. Nonchalance refers to casual/non-serious/indifferent attitude. The author wants to convey that in spite of the seriousness of the situation, he was very casual.

Not

Ans. (d)

Given the seriousness of the situation that he had to face, his was impressive. (b) nomenclature (d) nonchalance (GATE 2012, 1 Mark)

is

(b) 10 meters (d) 14 meters (GATE 2012, 2 Marks)

For y (height) to be maximum,

dy =0 dx

dy = 2x − 2x = 0 dx ⇒ x = 10 d 2y

= −0.2(−ve),

dx 2

d 2y dx 2

y > 1, which of the following must be true?

(i) ln x > ln y (ii) ex > ey (iii) yx > xy (iv) cos x > cos y

(b) leads to (d) affects (GATE 2015, 1 Mark)

Solution:  Apparent: looks like Dormant: hidden Harbour: give shelter Effect (verb): results in Ans. (a)

(a) (i) and (ii) 204. Choose the statement where underlined word is used correctly.

(b) (i) and (iii) (c) (iii) and (iv) (d) (ii) and (iv) (GATE 2015, 1 Mark) Solution:  For whole numbers, greater the value, greater will be its logarithmic. Same logic applies for power of e. Ans. (a) 201. Which of the following options is the closest in meaning to the sentence below? She enjoyed herself immensely at the party.

GATE_GA-Questions.indd 789

(a) When the teacher eludes to different authors, he is being elusive. (b) When the thief keeps eluding the police, he is being elusive. (c) Matters that are difficult to understand, identify or remember are allusive. (d) Mirages can be allusive, but a better way to express them is illusory. (GATE 2015, 1 Mark) Solution:  Elusive: difficult to answer. Ans. (b)

8/22/2017 5:35:50 PM

790     GENERAL APTITUDE

205. Fill in the blank with the correct idiom/phrase. That boy from the town was a in the sleepy village. (a) dog out of herd (b) sheep from the heap (c) fish out of water (d) bird from the flock

AB = BC = 6 m Thus, AB¢′ = B¢C′ = 6 cos 60° = 3 m Hence, AC = 3 + 3 = 6 m AE¢′ = 6 - 4 = 2 m EE¢′ = 2 m Hence, using Pythagoras theorem, we get AE = (EE ′)2 + (AE ′)2 = 22 + 22 = 8 = 2 2 m

(GATE 2015, 1 Mark) Ans. (a) Solution:  From the statement, it appears that boy found it tough to adapt to a very different situation. Ans. (c) 206. Tanya is older than Eric. Cliff is older than Tanya. Eric is older than Cliff. If the first two statements are true, then the third statement is (a) True (b) False (c) Uncertain (d) Data insufficient

Solution:  The correct answer is option (b). Ans. (b) 207. Mr. Vivek walks 6 m North-east, then turns and walks 6 m South-east, both at 60° to East. He further moves 2 m South and 4 m West. What is the straight distance in meters between the point he started from and the point he finally reached? (b) 2 (d) 1/ 2

(b) He ensured that the company will not have to bear any loss. (c) The actor got himself ensured against any accident. (d) The teacher insured students of good results.

Solution:  Consider the figure formed from the data given in the question, where A is the starting point and E is the destination. B 60°

6m

Solution:  Insured — the person, group, or organization whose life or property is covered by an insurance policy. Ensured — to secure or guarantee. Ans. (b) 209. Which word is not a synonym for the word vernacular? (a) Regional (b) Indigenous (c) Indigent (d) Colloquial (GATE 2015, 1 Mark)

(GATE 2015, 1 Mark)

60°

(a) The minister insured the victims that everything would be all right.

(GATE 2015, 1 Mark)

(GATE 2015, 1 Mark)

(a) 2 2 (c) 2

208. Choose the statement where underlined word is used correctly.

Solution:  Vernacular — expressed or written in the native language of a place. Indigent — deficient in what is requisite. Ans. (c) 210. Four cards are randomly selected from a pack of 52 cards. If the first two cards are kings, what is the probability that the third card is a king? (a) 4/52 (b) 2/50 (c) (1/52) × (1/52) (d) (1/52) × (1/51) × (1/50)

6m B′ 60°

A

C

(GATE 2015, 1 Mark)

E′ 2m E

GATE_GA-Questions.indd 790

4m

D

Solution:  There are 4 kings in a pack of 52 cards. If 2 cards are selected and both are kings, remaining cards will be 50 out of which 2 will be kings.

8/22/2017 5:35:51 PM

SOLVED GATE PREVIOUS YEARS’ QUESTIONS     791

Thus, the probability =

2 50

Ans. (b)

213. The given statement is followed by some course of action. Assuming the statement to be true, decide the correct option. Statement:

211. Choose the most appropriate word from the options given below to complete the following sentence:

There has been a significant drop in the water level in the lakes supplying water to the city.

The official answered that the complaints of the citizen would be looked into.

Course of action: (a) respectably (b) respectfully (c) reputably (d) respectively

(i) The water supply authority should impose a partial cut in supply to tackle the situation. (ii) The government should appeal to all the residents through mass media for minimal use of water. (iii) The government should ban the water supply in lower areas.

(GATE 2015, 1 Mark) Solution:  The correct answer is `respectfully’. Ans. (b)

(a) Statements (i) and (ii) follow (b) Statements (i) and (iii) follow (c) Statements (ii) and (iii) follow (d) All statements follow (GATE 2015, 2 Marks)

212. The pie chart below has the breakup of the number of students from different departments in an ­engineering college for the year 2012. The proportion of male to female students in each department is 5:4. There are 40 males in Electrical Engineering. What is the difference between the numbers of female students in the Civil department and the female students in the Mechanical department?

Solution:  Both statements (i) and (ii) follow. Ans. (a) 214. Select the alternative meaning of the underlined part of the sentence. The chain snatchers took to their heels when the police party arrived.

Mechanical, 10%

Computer Science, 40%

(a) took shelter in a thick jungle (b) open indiscriminate fire (c) took to flight (d) unconditionally surrendered

Electrical, 20%

(GATE 2015, 2 Marks) Solution:  `Took to flight’ has the same meaning. Ans. (c)

Civil, 30% (GATE 2015, 2 Marks) Solution: Electrical male students = 40 Thus, electrical female students =

4 × 40 = 32 5

Total female students in Mechanical department 32 = = 16 2 Total female students in Civil department = 3 32 × = 48 2

215. The number of students in a class, who have answered correctly, wrongly, or not attempted each question in exam, are listed in the table below. The marks from each question are also listed. There is no negative or partial marking. Q. Marks Answered Answered Not No. Correctly Wrongly Attempted 1

2

21

14

6

2

3

15

27

2

3

1

11

29

4

4

2

23

18

3

5

5

31

12

1

Difference = 48 − 16 = 32 Ans. 32

GATE_GA-Questions.indd 791

8/22/2017 5:35:53 PM

792     GENERAL APTITUDE

What is the average of the marks obtained by the class in the examination? (a) 2.290 (c) 6.795

Putting above value in Eq. (iv), we get p + c + m − 0.7 = 0.65

(b) 2.970 (d) 8.795

p + c + m = 1.35 =

27 20 Ans. (d)

(GATE 2015, 2 Marks)

217. If the list of letters, P, R, S, T, U, is an arithmetic Solution:  Total students = 21 + 17 + 6 = 44 Total marks = 2 × 21 + 3 × 15 + 1 × 11 + 2 × 23 + 5 × 31 = 299 sequence, which of the following are also in arithmetic sequence? × 15 + 1 × 11 + 2 × 23 + 5 × 31 = 299 Average marks =

I. 2P, 2R, 2S, 2T, 2U II. P−3, R−3, S−3, T−3, U−3 III. P2, R2, S2, T2, U2

299 = 6.795 44 Ans. (c)

216. The probabilities that a student passes in Mathematics, Physics and Chemistry are m, p and c, respectively. Of these subjects, the student has a 75% chance of passing in at least one, a 50% chance of passing in at least two and a 40% chance of passing in exactly two. Following relations are drawn in m, p and c. (i) p + m + c = 27/20 (ii) p + m + c = 13/20 (iii) (p) × (m) × (c) = 1/10 (a) Only relation (i) is true (b) Only relation (ii) is true (c) Relations (ii) and (iii) are true (d) Relations (i) and (iii) are true

(GATE 2015, 2 Marks) Solution:  If P, R, S, T, U are in arithmetic sequence with a common difference `d’. Then, 2P, 2R, 2S, 2T, 2U will be an arithmetic sequence with common difference `2d’. P − 3, R − 3, S − 3, T − 3, U − 3 will be an ­arithmetic sequence with common difference `d’. P2 , R 2 , S2 , T2 , U2 will not be an arithmetic sequence. Ans. (b)

(GATE 2015, 2 Marks) Solution:  Probability of passing in at least one subject 1 − (1 − m)(1 − p)(1 − c) = 0.75 

(a) (I) only (b) (I) and (II) (c) (II) and (III) (d) (I) and (III)

218. In a triangle PQR, PS is the angle bisector of ∠QPR and ∠QPS = 60°. What is the length of PS?

P

(i)

r

q

Probability of passing in at least two subjects (1 − m)pc + (1 − p)mc + (1 − c)mp + mpc = 0.5 (ii)

s Q

Probability of passing in exactly two subjects (1 − m)pc + (1 − p)mc + (1 − c)mp = 0.4  (iii) (a)

(q + r) qr

(c) (q 2 + r 2 )

(b)

qr (q + r)

(d)

Subtracting Eq. (iii) from Eq. (ii), we get mpc = 0.1 Simplifying Eq. (i), we get

After simplifying Eq. (iii), we get pc + mc + mp − 3mpc = 0.4 Putting value of mpc, we get pc + mc + mp = 0.7

GATE_GA-Questions.indd 792

(q + r)2 qr

(GATE 2015, 2 Marks)

p + c + m − (mp + mc + pc) + mpc = 0.75 p + c + m − (mp + mc + pc) = 0.65 

R P

Solution: (iv)

 ∠QPR  2qr cos   2  q+r ∠QPR = 2 × ∠QPS = 120 120   qr  2qr cos   PS = =  2   q + r  q+r PS =

Ans. (b)

8/22/2017 5:36:01 PM

SOLVED GATE PREVIOUS YEARS’ QUESTIONS     793

219. Out of the following four sentences, select the most suitable sentence with respect to grammar and usage:

M

(a) Since the report lacked needed information, it was of no use to them. (b) The report was useless to them because there were no needed information in it. (c) Since the report did not contain the needed information, it was not real useful to them. (d) Since the report lacked needed information, it would not have been useful to them.

O

S 45°

T

N

(GATE 2015, 2 Marks) Solution:  Option (a) is the most suitable sentence. Ans. (a)

P 1

SN = 1 sin 45° =

= OT

220. If p, q, r, s are distinct integers such that:

2

f(p, q, r, s) = max(p, q, r, s) g(p, q, r, s) = min(p, q, r, s) h(p, q, r, s) = remainder of (p × q)/(r × s) if (p × q) > (r × s) or remainder of (r × s)/(p × q) of (r × s) > (p × q)

 1  Vertical distance between M and P = 4 + 2 −   2  1 Horizontal distance = 2 Distance between M and P =

 2  2 6 − 1  +  1  = 5.34 km  2   2 

Also a function fgh(p, q, r, s) = f(p, q, r, s) × 2    2 6 − 1  +  1  = 5.34 km g(p, q, r, s) × h(p, q, r, s)   2  2  Ans. (a) Also the same operations are valid with two variable functions of the form f(p, q). 222. Choose the most appropriate equation for the function drawn as a thick line, in the plot below. What is the value of fg(h(2, 5, 7, 3), 4, 6, 8)? (GATE 2015, 2 Marks) y Solution: (2,0)

x

fg(h(2, 5, 7, 3), 4, 6, 8) (0,−1) = fg(1, 4, 6, 8) = f (1, 4, 6, 8) × g(1, 4, 6, 8) = 8 × 1 = 8 Ans. 8 221. Four branches of company are located at M, N, O and P. M is north of N at a distance of 4 km; P is south of O at a distance of 2 km; N is southeast of O by 1 km. What is the distance between M and P in km? (a) 5.34 (c) 28.5



(b) 6.74 (d) 45.49 (GATE 2015, 2 Marks)

Solution:  Given the data in the question, we can form the following figure:

GATE_GA-Questions.indd 793

(a) x = y − |y | (b) x = −(y − |y |) (c) x = y + |y | (d) x = −(y + |y |) (GATE 2015, 2 Marks) Solution: When y = −1, x = 2. Also, when y is positive x = 0. Thus, x = − (y − y ) Ans. (b)

8/22/2017 5:36:04 PM

794     GENERAL APTITUDE

223. Alexander turned his attention toward India, since he had conquered Persia. Which one of the statements below is logically valid and can be inferred from the above sentence?

Which of the statement(s) below is/are logically valid and can be inferred from the above passage?

(a) Alexander would not have turned his attention towards India had he not conquered Persia. (b) Alexander was not ready to rest on his laurels and wanted to march to India. (c) Alexander was completely in control of his army  and could command it to move toward India. (d) Since Alexander’s kingdom extended to Indian borders after the conquest of Persia, he was keen to move further.

(i) He was already a successful batsman at the highest level. (ii) He has to improve his temperament in order to become a great batsman. (iii) He failed to make many of his good starts count. (iv) Improving his technical skills will guarantee success. (a) (iii) and (iv) (b) (ii) and (iii) (c) (i), (ii), and (iii) (d) (ii) only

(GATE 2015, 2 Marks)

(GATE 2015, 2 Marks)

Solution:  Option (a) is logically valid. Ans. (a) 224. The exports and imports (in crores of Rs) of a country from the year 2000 to 2007 are given in the following bar chart. In which year is the combined percentage increase in imports and exports the highest? 120 110 100 90 80 70 60 50 40 30 20 10 0

out that until he addresses this problem, success at the highest level will continue to elude him.

Solution:  Statements (ii) and (iii) are logically valid. Ans. (b) 226. The head of a newly formed government desires to appoint five of the six selected members P, Q, R, S, T and U to portfolios of Home, Power, Defense, Telecom, and Finance. U does not want any portfolio if S gets one of the five. R wants either Home or Finance or no portfolio. Q says that if S gets either Power or Telecom, then she must get the other one. T insists on a portfolio if P gets one. Which is the valid distribution of portfolios?

2000

2001

2002

2003

2004

Exports

2005

2006

2007

Imports

(GATE 2015, 2 Marks)

(a) P — Home, Q — Power, R — Defense, S — Telecom, T − Finance (b) R — Home, S — Power, P — Defense, Q — Telecom, T = Finance (c)  P — Home, Q — Power, T — Defense, S — Telecom, U — Finance (d) Q — Home, U — Power, T — Defense, R — Telecom, P — Finance (GATE 2015, 2 Marks)

Solution: Increase in exports in 2006 =

100 − 70 = 42.8% 70

Increase in imports in 2006 =

120 − 90 = 33.3% 90

Solution:  Since U does not want any portfolio, (c) and (d) are ruled out. R wants Home, or Finance or no portfolio, therefore (a) is not valid. Thus, option (b) is correct. Ans. (b)

which is more than any other year. Ans. 2006 225. Most experts feel that in spite of possessing all the technical skills required to be a batsman of the highest order, he is unlikely to be so due to the lack of requisite temperament. He was guilty of throwing away his wicket several times after working hard to lay a strong foundation. His critics pointed

GATE_GA-Questions.indd 794

227. A cube of side 3 units is formed using a set of smaller cubes of side 1 unit. Find the proportion of the number of faces of the smaller cubes visible to those which are NOT visible. (a) 1:4



(b) 1:3

(c) 1:2



(d) 2:3 (GATE 2015, 2 Marks)

8/22/2017 5:36:04 PM

SOLVED GATE PREVIOUS YEARS’ QUESTIONS     795

Solution:  Number of faces per cube = 6 Total number of cubes = 9 × 3 = 27

230. Find the missing value:

Therefore, total number of faces = 27 × 6 = 162 Hence, total number of non-visible faces = 162 − 54 = 108

1

6

5

4

7

4

7

2

1

9

2

8

1

2

4

1

5

2

3

3

?

3

1

Therefore, Number of visible faces 54 1 = = Number of non-visible faces 108 2

Ans. (c)

228. The following question presents a sentence, part of which is underlined. Beneath the sentence you find four ways of phrasing the underlined part. Following the requirements of the standard written English, select the answer that produces the most effective sentence. Tuberculosis, together with its effects, ranks one of the leading causes of death in India. (a) ranks as one of the leading causes of death (b) rank as one of the leading causes of death (c) has the rank of the leading causes of death (d) are one of the leading causes of death (GATE 2015, 2 Marks) Solution:  `Ranks as one of the leading causes of death’ produces most the effective sentence. Ans. (a) 229. Humpty Dumpty sits on a wall every day while having lunch. The wall sometimes breaks. A person sitting on the wall falls if the wall breaks. Which one of the statements below is logically valid and can be inferred from the above sentences? (a) Humpty Dumpty always falls while having lunch. (b) Humpty Dumpty does not fall sometimes while having lunch.

(GATE 2015, 2 Marks) Solution:  Middle number is the average of the numbers on both sides. Average of 6 and 4 is 5 Average of (7 + 4) and (2 + 1) is 7. Average of (1 + 9 + 2) and (1 + 2 + 1) is 8. Average of (4 + 1) and (2 + 3) is 5. Therefore, average of 3 and 3 is 3. Ans. 3 231. Read the following paragraph and choose the correct statement. Climate change has reduced human security and threatened human well-being. An ignored reality of human progress is that human security largely depends upon environmental security. But on the contrary, human progress seems contradictory to environment security. To keep up both at the required level is a challenge to be addressed by one and all. One of the ways to curb the climate change may be suitable scientific innovations, while the other may be the Gandhian perspective on small scale progress with focus on sustainability. (a) Human progress and security are positively associated with environmental security. (b) Human progress is contradictory to environmental security. (c) Human security is contradictory to environmental security. (d) Human progress depends upon environmental security. (GATE 2015, 2 Marks)

(c) Humpty Dumpty never falls during dinner. (d) When Humpty Dumpty does not sit on the wall, the wall does not break. (GATE 2015, 2 Marks) Solution:  Option (b) is logically valid. Ans. (b)

GATE_GA-Questions.indd 795

Solution:  The correct answer is option (b). Ans. (b) 232. Lamenting the gradual sidelining of the arts in school curricula, a group of prominent artists wrote to the Chief Minister last year, asking him to allocate more funds to support arts education

8/22/2017 5:36:06 PM

796     GENERAL APTITUDE

in schools. However, no such increase has been announced in this year’s budget. The artists expressed their deep anguish at their request not being approved, but many of them remain optimistic about funding in the future.



Which of the statement(s) below is/are logically valid and can be inferred from the above statements? (i) The artists expected funding for the arts to increase this year. (ii) The Chief Minister was receptive to the idea of increasing funding for the arts. (iii) The Chief Minister is a prominent artist. (iv) Schools are giving less importance to arts education nowadays.

235. Given below are two statements followed by two conclusions. Assuming these statements to be true, decide which one logically follows.

Statements: (i) All film stars are playback singers. (ii) All film directors are film stars.



Conclusions: (i) All film directors are playback singers. (ii) Some film stars are film directors. (a) Only conclusion (i) follows. (b) Only conclusion (ii) follows. (c) Neither conclusion (i) or (ii) follows. (d) Both conclusions (i) and (ii) follow. (GATE 2015, 2 Marks)

(a) (iii) and (iv) (b) (i) and (iv) (c) (i), (ii) and (iv) (d) (i) and (iii)

Solution:  Both conclusions (i) and (ii) follow. Ans. (d) (GATE 2015, 2 Marks)

Solution:  (i), (ii) and (iv) are logically valid. Ans. (c)

236. A tiger is 50 leaps of its own behind a deer. The tiger takes 5 leaps per minute to the deer’s 4. If the tiger and the deer cover 8 m and 5 m per leap, respectively, what distance in metres will the tiger have to run to catch the deer?

233. If a2 + b2 + c2 = 1, then ab + bc + ac lies in the interval (a) [1, 2/3] (c) [-1, 1/2]

(b) [-1/2, 1] (d) [2, 4] (GATE 2015, 2 Marks)

Solution:  It lies in the interval [-1/2, 1]. Ans. (b) 234. In the following sentence, certain parts are underlined and marked P, Q and R. One of the parts may contain certain error or may not be acceptable in standard written communication. Select the part containing an error. Choose D as your answer if there is no error. all the errors The student corrected that P the instructor marked answer book. on the R Q (a) P



(b) Q

(c) R



(d) No Error

(GATE 2015, 2 Marks) Solution:  We are given 1 leap of tiger = 8 m Speed = 5 × 8 = 40 m/min 1 leap of deer = 5 m Speed = 5 × 4 = 20 m/min Assume that at time `t’ the tiger catches the deer. Thus, distance travelled by deer + initial distance between them Distance covered by tiger = 50 × 8 = 400 m ⇒ 40 × t = 400 + 20t ⇒ t =

400 = 20 min 20

Total distance = 400 + 20t = 400 + 400 = 800 m

Ans. 800 m

237. Ms. X will be in Bagdogra from 01/05/2014 to 20/05/2014 and from 22/05/2014 to 31/05/2014. On the morning of 21/05/2014, she will reach Kochi via Mumbai. Which one of the statements below is logically valid and can be inferred from the above sentences?

(GATE 2015, 2 Marks) Solution:  Part Q contains error.

GATE_GA-Questions.indd 796

Ans. (b)

(a) Ms. X will be in Kochi for one day, only in May. (b) Ms. X will be in Kochi for only one day in May.

8/22/2017 5:36:10 PM

SOLVED GATE PREVIOUS YEARS’ QUESTIONS     797

(c) Ms. X will be only in Kochi for one day in May. (d) Only Ms. X will be in Kochi for one day in May. (GATE 2015, 2 Marks) Solution:  Second sentence says that Ms. X reaches Kochi on 21/05/2014. Also she has to be in Bagdogra on 22/05/2014. Therefore, she stays in Kochi for only one day in May. Ans. (b) 238. log tan 1° + log tan 2° + is .

+ log tan 89°

(a) 1

(b) 1 / 2

(c) 0

(d) -1 (GATE 2015, 2 Marks)

Solution:  log tan 1 + log tan 89 log tan 1 + log tan 89 = log(tan 1 × tan 89 ) = log(tan 1 × cot 1 ) = log(tan 1 × tan 89 ) = log(tan 1 × cot 1 ) = log 1 = 0 = log 1 = 0 Using the same logic total sum is `0’.

Solution:   The correct way to express is `It would remain between you and me’. Ans. (a) 241. In the following question, the first and the last sentence of the passage are in order and numbered 1 and 6. The rest of the passage is split into four parts and numbered as 2, 3, 4 and 5. These four parts are not arranged in proper order. Read the sentences and arrange them in a logical sequence to make a passage and choose the correct sequence from the given options. 1. On Diwali, the family rises early in the morning. 2. The whole family, including the young and the old enjoy doing this. 3. Children let off fireworks later in the night with their friends. 4. At sunset, the lamps are lit and the family performs various rituals. 5. Father, mother, and children visit relatives and exchange gifts and sweets. 6. Houses look so pretty with lighted lamps all around. (a) 2, 5, 3, 4       (b) 5, 2, 4, 3

Ans. (c) (c) 3, 5, 4, 2       (d) 4, 5, 2, 3

239. From a circular sheet of paper of radius 30 cm, a sector of 10% area is removed. If the remaining part is used to make a conical surface, then the ratio of the radius and height of the cone is . (GATE 2015, 2 Marks) Solution:  90% of area of sheet = Cross-sectional area of cone ⇒ 0.9p × 30 × 30 = p × r1 × 30 ⇒ r1 = 27 Therefore, height of the cone = 2

240. Ram and Shyam shared a secret and promised to each other it would remain between them. Ram expressed himself in one of the following ways as given in the choices below. Identify the correct way as per standard English. (a) It would remain between you and me. (b) It would remain between I and you. (c) It would remain between you and I. (d) It would remain with me. (GATE 2015, 2 Marks)

GATE_GA-Questions.indd 797

Solution:  The correct sequence is `5, 2, 4, 3’. Ans. (b) 242. Right triangle PQR is to be constructed in the xyplane so that the right angle is at P and line PR is parallel to the x-axis. The x and y coordinates of P, Q, and R are to be integers that satisfy the inequalities: 4 ≤ x ≤ 5 and 6 ≤ y ≤ 16. How many different triangles could be constructed with these properties?

302 − 272 = 13.08 cm

30 − 27 = 13.08 cm Ans. 13.08 cm 2

(GATE 2015, 2 Marks)

(a) 110 (c) 9900

(b) 1100 (d) 10,000 (GATE 2015, 2 Marks)

Solution:  P can take a total of 10 × 11 = 110 coordinates. For a given P, R can take a total of 9 coordinates and Q can take a total of 10 coordinates. Hence, total number of different triangles = 110 × 9 × 10 = 9900. Ans. (c) 243. A coin is tossed thrice. Let X be the event that head occurs in each of the first two tosses. Let Y  be

8/22/2017 5:36:12 PM

798     GENERAL APTITUDE

the event that a tail occurs on the third toss. Let Z be the event that two tails occur in three tosses. Based on the above information, which one of the following is TRUE?

Therefore, area of triangle QTS =

1 × SQ × QT 2

 2  1 2 1  1  1 ×  PQ ×  QR  = × ×  × PQ × QR      7  4 7 2  2 4 1 = × Area of ∆PQR 14 Given, 1 20 = × A ⇒ A = 14 × 20 = 280 cm2 14 Ans. 280 cm2 =

(a) X and Y are not independent (b) Y and Z are dependent (c) Y and Z are independent (d) X and Z are independent (GATE 2015, 2 Marks) Solution:  Let Y be the event that tail occurred in third toss. And Z be two tails in third toss which can be {TTH, THT, HTT}. Y = {TTH, TTT} Thus, both Y and Z are dependent. Ans. (b) 244. Given below are two statements followed by two conclusions. Assuming these statements to be true, decide which one logically follows. Statements: (i) No manager is a leader. (ii) All leaders are executives. Conclusions: (i) No manager is an executive. (ii) No executive is a manager.

246. Select the appropriate option in place of underlined part of the sentence. Increased productivity necessary reflects greater efforts made by the employees. (a) Increase in productivity necessary (b) Increase productivity is necessary (c)  Increase in productivity necessarily (d) No improvement required. (GATE 2015, 2 Marks) Solution:  The correct answer is option (c). Ans. (c)

(a) Only conclusion (i) follows. (b) Only conclusion (ii) follows. (c)  Neither conclusion (i) nor (ii) follows. (d) Both conclusions (i) and (ii) follow. (GATE 2015, 2 Marks) Solution:  The correct answer is option (c). Ans. (c) 245. In the given figure, angle Q is a right angle, PS:QS = 3:1, RT:QT = 5:2 and PU:UR = 1:1. If area of triangle QTS is 20 cm2, then the area of triangle PQR in cm2 is . R

U T

247. Read the following table giving sales data of five types of batteries for years 2006—2012. Year

Type I

Type II

Type III

Type IV

Type V

2006

75

144

114

102

108

2007

90

126

102

84

126

2008

96

114

75

105

135

2009

105

90

150

90

75

2010

90

75

135

75

90

2011

105

60

165

45

120

2012

115

85

160

100

145

Out of the following, which type of battery achieved highest growth between the years 2006 and 2012? (a) Type V (b) Type III

P S

Q (c) Type II

(GATE 2015, 2 Marks) (d) Type I Solution:  Let area of triangle PQR be `A’ 1 1 SQ = = 4 PQ 1 + 3 2 2 QT = = 2 + 5 7 QR

GATE_GA-Questions.indd 798

(GATE 2015, 2 Marks) Solution:  Type I achieved a growth of 53% in the period which is higher than any other type of battery. Ans. (d)

8/22/2017 5:36:14 PM

SOLVED GATE PREVIOUS YEARS’ QUESTIONS     799

248. There are 16 teachers who can teach Thermo­ dynamics (TD), 11 who can teach Electrical Sciences (ES), and 5 who can teach both TD and Engineering Mechanics (EM). There are a total of 40 teachers, 6 cannot teach any of the three subjects, that is EM, ES or TD. Six can teach only ES. Four teach all three subjects, that is EM, ES and TD. Four can teach ES and TD. How many can teach both ES and EM but not TD?

251. The given question is followed by two statements; select the most appropriate option that solves the question. Capacity of a solution tank A is 70% of the capacity of tank B. How many gallons of solution are in tank A and tank B?

(a) 1

(b) 2

Statements: (i) Tank A is 80% full and tank B is 40% full (ii) Tank A is full and contains 14,000 gallons of solution.

(c) 3

(d) 4

(a) Statement (i) alone is sufficient

(GATE 2015, 2 Marks)

(b) Statement (ii) alone is sufficient



(c) Either statement (i) or (ii) alone is sufficient. Solution:  Using the data given in the question, we can form the Venn diagram as follows: TD 11

(d) Both the statements (i) and (ii) together are sufficient.

ES

(GATE 2015, 2 Marks)

6

Solution:  Statement I can be used to solve the question if capacity of both tanks is already known. Statement II can be used if it is known what quantity of each tank is full/empty. Now, let capacity of tank B be x.

0 4

1

1 11 EM Ans. (a)

249. How many four digit numbers can be formed with the 10 digits, 0, 1, 2, ..., 9, if no number can start with 0 and if repetitions are not allowed? (GATE 2015, 2 Marks) Solution:  In thousands place, 9 digits except 0 can be placed. In hundreds place, 9 digits can be placed (including 0, excluding the one used in thousands place). In tens place, 8 digits can be placed (excluding the ones used in thousands and hundreds place). In ones place, 7 digits can be placed (excluding the one used in thousands, hundreds and tens place). Total number of combinations = 9 × 9 × 8 × 7 = 4536 Ans. 4536 250. The word similar in meaning to `dreary’ is (a) Cheerful (b) Dreamy (c) Hard (d) Dismal

70 x = 14000 100 ⇒ x = 2000 gallons 80 × 14000 = 11200 gallons 100 40 Solution in tank B = × 20000 = 8000 gallons 100 Thus, total solution = 11200 + 8000 = 19200 gallons Ans. (d) Solution in tank A =

252. Out of the following four sentences, select the most suitable sentence with respect to grammar and usage. (a) I will not leave the place until the minister does not meet me. (b) I will not leave the place until the minister doesn’t meet me. (c) I will not leave the place until the minister meet me. (d) I will not leave the place until the minister meets me. (GATE 2016, 1 Mark)

(GATE 2015, 2 Marks) Solution:  Dreary — depressingly dull and bleak or repetitive. Ans. (d)

GATE_GA-Questions.indd 799

Solution:  `Until’ itself is negative word hence the sentence after `until’ must be affirmative. Minister is singular and hence `s’ is added with the verb. Ans. (d)

8/22/2017 5:36:15 PM

800     GENERAL APTITUDE

253. A rewording of something written or spoken is a __________. (a) paraphrase (b) paradox (c) paradigm (d) paraffin

(a) 56 (b) 64 (c) 72 (d) 96 (GATE 2016, 1 Mark)

(GATE 2016, 1 Mark) Solution:  If any written or spoken sentence is reworded, it is called paraphrasing. Ans. (a) 254. Archimedes said, “Give me a lever long enough and a fulcrum on which to place it, and I will move the world.” The sentence above is an example of a __________ statement. (a) figurative (b) collateral (c) literal (d) figurine

Solution:  Side of a cube = 1 unit. Assume that side of larger cube is made of X units. Then volume of larger cube = Volume of all 64 cubes. X 3 = 64 × (1)3 X=

3

64 = 4 units

2

Surface area, A = 6 × 4  = 96 units If one cube is removed from each corner, then three surfaces one on each side will be removed and three surfaces will also be added. Hence, no change in surface area will be there. Ans. (d) 257. The man who is now Municipal Commissioner worked as __________.

(GATE 2016, 1 Mark) Solution:  The sentence under quotes is denoting figures and hence this sentence is figurative. Ans. (a)

(a) the security guard at a university (b) a security guard at the university (c) a security guard at university (d) the security guard at the university (GATE 2016, 1 Mark)

255. If `relftaga’ means carefree, `otaga’ means careful and `fertaga’ means careless, which of the following could mean `aftercare’? (a) zentaga (b) tagafer (c) tagazen (d) relffer

Solution:  The man who is now Municipal Commissioner worked as the security guard at a university. Ans. (a) 258. Nobody knows how the Indian cricket team is going to cope with the difficult and seamer-friendly wickets in Australia.

(GATE 2016, 1 Mark) Solution:  refltaga = care free otaga = care full

Choose the option which is closest in meaning to the underlined phrase in the above sentence. (a) put (b) put (c) put (d) put

Hence, `taga’ = care Now, fertaga = care less

up with in with down to up against

Hence, `fer’ = less

(GATE 2016, 1 Mark)

Therefore, tagazen = aftercare Solution:  Cope with = Put up with Ans. (c) Ans. (a) 256. A cube is built using 64 cubic blocks of side one unit. After it is built, one cubic block is removed from every corner of the cube. The resulting surface area of the body (in square units) after the removal is __________.

GATE_GA-Questions.indd 800

259. Find the odd one in the following group of words: mock, deride, praise, jeer (a) mock (b) deride

8/22/2017 5:36:16 PM

SOLVED GATE PREVIOUS YEARS’ QUESTIONS     801

(c) praise (d) jeer

= (GATE 2016, 1 Mark)

Solution:  Praise is the odd word. Its meaning is opposite to that of the other words. Ans. (c)

an + bn an + bn = n 1 1 b + an + n n a b a nb n

= an ⋅ bn

(a n + b n ) = (ab )n = 4n an + bn Ans. (b)

260. Pick the odd one from the following options: 262. Which of the following is CORRECT with respect to grammar and usage?

(a) CADBE (b) JHKIL (c) XVYWZ (d) ONPMQ

Mount Everest is __________.

(GATE 2016, 1 Mark) Solution:  Number the alphabet as shown in the following figure:

(a) the highest peak in the world (b) highest peak in the world (c) one of highest peak in the world (d) one of the highest peak in the world (GATE 2016, 1 Mark)

1

2

3

4

5

A

B

C

D

E F

H

I

J

K

L

M

N

O

P

Q R

U V

W

Y

Z

Solution:  The sentence is stating the highest peak in the world. Since it is a specific thing, we need to use the definite article `the’ before it. Also the sentence is using the superlative degree and so we say `the highest peak in the world’ making option (a) the correct answer. There cannot be many highest peaks in the world and so options (c) and (d) are incorrect. Ans. (a)

G

Options (a), (b) and (c) are in the order 31425.

263. The policeman asked the victim of a theft, “What did you __________?”

Only option (d) is in the order 32415. (a) loose

(b) lose

(c) loss

(c) louse

Ans. (d) 261. In a quadratic function, the value of the product of the roots (α, β) is 4. Find the value of an + b n a −n + b −n (a) n4 (b) 4n (c) 22n−1 (d) 4n−1 (GATE 2016, 1 Mark) Solution:  αβ = 4  (Given) Now, an + bn a −n + b −n

GATE_GA-Questions.indd 801

(GATE 2016, 1 Mark) Solution:  The context of the sentence is asking a person who has been deprived of something because of a theft. The word to be used to fill the blank is `lose’ which means to be deprived of something. `Loose’ means something that is not fitted. `Louse’ is the singular form of the word `lice’ that is a parasite that lives in the skin of mammals and birds. `Loss’ is a noun that means the process of losing someone or something. For example, he suffered tremendous loss in his business. Ans. (b) 264. Despite the new medicines __________ in treating diabetes, it is not __________ widely. (a) effectiveness — prescribed (b) availability — used

8/22/2017 5:36:18 PM

802     GENERAL APTITUDE

(c) prescription — available (d) acceptance — proscribed (GATE 2016, 1 Mark) Solution:  The sentence is looking for a contrast as it is joined by the conjunction `despite’. The best pair of words that can fit the context of the sentence is `effectiveness — prescribed’. Though the medicine is `effective’ in treating diabetes, it is not being `prescribed’ widely. A new medicine cannot have a `prescription’ or `availability’ for treating a disease. `Proscribed’ means forbidden by law. In case we use `acceptance — proscribed’ the sentence will not make any sense because it will mean that though the medicine is accepted widely, it is not forbidden by law. Ans. (a) 265. In a huge pile of apples and oranges, both ripe and unripe mixed together, 15% are unripe fruits. Of the unripe fruits, 45% are apples. Of the ripe ones, 66% are oranges. If the pile contains a total of 5,692,000 fruits, how many of them are apples? (a) 2,029,198

(b) 2,467,482

(c) 2,789,080

(d) 3,577,422

where I live. Arun is farther away than Ahmed but closer than Susan from where I live. From the information provided here, what is one possible distance (in km) at which I live from Arun’s place? (a) 3.00

(b) 4.99

(c) 6.02

(d) 7.01 (GATE 2016, 1 Mark)

Solution:  In the question, it is given that Ahmed is 5 km away and Susan is 7 km away from where I live. Further, it is given that Arun is farther away than Ahmed from where I live and not as far as Susan. That means Arun must be living at distance more than 5 km but less than 7 km from my house which is according to given options can be 6.02 km. Ans. (c) 267. Based on the given statements, select the appropriate option with respect to grammar and usage. Statements: (i) The height of Mr. X is 6 feet. (ii) The height of Mr. Y is 5 feet. (a) Mr. (b) Mr. (c) Mr. (d) Mr.

X X X X

is is is is

longer than Mr. Y. more elongated than Mr. Y. taller than Mr. Y. lengthier than Mr. Y.

(GATE 2016, 1 Mark) (GATE 2016, 1 Mark) Solution:  Let Solution:  T = Total number of fruits = 5,692,000

`Mr. X is taller than Mr. Y’ is correct. Ans. (c)

R = Ripe fruits 268. The students __________ the teacher on teachers’ day for twenty years of dedicated teaching.

U = Unripe fruits A = Apple

(a) facilitated

O = Oranges

(b) felicitated (c) fantasized

Given U = 15% of T:

(d) facilitated

(15/100) × 5,692,000 = 853,800

(GATE 2016, 1 Mark) R = T − U = 4,838,200 A(U) = 45% of U:

Solution: 

Felicitated is correct. Ans. (b)

(45/100) × 853,800 = 384,210 A(R) = (100 — 66)% of R:

269. After India’s cricket world cup victory in 1985, Shrotria who was playing both tennis and cricket till then, decided to concentrate only on cricket. And the rest is history.

(34/100) × 4,838,200 = 1,644,988 Therefore, A(U) + A(R) = 2,029,198 Ans. (a) 266. Michael lives 10 km away from where I live. Ahmed lives 5 km away and Susan lives 7 km away from

GATE_GA-Questions.indd 802

What does the underlined phrase mean in this context? (a) History will rest in peace. (b) Rest is recorded in history books.

8/22/2017 5:36:18 PM

SOLVED GATE PREVIOUS YEARS’ QUESTIONS     803

(c) Rest is well known. (d) Rest is archaic. (GATE 2016, 1 Mark) Solution: 

`Rest is well known’ is correct.

(a) The price of an apple is greater than an onion. (b) The price of an apple is more than onion. (c) The price of an apple is greater than that of an onion. (d) Apples are costlier than onions.

Ans. (c) (GATE 2016, 1 Mark) 270. Given (9 inches)1/2 = (0.25 yards)1/2, which one of the following statements is TRUE? (a) 3 inches = 0.5 yards (b) 9 inches = 1.5 yards (c) 9 inches = 0.25 yards (d) 81 inches = 0.0625 yards (GATE 2016, 1 Mark) Solution: 

(9 inches)1/2 = (0.25 yards)1/2 ⇒ 9 inches =

0.25 yards

Squaring on both sides, we get 9 inches = 0.25 yards

Solution:  The information given to us about both apples and onions is its price. We see that an apple costs more than an onion. The best way to frame the information with respect to grammar and usage in the given options is option (c). `More costlier’ is wrong grammatically because `costlier’ is the comparative degree of comparison and hence does not require `more’ before it. Since the prices are being compared, we cannot say the price of an apple is greater than an onion. Thus option (a) is also wrong. Option (b) is incorrect without any article before `onion’. Thus, option (c) that correctly compares the price of both the items is the correct answer. Ans. (c)

Ans. (c) 271. S, M, E and F are working in shifts in a team to finish a project. M works with twice the efficiency of others but for half as many days as E worked. S and M have 6 hour shifts in a day, whereas E and F have 12 hours shifts. What is the ratio of contribution of M to contribution of E in the project? (a) 1:1 (b) 1:2 (c) 1:4 (d) 2:1

273. The Buddha said, `Holding on to anger is like grasping a hot coal with the intent of throwing it at someone else; you are the one who gets burnt’. Select the word below which is closest in meaning to the word underlined above. (a) burning (b) igniting (c) clutching (d) flinging (GATE 2016, 1 Mark)

(GATE 2016, 1 Mark) Solution:  Ratio of contribution of M to the ­contribution of E in the project is given as d  6 × 2  × 2 1 Contribution of M = = Contribution of E [12 × d ] 2 d days have been used for M as it is 2 given that M works for half as many days as E works. An extra factor of 2 is used for M because M has twice the efficiency of others. Ans. (b) Note that

272. An apple costs Rs. 10. An onion costs Rs. 8. Select the most suitable sentence with respect to grammar and usage.

GATE_GA-Questions.indd 803

Solution:  `Burning’ or igniting’ means to give fire to something. `Clutching’ means to hold on to something. `Flinging’ means to hurl something forcefully. From the options, we see that the word closest in meaning to the word `grasping’ is clutching’. Ans. (c) 274. M has a son Q and a daughter R. He has no other children. E is the mother of P and daughter-in-law of M. How is P related to M? (a) P (b) P (c) P (d) P

is is is is

the the the the

son-in-law of M grandchild of M daughter-in law of M grandfather of M (GATE 2016, 1 Mark)

8/22/2017 5:36:19 PM

804     GENERAL APTITUDE

Solution:  Given information can be represented as

277. The chairman requested the aggrieved shareholders to ________ him.

Meaning



(a) bare with



(b) bore with



(c) bear with



(d) bare

M

(GATE 2016, 1 Mark) E

Q

Solution:  The chairman requested the aggrieved shareholders to bear with him. Ans. (c)

R

278. Identify the correct spelling out of the given options: Hence, we can say that, P is grandchild of M. Ans. (b)



(a) Managable



(b) Manageable



(c) Mangaeble



(d)  Managible

275. The number that least fits this set: (324, 441, 97 and 64) is __________.

(GATE 2016, 1 Mark) Solution:  Manageable has the correct spelling. Ans. (b)

(a) 324 (b) 441 (c) 97 (d) 64

279. Pick the odd one out in the following: 13, 23, 33, 43, 53 (GATE 2016, 1 Mark)



(a) 23





(b) 33

Solution:  Analyzing given numbers given, we get



(c) 43





(d) 53

2

324 = 18

(GATE 2016, 1 Mark)

2

441 = 21

Solution:  33 is not a prime number. 13, 23, 43, 53 are primes. Ans. (b)

64 = 82 But 97 is not square of any numbers, hence, least fits the given set.

280. R2D2 is a robot, R2D2 can repair aeroplanes. No other robot can repair aeroplanes.

Ans. (c) 276. It takes 10 s and 15 s, respectively, for two trains travelling at different constant speeds to completely pass a telegraph post. The length of the first train is 120 m and that of the second train is 150 m. The magnitude of the difference in the speeds of the two trains (in m/s) is __________.

Which of the following can be logically inferred from the above statements?

(a) 2.0 (b) 10.0 (c) 12.0 (d) 22.0



(GATE 2016, 1 Mark) (GATE 2016, 1 Mark)

Solution:  If a train crosses a pole, it has to travel distance equal to its own length. So, Speed of first train = 120/10 = 12 m/s Speed of second train = 150/15 = 10 m/s Therefore, Difference in the speed = 12 − 10 = 2 m/s Ans. (a)

GATE_GA-Questions.indd 804

(a) R2D2 is a robot which can only repair aeroplanes. (b) R2D2 is the only robot which can repair aeroplanes. (c) R2D2 is a robot which can repair only aeroplanes. (d) Only R2D2 is a robot.

Solution:  Since no other robot can repair airplanes, R2D2 is the only robot that can repair aeroplanes. Ans. (b) 281. If ½| 9y - 6½ | = 3, then y3 - 4y/3 is ________. 1 (a) 0 (b) + 3 1 (c) − (d) Undefined 3 (GATE 2016, 1 Mark)

8/22/2017 5:36:20 PM

SOLVED GATE PREVIOUS YEARS’ QUESTIONS     805

Solution:  9y - 6 = ±3

Which one of the options given below is NOT logically possible, based on the given fact?



      ⇒ 3y - 2 = ±1



      ⇒ 9y2 + 4 - 12y = -1       ⇒    9y2 - 12y = -3 12 −3       ⇒ y 2 − y = 9 9 4 −1       ⇒ y 2 − y = 3 3

(a) If (iii), then (iv). (c) If (i), then (ii).

Ans. (c)

282. The volume of a sphere of diameter 1 unit is     than the volume of a cube of side 1 unit. (a) least (c) lesser

(b) less (d) low (GATE 2016, 1 Mark)

Solution: Volume of sphere =

π d3 π 3 π = (1) = = 0.5236 6 6 6

3 3 Volume of cube = a = (1) = 1

Clearly, volume of sphere is less than volume of cube. Ans. (b) 283. The unruly crowd demanded that the accused be     without trial. (a) hanged (c) hankering

(b) hanging (d) hung (GATE 2016, 1 Mark)

(b) If (i), then (iii). (d) If (ii), then (iv). (GATE 2016, 1 Mark)

Solution:  (a) If (iii), then (iv)    ⇒ If the field is wet, then it did not rain. (b) If (i), then (iii)    ⇒ If it rains, then the field is wet. (c) If (i), then (ii)    ⇒ If it rains, then the field is not wet. (d) If (ii), then (iv).    ⇒ If the field is not wet, then it did not rain. According to the given fact, if it rains, then the field is wet. Opposite of this may not be true. Therefore, option (c) is not logically possible. Ans. (c) 286. A window is made up of a square portion and an equilateral triangle portion above it. The base of the triangular portion coincides with the upper side of the square. If the perimeter of the window is 6 m, the area of the window in m2 is     . (a) 1.43 (c) 2.68

(b) 2.06 (d) 2.88 (GATE 2016, 1 Mark)

Solution:

Solution:  The correct entry for the blank is hanged. Ans. (a)

x

x x

284. Choose the statement(s) where the underlined word is used correctly: (i) A prone is a dried plum. (ii) He was lying prone on the floor. (iii) People who eat a lot of fat are prone to heart disease. (a) (i) and (iii) only (c) (i) and (ii) only

x

x

x

Perimeter of the window = 6 m ⇒ (x + x + x) + (x + x) = 6 m

(b) (iii) only (d) (ii) and (iii) only (GATE 2016, 1 Mark)

⇒ 5x = 6 ⇒ x = 6/5 Now, area of the window is,

Solution:  The word `prone’ has been correctly used only in statements (ii) and (iii). Ans. (d) 285. Fact: If it rains, then the field is wet.

2 = x +

2  3 2 3  6  3 x = x2  1 +  =    1 +  4 4  5  4  

= 2.06 m2

Read the following statements: (i) It rains. (iii) The field is wet.

GATE_GA-Questions.indd 805

(ii) The field is not wet. (iv) It did not rain.

Ans. (b) 287. If I were you, I     expensive.

that laptop. It’s much too

8/22/2017 5:36:23 PM

806     GENERAL APTITUDE

(a) won’t buy (c) wouldn’t buy

(b) shan’t buy (d) would buy (GATE 2016, 1 Mark)

Solution:  First sentence is influencing the second form of helping verb in the blank. Hence `would not buy’ is correct Ans. (c)

What does the underlined phrasal verb mean? (b) appreciated (d) returned (GATE 2016, 1 Mark)

Solution:  `Turned a deaf ear’ is a phrasal verb which means disregard something. Hence, `ignored’ is correct. Ans. (a) 289. Choose the most appropriate set of words from the options given below to complete the following sentence.         is a will,     is a way. (a) Wear, there, their (b) Were, their, there (c) Where, there, there (d) Where, their, their (GATE 2016, 1 Mark) Solution:  Where there is a will, there is a way. Ans. (c) 290. (x% of y) + (y% of x) is equivalent to     . (a) 2% of xy (c) xy% of 100

(10b + a) - (10a + b) = 54

−9a + 9b = 54 a - b = −6



288. He turned a deaf ear to my request.

(a) ignored (c) twisted

Then, a + b = 12 (1) Reversing the digits means the new number is ba. This number can be represented as (10b + a). Now ba is greater than ab by 54. Therefore,

(b) 2% of (xy/100) (d) 100% of xy

(2)

Adding Eqs. (1) and (2), we get a + b = 12

⇒ a - b = −6



⇒ 2a = 6



⇒ a = 3 and b = 9

Hence, the number is 39. Ans. (a) 292. A shaving set company sells four different types of razors, Elegance, Smooth, Soft and Executive. Elegance sells at Rs. 48, Smooth at Rs. 63, Soft at Rs. 78 and Executive at Rs. 173 per piece. The table below shows the numbers of each razor sold in each quarter of a year. Quarter/ Product Q1 Q2 Q3 Q4

Elegance Smooth Soft Executive 27300 25222 28976 21012

20009 19392 22429 18229

17602 18445 19544 16595

9999 8942 10234 10109

(GATE 2016, 1 Mark) x y Solution:  x% of y + y% of x = ×y + ×x 100 100 =

xy yx + 100 100

=

2xy 2 = × xy = 2% of xy 100 100

Which product contributes the greatest fraction to the revenue of the company in that year? (a) Elegance (b) Executive (c) Smooth (d) Soft (GATE 2016, 2 Marks)

Ans. (a)

Solution: (i) Elegance = ∑Q × 48 Rs.

291. The sum of the digits of a two digit number is 12. If the new number formed by reversing the digits is greater than the original number by 54, find the original number. (a) 39 (c) 66

(b) 57 (d) 93 (GATE 2016, 1 Mark)

Solution:  Let the number be ab. So it can be represented as (10a + b).

GATE_GA-Questions.indd 806



 = 1,02,510 × 48



 = Rs. 49,20,480

(ii) Smooth = ∑Q × 63 Rs.

= 80,059 × 63



= Rs. 50,43,717

(iii) Soft = ∑Q × 78 Rs.

= 72,186 × 78 = Rs. 56,30,508

8/22/2017 5:36:24 PM

SOLVED GATE PREVIOUS YEARS’ QUESTIONS     807

(iv) Executive = ∑Q × 173 Rs.

= 39,284 × 173



= Rs. 67,96,132

Hence, maximum Executive.

contribution

is

made

by

(b) R always beats S, R always loses to Q and P always beats Q means P is better than Q and R but S sometimes loses to P hence S is not the absolute worst player. Hence, statement (ii) also is not logical. Ans. (d)

Ans. (b) 293. Indian currency notes show the denomination indicated in at least seventeen languages. If this is not an indication of the nation’s diversity, nothing else is. Which of the following can be logically inferred from the above sentences? (a) India is a country of exactly seventeen languages. (b) Linguistic pluralism is the only indicator of a nation’s diversity. (c) Indian currency notes have sufficient space for all the Indian languages. (d) Linguistic pluralism is strong evidence of India’s diversity.

295. If f(x) = 2x7 + 3x − 5, which of the following is a factor of f(x)? (a) (x3 + 8) (b) (x − 1) (c) (2x − 5) (d) (x + 1) (GATE 2016, 2 Marks) Solution:  f(x) = 2x7 + 3x − 5 Put x = 1 in the above equation, we get f(1) = 2 × 17 + 3 × 1 − 5 = 0 Hence, (x − 1) is a factor of f(x). Ans. (b)

(GATE 2016, 2 Marks) Solution:  Linguistic pluralism means existence of different languages, and this is a strong evidence of nation’s diversity. Hence, the best inference can be made is explained in last line. Ans. (d) 294. Consider the following statements relating to the level of poker play of four players P, Q, R and S. I. P always beats Q II. R always beats S III. S loses to P only sometimes IV. R always loses to Q Which of the following can be logically inferred from the above statements? (i) P is likely to beat all the three other players

296. In a process, the number of cycles to failure decreases exponentially with an increase in load. At a load of 80 units, it takes 100 cycles for failure. When the load is halved, it takes 10000 cycles for failure. The load for which the failure will happen in 5000 cycles is __________. (a) 40.00 (b) 46.02 (c) 60.01 (d) 92.02 (GATE 2016, 2 Marks) Solution:  Let the number of cycles = N and load = P. Exponential decrease can be represented as N = C1e − C2P

(1)

(ii) S is the absolute worst player in the set where C1 and C2 are arbitrary constants. (a) (i) only

As per the given conditions:

(b) (ii) only

100 = C1e −80 C2

(2)

10000 = C1e −40 C2

(3)

(c) (i) and (ii) (d) Neither (i) nor (ii) (GATE 2016, 2 Marks) Solution:  (a) S loses to P only sometimes means S beats P most of the times. Hence, statement (i) is not logical.

GATE_GA-Questions.indd 807

Using Eqs. (2) and (3) to find the values of the constant, we have C2 = 0.1151, C1 = 106

8/22/2017 5:36:26 PM

808     GENERAL APTITUDE

Substituting the values of C1 and C2 in Eq. (1), we have N = (106 )e −0.1151P

(4)

(a) (i) only (b) (ii) only (c) Both (i) and (ii) (d) Neither (i) nor (ii)

Now, for N = 5000 cycles find the value of P, using Eq. (4). So

P  = 46.02 Ans. (b)

297. Among 150 faculty members in an institute. 55 are connected with each other through Facebook and 85 are connected through WhatsApp. 30 faculty members do not have Facebook or WhatsApp accounts. The member of faculty members connected only through Facebook accounts is ________. (a) 35 (b) 45 (c) 65 (d) 90

Solution:  The author calls the internet an ­“unintended consequence”. Hence, (ii) cannot be true. The author is questioning if internet and mobile communications are good or not. Therefore, (i) is not an inference. Ans. (d) 299. All hill-stations have a lake. Ooty has two lakes. Which of the statement(s) below is/are logically valid and can be inferred from the above sentences? (i) Ooty is not a hill-station. (ii) No hill-station can have more than one lake.

(GATE 2016, 2 Marks) Solution:  Let faculties on Facebook = v = 55 Let faculties on WhatsApp = w = 85 Let faculties on both = x Let faculties on none = y = 30 Therefore, v + w − x + y = 150 ⇒55 + 85 − x + 30 = 150 ⇒ x = 20 Out of 55 on Facebook, 20 are also on WhatsApp. That is, 55 − 20 = 35 are only on Facebook. Ans. (a) 298. Computers were invented for performing only highend useful computations. However, it is no understatement that they have taken over our world today. The internet, for example, is ubiquitous. Many believe that the internet itself is an unintended consequence of the original invention. With the advent of mobile computing on our phones, a whole new dimension is now enabled. One is left wondering if all these developments are good or, more importantly, required. Which of the statement(s) below is/are logically valid and can be inferred from the above paragraph? (i) The author believes that computers are not good for us. (ii) Mobile computers and the internet are both intended inventions.

GATE_GA-Questions.indd 808

(GATE 2016, 2 Marks)

(a) (i) only (b) (ii) only (c) Both (i) and (ii) (d) Neither (i) nor (ii) (GATE 2016, 2 Marks) Solution:  (i) can be correct when “All hill-­stations should have at most one lake. Ooty has 2 lakes. Then (i) can be inferred. Hence, false. Option (ii) also has similar explanation. Ans. (d) 300. In a 2 × 4 rectangle grid shown below, each cell is a rectangle. How many rectangles can be observed in the grid?

(a) 21 (c) 27 (c) 30 (d) 36 (GATE 2016, 2 Marks) Solution:  In a 4 × 2 grid, we have 5 × 3 grid points. Any rectangle will need 2 points on a column and 2 points on a row. Therefore,     5! 3!  ×  = 30 ( 5 − 2 )! 2 ! ( 3 2 )! 2 ! −     Ans. (c)

8/22/2017 5:36:27 PM

SOLVED GATE PREVIOUS YEARS’ QUESTIONS     809

Therefore, percentage probability of person moving though a TB prone zone remaining infected but not showing symptoms

301. 2.5 f(x) 2

= P (A) ⋅ P (B) = (50/100) × (7 0/100) = 35%

1.5 1

Ans. (c)

0.5 −4

−3

−2

0 −1 −0.5 0

1

2

3

x 4

−1 −1.5 −2

303. In a world filled with uncertainty, he was glad to have many good friends. He had always assisted them in times of need and was confident that they would reciprocate. However, the events of the last week proved him wrong. Which of the following inference(s) is/are logically valid and can be inferred from the above passage?

−2.5

Choose the correct expression for f(x) given in the above graph.

(i) His friends were always asking him to help them.

(a) f(x) = 1 − |x − 1|

(ii) He felt that when in need of help, his friends would let him down.

(b) f(x) = 1 + |x − 1| (c) f(x) = 2 − |x − 1|

(iii) He was sure that his friends would help him when in need.

(d) f(x) = 2 + |x − 1|

(iv) His friends did not help him last week. (GATE 2016, 2 Marks)

Solution:  Given graph has two parts. For x = − 3 to x = 1, equation of line is

(a) (i) and (ii)

(b) (iii) and (iv)

(c) (iii) only

(d) (iv) only (GATE 2016, 2 Marks)

f(x) = x + 1 For x = 1 to x = 3, equation of line is f(x) = − x + 3 Hence, option (c) fits these cases. Ans. (c) 302. A person moving through a tuberculosis prone zone has a 50% probability of becoming infected. However, only 30% of infected people develop the disease. What percentage of people moving through a tuberculosis prone zone remains infected but does not show symptoms of disease? (a) 15

(b) 33

(c) 35

(d) 37

Solution:  The paragraph states that the subject was very confident about his good friends helping him in his times of need because he had always helped them before in their time. Thus, inference (iii) follows. Since the events of the last week proved him wrong, this means that his confidence was broken and his friends had not helped him. Thus, inference (iv) also follows. Ans. (b) 304. Leela is older than her cousin Pavithra. Pavithra’s brother Shiva is older than Leela. When Pavithra and Shiva are visiting Leela, all three like to play chess. Pavithra wins more often than Leela does. Which one of the following statements must be TRUE based on the above?

(GATE 2016, 2 Marks) (a) When Shiva plays chess with Leela and Pavithra, he often loses. Solution:  Percentage probability of being infected = P(A) = 50% Percentage probability of infected person developing disease is having system = P(B) = 30%

(b) Leela is the oldest of the three. (c) Shiva is a better chess player than Pavithra. (d) Pavithra is the youngest of the three.

Therefore, percentage probability of infected person not showing symptoms = P(B ) = 70%

GATE_GA-Questions.indd 809

(GATE 2016, 2 Marks)

8/22/2017 5:36:28 PM

810     GENERAL APTITUDE

Solution:  According to given information, the points we got are as follows: (i) Shiva is the brother of Pavithra. (ii) Shiva and Pavithra are cousins of Leela. (iii) According to their ages, Shiva > Leela > Pavithra. (iv) They all like to play chess. (v) Pavithra wins more often than Leela but ­information about winning cases of Shiva is not given. So from the given option statements, which one is clearly true is that Pavithra is the youngest of all. Ans. (d)

Solution:  After 7 days from start of project, Q took sick leave on first 2 days. Therefore, man hours by Q = 5 × 12 Therefore, work done by Q = 5 × 12 × [1/(25 × 12)] = 15 man hours by R = 7 × 18 Therefore, work done by R = [1/(50 × 12)] × 7 × 18 = 21,100 Therefore, ratio of work done by Q to work done by R = (1/5):(21/100) = 100/(5 × 21) = 20:21 Ans. (c) 307. The Venn diagram shows the preference of the student population for leisure activities.

1 1 1 305. If 𝑞−a =  and r − b = and s− c = , the value of abc r s q is __________. (a) (rqs)−1

(b) 0

(c) 1

(d) r + q + s

12 19

13 7

17

44

(GATE 2016, 2 Marks) Solution:  q−a = 1/r; r−b = 1/s and s−c = 1/q Therefore,

15

qa = r; rb = s and sc = q From the data given, the number of students who like to read books or play sports is __________.

Therefore, a log q = log r

(1) (a) 44

and b log r = log s

(2)

(b) 51 (c) 79

and c log s = log q

(3)

(d) 108 (GATE 2016, 2 Marks)

Multiplying Eqs. (1)—(3), we get abc(log q)(log r)(log s) = (log r)(log s)(log q) Therefore,

Solution:  From the given Venn diagram: Total students who play sports = 15 + 44 + 17 + 7 = 83 Total students who read books = 13 + 44 + 7 + 12 = 76

abc = 1 Ans. (c)

Total students who either play sports or read books = 83 + 76 − (44 + 7) = 108

306. P, Q, R and S are working on a project. Q can finish the task in 25 days, working alone for 12 hours a day. R can finish the task in 50 days, working alone for 12 hours per day. Q worked 12 hours a day but took sick leave in the beginning for two days. R worked 18 hours a day on all days. What is the ratio of work done by Q and R after 7 days from the start of the project?

Ans. (d)

(a) 10:11

(b) 11:10

(c) 20:21

(d) 21:20 (GATE 2016, 2 Marks)

GATE_GA-Questions.indd 810

308. Social science disciplines were in existence in an amorphous form until the colonial period when they were institutionalized. In varying degrees, they were intended to further the colonial interest. In the time of globalization and the economic rise of postcolonial countries like India, conventional ways of knowledge production have become obsolete. Which of the following can be logically inferred from the above statements?

8/22/2017 5:36:29 PM

SOLVED GATE PREVIOUS YEARS’ QUESTIONS     811

(i) Social science obsolete.

disciplines

have

become

(ii) Social science discipline had a pre-colonial origin. (iii) Social science disciplines always promote colonialism. (iv) Social science must maintain disciplinary boundaries.

310. M and N start from the same location. M travels 10 km East and then 10 km North-East. N travels 5 km South and then 4 km South-East. What is the shortest distance (in km) between M and N at the end of their travel? (a) 18.60 (b) 22.50 (c) 20.61 (d) 25.00

(a) (ii) only

(GATE 2016, 2 Marks)

(b) (i) and (ii) only

Solution: 

(c) (ii) and (iv) only

N

(d) (iii) and (iv) only (GATE 2016, 2 Marks)

M

309. Two and a quarter hours back, when seen in a mirror, the reflection of a wall clock without number markings seemed to show 1:30. What is the actual current time shown by the clock?

10

Solution:  On the basis of given passage, it can be concluded that statement (ii) `social science disciplines had a pre-colonial origin’ is correct. Therefore, option (a) is only correct. Ans. (a)

km

(x1,y1)

E

W 10 km 5 km 4 km (x2,y2) N S Position of M ≡ (17.07, 7.07 ) Position of N ≡ (2.03, − 7.03)

(a) 8:15 The shortest distance between their final positions is (b) 11:15 (x1 − x2 )2 + (y1 + y2 )2 = 20.61

(c) 12:15 (d) 12:45

Ans. (c) (GATE 2016, 2 Marks) Solution:  Two and a quarter hours back, the mirror image of the wall clock was as shown below:

Therefore, actual time was as shown below:

311. A wire of length 340 mm is to be cut into two parts. One of the parts is to be made into a square and the other into a rectangle where sides are in the ratio of 1:2. What is the length of the side of the square (in mm) such that the combined area of the square and the rectangle is a MINIMUM? (a) 30 (b) 40 (c) 120 (d) 180 (GATE 2016, 2 Marks) Solution:

The actual time, two and a quarter hours back, was 10:30.

x/4 x/4

Actual current time shown by the clock = 10:30 + 2 Hours 15 minutes b  = 12:45 Ans. (d)

GATE_GA-Questions.indd 811

8/22/2017 5:36:31 PM

812     GENERAL APTITUDE

Let the two parts of wire are of x min and (340 − x) mm, respectively. x mm 4 Perimeter of the rectangle = 2(a + b) = 340 − x Side of the square =

6b = 340 − x 340 x ⇒b= − 6 6

340 x − 3 3

x  340 x  + 2 −   6 16 6

Ans. (d) 313. The overwhelming number of people infected with rabies in India has been flagged by the World Health Organization as a source of concern. It is estimated that inoculating 70% of pets and stray dogs against rabies can lead to a significant reduction in the number of people infected with rabies.

2

For A to be minimum, dA =0 dx 2x 1  340 x  ⇒ + 4 − x− =0  6 16 6 6 x x 340 ⇒ + − =0 8 9 9 x ⇒ = 40 4

Since at

Which of the following can be logically inferred from the above sentences? (a) The number of people in India infected with rabies is high. (b) The number of people in other parts of the world who are infected with rabies is low. (c) Rabies can be eradicated in India by vaccinating 70% of stray dogs.

d2A > 0 for any value of x, A has a minima dx2

(d) Stray dogs are the main source of rabies worldwide. (GATE 2016, 2 Marks)

x = 40 . 4 Ans. (b)

312. The velocity V of a vehicle along a straight line is measured in m/s and plotted as shown with respect to time in seconds. At the end of the 7 seconds, how much will the odometer reading increase by (in m)? V (m/s)

Solution:  The information given in the passage does not help us conclude whether stray dogs are the main source of rabies globally. Thus option (d) cannot be inferred. It is said that 70% of both pets and stray dogs need to be inoculated to reduce the number of people infected with rabies. Thus option (c) also cannot be inferred. No information about the number of people infected in other parts of the world has been given. Thus option (b) is also eliminated.

2

1

1 −1

(d) 5

 1  1  1  1 =   ×1 × 1 + 1 × 1 + 1 × 1 +   × 1 × 1 +   × 1 × 2 +   × 2 × 1  2  2  2  2 1 1 = +1+1+ +1+1= 5 2 2

Combined area of square and rectangle A=

(c) 4

Solution:  Odometer reading = Overall area

Therefore,

2

(b) 3

(GATE 2016, 2 Marks)

But it is given that a = 2b. Therefore,

a = 2b =

(a) 0

2

3

4

5

6

7 Time (s)

The passage states that WHO is concerned because of the huge number of rabies patients in India. This means that the number is quite high otherwise the WHO would not have been concerned. Thus option (a) is the correct inference. Ans. (a)

GATE_GA-Questions.indd 812

8/22/2017 5:36:34 PM

SOLVED GATE PREVIOUS YEARS’ QUESTIONS     813

314. A flat is shared by four first year undergraduate students. They agreed to allow the oldest of them to enjoy some extra space in the flat. Manu is two months older than Sravan, who is three months younger than Trideep. Pavan is one month older than Sravan. Who should occupy the extra space in the flat?

Solving Eqs. (1) and (2), we get the points of intersection as B(4, 1). Now find the coordinates of the point of intersection of each line with the coordinate axes. Therefore, the required area A = A1 + A2   5   4 +  2   × 1   [Area of ∆ ABC + Area of trapezium BCOE ] ⇒ A = 15.25 Ans. (b)  1  1 ⇒ A=   ×6 × 4+    2  2

(a) Manu (b) Sravan (c) Trideep (d) Pavan (GATE 2016, 2 Marks) Solution:  Manu is two months older than Sravan, who is three months younger than Trideep, that means

Pavan is one month older than Sravan So,

316. A straight line is fit to a data set ln (x, y). This line intercepts the abscissa at ln x = 0.1 and has a slope of −0.02. What is the value of y at x = 5 from the fit? (a) −0.030 (b) −0.014 (c) 0.014 (d) 0.030 (GATE 2016, 2 Marks) Solution: 

That means Trideep is the oldest among then and hence occupy the extra space in the flat. Ans. (c)

y

315. Find the area bounded by the lines 3x + 2y = 14, 2x − 3y = 5 in the first quadrant. ln5 ln or z

(a) 14.95 (b) 15.25 (c) 15.70 (d) 20.35

0.1 y (GATE 2016, 2 Marks)

Solution: 

7

Y

Since the figure will be straight line having intercept 0.1 at abscissa and slope = − 0.2

A

Hence, equation of line

C O

F

2

D 2x—3y=5

 1 z =   y +c  m

(4,1) B

1

5 3

5 2

14 3

X 3x+2y=14

 1  ⇒ z=− y + 0.1  0.02   1  ⇒ z = ln x = −  + 0.1  0.02  Now at x = 5, z = ln 5 = −(1/0.02) + 0.1

GATE_GA-Questions.indd 813

3x + 2y = 14

(1)

2x − 3y = 5

(2)

⇒ y = −0.030 Ans. (a)

8/22/2017 5:36:37 PM

814     GENERAL APTITUDE

317. The following graph represents the installed capacity for cement production (in tonnes) and the actual production (in tonnes) of nine cement plants of a cement company. Capacity utilization of a plant is defined as ratio of actual production of cement to installed capacity. A plant with installed capacity of at least 200 tonnes is called a large plant and a plant with lesser capacity is called a small plant. The difference between total production of large plants and small plants, in tonnes is ________.

Capacity/production (tonnes)

300

Installed capacity

250 230

220 200

200 190 190 180 160 150 160 160

(c) Mechanical engineering is a profession well suited for women with masters or higher degrees in mechanical engineering.



(d) The number of women pursuing higher degrees in mechanical engineering is small. (GATE 2016, 2 Marks) Solution:  Option (a) can be logically inferred from the given paragraph. Ans. (a)

319. Sourya committee had proposed the establishment of Sourya Institutes of Technology (SITs) in line with Indian Institutes of Technology (IITs) to cater to the technological and industrial needs of a developing country.

Actual production

250



200 190

Which of the following can be logically inferred from the above sentence?

150 140 120

150 120

Based on the proposal,

100 100

50



0 1

2

3

4 5 6 Plant number

7

8

9

(GATE 2016, 2 Marks) Solution:  Plants 1, 4, 8 and 9 are large plants. Total actual production of large plants = 160 + 190 + 230 + 190 = 770 tonnes = A

(i) In the initial years, SIT students will get degrees from IIT. (ii) SITs will have a distinct national objective. (iii) SIT like institutions can only be established in consultation with IIT. (iv) SITs will serve technological needs of a developing country.



(a) (iii) and (iv) only



(b) (i) and (iv) only



(c) (ii) and (iv) only



(d) (ii) and (iii) only

Total actual production of small plants = 150 + 160 + 120 + 100 + 120 = 650 tonnes = B

(GATE 2016, 2 Marks) Solution:  Option (c) can be logically inferred from the given sentence. Ans. (c)

Therefore, A - B = 120 tonnes

Ans. 120

318. A poll of students appearing for masters in engineering indicated that 60% of the students believed that mechanical engineering is a profession unsuitable for women. A research study on women with masters or higher degrees in mechanical engineering found that 99% of such women were successful in their professions.

320. Shaquille O’ Neal is a 60% career free throw shooter, meaning that he successfully makes 60 free throws out of 100 attempts on average. What is the probability that he will successfully make exactly 6 free throws in 10 attempts?

(a) 0.2508



(b) 02816



(c) 0.2934



(d) 0.6000 (GATE 2016, 2 Marks)

Which of the following can be logically inferred from the above paragraph?



(a) Many students have misconceptions regarding various engineering disciplines. (b) Men with advanced degrees in mechanical engineering believe women are well suited to be mechanical engineers.

GATE_GA-Questions.indd 814

Solution:  Probability of successful free throw = 0.6 Probability of unsuccessful free throw = 0.4 Any 6 throws out of 10 may be successful, so choose 6 out of 10 as 10C6. Therefore, P=

10

C6 (0.6)6×(0.4)4 = 0.2508 Ans. (a)

8/22/2017 5:36:37 PM

SOLVED GATE PREVIOUS YEARS’ QUESTIONS     815

321. The numeral in the units position of 211870 + 146127 × 3424 is ________.

Distribution of marks of group Q is shown below:

(GATE 2016, 2 Marks) Solution:  Considering the given expression 211870 + 146127 × 3424

5

For unit place, we will find the unit place of each of the number (i.e. 211870, 146127 and 3424) given in the above expression. Unit place of 211870 = (1)870 = 1 Unit place of 146127 = (6)127 = 6 Unit place of 3424 = (3)424 = ((3)5)84 × (3)4

= (3)84 × (3)4



= ((3)5)16 × (3)4 × (3)4



= (3)16 × (3)8



= ((3)5)3 × (3)9 = (3)12



= ((3)5)2 × (3)2



= (3)4 = 1

85

Since σ Q (= 5) < σ P (= 25) , most of the students of group Q scored marks in a narrower range than the students of group P. Ans. (c) 323. A smart city integrates all modes of transport, uses clean energy and promotes sustainable use of resources. It also uses technology to ensure safety and security of the city, something which critics argue, will lead to a surveillance state.

Hence, overall the unit place of the expression is 1+6×1=1+6=7 Ans. 7 322. Students taking an exam are divided into two groups, P and Q such that each group has the same number of students. The performance of each of the students in a test was evaluated out of 200 marks. It was observed that the mean of group P was 105, while that of group Q was 85. The standard deviation of group P was 25, while that of group Q was 5. Assuming that the marks were distributed on a normal distribution, which of the following statements will have the highest probability of being TRUE? (a) No student in group Q scored less marks than any student in group P. (b) No student in group P scored less marks than any student in group Q. (c) Most students of group Q scored marks in a narrower range than students in group P. (d) The median of the marks of group P is 100. (GATE 2016, 2 Marks)

Which of the following can be logically inferred from the above paragraph? (i) All smart cities encourage the formation of surveillance states. (ii) Surveillance is an integral part of a smart city. (iii) Sustainability and surveillance go hand in hand in a smart city. (iv) There is a perception that smart cities promote surveillance. (a) (i) and (iv) only (b) (ii) and (iii) only (c) (iv) only (d) (i) only (GATE 2016, 2 Marks) Solution:  Sustainability and surveillance go hand in hand in a smart city. Ans. (c) 324. Find the missing sequence in the letter series. B, FH, LNP,      . (a) SUWY (c) TVXZ

(b) TUVW (d) TWXZ (GATE 2016, 2 Marks)

Solution: B,

Solution:  Distribution of marks of group P is shown below:

↓ 2

FH , ↓ 68

L N P ↓ 12 14 16

The next sequence must be 20 22 24 26 ↓ ↓ ↓ ↓ T V X Z

25

105

GATE_GA-Questions.indd 815

Ans. (c)

8/22/2017 5:36:39 PM

1 0.8 0.6 0.4 0.2 0

816     GENERAL APTITUDE

(c)

325. The binary operation is defined as a b = ab + (a + b), where a and b are any two real numbers. The value of the identity element of this operation, defined as the number x such that a x = a, for any a, is     .

−2



2

−2



2

1.2 1 0.8 0.6 0.4

(a) 0  (b) 1  (c) 2  (d) 10

0.2

(GATE 2016, 2 Marks) x = ax + (a + x) = a (given) ⇒ x(a + 1) + a = a ⇒ x(a + 1) = 0 This product has to be zero for any a. It is possible only with x = 0. Ans. (a) Solution:  a

0

(d)

(GATE 2016, 2 Marks) Solution:  Here option (c) is correct since the given function is a periodic function with period p and a maximum value of 1. Ans. (c)

326. Which of the following curves represents the function y = ln (|e[|sin(|x|)|]|) for | x | < 2p? Here, x represents the abscissa and y represents the ordinate. 1.5 1.5 11

327. Two finance companies, P and Q, declared fixed annual rates of interest on the amounts invested with them. The rates of interest offered by these companies may differ from year to year. Year-wise annual rates of interest offered by these companies are shown by the line graph provided below.

0.5 0.5

Q

P

00 −0.5 −0.5

7

−1 −1 −1.5 −1.5

(a) (a)

9 8

9.5

10

8

8

9 7.5

6.5 −2 −2

−−

8 6.5

6 4

22

1.5 1.5 2000

11 0.5 0.5

−0.5 −0.5 −1 −1

(b) (b)

2003

2004

−2 −2

−−

22

2006

(GATE 2016, 2 Marks)

1.2 1.2 11

Solution:  Amounts invested:

0.8 0.8 0.6 0.6

By Q = 9x at 4%

00

2005

(a) 2:3   (b) 3:4   (c) 6:7   (d) 4:3

By P = 8x at 6%

0.4 0.4 0.2 0.2

(c) (c)

2002

If the amounts invested in the companies, P and Q, in 2006 are in the ratio 8:9, then the amounts received after one year as interests from companies P and Q would be in the ratio:

00

−1.5 −1.5

2001

−2 −2

−−

22

−2 −2

−−

22

1.2 1.2 11

Interest received by P after 1 year =

6 × 8x = 0.48x 100

Interest received by Q after 1 year =

4 × 9x = 0.36x 100

0.8 0.8 0.6 0.6 0.4 0.4 0.2 0.2

(d) (d)

00

GATE_GA-Questions.indd 816

8/22/2017 5:36:41 PM

SOLVED GATE PREVIOUS YEARS’ QUESTIONS     817

Therefore, Ratio =

330. A square pyramid has a base perimeter x, and the slant height is half of the perimeter. What is the lateral surface area of the pyramid?

0.48x 4 = 0.36x 3 Ans. (d)

328. Today, we consider Ashoka as a great ruler because of the copious evidence he left behind in the form of stone carved edicts. Historians tend to correlate greatness of a king at his time with the availability of evidence today. Which of the following can be logically inferred from the above sentences? (a) Emperors who do not leave significant sculpted evidence are completely forgotten. (b) Ashoka produced stone carved edicts to ensure that later historians will respect him. (c) Statues of kings are a reminder of their greatness. (d) A king’s greatness, as we know him today, is interpreted by historians. (GATE 2016, 2 Marks) Solution:  Historians tend to correlate greatness of king with the available evidence today. This relates to last option (d). Ans. (d) 329. Fact 1: Humans are mammals. Fact 2: Some humans are engineers. Fact 3: Engineers build houses.

(a) x2 (c) 0.50x2

(b) 0.75x2 (d) 0.25x2 (GATE 2016, 2 Marks)

Solution:

Slant height (b)

a a

Base perimeter of square pyramid, x = 4a a=

x 4

b=

x 2

Slant height,

Therefore, Lateral surface area = 4 × Single face area

If the above statements are facts, which of the following can be logically inferred?

1 x x   = 4 ×  × ×  2 4 2

I. All mammals build houses. II. Engineers are mammals. III. Some humans are not engineers.

  = 4 ×

(a) II only (c) I, II and III



x2 x2 = 16 4

= 0.25 x2

(b) III only (d) I only (GATE 2016, 2 Marks)

Solution:  Draw the Venn diagram, assuming Humans = X, Mammals = Y, Engineers = Z. Y X

Ans. (d) 331. Ananth takes 6 hours and Bharath takes 4 hours to read a book. Both started reading copies of the book at the same time. After how many hours is the number of pages to be read by Ananth, twice that to be read by Bharath? Assume Ananth and Bharath read all the pages with constant pace.

Z

(a) 1  (b) 2  (c) 3  (d) 4 (GATE 2016, 2 Marks) Solution:  Total time taken by Ananth, tA = 6 hours Total time taken by Bharath, tB = 4 hours In 1 hour,

Hence, Statement I is wrong. Statement II is wrong. Statement III is right. Ans. (b)

GATE_GA-Questions.indd 817

Ananth reads

1 th of the book. 6

8/22/2017 5:36:44 PM

818     GENERAL APTITUDE

Bharath reads

Solution: 

1 th of the book. 4

Murali

After x hours, Number of pages to be read by Ananth = 1 −

x 6

Number of pages to be read by Bharath = 1 −

x 4

Arul

Rahul

Now, 1−

x  x = 1 −  × 2 6  4

Srinivas 2x x − = 2 −1 ⇒ 4 6

From the above figure, we can conclude that Srinivas and Murali are seated opposite each other.

2x =1 6 ⇒ x = 3 hours

Ans. (c)



335. Find the smallest number y such that the y × 162 is a perfect cube. Ans. (c)

332. After Rajendra Chola returned from his voyage to Indonesia, he ________ to visit the temple in Thanjavur. (a) was wishing (c) wished

(b) is wishing (d) had wished (GATE 2017, 1 Mark)

Solution:  (d) After Rajendra Chola returned from his voyage to Indonesia, he wished to visit the temple in Thanjavur. Ans. (c)

(a) 24

(b) 27

Solution:  y × 162 should be a perfect cube. Now, 162 = 2 × 81 = 2 × 34 For this to be a cube, we additionally need 22 and 32, i.e. 4 × 9 = 36 Ans. (d) 336. The probability that a k-digit number does NOT contain the digits 0, 5 or 9 is (a) 0.3k

(b) beside money (d) besides money (GATE 2017, 1 Mark)

(d) 36

(GATE 2017, 1 Mark)

(b) 0.6k

333. Research in the workplace reveal that people work for many reasons _________. (a) money beside (c) money besides

(c) 32

(c) 0.7k

(d) 0.9k

(GATE 2017, 1 Mark) Solution:  Let us first find out the total number of ways. The first place from the left can be filled in 9 ways. The second place can be filled in 10 ways.

Solution:  Research in the workplace reveal that people work for many reasons besides money.

The third place can be filled in 10 ways and so on till the unit’s place.

Ans. (d) 334. Rahul, Murali, Srinivas and Arul are seated around a square table. Rahul is sitting to the left of Murali. Srinivas is sitting to the right of Arul. Which of the following pairs are seated opposite each other? (a) Rahul and Murali (b) Srinivas and Arul (c) Srinivas and Murali (d) Srinivas and Rahul (GATE 2017, 1 Mark)

GATE_GA-Questions.indd 818

The total number of ways = 9 × 10 × 10 × 10 × 10 … (k - 1) times = 9 × 10k - 1 The favourable number of ways = 7 × 7 × 7 × 7 … k times Therefore, Required probability = 7k/(9 × 10k - 1)

8/22/2017 5:36:46 PM

SOLVED GATE PREVIOUS YEARS’ QUESTIONS     819

If we check with the options, the closest such value will be 0.7k. Ans. (c)

many multiple choice questions does the exam have? (a) 12

(b) 15

(c) 18

(d) 19

337. Choose the option with words that are not synonyms. (GATE 2017, 1 Mark) (a) aversion, dislike (c) plunder, loot

(b) luminous, radiant (d) yielding, resistant (GATE 2017, 1 Mark)

Solution:  The words “yielding” and “resistant” are not synonyms. Ans. (d)

Solution:  Let x be the number of questions of 3 marks. Therefore, the number of questions of 11 marks will be 20 - x. Also, 3x + 11(20 - x) = 100 ⇒ 3x + 220 - 11x = 100 ⇒ 8x = 120 ⇒  x = 15

338. Saturn is _________ to be seen on a clear night with the naked eye. (a) enough bright (b) bright enough (c) as enough bright (d) bright as enough

Ans. (b) 341. There are 3 red socks, 4 green socks and 3 blue socks. You choose 2 socks. The probability that they are of the same colour is (GATE 2017, 1 Mark) (a) 1/5

(b) 7/20

(c) 1/4

(d) 4/15

Solution:  The correct answer is bright enough. Solution: Ans. (b) 339. There are five buildings called V, W, X, Y and Z in a row (not necessarily in that order). V is to the West of W. Z is to the East of X and the West of V. W is to the West of Y. Which is the building in the middle? (a) V

(b) W

(c) X

(d) Y

Favourable number of ways = 3C2 + 4C2 + 3C2 = 3 + 6 + 3 = 12 Total number of ways =

10

C2 = 45

Therefore, Required probability = 12/45 = 4/15 Ans. (d)

(GATE 2017, 1 Mark) 342. She has a sharp tongue and it can occasionally turn _________.

Solution:  E N

(a) hurtful (c) methodical

(b) left (d) vital

S (GATE 2017, 1 Mark) W Solution:  She has a sharp tongue and it can occasionally turn hurtful.

The buildings will be as shown: Y W V Z X

Ans. (a) 343. I __________ made arrangements had I __________ informed earlier.

Therefore, building V is in the middle.

(a) could have, been

(b) would have, being

(c) had, have

(d) had been, been

Ans. (a) (GATE 2017, 1 Mark) 340. A test has twenty questions worth 100 marks in total. There are two types of questions. Multiple choice questions are worth 3 marks each and essay questions are worth 11 marks each. How

GATE_GA-Questions.indd 819

Solution:  I could have made arrangements had I been informed earlier. Ans. (a)

8/22/2017 5:36:47 PM

820     GENERAL APTITUDE

344. In the summer, water consumption is known to decrease overall by 25%. A Water Board official states that in the summer household consumption decreases by 20%, while other consumption increases by 70%.

(a) Only (i) (c) Only (ii) and (iii)

(b) Only (ii) (d) Only (iv)

B T

S

S

B

T

C

Which of the following statements is correct? (a) The ratio of household to other consumption is 8/17.

C

(b) The ratio of household to other consumption is 1/17. T

S

(c) The ratio of household to other consumption is 17/8.

B

(d) There are errors in the official’s statement. C (GATE 2017, 1 Mark) (GATE 2017, 1 Mark)

Solution:  Let the ratio of household consumption to other consumption be x:y.

Solution:  According to the Venn diagram shown above, option (b) is correct. Ans. (b)

Therefore, -20x + 70y = -25(x + y) ⇒ -20x + 70y = -25x - 25y

347. The ninth and the tenth of this month are Monday and Tuesday _________.

⇒ 5x = -95y

(a) figuratively (c) respectively

which is not possible.

(b) retrospectively (d) rightfully

Therefore, option (d) is correct. (GATE 2017, 1 Mark) Ans. (d) 345. 40% of deaths on city roads may be attributed to drunken driving. The number of degrees needed to represent this as a slice of a pie chart is (a) 120

(b) 144

(c) 160

(d) 212

(GATE 2017, 1 Mark) Solution:  100% = 360° ⇒ 1% = 3.6°

Solution:  The ninth and the tenth of this month are Monday and Tuesday, respectively. Ans. (c) 348. It is _________ to read this year’s textbook _________ the last year’s. (a) easier, than (c) easier, from

(b) most easy, than (d) easiest, from (GATE 2017, 1 Mark)

Now,

Solution:  It is easier to read this year’s textbook than the last year’s. Ans. (a)

40% = 40 × 3.6 = 4 × 36 = 144° Ans. (b) 346. Some tables are shelves. Some shelves are chairs. All chairs are benches. Which of the following conclusions can be deduced from the preceding sentences?     (i) At least one bench is a table   (ii) At least one shelf is a bench (iii) At least one chair is a table (iv) All benches are chairs

GATE_GA-Questions.indd 820

349. A rule states that in order to drink beer, one must be over 18 years old. In a bar there are 4 people. P is 16 years old, Q is 25 years old, R is drinking milkshake and S is drinking a beer. What must be checked to ensure that the rule is being followed? (a) Only P  ’s drink (b) Only P ’s drink and S ’s age (c) Only S ’s age (d) Only P ’s drink, Q’s drink and S ’s age (GATE 2017, 1 Mark)

8/22/2017 5:36:48 PM

SOLVED GATE PREVIOUS YEARS’ QUESTIONS     821

Solution:  Only P ’s drink and S ’s age must be checked to ensure that the rule is being followed. Ans. (b) 350. Fatima starts from point P, goes North for 3 km and then East for 4 km to reach point Q. She then turns to face point P and goes 15 km in that direction. She then goes North for 6 km. How far is she from point P and in which direction should she go to reach point P  ? (a) 8 km, East (c) 6 km, East

(b) 12 km, North (d) 10 km, North (GATE 2017, 1 Mark)

Solution:  We know, n(A ∪ B ∪ C) = n(A) + n(B) + n(C) - n(A ∩ B) - n(B ∩ C) - n(A ∩ C) + n(A ∩ B ∩ C) Let x be the number of students who take all the three subjects. Therefore, from the above formula, we have    500 = 329 + 186 + 295 - 83 - 217 - 63 - x ⇒ 500 = 447 + x ⇒ x = 500 - 447 = 53 Ans. (d) 352. He was one of my best _________ and I felt his loss _________.

Solution: (a) friend, keenly (c) friend, keener

E

(b) friends, keen (d) friends, keenly (GATE 2017, 1 Mark)

N

S Solution:  He was one of my best friends and I felt his loss keenly. Ans. (d)

W

Q

353. As the two speakers became increasingly agitated, the debate became _________.

5

(a) lukewarm (c) forgiving

4

(b) poetic (d) heated (GATE 2017, 1 Mark)

P 10 Solution:  As the two speakers became increasingly agitated, the debate became heated. Ans. (d)

3

6

354. A right-angled cone (with base radius 5 cm and height 12 cm), as shown in the figure below is rolled on the ground keeping the point P fixed until the point Q (at the base of the cone, as shown) touches the ground again. 360 deg.

351. 500 students are taking one or more courses out of Chemistry, Physics and Mathematics. Registration records indicate course enrolment as follows: Chemistry (329), Physics (186), Mathematics (295), Chemistry and Physics (83), Chemistry and Mathematics (217), and Physics and Mathematics (63). How many students are taking all 3 subjects?

r= 5c m

? Option (a) is correct as Fatima is at the position as shown and therefore needs to go 8 km towards the East direction. Ans. (a)

h= 12 c m

Q

(b) 43

(c) 47

(d) 53

(GATE 2017, 1 Mark)

GATE_GA-Questions.indd 821

Ground

By what angle (in radians) about P does the cone travel? (a)

(a) 37

P

5p 12

(b)

5p 24

(c)

24p 5

(d)

10p 13

(GATE 2017, 1 Mark)

8/22/2017 5:36:50 PM

822     GENERAL APTITUDE

Solution:  Let l represent the slant height of the cone. Q

If the number of cars with S is 0, then only the statement given by Q is correct. Ans. (a)

l Rq

numerical value for which exactly one of the three statements is true will be 0.

q

P

(height cone)

357. The ways in which this game can be played _________ potentially infinite. (a) is (c) are

R l2 = 52 + 122 = 25 + 144 = 169 ⇒ l = 13



(b) is being (d) are being (GATE 2017, 1 Mark)

Circumference of base circle = Length of arc QR ⇒ 2p r = Rq (R = slant height of the cone) ⇒ 2p r = 13q ⇒ 2p × 5 = 13q 10p ⇒q = 13

Solution:  The ways in which this game can be played are potentially infinite. Ans. (c) 358. If you choose plan P, you will have to ________ plan Q, as these two are mutually ________. (a) forgo, exclusive (c) accept, exhaustive

(b) forget, inclusive (d) adopt, intrusive (GATE 2017, 1 Mark)

Ans. (d) 355. In a company with 100 employees, 45 earn Rs. 20,000 per month, 25 earn Rs. 30,000, 20 earn Rs. 40,000, 8 earn Rs. 60,000 and 2 earn Rs. 150,000. The median of the salaries is (a) Rs. 20,000 (c) Rs. 32,300

Solution:  Mutually exclusive is an event where only one of two given events can happen. Therefore, option (a) is correct. Ans. (a) 359. If a and b are integers and a - b is even, which of the following must always be even?

(b) Rs. 30,000 (d) Rs. 40,000 (GATE 2017, 1 Mark)

Solution:  Arrange all the values in ascending or descending order first. Now, the number of observations is equal to 100 [even]. Therefore, Median of these values = Average of the two middlemost observations 50 th Observation + 51th Observation = 2 30,000 + 30,000 = = 30,000 2

(b) a2 + b2 + 1 (d) ab - b

(a) ab (c) a2 + b + 1

(GATE 2017, 1 Mark) Solution:  If a - b is even either both a and b are even or both a and b are odd. If a and b are both odd, •• •• •• ••

ab will be odd. a2 + b2 + 1 will be odd. a2 + b + 1 will be odd. But ab - b will always be even. Ans. (d)

Ans. (b) 356. P, Q and R talk about S ’s car collection. P states that S has at least 3 cars. Q believes that S has less than 3 cars. R indicates that to his knowledge, S has at least one car. Only one of P, Q and R is right. The number of cars owned by S is (a) 0 (c) 3



(b) 1 (d) Cannot be determined (GATE 2017, 1 Mark)

Solution:  It is mentioned that only one of them gave a correct statement. The only possible

GATE_GA-Questions.indd 822

360. A couple has 2 children. The probability that both children are boys if the older one is a boy is (a) 1/4

(b) 1/3

(c) 1/2

(d) 1

(GATE 2017, 1 Mark) Solution:  A couple has 2 children in 4 ways: BG, BB, GB, GG Therefore, the probability that both children are boys if the older one is a boy is 1/2. Ans. (c)

8/22/2017 5:36:52 PM

SOLVED GATE PREVIOUS YEARS’ QUESTIONS     823

(a) Only (i) (c) Both (i) and (ii)

361. P looks at Q while Q looks at R. P is married, R is not. The number of people in which a married person is looking at an unmarried person is

(b) Only (ii) (d) Neither (i) nor (ii) (GATE 2017, 1 Mark)

(a) 0 (c) 2



(b) 1 (d) Cannot be determined

Solution:  Neither (i) nor (ii) is correct since some lamps are beds or some beds are lamps is only a possibility and not a definite conclusion. Ans. (d)

(GATE 2017, 1 Mark) Solution:  In this question, we do not know about the marital status of Q. (a) Let Q be married. P looks at Q: Married looking at married: Not considered. Q looks at R: Married looking at unmarried: Considered.

365. If the radius of a right circular cone is increased by 50%, its volume increases by (a) 75% (c) 125%

Solution:  The formula for the volume of a cone is Πr2h. If the radius increases by 50%, the volume will increase by the successive effect of r since the formula has r2 in it.

(b) Let Q be unmarried. P looks at Q: Married looking at unmarried: Considered. Q looks at R: unmarried looking at unmarried: Not Considered.

% change in volume = 50 + 50 + [(50 × 50)/100] = 100 + (2500/100) = 125% Ans. (c)

Therefore, count is 1.

362. The bacteria in milk are destroyed when it ______________ heated to 80 degree Celsius. (a) would be (c) is

(b) 100% (d) 237.5% (GATE 2017, 1 Mark)

Therefore, count is 1.

In both the cases, irrespective of the marital status of Q, the answer will be 1. Ans. (b)



366. The following sequence of numbers is arranged in increasing order, 1, x, x, x, y, y, 9, 16, 18. Given that the mean and median are equal, and are also equal to twice the mode, the value of y is (a) 5

(b) will be (d) was

(b) 6

(c) 7

(d) 8

(GATE 2017, 1 Mark)

(GATE 2017, 1 Mark) Solution:  The given data is 1, x, x, x, y, y, 9, 16, 18. Solution:  The bacteria in milk are destroyed when it is heated to 80 degree Celsius.

Median is the middlemost data which is y. Mean is given by (1 + 3x + 2y + 9 + 16 + 18)/9. Since mean and median are equal, we have

Ans. (c) 363. _________ with someone else’s email account is now a very serious offence. (a) Involving (c) Tampering

(b) Assisting (d) Incubating



All benches are beds. No bed is a bulb. Some bulbs are lamps.

Solving Eqs. (i) and (ii), we have

GATE_GA-Questions.indd 823

    3x - 14x = -44 ⇒ -11x = -44 ⇒x=4 Therefore, from Eq. (ii)

Which of the following can be inferred? (i) Some beds are lamps. (ii) Some lamps are beds.

(i)

Also, since x is the mode and median is twice of mode, we have y = 2x (ii)

(GATE 2017, 1 Mark) Solution:  Tampering with someone else’s email account is now a very serious offence. Ans. (c) 364. Consider the following sentences:

(1 + 3x + 2y + 9 + 16 + 18)/9 = y ⇒ 3x + 2y + 44 = 9y ⇒ 3x - 7y = -44



y = 2x = 8 Ans. (d)

8/22/2017 5:36:52 PM

824     GENERAL APTITUDE

367. The event would have been successful if you ____________ able to come.

(a) are (c) have been

Solution: x+2

(b) had been (d) would have been (GATE 2017, 1 Mark)



Which of the words below is closest in meaning to the underlined word above? (a) pretty (c) sloppy

(GATE 2017, 1 Mark) Solution:  Complete is closest in meaning to the word thorough. Ans. (b) 369. Four cards lie on a table. Each card has a number printed on one side and a colour on the other. The faces visible on the cards are 2, 3, red, and blue. Proposition: If a card has an even value on one side, then its opposite face is red. The cards which MUST be turned over to verify the above proposition are

(b) 2, 3, red (d) 2, red, blue

Solution:  (a) The proposition says that if one side is an even number, then the other side will be red. It does not say anything when the number visible is not even. Also if a side is red, then the number on the opposite side could be an odd number also. Therefore, to verify the proposition, the two cards must be turned are the one showing blue on its face and the other one showing 2 on its face. Therefore, option (c) is the correct answer. Ans. (c) 370. What is the value of x when x+2



2x + 4

= 144

Equating the powers of 3 or the powers of 2, we get x = -1. Ans. (b)

(a) 1/9

(b) 2/9

(c) 1/3

(d) 4/9

(GATE 2017, 1 Mark) Solution:  The possible options for the product are {1, 2, 3, 4, 5, 6, 2, 4, 6, 8, 10, 12, 3, 6, 9, 12, 15, 18, 4, 8, 12, 16, 20, 24, 5, 10, 15, 20, 25, 30, 6, 12, 18, 24, 30, 36}. The perfect squares are {1, 4, 4, 4, 9, 16, 25, 36} which will form the favourable cases. Therefore, Required probability = 8/36 = 2/9

372. “The hold of the nationalist imagination on our colonial past is such that anything inadequately or improperly nationalist is just not history.” Which of the following statements best reflects the author’s opinion? (a) Nationalists are highly imaginative. (b) History is viewed through the filter or nationalism. (c) Our colonial past never happened. (d) Nationalism has to be both adequately and properly imagined. (GATE 2017, 2 Marks) Solution:  Statement (b) best reflects the author’s opinion.

2x + 4

3 ÷  = 144 ? 5 (b) -1 (d) Cannot be determined (GATE 2017, 1 Mark)

GATE_GA-Questions.indd 824

 3 ÷   5

Ans. (b)

(GATE 2017, 1 Mark)

 16  81 ×    25  (a) 1 (c) -2

 (24 )x + 2 ⇒ 34  2 x + 2  (5 )

= 144

371. Two dice are thrown simultaneously. The probability that the product of the numbers appearing on the top faces of the dice is a perfect square is

(b) complete (d) haphazard

(a) 2, red (c) 2, blue

3 ÷  5

 2 4 x + 8   52 x + 4  ⇒ 34  2 x + 4   2 x + 4  = 144 5  3  4 −2 x − 4 ⇒3 × 2 4 x + 8 = 2 4 × 32

Solution:  The event would have been successful if you had been able to come. Ans. (b) 368. There was no doubt that their work was thorough.

2x + 4

 16  81 ×    25 

Ans. (b) 373. Six people are seated around a circular table. There are at least two men and two women. There are at least three right-handed persons. Every woman has a left-handed person to her immediate right.

8/22/2017 5:36:53 PM

SOLVED GATE PREVIOUS YEARS’ QUESTIONS     825

None of the women are right-handed. The number of women at the table is (a) 2 (c) 4



(b) 3 (d) Cannot be determined

colour red and Shwetha dislikes the colour white. Gulab and Neel like all the colours. In how many different ways can they choose the shirts so that no one has a shirt with a colour he or she dislikes? (a) 21

(b) 18

(c) 16

(d) 14

(GATE 2017, 2 Marks) (GATE 2017, 2 Marks) Solution:  There are 4 men and 2 women. Also, out of the men 3 will be right handed and one man will be left handed.

Solution:  We will take 2 cases: 1. Arun selects white. The other three people can select in 3! ways, i.e. 6 ways.

M, Right Handed

2. Arun does not select white. Arun can select in 2 ways (since red cannot be selected). Now, Shweta can select in 2 ways and the remaining two people can select in 2 ways. Therefore, M, Right Handed

Total number of ways = 2 × 2 × 2 = 8 ways Hence, Total of both cases 1 and 2 = 6 + 8 = 14 ways

M, Right Handed

Ans. (d)

Ans. (a) (x + y) − x − y is equal to 2 (a) the maximum of x and y (b) the minimum of x and y (c) 1 (d) none of the above

374. The expression

376. A contour line joins locations having the same height above the mean sea level. The following is a contour plot of a geographical region. Contour lines are shown at 25 m intervals in this plot. If in a flood, the water level rises to 525 m, which of the villages P, Q, R, S, T get submerged? Q 425

R

550

450

(GATE 2017, 2 Marks) P 550

Solution:  The given expression is [(x + y) - | x - y  | ]/2

T 500 450

There are 2 cases:

S

500

1. If x > y, then | x - y | = x - y   Therefore, (x + y - x + y)/2 = 2y/2 = y

(a) P, Q

2. If x < y, then | x - y | = -(x - y)   Therefore, (x + y + x - y)/2 = 2x/2 = x

(b) P, Q, T (c) R, S, T

(d) Q, R, S

(GATE 2017, 2 Marks)

The expression will be equal to the minimum of x or y.

Solution:  R, S and T are all below 525 m and will get submerged. Ans. (c)

Ans. (b) 375. Arun, Gulab, Neel and Shweta must choose one shirt each from a pile of four shirts coloured red, pink, blue and white, respectively. Arun dislikes the

GATE_GA-Questions.indd 825

377. “We lived in a culture that denied any merit to literary works, considering them important only when they were handmaidens to something seemingly more urgent namely ideology. This was a

8/22/2017 5:36:54 PM

826     GENERAL APTITUDE

country where all gestures, even the most private, were interpreted in political terms.” The author’s belief that ideology is not as important as literature is revealed by the word: (a) culture (c) urgent

(a) 0

(b) 1

(c) 2

(d) 3

(GATE 2017, 2 Marks) Solution: 

(b) seemingly (d) political

6

(GATE 2017, 2 Marks) 4

Solution:  The author’s belief that ideology is not as important as literature is revealed by the word seemingly. Ans. (b)

2

−3

(a) The box labelled “apples” (b) The box labelled “apples and oranges” (c) The box labelled “oranges” (d) Cannot be determined (GATE 2017, 2 Marks)

379. X is a 30 digit number starting with the digit 4 followed by the digit 7. Then the number X3 will have (a) 90 digits (c) 92 digits

(b) 91 digits (d) 93 digits

2

2

3 ex 2 − 0.5x2

−2

As can be clearly seen from the above graph, there will be two solutions in the range of -5 to 5. Therefore, option (c) is correct. Ans. (c) 381. An air pressure contour line joins locations in a region having the same atmospheric pressure. The following is an air pressure contour plot is a geographical region. Contour lines are shown at 0.05 bar intervals in this plot. R 0.65 0.1

0.9 S

P

0.9 0.8

Q 0.8

0.75 0

2 km

1

If the possibility of a thunderstorm is given by how fast air pressure rises or drops over a region, which of the following regions is most likely to have a thunderstorm? (a) P

(GATE 2017, 2 Marks)

−1

0.95

Solution:  It is given that all the three boxes have been incorrectly labelled. we will open the box which is labelled as “apples and oranges”. Let us suppose this box contains apples. Then we are left with two boxes labelled “oranges” and “apples”. One of these two boxes will have oranges and the other will have apples and oranges. Now, the box having the label “oranges” cannot have oranges. Therefore, the box labelled “oranges” will have apples and oranges while the box labelled “apples” will have oranges. The same will be true if the box having the label “apples and oranges” contains oranges. Therefore, option (b) is correct. Ans. (b)

−2

0.95

378. There are three boxes. One contains apples, another contains oranges and the last one contains both apples and oranges. All three are known to be incorrectly labeled. If you are permitted to open just one box and then pull out and inspect only one fruit, which box would you open to determine the contents of all three boxes?

(b) Q

(c) R

(d) S

(GATE 2017, 2 Marks)

Solution:  The number X3 will have 90 digits. Ans. (a) 380. The number of roots of ex + 0.5x2 - 2 = 0 in the range [−5, 5] is

GATE_GA-Questions.indd 826

Solution:  Air pressure drop is the maximum in the region R as can be seen from the contour lines. Therefore, option (c) is correct. Ans. (c)

8/22/2017 5:36:55 PM

SOLVED GATE PREVIOUS YEARS’ QUESTIONS     827

382. “If you are looking for a history of India, or for an account of the rise and fall of the British Raj, or for the reason of the cleaving of the subcontinent into two mutually antagonistic parts and the effects this mutilation will have in the respective sections, and ultimately on Asia, you will not find it in these pages, for though I have spent a lifetime in the country, I lived too near the seat of events, and was too intimately associated with the actors, to get the perspective needed for the impartial recording of these matters.” Here, the word “antagonistic” is closest in meaning to (a) impartial

(b) argumentative

(c) separated

(d) hostile

Therefore, from the above figure, X is the person who is third to the left of V. Ans. (a) 384. Trucks (10 m long) and cars (5 m long) go on a single lane bridge. There must be a gap of at least 20 m after each truck and a gap of at least 15 m after each car. Trucks and cars travel at a speed of 36 km/h. If cars and trucks go alternately, what is the maximum number of vehicles that can use the bridge in one hour? (a) 1440

(b) 1200

(c) 720

(d) 600

(GATE 2017, 2 Marks) Solution:  Given that speeds of both car and truck = 36 km/hour

(GATE 2017, 2 Marks) Solution:  Here, the word “antagonistic” is closest in meaning to hostile. Ans. (d) 383. S, T, U, V, W, X, Y and Z are seated around a circular table. T’s neighbours are Y and V, Z is seated third to the left to T and second to the right of s. U’s neighbours are S and Y; and T and W are not seated opposite each other. Who is third to the left of V? (a) X

(b) W

(c) U

In 1 h, they travel = 36 km = 36000 m 5m

15 m truck

20 m

car

gap

gap 1 h = 36 km = 36000 m

10 m

Therefore, the maximum number of vehicles that can use the bridge in 1 h =

36000 × 2 = 720 × 2 = 1440 vehicles 50

(d) T

Ans. (a)

(GATE 2017, 2 Marks) 385. There are 3 Indians and 3 Chinese in a group of 6 people. How many subgroups of this group can we choose so that every subgroup has at least one Indian?

Solution:

(a) 56 Y

(b) 52

(c) 48

(d) 44

V (GATE 2017, 2 Marks) Solution:  Total number of ways of selection = 6C1 + 6C2 + 6C3 + 6C4 + 6C5 + 6C6 W

= 6 + 15 + 20 + 15 + 6 + 1 = 63 wayss Total number of ways of selection such that there is no Indian = 3C1 + 3C2 + 3C3

Z

X

Therefore,

= 3 + 3 + 1 = 7 ways

Total number of ways of selection such that there is at least one Indian = 63 - 7 = 56 ways Ans. (a)

GATE_GA-Questions.indd 827

8/22/2017 5:36:57 PM

828     GENERAL APTITUDE

Solution:  Option (a) best reflects the author’s opinion. Ans. (a)

386. A contour line joins locations having the same height above the mean sea level. The following is a contour plot of a geographical region. Contour lines are shown at 25 m intervals in this plot. 425 450

Q 575

550

P

388. Each of P, Q, R, S, W, X, Y and Z has been married at most once. X and Y are married and have two children P and Q, Z is the grandfather of the daughter S of P. Further, Z and W are married and are parents of R. Which one of the following must necessarily be FALSE?

550

(a) X is the mother-in-law of R. (b) P and R are not married to each other. (c) P is a son of X and Y. (d) Q cannot be married to R.

500

500

475

(GATE 2017, 2 Marks) The path from P to Q is best described by Solution:  (a) Up-Down-Up-Down (b) Down-Up-Down-Up (c) Down-Up-Down (d) Up-Down-Up

Z

W

X

Y

(GATE 2017, 2 Marks) Solution:  The path from P to Q is best described as follows: 550 _ 500 _ 575 _ 550 R

Down

Up

Therefore, option (c) is the correct answer. Ans. (c) 387. “If you are looking for a history of India, or for an account of the rise and fall of the British Raj or for the reason of the cleaving of the subcontinent into two mutually antagonistic parts and the effects this mutilation will have in the respective sections, and ultimately or Asia, you will not find it in these pages; for though I have spent a lifetime in the country. I lived too near the seat of events, and was too intimately associated with the actors, to get the perspective needed for the impartial recording of these matters.”

Q

From the figure, P and R are married. So, option b is necessarily false. Ans. (b) 389. 1200 men and 500 women can build a bridge in 2 weeks, 900 men and 250 women will take 3 weeks to build the same bridge. How many men will be needed to build the bridge in one week? (a) 3000

(a) An intimate association does not allow for the necessary perspective. (b) Matters are recorded with an impartial perspective. (c) An intimate association offers an impartial perspective. (d) Actors are typically associated with the impartial recording of matters. (GATE 2017, 2 Marks)

(b) 3300

(c) 3600

(d) 3900

(GATE 2017, 2 Marks) Solution:  Let the total work be 42 units. 1200 M + 500 W = 14 days; 3 units per day (i) (2 weeks) 900 M + 250 W = 21 days; 2 units per day (ii) (3 weeks)

Which of the following statements best reflects the author’s opinion?

GATE_GA-Questions.indd 828

P

Down

From Eq. (i), we have

1200 M + 500 W = 3 units

(iii)

From Eq. (ii), we have

1800 M + 500 W = 4 units

(iv)

Solving Eqs. (iii) and (iv), we get 600 M = 1 unit ⇒ 1 M = 1/600 units

8/22/2017 5:36:58 PM

SOLVED GATE PREVIOUS YEARS’ QUESTIONS     829

Also,

a contour plot of a geographical region. Contour lines are shown at 25 m intervals in this plot.

500 W = 1 unit ⇒ 1 W = 1/500 units

R Now, the same work has to be completed in 1 week, i.e. 7 days. If 42 units has to be done in 7 days, 6 units will be done per day. 600 men are able to do 1 unit work per day. Therefore, for 6 units, 3600 men will be required.

425 450

550 575

P 57 5 550

S 500

Ans. (c)

T 500

475

390. The number of 3-digit numbers such that the digit 1 is never to the immediate right of 2 is

0 (a) 781

(b) 791

(c) 881

1

2 km

(d) 891

(GATE 2017, 2 Marks)

Which of the following is the steepest path leaving from P? (a) P to Q (b) P to R

Solution:  The total number of three digit numbers (from 100 to 999) = 900 Let 2 be at the hundred’s place and so 1 will be at the ten’s place. Hundred’s Ten’s Place Place

Unit’s Place

Digit 2

Digit 1

Any of the 10 digits

1 way

1 way

10 ways

Therefore, there are total 10 such numbers. Let 2 be at the ten’s place and so 1 will be at the unit’s place. Hundred’s Place

Ten’s Place

Unit’s Place

Any digit from 1 to 9

Digit 2

Digit 1

9 way

1 way

1 way

Therefore, there are total 9 such numbers. Total 3 digit numbers such that the digit 1 is to the immediate right of digit 2 = 19. Therefore, Required answer = 900 - 19 = 881 numbers

(c) P to S

(d) P to T

(GATE 2017, 2 Marks) Solution:  P to R path is the deepest path from sea level. Therefore, it is the steepest path. Ans. (b) 392. “Here, throughout the early 1820s, Stuart continued to fight his losing battle to allow his sepoys to wear their caste-marks and their own choice of facial hair on parade, being again reprimanded by the commander-in-chief. His retort that ‘A stronger instance than this of European prejudice with relation to this country has never come under my observations’ had no effect on his superiors.” According to this paragraph, which of the statements below is most accurate? (a) Stuart’s commander-in-chief was moved by this demonstration of his prejudice. (b) The Europeans were accommodating of the sepoys’ desire to wear their caste-marks. (c) Stuart’s “losing battle” refers to his inability to succeed in enabling sepoys to wear caste-marks. (d) The commander-in-Chief was exempt from the European prejudice that dictated how the sepoys were to dress. (GATE 2017, 2 Marks)

Ans. (c) 391. A contour line joins locations having the same height above the mean sea level. The following is

GATE_GA-Questions.indd 829

Solution:  According to the paragraph, option (c) is the most accurate. Ans. (c)

8/22/2017 5:36:59 PM

830        GENERAL APTITUDE

393. What is the sum of the missing digits in the subtraction problem below?

Taking this condition, we have

5____ -4 8 _ 8 9     ________ 1111



−x + 1 ≤ 2 ⇒ −x ≤ 1 ⇒ x ≥ −1

(4)

Combining Eqs. (3) and (4), we have (a) 8 (c) 11

(b) 10 (d) Cannot be determined



Solution:  There are two possible cases:



  (i) Considering

For y + 2 ≤ 3

50000 - 48889 = 1111

−1 ≤ x ≤ 3

y + 2 = y + 2,



0+0+0+0+8 = 8

(B)

Combining Eqs. (A) and (B), we have

(GATE 2017, 2 Marks)

Therefore, the sum of the missing digits is

−1 ≤ x ≤ 1

(C)

if y + 2 ≥ 0 or y ≥ − 2 (5)

Taking this condition, we have y + 2 ≤ 3 ⇒ y ≤ 1

(ii) Considering

(6)

50100 - 48989 = 1111 Combining Eqs. (5) and (6), we have Therefore, the sum of the missing digits is 0 + 1 + 0 + 0 + 9 = 10



Hence, the correct option is (d) as the answer cannot be determined. Ans. (d) 394. Let S1 be the plane figure consisting of the points (x, y) given by the inequalities |x - 1| ≤ 2 and |y + 2| ≤ 3. Let S2 be the plane figure given by the inequalities x - y ≥ -2, y ≥ 1 and x ≤ 3. Let S be the union of S1 and S2. The area of S is (a) 26

(b) 28

(c) 32



−2 ≤ y ≤ 1

y + 2 = − y − 2,

(D)

if y + 2 < 0 or y < − 2

Taking this condition, we have −y − 2 ≤ 3 ⇒ −y ≤ 5 ⇒ y ≥ −5



(E)

Combining Eqs. (D) and (E), we have

−5 ≤ y ≤ 1

(F)

(d) 34 We will now plot the graph of the union of

(GATE 2017, 2 Marks) x − y ≥ −2, y ≥ 1, x ≤ 3, −1 ≤ x ≤ 3 and −5 ≤ y ≤ 1 Solution: 

The required area is the area of ∆ABC and the area of rectangle ABDE.

For x − 1 ≤ 2 x − 1 = x − 1, if x − 1 ≥ 0 or x ≥ 1



(1)

=

Taking this condition,

Area of ∆ABC

x −1 ≤ 2 ⇒x≤3

1 1 × AB × BC = × 4 units × 4 units = 8 sq. units 2 2

Area of rectangle ABDE (2) = AB × BD = 4 units × 6 units = 24 sq. units

Combining Eqs. (1) and (2), we have Therefore,     

GATE_GA-Questions.indd 830

1≤ x≤ 3

x − 1 = −x + 1,

(A) if x < 1

(3)

Total area = 8 units + 24 units = 32 sq. units Ans. (c)

25-08-17 3:02:56 PM

SOLVED GATE PREVIOUS YEARS’ QUESTIONS     831

x−y =2

6 C

5 E 4

4 Unit 3

4 × 4 = 16 U × 2 = 8U

2 B

4 Unit

A 1 −6

−5

−4

−3

−2

−1

0

1

2

3

4

6

5

4 × 6 = 24 U

−1 6 −2 −3 −4 x−y ≤2

E

D

−5

395. Two very famous sportsmen Mark and Steve happened to be brothers, and played for country K. Mark teased James, an opponent from country E. “There is no way you are good enough to play for your country.” James replied, “Maybe not, but at least I am the best player in my own family.” Which one of the following can be inferred from this conversation? (a) Mark was known to play better than James. (b) Steve was known to play better than Mark. (c) James and Steve were good friends. (d) James played better than Steve.

Population density

−6

1.0 0.9 0.8 0.7 0.6 0.5 0.4 0.3 0.2 0.1 0.0

37°C 25°C

0 20 40 60 80 100 120 140 160 180 200

(GATE 2017, 2 Marks) Solution:  According to the statement given by James, Mark is not the best player in his (Mark’s) family. From this statement, we can infer that Steve is a better player than Mark. Ans. (b) 396. The growth of bacteria (lactobacillus) in milk leads to curd formation. A minimum bacterial population density of 0.8 (in suitable units) is needed to form curd. In the graph below, the population density of lactobacillus in 1 litre of milk is plotted as a function of time, at two different temperatures, 25°C and 37°C.

GATE_GA-Questions.indd 831

Consider the following statements based on the data shown above:    (i) The growth in bacterial population stops earlier at 37°C as compared to 25°C. (ii) The time taken for curd formation at 25°C is twice the time taken at 37°C. Which one of the following options is correct? (a) Only (i) (c) Both (i) and (ii)

(b) Only (ii) (d) Neither (i) nor (ii) (GATE 2017, 2 Marks)

8/22/2017 5:37:06 PM

832     GENERAL APTITUDE

Solution:  From the graph, it can be seen that statement (i) is true whereas statement (ii) is not correct. Ans. (a) 397. “If you are looking for a history of India, or for an account of the rise and fall of the British Raj, or for the reason of the cleaving of the subcontinent into two mutually antagonistic parts and the effects this mutilation will have in the respective sections, and ultimately on Asia, you will not find it in these pages, for though I have spent a lifetime in the country, I lived too near the seat of events, and was too intimately associated with the actors, to get the perspective needed for the impartial recording of these matters.” Which of the following is closest in meaning to “cleaving”? (a) Deteriorating (b) Arguing (c) Departing (d) Splitting (GATE 2017, 2 Marks) Solution:  “Splitting” is closest in meaning to “cleaving.” Ans. (d) 398. X bullocks and Y tractors take 8 days to plough a field. If we halve the number of bullocks and double the number of tractors, it takes 5 days to plough the same filed. How many days will it take X bullocks alone to plough the field? (a) 30

(b) 35

(c) 40

Therefore, Time taken = 40/X = 40 × 3/4 = 30 days Ans. (a) 399. There are 4 women P, Q, R, S and 5 men V, W, X, Y, Z in a group. We are required to form pairs each consisting of one woman and one man. P is not to be paired with Z, and Y must necessarily be paired with someone. In how many ways can 4 such pairs be formed? (a) 74

(b) 76

(c) 78

(d) 80

(GATE 2017, 2 Marks) Solution:  There are two possibilities: (a) If Z is not selected.     The four pairs can be formed in 4! ways, i.e. 24 ways. (b)  If Z is selected.      Z can first form a pair in 3 ways. (P is not paired with Z).    Also, Y has to be definitely selected. The remaining 2 people from the 3 men available can be chosen in 3C2, i.e. 3 ways. The 3 people selected including Y can now form a pair in 3! ways = 6 ways. Total number of ways in the second case = 3 ways × 3 ways × 6 ways = 54 ways. Therefore, from the two possibilities, Total number of ways = 24 + 54 = 78 ways

(d) 45

Ans. (c) (GATE 2017, 2 Marks) Solution:  Let the work be 40 units.

X + Y = 8

5 units per day

(i)



X + 2Y = 5 2

8 units per day

(ii) P says “Both of us are knights”. Q says “None of us are knaves”.

Eq. (i) can be written as



2X + 2Y = 10



(iii)

Which one of the following can be logically inferred from the above?

(iv)

(a) Both P and Q are knights. (b) P is a knight; Q is a knave. (c) Both P and Q are knaves. (d) The identities of P and Q cannot be determined.

Eq. (ii) can be written as



X + 2Y = 8 2

400. All people in a certain island are either “Knights” or “Knaves” and each person knows every other person’s identity. Knights NEVER lie, and knaves ALWAYS lie.



Solving Eqs. (iii) and (iv), we get (GATE 2017, 2 Marks) X 3 = 2 2 ⇒ 3X = 4 ⇒ X = 4/3

GATE_GA-Questions.indd 832

Solution:  There are two possible cases: 1. If P is a knight, then the statement given by P is true as per which both P and Q are knights.

8/22/2017 5:37:07 PM

SOLVED GATE PREVIOUS YEARS’ QUESTIONS     833

Also, Q says that “none of us are knaves” which is true. This further verifies that Q is also a knight.

construction of the new metro line in the area. Modern technology for underground metro construction tried to mitigate the impact of pressurized air pockets created by the excavation of large amounts of soil. But even with these safeguards, it was feared that the soil below the concert hall would not be stable.

2. If P is a knave, then the statement given by P is not true. Also, the statement given by Q is also not true. This leads us to the conclusion that both P and Q are knaves. Therefore, the correct option is (d).

From this, one can infer that Ans. (d) (a) the foundations of old buildings create pressurized air pockets underground, which are difficult to handle during metro construction. (b) metro construction has to be done carefully considering its impact on the foundations of existing buildings. (c) old buildings in an area form an impossible hurdle to metro construction in that area. (d) pressurized air can be used to excavate large amounts of soil from underground areas.

Pollutant Concentration (ppm)

401. In the graph below, the concentration of a particular pollutant in a lake is plotted over (alternate) days of a month in winter (average temperature 10°C) and a month in summer (average temperature 30°C). 11 10 9 8 7 6 5 4 3 2 1 0

Winter Summer

(GATE 2017, 2 Marks)

0

2

4

6

8

Solution:  From the given paragraph, we can infer that metro construction has to be done carefully considering its impact on the foundations of existing buildings. Ans. (b)

10 12 14 16 18 20 22 24 26 28 30 Day of the month

Consider the following statements based on the data shown above:    (i) Over the given months, the difference between the maximum and the minimum pollutant concentrations is the same in both winter and summer. (ii) There are at last four days in the summer month such that the pollutant concentrations on those days are within 1 ppm of the pollutant concentrations on the corresponding days in the winter month.

403. Students applying for hostel rooms are allotted rooms in order of seniority. Students already staying in a room will move if they get a room in their preferred list. Preferences of lower ranked applicants are ignored during allocation. Given the data below, which room will Ajit stay in?

Names

Student Seniority

Current Room

Room Preference List

Which one of the following options is correct?

Amar

1

P

R, S, Q

(a) Only (i) (c) Both (i) and (ii)

Akbar

2

None

R, S

Anthony

3

Q

P

Ajit

4

S

Q, P, R

(b) Only (ii) (d) Neither (i) nor (ii) (GATE 2017, 2 Marks)

Solution:  The difference between the maximum and the minimum pollutant concentrations in winter = 8 - 0 = 8. The difference between the maximum and the minimum pollutant concentrations in summer = 10.5 - 1.5 = 9. Statement (i) is false and statement (ii) is correct from the graph. Ans. (b) 402. The old concert hall was demolished because of fears that the foundation would be affected by the

GATE_GA-Questions.indd 833

(a) P

(b) Q

(c) R

(d) S

(GATE 2017, 2 Marks) Solution:  As per the preferences given, Amar will go to room R. Akbar will go to room S. Anthony will go to room P. Ajit will go to room Q. Ans. (b)

8/22/2017 5:37:07 PM

404. The last digit of (2171)7 + (2172)9 + (2173)11 + (2174)13 is (a) 2

(b) 4

(c) 6

(d) 8

(GATE 2017, 2 Marks) Solution:  Given, (2171)7 + (2172)9 + (2173)11 + (2174)13 The last digit of 21717 is 1. When 9 is divided by 4, the remainder obtained is 1. Therefore, the last digit of 21729 is same as the last digit of 21721 which is 2. When 11 is divided by 4, the remainder obtained is 3. Therefore, the last digit of 217311 is same as the last digit of 21733 which is 7. When 13 is divided by 4, the remainder obtained is 1. Therefore, the last digit of 217413 is same as the last digit of 21741 which is 4. Hence,

Number of furniture items

834     GENERAL APTITUDE

20 18 16 14 12 10 8 6 4 2 0

Bed Table Chair

C1

C2 C3 C4 Carpenter (C)

C5

Consider the following statements.    (i) The number of beds made by carpenter C2 is exactly the same as the number of tables made by carpenter C3. (ii) The total number of chairs made by all carpenters is less than the total number of tables. Which one of the following is true?

Required last digit = 1 + 2 + 7 + 4 = 14 Therefore, the required last digit is 4.

(a) Only (i) (c) Both (i) and (ii)

(b) Only (ii) (d) Neither (i) nor (ii)

Ans. (b) (GATE 2017, 2 Marks) 405. Two machines M1 and M2 are able to execute any of four jobs P, Q, R and S. The machines can perform one job on one object at a time. Jobs P, Q, R and S take 30 minutes, 20 minutes, 60 minutes and 15 minutes each respectively. There are 10 objects each requiring exactly 1 job. Job P is to be performed on 2 objects. Job Q on 3 objects, Job R on 1 object and Job S on 4 objects. What is the minimum time needed to complete all the jobs? (a) 2 hours (c) 3 hours

(b) 2.5 hours (d) 3.5 hours (GATE 2017, 2 Marks)

Solution: M1: P: 1 hour and R: 1 hour M2: Q: 1 hour and S: 1 hour Therefore, the minimum time needed to complete all the jobs is 2 hours. Ans. (a) 406. The bar graph below shows the output of five carpenters over one month, each of whom made different items of furniture: chairs, tables, and beds.

GATE_GA-Questions.indd 834

Solution:  The number of beds made by carpenter C2 = 8 (12 to 20) The number of tables made by carpenter C3 = 8 (5 to 13) Therefore, statement (i) is true. Now, Total number of chairs made by all the carpenters = 2 + 10 + 5 + 2 + 4 = 23 chairs Total number of tables made by all the carpenters = 7 + 2 + 8 + 3 + 10 = 30 tables Therefore, statement (ii) is also true. Ans. (c) 407. Bhaichung was observing the pattern of people entering and leaving a car service centre. There was a single window where customers were being served. He saw that people inevitably came out of the centre in the order that they went in. However, the time they spent inside seemed to vary a lot: some people came out in a matter of minutes while for others it took much longer. From this, what can one conclude?

8/22/2017 5:37:08 PM

SOLVED GATE PREVIOUS YEARS’ QUESTIONS     835

(a) The centre operates on a first-come-first-served basis, but with variable service times, depending on specific customer needs. (b) Customers were served in an arbitrary order, since they took varying amounts of time for service completion in the centre. (c) Since some people came out within a few minutes of entering the centre, the system is likely to operate on a last-come-first-served basis. (d) Entering the centre early ensured that one would have shorter service times and most people attempted to do this.

is seated three places to the left of R. S is seated opposite U. If P and U now switch seats, which of the following must necessarily be true? (a) P is immediately to the right of R. (b) T is immediately to the left of P. (c) T is immediately to the left of P or P is immediately to the right of Q. (d) U is immediately to the right of R or P is immediately to the left of T. (GATE 2017, 2 Marks) Solution:  Consider the following figure.

(GATE 2017, 2 Marks) U/S Solution:  We can conclude that the centre operates on a first-come-first-served basis, but with variable service times, depending on specific customer needs. Ans. (a)

Q

R

408. A map shows the elevations of Darjeeling, Gangtok, Kalimpong, Pelling, and Siliguri. Kalimpong is at a lower elevation than Gangtok. Pelling is at a lower elevation than Gangtok. Pelling is at a higher elevation than Siliguri. Darjeeling is at a higher elevation than Gangtok.

P

T

U/S

Which of the following statements can be inferred from the paragraph above?     (i) Pelling is at a higher elevation than Kalimpong.   (ii) Kalimpong is at a lower elevation than Darjeeling. (iii) Kalimpong is at a higher elevation than Siliguri. (iv) Siliguri is at a lower elevation than Gangtok. (a) Only (ii) (c) Only (ii) and (iv)

(b) Only (ii) and (iii) (d) Only (iii) and (iv) (GATE 2017, 2 Marks)

Solution:  As per the question, we have Kalimpong < Gangtok

If P and U interchange their positions, either T will be immediately to the left of P or P will be immediately to the right of Q. Therefore, option (c) is true. Ans. (c) 410. Budhan covers a distance of 19 km in 2 hours by cycling one fourth of the time and walking the rest. The next day he cycles (at the same speed as before) for half the time and walks the rest (at the same speed as before) and covers 26 km in 2 hours. The speed in km/h at which Budhan walks is (a) 1

(b) 4

Pelling < Gangtok

(c) 5

(d) 6

(GATE 2017, 2 Marks)

Siliguri < Pelling Gangtok < Darjeeling

Solution:  Let the speed for cycling be C km/h and the speed of walking be W km/h. On the first day, we have

Combining the statements, we have

(2/4) × C + (6/4) × W = 19

Kalimpong < Gangtok < Darjeeling Siliguri < Pelling < Gangtok < Darjeeling



⇒ C/2 + 3W/2 = 19 ⇒ C + 3W = 38

(1)

Therefore, only statements (ii) and (iv) are correct. Ans. (c) 409. P, Q, R, S, T and U are seated around a circular table. R is seated two places to the right of Q. P

GATE_GA-Questions.indd 835

On the second day, we have (2/2) × C + (2/2) × W = 26   ⇒ C + W = 26 

(2)

25-08-17 2:55:42 PM

836     GENERAL APTITUDE

Solving Eqs. (1) and (2), we have

Which of the following statements are correct?

2W = 12 W = 6 km/h Ans. (d) 411. The points in the graph below represent the halts of a lift for durations of 1 minute, over a period of 1 hour.

   (i) T  he elevator never moves directly from any non-ground floor to another non-ground floor over the one hour period. (ii) T  he elevator stays on the fourth floor for the longest duration over the one hour period. (a) Only (i) (c) Both (i) and (ii)

(b) Only (ii) (d) Neither (i) nor (ii)

Floor Number

5 4

(GATE 2017, 2 Marks)

3

Solution:  Statement (i) is not correct since the elevator moved from the 2nd floor to the 5th floor during the time period of 25 minutes to 30 minutes.

2 1

Statement (ii) is not correct since the elevator has stopped in the ground floor for the maximum time.

0 0

5

10 15 20 25 30 35 40 45 50 55 60 Ans. (d) Time (min)

GATE_GA-Questions.indd 836

25-08-17 2:55:42 PM